You are on page 1of 1123

Laboratory

New SimpCijkd L/Wanua/^

tITSCS \

A Reference Book For Class XII


I

S.L ARORA

DHANPAT RAI & Co.


Re
Fi ad
nd
Y
ou
r
Yo
eB
oo
ur
ks
for
Flo
Fr w
ee
new Simplified Physics
Class XII
VOLUME I

s*

i
X
V
<

j
r?

A Reference Book For Class XII

S.L. Arora
/

Vv

DHANPAT RAI & CO. (Pvt.) Ltd.


EDUCATIONAL & TECHNICAL PUBUSHERS
Published by: GAGAN KAPUR
Dhanpat Rai 8: Co. (P) Ltd., Delhi

Regd. Office : 4576/15, Agarwal Road


Darya Ganj, New Delhi-110002
Phone : 2324 7736, 37, 38, dhanpatrai@gmail.com

w
Flo
ee
Fr
© S.L. Arora

Edited by : Bhupesh Arora and Kanta Arora


for
ur
ks
Yo
oo
eB

No part and style of the book may be copied in any form without the written
permission of the author.
r
ou
ad
Y

EDITIONS : 2006, 2007, 2009, 2010, 2013, 2014, 2016-2021, 2022


REPRINTS : 2008, 2011, 2012, 2015
nd
Re

FOURTEENTH EDITION: 2023


Fi

PRICE : ? 1450/- (for Vol. I & 11 +Lab Manual of Physics)

X5rpesetting by : North Delhi Computer Convention, Delhi-110009,


ndcc.in@gmail.com
Printed at: Natraj Offset, Delhi
PfEfacE to fhe Fourteenfh Edition

The author is highly thankful to the readers for the book continues to enjoy the popularity for the last
several years. In fact, this book is an outcome of continuous feedback and suggestions from the
esteemed colleagues and students. Many readers have also sent in compliments, suggestions and
pointed out some printing errors. The author is highly thankful to all of them. Every effort has been made
to incorporate their suggestions and to remove these errors in the present edition. Keeping in view the
latest Sample Paper issued by CBSE, a new section of "Objective Type Questions (OTQs)" has been added
in each chapter. This section includes Multiple Choice, Case Study Based, Assertion-Reason Type Questions.
A special feature of this edition is the Competition Section in which a vast variety of questions from latest
JEE Main and NEET examinations have been given in a topicwise manner. The author is of the firm view
that this feature of the book will help the students a long way in preparing for their competitive
examinations. Also questions from the latest JEE Advance examinations have been solved in the simplest
possible manner. All chapters have been updated with questions from the latest board papers. The
chapter of 'Communication Systems' and also many topics in other chapters, which are not included in
the latest CBSE syllabus, have been retained in the book for the convenience of the students preparing
for JEE Main and NEET examinations.

The author expresses deep regards for Mr. Jagdish Kapur and Mr. Gagan Kapur, Dhanpat Rai and Co.,
who always encourage and cooperate for large scale constructive changes for every new edition of the
book. Any suggestion towards the further improvement of the book shall be gratefully received and
acknowledged.

January, 2023 AUTHOR

shyamlal.arora@gmail.com
PrefacE

This book is an humble attempt at presenting to the students of class XII their complete syllabus prescribed by
CBSE in a clear-cut and coordinated manner. The book, based on NCERT pattern, has been carefully designed
to make physics learning as easy and straight forward as possible. The present book is the outcome of the
author's consistent interaction in the classroom with his students for more than twenty five years. Observing
that there is a wide gap between the syllabi of secondary and senior secondary classes, the author has
developed the subject matter from the very basic principles of physics. At every stage, student's 'whys' and
'hows' have been anticipated and fully answered so that even the students having weak background of physics
and mathematics may easily learn the advanced concepts of physics.

w
Some of the distinguishing features of the book are :

1. The subject matter has been arranged in a systematic manner strictly in accordance with the CBSE

Flo
syllabus.

ee
2. The language used in the book is simple, lucid and easily understandable.
3. 'Glimpses' given in each chapter provide an insight into the entire chapter. These can be used for

Fr
quick revision also.
4. The book contains a very large number of illustrative diagrams. They are neat, explicit, well-labelled
for
ur
and can be easily resketched by the students.
5. A very large number of solved and unsolved numerical problems on all topics have been given
under the appropriate articles for the guidance of the students. It is hoped, doubtlessly, that after a
ks

student goes through the solved examples of any topic, sufficient confidence will build up in him to
Yo
oo

work out unsolved problems of that topic. To further help the students, useful hints or complete
solutions of difficult problems have been given at the end of each exercise.
eB

6. 'For Your Knowledge' boxes given in the book at many places provide additional information to the
students which may prove useful for competitive examinations.
r

7. Another special feature of the book is that it contains a very large number of reasoning based
ou
ad

solved problems in the sections 'Very Short Answer Conceptual Problems' and 'Short Answer
Y

Conceptual Problems'.
8. At the end of each chapter, there is a 'Text Based Exercise'. Each exercise includes a vast collection
nd
Re

of questions asked in previous examination papers of CBSE, ISCE and many other State Boards of
India. The exercise has been split into three sections : (a) Type A contains very short answer
Fi

questions, each carrying 1 mark; (b) Type B contains short answer questions, each carrying 2 or 3
marks and (c) Type C contains long answer questions, each carrying 5 marks. Answers of all these
questions have been provided at the end of each section.

I shall fee! amply rewarded for the time and labour Ihave put in during the preparation of this volume, if the
students and their teachers find it adequate for their requirements. Ineed hardly add that suggestions and
constructive criticism to improve the book further will be gratefully accepted and incorporated in the next
edition of the book.

AUTHOR
Syllabus

Time : 3 Hours THEORY Marks : 70

Unit Marks

I. Electrostatics
16
II. Current Electricity
III. Magnetic Effects of Current & Magnetism
17
IV. Electromagnetic Induction and Alternating Currents
V. ElectromagneticWaves
18
VI. Optics
VII. Dual Nature of Matter
12
VIII. Atoms and Nuclei

IX. Electronic Devices 7

Unit 1 Electrostatics 26 Periods

Chapter 1; Electric Charges and Fields. Electric Charges; Conservation of charge, Coulomb's law-force between two
point charges, forces between multiple charges; superposition principle and continuous charge distribution.
Electric field, electric field due to a point charge, electric field lines; electric dipole, electric field due to a dipole;
torque on a dipole in uniform electric field.
applications to find field due to infinitely long straight wire,
Electric flux, statement of Gauss's theorem and its
uniformly charged infinite plane sheet and uniformly charged thin spherical shell (field inside and outside).
Chapter 2 : Electrostatic Potential and Capacitance. Electric potential, potential difference, electric potential due
to a point charge, a dipole and system of charges; equipotential surfaces, electrical potential energy of a system
of two point charges and of electric dipole in an electrostatic field.
Conductors and insulators, free charges and bound charges inside a conductor. Dielectrics and electric
polarisation, capacitors and capacitance, combination of capacitors in series and in parallel, capacitance of a
parallel plate capacitor with and without dielectric medium between the plates, energy stored in a capacitor (no
derivation, formulae only).

Unit II Current Electricity 18 Periods

Chapter 3 ; Current Electricity. Electric current, flow of electric charges in a metallic conductor, drift velocity,
mobility and their relation with electric current; Ohm's law, V-I characteristics (linear and non-Unear), electrical
energy and power, electrical resistivity and conductivity, temperature dependence of resistance.
Internal resistance of a cell, potential difference and emf of a cell, combination of cells in series and in parallel.
Kirchhoff's rules. Wheatstone bridge.
Unit III Magnetic Effects of Current & Magnetism 25 Periods

Chapter 4 : Moving Charges and Magnetism. Concept of magnetic field, Oersted's experiment.
Biot-Savart law and its application to current carrying circular loop.
Ampere's law and its applications to infinitely long straight wire. Straight solenoid (only qualitative treatment),
force on a moving charge in uniform magnetic and electric fields.
Force on a current-carrying conductor in a uniform magnetic field. Force between two parallel current-carrying
conductors - definition of ampere. Torque experienced by a current loop in uniform magnetic field ; moving coil
galvanometer - its current sensitivity and conversion to ammeter and voltmeter.

Chapter 5 : Magnetism and Matter. Bar magnet, bar magnet as an equivalent solenoid (qualitative treatment
only), magnetic field intensity due to a magnetic dipole (bar magnet) along its axis and perpendicular to its axis

w
(qualitative treatment only), torque on a magnetic dipole (bar magnet) in a uniform magnetic field (qualitative
treatment only), magnetic field lines. Current loop as a magnetic dipole and its magnetic dipole moment.

Flo
Magnetic properties of materials- Para-, dia- and ferro - magnetic substances with examples. Magnetization of
materials, effect of temperature on magnetic properties.

ee
Fr
Unit IV Electromagnetic Induction and Alternating Currents 24 Periods

Chapter 6 : Electromagnetic Induction. Electromagnetic induction ; Faraday's Laws, induced emf and current ;
Lenz's Law, Self and mutual induction.
for
ur
Chapter 7 : Alternating Current. Alternating currents, peak and rms value of alternating current/voltage ;
reactance and impedance ; LCR series circuit (phasors only), resonance ; power in AC circuits, power factor ;
ks

wattless current. AC generator and transformer.


Yo
oo
eB

Unit V Electromagnetic Waves 04 Periods

Chapter 8 : Electromagnetic Waves. Basic idea of displacement current, Electromagnetic waves, their
r

characteristics, their transverse nature (qualitative ideas only).


ou
ad

Electromagnetic spectrum (radio waves, microwaves, infrared, visible, ultraviolet. X-rays, gamma rays) including
Y

elementry facts about their uses.


nd
Re

Unit VI Optics 30 Periods


Fi

Chapter 9 : Ray Optics And Optical Instruments. Ray Optics : Reflection of light, spherical mirrors, mirror
formula, refraction of light, total internal reflection and optical fibres, refraction at spherical surfaces,
lenses, thin lens formula, lensmaker's formula, magnification, power of a lens, combination of thin lenses
in contact, refraction of Ught through a prism.
Optical Instruments : Microscopes and astronomical telescopes (reflecting and refracting) and their
magnifying powers.

Chapter 10 : Wave Optics. Wavefront and Huygen's principle, reflection and refraction of plane wave at a plane
surface using wavefronts. Proof of laws of reflection and refraction using Huygens' principle. Interference, Young's
double sUt experiment and expression for fringe width (No derivation, final expression only), coherent sources
and sustained interference of Ught. Diffraction due to a single sUt, width of central maxima (quaUtative treatment
only).
UNIT VII DUAL NATURE OF RADIATION AND MATTER 08 Periods
Chapter 11 : Dual Nature of Radiation and Matter. Dual nature of radiation. Photoelectric effect, Hertz and
Lenard's observations ; Einstein's photoelectric equation - particle nature of light.
Experimental study of photoelectric effect
Matter waves - wave nature of particles, de Broglie relation.

UNIT VIII ATOMS AND NUCLEI 15 Periods

Chapter 12 : Atoms. Alpha-particle scattering experiment; Rutherford's model of atom; Bohr model of hydrogen atom,
Expression for radius of nth possible orbit, velocity and energy of electron in this orbit, hydrogen line spectra
(qualitative treatment only).
Chapter 13 : Nuclei. Composition and size of nucleus, nuclear force.
Mass-energy relation, mass defect ; binding energy per nucleon and its variation with mass number ; nuclear
fission, nuclear fusion.

UNIT IX ELECTRONIC DEVICES 10 Periods

Chapter 14 : Semiconductor Electronics : Materials, Devices and Simple Circuits. Energy bands in conductors,
semiconductors and insulators ( qualitative ideas only). Intrinsic and extrinsic semiconductors p- and n-type, p-n
junction.
Semiconductor diode : 1-V characteristics in forward and reverse bias, application of junction diode - diode as a
rectifier.

--------•�-------
To My Grandchildren

w
Nihit, Nayraa and Jivin
whose smiling faces inspire me.
I God bless them!

Flo
ee
Fr
for
ur
ks
Yo
oo
r eB
ou
ad
Y
nd
Re
Fi
Confente

^^hapter 1 Electric Charges and Field 1.1 -1.148

1.1 Frictional Electricity 1.1 1.25 Electric Field at an Equatorial Point of a Dipole
1.2 Electric Charge 1.1 1.39

1.3 Electrostatics 1.2 1.26 Torque on a Dipole in a Uniform ElecOic Field 1.40

1.4 Two Kinds of Electric Charges 1.2 1.27 Dipole in a Non-uniform Electric Field 1.41

1.5 Electronic Theory of Frictional Electricity 1.3 1.28 Electric Field Lines 1.44

1.6 Conductors and Insulators 1.4 1.29 Electric Field Lines for Different Charged
J.7 Electrostatic Induction 1,4 Conductors 1.45

1.30 Area Vector 1.47


1.8 Basic Properties of Electric Charge 1.6

1.9 Additivity of Electric Charge 1.6 1.31 Electric Flux 1.47

1.32 Gauss's Theorem 1.48


1.10 Quantization of Electric Charge 1.6
1.33 Gaussian Surface 1.49
1.11 Conservation of Charge 1.8

1.12 Electric Charge \Js. Mass 1.8 1.34 Coulomb's Law from Gauss's Theorem 1.49

1.13 Coulomb's Law of Electric Force 1.9 1.35 Field due to an Infinitely Long Charged Wire 1.55
1.14 Coulomb's Law in Vector Form 1.10 1.36 Electric Field due to a Uniformly
Charged Infinite Plane Sheet 1.55
1.15 Dielectric Constant : Relative Permittivity 1.10

1.16 Comparing Electrostatic and 1.37 Field due to a Uniformly Charged


Qravitational Forces 1.18
Thin Spherical Shell 1.56

1.38 Field Due to a Uniformly


1.17 Forces between Multiple Charges ;
The Superposition Principle 1.19 Charged Insulating Sphere* 1.58

1.18 Electric Field 1.24 Very short answer conceptual problems 1.62

Short answer conceptual problems 1.67


1.19 Electric Field due to a Point Charge 1.28

1.20 Electric Field due to a System of Problems on higher order thinking skills 1.72

Point Charges 1.29 Guidelines to NCERT exercises 1.80

1.21 Continuous Charge Distribution 1.34 (OTQs) Objective Type Questions 1.87

1.22 Electric Dipole 1.38 Text Based Exercises 1.96

1.23 Dipole Field 1.39 Competition Section 1.109

1.24 Electric Field at an Axial Point of a Dipole 1.39

^^hapter 2 Electrostatic Potential and Capacitance 2.1 -2.180

2.1 Electrostatic Potential and Potential Difference 2.6 Electric Potential due to a Uniformly Charged Thin
2.1 Spherical Shell 2.5

2.7 Relation between Electric


2.2 Electric Potential due to a Point Charge 2.2
Field and Potential 2.11
2.3 Electric Potential due to a Dipole 2.2
2.8 Equipotential Surfaces and their Properties 2.14
2.4 Electric Potential due to a System of Charges
2.4
2.9 Equipotential Surfaces of Various
Charge Systems 2.14
2.5 Electric Potential due to a Continuous
Charge Distribution 2.4 2.10 Electric Potential Energy 2.15
2.11 Potential Energy in an External Field 2.17 2.27 Dielectrics and their Polarization 2.55

2.12 Potential Energy of a Dipole in a 2.28 Dielectric Strength 2.58


Uniform Electric Field 2.18
2.29 Capacitance of a Parallel Plate Capacitor
2.13 Conductors and Insulators 2.23 with a Dielectric Slab 2.58

2.14 Free and Bound Charges 2.23 2.30 Capacitance of a Parallel Plate Capacitor
2.15 Behaviour of Conductors in with a Conducting Slab 2.59

Electrostatic Fields 2.24 2.31 Uses of Capacitors 2.59

2.16 Electrostatic Shielding 2.25 2.32 Effect of Dielectric on Various Parameters 2.59

2.17 Electrical Capacitance of a Conductor 2.26 2.33 Discharging Action of Sharp Points :
2.18 Capacitance of an Isolated Corona Discharge 2.66

Spherical Capacitor 2.26 2.34 Collecting Action of a hollow Conductor 2.66

w
2.19 Concept of a Capacitor and its Principle 2.28 2.35 Van De Qraaff Generator* 2.66

2.20 Parallel Plate Capacitor 2.29 Very short answer conceptual problems 2.67

Flo
2.21 Spherical Capacitor* 2.30 Short answer conceptual problems 2.73

2.22 Cylindrical Capacitor* 2.30 Problems on higher order thinking skills 2.80

ee
2.23 Combination of Capacitors in Series Guidelines to hCERT exercises 2.92

and in Parallel 2.33

Fr
(OTQs) Objective Type Questions 2.103

2.24 Energy Stored in a Capacitor 2.47 Text Based Exercises 2.119

2.25 Energy Density of an Electric Field 2.47


Competition Section 2.132

for
ur
2.26 Redistribution of Charges 2.48
ks

^^hapter 3
Yo

Current Electricity 3.1 - 3.248


oo
eB

3.1 Current Electricity 3.1 3.17 Superconductivity' 3.29

3.2 Electric Current 3.1 3.18 Resistances in Series and Parallel* 3.30

3.3 Maintenance of Steady Current in a Circuit 3.4 3.19 Internal Resistance of a Cell 3.45
r

3.4 Electromotive Force : EMF 3.4 3.20 Relation between Internal Resistance, EMF
ou
ad

3.5 EMF vs. Potential Difference 3.5 & Terminal Potential Difference of a Cell 3.45
Y

3.6 OhM's Law : Resistance 3.6 3.21 Combinations of Cells in Series & Parallel 3.51

3.22 heating Effect of Current 3.59


3.7 Factors Affecting the Resistance :
nd
Re

Resistivity 3.6 3.23 heat Produced By Electric Current :


Joule's Law 3.60
3.8 Current Density, Conductance and
Fi

Conductivity 3.7 3.24 Electric Power 3.60

3.9 Classification of Materials in 3.25 Electric Energy 3.61

Terms of Resistivity 3.8 3.61


3.26 Power Rating
3.10 Colour Code for Carbon Resistors* 3.9
3.27 Power Consumption in a Combination of
3.11 Carriers of Current 3.15 Appliances 3.61

3.12 Mechanism of Current Flow in a Conductor : 3.28 Efficiency of a Source of Emf 3.62

Drift Velocity and Relaxation Time 3.15


3.29 Efficiency of an Electric Device 3.63

3.13 Relation between Electric Current and


3.30 Applications of Heating
Drift Velocity ; Derivation of Ohm's Law 3.16 Effect of Current 3.64

3.14 Mobility of Charge Carriers 3.22 3.31 Kirchhoffs Laws 3.79

3.15 Temperature Dependence of Resistivity 3.24 3.32 Potentiometer’ 3.96

3.16 Limitations of Ohm's Law : 3.96


3.33 Applications of a Potentiometer*
Ohmic and Mon-ohmic Conductors 3.28
3.34 Sensitiveness of a Potentiometer* 3.98
3.35 Wheatstone Bridge 3.104 Guidelines to hCERT exercises 3.148

3.36 Metre Bridge or Slide Wire Bridge 3.105 (OTQs) Objective Type Questions 3.155

Very short answer conceptual problems 3.115 Text Based Exercises 3.178

Short answer conceptual problems 3.122 Competition Section 3.192

Problems on higher order thinking skills 3.131

apter 4 Magnetic Effect of Current 4.1 -4.192

4.1 Concept of Magnetic Field 4.1 4.16 Cyclotron* 4.40

4.2 Oersted's Experiment 4.1 4.17 Force on a Current Carrying Conductor in a


4.3 Biot-Savart Law 4.2 Magnetic Fieid 4,43

4.4 Biot-Savart Law vs. Coulomb's Law 4.3 4.18 Forces between Two Parallel

4.5 Current-carrying Conductors 4.48


Magnetic Field due to a Long Straight
4.19 Torque Experienced by a Current Loop in a
Current Carrying Conductor* 4.5

4.6 Uniform Magnetic Field 4.53


Magnetic Field at the Centre of
4.20 Moving Coil Galvanometer 4.57
Circular Current Loop 4.12

4.7 4.21 Sensitivity of a Galvanometer 4.59


Magnetic Field on the Axis of a
Circular Current Loop 4.13 4.22 Measurement of Current and Voltage 4.62

4.8 Ampere's Circuital Law and its Application 4.23 Conversion of a Gaivanometer into an
to Infinitely Long Straight Wire 4.22 Ammeter 4.63
4.9 Magnetic Field Inside a Straight Solenoid 4.23 4.24 Conversion of a Galvanometer into a

4.10 Magnetic Field due to a Toroidal Solenoid 4.24 Voltmeter 4.63

4.11 Force on a Moving Charge in a Very short answer conceptual problems 4.73

Magnetic Field 4.28 Short answer conceptual problems 4.79

4.12 Lorentz Force 4.30 Problems on higher order thinking skilis 4.87

4.13 Work done by a Magnetic Force on a Guideiines to MCERT exercises 4.96

Charged Particle is Zero 4.33


(OTQs) Objective Type Questions 4.103
4.14 Motion of a Charged Particle in a Text Based Exercises 4.120
Uniform Magnetic Field 4.33
Competition Section 4.136
4.15 Motion of a Charge in Perpendicular
Magnetic and Electric Fields 4.34

(Chapter 5 Magnetism 5.1-5.108

5.1 Introduction 5.1 5.9 Magnetic Field of a Bar Magnet at


5.2 Artificial Magnets 5.1 an Equatorial Point 5.7

5.3 5.10 Torque on a Magnetic Dipole in


Basic Properties of Magnets 5.2
5.4 a Magnetic Field* 5.10
Some Important Definitions Connected
5.11 Potential Energy of a Magnetic Dipole in a
with Magnetism 5.2

5.5 Magnetic Field 5.11


Coulomb's Law of Magnetic Force 5.3
5.12 Current Loop as a Magnetic Dipole 5.11
5.6 Magnetic Dipoie & Magnetic Dipole Moment 5.3
5.13 Magnetic Dipole Moment of
5.7 Magnetic Field Lines 5.6

5.8
a Revolving Electron 5.12
Magnetic Field of a Bar Magnet at an
5.14 Bar Magnet as an Equivalent Solenoid 5.17
Axial Point 5.7

L
5.15 Gauss's Law in Magnetism 5.19 5.28 Properties of Diamagnetic Substances 5.32

5.16 Magnetic Field of the Earth' 5.19 5.29 Properties of Paramagnetic Substances 5.33

5.17 Origin of Earth's Magnetic Field* 5.20 5.30 Properties of Ferromagnetic Substances 5.34

5.18 5ome Definitions in Connection 5.31 Hysteresis 5.36

with Earth's Magnetism* 5.20


5.32 Permanent Magnets and Electromagnets 5.40

5.19 Elements of Earth's Magnetic Field* 5.21


5.33 Tangent Galvanometer 5.41
5.20 Global Variations in the
5.34 Oscillations of a Freely Suspended Magnet 5.42
Earth's Magnetic Field* 5.22
5.35 Vibration Magnetometer 5.43
5.21 Temporal Variations in the 5.45
Very short answer conceptual problems
Earth's Magnetic Field* 5.23
Short answer conceptual problems 5.49
5.22 lieutral Point

w
5.23
Problems on higher order thinhing skills 5.55
5.23 Some Important Terms used to Describe
Guidelines to MCERT exercises 5.61
Magnetic Properties of Materials 5.29
(OTQs) Objective Type Questions 5.68

Flo
5.24 Classification of Magnetic Materials 5.30
Text Based Exercises 5.77
5.25 Origin of Diamagnetism 5.30
Competition Section 5.87

ee
5.26 Origin of Paramagnetism 5.31

Fr
5.27 Origin of Ferromagnetism :
Domain Theory 5.31

for
ur
^^hapter 6 Electromagnetic Induction 6.1 - 6.98
ks
6.1 Magnetic Flux 6.1 6.13 Self-inductance of a Long Solenoid 6.21
Yo

6.2 Electromagnetic Induction : An Introduction 6.2 6.14 Phenomena Associated with Self-induction 6.21
oo

6.3 Faraday's Experiments 6.2 6.15 Mutual Induction 6.22


eB

6.4 Laws of Electromagnetic Induction 6.4 6.16 Mutual Inductance of Two Long Solenoids 6.23

6.5 Explanation of Lenz's Law 6.4 6.17 Grouping of Inductances* 6.24


r

6.6 Motional EMF from Faraday's Law 6.9 Very short answer conceptual problems 6.30
ou
ad

Short answer conceptual problems 6.35


6.7 Fleming's Right Hand Rule 6.9
Y

6.8 Motional EMF from Lorentz Force Problems on higher order thinking skills 6.44

and Energy Consideration 6.9 Guidelines to MCERT exercises 6.51


nd

6.9 Relation between Induced Charge and (OTQs) Objective Type Questions 6.57
Re

Change in Magnetic Flux 6.10


Text Based Exercises 6.64
Fi

6.10 Methods of Generating Induced Emf 6.15


6.71
Competition Section
6.11 Eddy Currents 6.18

6.12 Self-induction 6.20

^Jhapter 7 Alternating Current and Electrical Machines 7.1 - 7.134

7.1 Alternating Current 7.1 7.6 A.C. Circuit Containing only a Resistor 7.7

7.2 Mean or Average Value of A.C. 7.2 7.7 A.C. Circuit Containing only an Inductor 7.7

7.3 Root Mean Square (RMS) or Virtual or 7.8 A.C. Circuit Containing only a Capacitor 7.8

Effective Value of A.C. 7.3 7.9 A.C. Circuit with Resistance and

7.4 Root Mean Square Value of an Inductance in Series 7.12

Alternating EMF 7.3 7.10 A.C. Circuit with Resistance and

7.5 Phasors and Phaser Diagrams 7.6 Capacitor in Series 7.17


7.11 Series LCR-circuit 7.21 7.23 Uses of Transformers 7.45

7.12 Resonance Condition of a 7.24 Long Distance Transmission


Series LCR-circuit 7.22 of Electrical Power 7.45

7.13 Sharpness of Resonance ; Q-factor 7.23 7.25 A.C, Generator 7.48


7.14 Choke Coil* 7.30 7.26 Advantages and Disadvantages of
7.15 Power in an A.C. Circuit 7.30 A.C. Over D.C, 7.50

7.16 Power Factor 7.31 Very short answer conceptual problems 7.52

7.17 Wattless Current 7.31 Short answer conceptual problems 7.55

7.18 Average Power Associated Problems on higher order thinking skills 7.63

with a Resistor 7.32 Guidelines to MCERT exercises 7.68

7.19 Energy and Average Power Associated (OTQs) Objective Type Questions 7.75
with a Pure Inductor 7.32 Text Based Exercises 7.86

7.20 Energy and Average Power Associated Competition Section 7.99


with a Pure Capacitor 7.33

7.21 LC-OsdIlations 7.39


7.22 Transformer 7.43

Chapters 8 to 15 are in Vol. II

* marked articles are not included in the latest CBSE syllabus.


□ □□□
D iJD
C H A PTER 1 DP
D

' ■ *i‘

EIecMc Charges

w
and Field

Flo
ee
Fr
for
ur
II FRICTIONAL ELECTRICITY dectrica for such substances. In fact, the Greek name for
amber is elektron which is the origin of all such words :
1. WJiat is frictional electricity ? When is a body said electricity, electric force, electric charge and electron.
ks
to be electrified or charged ?
Yo

Frictional electricity. If a glass rod is rubbed with For Your Knowledge


oo

a
silk cloth, or a fountain-pen with a coat-sleeve, it is able
> Amber is a yellow resinous (gum like) substance
eB

to attract small pieces of paper, straw, lint, light feathers,


found on the shores of the Baltic sea.
etc. Similarly, a plastic comb passed through dry hair
can attract such light objects. In all these examples, we > Both electric and magnetic phenomena can be derived
r

can say that the rubbed substance has become electrified from charged particles. Magnetism arises from
ou

charges in motion. The charged particles in motion


ad

or electrically charged. It is because of friction that the


substances get charged on rubbing. exert both electric and magnetic forces on each other.
Y

Hence electricity and magnetismare studied together


The property of rubbed substances due to which thei/
as electromagnetism.
attract light objects is called electricity. The electricity
nd
Re

developed by rubbing or friction is called frictional or static 12 ELECTRIC CHARGE


electricity. The rubbed substances lohich show this p^roperty
Fi

of attraction are said to have become electrified or 3. What is electric charge ? Is it a scalar or vector
electrically charged. quantity ? Name its SI unit.
2. Give a historical view offrictional electricity. From Electric charge. Electric charge is an intrinsic property
of the elementary particles like electrons, protons, etc.,
where did the term electricity get its origin 7 of which all the objects are made up of. It is because of
Historical vieTv of frictional electricity. In 600 B.C., these electric charges that various objects exert strong
Tljales of Miletus, one of the founders of Greek science, electric forces of attraction or repulsion on each other.
first noticed that if a piece of amber is rubbed with a Electric charge is an intrinsic property of elementary
woollen cloth, it then acquires the property of attrac particles of matter zohich gives rise to electric force between
ting light feathers, dust, lint, pieces of leaves, etc. various objects.
In 1600 A.D., William Gillbert, the personal doctor to Electric charge is a scalar quantity. Its SI unit is
Queen Elizabeth -1 of England, made a systematic study coulomb (C). A proton has a positive charge (-i- e) and an
of the substances that behave like amber. In his book electron has a negative charge (-e), where
-19
De Magnete (on the magnet), he introduced the name e = 1.6xl0 coulomb

(1.1)
1.2 PHYSICS-XII

Large-scale matter that consists of equal number of (n) Bring a plastic rod rubbed with wool near the
electrons and protons is electrically neutral. If there is charged glass rod. The two rods attract each
an excess of electrons, the body has a negative charge other [Fig. l-l(b)].
and an excess of protons results in a positive charge. (Hi) Now rub a plastic rod with wool and suspend it
from a rigid support. Bring another similarly
1.3 ELECTROSTATICS charged plastic rod near it. There will be a
4. What is electrostatics ? Mention some of Us repulsion between the two rods [Fig. 1.1(c)).
important applications. Exi’ERIMENT 2. If a glass rod, rubbed with silk, is
Electrostatics. Electrostatics is the study of electric made to touch two small pith balls (or polystyrene
charges at rest. Here we study the forces, fields and balls) which are suspended by silk threads, then the two
potentials associated with static charges. balls repel each other, as shown in Fig. 1.2(a). Similarly,
Applications of electrostatics. The attraction and two pith balls touched with a plastic rod rubbed with
repulsion between charged bodies have many indus fur are found to repel each other [Fig. 1.2(1>)]. But it is
trial applications. Some of these are as follows : seen that a pith ball touched with glass rod attracts
1. In electrostatic loudspeaker. another pith ball touched with a plastic rod [Fig. 1.2(c)).
2. In electrostatic spraying of paints and powder
coating.
3. In flyash collection in chimneys. ^^Silk
4. In a Xerox copying machine. \ thread
5. In the design of a cathode-ray tube used in
television and radar.

(fl) Repulsion (1>) Repulsion


1.4 TWO KINDS OF ELECTRIC CHARGES
5. Hoio will you show experimentally that (i) there are
only two kinds of electric charges and (ii) like charges
repel and unlike charges attract each other ?
Two kinds of electric charges. About 100 years ago,
Charles Du Fay of France showed that electric charges
on various objects are of only two kinds. The following
simple experiments prove this fact.
(c) Attraction
Experiment 1

(i) Rub a glass rod with silk and suspend it from a Fig. 1.2 Like charges repel and unlike charges attract.
rigid support by means of a silk thread. Bring
another similarly charged rod near it. The two From the above experiments, we note that the
rods repel each other [Fig. 1.1(a)). charge produced on a glass rod is different from the
charge produced on a plastic rod. Also the charge
produced on a pith ball touched with a glass rod is
different from the charge produced on pith ball
touched with a plastic rod. We can conclude that :
VXasyS * Attraction
1. There are only two kinds of electric charges - positive
and negative.
2. Like charges repel and unlike charges attract each other.
The statement 2 is known as the fundamental law
Silk of electrostatics.
Plastic The above experiments also demonstrate that the
Repulsion charges are transferred from the rods to the pith balls
on contact. We say that the pith balls have been
electrified or charged by contact. This property which
(c)
distinguishes the two kinds of charges is called the polarity
Fig. 1.1 Like charges repel and unlike charges of charge.
attract each other.
ELECTRIC CHARGES AND FIELD 1.3

6. Wliat are vitreous and resinous charges ? What turns out to be negative in this convention. It would
was wrong zvith this nomenclature ? have been more convenient if electrons were assigned
Vitreous and resinous charges. Charles Du Fay used the positivecharge. But in science, sometimeswe have to
live with the historical conventions.
terms vitreous and resinous for the tioo kinds of charges.
1. The charge developed on glass rod zvben rubbed zoith > Different substances can be arranged in a series in
silk zoos called vitreous charge (Latin virtuni = glass). such a way that if any two of them are rubbed together,
then the one occurring earlier in the series acquires a
2. The charge developed on amber when rubbed with
positive charge while the other occurring later acquires
wool zoas called resinous charge (amber is a resin).
a negative charge :
But later on, these terms were found to be
1. Fur 2. Flannel 3. Sealing wax
misleading. For example, a ground glass rod develops 4. Glass 5. Cotton 6. Paper
resinous electricity while a liighly polished ebonite rod
7. Silk 8. Human body 9. Wood
develops vitreous electricity.

w
10. Metals 11. Rubber 12. Resin
7. What are positive and negative charges ? What is 13. Amber 14.
the nature of charge on an electron in this convention ? Sulphur 15. Ebonite

16. Guta parcha


Positive and negative charges. Benjamin Franklin

Flo
(1706-1790), an American pioneer of electrostatics Thus glass acquires a positive charge when rubbed
introduced the present-day convention by replacing with silk but it acquires negative charge when rubbed

ee
the terms vitreous and resinous by positive and with flannel.

Fr
negative, respectively. According to this convention :

1. The charge developed on a glass rod zohen rubbed 15 ELECTRONIC THEORY OF FRICTIONAL
with silk is called positive charge. ELECTRICITY
for
ur
2. The charge developed on a plastic rod zohen rubbed
8. Describe the electronic theory of frictional
zvith wool is called negative charge.
electricity. Are the factional forces electric in origin ?
The above convention is consistent with the fact
ks
that when two opposite kinds of charges are brought in Electronic theory of frictional electricity. All matter
Yo

is made of atoms. An atom consists of a small central


contact, they tend to cancel each other's effect. According
oo

to this convention, the charge on an electron is negative. nucleus containing protons and neutrons, around
which revolve a number of electrons. In any piece of
eB

Table 1.1 gives a list of the pairs of objects which get matter, the positive proton charges and the negative
charged on rubbing against each other. On rubbing, an electron charges cancel each other and so the matter in
object of column I will acquire positive charge while
r

bulk is electrically neutral.


that of column II will acquire negative charge.
ou
ad

The electrons of the outer shell of an atom are


Table 1.1 Two kinds of charges developed on
rubbing loosely bound to the nucleus. Tine energy requiredto
Y

remove an electron from the surface of a material is


Column I Column n
called its 'work function'. When two different bodies
nd

(Positive charge) (Negative charge)


Re

are rubbed against each other, electrons are transferred


Glass rod Silk cloth from the material with lower work function to the
Fi

Flannel or cat skin Ebonite rod material with higher work function. For example,
Woollen cloth Amber rod when a glass rod is rubbed with a silk cloth, some
Woollen coat Plastic seat electrons are transferred from glass rod to silk. The
Woollen carpet Rubber shoes
glass rod develops a positive charge due to deficiency
of electrons while the silk cloth develops an equal
Obviously, any two charged objects belonging to negative charge due to excess of electrons. The
combined total charge of the glass rod and silk cloth is
the same column will repel each other while those of
two different columns will attract each other. still ;:ero, as it was before rubbing i.e., electric charge is
conserved during rubbing.
For Your Knowledge Electric origin of frictional forces. The only way by
> Benjamine's choice of positive and negative charges is which an electron can be pulled away from an atom is
purely conventional one. However, it is unfortunate to exert a strong electric force on it. As electrons are
that the charge on an electron (which is so important actually transferred from one body to another during
to physical and chemical properties of materials) Ribbing, so frictional forces must have an electric origin.
PHYSICS-Xll

For Your Knowledge


^ The cause of charging is the actual transfer of elec
trons from one material to another during rubbing.
Protons are not transferred during rubbing. U UJ
U UJ

> The material with lower work function loses electrons


(«) {b)
and becomes positively charged.
> As an electron has a finite mass, therefore, there always Fig. 1.3 Neutralisation of a (a) positively charged,
occurs some change in mass during charging. The (b) negatively charged, earthed body.
mass of a positively charged body slightly decreases The electricity from the mains is supplied to our
due to loss of some electrons. The mass of a negatively houses using a three-core wiring : live, neutral and
charged body slightly increases due to gain in some earth wires. The live wire red in colour brings in the
electrons. current. The black neutral wire is the return wire. The
green earth wire is connected to a thick metal plate
1.6 CONDUCTORS AND INSULATORS buried deep into the earth. The metallic bodies of the
9. How do the conductors differ from the insulators ? electric appliances such as electric iron, refrigerator, TV,
etc. are connected to the earth wire. When any fault
Why cannot we electrify a metal rod by rubbing it while occurs or live wire touches the metallic body, the charge
holding it in our hand ? How can we charge it ? flows to the earth and the person who happens to touch
Conductors. TJjc substances through which electric
the body of the appliance does not receive any shock.
charges can flow easily are called conductors. They contain
a large number of free electrons which make them ^ J ELECTROSTATIC INDUCTION
good conductor of electricity. Metals, human and animal
bodies, graphite, acids, alkalies, etc. are conductors. 11. Wlmt is meant by electrostatic mduction ?
Insulators. The substances through which electric charges Electrostatic induction. As shown in Fig. 1.4, hold a
cannot flow easily are called insulators. In the atoms of conducting rod AB over an insulating stand. Bring a
near its end A The free
such substances, electrons of the outer shell are tightly positively charged glass rod
bound to the nucleus. Due to the absence of free charge electrons of the conducting rod get attracted towards
carriers, these substances offer high resistance to the the end Awhile the end Bbecomes electron deficient,
flow of electricity through them. Most of the non- The closer end A acquires a negative charge while the
metals like glass, diamond, porcelain, plastic, nylon, remote end B acquires an equal positive charge. As
wood, mica, etc. are insulators. soon as the glass rod is taken away, the charges at the
An important difference between conductors and ends A and B disappear, Conducting rod
insulators is that when some charge is transferred to a
conductor, it readily gets distributed over its entire
A B
surface. On the other hand, if some charge is put on an
insulator, it stays at the same place. We shall discuss
Excess of Deficiency of
this distinguishing feature in the next chapter. electrons electrons
A metal rod held in hand and rubbed with wool
does not develop any charge. This is because the
human body is a good conductor of electricity, so any Insulating
stand
charge developed on the metal rod is transferred to the
earth through the human body. We can electrify the
rod by providing it a plastic or a rubber handle and Fig. 1.4 Electrostatic induction.
rubbing it without touching its metal part.
10. What is meant by earthing or grounding in Electrostatic induction is the phenomenon of
household circuits ? What is its importance ? temporary electrification of a conductor in which opposite
Earthing and safety. When a charged body is charges appear at its closer md and similar charges appear at
brought m?ontacf with the earth (through a connecting Us farther end in the presence of a nearbp charged body.
conductor), its entire charge passes to the ground in The positive and negative charges produced at the
the form of a momentary current. This process in which a ends of the conducting rod are called induced charges and
body shares its charges loith the earth is called grounding or the charge on the glass rod which induces these
earthing. charges on conducting rod is called inducing charge.
ELECTRIC CHARGES AND FIELD 1.5

12. Describe how tzuo metal spheres can he opposite!}/ 13. Hoiv can you charge a metal sphere positively
charged by induction. loithout touching it ?
Charging of two spheres by induction. Figure 1.5 Charging of a sphere by induction. Fig. 1.6 shows
shows the various steps involved in inducing opposite the various steps involved in inducing a positive
charges on two metal spheres. charge on a metal sphere,
(fl) Hold the metal sphere on an insulating stand.
Bring a negatively charged plastic rod near it.
The free electrons of the sphere are repelled to
the farther end. The near end becomes posi
tively charged due to deficit of electrons,
(ij) When the far end of the sphere is connected to
the ground by a connecting wire, its free
electrons flow to the ground,

w
(c) When the sphere is disconnected from the
ground, its positive charge at the near end
remains held there due to the attractive force of

Flo
the external charge,
(rf) When the plastic rod is removed, the positive

ee
charge spreads uniformly on the sphere.

Fr
Plastic
Fig. 1.5 Two metal spheres get oppositely charged by induction,
for
ur
Ground
(fl) Hold the two metal spheres on insulating stands
(/’)
and place them in contact, as shown in Fig. 1.50?).
ks
(b) Bring a positively charged glass rod near the left
Yo

sphere. The free electrons of the spheres get attracted


oo

towards the glass rod. The left surface of the left sphere
develops an excess of negative charge while the right
eB

(rf)
side of the right sphere develops an excess of positive
charge. However, all of the electrons of the spheres do
Fig. 1.6 Charging by induction.
not collect at the left face. As the negative charge
r
ou

begins to build up at the left face, it starts repelling the Similarly, the metal sphere can be negatively charged
ad

new incoming electrons. Soon an equilibrium is by bringing a positively charged glass rod near it.
Y

established under the action of force of attraction of the


rod and the force of repulsion due to the accumulated For Your Knowledge
electrons. The equilibrium situation is shown in
nd
Re

Fig. 1.5(b). Gold-leaf electroscope. It is a device used for detecting


an electric charge and identifying its polarity. It
Fi

(c) Holding the glass rod near the left sphere, sepa consists of a vertical conducting rod passing through
rate the two spheres by a small distance, as shown in a rubber stopper fitted in the mouth of a glass vessel.
Fig. 1.5(c). The two spheres now have opposite charges. Two tlnn gold leaves are attached to lower end of the
(d) Remove the glass rod. The charges on the spheres rod. When a charged object touches the metal knob at
get redistributed. Their positive and negative charges the outer end of the rod, the charge flows down to the
face each other, as shown in Fig. 1.5(rf). The two leaves. The leaves
spheres attract each other, diverge due to Metal
●Metal rod
(e) When the two spheres are separated quite apart, repulsion of the like knob
- Rubber stopper
the charges on them get uniformly distributed, as charges they have
shown in Fig. 1.5(e). received. Tlie degree <—'Glass vessel
of divergent^ of the
Thus the two metal spheres get charged by a
leaves gives a Gold leaf
process called charging by induction. In contrast to the measure of the
process of charging by contact, here the glass rod does
not lose any of its charge.
amount of charge. l: Tin foil
1.6 PHYSICS-XII

18 BASIC PROPERTIES OF ELECTRIC CHARGE Vie experimental fact that electric charges occur in
discrete amounts instead of continuous amounts is called
It is observed from experimeiits that electric charge quantization of electric charge. The quantization of electric
has following three basic properties : charge means that the total charge (q) of a body is always an
1. Additivity 2. Quantization 3. Conservation. integral multiple of a basic quantum of charge (e), i.e.,
We shall discuss these properties in detail in the q = ne , where =0, + 1, ± 2, ± 3,
next few sections.
Cause of quantization. The basic cause of quanti
zation of electric charge is that during rubbing only an
19 ADDITIVITY OF ELECTRIC CHARGE
integral number of electrons can be transferred from
14. What do you mean by additive nature of electric one body to another.
charge ? Quantization of electric charge is an experi-
Additive nature of electric charge. Like mass, mentally verified law :
electric charge is a scalar quantity. Just as the mass of 1. The experimental laws of electrolysis discov
an extended body is the sum of the masses of its ered by Faraday first suggested the quanti
individual particles, the total charge of an extended zation of electric charge.
body is the algebraic sum {i.e., the sum taking into 2. Millikan's oil drop experiment in 1912 on the
account the positive and negative signs) of all the measurement of electric charge further estab
charges located at different points inside it. Thus, the lished the quantization of electric charge.
electric charge is additive in nature. 17. Can zve ignore the quantization of electric
Additivity of electric charge means that the total charge ? If yes, under zuhat conditions ?
charge of a system is the algebraic sum of all the individual When can we ignore the quantization of electric
charges located at different points inside the system.
cliarge. While dealing with macroscopic charges (q = ne),
If a system contains charges , q^^, then its we can ignore the quantization of electric charge. This
total charge is is because the basic charge e is very small and n is very
q = q^+q2 + large in most practical situations, so q behaves as if it
were continuous i.e., as if a large amount of charge
Tlie total charge of a system containing four were flowing. For example, when we switch on a 60 W
charges 2 pC, - 3 pC 4 pC and - 5 pC is bulb, nearly 2 x 10^® electrons pass through its filament
q = 2 pC-3pC + 4pC-5pC=-2 pC per second. Here the graininess or structure of charge
does not show up i.e., the bulb does not flicker with the
110 QUANTIZATION OF ELECTRIC CHARGE entry of each electron. Quantization of charge becomes
15. What is meant by quantization of a physical important at the microscopic level, where the charges
involved are of the order of a few tens or hundreds of e.
quantity ?
Quantization of a physical quantity. Vie quanti For Your Knowledge
zation of a physical quantity means that it cannot vary conti > The smallest amount of charge or basic quantum of
nuously to have any arbitrary value but it can change disconti- charge is the charge on an electron-19or a proton. Its
nuously to take any one of only a discrete set of values. For exact magnitude is c = 1.602192 x 10 c
example, a building can have different floors (ground, > Quantization of electric charge cannot be explained on
first, second, etc.) from the ground floor upwards but it the basis of classical electrodynamics or even modem
cannot have a floor of the value in-between. Thus the
physics. However, the physical and cliemical properties
energy of an electron in atom or the electric charge of a of atoms, molecules and bulk matter cannot be explained
system is quantized. Vie minimum amount by zvhich a without considering the quantization of electric charge.
physical quantity can change is called its quantum. > Recent discoveries in high energy physics have indi
16. What is meant by quantization of electric charge ? cated that the elementary particles like protons and
What is the cause of quantization of electric charge ? neutrons are themselves built out of more elementary
units, called quarks, which have charges (2/3) e and
Quantization of electric charge. It is found
(- 1/3) £ Even if quark-model is established in future,
experimentally that the electric charge of any body, the quantization of charge will still hold. Only the
large or small, is always an integral multiple of a quantum of charge will reduce from e to e/3.
certain minimum amount of charge. This basic charge > Quantization is a universal law of nature. Like charge,
is the charge on an electron, which is denoted by e and energy and angular momentum of an electron are also
has magnitude 1.6 x 10"^^ coulomb. Thus the charge on quantized. However, quantization of mass is yet to be
an electron is -e, on a proton is + e and that on established.
a-particle is -i- 2 c.
ELECTRIC CHARGES AND FIELD l.l

Examples based on Example 4. How much positive and negative charge is


Quantisation of Electric Charge there in a cup of water ? (NCERT]

Formulae Used
Solution. Suppose the mass of water contained in a
cup is 250 g. The molecular mass of water is 18 g.
1. (/ = ne
Number of molecules present in 18 g of water
2. Mass transferred during charging = xn 23
= Avogadro's number =6.02 x 10
Units Used
Number of molecules present in a cup (or 250 g)
q and e are in coulomb, n is pure integer. of water

Constants Used 6.02 X 10^^ X 250 24


n = = 8.36x10
-19
c= 1.6x10 C, in, = 9.1 X 10“^^ kg 18

Each molecule of water (H^O) contains 2 +8=10


Example 1. Wiiich is bigger-a coulomb or a charge on

w
an electrons as well as 10 protons.
electron ? Hozo many electronic charges form one coulomb of Total number of electrons or protons present in a
charge ? [Haryana 01)
cup of water.
Solution. One coulomb of charge is bigger than the

Flo
25
n'= n X 10 =8.36 x 10
charge on an electron.
-19 Total negative charge carried by electrons or total
Charge on one electron, e = 1.6 x 10 c

ee
positive charge carried by protons in a cup of water,
.●. Number of electronic charges in 1 coulomb,

Fr
1C
q = n'e
»=2 = 6.25x 10^®. = 8.36 X 10
25
X 1.6x10
-19
C = 1.33x lO^C
e 1.6x10 -19 c

for
ur
Example 2. A comb drawn through person's hair on a dry roblems For Practice
day causes 10^^ electrons to leave the person's hair and stick
to the comb. Calculate the charge carried by the comb. 1. Calculate the charge carried by 12.5 x 10® electrons.
ks
Solution. Here n =10^^ e =1.6 x 10“^^ C ICBSE D 92]
Yo

-10
oo

(Ans. 2 X 10 C)
i/ = w = 10“xl.6xl0
-19
= 1.6xlO^C
As the comb has excess of electrons,
2. How many electrons would have to be removed
eB

from a copper penny to leave it with a positive


.●. Charge on comb = -1.6 x 10® C. charge of 10“^ C ? (Ans. 6.25 x lo” electrons)
Example 3. If a body gives out 10^ electrons every second, 3. Calculate the charge on an alpha particle. Given
r

how much time is required to get a total charge ofl Cfrom charge on a proton = 1.6 x 10”'^ C.
ou
ad

it ? [NCERT] (Ans. + 3.2xlO‘^^Q


Y

Solution. Number of electrons given out by the 4.


Calculate the charge on 2^Fe nucleus. Given charge on
body in one second =10^ a proton = 1.6 x 10“^^ C. (Ans. + 4.16 x 10"^® C)
Charge given out by the body in one second
nd

5. Determine the total charge on 75.0 kg of electrons.


Re

= ;ie = 10^xl.6xl0“^^C (Ans.-1.33x10^® C)


Fi

= 1.6x 10 — 10 ^ 6. How many mega coulombs of positive (or


Time required to get a charge of 1.6 x 10
-10
C negative) charge are present in 2.0 mole of neutral
= 1 s hydrogen (H2) gas ?
7. Estimate the total number of electrons present in
Time required to get a charge of 1 C 100 g of water. How much is the total negative
1
-10
s = 6.25 X 10’s charge carried by these electrons ? Avogadro's
1.6 X 10 number = 6.02 x 10^ and molecular mass of water
6.25 X 10^ = ia (Ans. 5.35 x 10® Q
years = 198.18 years.
365 X 24 X 3600
HINTS

Thus from a body emitting 10^ electrons per 3. An alpha particle contains 2 protons and
second, it will take nearly 200 years to get a charge of 2 neutrons.
1 C from that body. This shows how large is one
q = + 2e.
coulomb as the unit of charge.
PHYSICS-XIl

Fe nucleus contains 26 protons and 30 neutrons. 4. Electric charge is conserved during the
q = + 26 e phenomenon of pair production in which a y-ray
Total mass 75.0 25 31 photon materialises info an electron-positron pair.
5. n = = — xlO
Mass of an electron 9x10 -31 3 y-ray electron + positron
25 zero charge (-e) (+e)
-rn xlO^' xl.6 xl0'^^=-1.33 xlO'^ C
5. In annihilation of fnatter, an electron and a posi
tron on coming in contact destroy each other,
6. Number of moleailes in 2.0 mole of H2 gas
= 2.0 X 6.02 X10
23 producing two y-ray photons, each of energy
0.51 MeV.
As each H2 molecule contains 2 electrons/protons,
23
so electron + positron + 2 y - rays
fi = 2 X 2.0 X 6.02 X 10^ = 24.08 x 10 zero charge
23 -19
i-e) (+C)
q = ne = 24.08 x 10 X 1.6 X 10
= 0.3853 X 10*^ C = 0.3853 MC. For Your Knowledge
[1 MC=10®a
> Conservation of charge implies that electric charges
7, Proceed as in Example 4. can be created or destroyed always in tine form of equal
and opposite pairs but never in isolation. For example,
1.11 CONSERVATION OF CHARGE in the beta decay of a neutron (zero charge), a proton
(charge + e) and an electron (cliarge - c) are produced.
18. State the law of conservation of charge. Give some Total charge remains zero before and after the decay.
exai7tples to illustrate this law.
> Tlie law of conservation of charge is an exact law of
Law of conservation of charge. If some amount of nature. It is valid in all domains of nature. Even in the
matter is isolated in a certain region of space and no domains of high energy physics, where mass changes
matter either enters or leaves this region by moving into energy and vice-versa, the law of conservation of
across its boundary, then whatever other changes may charge strictly holds good.
occur in the matter inside, its total charge will not
change with time. This is the law of conservation of 2.12 ELECTRIC CHARGE VS AAASS
charge which states :
19. Compare the properties of electric charge with
1. The total charge of an isolated system remains constant. those of mass of a body.
2. The electric charges can neither be created nor destroyed,
they can only be transferred from one body to another. I able 1.2 Comparirisen of the properties of
The law of conservation of charge is obeyed both in electric chorge ond mass
large scale and microscopic processes. In fact, charge Electric charge Mass
conservation is a global phenomenon i.e., total charge of
the entire universe remains constant. 1. Electric charge may be Mass of a body is
positive, negative or zero. always positive.
Examples : 2. Electric charge is always Quantization of mass is
1. When a glass rod is rubbed with a silk cloth, it quantized : q = 7ie not yet established.
develops a positive charge. But at the same 3. Charge on a body does Mass of a body
time, the silk cloth develops an equal negative not depend on its speed. increases with its speed.
charge. Thus the net charge of the glass rod and 4. Charge is strictly Mass is not conserved by
the silk cloth is zero, as it was before rubbing. conserved. itself as some of the mass

2. The rocksalt ionises in aqueous solution as may get changed into


follows :
energy or vice versa.
5. Electrostatic forces Gravitational forces
NaCl ^ Na* + Cr
betw'een two charges between two masses are

As the total charge is zero before and after the may be attractive or always attractive.
ionisation, so charge is conserved. repulsive.
235
6. Electrostatic forces Gravitational forces
3. Charge is conserved during the fission of a 92 U between different between different bodies
nucleus by a neutron.
charges may cancel out. never cancel out.
Ba -t- 3^Kr + 3 Ju + Energy
1 235 141
U-> 56
0
n +
92 7. A charged body always A body possessing
possesses some mass. mass may not have any
Total charge before fission (0 + 92) net charge.
= Total charge after fission (56 -1- 36 -1- 3 x 0)
ELECTRIC CHARGES AND FIELD 1.9

20. How does the speed of an electrically charged where is called pennittivili/ of free space. So we can
particle affect its (i) mass and (ii) charge ? express Coulomb's law in SI units as
Effect of speed on mass and electric charge. According F =
1

to the special theory of relativity, the mass of a body 4TC 8„0


increases with its speed in accordance with the relation:
Units of charge. (/) The SI unit of charge is coulomb. In
Z!!o
m =
the above equation, ii =q2=lC and r = 1 m, then
1- 1
?■ f = = 9x 10^ N
4n £
0
where, Wq = rest mass of the body, c = speed of light, So one coulomb is that amount of charge that repels an
and m = mass of the body when moving with speed v. equal and similar charge with a force of 9x 10^ N when
As V <c, therefore, m > niQ. placed in vacuum at a distance of one metre from it.

w
In contrast to mass, the charge on a body remains (ii) In electrostatic cgs system, the unit of charge is
constant and does not change as the speed of the body known as electrostatic unit of charge (e.s.u. of charge) or
changes. statcoulomb (stat C).

Flo
One e.s.u. of charge or one statcoulomb is that charge
1.13 COULOMB'S LAW OF ELECTRIC FORCE which repels an identical charge in vacuum at a distance of
one centimetre from it with a force ofl dyne.

ee
21. State Coulomb's law in electrostatics. Express the
same in SI units. Name and define the units of electric 1 coulomb = 3 X 10^ statcoulomb

Fr
charge. = 3 X 10^ e.s.u. of charge
Coulomb's law. In 1785, the French physicist {Hi) In electromagnetic cgs system, the unit of
Charles Augustin Coulomb (1736-1806) experimentally charge is abcoulomb or electromagnetic unit of
for
ur
measured the electric forces between small charged charge (e.m.u. of charge).
spheres by using a torsion balance. He formulated his
abcoulomb= ^ e.m.u. of charge
1
1 coulomb =
observations in the form of Coulomb's law which is 10
ks
electrical analogue of Newton's law of Universal
Yo

Gravitation in mechanics. For Your Knowledge


oo

Coulomb's laiv states that the force of attraction or > A torsion balance is a sensitive device to measure force.
eB

repulsion betiueen tivo stationary point charges is (i) directly ' - When the linear sizes of charged bodies are much smaller
proportional to the product of the magnitudes of the two than the distance between them, their azes may be ignored
charges and (ii) inversely proportional to the square of the and the charged bodies are called point charges.
r

distance between them. This force acts along the line joining
ou

5s Coulomb's law is valid only for point charges.


ad

the two charges.


‘h : In SI units, the exact value of the combination 4jr is
Y

10^
LU ,.^2x1-1 -2
4rt£
0 “ -^C N m
Fig. 1.7 Coulomb's law. c^
nd

where cis the speed of light in vacuum having the exact


Re

If two point charges q^ and are separated by value Z99792458 x 10* ms"’.
distance r, then the force F of attraction or repulsion
Fi

between them is such that > Electrostatic force constant,


1 k= 8.98755 xl 0^ Nm^"^ =9x10^ Nm^
F cc q^q^ and F qc
> Permittivity offree space,
=8.8551485 xl0"’2
F oc or
F = k‘>4^ r
= 9x!0"’^C^N“’
> SI unit of permittivity
where kisa constant of proportionality, called electro
_ coulomb X coulomb = c2N'’m"^
static force constant. The value of k depends on the
newton x metre^
nature of the medium between the two charges and the
system of units chosen to measure F, qy qj and r. The unit is usually expressed as farad per
metre (Fm"’).
For the two charges located in free space and in SI
units, we have > More strictly, the SI unit of charge 1 coulomb is equal to
1 1 ampere-second, where 1 ampere is defined in terms of
k = = 9x10^ Nm^C"^
4jt e the magnetic force between two current carrying wires.
0
LIO PHYSICS-XM

1.14 COULOMB'S LAW IN VECTOR FORM Range of coulombian forces. Coulombian forces act
over an enormous range of separations (r), from
22. Write Coulomb's law in vector form. What is the nuclear dimensions (r = 10"^^ m) to macroscopic dis
importance of expressing it in vector form ? tances as large as 10^® m. Inverse square is valid over
Coulomb's law in vector form. As shown in Fig. 1.8, this range of separation to a high degree of accuracy.
consider two positive point charges and q2 placed in Limitations of Coulomb's law. Coulomb's law is
vacuum at distance rfrom each other. They repel each not applicable in all situations. It is valid only under
other.
the following conditions :
h
1. The electric charges must be at rest.
Fi2 F21 2. The electric charges must be point charges i.e.,
+ ‘7i + <?2
the extension of charges must be much smaller
'●12 ^21
than the separation between the charges.
Fig. 1.8 Repulsive coulombian forces for 3. The separation between the charges must be
In vector form. Coulomb's law may be expressed as greater than the nuclear size (10"^®m), because
for distances < 10"^^m, the strong nuclear force
= Force on charge 1^2 due to dominates over the electrostatic force.
1
12
115 DIELECTRIC CONSTANT :
4;r £q RELATIVE PERMITTIVITY
^ r

where r^2 = ~ / is a unit vector in the direction from 24. What do you mean by permittivity of a medium ?
Define dielectric constant in terms offorces between two
to ^2-
charges.
Similarly, Tj2 = Force on charge due to ^2 Permittivity : An introduction. When two charges
1 are placed in any medium other than air, the force
^1^2 p21 between them is greatly affected. Permittivity is a
47iSn0
property of the medium which determines the electric force
A r between two charges situated in that medium. For example,
where , is a unit vector in the direction from ^2 the force between two charges located some distance
to q^. apart in water is about l/80th of the force between
them when they are separated by same distance in air.
The coulombian forces between unlike charges This is because the absolute permittivity of water is
{q.^q2 <0) are attractive, as shown in Fig. 1.9. about 80 times greater than the absolute permittivity of
air or free space.
Dielectric constant or relative permittivity. Accor
-‘?2
Fi2 f.21 ding to Coulomb's law, the force between two point
'●12 '■21 charges and ^^2/ placed in vacuum at distance rfrom
each other, is given by
Fig. 1.9 Attractive coulombian forces for q^q^<0. 1
Fvac ...(1)
Importance of vector form. The vector form of cou 4k Eq
lomb's law gives the following additional information :

When the same two charges are placed same
1. As r2j = - r^2' therefore F21 = - Fj2. distance apart in any medium other than vacuum, the
force between them becomes
This means that the two charges exert equal and 1 ^1^2
opposite forces on each other. So Coulombian
F
med ...(2)
47t£
forces obey Newton's third law of motion.
The quantity e is called absolute permittivity or just
2. As the Coulombian forces act along F^2 permittivity of the intervening medium. Dividing
i.e., along the line joining the centres of two equation (1) by equation (2), we get
charges, so they are central forces.
Fvac 4tc E„0 8
23. What is the range over which Coulombian forces _

F
can act ? State the limitations of Coulomb's law in med ^0
electrostatics. 47tE
ELECTRIC CHARGES AND FIELD

The ratio{e/ EQ)oftheperjniHivih/{E)oflhe medium to the 3.7x10"’^ = 9 X 10^ X qx q


permittivity (e^) of free space is called relative permittivity (5x 10“^*’)“
(Ej. ) or dielectric constant (k) of the given medium. Thus 3.7 X 10'^ X 25x10”^° -38
F or = 10.28x10
e or K = —
E
vac 9x 10^
^0 ^med
or 7 = 3.2 X 10"^^ C
So one can define dielectric constant in terms of
Number of electrons missing from each ion is
forces between charges as follows : -19
q 3.2 X 10
The dielectric constant or relative permittivity of a n = - =
-19
= 2.
c 1.6x10
medium may be defined as the ratio of the force between tzuo
charges placed some distance apart in free space to the force Example 6. A free pith-ball A of 8 g carries a positive
between the same two charges when they are placed the same charge of 5xlO"®C What must be the nature and
distance apart in the given medium.

w
magnitude of charge that shoidd be given to a second
Clearly, when a material medium of dielectric pith-ball B fixed 5 cm below the former ball so that the upper
constant k is placed between the charges, the force ball is stationary ? [Haryana 01]
between them becomes 1 / k times the original force in

Flo
Solution. The pith-ball Bmust be of positive charge
vacuum. That is.
i.e., of same nature as that of A so that the upward
F
force of repulsion balances the weight of pith-ball A

ee
F vac
med
K
When the pith-ball A remains

Fr
Hence the Coulomb's law for any material medium stationary,
may be written as
Fmed
T F = m^g
for ^0 ‘h
ur
4;t CnK
0 or
^ ‘?1?2 =

47iEn0 5 cm
K (vacuum) = 1
ks
K (air) = 1.00054 But m
i=8g=8xl0-3 kg B
O <72
Yo

K (water) = 80. q^ =5x10“^ C


oo

r = 5 cm =0.05 m
Examples based on Fig. 1.10
eB

CouiDfnb's Law 9x10^x5x10


= 8 X 10'^ X 9.8
Formulae Used (0.05)^
r

1
M2 8 X 9.8 X (0.05)^ X 10"
ou

1. Fvac
ad

4n E„0 r^ or
^2 ~
9x5
Y

1 ?i ^2
2. Fmed
4rt £q K = 4.36 X 10“^C (positive).
Example 7. A particle of mass m and carrying charge - q^
nd
Re

Units Used
is moving around a charge + 72 along a circular path of
7j, 72 are in coulomb, F in newton and r in metre. radius r. Prove that the period of revolution of the charge - 7^
Fi

Constant Used about + 72 is given by


k = — = 9x10^ 16ti^ e„0 mr^
4n ,E 0 T =
*?1^2

Example 5. The electrostatic force of repulsion between two Solution. Suppose charge - 7, moves around the
positively charged ions carrying equal charges is3.7 x 10“^ N,
when they are separated by a distance of 5 A How many charge + q^ with speed v along the circular path of
radius r. Then
electrons are missing from each ion ?
Solution. Here F =3.7 x 10"^ N, Force of attraction between the two charges
= Centripetal force
r = 5 A = 5 X 10 m, q^=q^=q (say)
1 ^1^2 _ 1 qi?2
As Ml or or V =

47t£n0
47ie„0 r
■y 4jrEQ mr
L12 PHYSICS-XII

The period of revolution of charge - cj.^ around + c\2 {b) (0 When charge on each sphere is doubled, and
will be the distance between them is halved, the force of
2nr Zr, mr 167i^e«0 mr^ repulsion becomes
_ ^ 2q^.2q2
0
1 = = 2jir or T =
F'air = 16A:.
'{r/2f
V

Example 8. Tzvo identical charged particles each having a = 16xl.5xl0"^=0.24N.


mass 10 g and charge 2.0 x 10"^C are placed on a horizontal (n) The force between two charges placed in a
table with a separation L between them such that they stay medium of dielectric constant k is given by
in limited equilibrium. If the coefifcient of friction between F =
1
i ^
each particle and the table is 0.25, find the value of L.

47l€n0 K
[Useg = 10 ms [JEE Main June 22]
For water, k=80
Solution. For equilibrium of the charged particles. F. 1.5x10'^
air
Mutual electrostatic force = Force of friction Fwater
K 80
kq^ = 1.875xlO"‘^N-1.9xlO'^ N.
= pmg
1}
k
Example 10. Suppose the spheres A and B in Example 9 have
.-. L= xq identical sizes. A third sphere of the same size but uncharged
Ijpmg is brought in contact with the first, then brought in contact
with the second, and finally removed from both. What is the
= I 9x10^ x2 X 10 ^m new force of repulsion between A and B ? [NCERTI
“V 0.25x0.01x10 Solution. Charge on each of the spheres A and B is
= - X 10^ x 2 X 10"^ m = 12 X10"^ m =12 cm.
(j=6.5xlO“^C
5 When a similar but uncharged sphere C is placed in
contact with sphere A, each sphere shares a charge
Example 9. (fl) Two insulated charged copper spheres A
q/2, equally.
and B have their centres separated by a distance of 50 cm.
What is the mutual force of electrostatic repulsion if the *7 Charge = 0 V2 <1/2

charge on each is 6.5 x 10 ? The radii of A and B are © * ©


negligible compared to the distance of separation. Also *7/2 3*7/4 3<?/4
compare this force with their mutual gravitational attraction
if each weighs 0.5 kg. @ " © ®©
*7/2 3*7/4
(b) What is the force of repidsion if (/) each sphere is
charged double the above amount, and the distance between .0 ©
them is halved ; (ii) the two spheres are placed in water ?
Fig. 1.11
(Dielectric constant of water = 80). [NCERT]

Solution, (fl) Here = q2 =6.5 x 10”^ C, Now when the sphere C (with charge q/2) is placed
in contact with sphere B (with charge q), the charge is
r = 50 cm =0.50 m
redistributed equally, so that
*\
Using Coulomb's law. 1
Charge on sphere B or C =—2 . q + —
2.
=—
4

.0
F:
air
.'. New force of repulsion between A and B is
= 9x10^
9 6.5 X 10"^ X 6.5 X 10"^ N
3q q
(0.50)^ ^ _ 4 '2
= 1.5 X 10'^ N. 47teo‘
The mutual gravitational attraction. = - X 1.5 X 10"^ N = 0.5625 x 10”^ N
8
p _ r - 5.7 X 10"^ N.

-11 Example 11. Two similarly equally charged identical metal


6.67 X 10 X 0.5 X 0.5
spheres A and B repel each other with a force of 2.0 x 10"^ N.
-11
= 6.67x 10 N
(0.5)2 A third identical uncharged sphere C is touched to A, then
placed at the midpoint between A and B. Calculate the net
Clearly, fc « E,i, ● electrostatic force on C. (CBSE OD 03]
ELECTRIC CHARGES AND FIELD

Solution. Let the charge on each of the spheres A equilibrium ? In zvbich case the equilibrium loill be stable
and B be q. If the separation between A and B is r, then and in zuhich unstable ?
electrostatic force between spheres A and B will be Solution. Suppose the three charges are placed as
2
shown in Fig. 1.13. Let the charge q be positive.
F = cf.-^=2.0xl0"^N
+ 4c + e

When sphere C is touched to A, the spheres share


charge q/2 each, because both are identical.
U a-x
Force on C due to A

Fig. 1.13
along AC
(r/2f
Force on C due to B For the equilibrium of charge + q, we must have

w
Force of repulsion Fj between + 4c and + q
= alongSC
{r/2f f = Force of repulsion Fj between + e and + q
Since these forces act in opposite directions, 1 4ex q _ 1 q

Flo
or
therefore net force on C is 4rce
t) x'^ Ane^la-xf
= 2.0 X 10"® N, along BC. 4{a-xf = x^

ee
F' = k -k or

or 2ia~x)=±x

Fr
Example 12. Tzvo identical charges, Q each, are kept at a X-—
2a
or 2a
distance r from each other. A third charge q is placed on the 3
line joining the above tzvo charges such that all the three
for
As the charge q is placed between + 4e and + e, so
ur
charges are in equilibriuzn. What is the magnitude, sign and
only x=2a/3 is possible. Hence for equilibrium, the
position of the charge q ? [CBSE OD, 98]
charge q must be placed at a distance 2a/3 from the
Solution. Suppose the three charges be placed in charge + 4c.
ks
the manner, as shown in Fig. 1.12.
Yo

We have considered the charge q to be positive. If


oo

-w
w—

w-
we displace it slightly towards charge c, from the
equilibrium position, then Fj will decrease and F2 will
eB

A c
Q q Q increase and a net force (F, - Fj) will act on q towards
left i.e., towards the equilibrium position. Hence the
Fig. 1.12 equilibrium of positive q is stable.
r
ou

The charge q will be in equilibrium if the forces Now if we take charge q to be negative, the forces Fj
ad

exerted on it by the charges at A and C are equal and and F2 will be attractive, as shown in Fig. 1.14.
Y

opposite.
+ 4e -q + e
or x^ ={r-xf <■
nd

x^ {r-xf
Re

F, ^2
r a-x
Fi

or x~r-x or a: = -
2
Fig. 1.14
Since the charge at A is repelled by the similar
charge at C, so it will be in equilibrium if it is attracted ,, .
by the charge q at B, i.e., the sign of charge q should be The charge - q w,ll st.ll be m equilibrium at
opposite to thit of charge Q. ^towards
=2“/3- right, then F^ will decrease and F2 will
.-. Force of repulsion between charges at A and C
increase. A net force (F2 - Fj) will act on -q towards
= Force of attraction between charges right i.e., away from the equilibrium position. So the
at A and B
equilibrium of the negative q ivill be unstable.
Q-Q Q
= k Example 14 Tzvo 'free' point charges + 4e and +e are
''=T-
or or

{rl2f placed a distance 'a' apart. Where should a third point charge
Example 13. Tzvo pomt charges + 4e and + e are ‘fixed’ a q be placed betiveen them such that the entire system may be
distance 'a' apart. Where should a third point charge q be in equilibrium ? What should be the magnitude and sign ofq ?
placed on the Ime joining the tzvo charges so that it may be in What type of an equilibrium zvill it be ?
1.14 PHYSICS-XII

Solution. Suppose the charges are placed as shown Example ^6. A charge Q is to be divided on two objects.
in Fig. 1.15. What should be the values of the charges on the two objects
+ 4e + e
so that the force between the objects can be maximum ?
●4 ■4
F F' F2 Solution. Let q and Q- qhe the charges on the two
X a-x objects. Then force between the two objects is
a
F =
1 <?(Q~?)

Fig. 1.15
47teo ■
where r is the distance between the two objects.
As the charge + e exerts repulsion F on charge +4e,
so for the equilibrium of charge +4e, the charge -q For F to be maximum.
dF
must exert attraction f' on +4e. This requires the — =0
charge q to be negative. dq
For equilibrium of charge + 4e, or 1 ^ (ciQ-q^)-o
F = F' 4:ieQ r^ dq
1

4 OT 0
4ex e

a
2
4to 0
1 4ex q
X
2
or
^(qQ-q^)
dq =0
ex
2 or
Q-2q=0
or
^ = T2
a or

For equilibrium of charge - q,


i.e., the charge should be divided equally on the two
Attraction between +4eand -q objects.
= Attraction F2 between + e and - q Example 17. Two identical spheres, having charges of
1 4ex q _ 1 ex. q opposite sign attract each other with a force of 0.108 N when
4toq x~ 4K£Q{a-xf separated by 0.5 m. The spheres are connected by a conduc
or x^=4(a-xf ting wire, which then removed, and thereafter they repel each
x=2al3 other with a force of 0.036 N. What were the initial charges
on the spheres ?
Hence ex^ 4^ _ 4e
a
Solution. Let + and - qj be the initial charges on
the two spheres,
The equilibrium of the negative charge q will be
unstable. (fl) When the two spheres attract each other,
F = k
'?i‘?2 0.108=9xlO^-^
Example 15. Two point charges of charge values Q and q t.e.,

are placed at distances x and x/2 respectively from a third (0.5)2


charge of charge value 4q, all charges being in the same 0.108 X (0.5)2 = 3x 10
-12

straight line. Calculate the magnitude and nature of charge ^1^2 “


9x10^
Q, such that the net force experienced by the charge q is zero.
(CBSE D 98] (F) When the two spheres are connected by the
Solution. Suppose the three charges are placed as wire, they share the charges equally.
shown in Fig. 1.16. ?i ~?2
.●. Charge on each sphere =
iq <7 Q 2 2
4
A Fb C Fa B Force of repulsion between them is

Fig. 1.16 2 2
F =
For the equilibrium of charge q, the charge Q must
have the same sign as that of qor 4q, so that the forces
9x 10^
and Fg are equal and opposite. 0.036 =
(0.5)2 i 2
I.e.,
As ^a=Pb
1 4qxq ^ 1 qx Q 0.036 x (0.5)2 ^ 4 -12
(^1 ^2'^ ~ 9x10^
= 4x 10
4TOq {x/2f 4toq (x/2)2
-6
or
Q = 4^. or
q^-q2~2x\0 ,..(I)
Now (t/i + ~('7] Hi)
ELECTRIC CHARGES AND FIELD

^H\Hi
= {2x 10“^)^+ 4x3x10
= 16x 10
-12
-12
roblems For Practice

1. Obtain the dimensional formula of Eq.


1.15

/
= 4 X 10”^ (Ans.
...{ii)
2. Calculate coulomb force between two a-particles
On solving equations (/) and (ii), we get -15
separated by a distance of 3.2 x 10 m in air.

= 3 X 10"® C and O2 = 10'^ C [CBSE OD 92]

which are the initial charges on the two spheres. (Ans. 90 N)


Example 18. Tivo small spheres each having mass m kg 3. Calculate the distance between two protons such
and charge q coulomb are suspended from a point bp that the electrical repulsive force between them is
insulating threads each I metre long but of negligible mass. If equal to the weight of either. [CBSE D 94]

0 is the angle, each thread makes with the vertical when (Ans. 11.8 cm)

w
equilibrium has been attained, show that 4. How far apart should the two electrons be, if the
q^ = (4 mg'f sin^ 0 tan 0) 4ti e 0 [Punjab 95] force each exerts on the other is equal to the weight
-19
of the electron ? Given that e = 1.6x10 C and
Solution. The given situation is shown in Fig. 1.17.

Flo
Each of the spheres A and B is acted upon by the m^ = 9.1 X10"^’ kg. [Haryana 02]

following forces : (Ans. 5.08 m)

ee
(i) its weight mg, (ii) tension T in the string 5. A pith-ball A of mass 9x10“^ kg carries a charge of
5jiC. What must be the magnitude and sign of the

Fr
(Hi) the force of repulsion F given by
charge on a pith-ball B held 2 cm directly above the
1
F = ...(0 pith-ball A, such that the pith-ball A remains
471 e„0 stationary ?
for
ur
(Ans. 7.84 pC, sign opposite to that of A)
6. Two identical metal spheres having equal and
ks
similar charges repel each other with a force of
Yo

103 N when they are placed 10 cm apart in a medium


oo

1/
of dielectric constant 5. Determine the charge on
each sphere. (Ans. 23.9 x 10"® C)
eB

Ty
4 7. The distance between the electron and proton in
hydrogen atom is 5.3 x 10"” m. Determine the magni
AJ tude of the ratio of electrostatic and gravitational
r

force between them.


ou
ad

Given = 9.1 x 10“^^ kg, = 1.67x10“^^ kg,


Y

mg e=1.6xl0“^^Cand G = 6.67x10"” Nm^ kg"^.


Fig. 1.17 (Ans. FJ Fg = 2-27 x 10^^)
nd
Re

As the forces are in equilibrium, the three forces on 8. Two identical metallic spheres, having unequal,
opposite charges are placed at a distance 0.90 m
sphere A can be represented by the three sides of
Fi

A AOC taken in the same order. Hence apart in air. After bringing them in contact with
F T
each other, they are again placed at the same
mg
distance apart. Now the force of repulsion between
AC OC AO
them is 0.025 N. Calculate the final charge on each
AC of them. [CBSE D 02C1
or
F = mg X ...{ii)
OC
(Ans. 1.5xl0"®Q
From (f) and {ii), we have 9. A small brass sphere having a positive charge of
1 AC 1.7 X 10"® C is made to touch another sphere of the
= mgx
4jiEq AB^ OC same radius having a negative charge of 3.0 x 10"^ C.
Find the force between them when they are
But AC = / sin 0, OC = I cos 0, AB = 2 AC = 21 sin 0
separated by a distance of 20 cm. What will be the
1 I sin 0
force between them when they are immersed in an
= mgx
0
4/^sin^0 I cos 0 oil of dielectric constant 3 ?

(Ans. 1.1 X10"^ N ; 0.367 x IQ~^ N)


or
q^ - (4 jng "f sin^ 0 tan 0) 4tc£q.
1.16 PHYSICS-XII

10. The sum of two point charges is 7 \iC. They repel threads 0.5 m long. On charging the balls equally,
each other with a force of 1 N when kept 30 cm they are found to repel each other to a distance of
apart in free space. Calculate the value of each 0.2 m. Calculate the charge on each ball.
charge. ICBSE F 091 [Haryana 2002]
(Ans. 5)iC, 2pC) (Ans. 2.357 x 10" ^ C)
11. Two point charges = 5xlO"^C and £^2 = 3 X 10"^C HINTS
are located at positions (1 m, 3 m, 2 m) and (3 m.
->
1, F = ^ HlHi or
_ I2
^0 = 4nFr^
5 m, 1 m) respectively. Find the forces 0

using vector form of Coulomb's law. AT. AT


M = MLT"^L^' = IM"^L"^T^ A^l.
[Ans. ^^=-5xl0-^{2i+2}-k )N, -19 -15
2. Here = £/2 = 2e = 3.2 x 10 C r = 3.2 X 10 m
= 5xl0“^(2i' + 2j~k)N]
12. Tlnree equally charged small objects are placed as 4nsQ
shown in Fig. 1.18. The object A exerts an electric
force on object B equal to 3.0 x 10"^N. 9 X 10^ X 3.2 X 10'^^ X 3.2 x 10"’’
= 90N.
(3.2x10'^^)^
A B C

3. For a proton, m = 1.67 x 10“^^ kg,


-19
2 cm 1 cm >( C.
M- ♦«
q = + e = 1.6x10
Weight of proton = Electrical repulsive force
Fig. 1.18
or
(i) What electric force does C exert on B ?
(i7) What is the net electric force on B ? 2 _ kq^ _ 9x10^ x(1.6xl0~^^)‘
[.A.ns. (0 12.0 X 10“* N, along BA ^ 1.67x10"^^ X 9.8
{ii) 9.0 X 10"^ N, along BA] 23.04
X 10"^ = 0.014
13. Two identical metallic spheres A and B, each carry 16.36
ing a charge q, repel each other with a force f. A or r= 0.118 m =11.8 cm.
third metallic sphere C of the same size, but un e xe

charged, is successively made to touch the spheres 4. g = k.


A and B, and then removed away. What is the force
of repulsion between A and B ? (Ans. 3f / 8) or r^ =
ke^ 9x10^ x(1.6xl0“^^)^ = 25.84
-31
9.1x10 x9.8
14. Two point charges + 9e and + e are kept at a distance
a from each-other. Where should we place a third r= 5.08 m.
charge q on the line joining the two charges so that 5, The pith-ball B must have charge opposite to that of
it may be in equilibrium ? A so that the upward force of attraction balances
3£7
Ans. — from + 9e charge the weight of pith-ball A.
When the pith-ball A remains B
tO
15. Two point electric charges of values q and 2q are stationary.
kept at a distance d apart from each other in air. A 2 cm p
F = HIjg
third charge Q is to be kept along the same line in
1 'll‘72
such a way that the net force acting on q and 2q is or
4n .80
zero. Calculate the position of charge Qin terms of q ‘h
and d. ICBSE D 98] But Mjj = 9 X 10"^ kg, »'iS
(Ans. At a distance of (V2 - 1) d from charge q) £fj = 5pC = 5xlO"*C, Fig. 1.19
16. A charge q is placed at the centre of the line joining r = 2 cm = 0.02 m
two equal charges Q. Show that the system of three
charges will be in equilibrium ii q = -Q/ A 9x10^x5x10"^
^ = 9xl0"^x9.8
ICBSE OD 05] (0.02)2
17. Two pith-balls each weighing 10"^kg are or
£?2 = 7.84 X 10
-12
C = 7.84 pC
suspended from the same point by means of silk
ELECTRIC CHARGES AND FIELD

6. F= 1 'll <72 103 =


9 X 10^ X £j2 p -
12
4p Sq K 5x(0.10)^
or
17 = 23.9x10"^ C 9x10^ x5xl0~^ x3xl0~^ {2i + 2]-k)
7. Proceed as in illustrative problem on page 1.18. 3^ 3

8. The two spheres will share the final charge equally. = 5xlO“^(2f+2;-^)N
Let q be the charge on eacli sphere.
F =
1
.Mi = 0.025 N
Also, 4 = ^4 =-5xl0"^(2i' +2;) N.
4ti £n0 ' 12. Here AB = 2 cm = 0.02 m, BC = 1 cm = 0.01 m

or
9 X 10^ X17 X g = 0.025 4
<7

(0.90)^ A e
f BA
c

w
2 cm 1 cm
0.025 x(0.90)^
1^ =
-14
or = 225 X10
9x10^ Fig. 1.20
or
17 = 1.5x10"^ C Let q be the charge on each object.

Flo
9. Charge shared by each sphere 1

AtuEq (AB)^

ee
(17-3) X 10"^ = 7 X10'^ C
2
9 X 10% 17^

Fr
-6
or 3.0x10
9xl0^x(7xl0"^)^ (0.02)^
F:_ = = 1.1x10"%
(0.20)^
air

or -xl0"^%.
for
ur
3
9xl0^x(7xl0"^)^ = 0.367 X10"^ N. -19
^Oil = 1 qxq
= 9 X10^ x
4x10
3 X (0.20)^ (0 ,FBC
47ic„'(BCf 3 X (0.01)2
ks
10. Here F = 1N, r = 30 m
= 12.0x10"® N, along BA.
Yo

p-lcHlUl
oo

As
(n) Net force on charge at B,
-6
eB

9 X 10^ X £7j^2 ^^=^bc-^b.4=(12-0-3.0)x10


1 =
(0.30)2 = 9.0x10"® N, along BA.
-11 13. Proceed as in Example 10 on page 1.12.
r

or
q^qi = 10
ou

14. Force between + 9e and t7 = Force between+ eand q


ad

But g, + (72 = 7pC = 7xlO“®C ...(z)


e X <7
Y

Now (q^ - <72)2 = (<7, + <72)2 - 4<7j<72 x2 (rt-AT)2


= 49x10"^2_4^^q -11
3 I
nd

or X = 3fl / 4
Re

or
-12 X a - X
= 9x10
Fi

or
'll - ^2 = 3pC ...(zz) 15. For equilibrium of charges q and 2q, the charge Q
must have sign opposite to that of q or 2q. Suppose
On solving (i) and (zT), we get it is placed at distance x from charge q.
z7j = 5 qC and q2=2 pC 4
●-
Q 2q

11. Here ^ =(/+ 3; +2ft )m, ^=(3z+5; + cf)m X d-x

hi ~ = (3i +5; + ft )-(r + 3/ + 2ft ) Fig. 1.21


For equilibrium of charge <7,
= (2z + 2/ - ft ) m
.(0
,t2 <f2
1^2! =/2'2 + 22 + (-1)2 = 3 m
For equilibrium of charge 2q,
;'12 _“ ?2 2'z+2/-ft ...(zz)
3
1^2 I rf2 id~xf
PHYSICS-XII

From (i) and {ii), we get, 1.16 COMPARING ELECTROSTATIC AND


,.qQ f- GRAVITATIONAL FORCES
{d-xf
25. Give a comparison of the electrostatic and gravi
or 2x^=(d-xf tational forces.
or y[2x = d-x Electrostatic force vs gravitational force. Electro
or
1
X = -7= = (V2 - 1) d static force is the force of attraction or repulsion behoeen two
V2+ 1 charges at rest while the gravitational force is the force of
attraction behoeen tioo bodies by virtue of their masses.
i.e., the charge Q must be placed at a distance of
(V2 - 1) d from the charge q. Similarities :

16. Suppose the three charges are placed as shown in 1. Both forces obey inverse square law i.e.,
Fig. 1.22.
F x4r.
Q Q
C B 2. Both forces are proportional to product of
■H
M-
masses or charges.
Fig. 1.22 3. Both are central forces i.e., they act along the line
joining the centres of the two bodies.
Clearly, tlie net force on charge q is zero. So it is in 4. Both are conservative forces i.e., the work done
equilibrium, the net force on other two charges against these forces does not depend upon the
should also be zero.
path followed.
Total force on charge Q at point B is 5. Both forces can operate in vacuum.
1 QQ , ^ <lQ = 0
Dissimilarities :
4ne„'(2xf 4n ,c 0 x2
1. Gravitational force is attractive while electro
1 qQ^ 1 QQ
or static force may be attractive or repulsive.
47t£Q x^ 4n£fl {Ixf' 2. Gravitational force does not depend on the nature
or i? = -Q/4. of the medium while electrostatic force depends on
17. In A OCA of forces, we have the nature of the medium between the two charges.
F _ mg ^ r 3. Electrostatic forces are much stronger than
AC OC OA gravitational forces.
Illustrative Problem. Coulomb's law for electrical force
between hoo charges and Newton's laxu for gravitational force
behoeen hoo masses, both have inverse-square dependence on
the distance between charges/masses,
(a) Compare the strength of these forces by determining
the ratio of their inagnitude {i)for an electron and a
proton and {ii)for two protons,
(b) Estimate the accelerations for electron and proton
F- B due to the electrical force of their mutual attraction
when they are 1 A (’=10"^'^ m) apart.
How much is the electrostatic force stronger than
the gravitational force ?
(a) (/) From Coulomb's law, the electrostatic force
F = mg between an electron and a proton separated by
distance r is
1
= X
r' ,.^N2 _kj-e){e)_ ke'^
4ti Cf) AB^
9 X 10^ X 10"^ X 9.8 X 0.1
Negative sign indicates that the force is attractive.
(0.2)' ^(0.5)^ -(0.1)" From Newton's law of gravitation, the corresponding
q = 2.357x10"* C. gravitational attraction is
ELECTRIC CHARGES AND FIELD 1.19
m.. m
26. Give two examples zvhich illustrate that the
^G =
electrical forces are enormously stronger than the gravi
tational forces.
where and m^ are the masses of the proton and
electron. Examples : (/) A plastic comb passed through hair
Hence can easily lift a piece of paper upwards. The electro
F ke^ static attraction between the comb and the piece of
paper overcomes the force of gravity exerted by the
entire earth on the paper.
Butit=9xlO^Nm^C"^ e = 1.6x 10"’^ C, {ii) When we hold a book in our hand, the electric
(frictional) forces between the palm of our hand and
=1.67x10"^^ kg, =9.1x10"^^ kg, the book easily overcome the gravitational force on the
-11
G = 6.67 X 10 Nm^ kg ^ book due to the entire earth.

w
In the words of Feynman, if you stand at arm's
9 X 10^ X (1.6x10"^^)'
-11 -27 -31 length from your friend and instead of being electri
Pc 6.67x10 X 1.67x10 X 9.1x10
cally neutral each of you had an excess of electrons

Flo
39
= 2.27 X 10 over protons by just one per cent, then tlie force of
(a) (ii) Similar to that in part (/), the ratio of the repulsion between you would be enough to lift the

ee
entire earth.
magnitudes of electric force to the gravitational force

Fr
between two protons at a distance r is given by
1.17 FORCES BETWEEN MULTIPLE CHARGES :
F, ke^ 9x10% (1.6x10“^^)^ THE SUPERPOSITION PRINCIPLE
-11
Pc 6.67 X 10 X (1.67 X 10“^^)^ for
ur
27. State the principle of superposition of electrostatic
36
= 1.24 X 10 forces. Hence write an expression for the force on a point
Thus the large value of the (dimensionless) ratio of charge due to a distribution of N-1 point charges in
ks
the two forces indicates that the electrostatic forces are terms of their position vectors.
Yo
oo

enormously stronger than the gravitational forces. Principle of superposition of electrostatic forces.
Coulomb's law gives force between two point charges.
(b) The magnitude of the electric force exerted by a
eB

The principle of superposition enables us to find the


proton on an electron is equal to the magnitude of the
force on a point charge due to a group of point charges.
force exerted by an electron on a proton. The magni
tude of this force is This principle is based on the property that the forces
r

with which two charges attract or repel each other are


ou

ke^ 9 xiO^x (1.6x10“^^)'


ad

F = not affected by the presence of other charges.


(10-10)2
Y

-10
The principle of superposition states that when a
[v r = lA=10 m] number of charges are interacting, the total force on a given
nd

= 2.3xlO"“N charge is the vector sum of the forces exerted on it due to all
Re

Acceleration of the electron due to the mutual other charges. The force betiveen tzoo charges is not affected
Fi

attraction with the proton, by the presence of other charges.


f 2.3 X 10^^ N 22 -2
As shown in Fig. 1.24, consider N point charges
= 2.5 X 10 ms
‘Iz’^2' hr[ vacuum at points whose
m^ ~ 9.1 X 10“^^ kg . . . . — —> —» —» —>
position vectors w.r.t. origin O are r^, ry — /
Acceleration of the proton due to the mutual
attraction with the electron. respectively.
F 2.3 X 10“® N According to the principle of superposition, the
a = 1.3x 10^^ ms"^ total force on charge i/j is given by
^ 1.67x10"^^ kg
Clearly, the acceleration of an electron or a proton F, PlS'^ + F
IN

due to the electric force is much larger than the accele


ration due to gravity. So, we can neglect the effect of where Fp, ^3,...., 7^ IN
are the forces exerted on

gravitational field on the motion of the electron or the charge q.^ by the individual charges qy ' ^N
proton. respectively.
PHYSICS-XIl

z= Total force on rttli charge

f = % y - ZA
-♦ i3
0 i? = 1
b^a
'ab
^1:1
where a =1,2,3,..., N.
It may be noticed that for each choice of a, the
summation on b omits the value a. This is because
summation must be taken only over other charges. The
above expression can be written in a simpler way as
follows :

O F = Total force on charge q due to many point


Fig. 1.24 Superposition principle : Force on charges cf
—>

charge exerted by and q^. -r'


f=^
4tcs
According to Coulomb's law, the force exerted on 0 all point r
charges
charge due to ^2 is
F,12 L_^ P

4 ire
0 '12
12
Eacampte baSBCl on
-» rrtndple of SupHposition of
1 ^1^2 -^2 Electric Forces
4n:e 0 ,2
rj -r2 '1-^2 Formulae Used
-» ->
1 'i-^2
●'7i^2 “ -^12 -^13 -^14 ●●● IN
4 Tie 0
1 - ^2 I
P^^jFl + F^+lF^F^cosQ
r “ r
where n,» = —1 = a unit vector pointing from q^ to Units Used
i“* "*i
I >i - rj I Forces are in newton, charges in coulomb and
distances in metre.
and ^12 = I ? “ ^^ distance of ^2 from q^.
Hence the total force on charge q■^ is Example 19. An iufinite number of charges each equal to
1
4 pC are placed along x-axis at x=lm, x -2 m, x = 4 m,
F = p12 +, ^1% C13 I + +
IN x=8m and so on. Find the total force on a charge of 1 C
4;reo ^ 13 IN
placed at the origin. [HT 95]

y
Solution. Here g = 4 pC = 4xlO^C, gQ=lC
or
fl = Z- 2 'u
4t18
0 1 = 2 ii By the principle of superposition, the total force
In terms of position vectors, acting on a charge of 1 C placed at the origin is
-» 1 1
"l ' 'i -"3
4jte 0
'?1^2 IT + "71^3
+
4Tt8o [rf rf
lii-IP 'i -IP T 1 1
-^N = 9xl0^x 4xl0"^xl
+ ^l^N ~ .1^ 2^ 4^
"i -7w 1^I
-> Sum of the infinite geometric progression
N 7 - r
or
_ ‘^i I
a 1 4
4rte 0 1= 2
^1
1-r
1-^ 3
4

In general, force F^ on flth charge q^ located at due F=9xl0^x4xl0"^xli = 4.8x 10“ N.


to all other (N -1) charges may be written as 3
ELECTRIC CHARGES AND FIELD

Example 20. Consider three charges q^ % each equal to


qat the vertices of an equilateral triangle of side 1. What is the
force on a charge Q (with the same sign as q) placed at the
centroid of the triangle ? [NCERTJ

Solution. Suppose the given charges are placed as


shown in Fig. 1.25(fl).

AO

w
Let Q be the charge required to be kept at the
centroid G. Then,

= Force at A due to the charge at B

Flo
= J-l. along BA
4jte«0 /"'

ee
Let AO=BO^CO = r
1 (7^

Fr
Force on charge Q due to qy F2 = Force at A due to charge at C = 4jt8. ■ along CA
R =
1 Q^i AO 0

1
47iEq AO^ H + F, = 2H cos30°, alone GA =-73.— . %y,
I- 1
along GA
for
ur
^ 2 ^ ® 4;re„0
Force on charge Q due to ^2/
1 Qq Force at A due to charge at G
^2 = ^ BO
ks
47iSq BO^ _ 1 _ 1 Qq _ I ^Qq
Yo

Force on charge Q due to q^, 47ie(j AG^ 4718^ {liJsf 47U8p f


oo

^3 = ' %co
eB

This must be equal and opposite to (Fj+ f2)-


47t8o co^
By the principle of superposition, the total force on 3Qq=~JSq^ or Q 73*
charge Q is
r
ou

Example 22. Consider the charges q, q and -q placed at


ad

-*

F - Fi + F2 + the vertices of an equilateral triangle, as shown in Fig. 1.27.


Y

, Qq What is the force on each charge ? [NCERT]


[AO + BO + CO] [V = i?2 = ‘Is = q]
47r£n0 Solution. The forces of attraction or repulsion
nd
Re

between different pairs of charges are shown in


As shown in Fig. 1.25(b), the angle between each Fig. 1.27. Each such force has magnitude,
Fi

pair of tise unit vectors AO, BO and CO is 120°, so they


1 q"
form a triangle of cyclic vectors. Consequently, 4t:8„
ao + bo+co==o

Hence F =0i.e., the total force on charge Qis zero.


Example 21. Three point charges +q each are kept at the
vertices of an equilateral triangle of side 'I'. Determine the
jnagnitude and sign of the charge to be kept at its centroid so
that the charges at the vertices remain in equilibrium.
[CBSE F 2015]

Solution. At any vertex, the charge will be in


equilibrium if the net electric force due to the
remaining three charges is zero.
1.22 PHYSICS-XII

By the parallelogram law, the net force on charge = ^(180)^ + (180)^ + 2 X 180 X 180 xcos 120° N
IS
= 180^1+1+2 x(-1/2) N = 180 N
^ = ^F~+ F^+2Fx F cos 120° BC Let the resultant force F make an angle P with the
force F,. Then
= ^2F^ +2F^(-l/2) BC = F BC sin 120° 180 xsin 120°
tan p =
A

Fj + F2 cos 120° 180+ ISOcos 120°


where BC is a unit vector along BC.
180xV3/2
Similarly, total force on charge qj is = V3
180+ 180(- J)
F^ = FAC
p = 60°
where AC is a unit vector along AC.
i.e., the resultant force F is parallel to BC.
Total force on charge is
Example 24. Four equal point charges each 16 pC are
^ ^ 2 F X F cos 60° H =V3Fw placed on the four corners of a square of side 0.2 m. Calculate
the force on any one of the charges.
where n is a unit vector along the direction bisecting Solution. As shown in Fig. 1.29, suppose the four
ZACB.
charges are placed at the corners of the square ABCD.
Example 23. Charges of+ 5 pC, + 10 pC and -10 pC are Let us calculate the total force on q^.
placed in air at the corners A, B and C of an equilateral
triangle ABC, having each side equal to 5 cm. Determine the h
resultant force on the charge at A.
Solution. The charge at 6 repels the charge at A A
0.2 m ^4
'Z,
D
with a force.
^ , q^q2 9xl0^x(5xl0"^)x(10xl0'^) N
Fj - cf - e o
to

= 180 N, along BA

B
‘I2 0.2 tn %

Fig. 1.29

Here AB= BC = CD = AD =0.2 m

'll = “12 = ‘la = mC = 16 X10“^ C


Force exerted on q^ by q-^ is
9x10^ X 16x10"^ X 16x10"^
E1 =
(0.2)2
Fig. 1.28 = 57.6 N, along AD produced
Force exerted on q^ by q2 is
Tlie charge at C attracts the charge at A with a force 9x10^ X 16x10"^ X 16x10*^
_9xl0^x(5xl0-^)x(10xl0~^) ^2 =
N (0.2)^ + (0.2)2
^~ (0.05)2 = 28.8 N, along BD produced
= 180 N, along AC. Force exerted on q^ by q.^ is
By the parallelogram law of vector addition, the 9x10^ x 16x10"^ X 16x10"^
—►

magnitude of resultant force F on charge at A is ^3 =

F = ^F,2+ F/ + 2F1F2COS0 = 57.6 N, along CD produced


ELECTRIC CHARGES AND FIELD

As Fj and are perpendicular to each other, so Net force on charge at A will be zero if
their resultant force is
9 X 10^ X 2 X 10 -6
= 2.34
F' = + F^^ = ^57.6^ + 57.6^ (O.lx ^/3)^
= 57.6V2 = 81.5 N, in the direction of F^. 2.34 X 0.01 X 3
Hence total force on is or = 3.9xlO'^C = 3.9 pC
18x 10^
F = F2 + F' = 28.8 + 81.5
= 110.3 N, along BD produced. roblems for Practice
Example 25. Three point charges of + 2 pC -3 pC and
-3 pC are kept at the vertices A, B and C respectively of an 1. Ten positively charged particles are kept fixed on
equilateral triangle of side 20 cm as shown in Fig. 1.30. the :r-axis at points a: = 10 cm, 20 cm, 30 cm, ...,
What should be the sign and magnitude of the charge to be 100 cm. The first particle has a charge 1.0 x 10"® C,
the second 8 x 10“® C, third 27 x 10"® C, and so on.

w
placed at the midpoint (M) of side BC so that the charge at A
remains in equilibrium ? [CBSE D 05] The tenth particle has a charge 1000 x 10"® C. Find
the magnitude of the electric force acting on a 1 C
charge placed at the origin. (Ans. 4.95 X10®N)

Flo
2. Charges q^ = 1.5 mC, q2 = 0.2 mC and q^ =- 0.5 mC
are placed at the points A, Band Crespectively, as

ee
shown in Fig. 1.32. If rj = 12 m and r2 = 0.6 m,

Fr
calculate the magnitude of resultant force on q^.
(Ans. 3.125 x 10® N)

for
ur
Fig. 1.30
ks
Solution. As shown in Fig. 1.31, the force exerted on
Yo

charge + 2 pC by charge at B,
oo

1
F,1 =
eB

4 71 e 2
7
0

9 X 10% 2 X 10"^ X 3 X 10"®


r

(0.20)^
ou
ad

= 1.35 N, along AB
3. Two equal positive charges, each of 2pC interact
Y

Force exerted on charge + 2 pC by charge at C with a third positive charge of 3pC situated as
9xl0%2xl0"®x3xl0"® shown in Fig. 1.33. Find the magnitude and
nd

^^2 = direction of the force experienced by the charge of


Re

(0.20)^ 3pC. (Ans. 3.456 x 10 ® N, along OC produced)


Fi

= 1.35 N, along AC
Resultant force of and F2 i 2mC
F==^F^+ F} +2 F^Fj cos 60°
3 m
= -^1.35^ + 1.35^ + 2 X 1.35 x 1.35 x 0.5 ●'. 3 ^C
= 1.35 X V3 =2.34 N, along AM 0
4 m
-.*c

For the charge at A to be equilibrium, the charge q to


be placed at point M must be a positive charge so that it 3 m

exerts a force on +2pC charge along MA.


Now, AM = ^20^ - lOp- S 2nC
B
= ^/300 =10V3 cm

= 0.1x m Fig. 1.33


1.24 PHYSICS-XII

HINTS Similarly, force exerted by charge on


1. By the principle of superposition, the total force on = 2.16 X 10“^ N, along BC produced
the 1 C charge placed at the origin is Clearly, = Fg (in magnitude)
^0 = ^01 + ^02 + ^03 + -■ + 1^0 The components of F^ and Fg along Y-axis will
cancel out and get added along X-axis.
_ ‘Jo ^1 . 'll . % -I-.... +
'lio .'. Total force on 3 pC charge,
4n£o \r^ ^ h
F=2F^ cos 9=2x2.16x10“^ x|
b "10.
-8 -8
1.0x10 8x10
= 1X 9 X 10^ + = 3.456 xl0“^ N, along CX.
(0.10)^ (0.20)^
-8 -8 1.18 ELECTRIC FIELD
27x10 1000 X10
+ -I-... +
(0.30)^ (1.00)^ 28. Briefly develop the concept of electric field.
Concept of electric field. The electrostatic force acts
= 9x10^ xl0“^ [1+ 2+ 3 + ...-H 10] between two charged bodies even without any direct
= 9x55 xlO^ =4.95 X10® N. contact between them. The nature of this action-
af-distance force can be understood by introducing the
2. F,=
1 q^q^ _ 9xl0^x0.2xl0~®xl.5xl0~® concept of electric field.
47reo rf Source charge Test charge
= 1.875 X 10® N, along AB produced
^ <?2<?3 9 X10^ X 0.2 X10“^ X 0.5 X 10~® 0-0
%

47IE
rl (0.6f
J
0 P

= 2.5xlO®N, along BCJ.AB


As ^ 1 F2, so the resultant force on is Fig. 1.35 A charged body produces air electric field around it.

F = ^Ff + Ff = 3.125 X10® N. Consider a charged body carrying a positive charge q


placed at point O. It is assumed that the charge q
3. Here = lyg = 2pC = 2 x 10“ ^ C, produces an electrical environment in the surroun
f/(^ = 3pC = 3xlO“^C ding space, called electric field.
To test the existence of electric field at any point P,
AC= BC = ^j3^ + 4® we simply place a small positive charge t/g, called the
—>

test charge at the point P. If a force F is exerted on the


test charge, then we say that an electric field E exists at
the point P. The charge q is called the source charge as it
produces the field £.
29. Deifne electric ifeld at a point. Give its units and
dimensions.

Electric field. An electric field is said to exist at a point


if a force of electrical origin is exerted on a stationary charged
body placed at that point. Quantitatively, the electric field or
the electric intensity or the electric field strength E at a
point is defined as the force experienced by a unit positive
lest charge placed at that point, without disturbing the
position of source charge.
As shown in Fig. 1.35, suppose a test charge q^
->

4jiEo (AC)^ experiences a force F at the point P. Then the electric


9x10^ x2xl0"‘'’ x3xl0"^ field at that point will be
52
= 2.16 X10 ® N, along AC produced %
ELECTRIC CHARGES AND FIELD 1.25

There is a difficulty in defining the electric field by Thus an electric field plays an intermediary role in
the above equation. The test charge may disturb the the forces between two charges :
charge distribution of the source charge and hence Charge ^ Electric field ^ Charge.
change the electric field E which we want to measure. It is in this sense that the concept of electric field is
The test charge i/g must be small enough so that it does useful. Electric field is a characteristic of the system of
not change the value of £. It is better to define electric charges and is independent of the test charge that we
field as follows : place at a point to determine the field.
The electric field at a point is deifned as the electrostatic hTqwri on .
force per unit test charge acting on a vanishingli/ small Reiation bcpwaan Electric Field
positive test charge placed at that pomt. Hence Strength and Force
F Formulae Used

w
£ = lim
->
0
^0 %
e =L or
F - q^ E
%
The electric field E is a vector quantity whose

Flo
direction is same as that of the force f exerted on a
Units Used

positive test charge. When force is in newton, charge in coulomb and

ee
Units and dimensions of electric field. As the distance in metre, electric field strength is in
newton per coulomb (NC“*) or equivalently in

Fr
electric field is force per unit charge, so its SI unit is
volt per metre (Vm~^).
newton per coulomb (NCT^). It is equivalent to volt per
metre (Vm“^).
Example 26. Calculate the voltage needed to balance an oil
for
ur
The dimensions for £ can be determined as drop carrying 10 electrons when located betioeen the plates
follows : of a capacitor which are 5 mm apart. The mass of oil drop is
3 X 10"^^ kg. Take ^ = 10 ms~^. (CBSE OD 95C]
ks
Force MLT"^
[E] =
Solution. Here £j = 10e = 10xl.6x 10“^^ C
Yo

Charge C
oo

MLT“^ 1C1 d=5 mm = 5 x 10“^ m, w=3 x 10"’^ kg, ^ =10 ms“^


= 1MLT"^A"M
eB

●.* 1 A =
A.T 1 s I
qE
£
30. Give the physical significance of electric field.
r

Physical significance of electric field. The force


ou
ad

experienced by the test charge qQ is different at


Y

—►

different points. So E also varies from point to point. Fig. 1.36

In general, E is not a single vector but a set of infinite When the drop is held stationary.
nd
Re

Upward force on oil drop due to electric field


vectors. Each point r is associated with a unique
= Weight of oil drop
Fi

—>

vector E (r). So electric field is an example of vector field.


qE = mg
By knowing electric field at any point, we can
determine the force on a charge placed at that point.
V
£ =^
or q.-j^mg d
The Coulomb force on a charge q^ due to a source
charge q may be treated as two stage process : .-. V =
mgd _3xl0~^^x 10x5x10"^ = 9.375 V.
-19
(/) The source charge q produces a definite field ‘I 10x1.6x10

£(r) at every point r . Example 27. A positively charged oil drop is prevented
from falling under gravity by applying a vertical electric
(iV) The value of E(r) at any point r determines the field oflOOVnf^. If the mass of the drop is 1.6x.l0~^g,fijid
force on charge c/q at that point. This force is the number of electrons carried by the drop.
Solution. Here m = 1.6 x 10"^g = 1.6 x 10“^kg,
£ = 100Vm'^
Electrostatic force = Charge x Electric field.
1.26 PHYSICS-Xll

As the oil drop is positively qE Let T be the tension

charged, the field £ must act in the thread and 0 be


vertically upwards. When the drop 0 ^ the angle it makes with
vertical, as shown in
remains stationary.
Upward force of electric field mg Fig. 1.39. When the bob
is in equilibrium,
= Weight of oil drop Fig. 1.37
qE or iieE = mg T sin 0 = t/H;
Number of electrons carried by the oil drop, T cos 0 = mg
n - mg ^ 1.6xl0~^xl0 = 10^1 tan 0 = T sin Q ^ qE
eE ”l.6xl0“^^xl00 T cos 0 mg
Example 28. A simple pendulum consists of a small sphere 2 X 10"® X 2 X 10“^
of mass m suspended by a thread of length 1. The sphere 80 X 10"^ X 9.8
carries a positive charge q. The pendulum is placed in a 0 = 27°
or
uniform electric field of strength E directed vertically
downwards. Find the period of oscillation of the pendulum qE 2xl0"®x2xl0‘^
Also,T=- = 8.81 X 10"^ N.
due to the electrostatic force acting on the sphere, neglecting sin 0 sin27°

the effect of the gravitational force. [CBSE OD 19]


Example 30. An electron moves a distance of 6 cm when
Solution. Restoring force, F =-qEsinQ accelerated from rest by an electric field of strength
For small 0, 2 X 10^ NC~^. Calculate the time of travel. The mass-19and
charge of electron are 9xl0"^^fcg and 1.6x10 c

respectively. [CBSE D 91]

Solution. Force exerted on the electron by the


electric field.
F = eE

.'. Acceleration,

F_c£ 1.6x10
-19
x2 X 10^ 16 -2
a = = 0.35 X 10 ms
-31
m m 9x10

Now u =0, s =6.0 cm =0.06 m, a =0.35 x lO^^ms”^


As S=Iif +

0.06=0+ ^x0.35x 10^^ xt^


0.06 X 2
ma or t = = 0.585 X 10"® s.
^0.35 X 10
16

dh
or Example 31. An electron falls through a distance of 1.5 cm
dt^ -1
in a uniform electric field of magnitude 2.0 x 10"^ NC
This equation represents SHM of angular frequency. [Fig. 1.40(a)]. The direction of the field is reversed keeping
qE its magnitude unchanged and a proton falls through the
o> =
\ml same distatwe [Fig. 1.40(b) ]. Compute the time offall in each
case. Contrast the situation (a) with that of‘free fall under
2n ml
T = — =2n gravity'. [NCERT; CBSE 18C]
(0
]jqE + + + +

Example 29. A pendulum of mass 80 milligram carrying a -e + e

charge o/2xl0”®C is at rest in a horizontal uniform £ £

electric field of2 x 10“^ Vm~^. Find the tension in the thread
of the pendulum and the angle it makes with the vertical.
’ ’

Solution. Here m =80 mg =80 x 10"® kg, + +

(«)
+

(b)
(]=2xl0“®C, £=2xl0‘*Vm“^ Fig. 1.40
ELECTRIC CHARGES AND FIELD 121
7 —1
Solution. («) The upward field exerts a downward Solution. Here Dg =3 X 10 ms ,
force eE on the electron.
eE
y =2 mm =2 X :c=0.1m,
Acceleration of the electron, a = — £ = 18 Vcm“^ = 1800 V m“^

s=ut + -at^ =-at^


ma = eE or a = ^and( = i
As u =0, m V
2 2 0

Time of fall of the electron is 1^ ^


^' = 2 2 m ,u 0
● -.2

t ^ psm^. _ 2 X 1.5 X 10"^ X 9.1x10”^’


e_2yug_2x2x 10“^ X 9 X 10
14
V e£ 1.6 X 10“^^ X 2.0x10^
e

or

= 2.9 X 10"® s.
m Ex'^ 1800 X (O.lf
(I7) The downward field exerts a downward force eE = 2xlO^^Ckg"\

w
on the proton. Example 34. An electric field E is set up between the tzuo
eE
a parallel plates of a capacitor, as shown in Fig. 1.41. An
^ m electron enters the field symmetrically between the plates

Flo
V

Time of fall of the proton is with a speed Ug. The length of each plate is 1. Find the angle of
deviation of the path of the electron as it comes out of the field.
\lsm^

ee
'2s 2xl.5xl0~^xl.67xl0~^^
t
p
eE M 1.6 X 10"^® X 2.0x10“^

Fr
a
V
+ + + + + + t f +

= 1.25x10"^ s. e

Thus the heavier particle takes a greater time to fall Vo


for
ur
E
through the same distance. This is in contrast to the
situation of 'free fall under gravity' where the time of h !

fall is independent of the mass of the body. Here the


ks

acceleration due to gravity 'g', being negligibly small, Fig. 1.41


Yo
oo

has been ignored. Solution. Acceleration of the electron in the

Example 32. An electron is liberated from the lower of the upward direction.
eB

eE
two large parallel metal plates separated by a distance of a = —

20 mm. The upper plate has a potential of+ 2400 V relative m


I
to the lozoer plate. How long does the electron take to reach
r

Time taken to cross the field, t =


ou

the upper plate ? Take — of electrons 1.8 x 10^^ C kg~^.


V
ad

0
m
Upward component of electron velocity on
Y

Solution. Here V = 2400 V, =20 mm =0.02 m, emerging from field region,


-=1.8x 10^^ Ckg~^ eEl
v^=at = mv
nd
Re

m
0
Upward force on the electron exerted by electric
Horizontal component remains same, v^=v 0
Fi

field is
eV
F = eE = If 6 is the angle of deviation of the path of the
d electron, then
.-. Acceleration, V
eEl eEl
11
tan 0 = ^ or 0 = tan ^
mvl ■
2
£ eV 1.8x10“ X 2400 V mv
fl = — ms ^ =2.16 X 10^^ ms -2 X 0
m md 0.02
Example 35. A charged particle, of charge 2pC and mass
Using, s = ^ af^, we get 20 milligram, moving with a velocity of 1000 m/s entres a
'2s '2d 2 X 0.02 uniform electric field of strength 10^ NC~^ directed
t = s = 1.4 X 10 ® s.
16 perpendicular to its direction of motion. Find the velocity
a a
^2.16 X 10 and displacement, of the particle after 10 s. [CBSE SP 11]
Example 33. A stream of electrons moving with a velocity Solution. The velocity of the particle, normal to the
0/ 3 X 10^ ms"^ is deflected by 2 mm in traversing a distance direction of field.
of 0.1 m in a uniform electric field of strength 18 V an~^. = 1000 ms \ is constant
Determine e/m of electrons.
s
'
L28

The velocity of the particle, along the direction of


field, after 10 s, is given by
PHYSICS-XII

3. (/)«=- =
m

m
1.6x10"^^ X 3.34x10^
1.67 X10
-27

. 2 X10"^ X 10^ X10 -1 = 3.2x10“ ms" 2.


=0 + t = =2000 ms
m 10x10"^
The net velocity after 10 s,
(jj) S = 0+ I
2s 2 X 0.02
^v; + v^ =yl(1000f +(2000f =1000V5 = 3.54x10"2&
-1
V = ms t =
,11
a
■y 3.2x10
Displacement, along the :r-axis, after 10 s, {Hi) The field must act vertically downwards so that
X = 1000 X 10m = 10000 m
the positively charged proton falls downward.
Displacement along y-axis (in the direction of field) 4. As the particle is throvm against the field, so
after 10 s, f qE 40x10“^ X10^ = -4xl0^ms 2
1 2x10"^x102 -6
f2=-x x(10)2 m m 100x10
2 m 1 10x10"^ - iP~ + las
=10000 m
q2 =(200)2+ 2(-4xl0^)xs
Net displacement, " ~l
200x200
s = = 0.5 m.

r= = 7(10000)^+{10000)2 ^ 10000V2 m. 8x10

5. a =—F _qE 3.2x10'^^ xl.6xl0^


-27
= 8xl0“ms 2
roblems For Practice m m 6.4x10
-1^ = las
1. Calculate the electric field strength required to just
i>2-0 = 2x8xl0“x2xl0"2
support a water drop of mass 10"^ kg and having a
charge 1.6 X 10"^^ C. (CBSE OD 99] u = 4>/2xl0^ ms"^.
(Ans. 6.125x10^^ NC"^)
119 ELECTRIC FIELD DUE TO A POINT CHARGE
2. In Millikan's experiment, an oil drop of radius
10“'^ cm remains suspended between the plates 31. Obtain an expression for the electric field
which are 1 cm apart. If the drop has charge of 5e intensity at a point at a distance r from a charge q. What
over it, calculate the potential difference between is the nature of this field ?
the plates. The density of oil may be taken as Electric field due to a point charge. A single point
1.5 gcm“2. (Ans. 770 V) charge has the simplest electric field. As shown in
3. A proton falls down through a distance of 2 cm in a Fig. 1.42, consider a point charge q placed at the origin O.
uniform electric field of magnitude 3.34 x lO^NC"'. o p
F
Determine (j) the acceleration of the electron (ii) the %
Source
time taken by the proton to fall through the charge
Test

distanceof 2 cm, and {Hi) the directionof the electric charge


field. Mass of a proton is 1.67 x 10 kg. Fig. 1.42 Electric field of a point charge.
(Ans. 3.2 X 10“ms"2, 3.54 x lO"^ s.
We wish to determine its electric field at a point P at a
vertically downwards)
distance r from it. For this, imagine a test charge q^
4. A positive charge particle of 100 mg is thrown in placed at point P. According to Coulomb's law, the
opposite direction to a uniform electric field of force on charge is
strength 1 x 10^ NC"’. If the charge on the particle is
40 pC and the initial velocity is 200 ms"', how much
distance it will travel before coming to the rest '0

momentarily ? IJEE Main June 22j (Ans. 0.5 m)


where r is a unit vector in the direction from q to q^.
5. A a-particle of mass 6.4xl0"2^kg and charge Electric field at point P is
3.2xlO”'^C is situated in an electric field of
1.6 xl0^Vm"'.If the particle starts from rest, find its F
£ = —
1 q
y
-*
^
r
velocity at the end of 2xl0~2m path. % 47uEq r
(Ans. W2xl0^ms"')
The magnitude of the field £ is
HINTS
4 3
2. Use — nr p g ~ tie — .
3
V
d
£=—■4
4jten 0
ELECTRIC CHARGES AND FIELD 1.29

Clearly, £ <xl/r^. This means that at all points on Hence, the electric field at point P due to the system
the spherical surface drawn around the point charge, of N charges is
—►

the magnitude of £ is same and does not depend on


->

E = Ej + £2 +... + £^
the direction of r. Such a field is called spherically
symmetric or radial field, i.e., a field which looks the -^—\lLr +-^r 2P
+... + UtLrNP
same in all directions when seen from the point charge. 2P NP

1.20 ELECTRIC FIELD DUE TO A SYSTEM


or £ =
OF POINT CHARGES 4716
-0 i=l 'W £
32. Deduce an expression for the electric field at a
point due to a system ofN point charges.
'Vv

w
Electric field due to a system of point charges.
Consider a system of N point charges ^2' ●' ^N ' %
V'" A

having position vectors ...... ^ with respect to the £.

Flo
origin O. We wish to determine the electric field at

v-y

ee
£4
, u* '

Fr
►4

^ip £1
" >.''4P
for
ur
I
I
’’2P
ir3P
“I: I
ks
I
Yo
I
oo

*<?3
Fig. 1.44 Electric field at a point due to a system of
eB

Fig. 1.43 Notations used in the determination of electric charges is the vector sum of the electric
field at a point due to two point charges, fields at the point due to individual charges.

point P whose position vector is r*. According to


r

In terms of position vectors, we can write


ou
ad

—» —»

Coulomb's law, the force on charge test due to 1 w r - r

charge q^ is
E = - z
Y

47te
p- 1 Mo;IP
0 1 = 1
\r -r,f r ~ r
;

1 W
^ 47t6o ■ jfp
nd

-*

- z
Re

or £ = ('●
4716 i3
0 1 = 1
where r^p is a unit vector in the direction from to P r -r.\
Fi

and r^p is the distance between r/j and P. Hence the mples based on
electric field at point P due to charge q^ is
Electric Fields of Point Charges
F. 1
1
£1 = 2
r
IP Formulae Used
% 4716^ rfp
Similarly, electric field at P due to charge 1/2 is
1. £= — .4
4tc8o r
1
£2 = ● 2 7P
2. By the principle of superposition, electric field
47re
0
7
'2P due to a number of point charges.
-» —> —>

According to principle of superposition of electric E = ^ + ^+ ^ + ...


fields, the electric field at any point due to a group of charges
Units Used
is equal to the vector sum of the electric fields produced by
each charge individually at that point, when all other charges When q is in coulomb and r in metre; £ is in NC”^
are assumed to be absent. or Vm~L
1.30 PHYSICS-Xll

Example 36. Assuming that the charge on an atom is Example 39. Two point charges of +16 pC and -9 pC are
distributed unifortnli/ in a sphere of radius 10"^*^ m, zvhat placed 8 cm apart in air. Determine the positioii of the point
zvill be the electric field at the surface of the gold atom ? at which the resultant field is zero.
For gold, Z = 79. Solution. Let P be the point at distance x cm from
Solution. The charge may be assumed to be -to
con A, where the net field is zero.
centrated at the centre of the sphere of radius 10 m.

r = 10“^®m, q = Ze=79xl.6xlO-^^C = + 16 nC = - 9 |iC

1 q 9x10^x79x1.6x10
-19
A P B
E =
♦H- 8-x
4tc£q r^ (10-10)2
= 1.138 X 10^^ Fig. 1.46
Example 37. Two point charges A and B of magnitude
+8xlO"^C and -8xlO"^C respectively are placed at a At point P, + £2 = 0
distance d apart. The electric field at the middle point O kxlSxlO^^ kx{-9)xlQ-^
between the charges is 6.4xlO‘*NC"\ Find the distance'd ' = 0
between the point charges A and B. [jEE Main June 22) (ATX 10"^)^ [(8-;r)xl0“^f
Solution. 16 9
or
E
+ 8x10
-6
C -8x10
-6
C
(8-xf
A0- 4 4 3
d/2 O d/2 or — = +
x 8-x
^-0
32
= 2-^ —
8kq or x = — cm, 32 cm
7
32
{2j
At X = — cm, both E, and will be in the same
7 ^ ^
^2 _ ^ _8x9x10^x8x10"^ =9 direction, therefore, net electric field cannot be zero.

6.4x10^ Hence x = 32 cm

rf = 3 m. i.e., electric field is zero at a point 24 cm to the right of


-9 pC charge.
Example 38. Two point charges of +lpC and -i-4pC are
kept 30 cm apart. How far from the +lpC charge on the line Example 40. Two point charges q.^= +0.2 C and
joining the two charges, will the net electric field be zero ? 4/2 = + 0.4 C are placed 0.1 m apart. Calculate the electric
(CBSE OD 20] field at
Solution.
(rt) the midpoint between the charges,
= +1 pC (j2 = + 4pC
4 {b) a point on the line joining and ^2 smc/i that it is
A £2 P £1 B
0.05 m away from <^2 and 0.15 m away from q^
0.30 - X
Solution, (a) Let O be the midpoint between the
Fig. 1.45 two charges.
The electric field at point P will be zero if 9, = + 0.2C (?2 = + 0.4 C
£,=£2 A O B
-6

or
1 lxl0~^^ 1 4x10 £j £2
M- 0.1 m M
4716^ x^ 47ieg (0.30-x)^
or (0.30-x)^=4x^ or 0.30-x=+2x Fig. 1.47
or x=0.10m or x = -0.30m

At X = - 0.30 m i.e., at 30 cm to the left of q^ both Ej Electric field at O due to q^


and £2 will be in the same direction, therefore net _^kq^ _9xl0%0.2 = 7.2 X 10^^ NC"\
electric field cannot be zero. Net electric field will be
zero at x = +0.10m i.e., at 10 cm to the right of +lpC
r^ ~ {O.OSf
charge. acting along AO
ELECTRIC CHARGES AND FIELD 1.31

Electric field at O due to q2> The electric field vector Ej at A due to the negative
_ kq^ _9x 10^ X 0,4 charge £^2 points towards the right and it has a
^ ~ (0.05)^ magnitude,
9x10^x10'^
= 14.4x 10'^ NC \ acting along BO ^2 = NC”’ =3.6X 10“^ NC"^
Net field at O = £2 - £^ (0.05)^
= 7.2 X 10^' NC acting along BO. Magnitude of the total electric field at A
E^=E^ + £2
{b) Electric field at P due to q^, = 3.6 X 10^^ + 3.6 X 10^ = 7.2 x 10“^ NC"^
9 X 10^ X 0.2
, acting along AP
E^ is directed towards the right.
The electric field vector £^ at B due to the positive

w
Electric field at P due to q2,
_ 9 X 10^ X 0.4 , acting along BP
charge q^ points towards the left and it has a
(0.05)^ magnitude,

Flo
9x10^x10'®
E,1 = NC’=3.6x10^ NC’
i7i=+0.2C ^2 = + 0.4 C £] ►
(0.05)2

ee
A B
£2 ►
0.1m 0.05 m
The electric field vector £2 at B due to the negative

Fr
M-

charge ^2 points towards the right and it has a


Fig. 1.48 magnitude.
9x10^x10"^
NC’ = 4x10^ NC“’
Net electric field at point P is for
ur
^2 =
0.2 0.4 (0.15)2
£= £j + £2=9x10^ +
Magnitude of the total electric field at B
(0.15)2 (0.05)2
ks

= 1.52 X 10’2 NC'’, acting along AP.


£^ = £^ - £2 = 3.2 X 10^ NC'*
Yo
oo

Efj is directed towards the left.


Example 41. Two point charges q^ and f^2 of 10 ^ C and
Magnitude of each electric field vector, at point C,
eB

-10 C respectively are placed 0.1 m apart. Calculate the


electric fields at points A, B and C shown in Fig. 1.49. of charges £jj and £^2 is
[NCERT] 9x10^x10'®
£i = £2 = = 9x 10^ NC -I
r

(0.1)2
ou
ad

The directions in which these two vectors point are


shown in Fig. 1.49. The resultant of these vectors is
Y

given by
£^=^£j2+ £^2 + 2 £j £2 cos 0
nd
Re

= ^{9 X 10^)2 + (9 X 10^)2 + 2 X 9 X 10^ X 9 X 10^ cos 120“


Fi

=9x 10^.yi+l + 2(-l/2) NC“’ = 9x 10^ NC'’


Since £^ and £2 are equal in magnitude, so their
resultant acts along the bisector of the angle
Fig. 1.49 between £j and £2, i.e., towards right.

Solution. The electric field vector £j at A due to the Example 42. ABCD is a square of side 5 m. Charges of
+ 50 C, - 50 C and + 50 C £jre placed at A, C and D
positive charge £/^ points towards the right and it has a respectively. Find the resultant electric field at B.
magnitude.
Solution. Electric field at B due to + 50 C charge at
_ cf£/i _9x10^x10"^ NC
-I
A is
rf ~ (0.05)2
= 3.6x10^ NC"’ E.y = k ~
Y
=k 5
=2k, along AB
PHYSICS-XII

£3 Example 43. Four charges + q, + q,-q,-q arc placed


Ei
respectively at the four comers A, B, C and D of a square of
A 5 m 6X45°
+ 50C - - ►ar side 'a'. Calculate the electric field at the centre of the square.
[Punjab 96C]
E
£2 Solution. Let £'A' Eg, and £p be the electric fields
5 m
5 m
at the centre O of the square due to the charges at A, B,
C and D respectively. Their directions are as shown in
Fig.
+ 50C # -50C
D 5 m

Y
Fig. 1.50 y

Electric field at B due to -50 C charge at C is


£2 = k~=2k,
52
along BC

Electric field at B due to + 50 C charge at D is


50
£, = cf . —= = k, along DB Fig. 1.51

Since all the charges are of equal magnitude and at


Component of £j along x-axis =2k the same distance r from the centre O, so
(as it acts along x-axis)
Component of £2 along x-axis = 0 £^ = £g = ^ = £q = /r ~ a
2

(as it acts along y-axis)


J [●.* r^ + r^ = a^]
Component of £3 along x-axis
Because E^ and £^ act in the same direction, so
= £3 cos 45° = k V2- their resultant is
2kq 2kq _4kq
Total electric field at B along x-axis 2
a a
1 ^
= 2fc + 0 + 4 = k 2 +
Similarly, resultant of Eg and Eg, is
J2
Now, £ - £ + £ =1^2
a

Component of £j along y-axis =0


Now, the resultant of £^ and £2 will be
Component of £2 along y-axis =2k
Component of £3 along y-axis +
4kq ^2
2
a
Ey = £3 sin 45° = %/2
But the components of £, and £3 act in opposite direc
= 4^2 'i4,
a^
tions, therefore, total electric field at B along y-axis directed parallel tO/4D or BC, as shown in Fig. 1.51(fj).
k ( 1 ^
= 2k-4= = k 2- ^
^/2 I J2) cos ^p = -^£ = V2 ●. p=45°
Resultant electric field at B will be
i.e., the resultant field is inclined at an angle of 45° with AC.
£ =
Example 44. Two point charges+6q and -Sqare placedat
the vertices 'S' and 'C of an equilateral triangle ABC of side
k f 2+^
1 + k 2- ‘a' as shown in Fig. 1.52(a). Obtain the expression for (i) the
n magnitude and (ii) the direction of the resultant electric field
= 3fc =3 X 9 X 10^ NC "^ = 2.7 X 10^° NC at the vertex A due to these two charges. [CBSE OD 14C)
If the resultant field £ makes angle P with x-axis, then Solution, (i) As shown in Fig. 1.52(1?), the fields at
.._o ^ {2-lN2)k
t3n p — — — — = 0.4776 or p = 25.5°. point A due to the charges at B and C are £g^ and
£^ {2+l/^)k respectively.
ELECTRIC CHARGES AND FIELD

5. Two point charges +q and -2q are placed at the


vertices 'B' and 'C of an equilateral triangle ABC
of side ‘a' as shown in Fig. 1.53. Obtain the
1.33

/
expression for (i) the magnitude and (ii) the
direction of the resultant electric field at the vertex
A due to these two charges. [CBSE OD 14C1

(ii) 30° with AQ

Fig. 1.52

Their magnitudes are

w
1
.~=6E, where £ =
47TE a
0

£^c=—-^-8£

Flo
6. Find the magnitude and direction of electric field at
47te -2 0
a
point Pin Fig. 1.54.

ee
The magnitude of the resultant field is

Fr
£
net ~ -J^BA + ^AC +2 £g^ cosl20°

'(6E)^+(8£)^+2x6£x8£xf-i for
ur
I 2)

= £^/52 = 1 q-M
2
ks
4716 a
0
Yo

(if) If the resultant field makes an angle p with AC, then


oo

£^sinl20° 6£x(^^/2) 3^/3 Fig. 1.54


eB

tanp = -
+ £g^ cos 120' { =5
AC 8£+6£ - ^ 1 2q
along BP produced
4;t8r,0 '
3V3
r

p = tan ^ — 7. Three charges, each equal to q are placed at the tliree


ou
ad

s
corners of a square of side a. Find the electric field
at the fourth comer.
Y

roblems For Practice


Ans. (2v^+ 1) L
nd

HINTS
Re

1. An electron is separated from the proton through a


distance of 0.53 A. Calculate the electric field at the 1. Electric field at the location of the electron,
Fi

location of the electron.


(Ans. 5.1x10” NC"*) q _ 9x10^ x1.6x10"^9
= 5.1x10^^ NC"V
2. Determine the electric field produced by a helium 47160 ' (0.53x10"^®)^
nucleus at a distance of 1 A from it. -10
2. Here q = + 2e and r = 1 A = 10 m.

(Ans. 2.88x10” NC"’)


3. Two point charges + q and + 4q are separated by a 3. Suppose the electric field is zero at distance x from
distance of 6a. Find the point on the line joining the the charge + q. Then
two charges where the electric field is zero. _J_ 1 4q
(Ans. At a distance 2a from charge + q) 47T60 x'^ 471:60 (6a-x)^
4. Two point charges q^ and 4/2 of 2 x 10”®C and or (6a-x)^ = 4x^ or 6a X = 2x

- 2 X 10"^C respectively are placed 0.4 m apart. or x = 2a


Calculate the electric field at the centre of the line
Electric field is zero at distance 2a from the
joining the two charges. ICBSE F 94C1
charge + q.
(Ans. 900 NC~\ towards tlie -ve charge) 4. Proceed as in Exercise 1.8 on page 1.81.
PHYSiCS-XII

.^0
5. Proceed as in the solution of Example 44 on ciP = 1 ;
Am
page 1.32. 0

6. Here and are equal and opposite and hence


A r
cancel out. where r = — , is a unit
r
BP = n sin 45° = <i / V2
vector pointing from the
1
Hence E~ E B 1 small charge dq towards

4TrSg the point charge q^. By
the principle of super
4ire
^ ^ , along BP produced.
0
a
position, the total force
Fig. 1.56 Force on a point charge
on charge qQ will be the
7. Refer to Fig. 1.55. vector sum of the forces q g due to a continuous charge
distribution.

Ec E, exerted by all such


small charges and is
A D
given by

F = ■ r
■' 4718
0

9/ or

4718q ■*

(i
34. Name the different types of continuous charge
distributions. Deifne their respective charge densities.
Write expression for the electric field produced by each
Fig. 1.55
type of charge distribution. Hence write expression for
the electric ifeld of a general source charge distribution.
Different types of continuous charge distributions.
\2 There are three types of continuous charge distributions:
1 <1
+ +
(a) Volume charge distribution. It is a charge distri
\[AKBQa^ An 8n0 AKC^iflnf bution spread over a three dimensional volume or region Vof
space, as shown in Fig. 1.57. We define the volume charge
= (2V2 + 1)—2 density at any point in this volume as the charge contained
An e
0 871 Eq a^ per unit volume at that point, i.e.,
P=^
dV
1.21 CONTINUOUS CHARGE DISTRIBUTION
The SI unit for p is coulomb per cubic metre (Cm ).
33. What is a continuous charge distribution ? How
can we calculate the force on a point charge q due to a For example, if a
continuous charge distribution ? charge q is distributed
Continuous charge distribution. In practice, we
deal with charges much greater in magnitude than the of a sphere of radius R,
charge on an electron, so we can ignore the quantum then its volume charge
nature of charges and imagine that the charge is spread density i IS

in a region in a continuous manner. Such a charge


distribution is known as a continuous charge distribution. P = -P-Lcm'^
Calculation of the force on a charge due to a conti- 3
nuous charge distribution. As shown in Fig. 1.56,
The charge con
consider a point charge q^ lying near a region of contin- tained in small volume dq = pdV
uous charge distribution. This continuous charge distri

bution can be imagined to consist of a large number of


Fig. 1.57 Volume charge
small charges dq. According to Coulomb's law, the dq = p dV distribution
force on point charge qQ due to small charge dq is
ELECTRIC CHARGES AND FIELD

Total electrostatic force exerted on charge due to density at any point on this line as the charge per unit
the entire volume V is given by length of the line at that point, i.e.,
% do A
%
-*

f.V ~ -I'' = 4tis ^dVr


4tu£o 1. r 0 V

Electric field due to the volume charge distribution


at the location of charge is

E*V 1 ^dVr.
% 4tc£o J
(b) Surface charge distribution. It is a charge
distribution spread over a two-dimensional surface S in

w
space, as shown in Fig. 1.58. We define the surface
charge density at any point on this surface as the charge per
unit area at that point, i.e.,

Flo
G = —^
dq Fig. 1.59 Line charge distribution.
dS
For example, if a charge q is uniformly distributed

ee
The SI unit for a is Cm
over a ring of radius R, then its linear charge density is
%

Fr
2nR
r

The charge contained in small length dl is


for
ur
/+ + + +/+ ^
dq = XdL
//+ + +;> ;:
h * * >4<
/+ + + Total electrostatic force exerted on charge due to
^s; f s
ks
^ ^ the entire length Lis given by
Yo
+
/+_+ + +
~dLr
oo

4k£^0 ●’L r^
dq = adS
eB

Electric field due to the line charge distribution at


Fig. 1.58 Surface charge distribution.
the location of charge q^ is
For example, if a charge q is uniformly distributed Ei-A
r

X
over the surface of a spherical conductor of radius R, -^dlr
ou
ad

%
then its surface charge density is
Y

The total electric field due to a continuous charge


G -
■^Cm-2
4nR^ distribution is given by
nd

lorn =^v + '^s+h


Re

The charge contained in small area dS is


dq= a dS
Fi

1 X
Total electrostatic force exerted on charge <7^ due to
or £
cont KdVr-^i -^dSr+f Ardlr
the entire surface S is given by
4718
0 V s ^ I ^ \
So a General charge distribution. A general charge distri
fs = 47te
^dSr
bution consists of continuous as well as discrete charges.
0 s
Hence total electric field due to a general charge
Electric field due to the surface charge distribution distribution at the location of charge q^ is given by
at the location of charge q^ is
^ total ^ discrete + E
cont

1
tg -
% 4 TIE J or E
^ total
= 1
— Z
^ ''I- + ^dVr
0 s 4tc£ 0 i = l 7^
I V

(c) Line charge distribution. It is a charge


X , A
distribution along a one-dimensional curve or line L in
G
+ dS r -I- ^ dir
space, as shown in Fig. 1.59. We deifne the line charge s ^
PHYSICS-XII

A , .. Example 46. Sixhi four drops of radius 0.02 mand each


In all the above cases, r
carrying a charge of 5 nC are combined to form a bigger drop.
vector directed from each point of the volume, surface surface density of electrification will change if
or line charge distribution towards the location of the UEE Main June 221
no charge is lost.
point charge qQ. Solution. Volume of each small drop

i'ii t f
t ’
r

ife;■:,T. 171 (0.02 f


Formulae Used
Volume of 64 small drops =
|7c(0.02f x64
dq Let R be the radius of the bigger drop formed. Then
1. Volume charge density, p = dV

2. Surface charge density. dS


3 |::{0.02)^x64
dq
or R^={0.02)^x4^
3. Linear charge density, Cit-d
R = 0.02 x 4=0.08m

4. Force exerted on a charge i^q due to a continuous Charge on small drop = 5 pC = 5x10 ^ C
charge distribution. Surface charge density of small drop,
^ _ 5 X 10"^
4tiEq r^
(j
1
4tc?-^ 471(0.02)
2 Cm-^
5. Electric field due to a continuous charge distribution,
Surface charge density of bigger drop,
5 X 10~^ X 64
4;rs 0 02 =
Cm'^
4ti:(0.08)^
Units Used
p is inCrn”^, oin Cm"^, X in Cm
-1
and Ein NC ^
o
1 5xlQ-^ 471(0.08)^
_ -1-4
o
””471 (0.02)^ " 5 X 10“^ X 64 4
Example 45. A charged spherical conductor has a surface Example 47. Obtain the formula for the electric field due to
density of 0.7 CnC^. When its charge is increased by 0.44 C, a long thin wire of uniform linear charge density X without
the charge density changes by 0.14 Cm~^. Find the radius of using Gauss's lazo. {NCERT1301
the sphere and initial charge on it. Solution. Electric field of n line charge from
Coulomb's law. Consider an infinite line of charge with
Solution, o = —^
4717^ uniform line charge density X, as shown in Fig. 1.60.
We wish to calculate its electric field at any point P at a
In first case: 0.7 = —^ ●●●(0 distance y from it. The charge on small element dx of
4nr'^
the line charge will be
hi second case: dq = Xdx
0.7 + 0.14 =
q + 0.44
4tu r^
dE; dE^
0.84 =
q + 0.44 ...{ii)
or
4717^ e

Dividing (ii) by (/), we get,


0.44 0
0.84 (j + 0.44 or
6
-=1 +
0.7 4 5 9
y
.-. Initial charge, q = 2.2 C.
22 X
From (/), r=
ox 4ti 07 X 471
O -H frxH-
2.2x7
- 0.5 m.
0.7x4x22 Fig. 1.60 A section of an infinite line of charge.
ELECTRIC CHARGES AND FIELD

Tlie electric field at the point P due to the charge


element dq will be
Xdx
dE = 1
4nep 4;reQ
dE cos e

The field dE has two components :


dE^ = - dE sin Q and = dE cos 0

The negative sign in ;r-component indicates that


d E^ acts in the negative Ar-direction. Every charge ele


ment on the right has a corresponding charge element Fig. 1.61
on the left. The A-components of two such charge ->

elements will be equal and opposite and hence cancel .-. The magnitude of the field d E produced by the

w
—►
element dl at the field point P is
out. The resultant field £ gets contributions only from
y -components and is given by at
,r =+oo 2ii(i

Flo
£=£y=Jrf£^ = cos0rf£
As shown in Fig. 1.61, the field dE has two
X=-<r>

components :

ee
X =00
1 X dx
= 2 cos 0. — 1. the axial component dE cos 0, and

Fr
x = 0 47rSo ' y^+x^ 2. the perpendicular component dE sin 0.

A. dx Since the perpendicular components of any two


cos 0 — diametrically opposite elements are equal and
for
ur
9 2
2 OT
y + X
0 x = 0
opposite, they all cancel out in pairs. Only the axial
Now X = y tan 0 components will add up to produce the resultant field
ks
E at point P, which is given by
dx =y sec^0 frO
Yo

2jtn
oo

0 = n/2 £ = dECOS 0
E
X
COS 0
y sec^ 0 dQ
[●.' Only the axial components
eB

0
2 OT
0 9= 0 y^ (l + tan^O) contribute towards £]
2ra 2nn

X
0 = 7t/2 kq dl X _ kqx 1 dl
r

COS 0 fr0 = 2 OT r^ r 2to


ou

0 0
2JtSoy 2^£oy
ad

0=0
Q
COS 0 = —
^
Y

X n r
sin — sin 0
Ine^y 2 lin-2ruJ kqx 1
.2to
"2TO-r3l'Jn 2TO'(Ar^ + fl2“)3/2
nd
Re

X
or £ = [v =
2TOpy
Fi

or £ =
kqx 1 qx
Example 48. A charge is distributed uniformly over a ring
of radius 'a'. Obtain an expression for the electric intensity £
at a point on the axis of the ring. Hence show that for points Special Case
at large distances from the ring, it behaves like a point For points at large distances from the ring, x»a
charge. (CBSE D 16, 20] £ = -^=^—
Solution. Suppose that the ring is placed with its 4TOq
plane perpendicular to the x-axis, as shown in Fig 1.61. This is the same as the field due to a point charge,
Consider a small element dl of the ring. indicating that for far off axial points, the charged ring
As the total charge q is uniformly distributed, the behaves as a point charge.
charge dq on the element dl is Example 49. A thin semicircular ring of radius a is
dq - —^. dl 2k a
charged uniformly and the charge per unit length is X. Find
the electric field at its centre. |CBSE PMT 2000, AIEEE 2010]
138 PHYSICS-XIl

Solution. Consider two symmetric elements eacli HINTS


of length dl at A and R The electric fields of the two 1. Usec7 = —
elements perpendicular to PO get cancelled while 47ir^ ’
those along PO get added. 2. Use q = 4rcr^a
Electric field at O due to an element of length dl is 3. Surface area of cube = 6 x /^ = 6 x 0.01 = 0.06
1 dq
dE =
j-cosQ [Along PO] 4. -7iR^=2x-7ir^ or R= r
4;rsn0 a 3 3
2
1 Xdl
q 4k P} _ 2^ ?
47cen0 a
^cosO o.
1 _ = 2^'^ : 2
^2 4nr^ 2q 2r^ 2r^
1 X{adQ)
2—COsO [dl = adQ]
4ks
0
a 1.22 ELECTRIC DIPOLE
35. What is an electric dipole ? Define dipole moment
and give its SI unit. Give some examples of electric
dipoles. What are ideal or point dipoles ?
dE
Electric dipole. A pair of equal and opposite charges
t separated by a small distance is called an electric dipole.
+ Dipole moment. It measures the strength of an
electric dipole. The dipole moment of an electric dipole is a
vector whose magnitude is either charge times the separation
dE betzveen the two opposite charges and the direction is along
the dipole axis from the negative to the positive charge.
-t + As shown in Fig. 1.63, consider an electric dipole
consisting of charges + q and - q and separated by dis
Fig. 1.62 tance 2 a. The line joining the charges is called dipole axis.
Total electric field at the centre O is -4 la + <7

nil n/2 P
1 XcosOdO
£= Jd£=2 4;t6 0 a Fig. 1.63
-nil 0

1 X it/2 _ \ X X Dipole moment = Either charge x a vector drawn


-[sinG] 0 from negative to positive charge
Incr.0 a InZr.0 a 27ts„fl
0
->

7>
or p=qx2a
roblems for Practice
Thus the dipole moment is a vector quantity. Its
1. A uniformly charged sphere carries a total charge direction is along the dipole axis from ~qto + q and its
of 2n X 10 ’^C. Its radius is 5 cm and is placed in magnitude is
vacuum. Determine its surface charge density.
p = qx2a
Cm-2)
-10
(Ans. 2 X10
2. What charge would be required to electrify a The SI unit of dipole moment is coulomb metre (Cm).
sphere of radius 15 cm so as to get a surface charge When both the charge q and separation 2a are finite,
densit of-^ Cm'^ ? dipole has a finite size (equal to 2a), a location
erisiyo ● (Ans. 1.8 X10"’'C) (midpoint between +q and -q), a direction and a
3. A metal cube of length 0.1 m isis charged by 12 pC. strength.
Calculate its surface charge density. Examples of electric dipoles. Dipoles are common
(Ans. 2 X10"* Cm"^) in nature. In molecules like H2O, HCl, C2H5OH
4. Two equal spheres of water having equal and CH3COOH etc, tlie centre of positive charges does not
similar charges coalesce to form a large sphere. If fall exactly over the centre of negative charges. Such
no
charge is lost, how will the surface densities of molecules are electric dipoles. They have a permanent
electrification change ? (Ans. <^1:^2 = 2^^^ : 2) dipole moment.
ELECTRIC CHARGES AND FIELD

Idctil or point dipoio. We can think of a dipole in Electric field due to charge + ij at point P is
which sizeZfl 0 and charge ^ co in such a way that q
the dipole moment, p = qx2a has a finite value. Such a
E
+‘J J p (towards right)
47t£o (r-fl)
dipole of negligibly simll size is called an ideal or point
dipole. Hence the resultant electric field at point P is
Dipoles associated witli individual atoms or molecules E
axial
= r + r
-0
may be treated as ideal dipoles. An ideal dipole is
specified only by its location and a dipole moment, as 1 1

4k£q [{r-af (r + af ^
it has no finite size.

123 DIPOLE FIELD q Aar «

36. What is a dipole ifeld ? Why does the dipole field at 4tiEo ^
large distance falls offfaster than 1/r^ ?

w
-» 1 2pr
Dipole field. The electric field produced by an electric
or
^axial
AkEq {,
dipole is called a dipole field. This can be determined by
Here p = qx2a = dipole moment.

Flo
using (a) the formula for the field of a point charge and ●y 'j
(b) the principle of superposition. For r» a, a can be neglected compared to r .
Variation of dipole field with distance. The total

ee
t'axial 1 2p A
charge of an electric dipole is zero. But the electric field .-f p (towards right)

Fr
of an electric dipole is not zero. This is because the 4nep r^
charges + q and - q are separated by some distance, so Clearly, electric field at any axial point of the dipole
the electric fields due to them when added do not
acts along the dipole axis from negative to positive
for
ur
exactly cancel out. However, at distances much larger
than the dipole size (r»2a), the fields of + q and -q charge i.e., in the direction of dipole moment p.
nearly cancel out. Hence we expect a dipole field to fall
ks
1.25 ELECTRIC FIELD AT AN EQUATORIAL
off, at larger distance, faster than 1/r^, typical of the
Yo

POINT OF A DIPOLE
field due to a single charge. In fact a dipole field at
oo

larger distances falls off as 1 / r^. 38. Derive an expression for the electric field at any
eB

point on the equatorial line of an electric dipole.


1.24 ELECTRIC FIELD AT AN AXIAL POINT
OF A DIPOLE Electric field at an equatorial point of a dipole. As
shown in Fig. 1.65, consider an electric dipole consis
r

37. Derive an expression for the electric field at any ting of charges -q and + q, separated by distance 2 a
ou
ad

point on the axial line of an electric dipole. and placed in vacuum. Let P be a point on the equa
Y

Electric field at an axial point of an electric dipole. torial line of the dipole at distance r from it.
As shown in Fig. 1.64, consider an electric dipole i.e.,
consisting of charges + q and - q, separated by distance
nd
Re

2a and placed in vacuum. Let Pbe a point on the axial


line at distance r from the centre O of the dipole on the
Fi

side of the charge + q.


V
E
A 0

M- 2a M

Fig. 1.64 Electric field at an axial point of dipole.

Electric field due to charge - q at point P is


E = ^ (towards left)
-1 47tE„(r+«)"''
where p is a unit vector along the dipole axis from -q

to +q. Fig. 1.65 Electric field at an equatorial point of a dipole.


.40 PHYSICS-Xll

Electric field at point P due to + q charge is Clearly, Eaxia. =2 .£equa


1
£
4jten0 r~ +
^ ^ , directed along BP Hence the electric field of a short dipole at a dista7icc r
along its axis is twice the electric ifeld at the same distance
Electric field at point P due to ~q charge is along the equatorial line.

?, = 4jie„
^ . r + a-^ , directed along PA
0
1.26 TORQUE ON A DIPOLE IN A UNIFORM
ELECTRIC FIELD

Thus the magnitudes of £_ ^ and £+<? are equal i.e., 40. Derive an expression for the torque on an electric
1 dipole placed in a uniform electric field. Hence define
E-, = £ 47TE(, V + dipole moment.
—>
Torque on a dipole in a uniform electric field. As
normal to
Clearly, the components of £_^ and £+<j shown in Fig. 1.66(fl), consider an electric dipole
the dipole axis will cancel out. The components consisting of charges + q and - q and of length 2a
parallel to the dipole axis add up. The total electric ^ ^ uniform electric field ? making an angle 0
field ^^3 is opposite to p. with it. It has a dipole moment of magnitude,
p = qx 2a
E
cqua - - (E_ ^ cos 0 + £+ q cos 0) p -> ->

Force exerted on charge + qhy field E - q E


- 2 £_ ,^ cos 0 p -*

(along £)
= -2 .
1 ^ a

2^
4k&q ./TTfl Force exerted on charge ~qhy field ? =-q^
—►

cos 0 =
a
(opposite to £)
0

■V
r +
^Total = + q~^ -q^ =0.
1 p '' *■
E
4tiEq {f-i + a^f^^ ^
or
equn + qE
-►

where p=2qa, is the electric dipole moment. OS


c

£
If the point P is located far away from the dipole, P
7

r » a, then
1 P A
^xjua ~ 4rtEn0 4

(«)
Clearly, the direction of electric field at any point
on the equatorial line of the dipole will be antiparallel P
—>

to the dipole moment p. 6

39. Give a comparison of the magnitudes of electric E


fields of a short dipole at axial and equatorial points.
Comparison of electric fields of a short dipole at
axial and equatorial points. Tlie magnitude of the
electric field of a short dipole at an axial point at X =p xE
distance r from its centre is ih)
1 2p
^axinl 3 Fig. 1.66 (a) Torque on a dipole in a uniform electric field,
47I£n0 r
(b) Direction of torque as given by right hand screw rule.
Electric field at an equatorial point at the same Hence the net translating force on a dipole in a
distance r is
uniform electric field is zero. But the two equal and
1 P
£
cqua
opposite forces act at different points of the dipole.
47ie„0
They form a couple which exerts a torque.
ELECTRIC CHARGES AND FIELD 1.41

Torque = Either force x Perpendicular distance When the dipole is parallel or antiparallcl to £. In a
between the two forces ^ —>

non-uniform field, if p is parallel to £ or antiparallel to


T = X 2^1 sin 0 =(i? X 2fl) E sin 9
E , the net torque on the dipole is zero (because the
or
T = p£ sin 0 (p = ty X 2a) forces on charges ± q become linear). However, there
is a net force on the dipole. As shown in Fig. 1.67, when
As the direction of torque t is perpendicular to
p is parallel to £, a net force acts on the dipole in the
both p and ^ , so we can write direction of increasing £. When p is antiparallel to £^ —»

^ a net force acts in the direction of decreasing £.


T = p X t
£
The direction of vector x is that in which a right
—►
Force on-q
handed screw would advance when rotated from p to

w
Force on + q
£. As shown in Fig. 1.66(b), the direction of vector t is
0-- --0
perpendicular to, and points into the plane of paper. -7
p
+ 7

Flo
When the dipole is released, the torque x tends to Direction of net force =

Direction of increasing field =


align the dipole with the field E i.e., tends to reduce

ee
(«)
angle 0 to 0. When the dipole gets aligned with £, the

Fr
£

torque x becomes zero.


Clearly, the torque on the dipole will be maximum o--- --0
for
ur
p
+ 7 -7
when the dipole is held perpendicular to £. Thus Force Force

= sin90" = p£. on + q on - ij
ks
Direction of net force =
Yo

Dipole moment. We know that the torque,


Direction of increasing field =
oo

X = pE sin 0 ib)
eB

If £ =1 unit, 0 =90°, then x = p


Fig. 1.67 Forces on a dipole (o) when p is parallel
Hence dipole moment may be defined as the torque ^ ^ ^

acting on an electric dipole, placed perpendictdar to a uniform to E and (fe) when p is antiparallel to E.
r

electric field of unit strength.


ou
ad

A comb run through dry hair attracts small pieces


1.27 DIPOLE IN A NON-UNIFORM of paper. As the comb runs through hair, it acquires
Y

ELECTRIC FIELD charge due to friction. When tlie charged comb is


brought closer to an uncharged piece of paper, it
nd
Re

41. What happens when an electric dipole is held in a polarises the piece of paper i.e., induces a net dipole
non-uniform electric field ? What will be the force and the moment in the direction of the field. But the electric
Fi

torque when the dipole is held parallel or anti-parallel to field due to the comb on the piece of paper is not uni
the electric field ? Hence explain why does a comb run form. It exerts a force in the direction of increasing field
through dry hair attract pieces of paper.
i.e., the piece of paper gets attracted towards the comb.
Dipole in a non-uniform electric field. In a non-
uniform electric field, the + q and - q charges of a dipole 42. Give the physical significance of electric dipoles.
experience different forces (not equal and opposite) at Physical significance of electric dipoles. Electric
slightly different positions in the field and hence a net dipoles have a common occurrence in nature. A
force F acts on the dipole in a non-uniform field. Also, molecule consisting of positive and negative ions is an
electric dipole. Moreover, a complicated array of
a net torque acts on the dipole which depends on the
charges can be described and analysed in terms of
location of the dipole in the non-uniform field. electric dipoles. The concept of electric dipole is used
X = p X (i) in the study of the effect of electric field on an
insulator, and (n) in the study of radiation of energy
where r is the position vector of the centre of the dipole. from an antenna.
s 1A2

For Your Knowledge


> In a uniform electric field, an electric dipole expe
riences no net force but a non zero torque.
PHYSICS-XII

Units Used

Charge is in coulomb, distance 2a in metre,


dipole moment p in coulomb metre (Cm), field £
in NC"^ or Vm"^
> As the net force on a dipole in a uniform electric field
is zero, therefore, no linear acceleration is produced.
> Torque on a dipole becomes zero when it aligns itself Example 50. Two charges, one -i- 5 pC and another - 5 pC
parallel to the field. are placed 1 mm apart. Calculate the dipole moment.
ICBSE OD 94C]
Torque on a dipole is maximum when it is held
perpendicular to the field £ .
Solution. Here ^y=5pC = 5x 10"^ C
> In a non-uniform electric field, a dipole experiences a =1 mm =10“^ m

non zero force and non zero torque. In the special case
when the dipole moment is parallel or antiparallel to
Dipole moment.
p= qx2a = 5xl0 xl0"^=5x 10”’ Cm.
-6
the field, the dipole experiences a zero torque and a
non zero force.
Example 51. An electric dipole is placed at an angle of 60°
> A non-uniform or
0] magnitude 4xlO^NC ^ It
A B C with an electric
specifically an

increasing £-field experiences a torque ofSjS Ntn. If the length of the dipole is
may be represen 4 cm, determine the magnitude of either charge of the dipole.
ted by field lines Direction of Solution.Here0 =60°, E = 4xlO^NC~\r =8>/3Nm
as shown. increasing
Fig. 1.68 E-field 2/7 =4cm =0.04 m
Clearly, E^<Eg< E^.
> The direction of the electric field at an axial point of an As T = p£ sin 0 = tjx2flx EsinG
electric dipole is same as that of its dipole moment X
and at an equatorial point it is opposite to that of
dipole moment. (2fl)Esin0 0.04x4xl0^xsin60°
> The strength of electric field at an axial point of a short V3xl0"^x2 = 10"^C = 1 mC.
dipole is twice the strength at the same distance on
2xf3
the equatorial line.
At larger distances, the dipole field (£ccl/r^) Example 52. An electric dipole consists of two opposite
decreases more rapidly than the electric field of a charges of magnitude 1 /3 x 10'^ C, separated by 2 cm. The
point charge (£ oc 1 / ). dipole is placed in an external field of 3 x 10^ What
Examples based on maximum torquedoes the electricfield exert on thedipole ?
Dipole Moment, Dipole Field and Solution. Here o = - x 10"^ C 2a =2 cm =0.02 m,
3
Torque on a Dipole
Formulae Used £=3x10^ NC"^
1. Dipole moment, p = q x2a; where 2a is the T
= p£ sin90° =/jX 2/7X £xl
max
distance between the two charges.
2. Dipole field at an axial point at distance r from the = -x 10“^ X 0.02 X 3 X 10^ X 1 = 0.02 Nm.
3
centre of the dipole is
1
^axial ~ Example 53. Calcidate the electric field due to an electric
4nsQ'A-a^y dipole of length 10 cm having charges ofl pC at an equatorial
Whenr»/7,
1 2p point 12 cm from the centre of the dipole.
4rceo‘r^ Solution. Here q =1 pC = 10“^ C r =12 cm - 0.12 m,
3. Dipole field at an equatorial point at distance r 2/7=10 cm, /7 =5 cm =0.05 m
from the centre of the dipole is
1 E E
1 2qa
qua equa
47ieQ ‘(r^ +
1
E
When r » a, qua
4n Sq
_ 9 X10^ X 2 X10"^ X 0.05 9 x 100
(0.12^-f0.05^)^^^ (0.13)^
4. Torque, x ~ p£sin 9, where 0 is the angle between
p and £. = 4.096 X 10^ NC'\
ELECTRIC CHARGES AND FIELD

Example 54. Tzoo point charges, each of 5 j^C but opposite 9 X 10^ X 10"^ X 5 X 10"^ 1
NC"
in sign, are placed 4 an apart. Calculate the electric field (15xl0"^f
intetzsity at a point distant 4 cm from the midpoint on the
axial line of the dipole. [Punjab 02] = 1.33 X 10^ NC“\ along BA .
Solution. Here q = 5x 10“^C, 2a =0.04 m, This field is directed opposite to the direction of the
a =0.02 m, r =0.04 m dipole moment vector, i.e., from + q to-q, as shown in
Fig. 1.70.
1 2pr
^axial
4iiZq (r^-a^y
1 2 {qx2a) r
{j^-a^y

w
-Fa
9x10^x2x5x10 X 0.04x0.04

[(0.04)2-(0.02)2 f

Flo
144
-8
= 10® NC‘\
144x10

ee
A
Example 55. Two charges ±10pC are placed 5.00 mm -10^C
O
+ 10 ^C
apart. Determine the electric field at (a) a point P on the axis

Fr
of the dipole 15 cm away from its centre O on the side of the Fig. 1.70
positive charge, (b) a point Q, 15 cm away from Oona line
passing through O and normal to the axis of the dipole. for
Example 56. The force experienced by a unit charge when
ur
[NCERT] placed at a distance of 0.10 m from the middle of an electric
Solution. Here (/ = 10pC = 10 dipole on its axial line is 0.025 N and when it is placed at a
ks
distance of 0.2 m, the force is reduced to 0.002 N. Calculate
2fl = 5 mm = 5 x 10”^ m
Yo

the dipole length.


oo

r = 15 cm = 15 X 10"2 m 1 2pr
Solution. £'axial
4nEo’(r2-fl2)2
eB

(a) Field at the axial point P of the dipole is


In first case:
4k e.^r^ r = 0.10 m, ,£‘axial = 0.025 N
r

0
ou
ad

2 X qx2a
0.025 =
9xl0^x2px0.10 ...(0
4k
[(0.10)2-fl2]:
Y

9 X 10^ X 2 X 10'^ X 5 X 10“^ -1


NC In second case :
(15x10-2)3
nd
Re

r = 0.2 m, .£'axial = 0.002 N

= 2.66 X 10® NC"' , along AB. 9xl0^x2px0.2


Fi

0.002 = ...(ii)
This field is directed along the direction of dipole [(0.2)2-a2]'
moment vector, i.e., from -q to +q, as shown in Dividing (/) by (ii), we get
Fig. 1.69.
0.025 _ 0.10 [(0.2)2-fl2f
A O B £p 0.002 0.2 ■ [(0.1)2-fl2]2
+

-10^C + 10mC p
or ^^1 [(0.2)2
2 2 ' [(0.1)2-fl2f
Fig. 1.69
or ^ _ 0.04 - fl2
(b) Field at the equatorial point Q of the dipole is ~ 0.01-fl2

F - P - a = 0.05 m
^ 4ke,j^
0
47t£«r®
0 Dipole length =2a = 0.10 m.
PHYSICS-XII

4. Here r»n
roblems For Practice
^axial
1. An electric dipole of dipole moment 4 x lO'^Cm is 4ti£q 4ti£q
placed in a uniform electric field of 10~^NC~^ 9xl0%2x0.2xlQ-‘^xl0~^
making an angle of 30° with the direction of the
field. Determine the torque exerted by the electric (0.1)='
field on the dipole. (Ans. 2 x 10"® Nm) = 3.6x10“^ NC^’.
2. A dipole consisting of an electron and a proton 5. Here q - 100 )iC = lO'^^C 2n = 10 cm = 0.10 m
separated by a distance of 4 x 10“'”m is situated in p^qx2a = 10~^ xO.lO = 10'^ Cm
an electric field of intensity 3xlO^NC“^ at an
angle of 30° with the field. Calculate the dipole
moment and the torque acting on it. Charge on an
electron = 1.602 x 10" C.

(Ans. 6.41 X10"^^ C m, 9.615 x lO"^”* Nm)


3. An electric dipole consists of two opposite charges
of magnitude 2 x 10"^ C each and separated by a
distance of 3 cm. It is placed in an electric field of
2x10^ NC~^. Determine the maximum torque on
the dipole. (Ans. 1.2 X 10"^ N m)
4. Two point charges of + 0.2 p pC and - 0.2 p pC are
separated by 10"^m. Determine the electric field at
an axial point at a distance of 0.1 m from their
midpoint. Use the standard value of Sq.
Fig. 1.71
(Ans, 3.6xl0"^NC“=)
5. Calculate the field due to an electric dipole of Clearly,
length 10 cm and consisting of charges of ± 100 pC (^2 ^2)1/2^ 20cm = 0.20 m
at a point 20 cm from each charge.
1 P
(Ans. 1125xlO^NC“=) .■qua
4ii8q ' (r^ +
HINTS
_ 9x10^ xlO"^ = - xlO^
I. T = sin 0 " (Oip 8

= 4x10"^ X 10"'^ X sin 30° = 1.125x10'^ NC"V


= 2x10"® Nm.

2. Here q = e = 1.602 x 10
-19
C, 2fl = 4xl0
-10
m. 1.28 ELECTRIC FIELD LINES
£ = 3x10^ NC"’, 0 = 30° 43. What are electric lines of force ? Give their
-19 -10
important properties.
p = q x2a = 1602 x 10 x4xl0
Electric lines of force. Michael Faraday (1791-1867)
= 6.41x10"^^ Cm. introduced the concept of lines of force to visualize
the nature of electric (and magnetic) fields. A small
X = pE sin 0
positive charge placed in an electric field experiences
= 6.41 X10"^^ X 3 X10^ xsin 30° a force in a definite direction and if it is free to move,
-24
= 9.615x10 Nm. it will start moving in that direction. The path
along which this charge would move will be a line of
3. Here q = 2x 10"^ C, 2« = 3 cm = 3 x 10“^ m, force.
£=2x10^ NC"’
An electric line of force may be defined as the curve
T
max = p £ sin 90° = q x2a X Exl along which a small positive charge would tend to move
= 2x10
,-6
x3xl0'^ x2xl0^ when free to do so in an electric field and the tangent to
which at any point gives the direction of the electric field at
= 1.2x10"^ Nm.
that point.
ELECTRIC CHARGES AND FIELD 1.45

In Fig. 1.72/ the curve PQR is an electric line of


force. The tangent drawn to this curve at the point P
gives the direction of the field Ep at the point P.
Similarly, the tangent at the point Q gives the direction i
P
of the field Eq at the point Q, and so on.
3q

Q
Fig. 1.73

5. The lines of force are always normal to the surface

w
E,, of a conductor on which the charges are in
equilibrium.
Reason. If the lines of force are not normal to the

Flo
P

conductor, the component of the field £ parallel

ee
to the surface would cause the electrons to move
Fig. 1.72 An electric line of force.
and would set up a current on the surface. But no

Fr
The lines of force do not really exist, they are current flows in the equilibrium condition.
imaginary curves. Yet the concept of lines of force is 6. The lines of force have a tendency to contract
very useful. Michael Faraday gave simple explana lengthwise. This explains attraction between two
for
ur
tions for many of his discoveries (in electricity and unlike charges.
magnetism) in terms of such lines of force. 7. The lines of force have a tendency to expand
laterally so as to exert a lateral pressure on neigh
ks

For Your Knowledge bouring lines of force. Tlais explains repulsion


Yo
oo

between two similar charges.


The lines of force are imaginary curves, but the field 8. The relative closeness of the lines of force gives a
eB

which they represent is real. measure of the strength of the electric field in any
The term ‘lines offeree' is misleading. It will be more region. The lines of force are
appropriate to call them electric (or magnetic) 'field (/) close together in a strong field.
r

lines'.
(ii) far apart in a weak field,
ou
ad

A field line is a space curve i.e., a curve in three


dimensions.
(j/j) parallel and equally spaced in a uniform field.
Y

9. The lines of force do not pass through a conductor


because the electric field inside a charged
nd

conductor is zero.
Properties of Electric Lines of Force i
Re

1. Tlie lines of force are continuous smooth curves 1.29 ELECTRIC FIELD LINES FOR DIFFERENT
Fi

without any breaks. CHARGED CONDUCTORS


2. The lines of force start at positive charges and end 44. Sketch and explain the field lines of (i) a positive
at negative charges - they cannot form closed point charge, (ii) a negative point charge, (Hi) two equal
loops. If there is a single charge, then the lines of opposite charges, (iv) two equal positive charges and
force will start or end at infinity. ^ positiveli/ charged plane conductor.
3. The tangent to a line of force at any point gives the Electric field lines for different charge systems :
direction of the electric field at that point.
(0 Field lines of a positive point charge. Fig. 1.74
4. No two lines of force can cross each other. shows the lines of force of an isolated positive point
charge. They are directed radially outwards because a
Reason. If they intersect, then there will be two small positive charge would be accelerated in the
tangents at the point of intersection (Fig. 1.73) and outward direction. They extend to infinity. The field is
hence two directions of the electric field at the
spherically symmetric i.e., it looks same in all
same point, which is not possible. directions, as seen from the point charge.
PHYSiCS-XII

\ /

Fig. 1.74 Field lines of a Fig. 1.75 Field lines of a


positive point charge. negative point charge.

Fig. 1.77 Field lines of two equal positive charges,


(ii) Field lines of a negative point charge. Like that
of a positive point charge, the electric field of a (u) Field lines of a positively charged plane
negative point charge is also spherically symmetric but conductor. Fig. 1.78 shows the pattern of lines of force
the lines of force point radially inwards as shown in of positively charged plane conductor. A small positive
Fig. 1.75. They start from infinity. charge would tend to move normally away from the
(in) Field lines of two equal and opposite point plane conductor. Thus the lines of force are parallel
charges. Fig. 1.76 shows the electric lines of force of an and normal to the surface of the conductor. They are
electric dipole i.e., a system of two equal and opposite
equispaced, indicating that electric field £ is uniform
point charges (± q) separated by a small distance. They
at all points near the plane conductor.
start from the positive charge and end on the negative
charge. The lines of force seem to contract lengthwise
as if the two charges are being pulled together. This
explains attraction between two unlike charges. The
field is cylindrically symmetric about the dipole axis
i.e., the field pattern is same in all planes passing -►

through the dipole axis. Clearly, the electric field at all


points on the equatorial line is parallel to the axis of the
dipole.
Fig. 1.78 Field pattern of a positively charged
plane conductor.

45. What is the relation betiveen the density of lines of


force and the electric ifeld strength ? Illustrate if in a
diagram.
Relation between electric field strength and density
of lines of force. Electric field strength is proportional
to the density of lines of force i.e., electric field strength
at a point is proportional to the number of lines of force
cutting a unit area element placed normal to the field at
that point. As illustrated in Fig. 1.79, the electric field at
Fig. 1.76 Field lines of an electric dipole. P is stronger than at Q.
Region of
weak field
(iv) Field lines of two equal and positive point
charges. Fig. 1.77 shows the lines of force of two equal
Q
and positive point charges. They seem to exert a lateral -F
pressure as if the two charges are being pushed away
from each other. This explains repulsion between two
—►

like charges. The field £ is zero at the middle point N -Region of


of the join of two charges. This point is called neutral strong field
point from which no line of force passes. This field also
has cylindrical symmetry. Fig. 1.79 Density of lines of force is proportional
to the electric field strength.
ELECTRIC CHARGES AND FIELD

d S = dS n
46. Show that the 1/r^ dependence of electric field of a
point charge is consistent with the concept of the electric Area = dS

field lines. 1
Consistency of the invoi’se square law with the \
electric field lines. As shown in Fig. 1.80, the number of
radial lines of force originating from a point charge q in
a given solid angle AQ is constant. Consider two points
Pj and ?2 at distances r^ and rj from the charge q. The («) (t)
same number of lines (say >i) cut an element of area
A £7 at Pj and an element of area A Q at P^. Fig. 1.81 (a) A planar area element, (b) An area
element of a curved surface.

In case of a curved surface, we can imagine it to be

w
divided into a large number of very small area
elements. Each small area element of the curved
surface can be treated as a planar area. By convention,

Flo
the direction of the vector associated with every area
element of a closed surface is along the outward drazun

ee
normal. As shown in Fig. 1.81(b), the area element rfSat

Fr
any point on the closed surface is equal to frS «, where
dS is the magnitude of the area element and « is a unit
for
ur
Number of lines of force cutting unit area vector in the direction of outward normal,
element at P,1 =
t?AQ 1.31 ELECTRIC FLUX
ks
1

48. Define the term electric flux. How is it related to


Yo

Number of lines of force cutting unit area element


oo

electric field intensity ? What is its SI imit ?


at P2 =
^ AQ Electric flux. The term flux implies some kind of
eB

As electric field strength cc Density of lines of force


flow. Flux is the property of any vector field. The
electric flux is a property of electric field.
E
^ AQ ^
r

1 _
The electric flux through a given area held inside an
ou
ad

£2 AD n
'1 electric field is the measure of the total number of electric
lines offeree passing normally through that area.
Y

i.e.,
Ecc^. As shown in Fig. 1.82, if an electric field t passes
nd

normally through an area element AS, then the electric


Re

1.30 AREA VECTOR flux through this area is


Fi

47. What is an area vector ? Hozv do we specify the = E AS


direction of a planar area vector ? How do we associate a
vector to the area of a curved surface ? Area = AS

Area vector. We come across many situations


where we need to know not only the magnitude of a ► H

surface area but also its direction. The direction of a £ £

planar area vector is specified by the normal to the plane. In


Fig. 1.81(a), a planar area element dS has been repre-
—♦ —^

sented by a normal vector dS . The length of vector dS


represents the magnitude dS of the area element. If n is Fig. 1.82 Electric flux through normal area.
a unit vector along the normal to the planar area, then
As shown in Fig. 1.83, if the normal drawn to the
dS = dS n area element AS makes an angle 0 with the uniform
1.48 PHYSICS-XII

->
Electric flux is a scalar quantity.
field £, then the component of £ normal to AS will be
£ cos 0, so that the electric flux is Unit of 4^ = Unit of E x unit of S
SI unit of electric flux
Ai^ = Normal component of £ x Surface area
= £ cos 0 X AS = NC“^m^=Nm^C‘V

or
A(j^ = £ AS cos 0 = E .AS Equivalently, SI unit of electric flux
= Vm''^m^= Vm.

1.32 GAUSS'S THEOREM


49. State and prove Gauss's theorem.
u
Gauss's theorem. This theorem gives a relationship
between the total flux passing through any closed
surface and the net charge enclosed within the surface.
Gauss theorem states that the total flux through a
closed surface is I/e^ times the net charge enclosed by the
closed surface.
Fig. 1.83 Flux through an inclined area. Mathematically, it can be expressed as

In case the field £ is non-uniform, we consider a 4^=1 £.d1


s
closed surface S lying inside the field, as shown in
Fig. 1.84. We can divide the surface S into small area Proof. For the sake of simplicity, we prove Gauss's
theorem for an isolated positive point charge q. As
elements : A^^, AS2, A^^. Let the corresponding shown in Fig. 1.85, suppose the surface S is a sphere of
electric fields at these elements be ^ , radius r centred on q. Then surface S is a Gaussian
Closed surface S
surface.
n

,9 dS
dS 1

<7 S
Fig. 1.84 Electric flux through a closed surface S.

Then the electric flux through the surface Swill be Spherical


Gaussian
-» ->
surface
4^ = £^. AS^ + £2 . AS2 + -I- £.,.
N
ASN

JV
= S f..AS: I I
Fig. 1.85 Flux through a sphere enclosing a point charge.
1 = 1

When the number of area elements becomes Electric field at any point on S is
infinitely large (N -> 00) and AS -> 0, the above sum 1 4
E =
approaches a surface integral taken over the closed 471 V
surface. Thus

N This field points radially outward at all points on S.


4t^ = lim Z £..' A§.' = <J1) £ . dS Also, any area element points radially outwards, so it
N-»co 1 = 1
AS-*0 is parallel to £, i.e., 0 =0°.

Thus the electric flux through any surface S, open Flux through area dS is
or closed, is equal to the surface integral of the electric -»

field £ taken over the surface S. d^ = E . dS = E dS cos 0° = EdS


ELECTRIC CHARGES AND FIELD 1.49

Total flux through surface S is a single fixed value at every point on the surface, we
can easily calculate the electric field of certain
4^ = (; =11 E dS = E <> dS
s s
symmetric charge distributions by applying Gauss's
theorem.
= £ X Total area of sphere
1.34 COULOMB'S LAW FROM
= -^.4.4.^
4jteg r
GAUSS'S THEOREM

51. Deduce Coulomb's law from Gauss's theorem.


or
■1^=- Deduction of Coulomb's law from Gauss's
^0
theorem. As shown in Fig. 1.86, consider an isolated
This proves Gauss's theorem. positive point charge q. We select a spherical surface S
of radius r centred at charge q as the Gaussian surface.

w
For Your Knowledge Spherical
.r—Gaussian
> Gauss's theorem is valid for a closed surface of any
\ surface

Flo
shape and for any general charge distribution.
> If the net charge enclosed by a closed surface is zero £

ee
*-(iS
(</ = 0), then flux through it is also zero.
'dS

Fr
= 0
^0
> The net flux through a closed surface due to a charge
lying outside the closed surface is zero.
for
ur
> The charge q appearing in the Gauss's theorem Fig. 1.86 Applying Gauss's theorem to a
includes the sum of all the charges located anywhere point charge.
inside the closed surface,
ks
Yo

> The electric field E appearing in Gauss's theorem is By symmetry, E has same magnitude at all points
●m
oo

due to all the charges, both inside and outside the


on S. Also £ and dS at any point on S are directed
closed surface. However, the charge q appearing in
eB

the theorem is only contained within the closed radially outward. Hence flux through area d^ is
surface.
d^^= E .dS = EdScosO° = EdS
r

> Gauss's theorem is based on the inverse square


ou

dependence on distance contained in the coulomb's


ad

Net flux through closed surface S is


law. In fact, it is applicable to any field obeying
4^ =|?.dS = u £dS= EirfS
Y

inverse square law. It will not hold in case of any


s s
departure from inverse square law.
= £X total surface area of S* Ex 47cr^
nd

> For a medium of absolute permittivity c or dielectric


Re

constant k, the Gauss's theorem can be expressed as Using Gauss's theorem.


Fi

^E.dS=^ = ^
e KE
0
^.1 £
0

1.33 GAUSSIAN SURFACE or £ X 4nr^ = —


50. What is a Gaussian surface ? Give its importance. ^0
Gaussian surface. Any hypothetical closed surface 1 ^
E =
enclosing a charge is called the Gaussian surface of that or

charge. It is chosen to evaluate the surface integral of 4K£g r


the electric field produced by the charge enclosed by it, The force on the point charge q^ if placed on surface
which, in turn, gives the total flux through the surface. S will be

Importance. By choosing a closed surface


F = qoE = 4nSn ?■
(Gaussian surface) in such a way that the electric field 0

£ has a normal component which is either zero or has This proves the Coulomb's law.
PHYSICS-XII

Examples based on Example 59. A cylinder is placed in a uniform electricifeld


Electric Flux and Gauss's Theorem E with its axis parallel to the field. Show that the total
electric flux through the cylinder is zero.
Formulae Used
Solution. The situation is shown in Fig. 1.87.
1. Electric flux through a plane surface area S held in
dS
a uniform electric field E is E

(j)£ = E . S = ES cos 0
where 0 is the angle which the normal to the ri—
dS yH-^dS
outward drawn normal to surface area S makes
—>

with the field E.


Fig. 1.87
2. According to Gauss's theorem, the total electric flux
through a closed surface S enclosing charge q is Flux through the entire cylinder,

’£ “ E.dS = -^ 'E.dS + [e.^ + fE.rfS


S curved
left plane right plane
Total flux _ face face surface
3. Flux density = S
Area
’ £ dS cos 180° + J £ dS cos 0°+ J £ dS cos 90°
Units Used -£ fdS+ E ffiS + 0
Electric flux (|>£ is in Nm^ and flux density in = - £ X ni^ + £ X = 0.
NC■^
Constant Used Example 60. Calculate the number of electric lines offorce
Permittivity constant of free space is originating from a charge of 1 C.
1 -12
Solution. Tlie number of lines of force originating
= 8.85x10
from a charge of 1 C
47Cx9x10"^
= Electric flux through a closed
surface enclosing a charge of 1 C
Examples?. //? =bi +3 / + 4: k , calculate the electric (I 1
= 1.129 X
- 12
flux through a surface of area 20 units in Y-Z plane. e
0
8.85 X 10
[Haryana 97}
—► A A A Example 61. A positive charge of 17.7 pC is placed at the
Solution. Electric field vector, E = 6 i + 3 j + 4 k centre of a hollow sphere of radius 0.5 m. Calculate the flux
density through the surface of the sphere.
As the area vector S in the Y-Z plane points along
Solution. From Gauss's theorem,
outward drawn normal i.e., along positive X-direction, so -6
17.7x10
s = 20 r Flux, -12
=2 x 10^ Nm-
8.85 X 10
Flux, (|^ = £ . S =(6 (■ +3/ +4fc ).20 )■ Total flux
Flux density =
= 120 units. Area

2x10^
Example 58. A circular plane sheet of radius 10 cm is = 6.4x 10^ NC~\
placed in a uniform electricfield of 5x10^ NC ^,makingan 47t (0.5)2
angle of 60"^ zvith the field. Calculate electricflux through the
sheet. Example 62. Calculate the electric flux through each of the
Solution. Here r = 10 cm = 0.1 m, £ = 5 x 10® NC
-1
six faces of a closed cube of length /, if a charge q is placed
(a) at its centre and (b) at one of its vertices.
As the angle between the plane sheet and the Solution, (fl) By symmetry, the flux through each of
electric field is 60°, angle made by the normal to the the six faces of the cube will be same when charge q is
plane sheet and the electric field is 0 =90° 60° =30° placed at its centre.
Flux, (f^ = £S cos 0 = £ X 71 X cos 0
= 5 X 10® X 3.14 X (0.1)2 ^ 3QO A -1 ±
^ 6'e
= 1.36x lO'* Nm2 Cr\ 0
ELECTRIC CHARGES AND FIELD 1.51

(b) When charge q is placed at one vertex, the flux (//) By Gauss's theorem, the total charge inside the
through each of the three faces meeting at this vertex cube is

will be zero, as r is parallel to these faces. As only 'I “ ^0 ‘fe “


1
X 1.05 = 9.27 X 10
-12
C.
47ux9x10
one-eighth of the flux emerging from the charge q
passes through the remaining three faces of the cube, Example 64. An electric field is uniform, and in the
so the flux through each such face is positive X direction for positive x and uniform with the same
J_ 3 magnitude in the negative x direction for negative x. It is
^ 3 8s 0 24 ■ 8 0 given that
->
-1
E = 200 i NC for X > 0
Example 63. The electric field components in Fig. 1.88 are
E^ = a = 0, in which a = 800 N / Cn^. Calcu¬
and £=-200 f NC~^for x<0.
late (i) the flux ^ through the cube and (ii) the charge within A right circular cylinder of length 20 cmand radius 5 cm

w
the cube. Assume that a =0.1 m
[NCERT] has its centre at the origin and its axis along the x-axis so
r that one face is of ;c = + 10 aiiand the other is at x = -10 cm.
(/) Wliat is the net outward flux through each flat face ?

Flo
(ii) Wlmt is the flux through the side of the cylinder ?
(Hi) What is the net outward flux through the cylinder ?

ee
a

(m) What is the net charge inside the a/linder ?

Fr
(CBSE 19C; NCERT]
■►X
.V
i.
a

for
ur
a
5 cm
a
2, /
ks
Fig. 1.88 £ £

Yo

Solution. (/) The electric field is acting only in


oo

X-direction and its Y-and Z-components are zero. For w- 20 cm -H


eB

AT = —10 cm A-=10 cm
the four non-shaded faces, the angle between E and
. —* —y Fig. 1.89
AS is + 7r/2. So flux (j)= E. AS is zero through each of
r

these faces.
Solution, (i) On the left face : £ =-200 i NC"^
ou
ad

The magnitude of the electric field at the left face is


AS = -ASi =-71(0.05)^ f m^
Y

1/2 1/2
= CLX- - = aa
[x = adi.\ the left face]

The outward flux through the left face is
nd

Flux, \ = E^ . AS = E^ AS cos 0 ^ ^ . AS
Re

= £^ j-og igQc = - = + 200 X 71 (0.05)2 I . t Nm2 C“^


Fi

[0 =180° for the left face] = +1.57 Nm^C"^ [/ ./ =1]


The magnitude of the electric field at the right face is On the right face :
1/2 1/2
£n
R
= ax = a(2a) -1
£ = 200 i NC
[x =2a at the right face]
Flux, ^ = E^ AS cos 0° = Ej^ fl2 AS = AS =7T (0.05)2 f ^2
[9 =0° for the right face] The outward flux through the right face is
Net flux through the cube
(j^ = £ . "aS =+1.57Nm2c“\
^=<^ + <^ = Ei^a^-E^a^
= n2(£^-£J = CUl2 [(277)1/2 _^l/2j (//) For any point on the side of the cylinder ^ 1 AS,

'-^/2[V2 -1] =800 (0.1)^/2


= aa
_ Flux through the side of the cylinder,
= 1.05 Nm^C"^ <^ = E . AS = £ AS cos 90° = 0.
1.52 PHYSICS-Xll

(Hi) Net outward flux through the cylinder, Example 66. Figure 1.91 shows five charged lumps of
plastic and an electricalhj neutral coin. The cross-section of a
(|)£ = 1.57 + 1.57 + 0 = 3.14 Nm^ C~\ Gaussian surface S is indicated. What is the net electric flux
(ill) By Gauss's theorem, the net charge inside the through the surface if
cylinder is =q^=+3.1n C, q^ = q^^-5.9 nC
-12 -11
Sy (|^ =8.854 X 10 X 3.14 = 2.78 X 10 c. and ^3 = -3.1 nC7
Example 65. Vow are given a charge + Qat the origin O
(Refer to Fig. 1.90). Consider a sphere S with centre (2, 0, 0)
of radius m. Consider another sphere of radius -J2 m

centered at the origin. Consider the spherical caps (/) PSQ


(ii) PRQ (Hi) PWQ, with normals outward to the respective
spheres, and (iv) the flat circle PTQ with normal along the
x-axis.
(a) What is the sign of electric flux through each of the
surfaces (i)-(iv) ? 43
0
(b) What is the relation between the magnitudes of
fluxes through surfaces (i)-(iv) ?
(c) Calcidate the flux through the surface (ii) directly. Fig. 1.91
Assume that the area of the cap (ii) is A. INCERT] Solution. The neutral coin and the outside charges
-y q^ and q^ make no contribution towards the net charge
enclosed by surface S. Applying Gauss's theorem, we
get
q +%
‘='0
SI IT \R W
+ 3.1 X 10'^ - 5.9 X10'^ -3.1 X10”^
(2,0,0) 12
8.85x10

= -666.67 Nrn^C”^.
Q
Example 67. Sj and $2 are two concentric spheres enclosing
Fig. 1.90 charges Q and 2Q respectively as shown in Fig. 1.92.
Solution. For the charge + Q situated at origin O, the (i) Wliat is the ratio of the
electric flux through Sj
field ? points along+vex-direction/.e., towards right, and $2 ?
(fl) Tlie outward drawn normal on cap PSQ points (ii) How will the electric
towards left while it points towards right for
flux through the sphere
caps PRQ, PWQ and circle PTQ. So the flux is Sj change, if a medium
negative for (i) and positive for the rest. of dielectric constant k is
(b) The same electric field lines crossing (/) also introduced in the space
cross (ii), (in). Also, by Gauss's law, the fluxes
inside Sj in place of air ?
through (Hi) and (iv) add upto zero. Hence, all
magnitudes of fluxes are equal, (Hi) How will the electric flux through sphere change,
(c) Given area of the cap (ii) = A if a medium of dielectric constant k is introduced in
the space inside S2 in place of air ?
Electric field through cap (ii) is [CBSE OD 02,14,14C1
Q
£ =
1
5=9x10^x
-
Solution, (i) By Gauss's theorem.
47ueQ r
-1
= 4.5 X 10^ Q NC Flux through Sj is =—
^0
Electric flux through the cap (11) is
2Q+Q_3Q
^ = EA Flux through $2 is <t^ =
^0 So
= 4.5x 10^ QA NC"^ml
ELECTRIC CHARGES AND FIELD L53

Ratio of electric flux through Sj and Sj is 7. A charge q is situated at the centre of an imaginary
hemispherical surface, as shown in Fig. 1.93. Using
-1:3 Gauss’s theorem and symmetry considerations, deter
4^ 3Q/£g 3 mine the electric flux due to this charge through the
(ii) If a medium of dielectric constant k is intro hemispherical surface. ^ q
duced in the space inside S,, then flux through Sj 2^0
becomes

{● —» —* r E 1
£.dS=l.Q

^ = iE'.dS = i — .dS =K K K
^0

(in) The flux through does not change with the


introduction of dielectric medium inside the sphere $2-
Fig. 1.93

w
roblems For Practice 8. A hollow cylindrical box of length 1 m and area of
cross-section 25 cm^ is placed in a three dimen
sional coordinate system as shown in Fig. 1.94. The

Flo
1. If the electric field is given by A

electric field in the region is given by £ = 50x/,


E=8/ + 4/ + 3fc NC"’, calculate the electric flux where £ is in NC~' and .v is in metres.

ee
through a surface of area 100 m^ lying in the X-V y
(Ans. 300Nm^C"^)

Fr
plane.
1 m

2. The electric field in a certain region of space is A


(5 'i + 4 / - 4 ) X 10^ NC
-1
. Calculate electric flux O
for
ur
1 m

due to this field over an area of(2 / - j ) x 10” ^ m^. Z,

(Ans. 6x10^ Nrn^C”^)


ks
Fig. 1.94
Yo

Find
3. Consider a uniform electric field £ =3x10^/ NC”^
oo

Calculate the flux of this field through a square (/) net flux through the cylinder,
eB

surface of area lOcm^ when (i7) charge enclosed by the cylinder. ICBSE D 131
(i) its plane is parallel to the y-z plane, and
9, The electric field in a region is given by £ = ●
i .

(ii) the normal to its plane makes a 60° angle with


r
ou

the x-axis. ICBSE D 13C] Find the charge contained in the cubical volume
ad

[Ans. (i) 30Nm^"' (ii) 15Nm^’^] bounded by the surfaces x = 0, x =a, y = 0,


Y

y = fl, 2 = 0 and z=a. Take = 5 x lO"'’NC”’,


4. Given a uniform electric field E =5xlCPiNC \ <7 = 1cm and h = 2cm. (Ans. 2.2xl0‘’^ C)
find the flux of this field through a square of 10 cm
nd

10. The electric field components due to a charge inside


Re

on a side whose plane is parallel to the Y-2 plane. the cube of side 0.1 m are as shown.
What would be the flux through the same square if
Fi

£^ = ax, where a - 400 N / C- m


the plane makes a 30° angle with the X-axis ?
ICBSE D 14]
^ = 0, ^ = 0.
[Ans. (0 50 Nrn^C”' (ii) 25 Nm^”^] y

5. A point charge of 17.7 pC is located at the centre of a


cube of side 0.03 m. Find the electric flux through
each face of the cube. [Himachal 93]

(Ans. 3.3x10^ Nm^C”^)


6. A spherical Gaussian surface encloses a charge of
-8
8.85x10 C. (i) Calculate the electric flux passing
through the surface. (//) If the radius of the
Gaussian surface is doubled, how would the flux
0.1 m
change ? [CBSE D 01, F 071 0,1 m

[Ans. (i) 10'* Nm^ C“* (ii) No change]


Fig. 1.95
PHYSICS-XII

Calculate (i) the flux through the cube, and (ii) the 8. (0 Flux through the curved surface of the cylinder
is zero.
charge inside the cube. [CBSE 18C ; SP 20]

[Ans. (0 ([)£ = 0.1 (ii) q = 8.854 xl0“^^ C] Magnitude of the electric field at the left face,
-1
E= 50xl = 50NC
n. A uniform electric field E = E^. i N / C for .v > 0 and
—>
(|)l= EScose = 50x25xl0’^cosl80°
E = / N / C for X < 0 are given. A right circular = -1250xl0“^Nm^“^
cylinder of length I cm and radius r cm has its centre
Magnitude of the electric field at the right face,
at the origin and its axis along the x-axis. Find out -I
E=50x2=100NC
the net outward flux. Using Gauss's law write the
expression for the net charge within the cylinder. (j)^ = 100x25xl0~^cos0°
ICBSE D 08C1
= 2500xlO-^Nm^“^
HINTS
= 4)l+ =(2500-1250)xl0“*Nm^
-1

1. E =8i+4j + 3k NC~\ S =mk = 1250xl0~^Nm^“


1

= 1.250x10"^ Nm^C"V
Flux, ^p = E.S = {8i+4j+3k).W0k
= 300 Nm^ {ii) Total charge enclosed by the cylinder,
q = 8q(|)£ = 8.854 x 10“^^ x 1250 x lO^^C
2. = E , S =(5i + 4/ - 4fc ) X10^ . (2i - / ) x 10“^ = 11067.5xl0"^'^C =1.107 pC.
= [5x2 + 4x{^l)-0] xlO^ Nm^C“*. 9. 4)£ = 4'i. +
= 6x10^ Nm^ C"\
.2
= - E,L + Ef,a^
K
=~ ^
+ . a

3. (/) Normal to the area points in the direction of b


the electric field, 0 = 0°.
E;, _5xl0^ x(0.0lf = 0.25Nm^C"'.
({)£= EScos0 = 3xlO^ x(0.10)^cos0° b~~ i
0.02
= 30 Nm^C"^ -12
X 0.25 = 2.2x10
-12
c.
ty = £y (j)g = 8.85 X 10
(”) = 3x10^ x(O.lO)^ xcos 60° = ISNm^C"’. YA

4. (/) ({)£ = £Scos9


= 5 X10^ x(0.10)^cos 0°= 50 Nm^C‘\
(/■/■) ({)£ =5x10^ x(0.10f cos(90°-30°) /

= 5x10^ x(0.10)^x^ = 25 Nm^C^’. £

5. Flux through each phase of the cube


-6
o
-1a _ 1 ^ 12.7x10-12 a

So “6^8.85x10 a

= 3.3x10^ Nm^ C"\


Z
8.85 X 10'® C
6. (0't>£=-^ = 8.85x10“’^ N~‘ m
-2
Fig. 1.96

10. 4'f = <1>R + <t'L


= 10“' Nm^ C"^
= [Eyfat.r - 2a)- £^(atx - a)] a~
(ii) (j)g = lO'^Nm^ C“S because the charge enclosed = [a(2fl)-a{fl)]
is the same as in the case (i). = an ® =400x(0.1)® =0.4 Nm^C"^
7. From Gauss's theorem, total flux through entire -12
x0.4= 3.542 xl0"^2 Q
q - ejj<j)£ = 8.854 xlO
spherical surface is
11. Proceed as in Example 64 on page 1.51.

From symmetry considerations, flux through the (i) ^,- = E^.k —f +


,100,
£.. Jt
r

vioo;
+ 0

hemivSpherical surface is
1 = 2Trr^E^(10r^ Nm^'\
'E =
2 E
0 (//) (y = e„£ = 27ir^8QE^(10)“‘* C.
ELECTRIC CHARGES AND FIELD

I
1.35 FIELD DUE TO AN INFINITELY
A

Vectorially, n
2 71
LONG CHARGED WIRE 0

52. Apply Gauss s theorem to calculate the electric y/here n is the radial unit vector in plane normal to the
field of a thin infinitely lon^ straight line of charge, with a . . ,
■r , j f -1 -1 Wire passing through the observation point.
uniform charge density of Cm . & r r
Electric field due to an infinitely long straight charged 136 ELECTRIC FIELD DUE TO A UNIFORMLY
wire. Consider a thin infinitely long strai^t wire having CHARGED INFINITE PLANE SHEET
a uniform linear charge density X Cm” . By symmetry. 53. Apply Gauss's theorem to calculate the electric

the field E of the line charge is directed radially outwards field due to an infinite plane sheet of charge.
and its magnitude is same at all points equidistant Electric field due to a uniformly charged infinite
from the line charge. To determine the field at a plane sheet. As shown in Fig. 1.98, consider a thin,
distance rfrom the line charge, we choose a cylindrical infinite plane sheet of charge with uniform surface

w
Gaussian surface of radius r, length / and with its axis charge density a. We wish to calculate its electric field
along the line charge. As shown in Fig. 1.97(a), it has at a point P at distance r from it.
curved surface Sj and flat circular ends S2 and S^.

Flo
Obviously, || ^, -L £ and ^^3 ± £ . So only the
Plane sheet,
●» + *
e ^ ^ charge density a

*///****
curved surface contributes towards the total flux. +

ee
+

E ( \
■fir 7T\ E
4 ■. +
+

Fr
1^90'
\1J \J
●►E

, 4

for r
ur
Cross-sectional t -ij.
area A
ks
/ S, C3—^ -►£ Fig. 1.98 Gaussian surface for a uniforinly
charged infinite plane sheet.
Yo
dS,
oo

E
r
By symmetry, electric field £ points outwards
normal to the sheet. Also, it must have same magni
eB

tude and opposite direction at two points P and F


L S3
●^90°/ ■E
equidistant from the sheet and on opposite sides. We
choose cylindrical Gaussian surface of cross- sectional
r
ou

area A and length 2r with its axis perpendicular to the


ad

H- ■« r
(a) (b) sheet.
Y

Fig. 1.97 (a) Cylindrical Gaussian surface for line charge.


As the lines of force are parallel to the curved
(b) Graph off vs. r.
surface of the cylinder, the flux through the curved
nd

surface is zero. The flux through the plane-end faces of


Re

=1^. d? 1
+
^.d\ + jE.ciS^ the cylinder is
Fi

s s1 S2 S3
^ = EA+ EA=2 EA
EdS^ cos 0° + EdS^ cos 90°-f- EdS^ cos 90° Charge enclosed by the Gaussian surface, q = <sA
Si S2 S3 According to Gauss's theorem.
= £ dSj + 0 + 0
4t=-
= £X area of the curved surface or ^ = Ex2nrl ^0
Charge enclosed by the Gaussian surface, q = Xl a A a
2 £A = or £ =
Using Gauss's theorem, ({^ = ql Sq , we get ^0 2e
0
XI X a A
or E.2nrl = — or £ =
2 71 e„r Vectorially, E = n
0 2e
0

Thus the electric field of a line charge is inversely propor


tional to the distance from the line charge. Figure 1.97(b) shows where n is the unit vector normal to the plane and
the variation of £ with distance r from the line charge. going away from it.
1.56 PHYSICS-Xll

Clearly, £is independent of r, the distance from the 55. Two infijnte parallel planes have uniform charge
plane sheet, densities ± a. Determine the electric field in (i) the region
(i) If the sheet is positively charged (a >0), the field between the planes, and (ii) outside it.
is directed away from it. Electric field of two oppositely charged plane
(ii) If the sheet is negatively charged (a <0), the field parallel plates. As shown in Fig. 1.100, consider two
is directed towards it. plane parallel sheets having uniform surface charge
For a finite large planar sheet, the above formula densities of ± o. Suppose r be a unit vector pointing
will be approximately valid in the middle regions of from left to right, + a - a
the sheet, away from its edges.
54. Two infinite parallel planes have uniform charge
densities of and <J2- Determine the electric field at I +
n III
points (i) to the left of the sheets, {ii) between them, and +

{Hi) to the right of the sheets. ●1


£2 E,
Electric field of two positively charged parallel
plates. Fig. 1.99 shows two tlnin plane parallel sheets of
charge having uniform charge densities and ©2 t r
A
Fig. 1.100
with Oj > 02 >0- Suppose r is a unit vector pointing Sheet 1 Sheet 2

from left to right. In the region I: Fields due to the two sheets are
A A
G a

'h.
1
r
£,
£i=- 26
+ +
n, o
+ + 2e 0
0
♦ +
A
I u + III

+ Total field, £,-%


I 1
+ £.,=
2 2 P
o + — 0=0
2e
+ 4 0
£2 £2 £2
In the region II: Fields due to the two sheets are
*

■f

+
r
r £1 = 2e cr,
2e 0
o

Sheet 1 Sheet 2 0
A

r r o
Fig. 1.99 Total field. % = 2e o +
2e 0
o = — r

0 ^0
In the region I : Fields due to the two sheets are
A o
In the region III : Fields due to the two sheets are
£1 ~^r.
26
£2=“ 26^r A
0 0
£,1 =-- £ =- —
From the principle of superposition, the total 2e
<y / o

electric field at any point of region / is 0

Total field, £II! = 0.


A

— (o^ + O2)
£^ = E| + £2 = ~ 26
0 Thus the electric field between two oppositely
In the region II: Fields due to the two sheets are charged plates of equal charge density is uniform
which is equal to — and is directed from the positive to
E=^P,
I
2e
^2=--^P
^ 2Sn
^0
0 0
the negative plate, while the field is zero on the outside
of the two sheets. This arrangement is used for
A

.●. Total field, £n ~ — -<^2) producing uniform electric ifeld.


2e 0

In the region III; Fields due to the two sheets are 2.37 FIELD DUE TO A UNIFORMLY
-> a G CHARGED THIN SPHERICAL SHELL
£
1
2.;
2e
2s
0 0 56. Apply Gauss's theorem to show that for a
A
spherical shell, the electric ifeld inside the shell vanishes,
Total field, £ HI — (Oj + 02) whereas outside it, the field is as if all the charge had been
2e 0
concentrated at the centre.
ELECTRIC CHARGES AND FIELD 1.57

Electric field due to a uniformly charged thin or [For r= R]


spherical shell. Consider a thin spherical shell of 4TI £r^R“
0

charge of radius R with uniform surface charge density o


or £ = — [●/ q - 47tR^ a]
a. From symmetry, we see that the electric field E at %
any point is radial and has same magnitude at points (c) Wlten point P lies inside the spherical shell. As is
equidistant from the centre of the shell i.e., the field is clear from Fig. 1.102(a), the charge enclosed by the
spherically symmetric. To determine electric field at any Gaussian surface is zero, i.e.,
point P at a distance r from O, we choose a concentric
sphere of radius r as the Gaussian surface.
Gaussian
£
surface
Gaussian
surface

w
●£
\
/ \
\
I
I

Flo
1
I
Q. r
1 J_
E<- T £ = Spherical shell,
'P 4 7t8o ^
\ R I
I
charge density = a

ee
\
/
s

—7^ Spherical shell, Fig. 1.102 (a) Gaussian surface for inside points

Fr
S
*' charge density = ct of a thin spherical shell of charge.

Flux through the Gaussian surface,


<1^ = £ X 4jrr^
for
ur
£

Fig. 1.101 Gaussian surface for outside points of Applying Gauss's theorem.
a thin spherical shell of charge.
ks
^0
Yo

(a) When point P lies outside the spherical shell. The


oo

£ X 4nr^ - 0
total charge q inside the Gaussian surface is the charge
on the shell of radius R and area 47cR^. £ =0 [For r < K]
eB

or

q = 4;tR^ a Hence electric field due to a uniformly charged spherical


Flux through the Gaussian surface, = Ex shell is zero at all points wside the shell.
r

By Gauss's theorem. Figure 1.102(1?) shows how £ varies with distance r


ou
ad

from the centre of the shell of radius R. E is zero from


●e -
Y

% r = 0 to r = R ; and beyond r = R, we have

Ex4ki^ = 5.
nd
Re

^0

E = -J— 1 £
Fi

or [For r> R]
4rt £n0 r

q A
Vectorially, ? —

Anr^

This field is the same as that produced by a charge q


placed at the centre O. Hence/or points outside the shell,
the field due to a uniformly charged shell is as if the entire
charge of the shell is concentrated at its centre,
0 R
(b) When point P lies on the spherical shell. The Gaussian r

surface just encloses the charged spherical shell.


Applying Gauss's theorem,
£x4nR^ = '^ Fig. 1.102 (b) Variation of E with r for a
^0 spherical shell of charge.
s 1.58

1.38 FIELD DUE TO A UNIFORMLY


CHARGED INSULATING SPHERE*
PHYSICS-XII

This field is same as that produced by a charge q


placed at the centre O. Hence for points outside the sphere
the field due to the uniformly charged sphere is as if the
57. .4 charge q is uniformly distributed within an entire charge of the sphere is concentrated at its centre,
insulating sphere of radius R. Apply Gauss's theorem to (b) Wlten the point P lies on the sphere. The Gaussian
find the electric field due to this charge distribution at a surface just encloses the charged sphere. Applying
point distant rfrom the centre of the sphere, where (a) r> R Gauss's theorem,
(b) r= R (c) 0 <r< R. Show the variation of E with r Ex4kR'^ = -^
graphically. E
0

Electric field due to uniformly charged insulating


sphere. Consider an insulating sphere (For example, a or [For r= R]
4tie„R^
nucleus with protons almost uniformly distributed 0

inside it) of radius A with uniform volume charge (c) When point P lies inside the sphere. The charge
density p. From symmetry, we see that the electric field enclosed by the Gaussian surface is
£ at any point is radial and has same magnitude at
points equidistant from the centre O of the sphere i.e., (f'3^3
= —Tcr^p = —Tcr^x 4_, R^
the field is spherically symmetric. To determine electric kR^
3
field at any point P at distance r from O, we choose a E
concentric sphere of radius r as the Gaussian surface,
(rt) When point P lies outside the sphere. The total
charge q inside the Gaussian surface is the charge
inside the sphere of radius R. E E

‘1 = -kR^P
3

Flux through the Gaussian surface, £

(j)g = Ex Fig. 1.103 (b) Gaussian surface for inside points


£ of an insulating sphere of charge.

By Gauss's theorem,
I \
£x 4jir^ =
/
r
^0
1
E
,'P
Ex 4nr^ = qr'
\ /
or

or g _ 1 /ir _ pr V q = -ttR^P
E 4tc£.0 R^ 3e,0 3

Fig. 1.103 (a) Gaussian surface for outside points At the centre, r =0 and hence £ =0.The variation of
of an insulating sphere of charge. £ with distance r from the centre of a sphere of uni
By Gauss's theorem. formly charged sphere is shown in the figure 1.103(c).
£
’£ “
^0 1 ^
£ mnx
4 TtEo
Ex 4717-2=-^
^0 1
£x —
r
or [For r> R]
4k8o r O r=R r

Vectorially, ? =—-— Fig. 1.103 (c) Variation of E with r for a


4n
uniformly charged sphere.
ELECTRIC CHARGES AND FIELD 1.59

Examples based on Example 68. Two long straight parallel wires carry
Applications of Gauss's Theorem charges and length. The separation between
their axes is d. Find the magnitude of the force exerted on
Formulae Used unit length of one due to the charge on the other.
1. Electric field of a long straight wire of uniform Solution. Electric field at the location of wire 2 due
linear charge density X, to charge on 1 is
X X]
E = £ =
2nSg r 2n£r.0 d

where r is the perpendicular distance of the Force per unit length of wire 2 due to the above
observation point from the wire. field
2. Electric field of an infinite plane sheet of uniform
/ = £ X charge on unit length of wire 2 = EX^
surface charge density a.

w
X.jX.2
or
/=
2 So 2rce.^0 d

3. Electric field of two positively charged parallel Example 69. An electric dipole consists of charges

Flo
plates with charge densities Oj and Oj such that ±2 X 10"^C separated by a distance of 2 mm. It is placed
> O2 > Q, near a long line charge of density 4.0x 10“'^ On~^, as shown

ee
1
£-± (CTj + Cj) (Outside the plates) in Fig. 1.104, such that the negative charge is at a distance of
2e 0 2 an from the line charge. Calcidate the force acting on the

Fr
1 dipole.
£= (Ci -CJ2) (Inside the plates)
2^0 +

for
ur
4. Electric field of two equally and oppositely
charged parallel plates, +

£=0 (For outside points)


ks
+

(For inside points)


Yo
+ -<7 + <?
oo

— 2 cm 2 mm—
5. Electric field of a thin spherical shell of charge +
eB

density o and radius R, +


1
1
E = For r>R (Outside points) +
4tc Sa0
r
ou

E= 0 For r<R (Inside points)


ad

E = For r=R (At the surface) Fig. 1.104


Y

4n€g R^
Solution. Electric field due to a line charge at
Here q = 4n R^ a. distance r from it.
nd
Re

6. Electric field of a solid sphere of uniform charge £ =


1 2X
density p and radius R : 47CE,,0 r
Fi

E = For r> R (Outside points) Force exerted by this field on charge q,


4ti r^
0
1 2qX
1 F = qE =
E = For r<R (Inside points) 4t:£q r
47t 0
R^
Force exerted on negative charge (r =0.02 m),
1
1 9xl0^x2x2xl0"®x4xl0
-4
£ = For r = R (At the surface)
4jc8.0 ■ R^ ^1 = N
0.02
Here ? = = 7.2 N, acting towards the line cliarge
Force exerted on positive charge (r = 2.2 x 10 m),
Units Used
9 X 10^ X 2 X 2 X 10"® X 4 X 10“^
Here charges are in coulomb, r and R in metre, X in ^2 =
Cm , a in Cm p in Cm and electric field £ in 2.2 X 10"^
NC"' or Vm"’.
= 6.5 N, acting away from the line charge
1.60 PHYSICS-XII

Net force on the dipole, Example 72. A charge of 17.7 C is distributed


f = f, -f2=7.2 -6.5 uniformly over a large sheet of area 200 tr?. Calculate the
electric field intensity at a distance of 20 anfrom it in air.
= 0.7 N, acting towards the line charge. ICB5E OD 03C)
Example 70. (a) An infinitely long positively charged wire Solution. Surface charge density of the sheet,
has a linear charge density XCnf^. An electron is revolving
around the wire as its centre with a constant velocity in a 7 _ 17.7 xlO^'* C = 8.85 X 10"*^ Cm"^
a = —
A 200
circular plane perpendicular to the wire. Deduce the expre
ssion for its kinetic energy, (b) Plot a graph of the kinetic Electric field at a distance of 20 cm from it in air,
energy as a function of charge density X. [CBSE F 13]
a 8.85 X 10"^
£ = = 5xlO^NC"\
Solution. The electrostatic force exerted by the line 2e 2x8.85x10
-12
0
charge on the electron provides the centripetal force
for the revolution of electron. Example 73. Two large, thin metal plates are parallel and
.'. Force exerted by electric field = Centripetal force close to each other. On their inner faces, the plates have
mv
2
surface charge densities of opposite signs and of magnitude
eE =
r
17.7x 10'"^C/ What is electric field intensity E
(a) in the outer region of the first plate, and
Here v is the orbital velocity of the electron
X {b) between the plates ? (CBSE SP 23]
But £ =
2tc«
0
r Solution, (a) In the outer region of the first plate,
2 the fields due to the two plates are equal and opposite.
eX mv 2 eX
or V So, £ = 0.
luEr.0 r r iKSn0 m
(fc) Electric field in the region between the two
Kinetic energy of the electron will be plates.
C 1 2
El, = — mv =
eX . a 17.7x10
-22
-10
2 4tis 0 £= —
-12
= 2.0 X 10 NC"\
^0 8.85x10

(b) As Ef. X X, the graph of


Example 74. A large plane sheet of charge having surface
kinetic energy Ef. vs. charge
density X.will be a straight line
charge density 5.0 x 10' Cm" ^ lies in the X-Y plane. Find
the electric flux through a circular area of radius 0.1 m, if the
as shown in Fig. 1.105.
Fig. 1.105 yiormal to the circular area makes an angle of 60° zvith the
Z-axis.
Example 71. A long cylindrical volume contains a -12
imiformly distributed charge of density p. The radius of Given that: ^0 = 8.85x10

cylindrical volume is R. A charge particle (q) revolves around Solution. Here a = 5.0x10
16
Cm“^ r = 0.1 m.
the cylinder in a circular path. Find the kinetic energy of the
0=60°
particle. (JEE Main June 22]

Solution. Charge per unit length of the cylinder. Field due to a plane sheet of charge.
a
£ =
2
2e
I I ^ 0

Flux through circular area,


£ =
X _ pnR^ _ pR^
2ner.r 2nSr.r 2s«r
0 0 0 4^ = £AS cos 0 - X cos 0
2
mu
qE = 5.0x10
-16
X 3.14 X (0.1)^ cos 60°
r
●f + + -12
2
V
2 X 8.85 X 10
mv I- i-

2e,.r
0
r
●( T
= 4.44X 10'^ Nm^C'\
+ + +
im, q
Example 75. A spherical conductor of radius 12 cm has
I + H

mv
2 a
charge of 1.6xl0'^C distributed uniformly over its
+ + +
2e
0 + + +

surface. What is the electric field (i) inside the sphere,


(ii) just outside the sphere, (Hi) at a point 18 cm from the
2 4e 0
centre of the sphere ? [NCERT]
ELECTRIC CHARGES AND FIELD L6J

Solution. Here ty = 1.6 x 10 ^ C 7. A spherical shell of metal has a radius of 0.25 m and
carries a charge of 0.2 pC. Calculate the electric field
1^=12 cm =0.12 m
intensity at a point (i) inside the shell, (jj) just
(/) Inside the sphere, E = 0. This is because the charge outside the shell and {Hi) 3.0 m from the centre of
resides on the outer surface of the spherical conductor. the shell. [Ans. (i) 0 (ii) 2.88 x 10^ NC ^
(//) Just outside the sphere, r = K =0.12 m. Here the (m) 200 NC'^j
charge may be assumed to be concentrated at the HINTS
centre of the sphere. Er 9 X10“^ X 0.04
1. X. = 2n£Q Er = 47t8Q— 9 X lO’ X 2
E =
471 Eq = 2x10'^ Cm“^
9x10% 1.6x10'^ -1
2. Here ?^ = 2xlO"®Cm , r = 0.2 m
(0.12)^ 2x2x10'®

w
%. ^ = 9 X 10^ X
= 10® NC"\ 271Sq r 47tE(j r 0.2

{Hi) At a pomt 18 cm from the centre, = 1800 Vm'^

Flo
r = 18 cm =0.18 m.
I 2X 9xl0%2xl0'“
3. £ = = 4.5 xlO® Vm'^
q 9x10% 1.6x10'^ 47iEq r 4

ee
47teQ r^ (0.18)^ 4.

Fr
= 4.44 X 10^ NC'\ ^0 E(, AS
ly = £o AS E = 8.85 x 10“^^ x 1 x 100
7^. roblems For Practice = 8.85 xlO -10
for c
ur
5. From Example 70,
], An infinite line charge produces a field of eX
9xl0‘*NC'^ at a distance of 4 cm. Calculate the = 9 X10^ X 1.6 X10'^ X 2.0 X10'®
47ie,0
ks
linear charge density. (Ans. 2x10'^ Cm'^)
Yo

= 2.88xlO'^^J.
2. A cylinder of large length carries a charge of
oo

2 X 10'®Cm'^ Find the electric field at a distance of 6. Upward electric force on particle
eB

1
0.2 m from it. (An?. 1800 Vm"‘) = Weight of the particle
3. An infinitely long wire is stretched horizontally mg = qE=q.^
4 metre above the surface of the earth. It carries a 2eq
r

charge IpC per cm of its length. Calculate its


ou
ad

or

electric field at a point on the earth's surface CT


Y

vertically below the wire. (Ans. 4.5x10®Vm 2x8.85x10


-12
X 5 X 10'^ X 9.8
-6
4. Two large metal plates each of area 1 m^ are placed 4x10
nd
Re

facing each other at a distance of 10 cm and carry = 2.16x10


-13
c.
equal and opposite charges on their faces. If the Number of electrons required to be removed,
Fi

electric field between the plates is 100 NC'\ find -13


^ _ 2.16x10
the charge on each plate. (Ans. 8.85x10-'° C) M = —
-19
= 1.355x10®.
e 1.6x10
5. An electron is revolving around a long line charge
having charge density 2xl0'®Cm'^ Find the kinetic 7, (/) Electric field at any point inside the shell = 0.
energy of the electron, assuming that it is (n) E=
independent of the radius of electron's orbit. 47reg Fr
(Ans, 2.88 X10“*^J) 9xlo’x0.2xl0'®
= 2.88x10^ NC
6. A particle of mass 5xl0'^g is kept over a large (0.25)^
horizontal sheet of charge density 4x10 Cm .
What charge should be given to this particle, so that (m)£ = 7^.4 4ne,^0
if released, it does not fall down. How many
9x10% 0.2x10'®
electrons should be removed to give this charge ? = 200 NC'^
(Ans. 2.16xl0'^®C 1.355x10®) (3.0)2
PHYSICS-XIl

Very Short Answer Conceptual Problems

Problem 1 The electric charge of any body is actually farther end of the paper. The rod exerts greater attraction
a surplus or deficit of electrons. Why not protons ? than repulsion on the paper because negative charge is
Solution. Electrons are loosely bound to atoms and positive charge. Hence the rod
can be readily exchanged during rubbing. Protons are attracts the piece of paper.
firmly bound inside the nucleus. They cannot be easily Problem 8. Can two like charges attract each other ? If
detached. Hence electric charge of any body is just a yes, how ?
surplus or deficit of electrons and not protons. Solution. Yes. If one charge is larger than the other, the
Problem 2. When a glass rod is rubbed with silk, charge induces equal and opposite charge on the
both acquire charges. What is the source of their electri nearer end of the body with smaller charge. The opposite
fication ? induced charge is larger than the small charge initially
present on it.
Solution. For the electrification of a body, only
Problem 9. Why do the gramophone records get
electrons are responsible. During rubbing electrons are
covered with dust easily ?
transferred from glass rod to silk. The glass rod acquires a
positive charge and silk acquires an equal negative charge. Solution. The gramophone records get charged due to
the rubbing action of the needle. So they attract the dust
Problem 3. Is the mass of a body affected on charging ? particles from the air.
Solution. Yes. Electrons have a definite mass. The
Problem 10. An ebonite rod held in hand can be
mass of a body slightly increases if it gains electrons while charged by rubbing with flannel but a copper rod
the mass decreases if the body loses electrons. cannot be charged like this. Why ? (Himachal 97]
Problem 4. Two identical metallic spheres of exactly
Solution. Ebonite rod is insulating. Whatever charge
equal masses are taken. One is given a positive charge q
appears on it due to rubbing, stays on it. Copper is good
coulombs and other an equal negative charge. Are their
conductor. Any charge developed on it flows to the earth
masses after charging equal ? [IITI
through our body. So copper rod cannot be charged like this.
Solution. No. The positive charge of a body is due to It can be charged by providing it a plastic or rubber handle.
deficit of electrons while the negative charge is due to
Problem 11. Electrostatic experiments do not work
surplus of electrons. Hence the mass of the negatively well on humid days. Give reason.
charged sphere will be slightly more than that of the
positively charged spheres. Solution. Electrostatic experiments require accumu
lation of charges. Whatever charges appear during the
Problem 5. A positively charged rod repels a sus
pended object. Can we conclude that the object is posi experimentation, they are drained away through humid
tively charged ? air which is more conducting than dry air due to the
presence of a larger number of charged particles in it.
Solution. Yes, the object is positively charged.
Repulsion is the surest test of electrification. Problem 12. A comb run through one's dry hair
attracts small bits of paper. Why ? What happens if the
Problem 6. A positively charged rod attracts a hair is wet or if it is a rainy day ? [NCERT]
suspended object. Can we conclude that the object is
negativelycharged ? Solution. When the comb runs through dry hair, it gets
charged by friction. Tlie molecules in the paper get
Solution. No. A positively charged rod can attract
polarized by the charged comb, resulting in a net force of
both a neutral object and a less positively charged object.
attraction. If the hair is wet, or if it is rainy day, friction
Problem 7. How does a positively charged glass rod between hair and the comb reduces. The comb does not
attract a neutral piece of paper ?
get charged and thus it will not attract small bits of paper.
Solution. The positively charged rod induces negative
Problem 13. Ordinary rubber is an insulator. But the
charge on the closer end and positive charge on the
special rubber tyres of aircrafts are made slightly
conducting. Why is this necessary ? [NCERT]
Solution. During landing, the tyres of aircraft may get
highly charged due to friction between tyres and the air
strip. If the tyres are made slightly conducting, they will
lose the charge to the earth otherwise too much of static
electricity accumulated may produce spark and result in
fire.
ELECTRIC CHARGES AND FIELD

Problem 14. Vehicles carrying inflammable materials Solu lion. The charge on any body is always an integral ?
usually have metallic ropes touching the groimd during multiple of e. Here
motion. Why ? [Himanchal 98 ; Punjab 99 ; NCERT]
-19
C
1.63

/
n= —
^ _ 0.8 X 10 = 0.5
Solution. Moving vehicle gets charged due to friction. e 1.6x10 -19 c
The inflammable material may catch fire due to the spark Tills is not an integer. So a body cannot have a charge
produced by charged vehicle. When metallic rope is used, of 0.8 X 10“
the charge developed on the vehicle is transferred to the
ground and so the fire is prevented. Problem 22. If the distance between two equal point
Problem 15. An inflated balloon is charged by charges is doubled and their individual charges are also
rubbing with fur. Will it stick readily to a conducting doubled, what would happen to the force between them ?
wall or to an insulatingwall ? Give reason. [Roorkee] Solution. Tine original force between the two charges is
Solution. It will stick readily to the conducting wall. It F =
1

induces an equal amount of charge on the conducting

w
0
wall and much smaller charge on insulating wall. So a
When the individual charges and the distance
large force of attraction acts between the balloon and the
between them are doubled, the force becomes
conducting wall.
1 2q x2q _ 1 q xq

Flo
Problem 16. A metal sphere is fixed on a smooth F' = = f
horizontal insulating plate. Another metal sphere is 4kZq ' (2rf 4ti£q
placed a small distance away. If the fixed sphere is given

ee
Hence the force will remain same.
a charge, how will the other sphere react ? Problem 23. The electrostatic force between two

Fr
Solution. The charge on the fixed sphere induces charges is a central force. Why ?
unlike charge at the closer end and like charge on the far
end of the free sphere. Net attraction acts on the free Solution. The electrostatic force between h\'0 charges
sphere and so it gets accelerated towards the fixed sphere, sets along the line joining the two charges. So it is a central
for
ur
force.
Problem 17. Is there some way of producing high
voltage on your body without getting a shock ? Problem 24. How is the Coulomb force between two
ks
Solution. If we stand on an insulating surface and charges affected by the presence of a third charge ?
Yo

touch the live wire of a high power supply, a high poten Solution. The Coulomb force between two charges
oo

tial is developed on our body, without causing any shock. does not depend on the presence of a third charge.
Problem 18. A charged rod attracts bits of dry cork
eB

Problem 25. Two equal balls having equal positive


which after touching the rod, often jump away from it charge 'q' coulombs are suspended by two insulating
violently. Why ?
strings of equal length. What would be the effect on the
Solution. The charged rod attracts the bits of dry cork
r

force when a plastic sheet is inserted between the two ?


ou

by inducing unlike charge at their near ends and like


ad

[CBSE OD 14]
charge at their far ends. When the cork bits touch the rod,
Y

Solution. The force between the two balls decreases


they share the charge of the rod of the same sign and so
get strongly repelled away. because K(Plastic) > 1 and F « 1/ k.
Problem 26. Force between two point charges kept at
nd

Problem 19. What does q^+q^-O signify in


Re

electrostatics ? [CBSE OD OICJ a distant d apart in air is F. If these charges are kept at
the same distance in water, how does the electric force
Fi

Or
between them change ? [CBSE OD 11]
Two charges <7j and (jj/ separated by a small distance
Solution. Dielectric constant for water, k = 80
satisfy the equation q^ +^2 “ '^st does it tell about F, F
air _
the charges ? [CBSE F 03] Fwater
K 80
Solution. The equation signifies that the electric
charges are algebraically additive and here q^ and q2 are Thus the force in water is 1/ 80 times the original force
in air.
equal and opposite.
Problem 20. Name the experiment which established Problem 27. The dielectric constant of water is 80.
the quantum nature of electric charge. [CBSE OD 981 What is its permittivity ? [Haryana 97C]
e
Solution. Millikan's oil drop experiment for deter Solution. Dielectric constant, k =
mining electronic charge.
-19 -12
x80
Problem 21. Can a body have a charge of 0.8 x 10 C? .●. Permittivity, e= ksq = 8.854 x 10
Justify your answer by comment ? [Himachal 99C] = 7.083 X10"^®
s L64 PHYSICS-XII

Problem 28, Give an example to illustrate that electro


static forces are much stronger than gravitational forces.
Solution. A charged glass rod can lift a piece of paper
Problem 35. An electron and a proton are kept in the
same electric field. Will they experience same force and
have same acceleration ?
against the gravitational pull of the earth on this piece. Solution. Both electron and proton will experience
This shows that the electrostatic force on the piece of force of same magnitude, P = e£ Since a proton has 1836
paper is much greater than the gravitational force on it. times more mass than an electron, so its acceleration will
Problem 29. Two electrically charged particles, be 1/1836 times that of the electron.
having charges of different magnitude, when placed at a
Problem 36. Why direction of an electric field is
distance 'd' from each other, experience a force of
taken outward (away) for a positive charge and inward
attraction TThese two particles are put in contact and
(towards) for a negative charge ?
again placed at the same distance from each other.
What is the nature of new force between them ? Solution. By convention, the direction of electric field
is the same as that of force on a unit positive charge. As
Is the magnitude of the force of interaction between
this force is outward in the field of a positive charge, and
them now more or less than F ? [CBSE Sample Paper 11 ] inward in the field of a negative charge, so the directions
Solution. When the two particles are put in contact, are taken accordingly.
they share the difference of charge identically. Hence the Problem 37. A charged particle is free to move in an
two particles repel, witlr a force less than F. electric field. Will it always move along an electric
Problem 30. An electron moves along a metal tube field ? [irri
with variable cross-section, as shown in Fig. 1.107. How
Solution. The tangent at any point to the line of force
will its velocity change when it approaches the neck of
the tube ? gives the direction of electric field and hence of force on a
charge at that point. If the charged particle starts from
V
rest, it will move along the line of force. If it is in motion
G- and moves initially at an angle with the line of force, then
resultant path is not along the line of force.
Problem 38. A small test charge is released at rest at a
Fig. 1.107
point in an electrostatic field configuration. Will it
Solution. The positive charge induced on the neck of travel along the line of force ? [NCERT)
the tube will accelerate the electron towards the neck.
Solution. Not necessarily. The test charge will move
Problem 31. Why should a test charge be of negli along the line of force only if it is a straight line. This is
gibly small magnitude ? because a line of force gives the direction of acceleration
Solution. The magnitude of the test charge must be and not that of velocity.
small enough so that it does not disturb the distribution of
Problem 39. Why do charges reside on the surface of
the charges whose electric field we wish to measure the conductor ?
otherwise the measured field will be different from the
actual field. Solution. Charges lie at the ends of lines of force.
Problem 32. In defining electric field due to a point These lines of force have a tendency to contract in length.
charge, the test charge has to be vanishingly small. How The lines of force pull charges from inside a conductor to
its outer surface.
this condition can be justified, when we know that charge
less than that on an electron or a proton is not possible ? Problem 40. Why is electric field zero inside a
Solution. Because of charge quantisation, the test charged conductor ?
charge cannot go below e. However, in macroscopic Solution. This is because charges reside on the surface
situations, the source charge is much larger than the of a conductor and not inside it.

charge on an electron or proton, so the limit qQ—>0 for the Problem 41. Why do the electrostatic field lines not
test charge is justified. form closed loops ? [CBSE OD 14, 15 ; D 20]
Problem 33. What is the advantage of introducing the
Solution. Electrostatic field lines start from a positive
concept of electric field ?
charge and end on a negative charge or they fade out at
Solution. By knowing the electrical field at a point, the infinity in case of isolated charges without forming any
force on a charge placed at that point can be determined.
closed loop.
Problem 34. How do charges interact ?
Alternatively, electrostatic field is a conservative field.
Solution. The electric field of one charge exerts a force The work done in moving a charge along a closed path
on the other charge and vice versa. must be zero. Hence, electrostatic field lines cannot form
Charge ^ Electric field ^ Charge. closed loops.
ELECTRIC CHARGES AND FIELD 1.6'5(
Problem 42. Why do tlie electric field lines never Solution. If the field lines are not normal, then the field
cross each other ? [CBSE OD 14 ; D 20]
£ would have a tangential component which will make
Solution. If two lines of force intersect, then there electrons move along the surface creating surface currents
would be two tangents and two directions of electric field and the conductor will not be in equilibriu m.
at the point of intersection, which is impossible. Problem 47. Fig. 1.111 shows two P, Pi
Problem 43. Draw large metal plates, ■P^ and tightly held
lines of force to represent
a uniform electric field.
against each other and placed between
two equal and unlike point charges
[CBSE OD 95] ^ perpendicular to the line joining them, + Q -Q
Solution. The lines of
force of a uniform electric
(i) What will happen to the plates
field
when they are released ?
are
equidistant
Fig. 1.108 Uniform electric field. (ii) Draw the pattern of the electric
parallel lines as shown in

w
Fig. 1.108. field lines for the system.
[CBSE F 09] Fig. 1.111
Problem 44. Fig. 1.109 shows electric lines of force
Solution.
due to point charges and q^ placed at points A and B

Flo
respectively. Write the nature of charge on them. (/) When released, the two plates tend to move
apart slightly due to the charges induced in them.
[CBSE F 03]

ee
(ii) The pattern of the electric field lines for the
system is shown in Fig. 1.112.

Fr
Pi Pz
+

for
ur
+

+
ks
Yo

+
oo

Solution. As the lines of force are pointing towards


as well as ij2/Sobothfyj and £^2 must be negative charges.
eB

Problem 45. A positive point charge (+Q) is kept in


the vicinity of an uncharged conducting plate. Sketch Hg. 1.112
electric field lines originating from the point charge on
r

Problem 48. In the electric field shown in Fig. 1.113,


ou

to the surface of the plate. [CBSE OD 09.17C]


ad

the electric field lines on the left have twice the


Solution. Starting from the charge +Q, the lines of separation as that between those on the irght. If the
Y

force will terminate at the metal plate, inducing negative magnitude of the field at point A is 40 NC“^, calculate
charge on it. At all positions, the lines of force will be
perpendicular to the metal surface, as shown in Fig. 1.110. the force experienced by a proton placed at point A Also
nd
Re

find the magnitude of electric field at point B.


+Q
Fi

IZ
●B
Induced ''

-ve charge

Fig. 1.113

Solution. Force on proton at point A,


Fig. 1.110 -19 -18
F = eE^= 1.6 X10 X 40=6.4x10 N

Problem 46. Why is it necessary that the field lines As the separation between the lines of force at point B
from a point charge placed in the vicinity of a conductor is twice of that at point A, so
must be normal to the conductor at every point.
[CBSE F 09] ^=1x40 = 20 NC"’.
L66 PHYSICS-XII

surface ‘S’. What is the electric flux due to this con-


Problem 49. The electric lines of force tend to _
ICBSE D 10]
contract lengthwise and expand laterally. What do they figuration through the surface S ?
indicate ?
Solution. _= Net charge enclosed by the surface S
Solution. The lengthwise contraction indicates ^0
attraction between imlike charges while lateral expansion + 2q-(i _ q
indicates repulsion between like charges. ®0 6o
Problem 50. A point charge placed at any point on the
axis of an electric dipole at some large distance Problem 55. Two charges of magnitudes -2Q and +Q
experiences a force F. What will be the force acting on are located at points (a, 0) and (4a, 0) respectively. What
the point charge when its distance from the dipole is is the electric flux due to these charges through a sphere of
doubled ? [CPMT 91]
radius '3<i' with its centre at the origin ? [CBSE OD 13]
Solution. At any axial point of a dipole, electric field Solution. Only the charge -2Q is enclosed by the
vanes as

1 f 1 ^ 1
sphere of radius 3a. By Gauss's theorem.
Ex-t or
2Q
r q ? ? £ “
%
.●. When the distance of the point charge is doubled,
the force reduces to F / 8. Problem 56. A point charge +Q is placed at the centre
Problem51. Considerthe situationshown in Fig. 1.114. O of an uncharged hollow spherical conductor of inner
What are the signs ofq^ and^j ● ®*'® <^rawn in radius 'a' and outer radius 'b'. Find the following :

proportion to the charge, (a) The magnitude and sign of


what is the ratio / <72 ● the charge induced on the
inner and outer surfaces of
Solution. Here <7i is a the conducting shell,
negative charge and is (b) The magnitude of electric
a positive charge. field vector at a distance

3l-A (i) r = —, and (li) r - 2b,


l'
18 Fig. 1.116
from the centre of the shell.
= 1:3. Fig. 1.114 [CBSE SP 18]

Problem 52. An arbitrary surface encloses a dipole. Solution, (fl) The charge +Q at the centre induces
What is the electric flux through this surface ? charge -Qon the inner surface of the shell and charge + Q
[Exemplar Problem] on the outer surface of the shell.
Solution. As the total charge of a dipole is zero, so by (b) (i) Imagine a concentric spherical surface of radius
Gauss's theorem, the electric flux through the closed r = - as the Gaussian surface. By symmetry, E will have
surface is zero. ^
Problem 53. The force on an electron kept in an
same magnitude at all points on this surface and will
point radially outwards.
electric field in a particular direction is f. What will be
the magnitude and direction of the force experienced by a Flux through the Gaussian surface,
proton at the same point in the field ? Mass of the proton (fig = £x4xr^
is 1836 times the mass of the electron. [CBSE F07]
Charge enclosed by the Gaussian surface = + Q.
Solution. A proton has charge equal and opposite to
that of an electron. Hence the proton will experience a By Gauss's theorem.
force equal and opposite to that of F. Q Q
or EX =
Problem 54. Figure 1.115 shows three charges + 2q, -q
and + 3q. Two charges + 2q and -q are enclosed within a 1 1 4Q
E=
■ .,2 ■
47teQ 471£q

(ii) For outside points like r = 2b, the field is similar to


+ 3q that of a point charge.
S E =
1 Q _ 1 Q
Fig. 1.115 AneQ {2bf Aubq 4b^
ELECTRIC CHARGES AND FIELD

Short Answer Conceptual Problems

Problem 1. Five balls, numbered 1 to 5 are suspen


ded using separate threads. Pairs (1,2), (2,4), (4,1) show 4neQ r
electrostatic attraction, while pairs (2, 3) and (4,5) show neglecting the sizes of spheres A and B in comparison to r.
repulsion. What is the nature of charge on ball 1 ? When an identical but uncharged sphere C touches A, the
Solution. Repulsion is the surest test of electrification. charges redistribute on A and C and, by symmetry, each
sphere carries a charge {q / 2). Similarly, after D touches B,
So balls 2,3,4 and 5 are electrically charged. Pairs (2,3) and
the redistributed charge on each is {q'/2). If now the
(4,5) are similarly charged. As pair (2,4) shows attraction,
separation between A and B is halved, the magnitude of
so balls 2 and 4 are oppositely charged. If balls (2, 3) are the electrostatic force on each is
positively charged, then(4, 5)be negatively charged or vice
f' = 1 {ql2){q'!2)_ 1 (qq') ^ ^

w
versa. The ball 1 is attracted both by 2 and 4 separately i.e.,
by both kinds of charges. Hence the ball 1 is neutral. 4Tt£Q (r/2)^ 4neQ
Problem 2. A charged metallic sphere A is suspended Thus the electrostatic force on A, due to B, remains
unaltered.
by a nylon thread. Another charged metallic sphere B

Flo
carried by an insulating handle is brought close to A Problem 3. Compare electrostatic and gravitational
such that the distance between their centres is 10 cm as interactions.

ee
shown in Fig. 1.117(fl). The resulting repulsion of A is Solution. Similarities between electrostatic and

Fr
noted (for example, by shining a beam of light and gravitational interactions :
measuring the deflection of its shadow on a calibrated 1. Both forces act according to similar laws :
screen). Spheres A and B are touched by uncharged
spheres C and D, respectively as shown in Fig. 1.117(b). C for and F=G^
ur
47re 0
and D are then removed and B is brought closer to A to a
distance of 5.0 cm between their centres, as shown in 2. Both are conservative forces.
Fig. 1.117(c). What is the expected repulsion of A on the
ks
3. Both are central forces.
basis of Coulomb's law ? Spheres A and C and spheres B
Yo

Dissimilarities between electrostatic and gravi


oo

and D have identical sizes. Ignore the sizes of A and B in


tational interactions :
comparison to the separation between their centres.
eB

[NCERT] 1. Electrostatic interactions may be attractive or


repulsive while gravitational interactions are
B
always attractive.
r

(fl) 2. Electrostatic interactions depend on the nature


ou

A
ad

of the medium while gravitational interactions


M- 10 cm ♦I
do not depend on the nature of the medium.
Y

3. Electrostatic interactions are much stronger


than gravitationalinteractions.
nd
Re

Problem 4. Distinguish between electric charge and


mass.
Fi

(b) Solution.
C

Electric Charge Mass

1. Electric charge can be Mass is always positive.


positive, negative or zero.
2. Electric charge is always Quantisation of mass is
B quantised. yet not established.
(c)
3. Charge on a body does Mass of a body
N— 5 cm —M not change with its increases with its speed.
speed.
Fig. 1.117
4. Electric charge is always Mass is not conserved
Solution. Let the original charge on sphere A be and conserved. by itself as it can be
that on B be q'. At a distance r between their centres, the changed into energy
and vice versa.
magnitude of the electrostatic force on each is given by
s L68

Problem 5. What is an electric line of force ? What is


its importance ?
PHYSICS-Xtl

{Ihinjab 98C]
Solution. An electric line of force may be defined as
Solution. The lim indicates that the test charge q is

small enough so that its presence does not affect the


the path straight or curved, along which a unit positive distribution of source charge and hence does not change
the value of electric field which we wish to measure.
charge would tend to move if free to do so.
For electric lines of force of point charge Q, see
Importance :
Fig. 1.74 and Fig. 1.75.
(i) The tangent at any point on the curve gives the
Problem 9. A charge Q located at a point is in
direction of the electric field at that point.
(ii) The relative closeness of the lines of force
equilibrium under the combined electric field of three
indicates the relative strength of electric field charges q-^,q2'^z’ charges ^^,^2 located at
at different points. points and respectively, find the direction of the
Problem 6. Represent the surface distribution of force onQ, due to in terms of q^ q^, ^ ^ and 7.
charge for a square metal plate by using dashes in such a
[CBSE Sample Paper 08]
way that the greater the surface density of charge, the
farther away are the dashes from the plate. Solution. The forces exerted by q^ and t/2 on Q are
shown in Fig. 1.119.
Solution. The distribution of surface charge density on
a square metal plate is as shown in Fig. 1.118. As the
surface charge density is proportional to the curvature
and curvature is maximum at the corners and zero at

plane surface, so dashes are equidistant from the straight


portion and far-away from the comers.

Qnarged
metal plate

Fig. 1.118 According to Coulomb's law, force exerted by q^ on


Problem 7. Two point electric charges of unknown Qis
magnitude and sign are placed a distance'd' apart. The 1 Qqi r -
E1 =
electric field intensity is zero at a point, not between the 4k£ 0 ~*i2 I ~*\
charges but on the line joining them. Write the essential r-t[\ I r - rjI
conditions for this to happen. (CBSE D 97] -»

1 ZA
Solution.
(/) The two charges must have opposite 4ti£ ->,3
0
r -rj 1
signs.
(ii) The magnitude of the charge lying near the point of Force exerted by q2 on Q is
zero electric field intensity must be smaller than the
magnitude of the other charge. ^2 = 1-Q^2 ^ -^2
4ne 0
Problem 8. The electric field E due to a point charge
F According to the principle of superposition, total force
at any point is defined as lim — , where q is the test
q^oq exerted by q^ and ^2 Q
charge and F is the force acting on it. What is the F = F^ + F2
physical significance of lim in this field expression ?
q 0
1 r - K ^ r -r^
Draw the electric lines of a point charge Q when (i)Q > 0 Qq. 1— + Qfl-
and (ii) Q < 0. [CBSE D 07]
471£„0 r
^,3
^ I
^
-'2N
ELECTRIC CHARGES AND FIELD

The charge Qwill be in equilibrium if the force exerted


Clearly, the magnitude of field £ will be same at all
by ^3 is equal and opposite to the combined force exerted
points on the surface of a sphere of radius r drawn around
by ^ and ^2-
the point charge and does not depend on the direction of
Force exerted by ^3 on Q = - F r*. Hence tlie field due to a point charge is spherically
-» -*

Q symmetric.
u. -^2
Ane
-r\^ Electric field at distance r on the equatorial line of an
0
-'ll 71
electric dipole of dipole moment p is given by

Q "1 - r £-— P
4ns
0
-» -»,3 47160 ■(r2 + fl2)3/2
r r

The electric field £ is same at all points which lie on a


The above vector gives the direction of the force on Q cylinder of radius r with its axis on the dipole axis and the

w
due to ^3. field pattern looks same in all planes passing through the
Problem 10. Two point charges and - q are placed dipole axis. We say that the electric field of an electric
a'd' distance apart. Draw the line on which the resultant dipole is cylindrically symmetric.

Flo
field is parallel to the line joining the two charges. Problem 13. An electric dipole free to move is placed
ICBSE OD 96C] in a uniform electric field. Explain alongwith diagram

ee
Solution. As shown in Fig. 1.120, the resultant at all its motion when it is placed,

Fr
points lying on the perpendicular bisector (dotted line) is (n) parallel to the field,
parallel to the line joining the charges + q and - q. (b) perpendicular to the field.
£ fCBSE Sample Paper 1990}
P f
for
ur
Solution. ('^) Since the line of action of the two forces
A passes through the same point, the net force and the net
torque acting on the dipole is zero. So no motion is
ks
produced when a dipole is placed parallel to the electric
Yo

field.
oo

I.
eB

0 + '7£
r

Fig. 1.120
ou
ad

Problem 11. Draw a diagram to show lines of force in E ..+qE


Y

a plane containing two equal point charges of opposite E P


-90
sign separated by a small distance. Giving reason, indi 0+9
cate on the diagram a point where a small positive charge p-^
nd
Re

,.-qE
experiences a force parallel to the line joining the two
charges. ICBSE D 93C]
Fi

0-9
Solution. As shown in Fig. 1.120, electric field at any
point Pon the equatorial line is parallel to the line joining
the two charges. So if a small positive charge is placed at («) (b)
such a point, it will experience a force parallel to the line
joining the two charges. Fig. 1.121 Electric dipole (a) parallel
Problem 12. What is meant by the statement that the (b) perpendicular to uniform electric field £.
electric field of a point charge has spherical symmetry
(b) Tlie two equal and opposite forces - qE and + qE
whereas that of an electric dipole is cylindrically
constitute a couple and hence a torque acts on the dipole,
symmetric ? [Haryana 96] given by
Solution. electric field due to a point charge q at T = p£ sin 90° - 9.2aE [V p = q.2a]
distance r from it is given by
This torque rotates the dipole about an axis
£ = perpendicular to the electric field and passing through the
4nsQ midpoint of the dipole.
1:70 PHYSICS-Xll

Problem 14. An electric dipole is a pair of equal and (i) When 0= 90° torque is maximum [Fig. 1.122(a)].
opposite charges, separated by a small fixed distance W = 90°=p£
between them. The dipole is free to move. What is the
action on it, when it is placed in (ii) When 0 = 30° or 150°, torque is half the maximum
value [Fig. 1.122(b)].
(t) a uniform electric field, and
(ji) a non-uniform electric field ?
X = p Esin (30°or 150°)
1 „ 1
Solution. (0 In a uniform electric field, an electric — p£ = — T max
dipole experiences two equal, opposite and parallel forces 2^ 2
at its two ends. The net force on it is zero but it
(m) When 0 = 0° or 180°, to is minimum

experiences a torque due to which it rotates about an axis [Fig. 1.122(c)]


perpendicular to the electric field and passing through its
mid-point, ^min = p £sln(0°or 180 /
(ii) In a non-uniform electric field, an electric dipole Problem 16. Two small identical electrical dipoles
experiences two unequal and non-parallel forces at its two AB and CD, each of dipole moment p are kept at an
ends. The two forces add up to give a resultant force
angle of 120° to each other in an external electric field
and a torque. So the dipole will accelerate linearly in the —>

direction of the resultant force and also accelerate E pointing along the x-axis as shown in Fig. 1.123.
y
angularily in the direction of the resultant torque.
+ q»A
Problem 15. An electric dipole of dipole moment p is -c\
D

placed in a uniform electric field E. Write the expression 120°


X’' X
for the torque 7 experienced by the dipole. Identify two 0/
c
pairs of perpendicular vectors in the expression. Show
Z' +q
diagrammatically the orientation of the dipole in the -<7*B
field for which the torque is ii) maximum Hi) half the r
maximum value (Hi) zero. [CBSE Sample Paper 08]
Solution. Torque experienced by the electric dipole of Fig. 1.123
Find the
dipole moment p in a uniform electric field £ is given by
(i) dipole moment of the arrangement, and
^
T =p X £ (iV) magnitude and direction of the net torque
acting on it. [CBSEDll;OD 20]
The pairs of perpendicular vectors are :
Solution. (0 The directions of the two dipole moments
1. T* and p 2. T and E
and their resultant are shown in Fig. 1.124.
/I,
+ 4 0
Pa \PR
■X
£
a

P Pc

Fig. 1.124
-q
Given p^ = p^ = p
(fl) ib) Resultant dipole moment.

Pr = yjp^ + p^ + 2 X p X p cos 120°


P = P-
£
-q -*-q
This dipole moment acts along the bisector of Z AOC
i.e., at an angle of 30° with +X direction.

ic)
(ii) Torque, t = p£ sin 30°=-^pE
By right hand rule, the torque t acts into the plane of
Fig. 1.122 paper along Z-direction.
ELECTRIC CHARGES AND FIELD 1

Problem 17. (fl) Define electric flux. Write its SI units. Problem 19. T'vo thin concentric and coplanar sphe
[CBSE 18C)
rical shells, of radii fl and h (h > fl) carry charges,^ andQ,
(b) A spherical balloon carries a charge that is respectively. Find the magnitude of the electric field, at
uniformly distributed over its surface. As the balloon is a point distant x, from their common centre for
blown up and increases in size, how does the total ii) 0<x <a Hi) a <b HU) b ^x <oo
electric flux coming out of the surface change ? Give ICBSE F 15, D 16C]
reason. [CBSE D 07]
Solution. (') For 0 < x < fl, the charge enclosed by
Solution, (fl) The electric flux through a given surface Gaussian surface I is zero.
area is the total number of electric lines of force passing
normally through that area.
It is given by
a4>£ = e . as

w
- £ AScos 0

SI unit of electric flux = Nm^C“^

Flo
(b) As the balloon is blown up and increases in size,
the total charge on its surface remains constant. Hence, by
Gauss's theorem, the total electric flux coming out of its

ee
surface remains unchanged.

Fr
Fig. 1.126
Problem 18. A thin metallic spherical shell of radius
Q:is By Gauss's theorem.
R carries a charge Q on its surface. A point charge
y
4,, = f£.rfS=i
for
ur
placed at its centreC and another charge +2Q is placed
outside the shell at a distance x from the centre as £x47CC^ = 0
shown in Fig. 1.125. £=0
ks
A
(ii) For a <x <h, the net charge enclosed by the
Yo
oo

2Q Gaussian surface II is q.
Ex4ra:^=-^ or E = _L _1
eB

Q*C 4tC£q
(m) For b < X < CO, the net charge enclosed by the
r

Gaussian surface III is {q+ Q).


ou
ad

Fig. 1.125
Y

Find ^0

H) the force on the charge at the centre of shell or £ =


1 q+Q
4nSrt0 x^
nd

and at the point A,


Re

Hi) the electric flux through the shell. Problem 20. A spherical rubber balloon carries a
Fi

[CBSE D 15] charge that is uniformly distributed over its surface. As


the balloon is blown up ; how does E vary for points
Solution. Nst force on the charge Q/2, placed at the
centre of the shell is zero.
H) inside the balloon. Hi) on the surface of the balloon
and HU) outside the balloon ?
Force on the charge 2Q kept at point A at distance r
from the centre is Solution. (0 For points inside the balloon, £ = 0.
(jj) As the balloon is blown up, surface charge density
F=Ex2Q = ' .^x2Q
4tI£
0
r a decreases and so the field, (£ = o/eQ) on its surface
decreases.
1 3Q^
47iSn0 (Hi) For points outside the balloon,

{ii) Electric flux through the shell. £=—■4


47t£o r
Q/2_ Q
£ “ As the balloon is blown up, the charge enclosed by
the Gaussian surface remains same, so Edoes not change.
PHYSICS-XII

Problems on Higher Order Thinking Skills

Problem 1. A metallic spherical shell has an inner Solution.

radius and outer radius i?2- A charge Q is placed at the


centre of the spherical cainty. What will be the surface charge
density on (i) the inner surface and (ii) the outer surface ?
(Exemplar Problem ; CBSE OD 19]
Solution. Charge -Q is induced on the inner surface
and charge + Q is induced on the outer surface of the
cavity.
-Q
Surface charge density on the inner surface =

Q (a) Top view (b) Side view


Surface charge density on the outer surface =
Fig. 1.128
Problem 2. The dimensions of an atom are of the order
of an Angstrom. Thus there must be large electric fields Problem 5. Tioo point charges placed at a distance r in
between the protons and electrons. Wlnj, there is the air exert a force F on each other. At zohat distance will these
electrostatic ifeld inside a conductor zero ? charges experience the same force F in a medium of dielectric
[Exemplar Problem] constant K ?
Solution. The electric fields of protons and Solution. Fair■ ]_ ^1^2 _ p -(0
electrons bind the atoms to a neutral entity. Fields are 47te„0
caused by excess charges, there can be no excess charge
on the inner surface of an isolated conductor. 1
F = p ...{ii)
med
4TTSnK
0
Problem 3. Two dipoles, made from charges ±qand ± Q,
respectively, have equal dipole moments. Give the (i) ratio Dividing (i) by (ii), we get
between the 'separations' of these two pairs of charges
(ii) angle between the dipole axes of these two dipoles. = 1
(CBSE Sample Paper 13]
Solution. As the two dipoles have equal dipole or
moments, so

{i)qa = Qa' => — =~ Problem 6. A force F is acting between two charges


a Q placed some distance apart in vacuum. If a brass rod is
{ii) their dipole axes must have same direction i.e placed between these tzoo charges, how does the force
0=0® ' change ?
Solution. For any metal, k = co
Problem 4. Sketch the electric ifeld lines for a uniformly
Fvac
charged hollow cylinder as shown in Fig. 1.127. ,Fbrass
F
— = 0
K 00

i.e., in the presence of brass rod, the force between the


two charges becomes zero.
Problem 7. Figure 1.129 sliozos the electric lines around
three charges A, B and C.
{a) Which charge is positive ?

Fig. 1.127 {b) Which charge has the largest magnitude ? Why ?
●I,.

ELECTRIC CHARGES AND FIELD 1.73 ^

(c) In which region or regions of the picture could the (i) When the spheres are slightly separated with
electric field be zero ? the glass rod undisturbed, there is little change
(i) near A (ii) near B (Hi) near C (iv) nowhere. in the distribution of charges, as shown in
Justify your answer. Fig. 1.131(fl).
[ii) When the glass rod is removed, there is
redistribution of charge on each sphere. Their
positive and negative charges will face each
other, as shown in Fig. 1.131(&).
(Hi) The charge on each sphere gets uniformly
distributed as shown in Fig. 1.131(c).
Problem 9. Figure 1.132 shows tracks of three charged
particles in a uniform electrostatic field. Give the signs of the

w
Fig. 1.129
three charges. Which particle has the highest charge to mass
ratio ?

Solution, (a) Charges A and C are positive because

Flo
the lines of force are emerging out from them,
+ + + + + + + +
l) y
2
(fa) Charge Chas the largest magnitude because the

ee
maximum number of field lines are associated with it. >

Fr
(c) (/) Near A No neutral point can exist between 3
unlike charges A and B or between B and C. The
neutral point exists between like charges A and C.
for
ur
Fig. 1.132
Also, the neutral point will be closer to the charge with
smaller magnitude. Hence, electric field is zero near (fl) Suppose that a particle is attracted towards the
charge A. positive plate : what must the charge on it be ?
ks

Problem 8. A glass rod rubbed zvith silk is brought close


Yo

(b) Suppose, two particles have identical curved trajectories.


oo

to tzvo imcharged spheres in contact zinth each other, inducing Which of the following are necessarily true ?
charges on them as shozvn in Fig. 1.130. Describe zvhat happen (i) They have same charge ; (ii) They have same
eB

luhen
mass; (Hi) The charges have the same sign; (iv) They
have the same e/m ratio,

) (c) You are given the initial velocity v of a beam particle


r
ou

and the length of the capacitor 1. What other


ad

+ +

measurement would enable one to find e/m ?


Y

[NCERT;CBSED01C]

Solution. Particles 1 and 2 have negative charges


nd

Fig. 1.130
Re

because they are being deflected towards the positive


(/) the spheres are slightly separated, and plate of the electrostatic field.
Fi

(ii) the glass rod is subsequently removed, ajzd ifnally Particle 3 has positive charge because it is being
(Hi) the spheres are separated far apart ? deflected towards the negative plate.
Solution. Acceleration acting on charge q in y-direction,

a-^-
A B A B m m

Therefore, deflection of charged particle in time t in


y-direction is
1
h =0 X t + -
(fl) (b) (c) 2 2 m

i.e.. h a: 3-
Fig. 1.131 m
1:74 PHYSICS-XII

As the particle 3 suffers maximum deflection in Solution. (0 As F ocir, so F versus r2


graph is a
y-direction, so it has highest charge to mass {q/m)
ratio. straight line for both pairs of charges.
(fl) Negative charge. (ii) For the pair (1 pC, 2 pC), F is repulsive, the graph
(b) (m) and (ii;). The particles must have charges of OA has a +ve slope. For the pair (2pC,-3pC), F is
same sign and same elm ratio. attractive, the graph OB has a -ve slope.
(c) To find ,
we measure the vertical displace- (iff) Clearly, F.,
auraction
=3F
repulsion'
, the slope of OB is
^
ment h as the particle crosses the capacitor jj^ree times the slope of OA
plates.
F,,
Time taken by a particle to cross the capacitor plates. A

/
f = -
V

1
e£ n f 0
^ = 135(2 =0.5- r
2

2 m m\vj

Problem 10. Figure 1.133 shows three different patterns


of electric field lines. In each pattern, a proton is released
from rest at point Pand then accelerated towards the point Q ■B
by the electric field. Rank the patterns according to the linear
momentum of the proton when it reaches Q, greatest first. Fig. 1.134

Problem 12. Two point charges, t/j and located at


Q
points (fl,0,0) and (0, b,0) respectively. Find the electric
>■
field, due to both these charges, at the point (0,0, c).
[CBSE Sample Paper 13]
Solution. Net electric field at the point (0,0, c) due to
(a)
the charges and q2 is
■>


Fnet — £j+ £2 ~ 4718
0 7

(b)
Y

‘?2 (0, b, 0)
/
ic)

Fig. 1.133 Hi (a, 0,0)


I ♦ ■X
/ 0 .."''ll
Solution. The lines of force near point P are closest /
r

to each other in pattern (c) and farthest apart in


1

0, 0, c)
pattern (a). Consequently, electric field near point P is El
strongest in case (c) and weakest in case (a).
.-. Momentum of proton at point Q in pattern (c)
Fig. 1.135
> Momentum of proton at point Qin pattern (b)
> Momentum of proton at point Q in pattern (a) But - -a i+ ck

Problem 11. p/ot a graph showing the variation of


( 1
, where r is the distance
'r2 = -bj + ck => r2 +
coulomb force (F) versus
r^J
between the two charges of each pair of charges: (1 pC, 2 pC) Enel —
1 q.^{-ai+ck) q^i-bj+ck)
and (2pC, -3pC). Interpret the graphs obtained. 4718 0 {<,2 + c2)3/2 +
[CBSE OD 11]
ELECTRIC CHARGES AND FIELD

Problem 13. /I point charge is placed at the centre of ^^1^5 and ^ are equal and opposite and also Eg
spherical Gaussian surface. How will electric flux (j^ change
if and Ep are equal and opposite. Hence the net field at
(i) the sphere is replaced by a cube of same or different the centre is zero.
volume,
(ii) In both figures 1.137(i) and (n), the potentials
(») a second charge is placed near, and outside, the due to positive charges are +ve and those due to
original sphere. negative charges are equally negative. Hence the net
(Hi) a second charge is placed inside the sphere, and potential at the centre is zero.
{iv) the original charge is replaced by an electric dipole ? Problem 15. Three charges + q, + q and -2q are placed
Solution. By Gauss's law, ^ = < ^.di =-1 at the vertices of an equilateral triangle. What is the dipole
s ^0 moment of the system ?

w
(i) does not change because it depends only on
the total charge enclosed by the Gaussian
surface and not its shape or size.

Flo
(ii) (}^ does not change because the total flux is
determined by the charge inside the surface and
not on the charge outside.

ee
(Hi) (|^ will change because the total charge inside

Fr
the surface has changed.
(iv) (j^ becomes zero, because a dipole consists of
two equal and opposite charges and so the net
for
ur
charge inside the surface is zero. '-b
Problem 14. Four point charges are placed at the four
Fig. 1.138
corners of a square in the two ways (i) and (ii) as shown in
ks

Fig. 1.136. Will the (i) electric field (ii) electric potential, at Solution. Let each side of the equilateral triangle be d.
Yo
oo

the centre of the square, be the same or different in the two Dipole moment along CA = qd
configurations and why ? [CBSE Sample Paper 08] Dipole moment along CB ~ qd
eB

-Q -Q -Q + Q By parallelogram law, the net dipole moment acts


D c D c along diagonal CD. Its magnitude is
r

P= +2qdxqd X cos
60° = Vs qd
ou
ad

This dipole moment acts along the bisector of the


Y

A B A B

+ Q
angle at charge -2 q.
+ Q +Q -Q
(0 (»■) Problem 16. Two similar balls each having mass m and
nd
Re

charge q are hung from a silk thread of length 1. Prove that


Fig. 1.136 equilibrium separation.
Fi

\l/3
Solution. (0 In Fig. 1.137(f), the electric fields E^ X =

1^2tu£o mg )
and get added and also Eg and Ep get added. Hence
there is a net field at the centre O. In Fig. 1.137(ff), the ivhen each thread makes a small angle 0 loith the vertical.
0EE Main July 21]
-Q -Q -Q + Q Solution. Refer to Fig. 1.139. According to
D C D C
Coulomb's law, the electrostatic repulsion between the
two balls will be

f =

A —Jb a B 4tC£(j
+ Q +Q -Q
(0 Hi) For small angular displacement 0, we have
arc x/2 _ X
sin 0 - 0 (rad) =
Fig. 1.137 radius I 21
PHYSICS-XII

Problem 18. A simple pendulum cotisists of a small


0 sphere of mass m suspended cfy a thread of length 1. The
/ 0
sphere carries a positive charge q. The pendulum is placed in
a uniform electric field of strerigth E directed vertically
I / I
upxvards. With what period will the pendulum oscillate if the
electrostatic force acting on the sphere is less than the
gravitational force ? Assume oscillations to be small.
q Solution. The sphere experiences an upward pull
qE.
x/2 Q\%.

Net force on the sphere
X -— S>
= mg- qE, in downward direction
Acceleration,
mg
_ Force _mg-qE qE
Fig. 1.139 g’ = =g
Mass m m

Restoring force on each ball / I


T = 2k -=2n
= mg smd-mg — u qE
21 s-^
m

For equilibrium. Problem 19. Four particles, each haznng a charge q, are
1
X
placed on the vertices of a regular pentagon. The distance of
0 each cornerfrom the centre is 'a'. Determine the electric field
-il/3 at the centre of the pentagon.
or x^ = or x = Solution. Suppose the four charges are placed at the
2 uSq mg 2k£q mg comers A, B, C and D of the pentagon ABODE. If we
place a charge q at the corner
Problem 17. Five point charges, each of value + q coulomb
E also, then by symmetry,
are placed on five vertices of a regular hexagon of side L metres. the total electric field at O
Find the magnitude of force on a charge - q coulomb placed must be zero. Thus, the field
at the centre of the hexagon. [IIT 92 ; CBSE OD 19] at the centre O due to the
Solution. The situation is shown in Fig. 1.140. charges A, B, C and D is
Forces on charge - qatO due to charges at A and D are equal and opposite to the
equal and opposite and hence cancel out. field due to the charge q at E
q q
alone.
A B
Electric field at O due to
L charge q atE
V it
1
\-q/
F L \C .3, a
oV- - - q 47te
0
a

.●. Electric field at O due to the charges at A, B, C


and D

1
^q q
D
along OE
47te a
0

Fig. 1.140
Problem 20. An mfinite number of charges, each equal
Similarly, the forces due to charges at B and E to q are placed along X-axis at x = 1, x =2, x = 4, x =8,
cancel out. and so on.

(0 Find the electric ifeld at the point x =0due to this set


The only force on charge ~q at O is exerted by
charge +q at C. It is given by
up of charges,
[ii) What will be the electric field, if in the above set up,
qq
F = k
-3 newton, along OC. the consecutive charges have opposite signs.
(OCf HIT 95]
ELECTRIC CHARGES AND FIELD

Solution, (i) At the point x = 0, the electric fields due Solution.

to all the charges are in the same x-direction and hence F =


kQq
2^
get added up. X
2
+-■
1 4
£ =
47reo Ll 2^ 4^ g22 + ...
Net force on charge q,
1 1 1 = 2 F cos 0
H + — + ... net
4;te
0 >-
4 16 64
47ueo'i-l 3n£
0
2kQqx
\3/2
{it) Electric field at ;<r = 0 is 2 cP-
u
X + Q
1 4
£ =
4 716
0 I-
P 2^ P For maximum value of Fig. 1.143

w
dF
1 (I
net
= 0
dx
47te
0 >-
4 16 64
- 4tcEo‘1-(-|) 57te
0
\l/2
M X2 + —
2kQq P + ^

Flo
x2a: =0
Problem 21. Two identical positive charges Q each are -x.y—
2 4
fixed at a distance of '2a' apart from each other. Another

ee
point charge with mass'm' is placed at midpoint between d

two fixed charges. For a small displacement along the line "" 242.'

Fr
joining the fixed charges, the charge executes SHM. Find the Problem 23. Eight identical point charges of q coulomb
time period of oscillation of charge qQ. [JEE Mam July 22] placed at the corners of a cube of each side 0.1 m.
Solution. Calculate the electric ifeld at the centre G of the cube.
for
ur
X Calculate the field at the centre when one of the corner
Q m Q charges is removed.
o-
ks
%
a+x
a-x—●{ Solution. Length of each side, / =0.1 m
Yo

Distance of each comer from the centre of the cube is


oo

Fig. 1.142 431 43x0.1


r - = SV3 X 10"^ m
Net force on the cliarge t/Q in the displaced position,
eB

2 2

F = 0 When all the eight point charges are placed at the


(a + x)^ {a-x)^ comers, electric fields of the diagonally opposite
r

charges cancel out in pairs.


ou
ad

{a-xf -{a-vx'f Net field at the centre =0.


= kQq. 0
Y

When a charge is removed from one comer, the


electric fields due to three pairs of charges cancel out.
Q% 4ar However, the charge at the remaining comer creates field,
[x « a]
nd
Re

47C£ ■ .3
0
a
£ =
1 qxl
4ns„0 ■?
Fi

= -kx
_3 ^ 9xl0'^yq
0 = 1.2x 10^^ <jNC"^
(5V3xlO“^)'
m mne,.a^
0
T = 2nJ— -2k The field points towards the comer having no charge.
k
K Q%
Problem 24. If the total charge enclosed by a surface is
4p£,.tna^
0 zero, does it imply that the electric field everywhere on the
surface is zero ? Conversely, if the electric field everywhere
V Q%
on the surface is zero, does it imply that the charge inside is
Problem 22. Two point charges Q each are placed at a zero ? [Exemplar Problem]
distance d apart. A third point charge q is placed at a distance Solution. As ^ = 0, so from the Gauss's theorem, we
x from the mid-point on the perpendicular bisector. Find the have
value of x at which charge q will experience the maximum
Coulomb's force. [JEE Main June 22] <1^ =1 =0
a,78 PHYSICS-XII

Clearly, the above equation does not imply that £ Problem 26. A small metal sphere carrying charge + Q
is necessarily zero at all points on the Gaussian surface. is located at the centre of a spherical cavity in a large
It may also be possible that £ is non-zero but it is uncharged metal sphere as shown in Fig. 1.145. Use Gauss's
theorem to ifnd electric field at points Pj and P^.
perpendicular to d S at all points on the surface, even ICBSE D 05, OD 14C]
then the integral
Metal
^.d^ would be zero.
-* Free space
However, the converse is true. If £ is zero at every
point on Gaussian surface, then from Gauss's theorem
=-1
^0
we get, ^ =0
i.e., no net charge is enclosed by the Gaussian surface. Fig. 1.145

Problem 25. Figure 1.144 shows a cylindrical Gaussian Solution. (/) To determine the electric field at point
surface for an infinitely long thin straight wire of uniform Py consider a concentric spherical surface of radius
linear charge density. as the Gaussian surface. By symmetry, the field £ will
, >
have same magnitude at all points on this surface and
will point radially outward.
E
.-. Flux through the Gaussian surface,

I
'T\ \
(j^ = E X 4nt^
Charge enclosed by Gaussian surface = + Q
+ + + + + + + + + + + +

I By Gauss's theorem.
Q Q 1 Q
‘tk= — or
Ex 4nif = or £ =
% ^0
Fig. 1.144
(n) As the electric field inside a conductor is zero, so
Answer the following : field at point = 0.
(i) For which surface is the electric flux zero ?
Problem 27. A spherical conducting shell of inner
(»') Oi»er which surface is E zero ? radius r^ and outer radius rj has a charge 'Q'. A charge 'q' is
—* placed at the centre of the shell.
{Hi) Over which surface is [ £ | constant ? (fl) What is the surface charge density on the (i) inner
surface, (ii) outer surface of the shell ?
{iv) Over which surface does \ E \ change ?
(b) Write the expression for the electric field at a point
Solution. Electric field of a line charge, X > from the centre of the shell. ICBSE OD 10}

Solution. The charge q at the centre induces charge


E = —^ , acting radially outward
-(yon the inner surface of the shell and charge + qon the
outer surface of the shell.
(0 At the two plane end faces,
Q+q
●1^ = E.d§ =0.
(«) For any finite distance from the line charge,
£ cannot be zero.

{Hi) At all points of the curved surface, 1 is


constant.

(iv) At the plane end faces, | ^ | decreases with the


increase in distance r.
ELECTRIC CHARGES AND FIELD L79

(fl) (i) Surface charge density on the inner surface. Solution. Figure 1.148 shows the charge distri
cr
bution for the given model of the atom.
1 “

(ii) Surface charge density on the outer surface,


0-. = +
Q + <?
2

(b) For X > T2, the field is similar to that of a point


charge.
£ =
1 Q+q
0

Problem IS.Electricfield in Fig. 1.146 is directed along Fig. 1.148 An early model of atom.

w
+X direction and given by =5 Ax+2 B, where E is in
NC~ ^ and x is in metre, A and y As the atom is neutral, the total negative charge in a
B are constants with dimen Sphere of radius R must be - Ze. If p is the negative

Flo
sions. Taking A = W charge density, then we must have
and B=5NC~^, calculate : Ze + inR^ n
_p=0 or P=-
3Ze

ee
(i) the electric flux through the X 4;tR^
cube, (ii) net charge enclosed 7H7 By spherical symmetry of the charge distribution.

Fr
z/
/■»10an
within the cube.
the electric field £ depends only on radial distance r
ICBSE Sample Paper 08l Fig. 1.147 and not on the direction of r . It should point radially
for
ur
Solution, (i) The electric field is acting only in inwards or outwards. So we imagine a spherical
X-direction and its Y and Z-components are zero. So Gaussian surface of radius reentered at the nucleus.
the flux passes only through faces M and N.
ks
(i) For r <R, Flux through the Gaussian surface,
Yo

The magnitude of the electric field at the face = £ X 4nr^


oo

M(x=0)is
£., =5 Ax + 2 B = 5xl0x0 + 2x5=10NC”^ Charge enclosed by the Gaussian surface,
eB

Flux,
q = Positive nuclear charge
+ Negative charge in a sphere of radius r
= £^/^ cos 0 =10 X (0.10)^ X cosl80°
r

47cr^ 47tr^ f 3Ze


= Ze + — p = Ze + .
ou

= -0.1Nm^C“’
ad

3 3 4nR^
The magnitude of the electric field at the face
Y

r^
N (x =10 cm) is = Ze 1-
r3
Ej^=5Ax+2 6 = 5x10x0.10 + 2x5
nd
Re

= 15 NC
-1
Applying Gauss's theorem, ^ =g/eQ, we get
Fi

Flux, <^ = Ej^l^cosQ £ X 47i:r^ = —


r3
= 15 X {0.10)^ X cos 0“ = 0.15 Nm
Ze f 1 r
Net flux through the cube, or E = (r<R)
47re r^ r3
(|^ = (|)^ + ^ = - 0.1 + 0.15 = 0.05 Nm 0

(ii) Total charge enclosed within the cube, The field £ points radially outward.
-12 -13 (ii) For r>R. As the atom is neutral, the total
q= gQ<j^ =8.854 X 10 X 0.05= 4.43 X 10 C.
charge enclosed by the Gaussian surface is zero. By
Problem 29. An early model of an atom considered it to Gauss's theorem.
have a positively charged point nucleus of charge Ze, surrounded £x 47tr^ =0
by a uniform density of negative charge up to a radius R. The
atom as a whole is neutral. For this model, what is the or £ = 0. (r>R)
electric field at a distance rfrom the nucleus ? (Hi) At r = R. Both of the above cases give the same
[NCERT; AIIMS 18] result: £ = 0.
L80 PHYSICS-XII

[ Guidelines to NCERT Exercises

1.1. What is the force between two small charged spheres The factor ke^ I Gm^mp represents the ratio of
having charges of 2 x 10“^ Cand 3xl0~^Cplaced 30 cm apart electrostatic force to the gravitational force between an
in air ?
electron and a proton. Also, the large value of the ratio
Ans. Here q^=2x 10"^C, q2=3xlO'^C, signifies that the electrostatic force is much stronger than
r = 30 cm = 0.30 m the gravitational force.
1.4. (0 Explain the meaning of the statement 'electric
According to Coulomb's law, charge of a body is quantised.'
2xl0'^x3xl0‘^
F = —.^
4;rEQ r
= 9xl0^x (0.30)^
(i7) Why can one ignore quantisation of electric charge
when dealing with macroscopic i.e„ large scale
charges ?
= 6 xlO ^ N (repulsive).
Ans. (0 Quantisation of electric charge means that the
1.2. The electrostatic force on a small sphere of charge total charge (q) of a body is always an integral multiple of
0.4 pC due to another small sphere of charge -O.SpC in air is
a basic charge (e) which is the charge on an electron. Thus
0.2 N. (i) What is the distance between two spheres ? (ii) What
q = ne, where n = 0, ± 1, ± 2, ± 3,
is the force on the second sphere due to the first ?
{ii) While dealing with macroscopic charges {q = ue),
Ans. (i) Here q^ = 0.4 pC = 0.4 x 10^ C we can ignore quantisation of electric charge. This is
(?2 = -0.8pC=-0.8xlO“^C, f = 0.2N, r = 7 because e is very small and nis very large and so q behaves
1 ?l‘?2 as if it were continuous i.e., as if a large amount of charge
As f =
47rEQ is flowing continuously.
1.5. When a glass rod is rubbed with a silk cloth, charges appear
on both. A similar phenomenon is observed with many other
pairs of bodies. Explain how this observation is consistent with
9xl0^x0.4xl0'^x0.8xl0~^ -A
the law of conservationof charge.
= 144x10
0.2
Ans. It is observed that the positive charge developed
or r= 12x10"^ = 0.12 m =12 cm. on the glass rod has the same magnitude as the negative
(ii) The two charges mutually exert equal and opposite charge developed on silk cloth. So total charge after
forces. rubbing is zero as before rubbing. Hence the law of
.'. Force on the second sphere due to the first conservation of charge is being obeyed here.
= 0.2 N (attractive). 1.6. Four point charges q^ = 2 pC, q^ =-5 pC = 2 pC
1.3. Check that the ratio ke^/Gmm..
e p
is dimensionless. Look qj^ = -5 pCare located at the corners of a square ABCD of side
10 cm. What is the force on a charge o/l pCplaced at the centre
up a table of physical constants and determine the value of this
of the square ?
ratio. What does this ratio signify ?
Ans. Here OA = OB = OC - OD =
^10^ + 10^
Ans. , =
[Nm^‘^] x[Cf = no unit 2

Gm^,nipj [Nm^kg x[kg][kg] = 5^2 cm = 5>^xl0"^m

As the ratio ke^ / Gm^m^ has no unit, so it is qo=-5nC 10 cm

dimensionless. Df- T*C

Now fc=9xl0^Nm^"^
Nm^ kg ^
,-11
G = 6.67 X 10

e = 1.6 X 10”^^ kg £ o

= 9.1 X 10“^^ kg
o

and
Wp = 1.66 X 10“^^ kg
9x10^ x(1.6xl0'*^)^
6.67 X10
-11
X 9.1x10
-31
X 1.66x10
-27
Ak- B
10 cm

= 2.287x10^®. Fig. 1.149


ELECTRIC CHARGES AND FIELD L81

Forces exerted on the charge of 1 pC located at the Electric field at the midpoint O due to cf^,
centre are

.r» 9 X lO’ X 2 X 10“^ X 1 X 10"^


1
id- 9 X10^ X 3 X 10“®
4nEQ (O.lOf
(5V2x10"^)2
= 2.7xl0^NC"\ along OB
= 3.6 N, along OC
Electric field at the midpoint Odue to q^,
-> _ 9x10^ x5xl0~^ xlxlO"^ 1 9 X 10^ X 3 X 10'^
®~ (5V2x10“^)2 P - - _

■ 4te£o ■ “ (0.10)^
= 9 N, along OB
= 2.7xlO^NC'\ along OB
? - 9x10^ x2xl0~^ xlxlO'^ Resultant field at the midpoint Ois
(5V2x10“^)2
£= + £g =(2.7+ 2.7)xl0^

w
= 3.6 N, along OA = 5.4 xlO^ NC"\ along OB.
^ _ 9xl0^x5xl0~^xlxlQ-^ (ii) Force on a negative charge of 1.5 x 10~^C placed at
° (5V2xlO“^)^ the midpoint 0,

Flo
£=(/£= 1.5x10"^ x5.4xl0^
= 9 H along OD
= 8.1 xlO'^N, along OA

ee
->

Clearly, and = - fg The force on a negative charge acts in a direction

Fr
Hence total force on 1 pC charge is opposite to that of the electric field.
1.9. A sysfe;7j has two charges q^ = 25 xl0“^ C a?id
?=£>£> f>F^ qg = -25 X 10"^ C located at points A (0,0, - 15 an) and
for
ur
-*
B (0,0, +15 cm) respectively. What is the total charge and
- Fg = zero N.
electric dipole moment of the system ?
1.7. (a) An electrostatic field line is a continuous curve. Ans. Clearly, the two charges lie on Z-axis on either
ks
That is, afield line cannot have sudden breaks. Why not ? side of the origin and at 15 cm from it, as shown in
Yo

(b) Explain why txvo field lines never cross each other at any Fig. 1.151.
oo

point ? [CBSE D 05, 03 ; OD 14] 2a = 30 cm = 0.30 m, = 2.5 x 10 ^ C


eB

Ans. ((?) Electric lines of force exist throughout the


Z
region of an electric field. The electric field of a charge
decreases gradually with increasing distance from it and -7
r

becomes zero at infinity i.e., electric field caimot vanish = - 2.5 X 10 C B (0, 0, + 15 cm)
ou
ad

abruptly. So a line of force cannot have sudden breaks, it


must be a continuous curve.
Y

(b) If two lines of force intersect, then there would be


O.
two tangents and hence two directions of electric field at
nd
Re

the point of intersection, which is not possible.


1.8. Two point charges =+3 pC and q^ = -3 pC are
Fi

located 20 cm apart in vacuum, (i) Find the electric field at the


midpoint O of the line AB joining the two charges, (ii) If a -7
A (0, 0, - 15 cm)
q,, = 2.5 X 10 C
negative test charge of magnitude IJ x 10" ^ C is placed at the X
centre, find the force experienced by the lest charge.
[CBSE OD 03]
Fig. 1.151
Ans. The directions of the fields E. and £n due to the

charges q^ and ijg at the midpoint P are as shown in Total charge = q^ + ijg = 2.5 x 10 ^ - 2.5 x 10 ^ = 0
Fig. 1.150.
Dipole moment.
(j^ = + 3pC > ^g=-3nC p = y X 2/? = 2.5 X 10 ^ X 0.30
10 cm 10 cm
A O B
= 0.75 xlO"^ Cm
The dipole moment acts in the direction from 6 to A
Fig. 1.150 i.e., along negative Z-axis.
1.82 PHYSICS-XII

1.10. Ayi electric dipole with dipole moment A x 10 ^ Cm is V y


aligned at 30° with the direction of a uniform electric field of ●(D
magnitudes xlO^ NC~^. Calculate the magnitude of the torque
acting on the dipole.
(3)
Ans. Here p = 4 x 10'^ Cm, 0 = 30°, £ = 5 x 10“^ NC
-1

.'. Torque, x = p£sin 0 Fig. 1.152

= 4x10”^ xSxlO^ xsin 30° Ans. Refer to the solution of Problem 9 on page 1.73.

= 10'^ Nm. 1.15. Consider a uniform electric field :


£ = 3 X 10^ / NC~^ (i) What is the flux of this field through a
1.11. A polythene piece rubbed with wool is found to have a
negative charge of 3.2 x 10"^ C. (i) Estimate the number of square of 10 cm on a side whose plane is parallel to the
Y-Z-plane ? (ii) What is the flux through the same square if the
electrons transferred, (ii) Is there a transfer of mass from wool to
ytormal to its plane makes a 60° angle with the X-axis ?
polythene ? [CBSE D 13C)
Ans. (i) Here </ = 3.2 x 10“^ C, e = 1.6x 10-19 c
Ans. (f) Normal to a plane parallel to Y-Z plane points
As q = ne, therefore in X-direction, so

Number of electrons transferred, AS =0.10x0.1of m^=0.0lf m


2

3.2 X 10~^ 12
Electric flux.
fi= - = 2x10
(|)£ = £ .^S=3xl0^ f .O.Oli'
-19
e 1.6x10

Since polytliene has negative charge, so electrons are = 30 r.f =30Nm^C"i.


transferred from wool to polythene during rubbing.
(i7) Here 0=60°
(ii) Yes, there is a transfer of mass from wool to
polythene because each electron has a finite mass of
<1)£ = £ AS cos 60° = 3 X 10^ x 0.01 cos 60°
9.1 X10“^^ kg. = 30x|=15 Nm^C'i.
Mass transferred 1.16. Consider a unipnn electric field :

= m^ xn = 9.1 X 10
-31
x2xl0
,12
£ = 3 X 10^ / NC"!. What is the net flux of this field through a
-18 cube of side 20 cm oriented so that its faces are parallel to the
= 1.82x10
coordinate planes ?
Clearly, the amount of mass transferred is negligibly Ans. The flux entering one face parallel to Y-Z plane is
small. equal to the flux leaving other face parallel to Y-Z plane.
Flux through other faces is zero. Hence net flux through
1.12. (a) Two insulated charged copper spheres A and B the cube is zero.
have their centres separated by a distance of 50 cm. What is the
1.17. Carefid measurement of the electric field at the surface
mutual force of electrostatic repulsion if the charge on each is
of a black box indicates that the net outward flux through the
65 X 10“^C ? The radii of A and B are negligible compared to
surface of the box is 8.0 x 10^ Nm^C~^. (i) Wlwt is the net charge
the distance of separation, (b) What is the force of repulsion if inside the box ? (ii) If the net outward flux through the surface of
each sphere is charged double the above amount, and the the box were zero, could you conclude that there were no charges
distance between them is halved ? inside the box ? Why or why 7iot ?
Ans. Refer to the solution of Example 9 on page 1.12.
Ans. (/) (j)^ = 8.0 X 10^ Nm^-^
1.13. Suppose the spheres A and B in Exercise 1.12 have Using Gauss theorem,
identical sizes. A third sphere of the same size but uncharged is
brought in contact with the first, then brought in contact with
^0
the second, and finally removed from both. What is the new 1
force of repidsion between A and B 7 Charge, = 8q . (j>£ = 8.0 X 10^ X 4k X 9 X10
9C
Ans. Refer to the solution of Example 10 on page 1.12.
= 0.07xl0"^C =0.07pC
1.14. Figure 1.152 shows tracks of three charged particles in
(ii) No, we cannot say that there are no charges at all
a uniform electrostatic field. Give the signs of the three charges.
inside the box. We can only say that the net charge inside
Which particle has the highest charge to mass ratio ? the box is zero.
ELECTRIC CHARGES AND FIELD L83

1.18. A point charge +10 pC is a distance 5 an directly 1.21. A conducting sphere of radius 10 cm has an unknown
above the centre of a square of side 10 cm as shown in charge. If the electricifeld 20 cmfrom the centre of the sphere is
Fig. 1.153(a). What is the magnitude of the electricflux through 15 X 10^ NC^ and points radially inward, what is the net
thesquare ? (Hint: Thinkofthe squareas onefaceofa cube with charge on the sphere ?
edge 10 cm) Ans. Electric field at the outside points of a conducting
Ans. We can imagine the square as face of a cube with sphere is
edge 10 cm and with the charge of + 10 pC placed at its
centre, as shown in Fig. 1.153(^).
4n:eQ

^ = 4n8p£r^ = -—^
9x10^
X 1.5 X 10^ X (0.20)^ C
+ q q = 6.67 X 10"^ C = 6.67 nC

w
tn IT) As the field acts inwards, the charge q must be
negative.
q=- 6.67nC.

Flo
10 cm

(«) (b) 1.22. A uniformly charged conducting sphere of 2.4 m


diameter has a surface charge density of 80.0 pC/m^. (i) Find

ee
Fig. 1.153 the charge on the sphere, (ii) What is the total electric flux
leaving the surface of the sphere ?

Fr
[CBSE D 09C]
Symmetry of six faces of a cube about its centre
2.4
ensures that the flux ^ through each square face is same Ans. Here R = = 1.2m
2
when the charge q is placed at the centre.
for
ur
.'. Total flux. CT= SO.OpCm"^ = 80 X 10'^ Cm"^
= 6 X 4^ = — (j) Charge on the sphere is
^0
ks
q = 4n R^ct=4x3.14x(1.2)^ xSOxlO'^C
<K: = -^ = -xl0x 10"^ X 4ji X 9 X 10^
Yo

or
= 1.45x10'^ C.
^ 68o 6
oo

= 1.88x10^ Nm^C"\ (ii) Flux,


eB

= -1 = 1.45 X 10"^ x4;t x9 X 10^


1.19. A point charge of 2.0 pC is at the centre of a cubic
Gaussian surface 9.0 cm on edge. What is the net electric flux = 1.6x10* Nm^ C‘\
r

through the surface ?


ou
ad

Ans. Here = 2.0 p C = 2.0 x 10”^C, 1.23. An inifnite line charge produces a field of
= 8.85xl0"’^C2N"^m-2 9 X 10^ NC * at a distance of 2 cm. Calculate the linear charge
Y

density.
By Gauss's theorem, electric flux is Ans. £ = 9x10** NC'\ r = 2cm=0.02m
nd

2.0 X 10"^
Re

8.85 X 10
-12
= 2.26x10* Nm^C'^ Electric field of a line charge, E = —^
2ncr.r 0
Fi

1.20. A point charge causes an electric flux of -1.0 x 10* .●. Linear charge density,
Nm^C^ to pass through a spherical Gaussian surface of X= 2nE^Er
0
= 2K x
4n x9 xlO
g X 9 X 10'* X 0.02
10.0 cm radius centred on the charge, (i) If the radius of the
Gaussian surface were doubled, how much flux would pass = 0.01x10^* Cm"* =0.1pCm"*.
through the surface ? (ii) What is the value of the point charge ?
1.24. Two large, thin metal plates are parallel and close to
Ans. (i) = -10* Nm^”^ because the charge each other. On their inner faces, the plates have surface charge
enclosed is the same in both the cases.
densities of opposite signs and of magnitude 17.0x10“^ Cm~^.
(ii) Charge, What is £ (a) to the left of the plates, (b) to the right of the plates,
- Eg and (c) between the plates ?
Ans. Here a = 17.0 x 10“^ Cm“^
g-x(- 1.0x10*)
4n X 9 X lO’
(a) On the left, the fields of the two plates are
= - 8.84 X 10"^ C = - 8.84 nC. equal and opposite, so £ = Zero.
PHYSICS-XI!

(b) On the right, the fields of the two plates are


equal and opposite, so £ = Zero,
(c) Between the plates, the fields due to both
plates are in same direction. So the resultant
field is
O’
£ = —+ — = 17x10"^ x4nx9x 10^
2e
0 2eq Sq
= 19.2x10"^° NC"^
1.25. An oil drop of 12 excess electrons is held stationary
under a constant electric field of255 x in Millikan's
oil drop experiment. The density of the oil is
1.26 g cm~^. Estimate the radius of the drop. (g~ 9.81 ;
e=1.60xl0"^^O
Ans. Force on the oil drop due to electric field
= qE= neE
Weight of oil drop
4
** 3
= mg = volume x density x g = -

The field £ must act vertically <1^


downward so that the negatively
charged oil drop experiences an
upward force and balances the weight 0 ^
of the drop.
mg
When the drop is held stationary.
Fig. 1.154
Weight of oil drop
= Force on the oil drop due to electric field
1/3
43 _ 3neE
or - 7T r pg = neE r =

4npg
Now n= 12, e = 1.6 X 10"^^ C, Figure 1.155(c) is right because it satisfies all the
properties of lines of force.
£ = 2.55 X lO'* Vm"^, g = 9.81 ms“^
Figure 1.155(d) is wrong because lines of force cannot
p = 1.26g cm"^ = 1.26 xlO^ kg m’^ intersect each other.
-1I/3
3x12x1.6x10 x2.55 X lO'*
-19
Figure 1.155(c) is wrong because electrostatic field
r -
4 X 3.14 X 1.26 X 10^ X 9.81 lines cannot form closed loops.
1.27. In a certain region of space, electric field is along the
-1I/3
9 X 16 X 255 -15 Z-direction throughout. The magnitude of electric field is,
xlO
314x126x981 however, not constant but increases unipnnly alo)ig the positive
Z-direction at the rate of 10^ NC~^m~ . What are the force and
= (9.46x10"^)^^^ xlO"^ torque experienced by a system having a total dipole moment
= 0.0981 X 10”^ m = 9.81 xlO"^ mm.
equal to 10“^ C m in the negative Z-direction ? [AllMS 18]
Ans. The situation is shown in Fig. 1.156.
1.26. Which among the curves shown in Fig. 1.155, cannot
As the electric field changes uniformly in the positive
possibly represent electrostatic field lines ? Z-direction, so
Ans. Only Fig. 1.155(c) is right and the remaining
figures cannot represent the electrostatic field lines. = + 10^ NC-^m’^, d£. = 0, = 0
dz dx
Figure 1.155(a) is wrong because field lines must be
normal to a conductor. As the system has a total dipole moment in the
negative Z-direction, so
Figure 1.155(b) is wrong because lines of force cannot
start from a negative charge. p^ = -10“^Cm, p^=0, p^=0
ELECTRIC CHARGES AND FIELD

Gaussian
surface
n
-‘J

E
I

V
B IM
\ A

Fig. 1.158

+ <? By Gauss's theorem.


X
= ) £ . dS =
Total charge = 0

w
Fig. 1.156
i.e., the total charge enclosed by the Gaussian surface must
In a non-uniform electric field, the force on the dipole be zero. This requires a charge of - </ units to be induced
will be
on inner surface of conductor A. But an equal and

Flo
at opposite charge of + units must appear on outer surface
+1 dz
By A so that charge on the surface of A is Q + ty.

ee
= 0-h0-10"’'xl0^=-10"^N Hence the total charge on the surface of A is Q + <j.

Fr
{Hi) The instrument should be enclosed in a metallic
The negative sign shows that the force on the dipole case. This will provide an electrostatic shielding to the
acts in the negative Z-direction. instrument.

for
ur
As the dipole moment p acts in the negative 1.29. A hollow charged conductor has a tiny hole cut into its
Z-direction while the electric field E acts in the positive surface. Show that the electric field in the hole is G n, where ii
Z-direction, so 9 = 180°. 2e
0
ks
Torque, r = pEsin 180° = p£ x 0 = 0. is the U7iit vector in the outward normal direction, and a is the
Yo

1.28. (i) A conductor A with a cavity [Fig. 1.157(a)] is surface charge density near the hole.
oo

given a charge Q. Show that the entire charge must appear on Ans. Consider the charged conductor with the hole
filled up, as shown by shaded portion in Fig. 1.159.
eB

the outer surface of the conductor,


Applying Gauss's theorem, we find that field just outside
(ii) Another conductor B with charge q is inserted into
the cavity keeping B insulated from A. Show that the total is — n and is zero inside. This field can be viewed as the
r

^0
charge on the outside surface of A is Q+ q [Fig. 1.157(b)].
ou

superposition of the field due to the filled up hole plus


ad

(Hi) A sensitive instrument is to be shielded from the strong


Y

electrostatic fields in its environment. Suggest a possible way. B

£2
nd
Re

E.
Fi

'' Ej
A

E)

Fig. 1.157
Fig. 1.159

Ans. 0) Refer answer to Q.25(6) on page 2.25. the field due to the rest of the charged conductor. Since
(ii) Consider a Gaussian surface inside the conductor inside the conductor the field vanishes, the two fields

but quite close to the cavity. must be equal and opposite, i.e.,
Inside the conductor, E = 0. ^-^ = 0 ...{!)
L86 PHYSICS-Xtl

And outside the conductor, the fields are added up ; 1.32. (a) Consider an arbitrary electrostatic field configu
_ r, a ration. A small test charge is placed at a null point (i.e., where
^ + ^=- ...(2)
8
0 ^ = 0) of the configuration. Shoxu that the equilibrium of the
Adding equations (1) and (2), we get test charge is necessarih/ inistable.
a o
2J- =- or
Ei = (b) Verify this result for the simple configuration of two
^0 2eo charges of the same magnitude and sign placed a certain
Hence the field due to the rest of the conductor or the distance apart.
field in the hole is
Ans. (fl) We can prove it by contradiction. Suppose the
o
E = jj test charge placed at null point be in stable equilibrium.
2e
0 Since the stable equilibrium requires restoring force in all
where n is a unit vector in the outward normal direction.
directions, therefore, the test charge displaced slightly in
any direction will experience a restoring force towards the
1.30. Obtain the formida for the electric field due to a long null point. That is, all field lines near the null point should
thin wire of uniform linear charge density k without using be directed towards the null point. This indicates that
Gauss's law.
there is a net inward flux of electric field through a closed
[Hint. Use Coulomb's law directly and evaluate the surface around the null point. But, by Gauss's law, the
necessary integral.] flux of electric field through a surface enclosing no charge
Ans. Refer to the solution of Example 47 on page 1.36. must be zero. This contradicts our assumption. Hence the
1.31. It is 7tow believed that protons and treutroiis are test charge placed at the centre must be necessarily in
themselves built out of more elementary units called quarks. A unstable equilibrium.
proton and a neutron consists of three quarks each. Two types of (b) The null point lies on the midpoint of the line
quarks, the so called ‘up’ quark (denoted by u) of charge + (2/3) e, joining the two charges. If the test charge is displaced
atid the 'down' quark (denoted by d) of charge (-1/3) e, together slightly on either side of the null point along this line, it
with electrons build up ordmary matter. Suggest a possible will experience a restoring force. But if it is displaced
quark composition of a proton and neutron. normal to this line, the net force takes it away from the
Ans. Charge on 'up' quark (h) = + | ^ null point. That is, no restoring force acts in the normal
direction. But stable equilibrium demands restoring force
Charge on 'down' quark (d) = -~e in all directions, hence test charge placed at null point will
Charge on a proton = e not be in stable equilibrium.
Charge on a neutron = 0 1.33. Aparticle of mass mand charge (-q) enters the region
Let a proton contain x 'up' quarks and (3 - jr) 'down' between the tivo charged plates initially moving along x-axis
quarks. Then total charge on a proton is with speed (like particle 1 in Fig. 1.152). The length of plate is
ux + d (3 - x) = e L and a uniform electric field E is maintained between the
or
plates. Show that the vertical deflection of the particle at the far
2 a: edge of the plate is qEIr/{2m v^).
or - AT - 1+ - = 1
3 3 Compare this motion with motion of a projectile in
or .r = 2 and 3-a:=3-2=1 gravitational field.
Thus a proton contains 2 'up' quarks and 1 'down' Ans. The motion of the charge - in the region of the
quark. Its quark composition should be : uud. electricfield Ebetweenthe two charged plates is shown in
Let a neutron contain y 'up' quarks and (3 - y) 'down' Fig. 1.160.
quarks. Then total charge on a neutron must be
-H
uy+d(3-y) = 0 + + + + + + + + + +

or
|ey--le(3-y) = 0 y

or ^ -1+^=0
3^ 3

or y=l and 3-y = 3-l=2


Thus a neutron contains 1 'up' quark and 2 'down'
quarks. Its composition should be : udd. Fig. 1.160
ELECTRIC CHARGES AND FIELD 1.87

Force on the charge - in the upward direction is plates separated by 0.5 cm is 9.1 x 10^ N/C, where will the
electron strike the upper plate ?
ma = c\E (1 e i = 1.6 X 10"^’ C, = 9.1 X 10"^^ kg).
:. Acceleration,
qE
a = — Ans. Here i/ = 0.5 cm = 0.5 x 10' ^ m,
m
= 2.0x10^ ms■^ E = 9.1xl0^ NC'\ 1 = 7
Time taken to cross the field, t = — From the above exercise, the vertical deflection of an

electron is given by
Vertical deflection at the far edge of the plate will be eEl}
y =

y= ut + ^at^ = ,,0+ -1 fl£ =


qEl}
2 2 m
Imv] j2
eE
Like the motion of a projectile in gravitational field, 2 X 0,5 X10'^ x 9.1 X 10'^^ X 4 X 10^^
the path of a charged particle in an electric field is

w
1.6x10'^^ X 9.1x10^
parabolic. -4
= 2.5 X 10
1.34. Suppose that the particle in Exercise 1.33 is an electron
projected with velocity = 2.0 x 10^ ms~^. If E between the or L= 1.58x10'^ m =1.6 cm.

Flo
ee
OTQs : Objective Type Questions

Fr
■Type A : Multiple Choice Questions (i mark each)
for
ur
Based on Frictional Electricity, Electrostatic induction (fj) conservation of mass
and Basic Properties of Charges (c) conservation of energy
ks
1. Two charged spheres are separated by 2 mm. (d) quantisation of charge
Yo

13
Which of the following would yield the greatest
oo

5. A body has a charge of -2 pC. If it has 2.5 x 10


attractive force ?
protons, then how many electrons the body has ?
eB

(fl) +2q and -2q (b) + 2q and + 2q


(fl) 1.25x10
,13
(1?) 2.5xltf^
(c) ~2q and -2q (d) ~\q and -4q 13
(d) none of these
(c) 3.75x10
2. Consider an uncharged conducting sphere. A
r

6. In the process of charging, the mass of the


ou

positive point charge is placed outside the sphere. The


ad

negatively charged body


net charge on the sphere is then, [CI3SE SP 221
(6) decreases
Y

(fl) increases
(0 negative and uniformly distributed over the
surface of sphere (c) remains constant
(d) is not related to the charging process
nd

(ii) positive and uniformly distributed over the


Re

surface of sphere Based on Coulomb's Law


Fi

(in) negative and appears at a point on the surface of


the sphere closest to point charge 7. According to Coulomb's law, which is the correct
relation for the following figure ? (CBSE SP 23]
(iv) zero
3. A comb run through one's dry hair attracts small
^ 12 hi
bits of paper. This is due to
(fl) comb is a good conductor (fl) 0 (^) ‘ii'72 < ic)q^q2=0 {d)l>-i->Q
(b) paper is a good conductor
(c) the atoms in the paper get polarised by the 8. Two point charges placed in a medium of
charged comb dielectric constant 5 are at a distance r between them,
(d) the comb possesses magnetic properties experience an electrostatic force 'F. The electrostatic
4. Charge on a body is integral multiple of ±e. It force between them in vacuum at the same distance r
will be [CBSE SP 22]
implies the law of
(fl) conservation of charge (0 5 F (ii) F {in) f/2 (iv) f/5
PHYSICS-Xil

9. For coulomb force to be operative the least size of (a) 1.64 X 10'^^ N, 2.4 x 10"^^N
atom will be
-12 -15
(fc) 1.64x10'“^ N, 1.5xlO^N
-10 -8
(a) 10 m (b) 10 m (c) 10 id) 10 (c) 1.56 X 10“^® N, 2.4 X 10“^^N
m m

10. Two charges of equal magnitudes and at a (d) 1.5x10^ N, 2.4xlO“^^N


distance r exert a force F on each other. If the charges
15. A proton moving at constant velocity enters the
are halved and distance between them is doubled, then
region between two charged plates, as shown below.
the new force acting on eacli charge is A
F F
(«)t (c) 4 F
8 16
B

Based on Forces between Multiple Charges :


P
The Superposition Principle
D
11. Four charges are + (1 + 2q
arranged at the comers of a A B

square ABCD as shown in the E


O
figure. The force on the charge
Which of the paths shown correctly indicates the
kept at the centre O is
proton's trajectory after leaving the region between the
(i7) zero D C charged plates ?
{b) along the diagonal BD -2q + 4
(fl) A (&) B (c) C (d) D
(c) along the diagonal AC
16. The electric field required to keep a water drop
{d) perpendicular to side AR of mass'm' just to remain suspended, when charged
12. Four charged particles are placed at the vertices with one electron, is
of square as shown in the figure. An electron that is em
■(a) ms (b) (c) emg id) —
free to move is placed at the B
e 8
a
exact centre of the square. In A' B
which direction will the 17. A charge of magnitude 3e and mass 2 m is
electron move ?
moving in an electric field £. The acceleration
(a) It will move toward A imparted to the charge is
(b) It will move toward B 2Ee 3Ee 2m 3 m
D C (fl) (b) (c) (d)
(c) It will move toward C P
a 3 m 2m 3Ee 2Ee

(d) It will move toward D 18. A particle of mass m and charge q is placed at
13. Four charges as shown a
rest in a uniform electric field £ and then released, the
Q kinetic energy attained by the particle after moving
in figure are placed at the ^ a
comers of a square of side distance y, will be
length a. What is the ratio of n a
(a) (b)qEy {c)qE\j id)qEx/
(Q/ q) if net force on Q is zero ? 19. A charged particle of mass m and charge q
1 Q
(fl) (b) -2V2
a
initially at rest is released in an electric field of
2V2 magnitude £. Its kinetic energy after time t will be
2E^t^ c2„2.2
(^)4
V2
(«)
t q t
(b)
mq 2m

Based on Relation between Electric Field, (c)


Eq^m (rf)
Eqm
21
Electric Charge and Electric Force
14. A charged cloud system produces an electric 20. A small object with charge q and weight mg is
field in the air near the earth's surface. A particle of attached to one end of a string of length ®'L' attached
charge -2xlO“^Cis acted on by a downward electro to a stationary support. The system is placed in a
static force of 3x 10“^ N when placed in this field. The uniform horizontal electric field '£', as shown in the
gravitational and electrostatic force, respectively, accompanying figure. In the presence of the field, the
exerted on a proton placed in this field are string makes a constant angle 6 with the vertical.
ELECTRIC CHARGES AND FIELD 1.89

(a) electric force {b) electric field intensity


k (c) pressure (rf) electric charge
L
26. Four metal conductors having different shapes:
1. a sphere 2. cylinder
3. pear 4. lightning conductor
mg
are mounted on insulating stands and charged. The
one which is best suited to retain the charges for a
The sign and magnitude of q will be [CBSE SP 22] longer time is
(a) I {b)2 (c)3 (d)4
(fl) positive with magnitude 7tig / E
(^) positive with magnitude {mg / E)tan0 Based on Dipole Moment, Dipole Field
(c) negative with magnitude 7tig/ Etan0 and Torque on a Dipole

w
-10
(rf) positive with magnitude £tan0/ mg 27. Two equal and opposite charges of 2 x 10 C

are placed at a distance of 1 cm forming a dipole and


Based on Electric Fields of Point Charges
are placed in an electric field of 2xlO^N/C. The

Flo
21. Deuteron and alpha particle in air are at maximum torque on dipole is
separation 1 A. The magnitude of electric field
(fl) 2V2xlO“^ Nm (b) 8x10^ Nm

ee
intensity on a-particle due to deuteron is
(fl) 5.76x10” N/C (b) 144x10^* N/C (c)4xlQ-^ Nm (rf)4xl0"^ Nm

Fr
(c) 2828x10” N/C (d) zero
Based on Electric Field Lines
22. For a point charge, the graph between electric
28. Out of the following is not a property of field
field £ versus distance r is given by for
ur
lines
(«) (b) E4
(a) Field lines are continuous curves without any
ks
breaks
Yo

(b) Two field lines cannot cross each other


oo

(c) Field lines start at positive charges and end at


eB

(0 E negative charges
(d) They form closed loops
29. Electric field lines contract lengthwise, it shows
r

(a) repulsion between same charges


ou
ad

(1j) attraction between opposite charges


23. Two charges + 5 pC and +10 pCare placed 20 cm
Y

(c) no relation between force and contraction


apart. The electric field at the midpoint between the
two charges is (d) electric field lines do not move on straight path
nd
Re

(fl) 45 X10^ N / C towards + 5pC Based on Gauss's Theorem


Fi

(b) 13.5 X10^ N / C towards + 5pC 30. Which quantity is a vector quantity among the
(c) 45 X10*’ N / C towards + lOpC following ?
(a) Electric flux (b) Electric charge
(d) 13.5 X10^ N / C towards + lOpC
(c) Electric field (ri) Electric potential
24. Two small charged spheres A and B have
31. Gauss's law is valid for
cliarges 10 pC and 40 pC respectively, and are held at a
separation of 90 cm from each other. At what distance (a) Any closed surface
from A, electric intensity would be zero ? {b) Only regular closed surfaces
(rt) 22.5 cm (b) 18 cm (c) 36 cm (d) 30 cm (c) Any open surface
(d) Only irregular open surfaces.
Based on Continuous Charge Distributions
32. Which statement is true for Gauss law ?
25. If o= surface charge density, e- electric
(fl) All the charges whether inside or outside the
permittivity, the dimensions of — are same as Gaussian surface contribute to the electric flux.
c
PHYSICS-XII

(f>) Electric fiux depends upon the geometry of the 39. Above an infinitely large plane carrying
Gaussian surface, charge density a, the electric field points up and is
CT
(c) Gauss theorem can be applied to non-uniform equal to . What is the magnitude and direction of
electric field, 2e
0

(tf) The electric field over the Gaussian surface the electric field below the plane ?
remains continuous and uniform at every (fl) a / 28g, down (b) a/2£g,up
point. [CBSH SP 221
(c) a/ Eg, down (d) o/£g,up
33. Four charges +8C, -3C, +5C and -IOC are kept 40. Two parallel large thin metal sheets have equal
inside a closed surface. What will be the outgoing flux surface densities 26.4x 10~^^C/m^ of opposite signs.
through the surface ? The electric field between these sheets is
-16
(fl) 26 Vm (b) 0 Vm (c) 10 Vm {d) 8 Vm (fl) 1.5 N/C (b) 1.5 X10 N/C
34. The electric charges are distributed in a small (c) 3 X10'^^^ N/C (d) 3 N/C [CBSE SP 22]
volume. The flux of the electric field through a 41. A conducting sphere of radius R=20cm is
spherical surface of radius 10 cm surrounding the total
given a charge Q = 16 gC. What is E at its centre ?
charge is 20 Vm. The flux over a concentric sphere of
radius 20 cm will be (rt) 3.6xlO^N/C (b) 1.8xlO^N/C
{(?) 20 Vm (b) 25 Vm (c) 40 Vm (tf) 200 Vm (c) Zero (d) 0.9xl0^N/C
35. A soap bubble (or a neutral balloon) is given 42. A spherical shell of radius R has a charge + q
negative charge, its radius will units. The electric field due to the shell at a point
(fl) increase (b) decrease
(fl) inside is zero and varies as r~^ outside it
(c) remain unchanged (d) fluctuate
(b) inside is constant and varies as r outside it
36. A cylinder of radius rand length / is placed in a (c) inside is zero and varies as r~^ outside it
uniform electric field parallel £ to the axis of the
cylinder. The total flux for the surface of the cylinder is (d) inside is constant and varies as r~^ outside it
given by ICBSE SP 22] 43. A point charge is kept at the centre of a metallic
(fl) zero (c) Enr^ (d) lEnr'^ insulated spherical shell. Then :
{a) electric field outside the sphere is zero
Based on Applications of Gauss's Theorem
(b) electiic field inside the sphere is zero
37. Charge on a conducting metal sphere is present
(c) net induced charge on the sphere is zero
(fl) on the surface of sphere
(b) inside the sphere (d) electric potential inside the sphere is zero.
(c) outside the sphere 44. What is the net charge on a conducting sphere
of radius 10 cm ? Given that the electric field 15 cm
(d) both inside and outside of sphere
from the centre of the sphere is equal to 3 x 10^ N/C and
38. According to Gauss law, electric field of an is directed inward.
infinitely long straight wire is proportional to {fl)-7.5xlO“^C (b)-7.5xlO'^C
1
{a)r (b) 4
r‘
M4
r
(d)-
r
(c) 7.5 X10'^ C (d) 7.5xlO"^C

Answers and Explanations


1. (f?) Only+2and -2i/charges will exert attractive 3. (c) The comb run through dry hair gets charged.
force. The charged comb polarises the atoms of the paper.
Hence the small bits of paper get attracted by the
2. (m) The point charge +Q induces equal and
charged comb.
opposite charges on opposite ends of the spherical 4. (d) Quantisation of charge means the charge on a
conductor. The net charge on the conductor is zero.
body is an integral multiple of ±e.
5. (i:) Let be the number of electrons inside the
+4 -4 © body. Then
+Q
q=n^e +n^e
ELECTRIC CHARGES AND FIELD

-2 X 10"^ = -n X 1.6 X10"^^ +2.5 x 10^^ x 1.6 x 10"^^


S ■

e 17. (/’) Acceleration,


Force 3ex E 3Ee
= 3.75x10^^ a =
Mass 2m 2m

6. (n) Tlie mass of the negatively charged body 18. (1)) Increase in K.E. of the charged particle
increases due to gain of electrons.
= Work down on the particle by
7. (<7) As the forces are attractive, and q2 have the electric field

opposite charges. So q^q^ <0. = qExy = qE\j.


8-O') Pvac=^.ne,=^^‘ 19. (b) F ~ ma = Eq or a =
m
9. (n) The distance must be greater than the nuclear
size (alO'^^m). For r<10“^^m, the much stronger V = u + at ==0 +
Eq . t
nuclear force makes the coulombic force ineffective. m

w
‘1^
10. (d) Original force, F = I 2 1
K.E. = —mu = —m ' ■y -
2 2 nr 2m
‘1 <7
2 2 _
1

Flo
New force. f'=it F.
{2rf 16 20. (1;) ,—

I
TsinG =
11. (b) At the centre O, the forces due to the

ee
charges at A and C cancel out. The forces due to the T cos 0 = mg

Fr
T
charges at Band Dadd up along BD. The total force j k T cos G
qE
is along BD. tan 0 = 0
mg (I
12. (rf) Net force on the electron will be zero along
for
ur
T sin 0
the diagonal AC. It will move towards D due to mg tan0
q =
attraction by the proton. 1 £ ▼ '"8

13. (/)) Net force on charge Q = 0


ks
As the object is deflected towards the direction of £,
Yo

its charge q is positive.


oo

+ k
QQ -19

■■ V a
21. (t) Charge on a deutron =+e = 1,6 x 10 c
eB

or
^I^kqQ ,,QQ £ = ^—■4
47C8n r 0
a~
-19
r

1.6x10
■^ = 2^/2. =9xl0^x
ou
ad

or
(1x10^^°)“
Y

Q = 1.44x10”NC"\
Q and q must have opposite signs, so = -2y/2.
4 1
22. (1?) For a point charge, £ oc
nd

_ F^ 3xlQ-^N
Re

14. (/7) £=- = 1.5xl0^NC'^


q 2xlO“^C .●. £-rgraph is a rectangular hyperbola.
Fi

Electrostatic force on a proton 23. (n) At the midpoint, the fields of the two charges
= eE = 1.6 X 10“^^ X 1.5 X 10^ = 2.4 x 10“'^N are in opposite directions. The resultant field is
directed from larger charge to smaller charge.
Gravitational force on a proton 1 9(10-5)x 10"^
E = = 9x10
= mg = 1.67xl0“^^kgx 9.8 ms“^ 4jie 0 (10xl0“2)2
= 1.64xlO~^^N
= 4.5xlO^NC"\
15. (if) The electric field acts downwards. The
proton will be deflected along trajectory D. 24. (if)

16. (b) Force exerted by electric field Eb


10 |iC 40 nC
= Weight of water drop
A P B
t? £ = mg or £=^. X 0.90-X
e
7.92 PHYSICS-XII

At point P, F = F
36. (i?) As shown in tlie figure.
-6 -6
1 10x10 1 40x10 2
or
= -nr E
■}
4n:e
0 -Y“ 4716q (0.90-a:)^
1 4
or
X ^ (0.90-Y)^ 4i = 0
or 0.90-y = 2y

or X = 0.30 m = 30 cm.
(j
25. (/’) The relation between £, a and e is £ = , 2 E
e 4> = + itr £

26. (rt) The charge spreads uniformly on the sphere. Total electric flux =-7cr^E + 7ur^£ + 0 =0
The surface charge density {o = ql A) will be minimum
on its surface. Hence the sphere is best suited to retain 37. {n) Inside the sphere, £ = 0 ^ q=0
the charges for a longer time. Entire charge is present on the surface.
27. id) T = p£ = qxluK E
max
38. (rf) Electric field of an infinitely long straight wire,
= 2x10
-10
xlxlO'^x2xlO^Nm £ =
X
£qc-.
z.e..
= 4xl0“^Nm. 27uegr r

28. {(i) Electric field lines may not always form 39. (zt) Tlie plane is positively charged. Its field
closed loops. o/2eq must point downwards at a point below it.
+ <7
29. (b) Lengthwise contraction of the field lines
- a
40. (ii)
shows attraction between opposite charges.
^nel ~ ^+o ^-CT
30. (c) Only electric field is a vector quantity.
+

+
£
a a a
31. (/7) Gauss's law is valid for any closed surface. +

2e 0 2e
0 ^0
32. (c) The electric field over the Gaussian surface
+

+
£-a
remains continuous and uniform at every point. All 26.4x10
-12
+

other statements are not valid for Gauss law. -12 +


8.85x10
33. [b) Total charge, r/ = +8-3 + 5-10 =0
+

c: 3 NC'\
= 0 Vm. 41. (c) Electric field at any point inside the
^0
conducting sphere is zero.
34. (rt) The flux through a closed surface depends 1
42. (r) £inside = 0 and E
only on the enclosed charge and is independent of size 'outside
47ieg
and shape of the surface.
43. (1j) Electric field at every point inside the sphere
35. (rt) If a is the surface tension and r the radius of .
IS zero.

soap bubble, then


P
excess
4(j 44. {b) £=— ■4
4Tcen r 0
[r>R]
r

When the bubble is charged, q = -AtiEq. Er^ [Inward field]


4a
Pexcess = P-
electrostatic
+ 3xl0^x(0.15)^ = -7.5xlO"^C
r
9x10^
After electrification, surface tension decreases.

■ Type B : Case Study Based Questions


PARAGRAPH 1 Coulomb's lav/

This law is a quantitative statement of about the force between two point charges. When the linear sizes of
charged bodies are much smaller than the distance between them, their sizes may be ignored and the
charge bodies are called point charges. After retiring from his active services as a military engineer in 1776,
ELECTRIC CHARGES AND FIELD

Coulomb (a French physicist) discovered a torsion balance to measure a


small quantity of force and used it for determination of forces of
attraction or repulsion between small charged spheres. He thus arrived
in 1785 at the inverse square law relation, now known as Coulomb's law. Pi
+‘?i
He found that the force between two point charges varied inversely with
the square of the distance between the charges and was directly r

proportional to the product of the magnitude of the charges and acted ●4

along the line joining the two charges. Coulomb's law is an electrical <7jXi^2

analogue of Newton's law of Universal Gravitation in mechanics. r

QUESTIONS (Answer any four of the following questions)

w
1. Identify the wrong statement in the following, 3. Each of two point diarges is doubled and their
Coulomb's law correctly describes the electric force that distance is halved. Force of interaction becomes n times,
where n is
(a) binds the electrons of an atom to its nucleus

Flo
(b) binds the protons and neutrons in the nucleus of (a) 4 ib)l (c) 18 id) 16
an atom 4. Tlie minimum value of force acting between two
point charges placed 1 m apart from one another is

ee
(c) binds atoms together to form molecules
ke ke^
(ri) binds atoms and molecules to form solids. (fl) ke^

Fr
(b) ke
2. Two charges 3xlO“^C and 5xlO'*C are placed at a
distance 10 cm from each other. The value of electrostatic
5. A and Bare two identical spherical charged bodies
which repel each other with force f, kept at a finite distance.
for
ur
force acting between them is
A third uncharged sphere of the same size is brought in
(a) 13.5 X10” N {b) 40x10"N contact with sphere B and removed. It is then kept at
(c) ISOxlO^N (d) 13.5xl0'°N midpoint of A and B. Find the magnitude of force on C
ks
(fl)F/2 {b)F/8 {c)F (rf) Zero
Yo
oo

PARAGRAPH 2 Electric dipole


eB

The electric field due to a charge configuration with total charge zero is not zero, but for distances large
compared to the size of the configuration, its field falls off faster than 1/ typical of the field due to a
r

single charge. An electric dipole is the simplest example of this fact. An electric dipole is a pair of equal and
ou
ad

opposite charges + q and -q separated by some distance 2a. Its Hydrogen


Y

dipole moment vector p has magnitude 2qa and is in the Oxygen /


- i I

direction of the dipole axis from -qto+q. The electric field of


nd
Re

the pair of charges can be found out from Coulomb's law and / \
Fi

the superposition principle. The magnitude and the direction of Negative


1
I !
P
side
the dipole field depend not only on the distance r but also on \
>

\
/ Positive
the angle between the position vector r and the dipole moment side

_ - 7

p . In some molecules, like H2O, the centres of -^e charges and /


I
I
I
\
of +ve charges do not coincide. So they have permanent dipole
moment. Such molecules are called polar molecules. Hydrogen

QUESTIONS (Answer any four of the following questions)


6. What will be the value of electric field at the centre (i?) Equal to the electric field due to one charge at centre
of the electric dipole ? (c) Twice the electric field due to one charge at centre
(a) Zero (d) Half the value of electric field due to one charge at
centre
L94 PHYStCS-XII

7. If r is the distance of a point from the centre of a 9. An electric dipole of moment p is placed in a
short dipole, then the electric field intensity due to the
short dipole remains proportional to uniform electric field E. Tlie maximui^i torque
{a)r^ (b) (c) r-2 (rf) experienced by the dipole is

8. An electric dipole coincides on Z-axis and its {a)pE {b)plE {c)EJp {d)p.E
midpoint is on origin of the coordinate system. The 10. The frequency of oscillation of an electric dipole
electric field at an axial point at a distance z from origin is
having dipole moment p and rotational inertia I,
£. and electric field at an equatorial point at a distance y
oscillating in a uniform electric field E, is given by
from origin is E^. Here z = 1/» a, so 1-^ is equal to
l£,l ia)iV2K)^I/pE {h){\lliz)4pEI I
(a) I {b)4 (c)3 id) 2 (c) (2::) Vp£/ f {d){2n)^H pE
PARAGRAPH 3 Gauss's theorem

The term electric flux implies some kind of flow. Flux is the property of any vector field. Electric flux is a
property of Electric field. It is equal to the product of the given area and the normal component of the
electric field through it. Gauss's theorem gives a relationship between the total flux passing through any
closed surface S and the charge q enclosed within the surface. It states that the total flux through a closed
surface is 1 / Cq times the net charge enclosed by the surface.
Mathematically, (j)^ = E .dS = —■ , Cq = permittivity of free space
s ^0

Gauss's theorem is quite useful in calculating the -electric field in problems


where it is possible to choose a closed surface such that the electric field E has
a normal component which is either zero or has a single fixed value at every
point on the surface. Symmetry considerations in many problems make the
application of Gauss's theorem much easier. The closed surface we choose (having symmetry consideration
in view) to solve a given problem is called Gaussian surface. Gauss's theorem is based on inverse square
dependence on distance contained in Coulomb's law. Any violation of Gauss's theorem will reflect a
deviation from the inverse square law.

QUESTIONS (Answer any four of the following questions)


11. What is the SI unit of electric flux ? 14. The electric flux for Gaussian surface A that

N N encloses the charged particles in free space is


(")c
xm
2 (&)Nxm^ (c)^xC
m
id) m
Gaussian
surface A

12. If < I £ . dS = 0, inside a surface, that means there is

(fl) no net charge present inside the surface Gaussian

(1)} uniform electric field inside the surface surface B

(c) discontinuous field lines inside the surface


(Given c/j=-14nC, ij^=~56nC)
(d) some charge present inside the surface
(b) lO^CN'^m"^
-1
(a) 10^ Nm^
13. For a given surface the Gauss's law is stated as (d) 6.32x10^
(c) 6.32x10^ Nm^C
-1

E .dS =0. From this we can conclude that


15. Charge q is first kept in a sphere of radius 5 cm and
(a) E is necessarily zero on the surface then it is kept in a cube of side 5 cm. The outgoing flux
(b) E is perpendicular to the surface at every point (fl) will be more in case of sphere
(&) will be more in case of cube
(c) the total flux through the surface is zero
(c) will be same in both cases
(d) the flux is only going out of the surface
(d) cannot be determined
ELECTRIC CHARGES AND FIELD

Answers \"

1. (b) Coulomb's law applies to only charged particles. 2p


8. (d) For z » a,
Coulomb’s electrostatic force is not responsible for 4ner,z^
0

binding protons and neutrons inside a nucleus. P


9xl0^x3xl0"^x5xl0^
For y » a, I£,l =
2. (a) F =
(10xl0"^f
For z = y » a, —^ = 2.
= 13.5x10“ N. I£,l
1 ?1^2 9. (a) X max = pEsin90° = pE.
3.(rf)
4jreQ
10. (/j) Restoring torque for small 0,
T =-pEsinG =-p£0 [sin0-0]

w
r = = 16F «=16.
4^0* {r/2f or /a = -p£0
4. (f?) The value of force will be minimum between pE
a = - 0 i.e., a oc 0
/
two electrons placed one metre apart.

Flo
kex e to 1 pE
F = =kc'.
27t 2n I

ee
5.(c) <?
11. (f?) SI unit of electric flux

Fr
0 A
r 0 =unit of Ex unit of S =NC”^ X m
2 _Nm^
B C

Original force of repulsion, F=k^ for


ur
12. ((7) Inside a closed surface, E =0
(1
B q/2 Pa ‘7/2 E .dS =-i-=0 => ^=0
ks
^0
O0 0 0
Yo

r/2 r/2 No net charge is present inside the closed surface.


oo

A C B
c

Net force on C, 13. (c) Integral £ . dS gives flux through a surface.


eB

So option (c) is correct.


M _ ‘/net _(-14 + 78.85-56)xl0~^
14. (rt)
r

-12
*=0 8.85x10
ou
ad

= ef5^[2-l] = F. 8.85x10"^
-12
= 1000 Nm-C"'.
Y

8.85x10
6.(c) At the centre of an electric dipole, electric field
15. (c) The flux through a closed surface depends
is twice the electric field due to each charge.
nd

only on the enclosed charge and is independent of size


Re

7. (d) At far away points of a dipole, £ cc 4r. and shape of the surface.
Fi

■ Type C : Assertions and Reasons


DIRECTIONS

In the following questions, a statement of Assertion (A) is followed by a statement of Reason [R].
Mark the correct choice as :

(a) If both assertion and reason are true and reason is the correct explanation of the assertion.
{b) If both assertion and reason are true but reason is not the correct explanation of the assertion,
(c) If assertion is true but reason is false. (d) If both assertion and reason are false.

1. Assertion If the bob of a simple pendulum is kept in Reason. If bob is charged and kept in horizontal
a horizontal electric field, its period of oscillation will electric field, then the time period will be decreased.
remain same. [AIIMS 12]
1.96 PHYSICS-XII

2. Assertion Acceleration of charged particle in 5. Assertion. The coulomb force is the dominating
non-uniform electric field does not depend on velocity of force in the universe.

charged particle. Reason. The coulomb force is weaker than the

Reason. Charge is an invariant quantity. Tliat is the gravitational force. ]A[IMS 03)

amount of charge on particle does not depend on frame of 6. Assertion. The tyres of aircrafts are made slightly
reference. [AIIMS 17] conducting.
3. Assertion Net electric field inside a conductor is Reason. If a conductor is connected to the ground, the
zero. extra charge induced on the conductor will flow to the
ground.
Reason. Total positive charge equals to total negative
7. Assertion. In a non-uniform electric field, a dipole
charge in a charged conductor. [AIIMS 18J
will have translatory as well as rotatory motion.
4. Assertion All the charge in a conductor gets
distributed on whole of its outer surface.
Reason. In a non-uniform electric field, a dipole
experiences a force as well as a torque. [CBSE SP 21]
Reason. In a dynamic system, charges try to keep their
potential energy minimum.

Answers V

1. (c) When the uncharged bob is placed in an 3. (c) Assertion is true but reason is false.
electric field, its time period does not change. Assertion is 4. {a) Both assertion and reason are true and reason is
true.
the correct explanation of the assertion.
For the charged bob, time period increases in an 5. (d) Gravitational force is the dominating force in
electric field because of the increase in restoring force. nature due to its attractive nature. Coulomb force is 10^
Reason is false. times stronger than gravitational force.
6. (a) Any charge developed on the tyres due to

qE
2. (a) a = friction will flow to the ground due to conducting nature
m
of tyres. Tliis avoids electrical sparking.
As E changes, a also changes but it does not depend 7. (rt) Both assertion and reason are true and reason is
on velocity of the charged particle. the correct explanation of assertion.

Text Based Exercises

^YPE A : Very Short Answer Qu estions (i mark each)

1. What is the cause of charging a body ? 7. Is the force acting between two point electric
2. A glass rod is rubbed with silk. What type of charges and q2 kept at some distance in air,
charges do they acquire ? attractive or repulsive when ;
(') 'll 'll > 0 (ii) 'll <l2 < ^ ■ (CBSE 03, 07]
3. Why does an ebonite rod get negatively charged on
rubbing with wool ? 8. What happens to the mass of a body when it is
4. Consider three charged bodies P, Qand R. If Pand positively charged ?
Q repel each other and P attracts R, what is the 9. Usually it is the negative charge that is transferred
nature of the force between Q and R ? when two bodies are rubbed together. Give reason.

5. A positively charged glass rod is brought near an 10. Name any two basic properties of electric charges,
uncharged pith ball pendulum. What happens to n. What do you understand by quantisation of
the pith ball ? electric charges ?
6. When a polythene piece is rubbed with wool, it 12. What is the cause of quantisation of electric
acquires negative charge. Is there transfer of mass charge ?
from wool to polythene ? 13. What do you mean by additivity of electric charge ?
/

A
ELECTRIC CHARGES AND FIELD 1.97 ,

14. Both charge and mass are scalars and hence got the 27. Give the SI unit of electrical permittivity of free r
additive property. However, in adding charges it is space.
not enough to just add the amounts of charges. Why ? 28. Write down the value of absolute permittivity of
15. Is the total charge of the universe conserved ? free space.
16. A glass rod, when rubbed with silk cloth, acquires a 29. Deduce the dimensional formula for the propor
charge of 1.6 x 10"*^ C What is the charge on silk tionality constant k in Coulomb's law.
cloth ?
30. Write the dimensional formula for the permittivity
17. Two insulated charged copper spheres A and B of constant gg of free space.
identical size have the charges and q^ respec 31. What is the force of repulsion between two charges
tively. A tlrird sphere C of the same size but of 1 C each, kept 1 m apart in vacuum ?
uncharged is brought in contact with the first and 32. Two small balls, having equal positive charge q
then with the second and finally removed from both. coulomb are suspended by two insulating strings of
What are the new charges on A and B ? [CBSE F llj equal length I metre from a hook fixed to a stand.

w
18. Two identical conducting balls A and B have The whole set up is taken in a satellite into space
charges -Qand +3 Q respectively. Tlrey are brought where there is no gravity. What is the angle
in contact with each other and then separated by a between the two strings and the tension in each

Flo
distance d apart. Find the nature of the Coulomb string ?
force between them. [CBSE OD 19] 33. Define dielectric constant of a medium in terms of

ee
19. What is the least possible value of charge ? force between electric charges.
[CBSE DOS I1C;F 10]
20. 1 C of charge is equal to charge of n number of

Fr
electrons in magnitude. What is the value of n ? 34. How does the coulomb force between two point
21. Two metallic spheres A and B kept on insulating charges depend upon the dielectric constant of the
medium ? [CBSE OD 05]
stands are in contact with each other. A positively for
ur
charged rod P is brought near the sphere A as 35. In a medium the force of attraction between two
shown in Fig. 1.161.' The point electric charges, distance d apart, is F. What
ks
two spheres are sepa distance apart should these be kept in the same
rated from each other, medium so that the force between them becomes
Yo
oo

and the rod P is 3F?


removed. What will be
36. The force between two charges placed in vacuum is
eB

the nature of charges on F. What happens to the force if the two charges are
spheres A and B ? Fig. 1.161
dipped in kerosene oil of dielectric constant, k = 2?
[CBSE OD 19]
r

37. State the superposition principle for electrostatic


ou

22. A metal sphere is kept on an insulating stand. A


ad

force on a charge due to a number of charges.


negatively charged rod is brought near it, then the [NCERT]
Y

sphere is earthed as shown in Fig. 1.162. On 38. State the basic fact on which the principle of
removing the earthing, superposition of electrostatic forces is based.
and taking the negatively
nd
Re

charged rod away, what


39. At the first sight the principle of superposition of
will be the nature of electrostatic forces appears similar to the additive
Fi

property. But, it is not so. Can you guess the


charge on the sphere ? Ground
important difference ?
Give reason for your
answer.
(CBSE OD 19]
Fig. 1.162 40. A force F is acting between two point charges ^
and ^2- If a third charge q^ is placed quite close to q2,
23. State Coulomb's law of force between charges at what happens to the force between q^ and q^ ?
rest. Express the same in SI units. 41. Two point charges 'q.^' and ‘q^ are placed at a
24. In Coulomb's law, F = k , what are the factors distance'd' apart as shown in the Fig. 1.163. The
●P
on which the proportionality constant k depends ? 'll 42

25. Name and define the SI unit of charge. Fig. 1.163


26. In the relation F = it , what is the value of k in electric field intensity is zero at a point 'P' on the
K
line joining them as shown. Write two conclusions
free space ? that you can draw from this. [CBSE D 14C]
s 42. Two fixed point charges + 4e and + e units are
separated by a distance a. Where should a third
charge q be placed for it to be in equilibrium ?
PHYSICS-XII

■63. What is a point (ideal) dipole ? Give example.


64. How much is the dipole moment of non-polar
molecule ?
IQBSE'GD'Or;] 65. An electric dipole is placed in a uniform electric
43. Define volume charge density at a point. Write its field. What is the net force acting on it ?
SI unit. [GB9E©Q2C,;iF94G|
44. Define surface charge density at a point. Write its SI 66. What is the angle between the directions of electric
unit.
field at any (i) axial point and (n) equitorial point
45. Define line charge density at a point. Write its SI due to an electric dipole ? ;[.CBSE ST OH]
unit.
'67. Write the expression for the torque r acting on a
46. A metallic spherical shell has an inner radius jR, and
outer radius R2. A charge Qis placed at the centre of dipole of dipole moment p placed in an electric
the shell. What will be the surface charge density on field E.
[GBSEIFagj
the (i) inner surface, and (ii) outer surface of the
shell ? fCBSEOD 19) ●68. A dipole, of dipole moment j}, is present in a uni
47. Define electric field at a point. [Punjab 2nOQj
form electric field E . Write the value of the angle
48. Is electric field intensity a scalar or vector quantity ?
Give its SI unit. .(CBaE.D 99C| between p and E for which the torque, expe
49. Write the dimensional formula of electric field. rienced by the dipole, is minimum. [GBSEiTI)'09C]

50. Name the physical quantity whose SI unit is 69. When is the torque on a dipole in a field maximum ?
newton coulomb"^. [GBSEID‘9B| 70. What is the effect of torque on a dipole in an electric
field ?
51. Draw the pattern of electric field around a point
charge (i) > 0 and {ii) q <0. 71. When does an electric dipole placed in a
52. Sketch the lines of force due to two equal positive non-uniformelectric field experiencea zero torque
charges placed near each other. [CBSE D 96C, 03] but non-zero force ?

53. Draw the lines of force of an electric dipole. 72. What is the nature of symmetry of dipole field ?
54, Draw the pattern of electric field lines, when a 73. Will an electric dipole have translational motion
charge -Q is kept near an uncharged conducting when placed in a non-uniform electric field ? Give
plate. (CBSE'D 19] reason for your answer.
55, Do the electric lines of force really exist ? What is 74. Does the torque exerted on a dipole in a
about the field which they represent ? non-uniform field depend on the orientation of the
dipole with respect to the field ?
56. Two point charges q^ and ^2 placed a distance d
apart are such that there is no point where the field 75. What is the charge of a dipole ? |CBSE:D10C]
vanishes. What can be concluded from this ? 76. Define electric flux. ijCBSE 18,18C]
57, A proton is placed in a uniform electric field 77. What is the relation between electric intensity and
directed along the positive x-axis. In which flux ?
direction will it tend to move ? IGBSEiD laq 76. Is electric flux a scalar or a vector ? [CBSE 18]
56, Under what condition will a charged circular loop 79. Give the SI unit of electric flux ?
behave like a point charge in respect of its electric ‘ ,|CB9E.D 13C ;iF 16J
field ?
80. How is electric flux expressed in terms of surface
59. Two concentric spherical shells of radii R and 2R integral of the electric field ?
are given charges Qj and Q2 respectively. The 81. State Gauss theorem in electrostatics.
surface charge densities on the outer surfaces [CBSED f)8C)
are equal. Determine the ratio Qj: Q2.
[GBSET13] 82. Give the SI unit of surface integral E . dS ] of an

s
60. What is an electric dipole ? )OBSE OD 06, U.]
electric field ?
61. Define electric dipole moment. Write its SI unit.
83. What is the direction of an area vector ?
[GBSE OD 08,11 ;T13]
84. What is a Gaussian surface ?
62. Is electric dipole moment a scalar or vector
quantity ? [GBSE 06C ; T 13] 85. What is the use of Gaussian surface ?
ELECTRIC CHARGES AND FIELD 1.99

86. How much is the electric flux through a closed 91. A charge 'cf is placed at the centre of a cube. What is
surface due to a charge lying outside the closed the electric flux passing through the cube ?
surface ?
jGBSE'OD 12]
f?7. Two plane sheets of charge densities + aand - aare ●92. A charge Y is placed at the centre of a cube of side 1.
kept in air as shown in Fig. 1.164. What are the What is the electric flux passing through each face
electric field intensities at points A and B ? of the cube ? IGBSE'QD ig
[CBSEiD 03C| 93. A charge 'cf is placed at the centre of a cube of side 1.
●A
+ <y
What is the electric flux passing through two
opposite faces of the cube ? |CfiSE<QD 12]
●B
- CT 94. A charge QpC is placed at the centre of a cube.
What is the flux coming out from any one surface ?
Fig 1.164 .[GBSE T 10]
■95. Charges of magnitudes 2Q and -Q are located at

w
88. An electric dipole of dipole moment 20 x 10"^ Cm is
enclosed by a closed surface. What is the net flux points (a, o, o) and (4n, o, o). Find the ratio of the
coming out of the surface ? [CB5E D 03] flux of electric field, due to these charges, through
concentric spheres of radii 2n and 8a centered at the

Flo
89. What is the electric flux through a cube of side 1 cm
origin.
which encloses an electric dipole ? [GBSE D 15] [GBSESPllJ

90. How does the electric flux due to a point charge 96. Two charges of magnitudes -3Q and +2Q are

ee
enclosed by a spherical Gaussian surface get located at points (a,0) and (4fl,0) respectively. What

Fr
affected when its radius is increased ? is the electric flux due to these charges through a
(GBSE D16] sphere of radius 5a with its centre at the origin ?
fCBSE'ODlB]
for
ur
Answers
T
1. Charging occurs due to the transfer of electrons
ks
12. The basic cause of quantisation of electric charge is
from one body to another. that during rubbing only an integral number of
Yo
oo

2. The glass rod acquires positive charge and silk electrons can be transferred from one body to another.
acquires an equal negative charge. 13. Additivity of electric charge means that the total
eB

3. Electron, being less tightly bound in wool, get charge on a system is the algebraic sum (taking into
transferred to the ebonite rod on rubbing. account proper signs) of all individual charges in
4. Attractive. the system.
r
ou

5. The pith ball is attracted towards the rod, touches 14. Unlike mass, charges are of two different kinds
ad

it -

and then thrown away. positive and negative. While adding charges, we
Y

6. The polythene piece acquires negative charge due have to take into account their proper signs also.
to transfer of material particles like electrons from 15. Yes, charge conservation is a global phenomenon.
nd

wool to it, so there is a transfer of mass from wool 16. To conserve charge, the silk cloth acquires negative
Re

to polythene. charge of 1.6 xl0“*^C.


Fi

7. {«) When > 0/ the force is repulsive


(if) When q^ ^2 < 0/ the force is attractive. 17. New charge on sphere A q'^=^
8. The mass of the body decreases due to deficit of New charge on sphere B, q'j^ =
electrons. 2 4

9. The negatively charged electrons are very light and 18. Repulsive. When brought in contact, the two
loosely bound to the atoms while the positively spheres share charges + Q each and hence repel on

charged protons are tightly bound inside the nuclei. separation.


10. Electric charges are (i) quantised, (//) additive and 19. The least possible value of charge is the magnitude
(Hi) conserved. of the charge on an electron or proton and it is
19
e= 1.6x10 c.
11. Quantisation of electric charge means that the total
charge (q) of a body is always an integral multiple of 20. Number of electronic charges in 1 C,
a basic charge(e) which is the charge on an electron. 1C
,18
Thus 71 = - = 6.25x10
q - ne, where n = 0,±l, +2, ± 3, e 1.6xlO’'X
1.100 PHYSICS-XII

21. Due to electrostatic induction, sphere A will be 38. The principle of superposition of electrostatic
forces is based on the fact that the electrostatic force
negatively charged and sphere B will be positively
charged. between two charges is not affected by the presence
22. Due to electrostatic induction, the sphere will be of other charges.
positively charged. Its negative charge will be 39. In the principle of superposition, the electrostatic
drained away due to grounding. forces are added vectorially, not algebraically.
23. Refer to point 14 of Glimpses. 40. The force between and remains equal to F.
24. The proportionality constant k depends on the 41. (/) The point charges and n^ust be of opposite
nature of the medium between the two charges and nature or signs,
the system of units chosen. (/i) The magnitude of charge ijj must be greater
25. The SI unit of electric charge is coulomb. One than that of charge
coulomb is that amount of charge which repels an 42. Refer to the solution of Example 13 on page 1.13.
equal and similar charge with a force of 9 x lO^N 43. Refer answer to Q. 34 on page 1.34.
when placed in vacuum at a distance of 1 metre 44. Refer answer to Q. 34 on page 1.34.
from it.

26. it = 9xl0^ Nm^"^. 45. Refer answer to Q. 34 on page 1.34.


. 46. A charge - Q is induced on the inner surface and + Q
27. Slunitof Sq =C^N”^m on the outer surface of the shell.
28. Permittivity of free space, Q Q
(0 dinner- = (») = +

4tcR|
-2 outer
eo = 8.85xlO‘^^C^ m
47rR|^
Fr^ MLT 47. The electric field at a point is defined as the electro
29. [k] = = [ML^T'^A"^]. static force per unit positive charge acting on a
(AT)2
vanishingly small test charge placed at that point.
-JO If 1 = — =
{ATf f
■ 4nF' ? [MLT'^L^] Mathematically, E = lim
-»o
% %
31. F = 9xlO^N.
48. Electric field is a vector quantity. Its direction is
32. As the two balls are in the state of weightlessness, same as that of the force on a unit positive test
the strings would become horizontal due to the charge.
force of repulsion. -1
or Vm
SI unit of electric field = NC
.-. Angle between the two strings = 180°
49. [Electric Field]
1 R XT
Tension m each strmg = N■ Force MLT"^ MLT"2
AT
»[MLT'^A"^j.
Charge C
33. The dielectric constant of a medium is the ratio of
50. NC'^ is the SI unit of electric field.
the force between two charges placed some
distance apart in vacuum to the force between the 51. (0 See Fig. 1.74 on page 1.46
same two charges when they are placed the same
(i7) See Fig. 1.75 on page 1.46.
distance apart in the given medium. 52. See Fig. 1.77 on page 1.46.
Fvac 1
34. Fmed 4ed*-K 53. See Fig. 1.76 on page 1.46.
K
54.

35. As F cc
3F__^ or
F d'^ V3

36. Fkerosene
K 2

37. The principle of superposition states that the total


force on a given charge is the vector sum of the
individual forces exerted on it by all other charges,
the force between two charges being exerted in Induced charges
such a manner as if all other charges were absent.

F = F. + FIN
12 + Fj3 + Fig 1.165
ELECTRIC CHARGES AND FIELD 1.101

55, Lines of force do not really exist. These are 73. Yes. In a non-uniform electric field, an electric
hypothetical curves used to represent an electric dipole experiences unequal forces at its ends. The
field. But the electric field which they represent is two forces add up to give a resultant force which
real. gives a translatory motion to the dipole.
74. Yes. In a non-uniform electric field, the field vector
56. The point charges and ^2 equal and
opposite. £ (r*) changes from point to point, either in magni
57. As the proton has a positive charge, it will tend to tude or in direction or both. Therefore, the torque
move along positive .r-axis i.e., along the direction
of the electric field.
^ X £ (^) for a dipole located at V changes
with the change in orientation of the dipole with
58. When the observation point on the axis of the
respect to the field.
circular loop lies at a distancemuch greater than its
75. Zero.
radius, the electric field of the circular loop is

w
76. Electric flux over an area in an electric field repre
similar to that of a point charge.
sents the total number of electric lines of force crossing
59 Q = = 1:4. this area normally. If the normal drawn to the
■ Q2 4n(2Rfa

Flo
surface area AS makes an angle 0 with the field £,
60. An electric dipole is a pair of equal and opposite
then the electric flux through this area is
charges separated by some distance.

ee
61. Electric dipole moment is the product of either = EAS cos 9 = E . AS

Fr
charge q and the vector 2 /T drawn from the - ve
77. See the above equation.
charge to the + ve charge. 78. Electric flux is a scalar.

for
79. SI unit of electric flux = Nm^ C“*.
ur
p =qK.2a
Its SI unit is coulomb metre (Cm). 80. The electric flux (j>£ through any surface, open or
closed, is equal to the surface integral of the electric
62. Electric dipole moment is a vector quantity.
ks
field E over the surface S,
Yo

63. A point dipole is one which has negligibly small


oo

size. In such a dipole, charge 00 and size 2fl -> 0 E.dS


‘E -
in such a way that the product p = q.y2a has a finite
eB

value. Atomic dipoles are point dipoles. 81. Gauss's theorem states that the flux of electric field
64. Zero.
through any closed surface S is 1 / Eq times the total
r

65. Zero.
charge q enclosed by S
ou
ad

66. (i) At any axial point, £ acts in the direction jl. Mathematically,
Y

f a

(if) At any equatorial point, £ acts in the opposite (j)£ = ■> £ .dS = -^^.
s 80
direction of p.
nd

82. SI unit of f £ . dS = Nm^"’.


Re

Hence the angle between the directions of the


83. The direction of an area vector is along the outward
Fi

above two electric fields is 180°.


drawn normal to the surface.

67. T =pxE 84, An imaginary closed surface enclosing a charge is


68. Torque experienced by the dipole is minimum called the Gaussian surface of that charge.
85. By a clever choice of Gaussian surface, we can
when angle between ^ and £ is 0°. easily find the electric field produced by certain
r = p£ sin0° = 0. charge systems which are otherwise quite difficult
69. Torque is maximum when dipole is held perpen to determine by the application of Coulomb's law
dicular to the electric field. and superposition principle.
86. Zero.
70. Torque tends to align the dipole along the direction
a
of the electric field.
87. =0 and Eg =
71. When the dipole is placed parallel to the non- ^0
uniform electric field.
88. Zero, because the net charge on the dipole is zero.
72. The dipole field is cylindrically symmetric. 89. Zero.
L.L02. PHYStCS-Xli

.V
901. Electric flux remains unchanged 94. Flux through each face of the cube.
because the charge enclosed by the Gaussian 1 Q XT 2^'
surface remains same. £ =-.
6 s
—liNmC
0
_ ^ 1 <7
9T.. 4,, = 92. 4>e = 6 8
0 95. <}>(r=2^)^ 2Q = 2 : 1
^{r = 8a) 2Q-Q
95. 4>£ =
-2 ±_1 3-
6 8 3 8 Net charge enclosed _-3Q+2Q_ Q
0 0 96. (t)£ =
^0

i^YPE B : Short Answer Qu estions (2 marks each)


1- State the law of conservation of charge. Give two where p is the dipole moment of the dipole. What
examples to illustrate it.
is the net force experienced by the dipole ? Identify
[Har^'ana 2000 ; Punjab 06C, IOC] two pairs of perpendicular vectors in the
2. How does the speed of an electrically charged expression. [CBSE D 15C]
particle affects its (i) mass and (12) charge ? 13. Define an electric field line. Draw the pattern of the
3. State Coulomb's law of force between two electric field lines around a system of two equal positive
charges and state its limitations. Also define the charges separated by a small distance.
SI unit of electric charge. [Punjab 2003] [CB5E D 03 ; Sample Paper II]
4 Write Coulomb's law in vector form. What is the 14. Define electric line of force and give its two
importance of expressing it in vector form ? important properties. [CBSE DOS]

5 State Coulomb's law in vector form and prove 15. What do electric lines of force represent ? Explain
that
repulsion between two like charges on their basis.
[Pimjab'97C]
^1 ^ “ ^2
16. Define electric flux. Write its SI unit. A charge q
where letters have their usual meanings.
is enclosed by a spherical surface of radius k. If
6. Define electric field intensity. What is its SI unit ? the radius is reduced to half, how would the electric
What is relation betvv'een electric field and force ?
flux through the surface change ?
.T. Derive an expression for electric field intensity at a (CBSE OD'09’; F 16]
point at distance rfrom a point charge q. 17. State and prove Gauss's theorem in electrostatics.
8. Obtain the expression for the electric field at a point 18. Using Gauss's theorem, obtain an expression for
on the equatorial line of an electric dipole. the force between two point charges.
[CBSE D' 19, 19C]
19. Two large parallel plane sheets have uniform
9; Derive an expression for the electric field at a point charge densities +a and -g. Determine the electric
on the axis of an electric dipole of dipole moment field (2) between the sheets, and (ii) outside the
p . Also write its expression when the distance r » sheets. lOSEOD 19]
the length 'o' of the dipole. [CBSE OD 19] 20. State Gauss's theorem in electrostatics. Using this
10. Derive an expression for the torque acting on an theorem, prove that no electric field exists inside a
—>

electric dipole of dipole moment p placed in a hollow charged conducting sphere.


[OSE ID)02, OB’C]
uniform electric field E . Write the direction along 21. Apply Gauss's law to show that for a charged
which the torque acts. PCBSE OD 19] spherical shell, the electric field outside the shell is,
111'.. What is an electric dipole ? Derive an expression as if the entire charge were concentrated at the
for the torque acting on an electric dipole, when held centre. (CBSE OD 19]
in a uniformelectric field. Hence define the dipole 22. A thin straight infinitely long conducting wire
moment. [CBSE D OS; OD 03C] having charge density X. is enclosed by a cylin
VZ. An electric dipole is placed in uniform external drical surface of radius r and length I, its axis
electric field £ . Show that the torque on the dipole coinciding with the length of wire. Find the
expression for the electric flux through the surface
is given by T =p X £ of the cylinder. [CBSEOD 11]
ELECTRIC CHARGES AND FIELD

Amsiwers.
u 9i
Refer answer to Q. 18 on page 1.8. Refer answer to Q. 37 on page 1.39.
z im.
Refer answer to Q. 19 on page 1.8. Refer answer to Q. 40 on page 1.40.
3; lllL- Refer answer to Q. 40 on page 1.40.
Refer answer to Q. 21 on page 1.9 and Q. 23 on
page 1.10. HZ
Refer answer to Q. 40 on page 1.40.
4.
Refer answer to Q. 22 on page 1.10. 13.
See Fig. 1.77 on page 1.46.
5.
Refer answer to Q. 22 on page 1.10. 14.,
Refer answer to Q. 43 on page 1.44.
6.
The electric field intensity at a point is defined as 15.
Refer answer to Q. 44(fy) on page 1.46.
the electrostatic force per unit test charge acting on 16.
Refer to point 33 of Glimpses. If the radius of the
a vanishingly small positive test charge placed at
spherical surface is reduced to half, the electric flux

w
that point.
would not change as the charge enclosed remains
F the same.
E = —, for qQ^O 17.
Refer answer to Q. 49 on page 1.48.
%

Flo
18.
Refer answer to Q. 51 on page 1.49.
SI unit of £ = NC ^ 19.
Refer answer to Q. 55 on page 1.56.

ee
Electrostatic force = Charge x Electric field. 20.
Refer answer to Q. 56(c) on page 1.57.

Fr
/.
Refer answer to Q. 31 on page 1.28. 21..
Refer answer to Q. 56(fl) on page 1.57.
8.
Refer answer to Q. 38 on page 1.39. 22.
Refer answer to Q. 52 on page 1.55.

for
ur
^YPE C : Long Answer Q-U’ESTIGNS - I (3 marks each)
ks
1.
Define electrostatic induction. Briefly explain how (0 Using suitable diagram, show that it does not
Yo

an insulated metal sphere can be positively charged undergo any translatory motion, and
oo

by induction. ("■) Derive an expression for the torque acting on it


eB

Z
Give six properties of electric charges. and specify its direction. When is this torque
3.. maximum ? [CBBE ID'OE, 0»]
Define electric field intensity and derive an
expression for it at a point on the axial line of a 8; (a) Obtain the expression for the torque x*
r

dipole. Also determine its direction.


ou

experienced by an electric dipole of dipole


ad

(Punjab 21100,,01.; Hbryana^OZ; CBSE ID'951 _» -*


moment p in a uniform electric field E .
Y

4.
Define the term 'electric dipole moment'. Is it a
scalar or vector ? (&) What will happen if the field were not
uniform ?
nd

Deduce an expression for the electric field at a


Re

-*

point on the equatorial plane of an electric dipole (C) What would happen if the external field £ is
of length 2a. [lEBSE ID)Iir;:P 09';:On)’I[Jl'
Fi

5.
Define electric field intensity. Write its SI unit.
increasing (i) parallel to ^ and (ii) anti-parallel
Write the magnitude and direction of electric field to ^ ? [CQSE.E
intensity due to an electric dipole of length 2a at the
9; State Gauss's theorem and express it mathe
midpoint of the line joining the two charges.
matically. Using it, derive an expression for the
[OBEOD'(15]
electric field intensity at a point near a thin infinite
6.
(i) Derive the expression for electric field at a plane sheet of charge density oCm”^.
point on the equatorial line of an electric [jCBfiE mnEi-orom]
dipole. TO. {a) Using Gauss's law, prove that the electric field at
(ii) Depict the orientation of the dipole in (i) stable, a point due to a uniformly charged infinite plane
(a) unstable equilibrium in a uniform electric sheet is independent of the distance from it.
field. ljrBM'iD>ii7]:
{b) How is the field directed if (0 the sheet is
7.
An electric dipole is held in a uniform electric positively charged, (it) negatively charged ?
field. II018EP uqi
■L104 PHYSICS-XII

11. (fl) An infinitely long thin straight wire has a Plot a graph showing variation of electric field as
uniform linear charge density X. Obtain the a function of r> R and r< R (r being the
expression for the electric field (E) at a point distance from the centre of the shell).
lying at a distance r from the wire, using [CBSE OD 13,16 ; SP 23]
Gauss's law.
13. A charge +Q, is uniforml)' distributed within a
(1?) Show graphically the variation of this electric sphere of radius R. Find the electric field, due to this
field £ as a function of distance r from the wire. charge distribution, at a point distant r from the
ICBSE D 08. 09.17C ; OD 07. 17C, 20] centre of the sphere where :
12. Using Gauss' law, deduce the expression for the (i) 0 < r < R and (ii) r>R. [CBSE OD 16C]
electric field due to a uniformly charged spherical
conducting shell of radius R at a point (/) outside
and (ii) inside the shell.

Answers ▼

1. Refer answer to Q. 11 on page 1.4 and Q. 13 on page 1.5. 7. Refer answer to Q. 40 on page 1.40.
2. Refer to point 9 of Glimpses. 8. (fl) Refer answer to Q. 40 on page 1.40.
3. Refer answer to Q. 37 on page 1.39. (i?) If the field is not uniform, the dipole
4. Refer answer to Q. 38 on page 1.39. experiences unequal forces at its ends. The
net force on the dipole gives it a translatory
5. Refer answer to Q. 29 on page 1.24. At any motion,
equatorial point of a dipole,
(c) Refer answer to Q. 41 on page 1.41.
So ^
V A

^qua
4n Sq (r^ + a^) 9. Refer answer to Q. 53 on page 1.55.
10. Refer answer to Q. 53 on page 1.55.
At the midpoint of the dipole (r = 0), the magnitude
of the field is 11. (fl) Refer answer to Q. 52 on page 1.55.
i__P (b) See Fig. 1.97(b) on page 1.55.
^qua 4n Eq 12. Refer answer to Q. 56 on page 1.57 and see
The directionof the field is from +ve to -ve charge. Fig. 1.102(b).
6. (/) Refer answer to Q. 38 on page 1.39. 13. Refer answer to Q. 57 on page 1.58.
(ii) See Fig. 1.172 on page 1.107.

^YPE D ; Long Answer Questions II (5 marks each)


1. State the principle of superposition and use it to 3. A dipole is made up of two charges +q and -q
obtain the expression for the total force exerted on a separated by a distance 2a. Derive an expression for
point charge due to an assembly of (N - 1) discrete the electric field due to this dipole at a point
point charges. [Haryana 02]
distant r from the centre of the dipole on the
2. (fl) Consider a system of Mcharges7jj,£?2'-Wn'''ith
-> equatorial plane.
position vectors ^2, r^, ...,r^ relative to
some origin 'O'. Deduce the expression for the Draw the shape of the graph, between | EJ and r
net electric field E at a point P with position when r » <7. If this dipole were to be put in a uniform

vector due to this system of charges. external electric field E, obtain an expression for the
torque acting on the dipole. [CBSE SP 15]
[CBSE F 15]
(b) Two identical point charges, q each, are kept
2m apart in air. A third point charge Q of 4. (a) An electric dipole of dipole moment ^ consists
unknown magnitude and sign is placed on the of point charges +q and -q separated by a
line joining the charges such that the system distance 2a apart. Deduce the expression for the
remains in equilibrium. Find the position and
nature of Q. [CBSE D 19]
electric field E due to the dipole at a distancex
ELECTRIC CHARGES AND FIELD 1.105
A

from the centre of the dipole on its axial line in (c) Given the electric field in the region E =2xi,
terms of the dipole moment p. Hence show find the net electric flux through the cube and
that in the limit x » a, the charge enclosed by it.

£ -> 2p ! y

(b) Draw a graph of E versus r for r » a.


(c) If this dipole were kept in a uniform external
E
electric field , diagrammatically represent the
position of the dipole in stable and unstable
equilibrium and write the expressions for the
torque acting on the dipole in both the cases.

w
[CBSE D 15 ; OD 171
Fig. 1.166
5. (fl) Define an ideal electric dipole. Give an
example, (CBSE D 15 ; OD 15]

Flo
(b) Derive an expression for the torque 9. (ij) Using Gauss's law, deduce the expression for
experienced by an electric dipole in a uniform the electric field due to a uniformly charged
electric field. What is net force acting on this

ee
spherical conducting shell of radius R at a
dipole ? point (/) outside and (a) inside the shell,

Fr
(c) An electric dipole of length 2 cm is placed with (b) Plot a graph showing variation of electric field
its axis making an angle of 60° with respect to as a function of r > R and r < R (r being the
uniform electric field of 10^ N/C. If it distance from the centre of the shell),
for
ur
experiences a torque of bVs Nm, calculate the (c) A square plane sheet of side 10 cm is inclined
(i) magnitude of charge on the dipole, and its at an angle of 30° with the direction of a
potential energy. [CBSE SP 21] uniform electric field of 200NC“^ Calculate
ks
6. (fl) Use Gauss's theorem to find the electric the electric flux passing through the sheet.
Yo
oo

field due to a uniformly charged infinitely [CBSE OD 16, 19C]


large plane thin sheet with surface charge 10. {a) State Gauss's law in electrostatics. Using this
eB

density a theorem, show mathematically that for any


(b) An infinitely large thin plane sheet has a point outside the shell, the field due to
uniform surface charge density + o. Obtain the uniformly charged thin spherical shell is the
r

expression for the amount of work done in same as if entire charge of the shell is
ou
ad

bringing a point charge q from infinity to a concentrated at the centre. Why do you expect
the electric field inside the shell to be zero
Y

point, distant r, in front of the charged plane


sheet. [CBSE OD 17] according to this theorem ? [CBSE OD 20]
7. (fl) State Gauss's law. Using this law, obtain the (b) Draw the field lines when the charge density of
nd
Re

expression for the electric field due to an the sphere is (i) positive, (n) negative.
infinitely long straight conductor of linear [CBSE D 08 ; OD 06]
Fi

charge density h [CBSE SP 20] 11. (a) Define electric flux. Is it a scalar or a vector
(b) A wire ABof length L has linear charge density quantity ?
X = kx, where x is measured from the end A of
A point charge cj is at a distance of d/2 directly
the wire. This wire is enclosed by a Gaussian above the centre of a square of side d, as shown
hollow surface. Find the expression for the
in Fig. 1.167. Use Gauss' law to obtain the
electric flux through this surface.
expression for the electric flux through the
[CBSE OD I7C]
square.
8. (a) "Gauss's law in electrostatics is true for any
closed surface, no matter what its shape or size
d/2
is". Justify this statement with the help of a
suitable example,
d
(b) Use Gauss's law to prove that the electric field
d
inside a uniformly charged spherical shell is
zero.
Fig. 1.167
JQS PHYSICS-XII

ib) If the point charge is now moved to a distance (b) In Fig. 1.168, there are three infinite long thin
'd' from the centre of the square and the side of sheets having surface charge density +2cr, -2a
the square is doubled, explain how the electric and + a respectively. Give the magnitude and
flux will be affected. ICBSE D' 18]
2a -2a a

121. (a) Use Gauss's law to derive the expression for


the electric field (£ ) due to a straight uniformly
charged infinite line of charge density A. C/m. A B C D

ib) Draw a graph to show the variation of £ with


perpendicular distance r from the line of charge.
(c) Find the work done in bringing a charge q
from perpendicular distance to Fig. 1.168
[CBSE D 18]
13. {a) State Gauss's law in electrostatics. Show with direction of electric field at a point to the left of
the help of a suitable figure that outward flux sheet of charge density +2a and to the right of
due to a point charge Q, in vacuum within sheet of charge density +a [CBSE SP llT

Gaussian surface, is independent of its size and


shape.

Answers

1. Refer answer to Q. 27 on page 1.19. So the graph between qua and r is of the type as
2. (a) Refer answer to Q. 32 on page 1.29. shown in Fig. 1.170.
(1j) Suppose the three charges be placed in the
manner, as shown below :

H- 2m ■M

A B C E
equa

Q 9
H- +M- 2-x -H
r

Fig. 1.169 Eig.. L17a

For the equilibrium of charge Q,


4. (fl) Refer answer to Q. 37 on page 1.39.
Force between charges at A and B = Force between
For r » a,
charges at B and C
1 qQ _ 1 Qq ^axial “
27C6Q
4ii£q 4716(3
1
x=2-x ^axial ^3
=> x =l m (b) E versus r graph is of the type as shown in
Fig. 1.171.
The charge q at A will be repelled by the similar
£
charge ^ at C It will be in equilibrium only if it is
attracted by charge Qat B.So the nature of charge Q
must be opposite to the nature of charge q.
3. Refer answer to Q. 38 on page 1.39 and Q. 40 on
page 1.40.
For r » a,
1 r
oc
ua
r^-
Big. 1.171
ELECTRIC CHARGES AND FIELD

^ Gaussian
(c) (f) For stable equilibrium, p is parallel to £ . surface

(li) For unstable equilibrium, p is antiparallel B

to E.

T = p£sinG= p£sin(0°or 180°) =0


(/) Stable - £0

equilibrium P
o- ■o
(0 = 0») Fig. 1.173

£o Total charge on the wire of length L will be

w
(ii) Unstable £o q= dq= kxdx
2
equilibrium P
o- o
0 JO
(0 = 180°)
By Gauss's theorem, flux through the hollow

Flo
Gaussian surface will be

Fig. 1.172 kl}

ee
- ^ _
Torque is zero in both cases (i) and (ij). ^“e 0 2e0

Fr
5. (a) An electric dipole of negligibly small size (a) According to Gauss's law, the electric flux
{2a0, qco) is called an ideal dipole. through a closed surface depends on the net
Atomic or molecular dipoles are ideal dipoles, charge enclosed by the surface and not upon
for
ur
the size of the surface.
(b) Refer answer to Q. 40 on page 1.40.
For any closed surface of arbitrary shape
(c) Here 2a = 2 cm = 0.02 m, 0 = 60°,
ks
enclosing a charge, the outward flux is same as
£=10^NC-\ T = SV3Nm
Yo

that due to a spherical Gaussian surface


oo

T=p£sin0 = (7x2(7x£sin0 enclosing the same charge. This is because of


the fact that:
.’. sVs = q X 0.02 X 10^ X sin 60°
eB

(i) electric field is radial, and


sV3x2
= 8xlQ-^C
or

0.02x10^ xV3 (//) the electric field, E oc -i. r


r
ou

P.E. of the dipole is


ad

(b) Refer answer to Q. 56(c) on page 1.57.


U = ~ pE cos Q--qx2axE cos 0 (c) Only the faces perpendicular to the .r-axis
Y

contribute towards the electric flux. Tlie contri


= - 8 X 10“^ X 0.02 X 10^ X cos 60° = - 8 J.
bution from the remaining faces is zero.
nd

6. {a) Refer answer to Q. 53 on page 1.55.


Re

r, 1

(b) Work done in bringing a charge q from infmity


Fi

— ●>

to a point, distant r, in front of the positively /


/
/

-<
charged plane sheet is I
I
I
W=q E.dr -q £drcosl80° 'a
●►x

CO CO

cr
dr = - 3^
7J

2e 0 2e 0
00 Fig. 1.174

Flux through the left face,


[r-oo] = GO.
2eo <^i^= £Scosl80°-2(0)ff^(-l) = 0
7. (fl) Refer answer to Q. 52 on page 1.55. Flux through the right face,
= £Scos0°= 2a xa^ xl = 2^
(b) Charge on a small element of length of x of the
wire is Net flux through the cube.
dq - Xdx = kxdx <}>£ = 4>l +
s LI 08

9. (fl) Refer answer to Q. 56 on page 1.57.

.
(b) See Fig. 1.102(b) on page 1.57.
(c) <|)e = EScosQ =200x(0.10)^cos(90°-30°)
PHYSICS-XII

(c) Work done in moving the charge c\ through


small displacement dr,

dW = F.dr =qE.dr =qEdrcosO°


= 200x0.01 xO.5 Nm^“’
= 1.0 Nm^C"'’. or
dW = g X xdr
Ine^r
10. (a) Refer answer to Q. 56 on page 1.57. Any
Gaussian surface lying inside spherical shell Total work done in moving the charge q from
does not enclose any charge. So by Gauss's perpendicular distance to r2(r2 > r^),
theorem, electric field inside the shell is zero,
(b) The lines of force for positively and negatively W =
dW = |
charged spherical shells are shown below : { 27reo r
- ^ brfr=
j r
2tk
^
0^

- Jl
2tiEo 2jt£o
13. (<j) Gauss's theorem. It states that the total electric
Fig. 1.175 flux through a closed surface S is equal to I/cq
11. (fl) The electric flux through any surface, open or times the charge q enclosed by the surface S.
closed, is equal to the surface integral of the electric
9
field E over the surface S. (j>£ = ' £ .dS = —
^0

(j>£=J E.rfS Electric field at point P due to charge q,


Electric flux is a scalar quantity. E = —L
We can imagine the square as the face of a cube of
edge d and with the charge q placed at its centre, as
Flux through area element dS
shown in the figure. ●7

According to Gauss's /● I
d(j)£ = E .dS = EdScosQ
law, total flux through L
the cube is q dScosG _ q dn
_
I
I 47tEQ 7^ 4jc8 0

I d/2
dScosG
where dCi = = solid angle subtended by dS
Flux through the each of d
the six square faces. at 0.
d
1 Total solid angle subtended by surface Sat 0 = 47T
^~6'^"'6c0 Fig. 1.176
dO = ^ x4rc = —
47160 4iKo
(b) If the charge is moved to a distance d and the S

side of the square is doubled, the imaginary cube This proves Gauss's theorem for Gaussian surface
will have side 2d but the charge enclosed {q) will of any shape and size.
remain the same. Hence the total flux through the 2o 2ct a

cube and therefore the flux through the square will (b) E^= + 1- — - + —, towards left.
2e 0
remain same as before.
2Eo 2eo 26o
2ct 2ct a a ^ j ●
£r,= + h — = —, towards nght.
12. (a) Refer answer to Q. 52 on page 1.55. D
26o 26o 2eo 2so
See Fig. 1.97(b) on page 1.55.
COMPETITION SECTION

ElecMc Charges and Field

GLIMPSES

w
1. Electrostatics. It is the study of electric charges 8. Electroscope. A device used for detecting an
at rest. electric charge and identifying its polarity is

Flo
called electroscope.
2. Frictional electricity. The property of rubbed
substances due to which they attract light objects 9. Properties of electric charges are as follows :

ee
is called electricity. The electricity developed by (i) Electric charges are quantized. Basic

Fr
rubbing or friction is called frictional or static («) Electric charges are additive, properties
electricity. The rubbed substances which show
(m) Electric charges are conserved,
this property of attraction are said to be
(iz;) Like charges repel and unlike charges
for
ur
electrified or electrically charged substances.
attract each other,
3. Electric charge. It is an intrinsic property of
elementary particles of matter which gives rise (u) The magnitude of elementary negative
ks
to electric force between various objects. It is a charge is same as that of elementary -19
Yo

C
scalar quantity and its SI unit is coulomb (C). positive charge and is equal to 1.6 x 10
oo

(vi) Unlike mass, the electric charge on a body


4. Positive and negative charges. Benjamin Franklin
eB

is not affected by its motion.


introduced the present day convention that
(i) The charge developed on a glass rod when 10. Additivity of electric charge. This means that
rubbed with silk is called positive charge,
the total charge of a system is tlie algebraic sum
r

of all the individual charges located at different


ou

(fj) The charge developed on a plastic/ebonite


ad

points inside the system.


rod when rubbed with fur is called negative
Y

charge. 11. Quantization of electric charge. This means that


the total charge {q) of a body is always an integral
5. Fundamental law of electrostatics. Like charges
nd

multiple of a basic charge (e) which is the charge


Re

repel and unlike charges attract each other. on an electron. That is,
6. Electronic theory of frictional electricity. During
Fi

q = ne, where « =0,± 1, ±2, ±3,


rubbing, electrons are transferred from one
object to another. The object with excess of The basic cause of quantisation of electric
electrons develops a negative charge, while the charge is that during rubbing only an integral
number of electrons can be transferred from one
object with deficit of electrons develops a
positive charge. body to another.
Faraday's laws of electrolysis and Millikan's oil
7. Electrostatic induction. It is the phenomenon of
drop experiment established the quantum
temporary electrification of a conductor in
nature of electric charge.
which opposite charges appear at its closer end
and similar charges appear at its farther end in 12. Basic quantum of charge. The smallest amount
the presence of a nearby charged body. An of charge or tire basic quantum of charge is the
insulated conductor can be positively or charge on an electron or proton. Its exact
negatively charged by induction. magnitude is e = 1.602182 x 10 -19 c.
(1.109)
I.IIQ PHYSICS-XII

13. Law of consen^ation of charge. It states that the Coulomb's law for any medium other than
total charge of a system remains unchanged vacuum can be written as
with time. This means that when bodies are
charged through friction, there is only transfer F =-}—!h3i
of charge from one body to another but no net
creation or destruction of charge takes place. 1 ^1^2 _ vac

14. Coulomb's law. The force of attraction or 47ceQK K

repulsion between two stationary point charges 18. Electrostatic force 7)s. gravitational force.
c\^ and ^2 is directly proportional to the product Electrostatic forces are much stronger than
q^q2 and inversely proportional to the square of gravitational forces. The ratio of the electric
the distance r between them. Mathematically, force and gravitational force between a proton
and an electron is
F=k
F
e —
ke^ 39
-227x10
Gmjn
The proportionality constant k depends on the p

nature of the medium between the two charges 19. Trinclple'of superposition of electrostafcic forces.
and the system of units chosen to measure F, q^, q^ When a number of charges are interacting, the
and r. For free space and in SI units, total force on a given charge is the vector sum of
1 the forces exerted on it due to all other charges.
k = = 9x10^ Nm^C^^
4ne
0
The force between two charges is not affected
by the presence of other charges. The total force
Eq is called permittivity
-12
of free space and its on charge q^ due to the charges q2> q^,
value is 8.854 x 10 C^N-^m"^. will be

Hence Coulomb's law in SI units may be


expressed as ^1 “ -^12 -^13 ,+ KIN

1 N
F = ^1?2 'll
K.
1/
4ti8q 4 Tie
0 i = 2 "1/

15. SI unit of charge .is cauiomb i(^). It is that N

amount of charge that repels an equal and y O


4 Tie
similar charge with a force of 9 x 10^ N when 0 i = 2

placed in vacuum at a distance of one metre


from it. - .r

where .r1j _

16. PermittrviW (c). It is the property of a medium l?-?l


which determines the electric force between
two charges situated in that medium. = a unit vector pointing from g. to q^
20. Electricfield. An electric field is said to exist at a
17. Dielectric constant or relative permittivity. The
ratio (s/Eq) of the permittivity of the given point, if a force of electrical origin is exerted
medium to that of free space is known as on a stationary charge placed at that point.
relative permittivity ) or dielectic constant (k) Quantitatively, it is defined as the electrostatic
of the given medium. force per unit test charge acting on a
g Fvac vanishingly small positive test charge placed at
or K =
the given point.
^0 med
Mathematically,
The dielectric constant of a medium may be ->

defined as the ratio of the force between two £ = lim —


charges placed some distance apart in free space %
to the force between the same two charges when Electric field is a vector quantity whose
they are placed the same distance apart in the direction is same as that of the force exerted on a
given medium. positive test charge.
ELECTRIC CHARGES AND FIELDl .(ConTp£Elsiljion Section;) i

21. 'Units and dimensions of lelectmc ifield. The SI %


unit of electric field is newton per coulomb
or F
I P + l 4r dS + [ dl
V ^ ^
cont
4 Tie
0 L ^
(NC“^) or volt per metre (Vm"^). The
dimensions of electric field are -* F
_ cont
£
Force MLT"^ cont
IE] = %
Charge C 1 X A
r dL
MLT"2 4ns 0
AT
= [MLT"^A'^] V

26. Tilectric field due to a general charge distribution.


22. iElectric field due :to a point chiirge. The electric
field of a point charge q at distance r from it is It is given by
given by E
total
= £
'discrete
+ E'cont

£ = ^—●4 N

w
1
4jieo
47te
0 r=l r
V ^
If q is positive, £ points radially outwards and if
q is negative, £ points radially inwards. This X A

Flo
a
+ r dS+ -=■ r dL
field is spherically symmetric.
23. Electric field due to a system of point charges ;

ee
St^erpo.sition principle for electric fields. The
27.
Electric dipole and dipole moment. An electric

Fr
principle states that the electric field at any dipole is a pair of equal and opposite charges
point due to a group of point charges is equal to + q and - q separated by some distance 2a. Its
the vector sum of the electric fields produced by dipole moment is given by
for
ur
each charge individually at that point, when all p = Either charge x a vector drawn from - qto-i- q
other charges are assumed to be absent.
= ^/ X 2 d
ks
^

£ = £.j + £2 + + £w
N
Magnitude of dipole moment, p = qx2a
Yo
oo

1
y
Z-<
-^r
2 'j’P
Dipole moment is a vector quantity having
4ns i=l ^iP direction along the dipole axis from ~qto+ q. Its
eB

1
N SI unit is coulomb metre (Cm).
4 TIE
z 28.
Electric field at an axial point of a dipole. The
0 1 = 1
r

dipole field on the axis at distance r from the


ou
ad

centre is
24. Continuous charge distribution. When the
charge involved is much greater than the charge
1 2pr 1
Y

. for r » a.
^axial
on an electron, we can ignore its quantum 4usq' 4tce
0
r

nature and assume that the charge is distributed


nd

At any axial point, the direction of dipole field is


Re

in a continuous manner. This is known as a

continuous charge distribution. along the direction of dipole moment ^


Fi

dq Cm"^
Volume charge density, p = 29. Electric field at an equatorial point of a dipole.
dV
The electric field at a point on the perpendicular
Surface charge density, a = ^ Cm-^ bisector of the dipole at distance r from its
dS centre is

Linear charge density. k = ^ Cm -1


£
equa
1 V 1
—.
p for
, r » a.
dL
4%Sq 4ns„0 r

25. Electrostatic force and field due to a continuous


At any equatorial point, the direction of dipole
charge distribution. The total force on a charge field is antiparallel to the direction of dipole
q^ due to a continuous charge distribution is moment p.
given by
-*■ In contrast to 1 / dependence of the
^cont electric field of a point charge, the dipole field
.112 PHYSICS-Xil

-*
has 1/ dependence. Moreover, the electric E. dS
field due to a short dipole at a certain distance S

along the axis is twice the electric field at the


Electric flux is a scalar quantity.
same distance along the equatorial line.
SI unit of electric flux = Nm^ C"\
30. Torque on a dipole in a uniform electric field.
The torque on a dipole of moment p when placed 34. Gaussian surface. Any hypothetical closed
in a uniform electric field at an angle 0 with it is surface enclosing a charge is called the Gaussian
given by X = p£ sin 9 surface of that charge.
35. Gauss's theorem. The total flux of electric field
In vector rotation. t: = p X E
When the dipole is released, the torque x tends ~.i through a closed surface S is equal to I/Eq
to align the dipole along the field E . times the charge q enclosed by the surface S.

If £=1 unit and 9=90° then x = p. So dipole E.dS = —


moment may also be defined as the torque s ^0
acting on an electric dipole placed perpendi
cular to a uniform electric field of unit strength. 36. Electric field of a line charge. The electric field
31. Electric lines of force. An electric line of force
of a long straight wire of uniform linear charge
density X,
may be defined as the curve along which a
X
small positive charge would tend to move when E = i.e., £oc-
2ns^r
free to do so in an electric field and the tangent 0
r

to which at any point gives the direction of


electric field at that point. where r is the perpendicular distance of the
wire from the observation point.
32. Important properties of electric lines of force.
These are : 37. Electric field of an infinite plane sheet of charge.
(/) Lines of force are continuous curves It does not depend on the distance of the
without any breaks, observation point from the plane sheet.
(i/) No two lines of force can cross each other. £ =
CT

2s
(Hi) They start at positive charges and end at 0

negative charges—they cannot form closed where a = uniform surface charge density.
loops.
(iv) The relative closeness of the lines of force 38. Electric field of two positively charged parallel
indicates the strength of electric field at plates. If the two plates have surface charge
different points, densities Oj and ©2 such that CTj >(J2 >0, then
1
(i?) They are always normal to the surface of a
conductor.
E =±
2e 0
(oj -I- CT2) (Outside the plates)
(vi) They have a tendency to contract length 1
wise and expand laterally. E-~—
2 E
(oj -02) (Inside the plates)
0

33. Electric flux. The electric flux through a given


area represents the total number of electric lines 39. Electric field of two equally and oppositely
of force passing normally tlirough that area. If the charged parallel plates. If the two plates have
surface charge densities ± a, then
electric field £ makes an angle 0 with the normal
E=0 (For outside points)
to the area elements AS, then the electric flux is
a
£= — (For inside points)
A(f)£ - EAS cos 0 = £ . A S ^0
The electric flux through any surface S, open or 40. Electric field of a thin spherical shell. If R is the
closed, is equal to the surface integral of ^ over radius and a, the surface charge density of the
the surface S. shell, then
ELECTRIC CHARGES AND FlELDl (Competition Section) , 1,1 u
1 Forr> R (Outside points) E = ^— For r> R (Outside points)
471 £n0 r 4;i:6o r
£=0 For r<R (Inside points)
For r < R (Inside points)
47C6n0 R^
£ = 1 _ R For r = R (At the surface)
47CEg 1
£ = For r=R (At the surface)
where q=47iR^a 4Tcen0 R^
41. El: ‘■i.il. JiargL-ii ‘'oHd
4 ^
If p is the uniform volume charge where ct = — tiR p
^ 3
density and R radius of the sphere, then

w
WS:W JEEAdvance

Multiple Choice Questions with (b)

Flo
one correct answer
M

ee
'. Two small balls, each having a positive charge q

7)
coulomb, are suspended by two insulating strings of

Fr
equal length / metre from a hook fixed to a stand. The
whole set up is taken in a satellite into space where
there is no gravity. The angle between the two strings for
ur
and the tenstion in each string will be (c) (d)
1
(fl)180°;
ks
(£)90°;
' (2/) 47reQ
Yo

©
oo

(c) 180°; {d)180°;


47I6q 2r 47rsQ ?● © ©
eB

(IIT 211,'i
2. A charge q is placed at the centre of the line
r

Two large vertical and parallel metal plates


ou

joining two equal positive charges Q. The system of the


ad

three charges will be in equilibrium, if q is equal to having a separation of 1 cm are connected to a dc


Y

Q Q voltage source of potential difference X. A proton is


released at rest midway between the two plates. It is
nd

found to move at 45° to the vertical JUST after release.


Re

Q
Then X is nearly
llir 1987; AH-ILI: 2002|
Fi

(fl)lxlO“^V (l;)lxlO'^V
A small metal ball is suspended in a uniform
(c)lxlO“^V
-10

electric field with the help of an insulated thread. If (d)lxlO V jllT 12]

high energy X-ray beam falls on it, n. Point charge q moves from point P to point S along
{a) the ball will be deflected in the direction of the path PQRS (Figure below) in a uniform electric
field.
Y
(1?) the ball will be deflected opposite to the direc
tion of field,

(c) the ball will not deflect at all.


(d) the ball will move to infinity. HIT £

●1. Three positive charges of equal value q are placed Q


X

at the vertices of an equilateral triangle. The resulting


R
lines of force should be sketched as in
1.114 PHYSICS-XII

field £ pointing parallel to the positive direction of 10. Six charges, three positive and three negative of
X-axis. The coordinates of the points P,Q,R and S are equal magnitude are to be placed at the vertices of a
(fl,b,0), (2d,0,0). {a,-b,0) and (0,0,0) respectively. The regular hexagon, such that the electric field, at the
work done by the field in the above process is given by
the expression P Q

(a) q Ea (b) ~qEa


(c) qEaJI (d) qE^j{2af + b'^ U R
0
7. Consider the charge
configuration and a spherical /
Gaussian surface as shown in / I
r s
the figure while calculating the ‘ I

flux of the electric field over the \ 0h / point O is double the electric field, when only one
spherical surface, the electric positive charge of same magnitude is placed at R.
field will be due to Which of the following arrangements of charges is
(0)^2 (b) only the positive charges possible for P, Q, R, S, T and U respectively ?
(c) all the charges
+/ -I +/ “/ “/ + (^) “ +/ “ +/ “
(d) +q^ and -q^ ,i':
(c) +, +, -/ +, - - (d) lHT3il.U|
'i- A metallic shell has a point charge q kept inside
! i. Three concentric metallic spherical shells of
its cavity. Which one of the following diagrams correctly
radii R, 2R, 3R are given charges Qy Q2, Q3, respec
represents the electric lines of forces ? ; I

tively. It is found that the surface charge densities on


(«) (&) the outer surfaces of the shells are equal. Then, the
ratio of the charges given to the shells, : Q2: Qy is
(fl) 1 : 2 : 3 (b) 1 : 3 : 5
(c) 1 : 4 : 9 (d) 1 : 8 : 18 111 . '!

(c) id) 12. A disk of radius fl/4 having a uniformly


distributed charge 6 C is placed in the x-y plane with its
centre at (~fl/2, 0, 0). A rod of length a carrying a
uniformly distributed charge 8 C is placed on the
A:-axis from x=alAiox- 5c7/4. Two point charges -7C and
3C are placed at (fl/4, -a/4, 0) and (-3fl/4, 3o/4, 0),
9. Three infinitely long charge sheets are placed as respectively. Consider a cubical surface formed by six
shown in the figure. surfaces a: = ±rt/2, y =±a!2,z =±al2. The electric flux
through this cubical surface is
2
a
z = 3a y

● P

- 2 CT
z = a

- a
z = -n

The electric field at point P is -2C 2C


2a 2o ^ («) (b)
(a) k (b)- k ^0 ^0
^0
10c 12 c
ic) ^k id)
4a (c) id)
^0 ^0 HIT :iU09|
ELECTRIC CHARGES AND FIELDl (Competition Section) Lirj

13. A uniformly charged thin spherical shell of


radius R carries uniform surface charge density of o [a, 0, a) {a, a, a)
per unit area. It is made of two hemispherical shells.

(0,0, 0) (0, a, 0) V
F
X

(«)2E/ (b) ^/2 E/


held together by pressing them with force F (see
figure). F is proportional to (c) E/ (d)
V2 [HT

w
{a)—a^R^ {b)—a^R ,. Charges Q,2 Q and 4Q are uniformly distributed
^0 in three dielectric solid spheres 1,2 and 3 of radii R/2,
R and 2R respectively, as shown in the figure. If

Flo
la2
(c) id)
s.R
0 e,R^

ee
14. A wooden block performs SHM on a frictionless

Fr
surface with frequency, Vg. The block carries a charge
E

for
ur
+ Q
"TO
bphere 1
ks
+ Qon its surface. If now a uniform electrical field £ is magnitudes of the electric fields at point P at a distance
Yo

R from the centre of spheres 1,2 and 3 are Ej, £2 and £3


oo

switched on as shown, then the SHM of the block will be


respectively, then
(<7) of the same frequency and with shifted mean
eB

position (fl) Ej > £2 > £3 (b) £3 > Ej > £2


(b) of the same frequency and with the same mean (c) £2 > Ej > £3 (d) £3 > £2 > £j [JEE All-, if
r

position
Multiple Choice Questions with One
ou
ad

(c) of changed frequency and with shifted mean or More than One Correct Answers
position
Y

(d) of changed frequency and with the same mean 1^'. Two equal negative charges -q are fixed at the
position. points (0,fl) and (0,-fl) on the Y-axis. A positive charge
nd
Re

15. A tiny spherical oil drop carrying a net charge q Q is released from rest at the point (2a,0) on the X-axis.
Fi

is balanced in still air with a vertical uniform electric


The charge Q will
(fl) execute simple harmonic motion above the origin.
field of strength ^^xlO^Vm”^ When the field is (1?) move to the origin and remain at rest,
switched off, the drop is observed to fall with terminal (c) move to infinity.
velocity 2 x 10”^ms"^ Given g = 9.8 ms"^, viscosity of (d) execute oscillatory but not simple harmonic
the air = 1.8 x 10"^ Ns m"^ and the density of oil motion. [IIT
= 900kgm"^ the magnitude of q is ●. A positively charged thin metal irng of radius R
(n) 1.6xlO‘^^C (&) 3.2xlO“’^C is fixed in the XY-plane with its centre at the origin O.
(c) 4.8xlO‘^^C (d) S.OxlO'^^C A negatively charged particle P is released from rest
at the point (0,0,2g), where Zq >0. Then, the motion of
16. Consider an electric field £ = EqX, where £g is a P is

constant. The flux through the shaded area (as shown (a) periodic for all the values of Zq satisfying
in the figure) due to this field is 0 < 2„0 < CO
1.116 PHYSICS-XII

(b) simple harmonic for all the values of ,z 0 (c) At a finite distance to the left of the electric
satisfying 0 <Zq < R. field is zero

(c) approximately simple harmonic provided Zq « R. (d) At a finite distance to the right of Q2 the electric
(d) such that P crosses O and continues to move field is zero.

along the negative Z-axis towards z = -x. 21. A spherical metal shell A of radius R^ and a
[in
solid metal sphere B of radius Rg(< R^) are kept far
20. A non-conducting solid sphere of radius R is apart and each is given charge + Q. Now they are
uniformly charged. The magnitude of the electric field connected by a thin metal wire. Then
due to the sphere at a distance r from its centre inside
(«) El - =0 (b)QA>QB
{a) increases as r increases, for r « R.
R
(c)^ = 6 n surface surface
(b) decreases as r increases, for 0 < r < x (d) B
CT
B
(c) decreases as r increases, for R < r < x
(d) is discontinuous at r = R. jin 2.T. A cubical region of side a has its centre at the
origin. It encloses three fixed point charges, -q at
21. A metallic solid is placed in a uniform electric {0,-fl/4:,0),+3q at (0,0,0) and -q at (0,+ fl/4,0). Choose
field.
the correct option(s).
1 > ♦— 1

2
7 ♦— 2

3 * 3

d -k- d

-q

The lines of force follow the path(s) shown in the 3q


figure as :
X-
{a)l {h)l
(c)3 id) A iin

22. Under the influence of the Coulomb field of (fl) The net electric flux crossing the plane
charge +Q, a charge -q is moving around it in an x-+a!2 is equal to the net electric flux
elliptical orbit. Find out the correct statement(s). crossing the plane x=—aH.
(fl) The angular momentum of the charge -q is (b) The net electric flux crossing the plane
constant.
y=+al2 is more than the net electric flux
(b) The linear momentum of the charge -q is crossing the plane y =-a 12.
constant.
(c) The net electric flux crossing the entire
(c) The angular velocity of the charge-q is constant.

region is —.
^0
(d) The linear speed of the charge -q is constant.
[iiT (if) The net electric flux crossing the plane
23. A few electric field lines for a system of two z= + al2 is equal to the net electric flux
charges Qj and Q2 fixed at two different points on the crossing the plane x = +a f 2.
x-axis are shown in the figure. 2b. The figure below depicts two situations in
which two infinitely long static line charges of constant
positive line charge density X are kept parallel to each

X X X X

- - ►x - - ►x
+4 -4
These lines suggest that
(«)lQil>iQ2l (fc)IQjl<!Q2l
ELECTRIC CHARGES AND FIELDl (Compeiition Section) 1.117^

other. In their resulting electric field, point charges q 29. Let £j(r), E2(r) and £3(r) be the respective
and -q are kept in equilibrium between them. The electric fields at a distance r from a point charge Q, an
point charges are confined to move in the x direction infinitely long wire with constant linear charge density
only. If they are given a small displacement about their X, and an infinite plane with uniform surface charge
positions, then the correct statement(s) is(are)
density a If £i(?5)= £2(^0) “ £3(^0^^^ ® distance r^,
(a) Both charges execute simple harmonic motion. then

(b) Both charges will continue moving in the (fl) Q = 4oKr^


direction of their displacement,
(c) Charge +q executes simple harmonic motion 2 no

while charge -q continues moving in the (c) E^{rJl)=lE2{rJl)


direction of its displacement.
(d) E2%tl) = 4E^{rJ2) !iri A>iv. 1;]
(d) Charge ~q executes simple harmonic motion

w
while charge +q continues moving in the 30. An infinitely long thin non-conducting wire is
direction of its displacement. IN I: .Vilv. i"| parallel to the z-axis and carries a uniform line charge
27. Consider a uniform spherical
charge <^s*^sity It pierces a thin non-conducting spherical

Flo
distribution of radius R, centered at the origin O. In radius R in such a way that the arc PQ sub¬
this distribution, a tends an angle 120° at the centre O of the spherical shell, as

ee
spherical cavity of radius shown in the figure. The permittivity of free space is Sq.

Fr
R2 centred at P with
distance OP = a = R^-R^
(see figure) is made. If the
for
ur
electric field inside the

cavity at position r is
ks
—► —>

£ (r ), then the correct


Yo

statement(s) is(are)
oo
eB

(fl) £ is uniform, its magnitude is independent of


R2 but its direction depends on T
Which of the following statements is (are) true ?
r

(b) E is uniform, its magnitude depends on


ou

(fl) The electric flux through the shell is -JSRkle 0-


ad

and its direction depends on 7 (b) The 2-component of the electric field is zero at
Y

all the points on the surface of the shell,


(c) E is uniform, its magnitude is independent of (c) The electric flux through the shell is V2 RX,/e 0-
nd
Re

a but its direction depends on (rf) The electric field is normal to the surface of the
shell at all points.
Fi

[ti i' Auv IS III


(d) E is uniform and both its magnitude and
31. A charged shell of radius R carries a total charge
direction depend on Q. Given (j) as the flux of electric field through a closed
28. Two non-conducting solid spheres of radii R cylindrical surface of height h, radius r and with its
and 2R, having uniform volume charge densities p 1 centre same as that of the shell. Here, centre of the
and P2 respectively, touch each other. The net electric cylinder is a point on the axis of the cylinder which is
field at a distance 2 R from the centre of the smaller equidistant from its top and bottom surfaces. Which of
sphere, along the line joining the centre of the spheres following option(s) is/are correct ? [Sp is the
is zero. The ratio Pj /p2 can be permittivity of free space]
32 Q
(a)-4 {b)~ (a) li h >2 R and r > R, then (]i = -^
25 ^0
32 8R 3R
(c) {d)4 (b) lth< — and r = —, then d) = 0
25 II- :■ . V 5 5
1.118 PHYSICS-XII

4R Q
' (c)If/z>2Kandr = Linked Comprehension Type
5s
0
Paragraph for Q. Nos. 34 to 36
3R Q
(d)Jfh>2Randr = The nuclear charge {Ze) is non-uniformly
5s 0
distributed within a nucleus of radius R. The

'2. A uniform electric field, E =-400V3t/ NC*^ is charge density p(r) [charge per unit volume] is
dependent only on the radial distance r from the
applied in a region. A charged particle of mass m
centre of the nucleus as shown in the figure. The
carrying positive charge q is projected in this region
electric field is only along the radial direction.
with an initial speed of2VlO xlO^ms^Tlhis particle is p(r) >.

” £

T
●►r
R

5 m
> Tlie electric field at r = R is

(a) independent of a
aimed to hit a target T, which is 5 m away from its
{b) directly proportional to a
entry point into the field as shown schematically in the
(c) directly proportional to
figure. Take —m =10^^Ckg"L Then
(d) inversely proportional to a HIT :o;is|

(i?) the particle will hit T if projected at an angle 45® For a=0, the value of d (maximum value of p as
from the horizontal.
shown in the figure) is
(b) the particle will hit T if projected either at an 3Ze 3Ze
(a) ib)
angle 30® or 60® from the horizontal. 4iiR^ kR^

(c) time taken by tl^e particle to hit T could be JV —6 ps (c)


4Ze
(rf)
Ze

3nR^ 3kR^ [IIT 20081


as well as J—us.
V2 3f. The electric field within the nucleus is generally
[5 observed to be linearly dependent on r.
(d) time taken by the particle to hit T is ps.
This implies
R
(fl)= 0
' . Two identical non-conducting solid spheres of
2R
same mass and charge are suspended in air from a (c) a = R
common point by two non-conducting, massless |IIT
strings of same length. At equilibrium, the angle Paragraph for Q. Nos. 37 to 38
between the strings is a. The spheres are now
A dense collection of equal number of electrons and
immersed in a dielectric liquid of density 800 kgm
positive ions is called neutral plasma. Certain
and dielectric constant 21. If the angle between the
solids containing fixed positive ions surrounded by
strings remains the same after the immersion, then
free electrons can be treated as neutral plasma. Let
(a) electric force between the spheres remains N be the number of density of free electrons, each
unchanged of mass JH When the electrons are subjected to an
(b) electric force between the spheres reduces electric field, they are displaced relatively away
(c) mass density of the spheres is 840 kg m"^ from the heavy positive ions. If the electric field
(d) the tension in the strings holding the spheres becomes zero, the electrons begin to oscillate
remains unchanged about the positive ions with a natural angular
[JET frequency which is called the plasma frequency.
ELECTRIC CHARGES AND FIELDl (Competition Section) 1.119

To sustain the oscillations, a time varying electric 41. A charge q is surrounded by a closed surface
field needs to be applied that has an angular consisting of an inverted cone of height h and base
frequency (o, where a part of the energy is absorbed radius R, and a hemisphere of radius R as shown in the
and a part of it is reflected. As co approaches (O^ all figure. The electric flux through the conical surface is
the free electrons are set to resonance together and nq
(in SI units). Tlie value of ti is .
all the energy is reflected. This is the explanation of 6e 0
!M I \Jv. 1
high reflectivity of metals. lil J^\]\

3“. Taking the electronic charge as 'e' and the


permittivity as 'Bq, use dimensional analysis to
determine the correct expression for co^.
Ne me
0
(«) (b)
V Ne

w
me
0

Ne^ me
0
(c) {d)
1 Ne^ ' ■!2.Two large circular discs separated by a distance

Flo
me
0

of 0.01 m are connected to a battery via a switch as


38- Estimate the wavelength at which plasma
shown in the figure. Charged oil drops of density

ee
reflection will occur for a metal having the density of
900kgm"^ are released through a tiny hole at the centre
electrons N a;4xl0^^m"^. Take Eg and

Fr
where these quantities are in proper SI units.
(a) 800 nm (1;) 600 nm (c) 300 nm (d) 200 nm o 0
1
for
ur
e
Integer Answer Type Switch
0.01 m
3‘). A solid sphere of radius R has a charge Q
ks
200 Vt
distributed in the volume with a charge density p =kr‘\
Yo

where k and a are constants and r is the distance from


oo

of the top disc. Once some oil drops achieve terminal


its centre. If the electric field at r = — is - times that at
velocity, the switch is closed to apply a voltage of 200 V
eB

2 8
r = R, find the value of a. ill I
across the discs. As a result, an oil drop of radius
8x10 m stops moving vertically and floats between
4U. An infinitely long uniform line charge
the discs. The number of electrons present in this oil
r

distribution of charge per unit le^th X lies parallel to drop is


ou

. (neglect the buoyancy force, take


ad

^/3
the y-axis in the y-z plane at z = figure). If the acceleration due to gravity = 10 ms"^ and charge on d<
Y

electron (e) = 1.6 x 10“^^C) IJKf Ad\-. 20201


magnitude of the flux of the electric field though the
Numerical Value upto Second Decimal Place
rectangular surface ABCD l^ing in the x-y plane with
nd
Re

its centre at the origin is ne


(Eq = permittivity of free 43. A particle, of mass 10"^ kg and charge 1.0 C, is
Fi

0
initially at rest. At time t =0, the particle comes under
space), then the value of n is III I All'.,. I \
the influence of an electric field £ (f) = EgSincof i,
where £g = 1.0 NC“* and o) = 10^rads”^. Consider the
effect of only the electrical force on the particle. Then the
maximum speed, in ms"^, attained by the particle at
L
D
43 subsequent times is iiiT-: A.^. IS ji|
— a
-C 2
44. One end of a spring of negligible unstretched
O ^ length and spring constant k is fixed at the origin (0,0).
A point particle of mass m carrying a positive charge q
is attached at its other end. The entire system is kept on
'B
a smooth horizontal surface. When a point dipole p
pointing towards the charge q is fixed at the origin, the
s
●f V

PHYSiCS-XII

“y List-I List-n

P- Qi, Qi, Qi, Q4 3\\ positive 1. +x

p Q- Q' Qi positive; Q, Q negative 2. -X

♦●'OOflOOOOOOOOO - ->x
(0,0) R. Q, Qi positive; Q^, Q3 negative 3. +y
S. Q> positive; 0,, Qi negative 4.

spring gets stretched to a length I and attains a new Code :


equilibrium position {see figure above). If the point (a) P-3, Q-1, R-4, S-2 (b) P-4, Q-2, R-3, S-1
mass is now displaced slightly by Al«I from its
(c) P-3, Q-1, R-2, S-2 (d) P-4, Q-2, K-1, S-3
equilibrium position and released, it is found to 'I-'' \..r.
1 k
oscillate at frequency The value of 5 is
i"'. The electric field E is measured at a point
6 V m
P(0,0,ri) generated due to various charge distributions
and the dependence of £ on d is found to be different
43. A circular disc of radius R carries surface charge for different charge distributions. List-I contains
different relations between E and d. List-Il describes
density (7(r) = aQ V. 1—- , where 0 is a constant and r is
R, different electric charge distributions, along with their
the distance from the centre of the disc. Electric flux locations. Match the functions in List-I with the related

through a large spherical surface that encloses the charge distributions in List-II.
charged disc completely is (1)q. Electric flux through List-i List II

another spherical surface of radius — and concentric P. £ is indep 1. A point charge Qat the origin
endent of d
with the disc is (j). Then the ratio — is. Q. £x- 2.
A small dipole with point
4> d
charges Q at (0,0,/) and -Q at
(0,0,-/). Take 2/ «d

Match-Matrix Type R. £oc4t 3.


An infinite line charge coin
d^ cident with the x-axis, with
lu. Four charges and Qj of same uniform linear charge density X
magnitude are fixed along the x-axis .atx--2a,-a,+a S. Ecc4 4.
Two infinite wires carrying
and 4-2 fl, respectively. A positive charge q is placed on d^ uniform linear charge density
the positive y-axis at a distance b > 0. Four options of parallel to the x-axis. The one
the signs of these charges are given in List I. The along (1/ = 0, z = /) has a charge
direction of the forces on the charge q is given in List II. density -i-X and the one along
Match List I with List II and select the correct answer (y = 0, 2=-/) has a charge
density -X. Take 2/ « d
using the code given below the lists.
5.
● q(0,b) Infinite plane charge coinci
dent with the xy-plane with
uniform surface charge density
(fl) P-5, Q-(3, 4), R-1, S-2 {b) P-5, Q-3, R-{1, 4), S-2
Qi Qz Qs Q3
(- 2a, 0) (- a, 0) (+ a, 0) (+ 2a, 0) (c) P-5, Q-3, R-(l, 2), S-4 (d) P-4, Q-(2, 3), R-1, S-5
t I I

Answers and Explanations


1. (/?) As the two balls are in the state of 2. : For equilibrium, charge q must be negative.
weightlessness, the two strings would become Fi3 1 Fi2 2 3
horizontal due to the force of repulsion, ’«■
+ Q X X + Q
.●. Angle between the two strings =180°
Tension in each string Considering the equilibrium of charge 1, .
<7^ kQx Q _ kQxq
= Force of repulsion =
47180 (2/)
jN.
{2xf x^
or
-!■
ELECTRIC CHARGES AND FIELDl Lompetit’on Noaion) L12!

3. ((/ The X-ray beam knocks out electrons from kq


10. ui) I E 1 =
the metal ball. The ball becomes positively charged
and gets deflected in the direction of the electric P
field.

4. Electric lines of force do not form closed loops.


-/ ^ \'
a\ +
So options (b) and (d) are wrong. Morever, lines of R
force start from a positive charge and end on a
negative charge, so option (fl) is also wrong.
\t £/
5. When the proton moves at 45® with vertical,
T S
qE = mg
Fields at O due to P and S are equal and opposite
and cancel out. Fields at O due to Q and T also cancel

w
qE
+ out. But the fields at O due to R and U are in same
o- ●o

direction. The net field at O is 2 £.

mg

Flo
1 cm
11. The surface charge densities on the outer
surfaces of the shells will be equal if the charges on the

ee
X shells are proportional to the squares of their radii.
=>
Hence the charge distribution will be as shown.

Fr
-19 X -27 + 9Q,
=> 1.6x10 X = 1.6x10 xlO
0.01
for
ur
X = lxl0"^V.

6. The work done by the conservative field £ is


ks
Yo

independent of the path followed.


oo

'i
Field, E = E i
eB

A A

Displacement, r =PS = ~a i -b j
Q2 = 4Qi-Qi=3Qj
r

-*
ou

W={qE)- r
ad

Q3=9Q,-4Qi=5Q^
= -{qE't).{a'i +b'] ) =-qaE.
Y

Qi:Q2:Q3=1:3:5.
7. The electric flux through a Gaussian surface
nd

12. The point charge of 3 C lies outside the


Re

is only due to the charges present inside the


Gaussian surface. But the electric field on the Gaussian surface. Therefore, charge enclosed by
Fi

Gaussian surface,
Gaussian surface is due to the charges present both
inside and outside the Gaussian surface. It will be ^ ^isk ^rad ^point
due to all the charges.
-2C
8. .. Electric field lines are perpendicular to the 2 4

equipotential surfaces and electric field is zero Electric flux through the cubical surface
everywhere inside a conductor.
_ q _-2C
9. The fields due to the three sheets at point P ^0 ^0
will point in the negative Z-direction.
Total electric field at point P is 13. (;i Here q^
a 2a 2a ^ These charges are located at CM of hemisphere, at a
£ = (-fc) + — k .
2e 0 2e distance of — from centre
0 2 Eg ^0 2
U22 PHYSICS-XII

R
d = -x2 = R Therefore the electric flux through the shaded
2
portion whose area is a-.x-Jl a = \s
Tta^R^
F = X (j)= EAcosB
47iend^ 4;iEr.rf^ ^0
0 0
= Eq{22o^)cos45°
14. i ' ^ Frequency, 17. F
Cj
r2
1
— will remain same.
2k Vm F _k{2Q)_2kQ
^ R^ r2
An additional constant force will not change the
frequency but will shift the mean position through ^ _ cf(4Q)R _ /cQ
distance ^^£/fc in the forward direction. ^ (2Rf 2R^
15. (u) For equilibrium in the presence of electric Clearly, £2 > E, > £3 .
field, qE = mg 18. ( ●') Suppose at any instant the charge Q is at a
4 3
distance x from the origin. It experiences two forces F
qE = jKrpg
or

and F. On resolving the two forces, the components


f sin 0 cancel out. The resultant force is
When the drop descends with constant velocity v in
the absence of electric field, FnR = 2 F cos 0
43 .
mg = — nr pg =6nr\rv = 2x
kQq X
3
ylx^ + a^
or ^ ^ 9 qy ^ 9 TS>00;;^x2^n02^ 2kQqx
“2P5“2‘ 900x9.8

9 10
= —xl0“
49 That is, F^ is not proportional to x, the motion
cannot be simple harmonic. But this force acts towards
or r = -xl0“^ m the origin and causes oscillatory motion.
7
19. 1. ' and <<) Let OP = Zq
6Knru
E Z

6kx1.8x10 ^x3x10 ^x2x10 ^x7 P(0,0,2o)


81kx 10^x7 E F

= 8.0x10 -”c
R
0
16. ((,1 Given £ = EqX. This shows that the electric
field acts along + x direction and is a constant. The X

area vector makes an angle of 45® with the electric


field. Electric field at P is

1 0
z E =
4TOo(K2 + 22j3/2
7-1
(a, fl, rt) ^
(a, 0, a) This field acts away from O.
t 42/7 0 = 45“
Force on charge -q placed at point P is
^MlI
F =
1 Qt?z,0
0,0,|o) ^^"(0,/7,0) y 4rt£„(R2 + ^2)3/2
X
This force acts towards the centre O.
ELECTRIC CHARGES AND FIELD! (Competition Section)

Hence the motion of the negatively charged


particle is periodic. As
a
/I _ inRl
QB
For Zq «R, F cc ,z 0 4tiRIB
The motion of the particle P will be approximately
simple harmonic.
rI6
X
Qa.RI§-x
QB 47l8oRgV
20. (, Iand u) Inside the sphere {r< R),
RB
E =
1 Q . Option (c) is also correct
i.e., Eccr a
B

O a

Outside the sphere (r > R), Also, — and £g = —


1
^0

w
i.e., E oc
£ a R
A _ A _ B

E a
<1 £^ < Eg
The variation of £ with distance r from the centre is B B

Flo
shown in the figure. Option (d) is also correct.

25. (<0/ (c), (d) As the charges are symmetrically

ee
placed about:r=+fl/2 and x = -al2 planes, the flux

Fr
through them is same.
=> Option (fl) is correct.
Also, the charges are symmetrically placed about
for
ur
y -+al2 and y =-a 12.
=> Option (b) is wrong.
ks
21. I : 1No lines of force can exist inside a conductor According to Gauss law, the net flux through the
Yo
oo

cube is
because £ = 0. Also lines of force enter and emerge
_ Wl _ 7
normally from a conductor.
eB

E ~
^0 ^0
22. i.;) As torque t about Q is zero, L= constant
=> Option (c) is correct.
about +Q. Linear velocity, momentum and angular
r

speed may vary. Again, the charges are symmetrically placed about
ou
ad

23. I- ( IDensity of field lines is more near x= + al2 and z=+a 12 planes, the flux through them is
Y

same.

iQil>IQ2l .●. Option (d) is correct.


Qj and Q2 are of opposite signs.
nd

26. (.'» As +q is displaced towards right, the


Re

=> null point will be closer to the charge of small repulsion due to the right wire is more than that due to
Fi

magnitude i.e., Qj and to the right of Q2. the left wire. A net force on+q acts towards left.
Hence options (fl) and (d) are correct.
24. i/r), (/'). [c). li:'). Electric field inside a
F ^[_1
charged restoring 2ne 01- l-x l+x
spherical shell is zero, so = 0. Option («) is
correct. m2x) Xqx
When the shells are connected by a thin wire, their 27teQ(/^-x^)
charges are transferred till their potentials become
equal. F
restoring
CCX
=> +q executes SHM.
1 Qa _ 1 .Q 6 = V As -qis displaced towards left, the attraction due to
47U8qR^ 4tu80 Rg the right wire is more than that due to the left wire. A
As R^ > Rg, therefore, > Ag. Option (b) is also net force acte on -q towards right making it to move in
correct. the direction of its displacement.
1.12 1 PHYSICS-XII

27. uf) As the sphere has uniform charge density p, 29. ) Given = ^2%) =
it must be an insulating sphere. So the concepts appli Q X a

cable to conducting sphere/cavity cannot be applied. => —^ = =—

Q=2Tta}^
=> Option (fl) is incorrect.
X X
Also, = a =>
'0 =
nr. n<y
0

Option (b) is incorrect.

Now, £/^l = 4£j(r„)


Electric field at any point A inside the sphere,

E = ^ -i- E2
£2f7l = 2£2(’b)=2'^l('b)
-> ( r \ ( r
AP £ 0 ^2E, -a => Option (c) is correct.
3s
0
3s
0
'UJ A2
= -2-(0A+AP)
3s
0
= -^0P
3s
0 £3'f]=E3(>i,)=£2('b)4^2 UJ
(1]
or £=-P a
=> Option (rf) is correct. => Option (d) is incorrect.
3s
0 30. <

23
Here a is the position vector of the centre of cavity PQ =2 Ksin60® =2 Rx = 23R
2
w.r.t. the centre of the sphere.
By Gauss's theorem.
28. I .
! At point Pj,
_

l_Pi(4/3^_p^
4 Tie
0 {2Rf 3s 0
xx23R 2srx
P]R _ p2^ ^0
12 3
=> Option (fl) is true and
Pi option (c) is not true.
— = 4 => Option {d) is correct.
Field due to infinite charged
wire is perpendicular to the wire.
£2 = 0 at all points on the surface of the shell.
=> Option (b) is true.
As the shell is non-conducting, option (d) is not
true.

31. - I

For h>2R and r> R, the


At point .P2' cylinder encloses the shell.
Pj(4/3)nR^ P2(4/3)jc8R^ = 0
^etic ~ Q
{2Rf {5Rf Q h>ZR

Pi _ 32 ^0
—. => Option (fc) is correct.
P2 25
=> Option (a) is correct.
ELECTRIC CHARGES AND FIELDl (Cornpeiition Seciion; /./25

SR 3R
For ft < andr =
3R
Z1 =
2Msm30°_2x2VT0xl0^x(l/2)
5 5 ' 5
a 400^/3xl0^o
the shell encloses the
fs
cylinder.
4R
= j-xlO"^s =
5 6 1/2
q,...
enc = 0
4R
2Msin60° _2x2Vi0xl0S(V3/2)
5
(|) = 0 a 400V3xlO
10

=> Option (ft) is correct. -xl0"*s = j-


2
4R
For ft > 2 R and r= Options (ft) and (c) are correct.
5 '
33. (//), (c) Net force on each sphere decreases but

w
the Coulomb force between the two spheres remains
sm a = — r ^4 the same. Option (a) is correct.
R~5 r

Flo
3
cos a = — a ^
5

ee
4R
2Q
Qfnc g 5

Fr
V.

2Q
4> =
5e 0
for
ur
Option (c) is incorrect.
For equilibrium in air,
3R a
ks
For ft >2R and r = T cos
5 '
Yo
oo

3 4
sma = cos a = — When the spheres are immersed in dielectric liquid,
5 5 a remains same.
eB

0„, = Q|i-|l=f V by 5 Tcos^ = mg-Vp,g


Q
r

T'sin
ou

58 2 K
ad

?ng _ mg-Vp,g
Y

=> Option (d) is correct.


FJk
32. (ft), (,) fl = ^ = 10^° X 400V3 ms-2
nd

1 Pi 1=1-^
Re

m
K 21
sin20 Ps Ps
Fi

R = 21x800
a
Ps =
20
= 840kgm ^
g _ 4xlQxl0^^x sin29 10
=>
Option (c) is correct.
10 X 400%/3
r 1 r
Also, => r=—
u T K 21

I 5 m
=> Option (rf) is incorrect.
+ <? T 34. ( 0 Total charge inside the nucleus is Ze. By
Gauss's theorem.
V3 Ze
sin20 =
2
Ex47rR2 =

26 = 60° or 120°
Clearly, E is independent of the charge
0 =30° or 60°
distribution, instead it depends on the total charge Ze.
L126 PHYSICS-XII

35. (/?) From the similar triangles of the figure. By Gauss's theorem.
in
'£ -

£.47rr^ = 1 f kx‘’ dx
0

1 ^
EAnr^ = — kx‘"^7tx^dx
0

+ 3 +1
£ = k_ ^ or £ =
kr^
R a +3 £Q{fl + 3)
q = jdq = jp^dV = (R - r)~A‘JiP‘dr
R
= Ze
Now £ - -£
0 ^R/2 ~ o‘^R
8
j 32e
tzR^ k R 1 k 13+1
or [R]
36. (l') For uniform volume charge density, £o(fl + 3)L2. 8 eo(fl + 3)
pr
£ = => £ oc r or 2“+^ = 8 =2^ a = 2.
3e 0
40.

37. (c) Let cOp <x 0 0 0 0 6

Putting dimensions. Imagine a cylindrical Gaussian surface of length L


-l6
with its axis on the charged wire and having radius.
[T-’]cc[Q]
a

ML^T‘2 ^2
a

Ne^
On solving, co^ oc
Then, the flux through the ^
me
0

Option (c) is correct. rectangular surface ABCD "2^


will be
38. (1;) (j)£ = Flux through the cylindrical portion of
Ne^ the centre angle 60°
0) =2nv=2n—
p-yl me
0
X
or
1 XL XL
« = 6.
6 e ne
0 0
me
0
X. = 27tc
41.
Ne^
-30 -11
22 8 10 xlO
= 2 X —X 3x 10 m
7
^4xl0^^x(1.6xl0"’^y ‘I’total So
= 600 nm.

39. 'l*hKmispheri: 2cq


0 0
(J
cone
280
ELECTRIC CHARGES AND FIELDl (Competition Section) 1.127

-- £=
42. - —V _200 = 2xlO^Vm"^ If the point mass is displaced towards right by
rf’o.oi Al«I, then
qE
When the oil drop floats,
mg = qE
{l + Alf
—3 nt^pg = NeE
2k'pq
4 (8 xlO-^)x 900x10 = - k{l + Al)~
N = -7l' '
3 1.6xl0“^^x2xl0^ mg

= 71x64x3x10"^-6. = - k{l + Al)-kl 1-


43.2,110
F^,=-4kAl

w
£ (f)= £gSincofi => Spring constant = 4/c
F = qE = qEg sin (ot
Frequency,

Flo
2n\ m
sincof
dt m => 6=71 = 3.14.

ee
45. '>.411

Fr
E = E^siniat
(
V KJ

for
ur
t

^R/4 “^^^0
ks
Yo

T n
V will be maximum at f = —
oo

2 CO
eB

ti.max
;r/(o
dv =
qE.0 sin cofrit
m
0 0
r
ou

V.max
[V] 0
ad

[-coscof] 0
nm
<]r -2 kcSq
Y

0 2g£0
V
max [l-COS7t] =
nun mo3
nd
Re

2xl.0xl.0 1
ms = 2.00 ms"^
lO'^xlO^
Fi

44. >,I4
(t) 6 5 5
For equilibrium of point charge q at distance I,
46.-1

^sp=^e P. Component of forces along z-axis will vanish.


kl = k' -
1
I
Net force is along +ve y-axis.
4 TIC II -
P^3.

■ ‘y
f1

✓ V V S,
s
p / \
✓ S
(0,0) ✓ s

+ Q +Q +Q +Q
X

1.128 PHYSiCS-XII

Q. Component of forces along y-axis will vanish. 47. '

Net force is along +ve j:-axis, 1. For a point charge Q :

E =
1 Q
47l£r,0 (P-
1 1
£ oc

'3 2. For a point on the axis of a dipole :


0^
E =
1 2prf ^ 1 2(Qx2Q
+ Q +Q -Q -Q
47tEg
R. p2 = fg > Fj = Components along x-axis will 1
E oc—r-
vanish. Net force is along -ve y-axis. d^
.-. R-^4.
3. For an infinite line charge :
1 X 1
E = £ a: —
2Ker^d
0
d

X X
Ep -
+ Q -Q -Q + Q
Zne^id-l) 2%EQ{d + l)
X f 21 X(2/)
S. ^2 “ ^3 ^ Components along y-axis will
vanish. Net force is along -ve x-axis.
iTlEQ^d'^-f 27te«d^
0

1
.-. S->2. => Ep 0C--

4. For an infinite plane sheet of charge :


1

£ =^ => E is independent of d
28 0

The correct matching is : P-5, C^3, K-(l, 4), S-2


+ Q -Q + Q -Q

JEEMaIn
4
■ffij
I

Based on Coulomb's Law then brought in contact with C and finally removed
away from both. The new force of repulsion between
Let [Sq] denote the dimensional formula of the B & C is
permittivity of vacuum. If M= mass, L= length, 3F
T = time and A = electric current, then w-
(«)[£„] =[M-'rt"A]
3F
(f.)[£„]=[M-'rtW] ic)^8 (d)
8
(c)[£„] = [M-'lV'A-^]
(d) [£„] =IM-*l2t-'A] [jri'. M.iin 'Pl Three charges + Q, q, + Q are placed respectively,
at distance, 0, d/2 and d from the origin, on the x-axis.
2. Two spherical conductors Band Chaving equal
If the net force experienced by + Q, placed at x = 0, is
radii and carrying equal charges in them repel each
zero, then value of q is
other with a force F when kept apart at some distance.
A third spherical conductor having same radius as {a)+Q/2 (b)-Q/2
that of Bbut uncharged, is brought in contact with B, (c) -Q/4 lILr
ELECTRIC CHARGES AND FIELDl ((;(mii|)i.'Uiimi .HH.i.ioiij

: Two identical charged spheres are suspended by .‘i. Two charges, each equal to q, are kept at :r =-a
strings of equal lengths. The strings make an angle of and :t = a on the x-axis. A particle of mass m and charge
30° with each other. When suspended in a liquid of q
density 0.8 g cir-^ the angle remains the same. If =2 ^
density of the material of the sphere is 1.6 g cm the small displacement (y « a) along the y-axis, the net
dielectric constant of the liquid is force acting on the particle is proportional to
(«)1 {b)4 («)y W-y
(c)3 id) 2 1
(c) (^)-i
nil I'orcns bniwonn Multipin Cluiifjn:. ; y y

Tlie Su|)oiposition Principh; A charge Q is placed at each of the opposite


»»

Three identical charged balls each of charge 2C comers of a square. A charge q is placed at each of the

w
are suspended from a common point P by silk threads other two comers. If the net electrical force on Q is zero,
of 2 m each (as shown in figure). They form an then Q / g equals
equilateral triangle of side 1 (a) -2V2 ib) -1

Flo
1
(c)l id) -

ee
Lhcicil ni 1

Fr
Fl'.-'Ctl i(, < .i l(‘H i|- ''
1(1.
A vertical electric field of magnitude
4.9 X lO^N / C just prevents a water droplet of a mass
for
ur
0.1 g from falling. The value of charge on the droplet
will be (Given g = 9.8 m/s^)
ks
2
(a) 1.6xlO^^C {b) 2.0xl0“^C
Yo

The ratio of net force on a charged ball to the force (c) 3.2xlO~^C (d) 0.5 X 10“^C
oo

between any two charged balls will be


eB

{a) 1:1 (&) 1 : 4 11 The bob of a simple pendulum has mass 2 g and
a charge of 5.0 pC. It is at rest in a uniform electric field
(c)V3:2 (d)V3:l of intensity 2000 Vm“^ At equilibrium, the angle that the
r

Four charges equal to -Q are placed at the four pendulum makes with the vertical (take g = 10ms"') is
ou
ad

comers of a square and a charge q is at its centre. If the (a) tan"'(5.0) (b) tan"'(2.0)
Y

system is in equilibrium, the value of q is (c) tan”'(0.5) (d) tan"'(0.2)


(a) + (f>)5(i + 2V2) I -'. A simple pendulum of length L is placed
nd

4 4
between the plates of a parallel plate capacitor having
Re

(c) -|(1 + 2V2) (<i)S(i+2V2) electric field £, as shown in the figure. Its bob has mass
Fi

m and charge q. The time period of the pendulum is


V- Three charges placed as given by
L
shown in the figure. The x-component of the force on {a) In
qE L
-q^ is proportional to 1 8-’-
Vv r)
m
(fl) -^--^cosO
b a L
+

+
m
q
(h)2n j qE +

(i)| + %sme
b a mj
L L
(c)%+^cose
b^ ■X
(c)2k
2
\2
{d)2n
q^E^
I
-9i + ?2 8 + —
b m

(d)-%-%sin0
b^ lAIIv!;!- 100.’1
' Ii ●
PHYSICS-XII

I ). A particle of mass m and charge q is released qEd


(«)y = 2{x-d) (b)y =
from rest in a uniform electric field. If there is no other mv
0

force on the particle the dependence of its speed v on


qEdfd--X
the distance x travelled by it is correctly given by (c)y =
mvi\2 mvt0
(graphs are schematic and not drawn to scale) 0

(fl) (b)

.. Two point charges +8q and -2q are located at


X = 0 and X = L respectively. The location of a point on
X-axis at which the net electric field due to these two

(c) id) point charges is zero is


L
V. . 11.. (a) 21 (c) 8L (d) 4L

Charges Qj and Q2 are at points A and B of a


right angle triangle OAB (see figure). The resultant
> electric field at point O is perpendicular to the
hypotenuse, then Qj / Q2 is
proportional to
I !, A particle of mass m and charge q has an initial X?
X?
velocity v = VqJ . li an electric field E - EqI and X2
—► A B
^2
magnetic field B = act on the particle, its speed
will double after a time
(C)^ (d)^
^2
3mv 0 ■J2mv,0
(fl) (&)
^^0 ' Two point charges (/j(VlO|iC)and i?2(“25 )i,C)are
placed on the x-axis at x = 1 m and x = 4 m respectively.
V3mu 0 2mv
(c) id)
0
The electric field (in V/m) at a point y = 3 m on y-axis is,
^^0 ^^0
Take^—=9xlO^Nm^Cr^
4Tce
0
I ' A particle of charge q and mass m is subjected to
an electric field £ = EQ(l-iJX^)in thex-direction, where (rt) (-63? +27; )xl0^ (b) (81i -81; )xl0^
a and Eq are constants. Initially the particle was at rest
at X =0. Other than the initial point the kinetic energy (c) (63i-27; )xl0^ (d) (-8I1+81; )xl0^
of the particle becomes zero when the distance of the
Three charged particles A, Band C with charges
particle from the origin is
-Aq, 2q and -2q are present on the circumference of a
(«) j- (b)
1
{c)a circle of radius d. The charged particles A, C and centre
a a
O of the circle form an equilateral triangle as shown in
the figure. Electric field at O along x-direction is
In, A charged particle (mass m and charge <;)moves -JSq 3V3q
(b)
along X-axis with velocity Uq. Wlien it passes through TtBnd^
0 0

the origin it enters a region


yJSq
having uniform electric field r (C)
^ A
£
47t8gd^
£ = ~Ej which extends
o ■►X
2^'3q
upto X = d. Equation of path ^0 id)
7te«d^
of electron in the region ●—d
0

x>dis
ELECTRIC CHARGES AND FIELDl l ■ 'Iiiijjcij, ]()|I i:'( iriiiij /./ ' i

For point charges and-ty are placed on


charge 'a' on it. If the electric
y-axis at y = -2d, y = -d, y = +d and y = +2d, field on its axis at height 'h'
respectively. The magnitude of the electric field £ at « «)from its centre is given as
a point on the :t-axis at a: = D, with D » d, will behave 'Ch', then value of 'C is
as o
(«) — ih)
2az 4ns
1 0 0
(a) E oc
(6)£=c-^ (c)
a

8ne 0
id)
a

as
0
(c) £ oc iD (d)ExX For a uniformly charged irng of radius R, the
electric field on its axis has the largest magnitude at a
distance h from its centre. Then value of h is

w
.( 1 I. H. ii !●
R
‘ ■ li Ij H I' 'O ■.1 ,< 'I ● : («) (b)R
V5
R
A thin semi-circular ring of radius r has a (c) (d) Ry/2

Flo
positive charge q distributed uniformly over it. The net V2
field £ at the centre O is
●Charge is distributed within a sphere of radius

ee
R with a volume charge density = where

Fr
1 r

A and a are constants. If Q is the total charge of this


charge distribution, the radius R is
for
ur
Q
(b) fllogfl Q
a

O
I
(rt)-Iog 1-
2KaAj 2mA
ks
1 1
(b)—^
a
(c) fllog
2nh.r^
}
4n^enr^
1
(rf)^log \\-QI2mA
Yo

0 0
\-Ql2mA
oo
eB

] }
471 271 £n7^
0 0 f ■

A wire, of length L(=20cm), is bent into a


r

semicircular arc. If the two equal halves, of the arc, An electric dipole is placed at an angle of 30° to a
ou
ad

were each to be uniformly charged with charges ±Q, non-uniform electric field. The dipole will experience
Y

(r7) a torque as well as translational force.


(b) a torque only,
nd

(c) a translational force only in the direction of the


Re

H field,
Fi

(d) a translational force only in a direction normal


o X to direction of the field.
O

Determine the electric dipole moment of the


[| Q\ = coulomb, where Eq is the permittivity (in SI system of three charges, placed on the vertices of an
units) of free space] the net electric field at the centre O ^^^fi^teral triangle, as shown in the figure.
A A
of the semicircular arc would be
ji+n
(a) iql)
y/2
(fl)(50xl0^N/C)/ (b) (50xl0^N/C)(‘
(c)(25xlO^N/C)/ (d) (25xlO^N/C)i' (b) ^ql V2

(c) -^qlj
A thin disc of radius b =2i7has concentric hole of
radius 'a' in it (see figure). It carries uniform surface (d)2qlj
/. / 32 PHYSICS-XII

- " An electric dipole of moment


p =(-i-3y+2it ■ A long cylindrical shell carries positive surface
Cm is at the origin
(0, 0, 0). The electric field due to this dipole at
charge a in the upper half and negative surface charge
in the lower half. The electric field lines around the
-CT

r*= + i +3j +5k (note that T. p =0) is parallel to cylinder will look like figure given (figures are
schematic and not drawn to scale) in i
(a){+i+3j-2k ) (b) -3 j-2 k )

ic){-i-3]+2k) id){-i+3j-2k)

Charge -q and +q located at A and B,


respectively, constitute an electric dipole. Distance
AB=2a, O is the midpoint of the dipole and OP is
perpendicular to AB. A charge Q is placed at P where
OP = y and y»2a. The charge Q experiences

QfP' Two charged thin infinite plane sheets of


I

-q I + q uniform surface charge density cr^ and a_, where


A o B |o+l>l'^_!/ intersect at right angle. Which of the
following best represents the electric field lines for this
system ?
electrostatic force P. If Q is now moved along the
r v'l . V («) (b) a
CT.
equatorial line to F such that OF = — and —-3 »2a,

the force on Q will be close to

(b) 3f (c) 9f {b) 27F

' i An electric dipole has a fixed dipole moment p ,


(c) (d)
which makes angle 0 with respect to x-axis. When sub- a

—> A

jected to an electric field = £ /, it experiences a torque


—> A

Tj =tfc. When subjected to another electric field £2 = CT. CT

V3 Ej / , it experiences a torque T2 = -T^. The angle 0 is


(fl)30° (b)45° (c)60° (rf)90°

' ● An electric dipole is formed by two equal and In finding the electric field using Gauss law, the
opposite charges q with separation d. The charges have
is applicable. In the formula, is
nc
formula | £
same mass m. It is kept in a uniform electric field E. If it I '^1
is slightly rotated from its equilibrium orientation
permittivity of free space, A is the area of Gaussian
then its angular frequency co is
surface and is charge enclosed by the Gaussian
(«)
2qE
ib) {c)2
qE
(d) surface. This equation can be used in which of the
1/ md 2md V md V md following situation ?
(fl) For any choice of Gaussian surface.
»»V

ELECTRIC CHARGES AND FIELD! .< oni|j.;tiliun Section)

(b) Only when the Gaussian surface is an ●'I. A cliarged ball B hangs
P
equipotential surface, from a silk thread S, which

(c) Only when \E \- constant on the surface. makes an angle 0 with a large
charged conducting sheet P as
(d) Only when the Gaussian surface is an shown in the figure. The
equipotential surface and |E1 is constant on the surface charge density of the
surface. sheet is proportional to
■ If the amounts of electric flux entering and (fl) COS0 (b) cots

leaving an enclosed surface respectively are 4>j and 4»2, (c) sin0 {d) tan 0 [Aini iOi'-j

the electric charge inside the surface will be


!0. Two infinite planes each with uniform surface
(b) ((l)2 + ({)i)eQ charge density +cyare kept in such a way that the angle

w
between them is 30°. The
(C) (d) electric field in the region
^0 %
shown between them is

Flo
,'.v. Shown in the figure are two point charges +Q given by
and -Q inside the cavity of a spherical shell. The
(i+vS)i)+|
a a

ee
(«)
chargers are kept near the surface of the cavity on
opposite sides of the centre of
2e
o<- bqLI 2/2]

Fr
the shell. If a, is the surface a
rr,1- V3> ^ 1
(c)~ id)
charge on the inner surface 2e
0 2^0 LI if 2_
and Qj net charge on it and ^2 for
ur
the surface charge on the jll-r Main Ian 2C]
outer surface and Q2 net -U. In a uniformly charged sphere of total charge Q
ks
charge on it, then and radius R, the electric field £ is plotted as function
Yo

{(i) =0 ; of distance from the centre. The graph which would


oo

correspond to the above will be


(b) =0,Qj =0; a2=0,Q2=0
eB

(^) E. ib) £i|i


(c) CTj 0,Qj 0 ; ^2 0,Q2 0
(rf) =0 ; a2=0,Q2=0
r
ou
ad

●.*1
'' 'oo; 01*11
Y

i,s. A positive point charge is released from rest at a R R


distance from a positive line charge with uniform ic) id)
nd
Re

density. The speed (u)of the point charge, as a function E E.


of instantaneous distance r from line charge, is
Fi

proportional to

R R
''0
].\i.,r.L ;.;|

42. Consider the force F on a charge ‘q' due to a


uniformly charged spherical shell of radius R carrying
(r?) u X In — (b) u X In — charge Q distributed uniformly over it. Which one of
Tl UoJ ^0) the following statements is true for F, if 'q' is placed at
distance r from the centre of the shell ?

+r/r,0 r
(c) V cce id)v X [a)F = for all r
47iEn0 r
'b j n ’ ‘ :ni,-
' /
I >-i PHYSiCS-XII

1 cjQ The region between two concentric spheres of


(b) > F > 0 for r < R
47te„0 R^ radii V and 'If, respectively (see figure), has volume
1 Q(j charge density p = where is a
(c)F^ ^ for r > R
47i8n0 r
constant and ris the distance from
1 Qi? the centre. At the centre of the
(^)F = j for r < R
4rce„0 R
spheres is a point charge Q. The
value of A such that the electric
Shown in the figure is a shell
made of a conductor. It has inner field in the region between the
radius a and outer radius b, and spheres will be constant, is
carries charge Q. At its centre is a Q 2Q
(fl) (b)
—►

dipole p as shown.
27i(b^-a^) K(a^-b^)
In this case 2Q Q
(d)
na 2na^ ' ●
(fl) Surface charge density on the outer surface
. Consider a sphere of radius R which carries a
depends on | p | R .
uniform charge density p. If a sphere of radius — IS

(b) Surface charge density on the inner surface is


(Q/2)
uniform and equal to E.
4tna^ carved out of it, as shown, the ratio — of magnitude
(c) Electric field outside the shell is the same as that
of point charge at the centre of the shell
of electric field and Eg respectively, at points A and
(d) Surface charge density on the inner surface of
B due to the remaining portion is
the shell is zero everywhere . ■ .. i
Q
Let p(r) = jr be the charge density
kR

distribution for a solid sphere of radius R and total


charge Q for a point 'P' inside the sphere at distance
from the centre of the sphere, the magnitude of electric
field is
Q
(«)0 (b)

18
(c) id) («)rr
4jte.R^
0
34

17
Let there be a spherically symmetric charge (c) —
54
distribution with charge density varying as
(5
p(r)=p 0 IS. Let a total charge 2Qbe distributed in a sphere
4 R
of radius R, with the charge density given by p(r) = cfr,
upto r= R and p(r)=0 for r> R where ris the distance from the centre. Two charges A
where r is the distance from the origin. The electric and S, of-Q each, are placed on diametrically opposite
field at a distance r(r < R) from the origin is given by points, at equal distance, a, from the centre. If A and B
do not experience any force, then
(«) {b) 4^Po^f 5 _ r '
SEq .,4 R^ 3s,,0 Is R, 3R
(b) a = R/V3

(c) £oLfl_£ id) 4ppr^ 5 r ^ ●1/4 -1/4


4eqU Rj 3e« 14 R, (c)fl=2 R (d)a=8 R
0

: II 1 I
||H M
m;: I
ELECTRIC CHARGES AND FlELDl f(OmpiJliLioii Section) L135 4
4'

● .ilV.

1. ' F = 1 ?i^2 or
_ ^1^2 5.
^0 =
4tu8q 4jtfr^

Hence,

^ [AT]^
N.m^ ” MLT-^.L^
2. ( ●●
Charge = 0 <1 q/2 q/2

© +
© 0©
2
'?/2 3g/4 3q/i

w
9

© +
© ©© Force between any two charged balls,
^ kx2x2 = 4A:

Flo
?/2 3(?/4
B
0 r ©^ Net force on a charged ball,

ee
= ^F^ + F^+2F Fcos60° = V3F
f=^M

Fr
47reQ -5i^
F
= V3:l.
1 (|)(-j) 6. ( ● :Consider the equilibrium of charge -Q at
for
ur
F' =
4718 8 comer A.
0
Pd
ks
3. ([■) Few
Yo

A a B
+Q +Q ^-Q
oo

<7 6
-Q

,^0
X ✓

0 d/2 d /
eB

\ /

^ q ^
a X a
For equilibrium of charge +Q placed at at =0,
/0\
r

^_kQxq ^kQxQ =0
y
y s
ou
ad

y N

(rf/2) -Q^ ^-Q


D a C
Y

Q
F ~F
nd
Re

4. I !Using Lami's theorem for the equilibrium of


the sphere. F - ^ _ 1
Fi

F
= tanG
'■ 47180 (V2h)^ 47180 2fl^
7«g F =
1
-
Qg' 1 2Qg

In oil.
F'
= tanG
^ 4TC8o(^fl/2)^ 4jT8 0

mg For equilibrium of charge ~Q at comer A, the net


F
l=m. force on this charge along AC must be zero.
F' 8’ mg
Fq = F^ + Fg cos 45 ®+F£) cos 45®
Fq - ^F2 +2FgCos45

8 or
or K -

o or 1 Q\, 1
47te«0 a 47i8n2fl 47C8n0 -n/2
2_=2
0.8
0

1~
1.6
or <, = ■5(1
4
+ 272).
/./ >(> PHYSICS-XII

7.. Force exerted by +^2 on -q^ along X-axis, 11. I

Tsin0 = qE T COS0
E 2 T
12 “
47ce„0
(along +ve z-direction) T cos 0 = mg .0

Force exerted by -q^ on -q^ along X-axis, tan0 =


qE 4
Tsin 0
●2—► qE
mg
F,3=~.^sin0
4718 -2 0
a _ 5x10-^x2000 = 0.5
mg

2xl0"^xl0
(along +ve x-direction)
e = lan'^0.5).
Total force on -q^ along -i-ve x-direction is 12. ( j Acceleration produced due to electric field,
qE . , . „
F =
JS-f% + %sin0' , a = —, acting horizontally
m

8. <. i Clearly, = 2 F^ cos 0 +

j y +
4 qE
+
0-* -
+
»ig
+
y E

Acceleration due to gravity


= g, acting vertically downwards
Sk
y
S
j~2 2 2
<7 a (1
q

fa\

Fnet
f y J T^2n
L
=2k
I

[V y«a\
Tl %/
or \2
a
3
'qE
. m;

13. (<●,
9.1 j i Net force on charge Q =0
Gain in K.E. = Work done by electric field

^Q]\.
+ K k^- I, QQ ^ mv^ = Fx = qEx
a ^ - ■ (^af V

,QQ
m
or 2 2
2a^ V oc X
=s> Option (d) is correct.
Q
or = 2^/2. 14. ( c ) As V =VqJ, magnitude of velocity does not
change in y-z plane.
Q and q must have opposite signs, so — = -2 -J2.
v^^+vl = v^
10. If ) For equilibrium of the droplet, v^ + vl = {2vQf E = Eni
0
>
qE m, q
B = B.i
mg = qE V =
0
0

Velocity in the direction of E will change.


^ E v^^u^+a^l
0.1xl0”^x9.8 ■JStnv, 0
C-2.0X 10"^ C. ■J%Vn0 =0 + 4^0 t t-
4.9x10® mg m
^^0
ELECTRIC CHARGES AND FIELDl (Comp.Jlilion Section) /. / >7

15. (:/) £= ^ 4- £_^ - 0


,3
W =
qEdx = (?£q (l-ax^)dx = qE^ x^-^^ or 1 8q , 1 _-2q = 0
0 L ^ 4tcc„0 OP^ 4t7ie.0 AP^

As AK£ = W=0, so .X 1 8^7 1 -2q =0


0 “ or
a
4tiSq x^ 4k&q {x-lf
16. .;
4 1
or =0
x^ [x-Lf
2 1
or
X x-L

x=2L

w
or

18. (0
k_Q2

Flo
tan0 = h.=h.= 2
KQ
At point P, = V
0 ^2

ee
t,.,
y 0

Fr
^ - £i
m

qE d Qi ATz
m i>, 0
for
ur
V

Slope, m = tan 0 = —
'y _
2
mv
0 19. (c)
ks

Equation of line passing through the point P{d,-yQ)


Yo

‘^y
oo

E>
and slope m is
qEd 3'
jx + C
eB

y = ...(/)
mv.
0
\

\
At X = d, tn= — 2
r

^0 \
ou
ad

y=Q~- - 0
1
=> y=
1 qE ^ 1
\
Y

^ 2 2 m
0
\

X X ■►.T
From equation (i), (0,0) ] 2 3 4
nd
Re

qEd^
2 ■ mv^0 _,c
2 =. c=l5^
2 mvl E = 1 /}
Fi

mu
0 0
47tEQ
qEd 1 qEd^
9xio^x-^/ToxiQ-^j~-f+3y
j''^2 mv^
y = -
mv
£1 = 10 VTo
^(d_ ]
y =
mun 12
0
^ 9x10^x25x10"^
^2 =
17. ; 'The location of the charges is as shown. 25 5

► £o
E = Ej+ £2
+
-2q
■►X
0 A P = 9 X 10^(-1 + 3 / ) +1.8 X 10^[4 / -3 / ]
H- L -H

H- X -M
= (63j-27; )xl0^Vm"^
/. / ')( )
{ >

PHYSICS-Xil

20. u I The magnitude of electric field of each 2q 22. (i i Refer to the solution of Example 49 on page 1.38.
charge at O is
£ = ) ■
2jte„r
0
2;i:e„r
0 2nh.r^
0

23. (i ) Electric field at the centre of the semicircular


charged wire is
2Q
^ ^ _ L _ Q
27tenfl 2enL 2&nL e-L^
0

■►X lO^e 0
N/C
eo(20xlO'^)^
= 25X10^N/C, alongy-axis.
k{2q) V3 24. (' ) Electric field at an axial point of a charged
= 2x2£cos30°=4x disc at distance x is
" 2
A-j3q _ -J3q £ =
a
1-
X

2s
0 0 OL

21. Electric field due to the complete disc of R =2a at


height h,
2d
a h

d.-^2
£l = 2e
1-

6 0

D
0 ■►X
a h
1-— [For h « a]
-

2s 0>- 2a
-d El 1

+4

-2d d
Electric field due to the disc of R = fl at height h,
-4

^2 = 2s
a
1-^1
Electric field at point P,
a
o>-

£ = 2 £j cos -2 £2 cos ©2 Electric field due to the given annular disc,


oh a
= Ch => C=
2kq D 2kq D E=£i-E2 = 4s„fl isa
0
{d^+ D^)'(d^+ [(2d^)+ D^][{2df+D‘^f^
25. (.)
2kqD[{d^ + ~{4d^ + £ =
kQh
-3/2 n-3/2' h
_2kqD(.1 + d^~^ - 1 +
Ad^
For maximum £,

c
R
— = 0
dh
Using binomial expansion for d « D,

E^2J^ 1-
3 d^ 3 (R^ + h^f^-x^(R^ + h^)^^^.2h
2 2' ^ kQ = 0
(R^ + /i^)^
^2fc^Pri2d^ 3
” £>3 2 2 [R^ + 1i2-3/i2] = 0
(R^ + /i^f
£ = ?^ £oc — h =±
R
D^' J2-
ELECTRIC CHARGES AND FIELDl (Competition Section) L139

26. . ' At the equatorial point P,

E' =
kp 27kpQ
Jr 2 and F = = 27F
(y/3)
r

31. <n p = pcos0i+ psin0;


y

dQ = pdV = p X 4nr^dr fS
Tw
K
“e
Q= fdQ= x47n^rfr = 47tAfe"2''^“dr £, = Hi
0 0

w
47cAfl -2R/a
Q =
-2
]^=27u4fl[l-e -» -»

Torque, T = px E
g-2R/a ^ Q

Flo
2to7A
T^ + i^ = px{£j+^)
2R/a _ 1

ee
Tj-Tj = (pcos0i + psin0y )x(£i +J3E] )
2twA

Fr
R = -ln
a 1 0 = -TSpEcosOA: + pEsmQ(-k)
2 l^l-Q/2raAj
=> p£sin0 = ^/3pEcos0for
ur
27. In a non-uniform electric field, an electric
tan© = -73
dipole experiences both a torque and a translational
force. 0=60®.
ks
Yo

28. 32. (,n


oo
r eB

■►x
ou
ad
Y

P = Pi + Pi
Moment of inertia I 2xm

= -(j/cos30°; -qlcos30°j
nd
Re

A — A
Torque, t = /a
-2,1- j=~j3qlj .
Fi

qExdsmQ = x —
^ 2 dt^
-* -* —^ A A A

29. : Clearly, r .p =0 => r = + i +3j +5k


d^Q ^2qE .sin©
is an equatorial position of the dipole. dt^ md
—» —»

E is antiparallel to p For small 0, sin 0 - 0


A A A

Only vector (+i +3/ -2J: )is antiparallel to p d^Q ^2qE-.0


dt^ md
E is parallel to the vector (+1+3/ ~2k ).
2qE
30. I. ) At the equatorial point P, o> =
V md

E=^ and f =
kpQ
33. (luThe pattern of the field lines should resemble
that of an electric dipole.
/./●/(●; PHYSICS-XII

34. The electric field due to each infinite plane 39. 1. ] At the location of charge q,
sheet is uniform. This rules out options (a) and (c).

£,AI A£ 1
I
I I
I I
h
a.
P
I
I
E2 ^2 I
I

£ E

Now 1 cr^ I >1 o_|, so the electric field due to +ve sheet
in Y-direction is greater than the electric field due to For equilibrium of the forces.
-ve sheet in ar-direction. Clearly, option (d) is correct.
35. /! The magnitude of the electric field is constant jsinQ= F = qE = ^
everywhere and the electric field must be along the
area vector i.e., the surface is an equipotential surface. r cos 0 = mg
36. i.h By Gauss’s theorem. On dividing.
Net flux leaving the enclosed surface
tane = ^^
1
X Net charge enclosed
^0 e
1
or =
xq
^0
or
</= ((t>2 - (j)] )£q.
37. ( /1 Net charge enclosed by the cavity is zero.
The charges + Q and -Q do not induce any charges on
the inner and outer surfaces of the shell.

Q,=o, CTj =0
and Q2=o, a 2 =0.
38.
2ner.r
0
^net - £cos30°{-y )+ £sin30°(-3: )
V
V3V £a
= £ 1- — y —^
dV = Edr = -
2tcs r
2^2
V.- 0
'●o
7
V, -V.; = -
X
-In-
a ,1 VsVV —1''X .
/ 27re 0 ^0 2e
OLV 2 J 2 J
AK+Ali = 0 41. I, < Refer to the answer of Q. 20 on page 1.123.
42. (. ) Inside the shell (r < R),
U
—mv
2_oJ+,(V,-l/.) = 0 £ = 0 => F = qE=0

Outside the shell (r> R),

= ^lnl
2 OT 0 'b 47I£q r
1
i; Qc In—.
i 'b 47T£q
ELECTRIC CHARGES AND FiELDl iUiiii|HHiiion Nrmon) /./●//

43.1:) Final charge distribution


on the shell will be as shown in
the figure.
+

Total charge of the dipole = 0


Net charge induced on outer
surface = 0

But due to non-uniform electric field of the dipole, Charge on this shell,
the charge induced on the inner surface is non-zero
and non-uniform. dq = Akt dr.y p = Anr dr—
r
= AizArdr

At any point outside the shell, the resultant electric Charge enclosed from r = ator = r,
field is due to charge Q uniformly distributed on the
r r^T

w
outer surface is like that of a point charge at the centre
of the shell.
q = ArzA rdr = A'KA — =2TiA(i^-a^)
a -ia

£=^
By Gauss's theorem field at any point at distance r

Flo
44.1: Consider a Gaussian surface of radius r^. from the centre is given by

ee
£x47tr^ = Q+q _ Q + 27iA(r^~a^)

Fr
^0

£x A%^1 - £= Q .-A
for 47re/ 26q 2ey
ur
0

For field to be independent of r,


ks
Q Aa^ Q
Yo

47te«0 2e 0 2m^
oo

or £ =
eB

47.1') E =
3e
0

45. ●j Charge enclosed by a Gaussian sphere of pR


r

radius r(< K) is Ea=- ^ I£A


ou

3CnV2 , 6e
ad

n= prfV= p- ——-']Anr^dr £g| = £ (sphere of radius R)


Y

0
- E (sphere of radius R/2)
r ( c 5 '
nd
Re

r
= 4tip 0, dr = AnpQ - rV
4 R 3 4R ^■kR^P ky~
-n — p
0 V 0 0 3 3 l2 j
Fi

r2 3R ^2

- Anpr,
[5 —
r^ 2 )
° 4 3 4R
1
Using Gauss's law. -TiRp 1-
/ - 18

ExAnt^ = q _ AkPq 5 ^ =
1 4 „
X — jrRp X — -
17 17pR
^0” Eq 4'3 4R 47:Eq 3 18 54s 0

|E^l_ pR,^54£q
4EqL3 R. IEg 1 6bq 17pR
9
46. Consider a shell of radius rand thickness dr
17
as shown in the figure.
LI42 PHYSICS-XII

Electric field at point B,


Tcka^
£x47W^=-^ m —

H ^0

2Q„ 2Qa^
4e«0 kR'^ 4en0 4Tce«R^
0

P ‘4- ► FBA
‘ sphere B

dq = kr.x ‘int^dr
0 1 Q'
QE =
R
47CEo'(2fl)2
2Q = 4Ttfcfr^dr
_ 1
0
Qx
4ite„R‘^ 47ce„0 4a^
2Q^ = i^ k, = —.^
2Q 0
=>
4
a
Charge enclosed in a sphere of radius a, 8
a 4

t’H
= 4;r/:fr^dr = 47cA:— =idca^
4
0

V-'.'- J*' V

Based on Columb's Lav/ 4. The acceleration of an electron due to the mutual

1. When air is replaced by a dielectric medium of attraction between the electron and a proton when
dielectric constant k, the maximum force of attraction they are 1.6 A apart is (m^-9xlO“^^kg,E = 1.6xlO"^^C)
between two charges separated by a distance (Take
47te
^ =9xlO^Nm^C"^)
0
(fl) decreases k times (b) remains unchanged
(fl) lO^ms ^ (&) 10^^ms“2
(c) increases k times
(c) lO^ms"^ (d) 10^ms“^ V : .
(c) decreases times. * (>'●}

5. Two positive ions, each carrying a charge q, are


2. An electron is moving round the nucleus of a
hydrogen atom in a circular orbit of radius r, The separated by a distance d. If F is the force of repulsion
between the ions, the number of electrons missing
coulomb force F between the two is
from each ion will be (e being the charge of an electron)
4Tce^Fd^
0 47C8,,fe^
0
(«) (b)
d^

(d) 4ner.Fd^
0 4n8r.Fd^
0
(c) (d)
\K li-'i-
V
Two point charges A and B, having charges +Q
and -Q respectively, are placed at certain distance 11. Point charges +4q, ~q and +4q are kept on the
apart and force acting between them is f. If 25% charge X-axis at points x=0, x = a and x =2a respectively.
of A is transferred to B, then force between the charges (a) Only -q is in stable equilibrium.
becomes
9F 16f (&) None of the charges is in equilibrium,
(a)
16 (c) All the charges are in unstable equilibrium.
4F
(d)F (d) All the charges are in stable equilibrium.
|\! i.l .h'l
ELECTRIC CHARGES AND FIELD! fiompeuiion soaioni i .l-ij

. Two pith balls carrying equal charges are liobcd on R(;|(.iiion ir.t'.voon Ucctnc hekis,
suspended from a common point by strings of equal I'f:l' !C ●●’..I i(;i j -;b liiu.! cl>;cin<; i'oiCii.'
length, the equilibrium separation between them is r. I i. There is an electric field in the X-direction. If the

work done in moving a charge of 0.2 C through a


distance of 2 m along a line making an angle of 60® with
X-axis is 4 J, then what is the value of £ ?
-I -1
(«) ^/3NC (b) 4NC

(c)5NC^ (rf)20NCr' !■ .
Now the strings are rigidly clamped at half the height. I . An electron falls from rest through a vertical
The equilibrium separation between the balls now distance in a uniform and vertically upward directed

w
becomes electric field E. The direction of electric field is now

^2 reversed, keeping its magnitude the same. A proton is


(«)|4
IV2 allowed to fall from rest in it through the same vertical

Flo
distance h. The time of fall of the electron, in
2A
ic) comparison to the time of fall of the proton, is

ee
{a) smaller (b) equal

Fr
Suppose the charge of a proton and an electron (c) 10 times greater {d) 5 times greater
differ slightly. One of them is -e, the other is (e + Ae). If
the net of electrostatic force and gravitational force
for
ur
between two hydrogen atoms placed at a distance
(. I'. X , I

d (much greater than atomic size) apart is zero, then Ae I,li .. V-l iL.; ]. - lol i-Ui'lC'l'iS

is of the order of
i A semi-circular arc of radius a is charged
ks
[Given mass of hydrogen =1.67x10 kg] uniformly and the charge per unit length is X. The
Yo

electric field at the centre is


oo

(a) 10“^“C {b)W~^C


(c) lO'^^C (d) 10“^^C
eB

2
Two identical charged spheres suspended from a 47u^e,.<7
0 0

common point by two massless strings of lengths /, are


r

initially at a distance d{d «/) apart because of their (0^ (d)


ou
ad

2'KS^n 2ne^a
mutual repulsion. The charges begin to leak from both 0 0

the spheres at a constant rate. As a result, the spheres


Y

approach each other with a velocity v. Then v varies as ik:s i un i'i\ i i!-;i
1 ;
.-'lUwk. i i(.
a function of the distance .r between the spheres, as lk:i ipt(;n .'! I k[:ok
nd

(in;!
Re

(fl) CC x2 (b)v<xx 1 Three point charges+i?,-2 g and +g are placed at


Fi

1
-1
points {x =0,y =a,z=0), (x =0, y =0,z =0) and
(c) U OC X 2 (rf) Z? OC X {x =a,y =0,z=0) and {x-a,y=0,z=0) respectively.
ID. The ratio of Coulomb's electrostatic force to the The magrutude and direction of the electric dipole
gravitational force between an electron and a proton moment vector of this charge assembly are
39
separated by some distance is 2.4 x 10 . The ratio of the
(a) -J2 qa along the line joining the points
proportionality constant, k = to the Gravitational
{x=0,y =0,z=0) and {x ~a,y = a,z =0)
4 Tie 0

constant G is nearly. (b) qa along the line joining the points


(Given that the charge of the proton and electron each (x = 0, y = 0, z = 0) and (x = a, y = a, z = 0)
-19
= 1.6x10 C, the mass of the electron =9.11 x 10 ^^kg.
the mass of the proton = 1.67 x 10“^\g) (c) -\/2iy(7 along+X direction
20 30
(a) 10 (&) 10 {d) A/2gfl along-f-y direction
40
(c)10 (d) 10
/,/-/■/ PHYSICS-XII

I'i, Two point charges -q and +q are placed at a


distance of L, as shown in the figure.
+4}

The magnitude of electric field intensity at a


distance R{R» L) varies as

(«)4R^
1
(b) What are the signs of the two charges ?
R^
(fl) Both are negative
ic)-^
1
id) (b) Both are positive
R^
(c) q^ is positive but q2 is negative
The electric field at a point on the equatorial
t J,

(d) q^ is negative but q2 is positive.


plane at a distance rfrom the centre of a dipole having
dipole moment 'p is given by (r» separation of two c'l: 'H ●! 'I . ●! I ,

charges forming the dipole, Gq = permittivity of free -M. A point charge +qis placed at the midpoint of
space)
a cube of side 1. The electric flux emerging from the
cube is
(b) E
47tenr
0 0 (a) zero m-i-
2P 6ql^
{c)E = id) E ^ (c) id)-4
6lh
0
47te,.r^
0 \i I I ^0 0

I" A point P lies on the perpendicular bisector A charge q pC is placed at the centre of a cube of
of an electric dipole of dipole moment p. If the side 0.1 m. Then the electric flux diverging from each
distance of Pfrom the dipole is r(much larger than face of this cube is

the size of the dipole), then the electric field at P is


proportional to («) (fc)-i-xlO -4
^0 ^0
(fl) p~^ and (b) p and r
qxlQ-^
-i
(c) and (d) p and q'xlO
(c) id)
6s 6e
IS- Torque acting on electric dipole of dipole 0 0
-♦

moment p placed in uniform electric field E is -IS. A charge Qis placed at the comer of a cube. The
-» -*
electric flux through all the six faces of the cube is
(a) px E (b) p.E Q Q
(«) ib)
6s
^0 0
(c) px{Ex p) {d)E.p/f^
Q Q
An electric dipole is placed at an angle of 30° ic) id)
3s 0
with an electric field intensity 2 x lO^N / C It '0

experiences a torque equal to 4 Nm. The charge on the 2-1. An electric charge q is placed at one of the
dipole, if the dipole length is 2 cm, is comers of a cube of side a. The electric flux on one of its
(fl) 5 mC (b)7pC faces will be
(c) 8 mC (d) 2 mC i's I I 1.'

fls &r.a
BosecI on Blociric Hield Lines 0 0

20. Figure gives electric lines of force due to two ic) (^)r^
24 s
charges and q2- 0 0
ELECTRIC CHARGES AND FIELD! (Competition Section) IA45
■ «'■>

25. A square surface of side L metres is in the associated with the cun'ed B

surface B, the flux linked


plane of the paper. A uniform electric E (volt m"’), C A
with the plane surface A in
also in the plane of the paper, is limited only to the
unit of volt metre will be \J. \J
lower half of the square surface as shown in the figure.
1
(«)
2 8 2e
0

>

► F ^0 iCBSl; liuri
> £

30. The electric field in a certain region is acting


The electric flux (in SI units) associated with the radially outward and is given by £ = Ar. A charge
contained in a sphere of radius 'a' centered at the

w
surface is
origin of the field, will be given by
(rt) £L- (b)
2 e
0
(fl) (fj) 4nEQAa^

Flo
£L^
{d) zero
(c) (C) 47U8qA(3^ [●●MPMi I'l
[CBSl: 20061
Based on Applications of Gauss's Theorem

ee
26. What is the flux through a cube of side 'a ' if a
31. Two parallel infinite line charges with linear

Fr
point charge q is at one of its corners ? charge densities +A. C/m and -X C/m are placed at a
00 (^)r^
88
distance of 2R in free space. What is the electric field
0
midway between the two line charges ?
for
ur
2). X
ic)± (c) 2c
-^6(1^ («)
J18„K
~N/C (^)
tis^K
N/C
^0 0 [AIPMT 121 0 0
ks
X
27. A square surface of side L metre in the plane of (c) N/C (rf) Zero
Yo

2%Zr,R
the paper is placed in a uniform electric field £ (volt/m) 0 i.\EI-.i l-j
oo

acting along the same plane at an angle 6 with the 32.A hollow insulated conducting sphere is given a
eB

positive charge of 10 pC What will be the electric field


at the centre of the sphere, if its radius is 2 m ?
(fl) zero (b) 5pCm ^
r
ou
ad

(c) 20pCm'^ (d) 32pCm~- [CBSL-


Y

33. A spherical conductor of radius 10 cm has a


charge of 3.2 x 10"^C distributed uniformly. What is the
nd

magnitude of electric field at a point 15 cm from the


Re

horizontal side of the square as shown in the figure. The centre of the sphere ? — =9xlO^NmVc^
Fi

4ne
0
electric flux linked to the surface, in units of volt m , is
-1
(r?) e6 (b) EL^cosO (i7) 1.28x10^ NC (b) 1.28 X 10^ NC'^
(d) 1.28x lO'^NC
-1
(c) 1.28 X 10^ NC
-]

(c) EL^sinO (d) zero ICUSI: 2010)


[\Li-:i 2(

28. A charge Q is enclosed by a Gaussian spherical 34. A hollow metal sphere of radius R is uniformly
surface of radius R. If the radius is doubled, then the
charged. The electric field due to the sphere at a
distance r from the centre mi
outward electric flux will
(a) zero as r increases for r < R, decreases as r
(fl) increase four times (b) be reduced to half increases for r> R
(c) remain the same (d) be doubled
(b) zero as r increases for r<R, increases as r
ICBSii 2011) increases for r> R
29. A hollow cylinder has a charge q coulomb (c) decreases as rincreases for r< 1? and for r> R
within it. If <j> is the electric flux in unit of volt metre (d) increases as r increases for r < R and for r>R
●N,

■f*
L146 PHYSICS-XII

Answers and Explanations


F.
!●(«) 7. (/;) In ifrst case :
K

2. (t:) Charge on an electron = -e


Charge on nucleus of hydrogen =+e
fee' -*

7'
A r
Here r =— is unit vector along line joining
r

electron to the nucleus. The negative sign shows that


the force is of attraction.

3. {a)
+Q F -Q
o- ¥ ■o In second case :
/t r B

= mg tan 0'
(attraction) I
Fj! _ tan0'
I
tan0
When 25% charge of A is transferred to B,
kq- F/2
3Q Q 3Q
4 F' F'
4 O
4 _ y/2
A r B
fcfl' '■/2
y img
J3QY3Q^
r =
I 4 A 4.
7 ^ W

9kQ^ _ 9F (attraction)
8. (c) Net charge on one H-atom = -e + e + Ae = Ae
16r ~ 16 For net force between two H-atoms to be zero,
1
4. («i) F = - Fp (repulsive) = F^ (attractive)
47r£„0 r'
1 Ac' G?«'
F e'
4TOq d' rf'
47iEQ»i^.r'
t'

9x I0^(Ae') = 6.67x 10 (1.67x10-2^)'


-11
X

9xl0^x(1.6xlQ-^^)' ms -2 6.67x1.67x1.67 -74


Ae' = xlO
9xl0-^^x(1.6xl0-'‘’)' 9

= 10"ms^'. Ae «10"^'.
2..2
1 1 e n 9. (e)
5.(e) F = [v <j = »ie)
47:e
0 d~ 47T8^ d^ From AACO of forces.

or n =
47TSQFd'
^ e' ■
6. (c) The net force on each charge is zero.
Therefore, all the charges are in equilibrium. If we
slightly displace the charge ~q to the irght, the net force
of attraction will further displace it to the right i.c.,
away from its mean positive. The equilibrium is, 2kq^ _ mg
I
therefore, unstable.
ELECTRIC CHARGES AND FIELDl (Competition Section) 1.147

14. (^7) The given charge assembly is equivalent to


q^='^.x^
^ 2kl two dipoles. One dipole of charges -q and +q has
q^ = Cx^
dq = 3Cx^^
dt dt

V =
dx _ 2q dq
^~3Cx^ 'tT
dq
As —
dt
is constant for both spheres, so
3

a;2

w
D cc V oc
x2-
1

V <K X ^
dipole moment along +ve X-direction and other

Flo
—>

10. (,o
dipole of charges -q and +q has dipole moment pj
—^

kq^qi along + ve Y-direction. The resultant dipole moment

ee
p
E
E _
has magnitude.

Fr
P = Vpi + =42qa
k (1.6x10"^^)^
for
ur
-27 -31
G 1.67x10 x9.11xl0 p is directed along OP, where P is (o,fl,0).
20
39 k 1.6x1.6x10 15. (1») The given arrangement is an electric dipole
ks
2.4x10
G 1.67x9.11 of length L
Yo
oo

k 2.4x1.67x9.11x10
39 For R » L, the electric field at an axial point.
G
20
2P
eB

1.6x1.6x10 £ = , where p~qL


= 14.26X 10-^ = 1.426 x 10^” :i 10^®. 47te.P^
0

£oc —
11. {d) Here = 0.2 C, s = 2 m, 0 = 60°, W = 4 J
r
ou
ad

But W =s PscosG -qEscosQ 16. (a) Electric field at an equatorial point of a short
Y

W
or £ = dipole.
qs cos Q
nd

4 £=-
Re

= 20NC’. 47ie„r^
0
0.2 X 2 X cos60°
Fi

e£ 17. (d) At any far away equatorial point of an electric


12. (,7) F = ma = eE
dipole.
ni

1 £ = _L_Z i.e., E <x pr


s= ut + —at^ 4TtEr.0
2

;, = o+l£^,^ 18. (rt) Torque on a dipole, t = px E.


2 m
19. (d) X = pEsinO = g(2fl)£sin0
t =
eE 4 = qx2x lQ-~ X2 X10^sin30°
As electron has smaller mass, its time of fall will be
q = 2xlO~^C =2mC
smaller than that of proton. 20. (rt) Both the charges are negatively charged
because the lines of force are directed towards the two
13. (c) Refer to the solution of Example 49 on
page 1.38. charges.
.148 PHYSICS-XII

■i,:

21. (/’) By Gauss' theorem, (l)^ = — . 30. (b) (|) = EAS =


^0 ^0
-6

22 (rt) ({) ^ 6
Net charge enclosed _ t/x6e10 Aay. 4tw^ = —
0

23. (/O When a charge Q is placed at one comer of


the cube, only one-eighth of the flux emerging from 31. {b)
charge Q passes through all the six faces of the cube. 2

Q +

"“6b/ +

+ P*
24. id) When charge q is placed at one comer, the +
£2
+
flux through each of the three faces meeting at this +

comer will be zero, as £ is parallel to these faces. +X -X


2R
One-eighth of the flux emerging from charge q passes
through the remaining three faces, so the flux through £ = £| + £2
each such face is
X X
1 1 q J
^ 3 8 e.0 24 e 0 0 0

25. (d) As electric field is parallel to the square E =


X
NC~\
surface, electric flux crossing this surface will be zero. kZqR
26. (1j) When the charge ^is placed at one comer of
32. (rt) Electric field is zero at all points inside a
the cube, only one-eighth of the flux emerging from the hollow charged conducting sphere.
charge q passes through the cube.
33. (p)
^ 8e/ 47i£n0 r
0

27. (d) As the field E lies in the plane of the surface, 9x10^x3.2x10”^ -1
NC
it is perpendicular to the normal to this surface. (15x10"^)^
(|) = £ . S = £Scos90° = 0. = 1.28xlO^NC”V
34. (a)
28. (c) As the charge Q enclosed by the Gaussian
surface remains same, so E

4>=<7/e 0' will also remain same.

29. (rt) Let the electric flux linked with surfaces A, B 1


,2
and Cbe (j)g and (j)^ respectively.
Then ^ h+ =~ r=R
*■
% r

But
'/I For a uniformly charged hollow metal sphere.
£ = 0
or 24)^=^-(J.g
inside

^0 ^0 and £
'outside
47ier,0 r"
Given ’b ~
2 £-
\''Q
=> Eccl.
r
C H APTER

ElecfrosfBfic Pofentia

w
and Capacil'ance
Flo
ee
Fr
for
ur
charge + q. We then calculate the potential difference
2.1 ELECTROSTATIC POTENTIAL AND
ks
between points A and B by the equation :
POTENTIAL DIFFERENCE
Yo

W
oo

AB
...(2.1)
Introduction. The electric field around a charge can %
be described in two ways :
eB

—> So the potential difference between two points in an


(i) by electric field (E), and electric field may be defined as the amount of work done in
moving a unit positive charge from one point to the other
r

(//) by electrostatic or electric potential (V^).


ou

against the electrostatic forces.


ad

The electric field £ is a vector quantity, while In the above definition, we have assumed that the
Y

electric potential is a scalar quantity. Both of these test charge is so small that it does not disturb the
quantities are the characteristic properties of any point distribution of the source charge. Secondly, we just
nd

in a field and are inter-related.


Re

apply so much external force on the test charge that it


just balances the repulsive electric force on it and hence
Fi

1. Develop the concepts of potential difference and does not produce any acceleration in it.
electric potential. State and define their SI units. It has been
SI unit of potential difference is volt (V).
Potendal difference. As shown in Fig. 2.1, consider named after the Italian scientist Alessaytdro Volta.
a point charge + q located at a point O. Let A and Bbe
1 joule
two points in its electric field. When a test charge is 1 volt =
1 coulomb
moved from A to B, a work has to be done in
moving against the repulsive force exerted by the or lV = lNm C”^=l JC"^
Source Test Hence the potential difference between two points
charge charge in an electric field is said to be 1 volt if 1 joule of work has
to be done in moving a positive charge ofl coulomb from one
O B A point to the other against the electrostatic forces.
Electric potential. The electric potential at a point
Fig 2 1 define potential difference. located far away from a charge is taken to be zero.
(2.1)
2.2 PHYSICS-Xll

i
In Fig. 2.1, if the point A lies at infinity, then
r r
= 0, so
w = rfW = - Fdx = -
tliat
4ti e x~
CO CO 0
w
r

% f X -Ux=--^ iV
where W is the amount of work done in moving the
4ji s
0 CO 4iT Eg L X - CO

test charge from infinity to the point Band Vg refers 5% 1 1 1 n


to the potential at point B. 4TI Eg L'' CO
4tc Eg r
So the electric potential at a point in an electric ifeld is
Hence the work done in moving a unit test charge
the amount of work done in moving a unit positive charge
from infinity to the point P, or the electric potential at
from infnihj to that point against the electrostatic forces.
Work done point P is
Electric potential = W
1/ = — V = 1 /I
Charge or

% 4tc Eg r
SI unit of electric potential is volt (V). The electric
Clearly, V ocl/r. Thus the electric potential due to a
potential at a point in an electric ifeld is said to be 1 volt if
point charge is spherically symmetric as it depends only
one joule of work has to be done in moving a positive charge
on the distance of the observation point from the
of 1 coulomb from infinity to that point against the
charge and not on the direction of that point with
electros tatic forces. respect to the point charge. Moreover, we note that the
2.2 ELECTRIC POTENTIAL DUE TO potential at infinity is zero.
A POINT CHARGE Figure 2.3 shows the variation of electrostatic
potential (y oc 1 / r) and the electrostatic field (£ oc 1 / r^)
2. Derive an expression for the electric potential at a with distance r from a charge q.
distance r from a point charge q. What is the nature of 5

this potential ? 4.5

Electric potential due to a point charge. Consider a 4

positive point charge q placed at the origin O. We wish 3.5

to calculate its electric potential at a point P at distance 3

rfrom it, as shown in Fig. 2.2. By definition, the electric t2.5


potential at point P will be equal to the amount of work ^ ,
done in bringing a unit positive charge from infinity to ^ ^
the point P. 1
dx
‘I % F
0.5
©- 00

O P B A 0
H- r
0 0.5 1 1,5 2 2.5 3 3.5 4 4.5 5

H- X ♦I

Fig. 2.3 Variation of potential V and field E


Fig. 2.2 Electric potential due to a point charge. with r from a point charge q.
Suppose a test charge r/g is placed at point A at
distance a: from O. By Coulomb's law, the electrostatic 2.3 ELECTRIC POTENTIAL DUE TO A DIPOLE
force acting on charge ^g is 3. Derive an expression for the potential at a point
F = 1 /}% along the axial line of a short dipole.
4k En0 X^ Electric potential at an axial point of a dipole. As
-* shown in Fig. 2.4, consider an electric dipole consisting
The force P acts away from the charge q. The small point charges - q and + q and separated by
work done in moving the test charge ijg from A to B distance 2a. Let Pbe a point on the axis of the dipole at
a distance r from its centre O.
through small displacement dx against the electro -<1
static force is 1

A O B P
dW = F .dx - Fdx cos 180° = - Fdx
14- a -H4- a ■M

The total work done in moving the charge t/g from 14^ r -H

infinity to the point P will be Fig. 2.4 Potential at an axial point of a dipole.
J
ELECTROSTATIC POTENTIAL AND CAPACITANCE 2,3

Electric potential at point P due to the dipole is Let AP - and BP = T2- P

1 zi + _2_.X Net potential at point P due to the dipole is , /'


4ti e 0 AP 4ti 0
BP

1 ‘7 1 '7
+ ■
1 -‘7
+
1 ^ ^■2
4jt£q r+a 47C£y r~a 4ti e
0 ''l Ak e 0 ^2 r

/ v-.
1 1 <7 1 1

4n E r~a r+a 4n £ / P
0 L 0 L ^2 h A
9 ●
B

^ \h~h
O +<7
1 c\.yla M 2(J

47TE 0 4JTE,,0 471 E 0 '*1^2


Fig. 2.6

w
1 p
or V^ = [V p = qx2a] If the point P lies far away from the dipole, then
471 Eg - K, - AB cos 6 = 2 « cos 0 and
2 2 1 P
For a short dipole, o <<r, so V= .-;r. q 2a cos 0

Flo
^ 47iEg 4jt £y
4. Show mathematically that the potential at a point

ee
1 p cos 0
on the equatorial line of an electric dipole is zero. or V =
47t£ r
●>

Fr
0
Electric potential at an equatorial point of a dipole. ->
A

As shown in Fig. 2.5, consider an electric dipole or


1 P-f _ 1 p .r
consisting of charges - q and + q and separated by 4t; £ 0 471 £ 0
for
ur
distance 2a. Let P be a point on the perpendicular
bisector of the dipole at distance r from its centre O. Here p = qx 2a, is the dipole moment and r = r Ir,
—» —»

is a unit vector along the position vector OP = r .


ks
p
Yo

Special Cases
oo

(i) When the point P lies on the axial line of the


eB

dipole, 0 =0° or 180° and


r

V = ±
^ P
4tt Eg
r

a a
ou

-49. i.e., the potential has greatest positive or the


ad

+ 4
A O B
greatest negative value,
Y

Fig. 2.5 Potential at an equatorial point of a dipole. (li) When the point P lies on the equatorial line of the
dipole, 0=90°, and V =0, i.e., the potential at
nd

Electric potential at point P due to the dipole is any point on the equatorial line of the dipole is
Re

zero. However, the electric field at such points


Fi

is non-zero.
V=V, + V2 = 4t: £„0 AP 4?: BP
0
6. Give the contrasting features of electric potential of
1 *7 1
3— = o. a dipole from that due to a single charge.
4 71 E 0
Differences between electric potentials of a dipole
and a single charge.
5. Derive an expression for the electric potential at
any general point at distance rfrom the centre of a dipole. 1. The potential due to a dipole depends not only
on distance r but also on the angle between the position
Electric potential at any general point due to a
vector r of the observation point and the dipole moment
dipole. Consider an electric dipole consisting of two
point charges - q and + q and separated by distance 2fl, vector ]). The potential due to a single charge depends
as shown in Fig. 2.6. We wish to determine the potential only on r.
at a point P at a distance r from the centre O, the direc-
2. The potential due to a dipole is cylmdrically symmetric
tion OP making an angle 0 with dipole moment p. about the dipole axis. If we rotate the observation
PHYSICS-XII

point P about the dipole axis (keeping r and 0 fixed), 2.5 ELECTRIC POTENTIAL DUE TO A
the potential V does not change. The potential due to a CONTINUOUS CHARGE DISTRIBUTION
single charge is spherically symmetric.
8. Deduce an expression for the potential at a point
3. At large distance, the dipole potential falls off as
due to a continuous charge distribution. Hence write the
1 / while the potential due to a single charge falls off expression for the electric potential due to a general source.
as 1 / r.
Electric potential due to a continuous charge
2.4 ELECTRIC POTENTIAL DUE TO distribution. We can imagine that a continuous charge
A SYSTEM OF CHARGES distribution consists of a number of small charge
7. Derive an expression for the electric potential at a
elements located at positions If ^ is the position
point due to a group of N point charges. vector of point P, then the electric potential at point P
due to the continuous charge distribution can be
Electric potential due to a group of point charges. written as
As shown in Fig. 2.7, suppose N point charges 1 dq
q^, qi> lie at distances r^, r^, , from a 4ne
0
->

point P. r - I
When the charge is distributed continuously in a
volume V,dq=p dV, where p is volume charge density.
The potential at point P due to the volume charge
distribution will be
1 pdV
^v = 4n8..x ●’
●o‘?4 0 V \r - rI

When the charge is distributed continuously over


an area S, dq = cs dS where a is surface charge density,
Fig. 2.7 Potential at a point due to a 1 a dS
system of N point charges. ^s = 4718
0 s [ r - t;. I
Electric potential at point P due to charge t/j is
When the charge is distributed uniformly along a
K1 = line L, dq = XdL, where X is line charge density,
4ti: 8
0 'i 1 Xdl
1^^ =
Similarly, electric potentials at point P due to other 4ti:8
Q L \ r - .r ,

charges will be
^2 1 % 1 The net potential at the point P due to the conti
V2=
^ —47te 47re
KN “ nuous charge distribution will be the algebraic sum of
0 h 0 h 4718
0
the above potentials.
As electric potential is a scalar quantity, so the total
potential at point P will be equal to the algebraic sum
cont

of all the individual potentials, i.e., 1 pdV^ f <sdS f XdL


or .Vcont — + +
47t e I -4 L
0 v\r - r.\ s 1 r - r. I L I r -
1
Electric potential due to a general source. The
4tI8 0
"l ’2 'S potential due to a general source charge distribution,
which consists of continuous as well as discrete point
or i- I ^ charges, can be written as
4tC8q i = I V = V
cont
+ Kdiscrete

If Tj, r2« r^,...,r^ are the position vectors of the N 1 p dV odS


or

point charges, the electric potential at a point whose 47TS 0 -»

—► V I r - .r s \ r -r.
position vector is r, would be
N Xdl
l/ =
1
z —^ +
Z
47ie«0 /=],-♦
r - r
L \ r - rI Ali point I f - r
j
I charges
ELECTROSTATIC POTENTIAL AND CAPACITANCE 2.5

2.6 ELECTRIC POTENTIAL DUE TO A For Your Knowledge


UNIFORMLY CHARGED THIN
SPHERICAL SHELL > Electric potential is a scalar quantity while potential
gradient is a vector quantity.
9. Write expression for the electric potential due to a > The electric potential near an isolated positive charge is
uniformly charged spherical shell at a point (i) outside the positive because work has to be done by an external
shell, (ii) on the shell and (Hi) inside the shell. agent to push a positive charge in, from infinity.
Electric potential due to uniformly charged thin > The electric potential near an isolated negative charge is
spherical shell. Consider a uniformly charged spherical negative because the positive test charge is attracted by
shell of radius R and carrying charge q. We wish to the negative charge.
calculate its potential at point P at distance r from its
> The electric potential due to a charge q at its own
centre O, as shown in Fig. 2.8. location is not defined - it is infinite.

> Because of arbitrary clioice of the reference point, the

w
P
Shell with electric potential at a point is arbitrary to w’ithin an
r
charge Q additive constant. But it is immaterial because it is the
potential difference between two points which is

Flo
physically significant.
> For defining electric potential at any point, generally a

ee
point far away from the source charges is taken as tlie
O
reference point. Such a point is assumed to be at infinity.

Fr
R

> As the electrostatic force is a conservative force, so the


work done in moving a unit positive charge from one
for
ur
Fig. 2.8 Potential due to a Fig. 2.9 Variation of potential point to another or the potential difference between
spherical shell. due to charged shell with two points does not depend on the path along which
distance r from its centre. the test charge is moved.
ks

(/) What the point P lies outside the shell. We know


Yo

Examples based on
oo

that for a uniformly charged spherical shell, the electric Electric Potential
field outside the shell is as if the entire charge is
eB

concentrated at the centre. Hence electric potential at Formulae Used


an outside point is equal to that of a point charge Work done W
1. Potential difference = or V =—
located at the centre, which is given by Charge
r
ou
ad

[For r> R] 2. Electric potential due to a point charge q at


47re„0 r
distance r from it,
Y

(ii) When point P lies on the surface of the shell. Here V = -J—J
r = K. Hence tlie potential on the surface of the shell is Atieq ' r
nd
Re

1 ^
[For r = R] 3. Electric potential at a point due to N point charges,
4n:e.^0 R
1 N q.
Fi

{Hi) When point P lies inside the shell. The electric field 4nSo r = i .r
at any point inside the shell is zero. Hence electric
4. Electric potential at a point due to a dipole,
potential due to a uniformly charged spherical shell is
constant everywhere inside the shell and its value is V = 1 pcos 0 _ 1 P* . F
equal to that on the surface. Thus, 471 En0 4n Sn0 ’ r^
V =
1 g Units Used
[For r<R]
4rt£.,0 R
Charge q is in coulomb, distance r in metre,
Figure 2.9 shows the variation of the potential V work done W in joule and potential difference V in
due to a uniformly charged spherical shell with volt.
distance r measured from the centre of the shell. Note
that V is constant {= (// 47t£QR) from r = 0 to r = R along Example 1. The work done in moving a charge of 3Cfrom
a horizontal line and thereafter Vccl/r for points a point A, loliich is at a potential of ~2V, to another point B
outside the shell. is 18 }. Find .VB-
●<<

k 2.6 PHYSICS-XII

Solution. Here q^3C, W^y=18J We can consider tlie circular wire to be made of a

Work done
large number of elementary charges dq. Potential due
Potential difference = to one such elementary charge dq at the centre,
Charge 1 dq
dV =
W 47T s r
P,B 0

Total potential at the centre of the circular wire,


1 V 1
V=: dV = dq = [^]o £., r
0
WB = 6 -2 = 4 V.
1 _9x 10^x2 xlO"^ = 18 X 10^ V.
Example 2. Find the electric potential at the surface of an ~ ^ ●~ 0.10
ntoinic nucleus (Z = 50) of radius 9.0xl0“^^c?«. Given ®
charge on a proton =1.6x 10“^^C. Example 5. Electric field intensity at point 'B' due to a
Solution. As the nucleus is spherical, it behaves point charge Q kepH at point A is 24 NC and the
like a point charge for external points. As 2=50, it electric potential at point B due to same charge isl2 JC
contains 50 protons. Calculate the distance AB and also the magnitude of charge
Q- [CBSE OD 03C|
= ne = 50x 1.6x 10“^^C
r = 9.0xl0"’^cm = 9xl0 "m -15 Solution. Electric field of a point charge,

p = 1 _9xl0^x 50x1.6x10
-19
V
£ = i-.-^=24NC"’
-15
47teQ r 9x10
Electric potential of a point charge,
= 8xlO^V.
P = ^ ,^=12JC“^
4716 0 r
Example 3. (i) Calculate the potential at a point P due to a
charge of 10"^ C located 9 cm away, (ii) Hence obtain The distance AB is given by
the work done in bringing a charge o/ 2 x 10"^ C from P 12
— = 0.5 m
. -
infinity to the point P. Does the anszver depend on the path £ 24
along which the charge is brought ? [NCERT]
The magnitude of the charge,
Solution. (/) Here q = Ax 10"^ C, r=9 cm =0.09 m
Q = 4tU Cn V> = — X X 12 X 0.5= 0.667 X 10"® C
Electric potential at point P is ^ " 9x10^
I 4 X 10"^
P = .-5=9x10®x = 4x 10^ V. Example 6. To zvhat potential we must charge an insulated
4ti 6 0 0.09
r
sphere of radius 14 an so that the surface charge density is
Jj=4xl0"^c equal to\p Cm' ^ ?
P
00
Solution. Here r = 14 cm = 14 x 10"^ m.
O
9 m a = lp Cm“^=10“^ Cm“^
Fig. 2.10 1 1 4;i r^o 1
P = .4t: ro
(//) By definition, electric potential at point P is 47C6y r 4jre
0
r 4716
0

equal to the work done in bringing a unit positive


= 9xl0®x4x—x14x10"^x10“^ V
charge from infinity to the point P. Hence the work 7
done in bringing a charge of 2 x 10“ ® Cfrom infinity to = 15840 V.
the point P is
Example 7. A charge oflA pC is given to a hollow metallic
W = ijo P =2 X 10“® X 4 X 10^ = 8 X 10"® J sphere of radius 0.2 m Find the potential [CBSE D 95]
No, the answer does not depend on the path along
(i) at the surface of the sphere, and
which the charge is brought.
(ii) at a distance of 0.1 cm from the centre of the sphere.
Example 4. A metal wire is bent in a circle of radiuslQ cm
It is given a charge of 200 pC which spreads on it uniformly.
Solution. (/) q =24 pC =24 x 10"^ C, R =0.2 m
Calculate the electric potential at its centre. [CBSE OD 95C| Potential at the surface of the sphere is
Solution. Here q = 200 pC = 2 x 10"“^ C, P =
1 q 9 X 10® X 24x10"'^ V= 1.08x10^ V.
4tC6 R 0.2
r = 10 cm =0.10 m 0
ELECTROSTATIC POTENTIAL AND CAPACITANCE 2.7

(n) As potential at any point inside the sphere The other possibility is that x may also lie on OA
= Potential on the surface produced, as shown in Fig. 2.12.
Potential at a distance of 0.1 cm from the centre
(j, = 3xl0"*C ij2=-2xl0 C
= 1.08 X 10^ V. O- ■o-
o A

Example 8. Twenti/ seven drops of same size are charged at 0.15 .t-0.15

220 V each. They combine to form a bigger drop. Calculate


the potential of the bigger drop. [NEET21] Fig. 2.12
Solution. Let radius of each small drop = r
As v^,4-v,=o
Radius of large drop = R -8 -8
1 3x10 2x10
As the volume remains constant, =0
4 ^ 4 T 4k e. ;c-0.15
-nR^=27x-nr^

w
3 7
which gives x = 0.45 m = 45 cm
R=3r
Thus the electric potential is zero at 9 cm and 45 cm
By charge conservation, Q =27q away from the positive charge on the side of the

Flo
kq = 220 V negative charge.
smaller ^
Example 10. Calculate the electric potential at the

ee
centre
kQ_kx27q = 9 of a square of side -Jl m, having charges 100 pC, - 50 pC,

Fr
,V V
bigger R 3r
smaller
20 pC, and -60 pC at the four corners of the square.
[CBSE OD 06C|
= 9x220V=1980 V.
Solution. Diagonal of the square
for
ur
Example 9. Two charges3x 10"® Cand -2 x 10“® C are
located 15 an apart. At what point on the line joining the = ^{j2f+iJ2)-=2 m
two charges is the electric potential zero ? Take the potential
ks
Distance of each charge from the centre of the
at infinity to be zero. [NCERTl square is
Yo
oo

Solution. As shown in Fig. 2.11, suppose the two r = Half diagonal = 1 m


point charges are placed on X-axis with the positive .●. Potential at the centre of the square is
eB

charge located on the origin O.


-2xlO’'*C
!L + 5i + 5i + ^
(7j=3 X 10 ‘?2 =
4ji8q L r r r r
r

o- ■o
ou

O P A
ad

X 0.15-:r 100x10'^ 50x10"^


V=9xl0'^
Y

1 1
Fig. 2.11 Zero of electric potential for two charges.
20 X 10"^ 60 X 10”^
+
nd

Let the potential be zero at the point P and OP = x.


Re

1 1
For X <0 {i.e., to the left of O), the potentials of the two
= 9x 10% 10"^xl0=9x 10^ V.
Fi

charges cannot add up to zero. Clearly, x must be


positive. Example 11. Four charges + q, + q,~q and - qare placed
If x lies between O and A then respectively at the corners A, B, C and D of a square of side
'a' arranged in the given order. Calculate the electric
potential at the centre O. If
1
5l ^2 E and F are the midpoints
+

= 0
4ti e
0 L
X
0.15-.rj of sides BC and CD respec
tively, what will be the
-8
3x10 2x10"’ work done in carrying a
or 9x10^ =0
X 0.15-X charge 'e' from O to E and
from O to F ?
3 2 Solution. Let OA = +
or = 0
X 0.15-.T OB=OC = OD = r.

which gives x =0.09 m =9 cm


PHYSICS-XIl

Then the potential at the centre O is dipole in broad-side-on position (ii) electric field and electric
1 ^
potential at the same point after rotating the dipole through
V,o =0 90°.
~
47CEy \_r r r r
Solution. Dipole moment,
Again, the potential at point E is p=(jx2/=3xl0‘^x2xl0"^=6x 10“^ Cm
^£= J_ + A 2 ^ = 0 (0 Electric field in broad-side-on position is
^ —4716 AE BE CE DE
0 L
E =
1 p 9x10^x6x10"^ = 250 NCT^
[■: AE = DE, BE = CE] 4718,,0 r^ (0.6)'
\2
\[5a Electric potential in broad-side-on position, V = 0.
Now, AF=BF=^V a^+(-
V2J 2 (ii) When the dipole is rotated through 90°, the
Tlie potential at point F is same point is now in end-on-position with respect to
the dipole.
1 ^ JL +A 1 £ = —. ^ = 500 NC'^
4tC8 0 AF BF CF DF 4ti:£ 0
L

2q r 1 ^ [●.● AF = BFr CF = DF]


1 p 9x10^x6x10"^ = 150 V.
AkSq _AF CF 47180 (0.6)2
2t? r 2 2 1
-1 Example 14. Fwo charges -q and + q are located at points
4tc80 i^fSa a 7^e,^ a
0 I Vs /4(0,0,-fl) and B(0,0,+/7) respectively. Hozo much work is
done in moving a test charge from point P(7,0,0) to
Work done in moving the charge 'e' from O to £ is
Q(-3,0,0) ? [CBSE D 09]
W = e[V£-VQ] = ex0 = 0 Solution. Points P and Q are located on the equa
Work done in moving the charge 'e' from O to E is
torial line of the electric dipole and potential of the
W=e[V,-Uo] = e [Tte^flWS
-Lfl-i -0 dipole at any equatorial point is zero.
.'.Work done in moving a test charge (jy from PtoQ,
qe f 1 W = ‘7o(^q-^p) = ‘7o(0-0) = O.
7C8o«V V5 X

P{7,0, 0)
Example 12. A short electric dipole has dipole moment of
4 X 10'^ Cm Determine the electric potential due to the
dipole at a point distant 0.3 mfrom the centre of the dipole
situated (a) on the axial line (b) on equatorial line and (c) on + 4
a line making an angle of 60° with the dipole axis. ●●● ■►Z

Solution. Here p = 4xlO~^Cm, r = 0.3m. /^(O, 0. - rt) B(0, 0, a)

Q(-3, 0, 0)
(rt) Potential at a point on the axial line is
1 p _9x 10^x4x10"^ = 400 V. y
4718^0 p2 (0.3)^
Fig. 2.14
(1?) Potential at a point on the equatorial line = 0. roblems For Practice
(c) Potential at a point on a line that makes an angle
of 60° with dipole axis is 1. The electric potential at 0.9 m from a point charge is
+ 50V. What is the magnitude and sign of the
P =
1 pcos 0 charge ? ICBSE D 95C]
■>
4ke 0 r
(Ans. 6 X 10" ^ C, positive)
_ 9 X 10^ X 4 X 10"^ cos 60° = 200 V.
2. The electric field at a point due to a point charge is
“ (03? 20 NC"’ and the electric potential at that point is
10 JC”^. Calculate the distance of the point from the
Example 13. Two point charges of+3pC and-3 pC are charge and the magnitude of the charge.
placed2 x 10"' mapartfrom each other. Calculate (i) electric [CBSE D 06]
field and electric potential at a distance of 0.6 m from the (Ans. 0-5 m, 0.55 x 10"^C)
ELECTROSTATIC POTENTIAL AND CAPACITANCE 2.9

3. Two points A and B are located in diametrically + 2pC are placed at the vertices B and D respec
opposite directions of a point charge of + 2pC at tively. Calculate electric potentials at the vertices A
distances 2.0 m and 1.0 m respectively from it. and C. Also calculate the work done in carrying a
Determine the potential difference - V^. charge of 3 pC from A to C. (Ans. 2.52 J)
(Ans. - 9 X 10^ V) 11. Charges of 2.0 x 10"^ Cand 1.0 x 10“^C are placed at
4. A hollow metal sphere is charged with 0.4 pC of the corners A and B of a square of side 5.0 cm as
charge and has a radius of 0.1 m. Find the potential shown in Fig. 2.17. How much work will be done in
(0 at the surface (//) inside the sphere (Hi) at a moving a charge of 1.0 x 10“^C from C to D against
distance of 0.6 m from the centre. The sphere is the electric field ? (Ans. 0.053 J)
placed in air. (Ans. 36 kV, 36 kV, 6 kV) D 5 cm C

5. Two point charges of + 10 pC and + 20 pC are


placed in free space 2 cm apart. Find the electric
potential at the middle point of the line joining the un

w
two charges. (Ans. 27 MV)
6. Two point charges q and -2q are kept ‘d’ distance <J}
A B
‘?2

apart. Find the location of the point relative to

Flo
charge Y at which potential due to this system of Fig. 2.17
charges is zero. [CBSE OD 14C]
of + 1.0xl0"”c, -2.0x10 -11
12. Charges C,

ee
(Ans. At distance d / 3 from charge q) + 1.0x10
-11
C are placed respectively at the corners
7. Two point charges, one of + 100 pC and another of

Fr
B, C and D of a rectangle ABCD. Determine the
- 400 pC, are kept 30 cm apart. Find the points of potential at the corner A. Given AB = A cm and
zero potential on the line joining the two charges 6C = 3 cm. (Ans. 1.65 V)
(assume the potential at infinity to be zero).
for
ur
13. Positive charges of 6,12 and 24 nC are placed at the
(Ans. 6 cm from + 100 pC charge) three vertices of a square. What charge must be
8. A charge ly = + 1 pC is held at O between the points placed at the fourth vertex so that total potential at
A and B such that AO = 2 m and BO = 1 m, as shown
ks
the centre of the square is zero ? (Ans. - 42 nC)
in Fig. 2.15(a). Calculate the potential difference
Yo

Two equal charges, 2.0 x 10"^ C each are held fixed


oo

{Vyu - Vj^). What will be the value of the potential at a separation of 20 cm. A third charge of equal
difference {V^ - 1/^) if position of B is changed as magnitude is placed midway between the two
eB

shown in Fig. 2.15(&) ? (Ans. - 4500 V, - 4500 V)


charges. It is now moved to a point 20 cm from both
the charges. How much work is done by the electric
B 1 m O 2 m A
field during the process ? (Ans. - 3.6 X 10"^ J)
r

(fl)
ou

15. ABC is a right-angled triangle, where AB and BC


ad

Bf
are 25 cm and 60 cm respectively; a metal sphere of
Y

1 m 2 cm radius charged to a potential of 9 x 10^ V is


placed at B. Find the amount of work done in
carrying a positive charge of 1 C from C to A.
nd
Re

O 2 m A (Ans. 0.042 J)
Fi

(b)

Fig. 2.15

9. Two small spheres of radius 'a' each carrying


charges + q and - q are placed at points A and B,
distance 'd' apart. Calculate the potential difference
between points A and B. (Ans. Iql^KCgd) Fig. 2.18
10. The sides of rectangle ABCD are 15 cm and 5 cm, as HINTS
shown in Fig. 2.16. Point charges of - 5 pC and 1
A D 1. As V = ^ .-. 50=9x10^ x-^
+ 2pC 47ce0 r 0.9

50 X 0.9
5 cm or
q = = 5xlO"^C
9 X 10^
-5^C
B 15 cm As the potential is positive, the charge q must be
Fig. 2.16 positive.
'>v 2.10 PHYSICS-XII

2. Electric field of a point charge, 1.0x10"^ 1.0 X 10"'


1 8. 1^,4 - V'a = 9 X lO’ 2.0 1.0
. 4 = 20 NC
-1
£ =
4n s 0 r
= - 4500 V
Electric potential of a point charge,
As potential is a scalar quantity, so change in position
V = ^ .^ = 10 JC -1
of the charge does not affect the value of potential.
ATZ Eg T 1 1
V 10 9-
Clearly, distance r = — — = 0.5 m 47T£ 0 d 4ji Eg d AnCnd
0
E 20
2x10"^ 5x10"^
Magnitude of charge, 10. y^ = 9xl0^ = - 7.8 X 10^ V
0.15 0.05
10x0.5
q = AuSq .V.r - 9xl0’ = 0.55 xl0"‘^ C. 2 X lO'*^ 5 X10"^
= 9 X lO’ = 0.6 X 10^ V
0.05 0.15
3. Here q = 2[iC = 2.x lO'^ C, = 2 m, =1m
_ <1 1 1 W = q{V^-V^) = 3.0 X 10“‘’{ 0.6 x 10^ + 7.8 x 10^)
= 2.52 J.
1 'h
= 2x10"^ x9xl0^ --- V 11. ^C = +

2 1 4n E n AC BCJ
= -9x10^ V. "2.0x10*^ 1.0x10“^
= 9 X lO'^
-1. (/) Potential at the surface, V2 X 0.05 0.05
4 X 10'^ X 9 X 10^
2+42
0.1
47i£g r = 9000 V
42 X 0.05
= 36000 V = 36 kV.
1 ‘Jz
(ii) Potential inside a hollow conductor is the same D “
4Tre
0 I- AD BDJ
as on its surface.

(in) When r = 0.6 m.


'2.0x10'^ 1.0x10-6
= 9 X10^ +

9 X 10^ X 4 X10'^ 0.05 42 X 0.05


= 6000 V = 6 kV.
0.6 2V2+ 1
= 9000 V
1 42 X 0.05
"I ^ ^ Hz
47lEg _ r, T2
W = ‘j(Vo-^c)
' 10 X 10’^ 20 X10“^ '2V2+ l-2-^2
= 9 X10’ + = 1.0 X 10"^ X 9000 = 0.053 J.
0.01 0.01 42 X 0.05
= 27x10^ V= 27 MV. 12. AC = ^4^ + 3^ = 5 cm = 0.05 m, AD - BC= 0.03 m
6. Let the point P of zero potential lie at distance x 1 1.0x10
-11
2.0x10
-11
1.0x10
-n

from the charge q. v =


0.05 0.03
4jiEo _ 0.04

X d-x -2q
o- + .0 = 1.65 V.
A P B
13, Suppose a charge of q nC be placed at the fourth
vertex. Let length of half diagonal be x metre.
1/7,1 4~2q) = 0
4n£g .r 47rsg (d-x) 6x10"^ 12x10"^ 24x10"^
v,=9xl0^
0
+ +
d
or 1~ or .V = —
X X X

X d-x 3
+
i/ X 10 ^ = 0
7. Suppose the point of zero potential is located at Y

distance-Y metre from the charge of + 100 pC.Then 6 12 24


- + — + — + -4 = 0
1 100 X 10
-6
400 X10"^ or
Y Y Y Y
V = = 0.
47IE0 Y 0.30 - Y 42
or
X X
This gives y = 0.06 m = 6cm i.e., the point of zero
potential lies at 6 cm from the charge of + 100 )iC.
or q=-A2 nC
ELECTROSTATIC POTENTIAL AND CAPACITANCE 2.11

14. The situation is shown in Fig. 2.19. charge + q located at the origin O. Let A and Bbe two ●

1 r 2 X 10"^ 2 X 10"^ adjacent points separated by distance dr. The two


^C-^D =
47ceo 0.20 0.20 points are so close that electric field £ between them
remains almost constant. Let V and V + dV be the
1 2x10"^ 2x10”^
+ potentials at the two points.
4n£ 0.10 0.10
0 Vf,==V+dV
1.8 X 10"* V + 4 rfr £

O B A

Fig. 2.20 Relation between potential and field.

The external force required to move the test charge


q^ (without acceleration) against the electric field ^ i:

w
IS

given by

F ~ q^ E

Flo
The work done to move the test charge from A to B is
W = F .dr = -q^E. dr

ee
Also, the work in moving the test charge from A to

Fr
B is

= - 2 X 10"^ X 1.8 X 10“* = - 3.6 xlO"^ J. W = Charge x potential difference

for
ur
15. Potential of the charged sphere is
1
V =
*7 Equating the two works done, we get
4n s
0 dV
-q^E.dr=q^.dV
ks
or E =-
.-. 9x10^ = 9x10’ x-2 dr
Yo

0.02 dV
oo

0.02 The quantity is the rate of change of potential


or
^7 =
10"
= 2x10“^ =2pC dr
eB

with distance and is called potential gradient. Thus the


Potential at A due to charge q is electric field at any point is equal to the negative of the
g_9xl0’x2xl0“^ potential gradient at that point. The negative sign shows
r

V
that the direction of the electric field is in the direction
4n 8q r 0.25
ou
ad

of decreasing potential. Moreover, the field is in the


Potential at C due to charge q is direction where this decrease is steepest.
Y

9 X lO’ X 2 X lO"*^
V Properties relating electric field to electric potential:
0.60
(/) Electric field is in that direction in which the
nd
Re

Potential difference between A and C is


potential decrease is steepest.
1 1
-V^= 1.8x10"^ (ii) The magnitude of electric field is equal to the
Fi

V
0.25 0.60
change in the magnitude of potential per unit
1.8x7
V = 0.042 V
displacement (called potential gradient) normal
300 to the equipotential surface at the given point.
Work done in moving a charge of + 1C from C to A 11. How can we determine electric potential if electric
W = q(V^-V^)=lx 0.042 = 0.042 J. ifeld is known at any point ?
Computing electric potential from electric field.
2.7 RELATION BETWEEN ELECTRIC FIELD The relation between electric field and potential is
dV
AND POTENTIAL E =- or dV = -E.dr

10. Show that the electric field at any point is equal to d r

the negative of the potential gradient at that point.


Integrating the above equation between points ^
Computing electric field from electric potential. As
shown in Fig. 2.20, consider the electric field due to and r^, we get
PHYSiCS-XII

2 ''2 Example 15. Find the electric field between two metal
dV = - E.dr or E .d r plates 3 mm apart, connected to 12 V battery.
V'l Solution. Electric field,
'■i
12 V
£=^ = 4x 10^ Vm"^
where and are the potentials at and r^ d 3x10“^ m

respectively. If we take at infinity, then =0 and Example 16. Caladate the voltage needed to balance an oil
drop carrying 10 electrons when located between the plates
put r2=r, we get of a capacitor which are 5 mm apart (g =10 rns~^). The mass
r of oil drop is 3 x 10“^^ kg.
-19

V(r)=-J E . d r Solution. q = rie = 10x1.6x10 c

kg, d =5 mm =5x10“^
CO -16
m = 3x 10 m

Hence by knowing electric field at any point, we


can evaluate the electric potential at that point. £= ^ =- 3 Vm-'
d 5x10
12. Shoxu that the units volt/metre and newton/
For the charged oil drop to remain stationary in
coulomb are equivalent. To which physical quantity do electric field.
they refer ?
qE = mg
SI units of electric field. Electric field at any point is -19 V -16
equal to the negative of the potential gradient. It 10x1.6x10 X -:ix 10 X 10
5x10“^
suggests that the SI unit of electric field is zwlt per metre.
But electric field is also defined as the force 3x10
-16
X 10 X 5 X 10
or V = = 9.47 V.
experienced by a unit positive charge, so SI unit of 10x1.6x10
-19

electric field is newton per coulomb. Both of these units


are equivalent as shown below, Example 17. An mfinite plane sheet of charge density
volt _ joule / coulomb 10" ^ Ciif ^ is held in air. In this situation how far apart are
10
metre metre two equipotenfial surfaces, whose p.d. is 5V ?
newton - metre _ newton Solution. Electric field of an infinite plane sheet of
coulomb - metre coulomb charge.
-1 a
or 1 Vm"' =1 NC £ =
2e
0

Examples based on
If Ar is the separation between two equipotential
Relation between Electric Field
surfaces having potential difference AV^, then
and Potential
£=^
Ar
Formulae Used
a AV
1. Electric field in a region can be determined from 2e Ar
the electric potential by using relation, 0
-12
dV 2e„AV 2x8.85x10 X 5
£=- or Ar =
0
dr -8
o 10
dV dV dV
or
£r = - £,=- = 8.85 X 10 ^ m = 8.85 mm.
8x dy ' ' dz

2. Electric field between two parallel conductors,


Example 18. A spark passes in air when the potential
gradient at the surface of a charged conductor is
d 3x10'’ Vttf^. What must be the radius of an insulated
metal sphere luhich can be charged to a potential of3x 10^ V
3. Electric potential in a region can be determined before sparking into air ?
from the electric field by using the relation,
Solution. Potential gradient,
-j E.dr dV
= 3x 10^ Vm"^
t30
dr
Units Used or dV=3xlO^ dr
-I
£ is in NC or Vm \ V in volt, r in metre. or V=3xl0^r
ELECTROSTATIC POTENTIAL AND CAPACITANCE 2.13

But V^ = 3x 10^ V Solution. Points B and C lie on an equipotential


3xl0^r = 3xl0^ surface, so = Vg.
or r = 1 m. .●. P.D. between A and C = P.D. between A and B

= -EAx
Example 19. A uniform electric field £ of 300 NC~^ is AV
= -5xlO^NC“'x4xlO"^ m £ =-
directed along negative X-axis. A, B and C are three points Ax
in the field, having x and y coordinates (in metre), as shoiujj
in Fig. 2.21. Find the potential differences AV^g and
= -200 V. [Ax = AB = Vs^-3^ = 4 cm]
AV,
CA-
Example 21. If the potential in the region of space around
Y
the point
' 0 "y
(-1 m,2 m,3 m) IS given
~
by
C(-3,4) B (4, 4) V =(10a: + 5y -3z )i>olt, calculate the three components
● M- of electric field at this point.

w
Solution. Here .v = -lm, y=2m, z=3m
«♦
As V = 10x^ + 5y^-3z^

Flo
A (4,1) dV
E -♦
dx
-(10x^ + 5y^-3z^)
dx

ee
■►X
0
= -20;r=-20x(-l) = 20 Vm"^

Fr
dV
E — (10x“ + 5/-32^)=-10i/
^ 9y
= -10x2=-20 Vm“^
for
ur
Fig. 2.21
dV
Solution, (i) No work is done in moving a unit £z=- dz
= - —(10A-^
dz
+ 5y^-3z^)=6z
positive charge from A to B because the displacement
ks
= 6x3=18 Vm"''.
of the charge is perpendicular to the electric field. Thus
Yo
oo

the points A and B are at the same potential.


roblems For Practice
AV,BA = 0
eB

1. A uniform electric field of 20 NC"^ exists in the


(n) Work is done by the electric field as the positive
vertically downward direction. Determine the
charge moves from B to C {i.e., in the direction of £). increase in the electric potential as one goes up
r

Thus the point C is at a lower potential than the point B. through a height of 50 cm. (Ans. 10 V)
ou
ad

AV
As £ = - 2. A uniform electric field of 30 NC' exists along the
Ax
Y

X-axis. Calculate the potential difference Vq -


= “ E Ax = -300 NC ^ X 7 m between the points A (4 m, 2 m)and B(10 m, 5 m).
(Ans.-180 V)
nd

= - 2100 V.
Re

(Hi) Points A and 6 lie on an equipotential surface. 3. An electric field £ = 20 / -i- 30 j NC exists in free
Fi

So
KB = V^ space. If the potential at the origin is taken zero,
AV
CA determine the potential at point (2 m, 2 m).
= -2100 V. (Ans.-100 V)
—> A ''
Example 20. Three points A B and C lie in a uniform 4. The electric field in a region is given by E =
x^
i ■

electric field (E) o/5 x 10^ NC~^ as shown in the Fig. 2.22.
Find the potential difference between A and C (CBSE F 09] Write the SI unit for A. Write an expression for the
potential in the region assuming the potential at
infinity to be zero. Ans. Nm^C ^ —
A. B 2x^
S
r
5 cm’' ^
3 cm
5. Figure 2.23 shows some equipotential surfaces. What
► £
N I can you say about the magnitude and tire direction of
'●I
c the electric field ?

Fig. 2.22
(Ans. £ =
r
Vm ^ radially outward)
14 PHYSICS-XII

Equipotentia!
surface

Fig. 2.24 An equipotential surface.

Let A and Bbe two points over an equipotential surface,


as shown in Fig. 2.24. If tlie test clrarge is moved from
HINTS
A to B, the work done will be

L AV
Wab = Charge X potential difference
k' lOOL;
As the surface is equipotential, so Vg - =0
2: AVi
I
●180,
I
‘ ,:3u 'AV’=*: S3
20 k2 - 30iX 100^ Hence = 0.

4. V SI unit of electric field> NC“’ 2. Electric field is always normal to the equipo
tential surface at every point If the field were not normal
.●. SI unit of A = NC~^ xm^ = Nm^
to the equipotential surface, it would have a non-zero
component along the surface. So to move a test charge
PotCTtial, V\=
03
against this component, a work would have to be done.
But there is no potential difference between any two
5. For the equipotential surface of 60 V, points on an equipotential surface and consequently
no work is required to move a test charge on the
[■●●r ! io.lOiriL:;' surface. Hence the electric field must be normal to the
equipotential surface at every point.
or
kq= 60IV X 0.10 iri = 6 Viri L
3. Equipotential surfaces are closer together in the
regions of strong field and farther apart in the regions
of weak field. We know that electric field at any point
! ' Clearly^ ’E chorea ri j" [^! b!(|i ilectnc is equal to the negative of potential gradient at that
; field will be radially'dutwairiibecatise V decfeases point.
'Y' wilhir. ■ dV dV
i.e., E=- or dr = -
dr E
2.8 EQUIPOTENTIAL SURFACES AND THEIR For the same change in the value of dV i.e., when
PROPERTIES dV = constant, we have
ls. Wlmt is an equipotential surface ? Give an drol¬
E
example.
Equipotential surface. Any surface that has same Thus the spacing between the equipotential
electric potential at every point on it is called an equipo- surfaces will be smaller in the regions, where the
tential surface. The surface may be surface of a body or a electric field is stronger and vice versa,
surface in space. For example, as we shall see later on, 4. No two equipotential surfaces can intersect each
the surface of a charged conductor is an equipotential other. If they interesect, then there will be two values
surface. By joining points of constant potential, we can of electric potential at the point of intersection, which
draw equipotential surfaces throughout the region in jg impossible,
which an electric field exists.

14. State and prove the iniportatif properties of 2.9 EQUIPOTENTIAL SURFACES OF
equipotential surfaces. VARIOUS CHARGE SYSTEMS

Properties of equipotential surfaces : 1. No zvork is 15. Sketch and explain the equipotential surfaces for :
done in moving a test charge over an equipotential suiface. (i) a point charge, (ii) two point charges +q and -q
ELECTROSTATIC POTENTIAL AND CAPACITANCE 2J5

separated by a small distance, (Hi) tivo point charges + qand (in) Equipotential surfaces of tivo equal positive
+ q separated by a small distance and (iv) a uniform electric charges. Fig. 2.27 shows the equipotential surfaces of
field. two equal and positive charges, each equal to + q, sepa-
Equipotential surfaces of various charge systems, rated by a small distance. The equipotential surfaces
For the various charge systems, we represent equipo are far apart in the regions in between the two charges.
tential surfaces by dashed curves and lines of force by indicating a weak field in such regions,
full line curves. Between any two adjacent equipotential
surfaces, we assume a constant potential difference.
(i) Equipotential surfaces of a positive point
charge. The electric potential due to a point charge qat > f

distance r from it is given by


v =
1 '7 / S

w
e r
0

This shows that V is constant if ris constant. Thus,


the equipotential surfaces of a single point charge are

Flo
concentric spherical shells with their centres at the Fig. 2.27
point charge, as shown in Fig. 2.25. As the lines of force
(iv) Equipotential surfaces for a tinifortn electric

ee
point radially outwards, so they are perpendicular to
field. Fig. 2.28 shows the equipotential surfaces for a
the equipotential surfaces at all points.

Fr
uniform electric field. The lines of force are parallel
straight lines and equipotential surfaces are equidis
tant parallel planes perpendicular to the lines of force.
for
ur
Equipotential
Equipotentinls surfaces
ks
>■
Line of
force
Yo
oo

*■
eB

*■
*■
■P-
r

Fig. 2.28 Equipotential surfaces for a


ou

Fig. 2.25 Equipotential surface of a +ve point charge.


ad

uniform electric field.


Y

(ii) Equipotential surfaces of two equal and opposite 16. Give the importance of equipote?itial surfaces.
point charges : Electric dipole. Fig. 2.26 shows the Importance of equipotential surfaces. Like the lines
equipotential surfaces of two equal and opposite charges, of force, the equipotential surfaces give a visual picture
nd
Re

+ q and - q, separated by a small distance. They are close


of both the direction and the magnitude of field £ in a
together in the region in between the two charges.
Fi

region of space. If we draw equipotential surfaces at


regular intervals of V, we find that equipotential
surfaces are closer together in the regions of strong
field and farther apart in the regions of weak field.
Moreover, E is normal to the equipotential surface at
/
/
I 1 every point.
I
\ /
2.10 ELECTRIC POTENTIAL ENERGY
17. 'What is meant by electric potential energy of a
charge system ?
Electric potential energy. It is the energy possessed
by a system of charges by virtue of their positions.
Fig. 2.26 Equipotential surfaces for
When two like charges lie infinite distance apart, their
two equal and opposite charges. potential energy is zero because no work has to be
2A6 PHYSICS-XII

^ done in moving one charge at infinite distance from Therefore

the other. But when they are brought closer to one Wg = Potential at point due to and P2
another, work has to be done against the force of X charge
repulsion. As electrostatic force is a conservative force, 1 1
this work gets stored as the potential energy of the tw'o or
^3 = X
% = ^1% ^ ^1%
4ne 471 £ 0
charges. 0 /l3 ^23 hs hi

The electric potential energy of a system of point charges Hence the electrostatic potential energy of the
may be defined as the amount of work done in assembling the ' + ^2
charges at their locations by bringing them in, from infinity. U = Total work done to assemble the three charges
18. Deduce expressions for the potential energy of a = Wj + IV2 + W3
system of two point charges and three point charges and hence 1
generalise the result for a system ofN point charges. ‘h
or U =
4ne
0 hi '13 ^23
Potential energy of a system of two
point charges. Suppose a point charge /
Potential energy of a system of N point charges.
is at rest at a point in space, as shown in The expression for the potential energy of N point
Fig. 2.29. It takes no work to bring the first charges can be written as
JV N
charge because there is no field yet to /
- Z ^
1 1
(j=— z z -.r.
work against. 471E
0 all pairs 'ij
2
‘?2
1^)
W,1 =0
Pi ^12 P2 As double summation counts every pair twice, to
avoid this the factor 1/2 has been introduced.
Fig. 2.29 P.E. of two point charges.
Note The potential at ;th charge due to all other
Electric potential due to charge q^ at a point P^ at charges can be written as
distance r,^ from P,1 will be
12
y= ^ i
●' k=1 r:u
K =
1 k^i
4tc8
0 ^12
The expression for P.E. of N point charges can be written as
If charge 7/2 is moved in from infinity to point P^, the 1 w 1 N (7, 1 1 N
work required is U = - Z q.
47CEn *:=] r,-t 2i' = l ' ‘
W2 = Potential x charge ^ k*j
1 ^1^2
= V^xi/2=-4718 ●V For Your Knowledge
0 '12
. , 1 , . , , . , ^ Electric potential energy is a scalar quantity. While
As the work done is stored as the potential energy c- j- 1 i ..u i «: ● u l\.
r nndmg its value, the value ot various charges must be
U of the system + ^2)/ so substituted with their proper signs.
> The potential energy of two like charges (q^ q^ >0) is positive.
u = Wj + W2 As the electrostatic force is repulsive, so a positive
amount of work has to be done against this force to
bring the charges from infinity to a finite separation.
Potential energy of
a system of three point > The potejitial energy of tivo unlike charges (q^ q^ < Oi is
negative. As the electrostatic force is attractive, so a
charges. As shown in positive amount of work has to be done against tliis
Fig. 2.30, now we bring force to take the charges from the given locations to
in the charge q^ from infinity. Conversely, a negative amount of work is
infinity To the point Py needed to bring the charges from infinity to the present
Work has to be done locations, so the potential energy is negative.
against the forces ^ As electrostatic force is a conservative force, so the

exerted by q^ and q.y. potential energy of a charge configuration is inde


pendent of the manner in which the charges are
assembled to the present locations. The potential energy
Fig. 2.30 P.E. of three is a characteristic of the present state of configuration,
point charges. not on how this state is attained.
ELECTROSTATIC POTENTIAL AND CAPACITANCE 2,i7

^ Positive potential energy implies that work can be


obtained by releasing tine charges, while negative potential
energy indicates that an external agency will have to do
work to separate the charges infinite distance apart.
^ Electric potential is a characteristic of an electric field, it
does not matter whether a charged object is placed in
that field or not. It is measured in |C~’ or volt. On the
other hand, electric potential energy is the energy of a
charged object in an external electric field. More
precisely, it is the energy of the system consisting of the
charged object and the external electric field (or charges
producing that field). It is measured in joule.
Fig. 2.31 P.E. of two charges in an external field.

2.11 POTENTIAL ENERGY IN AN

w
EXTERNAL FIELD
Work done in bringing q^ from qo to r2 against the
external field
19. Write an expression for the potential energy of a

Flo
single charge in an external field. Hence define electric
potential.
Work done on qj against the force exerted by q^

ee
Potential energy of a single charge. We wish to
1
determine the potential energy of a charge q in an

Fr
4Tte
external electric field £ at a point P where the corres 0 ^12
ponding external potential is V. By definition, V at a where r^2 is the distance between q^ and </2-
point P is the amount of work done in bringing a unit for
ur
positive charge from infinity to the point P. Thus, the Total potential energy of the system = The work
work done in bringing a charge q from infinity to the done in assembling the two charges
point P will be qV, i.e., W =qV
ks
^_M2
—>

U = q,V{r^)+q^V(r^)^ 4 Tie
Yo

This work done is stored as the potential energy of or


0 ^12
oo

the charge q. U r is the position vector of point P


21. Define electron volt. Express it in joule.
eB

relative to some origin, then


Units of electrostatic potential energy. Suppose an
U{r)^qV{r) electron {q = 1.6 x C) is moved through a potential
P.E. of a charge in an external field difference of 1 volt, then the change in its P.E. would be
r

= Charge x external electric potential


ou

AU = gA\/=1.6xlO^^^CxlV=1.6xlO“^^ J
ad

As This is a commonly used unit of energy in atomic


Y

physics and we call it electron volt (eV).


So we can define electric potential at a given point in Thus electron volt is the potential energy gained or lost by
nd
Re

an external field as the potential energy of a unit positive an electron in moving through a potential di^ence of 1 volt.
charge at that point. -19
1
Fi

leV=1.6xlO
20. Write an expression for the potential energy of
two point charges q.^ and q2> separated by distance rin an Multiples and submultiples of eV
electric field 1 meV (milli electron volt)
Potential energy of a system of two point charges in = 10"^ eV =1.6x 10"^^ J
an external field. Let V() and V{rf) be the electric 1 keV (kilo electron volt)
= 10^eV =1.6x10“^^ J
potentials of the field £ at the points having position 1 MeV (million electron volt)
vectors r^ and r2 as shown in Fig. 2.31. = 10^ eV = 1.6x10“^^ J
1 GeV (giga electron volt)
Work done in bringing q^ from x to^ against the = 10%V = 1.6xl0“^^ J
external field
1 TeV (tera electron volt)
= 10^^ eV=1.6xlO"^ J.
2,18 PHYSICS-Xll

2.12 POTENTIAL ENERGY OF A DIPOLE IN Special Cases


A UNIFORM ELECTRIC FIELD 1. Position of stable equilibrium. When 0 =0°
22. Derive an expression for the potential energy of a U = ~ p£ cos 0° = - ipE
dipole in a uniform electric field. Discuss the conditions of Thus the potential energy of a dipole is minimum
stable and unstable equilibrium. zvhen its dipole moment is parallel to the external
Potential energy of a dipole placed in a uniform field. This is the position of stable equilibrium.
electric field. As shown in Fig. 2.32, consider an electric 2. Position of zero energy. Wlien 0 =90°
dipole placed in a uniform electric field ^ with its U=-pE cos 90°=0.
Thus the potential energy of a dipole is zero
dipole moment p making an angle 0 with the field.
when it is held perpendicular to the external
Two equal and opposite forces + q^ and - qE act on field. This can be explained as follows. If we
its two ends. The two forces form a couple. The torque hold the dipble perpendicular to the electric
exerted by the couple will be field and bring it from infinity into the field,
then the work done on charge + q by the
1 = qEx 2a sin 0 = p£ sin 0
external agent is equal to the work done on
where qx2a = p, is the dipole moment. charge - q. The net work done on the dipole will
*■ be zero and hence its potential energy is zero.
3. Position of unstable equilibrium. When 0 = 180°
ti = - p£ cos 180° = + p£
E Thus the potential energy of a dipole is maximum
7" 2a sin 0
P when its dipole moment is antiparallel to the external
*■
0 field. This is the position of unstable equilibrium.
-qE -q
Examples based on
Fig. 2.32 Torque on a dipole in a uniform electric field. Electric Potential Energy
Formulae Used
If the dipole is rotated through a small angle dQ
against the torque acting on it, then the small work 1. Electric potential energy of a system of two point
done is charges,
U =
1 ?2
dW = T d0 = p£ sin 0 fr0 4nSo‘ rj2
The total work done in rotating the dipole from its 2. Electric potential energy of a system of N point
orientation making an angle 0,, with the direction of charges.
the field to 02 will be U =

02 47T £q all pairs rjj^


w = dW= pE sin QdQ 3. Potential energy of an electric dipole in a uniform
electric field.
U = - pE (cos ©2 - cos 0J)
= pE [- cos 0]^2 = pE (cos 0j - cos ©2) If initially the dipole is perpendicular to the field
This work done is stored as the potential energy U £, = 90® and ©2 = 0 (say), then
—>

of the dipole. U = pE cos 0 = - p . £


U = pE {cos 0^ - cos @2) If irritially the dipole is parallel to the field E,
6j = 0° and O2 = 0 (say), then
If initially the dipole is oriented perpendicular to
U = - pE(cos 0 - 1) = p£(l- cos 0)
the direction of the field (0j =90°) and then brought to
Units Used
some orientation making an angle 0 with the field
(©2 = 0), then potential energy of the dipole will be Charges are in coulomb, distances in metre,
U = pE (cos 90° - cos 0) = pE (0 - cos 9) energy in joule or in electron volt (eV) and dipole
moment in coulomb metre (Cm).
or
U = -pE cos 0 = — p . ^ leV =1.6x10-'^ C, lMeV*1.6xlO‘^^C.
/
/●

ELECTROSTATIC POTENTIAL AND CAPACITANCE 2.19'

Example 22 Example 24. Two positive point charges of 0.2 pC and


(a) Determine the electrostatic potential energy of a 0.01 pC are placed 10 an apart. Calculate the work done in
system consisting of two charges 1 fiC and -2\xC reducing the distance to 5 cm
(and with no external ifeld) placed at (-9 cm, 0,0) Solution. Here q^ =0.2 x 10“^C, q^ =0.01x 10^‘^C
and (9 cm, 0,0) respectively,
Initial separation (r,) =10 cm =0.10 m
(b) How much work is required to separate the two
charges infinitely away from each other ? Final separation (rf) = 5 cm =0.05 m
(c) Suppose the same system of charges is nozo placed in Work done = Change in potential energy
an external electric field E = A{l/r^) ; = Final P.E. - Initial P.E.
A =9 X 10^ Cnf^. What would the electrostatic
energy of the configuration be ? [NCERT] 1
1 ^1^2 _ ^l^2_ _L _1
Solution, (a) q^ =7 pC =7 X 10“^ Q q^=-2x 10“'^ Q 4ne
0 7 4tc£.,0 r 4ne
0 7 'I-
r = 18 cm =0.18 m

w
1 1
= 0.2 X 10~^ X 0.01 X 10"^ X 9 X 10^
Electrostatic potential energy of the two charges is .0.05 0.10
U = ~^ *?1 ?2
= 1.8 X 10"“* J.
4n

Flo
Example 25. Two electrozis, each moving with a velocity of
_9xl0^x7xl0~^x(-2)xl0^^ = -0.7 J. 10^ ms~ \ are released toruards each other. What zvill be the

ee
0.18
closest distance of approach between them ?
(&) Work required to separate two charges infinitely Solution. Let be the distance of closest approach

Fr
away from each other.
of the two electrons. At this distance, the entire K.E. of
W =U2~U^=0-U =-(-0.7) = 0.7 J. the electrons changes into their P.E. Therefore,
(c) Energy of the two charges in the external electric field for
ur
1 2 1 2 1 ee
— mv + — mv
= Energy of interaction of two charges with the 2 2 471
external electric field
9x 10^ X (1.6x10^’^)^
ks
1
+ Mutual interaction energy of the two charges 7 =
Yo

1_ ^1^2 47ie„0 mv^ 9.1 X 10"^^ X (10^)^


oo

= gi^(^i) + ^2^(7) +
4t:£„0 = 2.53 X 10
-10
m.
eB

A A
V = Er = — Example 26. Tzvo particles have equal masses of 5.0 geach
r2 47r£o r
and opposite charges of+4x 10“^C and -4.0x 10"^C. They
are released from rest with a separation of 1.0 m between
7pC ^ -2pC x9xl0^ Cm"^-0.7J
r

them. Find the speeds of the particles when the separation is


ou
ad

0.09 m0.09 m
reduced to 50 an
Y

= (70 -20) -0.7 = 50 - 0.7 = 49.3 J. Solution. Here m = 5.0 g = 5 x 10“^ kg,
Example 23. Three charges - q, + Q and - q are placed at <j = ±4xl0~^Q rj=1.0m, r2 = 50 cm =0.50 m
equal distances on a straight line. If the potential energy of
nd
Re

the systezn of three charges is zero, ivhat is the ratio Q:q ? Let V = speed of each particle at the separation of
50 cm.
[CBSE SP 23)
Fi

Solution. As shown in Fig. 2.33, suppose the three From energy conservation principle,
charges are placed at points A, Band Crespectively on K.E. of the two particles at 50 cm separation
a straight line, such that AB = BC = r. + P.E. of the two particles at 50 cm separation
-(J +Q -7
= P.E. of the two particles at 1.0 m separation
A B C
1
r r 1
— mv
2 1
+ — mv
2
+
^1 ^2 ]_ ?2
2 2 4 Tie 4ti£
0 h 0 "i
Fig. 2.33
As the total P.E. of the system is zero, so 2 _ ^l^2_
mv
1 [-‘iQ +A-c})i-q) Q(-q)^ = 0 4ne
0 /i h
4ti £q L r 2r r

Q^l- 2 _ ?2
-Q+l~Q=0 or 2Q = ^
or V
or or = 1: 4.
471E,.0 m ¥2
q 4
2.20 PHYSICS-Xll

4 X 10'^ X (- 4 X 10'^) X 9 X 10^ r0.50 -1.0 (Fig. 235). If these charges are displaced to the midpoints
5x10“^ 1.0x0.50 Aj, Bj a}id Cj respectively, ifnd the amount of the work done
in shifting the charges to the neiv locations. [CBSE OD 2015]
= 2880 or y = 53.67 ms
AD 7

Example 27. Four charges are arranged at the corners of a Solution. Bj = B, C] = q = ^ ^


square ABCD of side d as shown in Fig. 2.34. (i) Find the
work required to put together this arrangement, (ii) A charge Initial P.E. of the system is
qQ is brought to the centre E of the square, the four charges
1 rQx2Q 2Qx(-3Q) Ox(-3Q)1
being held fixed at its corners. How much extra work is U,=
I

needed to do this ? 4tce I I I


(NCERT ; CBSE F 15] OL

+ ‘7 -4
^ D 1 7Q2
4 Tie I
0

E
d A
Final P.E. of the system is
1 Qx2Q , 2Qx(-3Q) . Qx(-3Q)
U
^ 4ite OL in in in
B C
-4 + 4
1 14Q2
Fig. 2.34 4716 0 /

Solution, (f) Given AB = BC = CD - AD = d Work done = U,-U,I


/

AC= BD = ^d^ + d^ =fld 1 14Q^ 1 7Q^


4ns I 4ns I
Work required to put the four charges together 0 0

= Total electrostatic P.E. of the four charges 1 7Q2


1 4 ns I
^ 4^4c ^ ‘Ja‘Id 4g4c ^ 4b4d 4c4p 0

4n e.,0 ■- AB AC AD BC BD CD _
Example 29 (i) Tiuo point charges + and -Q2 are placed
1
4^ I 4^ 4^ I 4^ r distance apart. Obtain the expression for the amount of
4rt £ 0 d fid d d fid d work done to place a third charge Q3 at the midpoint of the
line joining the two charges,
= -^(4-V2).
4jre, 0
(ii) At what distance from charge + Qj on the line joining the
two charges (in terms o/Qj, Qj and r) will this work done be
{ii) Extra work needed to bring charge q^ to centre £ zero ? [CBSE D 20]
W = q^x Electrostatic potential at £ due to the Solution. (/)
four charges ■i-Qi -Q2
4 -4
= 4o + A r
P r
B
Ansgidlfl) 4nsQ{dlfl) 2 2

4 -4 Fig. 2.36
+ = 0.
4tiEq (d/f2) 4k6o (dlf2)
Electric potential at midpoint P,
Example 28. Three point charges, + Q, +2Qand -SQare
V = +
placed at the vertices of an equilateral triangle ABC of side I
r/2 rl2 r
A[+Q)
Amount of work done in placing charge at
point P,
2k
W = VxQ^=~{Q^~Q^)Q^.

B{+2Q] -3Q) {ii) Let the work done W be zero for a point at
distance x from charge + Qy Then electric potential will
be zero at this point.
ELECTROSTATIC POTENTIAL AND CAPACITANCE 2.21

kQi = 0
7>. roblems For Practice
or
X r~x
1. Two point charges + lOpC and -lOpC are
separated by a distance of 2.0 cm in air. (z) Calculate
or
r-x _ Q2 the potential energy of the system, assuming the
X
Qi 2ero of the potential energy to be at infinity.
(it) Draw an equipotential surface of the system.
r Q
^ + 1 = Q1 + Q2
[CBSE D 041
or

Qi (Ans.-45J)
2. Two point charges A and B of values + 15 pC and
rQ^ + 9pC are kept 18 cm apart in air. Calculate the
Qi +Q2 work done when charge B is moved by 3 cm
towards A. ICBSE OD 20001
Example 30. An electricdipole of length 4 cm, when placed

w
(Ans. 1.35 J)
with its axis making an angle of 60° with a uniform electric
field experiences a torque of 4^3 Nm Calculate the (i) magni 3. Two point charges 2Oxl0"^C and -4xl0"^C are
tude of the electric field, (ii) potential energy of the dipole, if separated by a distance of 50 cm in air. (z) Find the

Flo
the dipole has charges of± 8 nC. [CBSE OD 04; D 06C. 14] point on the line joining the charges, where the
electric potential is zero, (z'z) Also find the electro
Solution. Here 2fl =4 cm =0.04 m, 0=60°
static potential energy of the system.

ee
T =4 Nm, q =8 nC =8 x 10‘.^ C ICBSE OD 08]

Fr
Dipole moment, [Ans. (z) 41 cm from the charge of 20 x lO”*^ C
p=(|x2fl=8x 10“^ X 0.04 =0.32 x 10“^ Cm. 00-144 J]
4. Two charges, of magnitude 5 nC and - 2 nC, are
for
ur
(r) As X = pE sin 0 placed at points (2 cm, 0, 0) and (a: cm, 0, 0) in a
T 4>/3 region of space, where there is no other external
.●. £=
psinO 0.32 X 10'^ X sin 60° field. If the electrostatic potential energy of the
ks
4 ^x 10^x2 system is - 0.5 pj, what is the value otx?
Yo

[CBSE D 08C1
oo

0.32 X V3
(Ans. x = 4 cm)
= 2.5x10^“ NCr\
eB

5. Three point charges are arranged as shown in


(zz) U = - pE cos 0 Fig. 2.37. What is their mutual potential energy ?
Take zj = 1.0 x 10“^ C and zz = 10 cm. (^s. 0.27 J)
= - 0.32 X 10“^ X 2.5 X 10^® x cos 60° = - 4 J.
r
ou

Example 31. A molecule of a substance has permanent


ad

electric dipole moment equal to 10~^^ Cm. A mole of this


Y

substance is polarized (at low temperature) by applying a


strong electrostatic field of magnitude (10^Vm~^). The
direction of the field is suddenly changed.by an angle of 60°.
nd
Re

Estimate the heat released by the substance in aligning its


dipoles along the new direction of the field. For simplicity Fig. 2.37 Fig. 2.38
Fi

assume 100% polarization of the sample. [NCERT] 6. Determine potential energy of the charge configu
Solution. Here p = 10“^^Cm, £=10^ Vm~\
2
ration shown in Fig. 2.38.
23
0=6O°,N=6x1O

Work required to bring one dipole from position 7. Find the amount of work
0 = 0° to position 0 is done in arranging the
W = pE - pE cos 0 = p£(l - cos 0) three point charges, on
-29
the vertices of an equi
= 10 10^(1 - cos 60°) J =0.5 X 10“^^ J lateral triangle ABC, of
side 10 cm, as shown in
Work required for one mole of dipoles
Fig. 2.39.
= W X N =0.5 X 10“^ X 6 X 10^ =3.0 J [CBSE Sample Paper 2011]
Heat released= Loss in P.E.* Work done= 3.0 J. (Ans.-3.24 J)
2.22 PHYSICS-XII

8. Calculate the work done to dissociate the system of HINTS


three charges placed on the vertices -10
of a triangle as 1. U=-l
shown in Fig. 2.40. Here = 1.6 x 10 c.
471 Eg r
. ICBSED08;OD13]
(Ans. 2.304 x 10'* J) = 9 X 10^ X 10x10'^ x(- 10)xl0'*
2.0x10'^

= -45 J.
For equipotential surface, see Fig. 2.26 on page 2.15.
2. W = Final P.E. - Initial P.E.

- J: ^
4tc So [ 'i
100 100
cm
= 9xl0-‘^ xl5 X 10'^ x9xl0”^
15 18
Fig. 2.40 Fig. 2.41
= 1.35 J.

9. What is the electrostatic potential energy of the 3. (i) Suppose the point of zero potential is located at
charge configuration shown in Fig. 2.41 ? distance x metre from the charge of 20 x 10"^ C.
Take 1 r 20 X 10''^ 4 X10'^
8 Then, P = = 0
= + 1.0 X10'° c = - 2-0 X10 C 47U8.0 X 0.50-X

73 = + 3.0 X 10“* C, 74 = + 2.0 X 10'* C This gives a: = 0.41 m = 41 cm.


and fl = 1.0 metre. (Ans. - 6.36 x 10'" J)
(n)U =
10. Three point charges + 7, + 27 and Qare placed at the 47te„ r
three vertices of an equilateral triangle. Find the
value of charge Q (in terms of 7), so that electric 9x10^ x20xl0'^x(-4)xl0'^
= -1.44 J.
0.50
potential energy of the system is zero.
1
(Ans. Q=-2ql3) 4. (J =
47tSQ r
11. An electron (charge = -e) is placed at each of the
eight comers of a cube of side a and an a-particle .-. -0.5x10'* =
9x10^ x5xlO'^x(-2)xlO'^
(charge = + 2c) at the centre of the cube. Calculate (x-2)xl0'^
the potential energy of the system. On solving, a: =4 cm.
(Ans. 3.89 x 10^*^ e^la joule)
5. U =
12. Two identical particles, each having a charge of
4rc Sq a
2.0 X10"^ C and mass of 10 g, are kept at a
separation of 10 cm and then released. What would 3x9x10^ x(1.0xl0~^)^
= 0.27 J.
be the speeds of the particles when the separation 0.10
becomes large ? (Ans. 600 ms ')
1
7. W =
13. Find the amount of work done in rotating an electric
47tSoL AB AC BC
dipole, of dipole moment 3.2x10'* Cm, from its
position of stable equilibrium, to the position of .. 1 , ?(-?) . ?(-?)
unstable equilibrium, in a uniform electric field of 47tSoL r r r

intensity 10‘*N/C [CBSE Sample Paper 2011]


(Ans. 6.4xlO'^J)
14. An electric dipole consists of two opposite charges 47ieQ r
each of magnitude 1 pC separated by 2 cm. The 9xl0^x(6xl0~*)^
dipole is placed in an external electric field of 0.10
10*NC'^ Find (/) the maximum torque exerted by = - 3.24 J.
the field on the dipole (//) the work which the
external agent will have to do in turning the dipole 8. Initial P.E. of the three charges.
through 180° starting from the position 0 = 0°. 1
, ‘^2% ,
u.- = +

[Ans, (0 2 X10"* Nm (//) 4 x 10'* J] 4Tr£oL r r r


ELECTROSTATIC POTENTIAL AND CAPACITANCE

13. Here 0, =0°, 02 = 180°,


1
q(-4q) ^ i-4q)x2q ^ qx2q £=10‘^N/C
4^£o L '■ r r

W = p£(cos0j -cos @2)


1 Wq^ _ 9x10^ xl0x(1.6xl0"’‘^)^
4ji8o' r 0.10 = 3.2 X 10“® X 10^{cos0°-cos 180°)
= 3.2 X10^ x(l + 1} = 6.4 xlO'^ J.
--2.304 x l0“^}
Final P.E., L/,=0
14. p=qx2a = 10*^ x0.02 = 2xlO"®Cni
/ -8
(0 = p£sin 90° = 2 X 10 xlO’’ xl
Work required to dissociate the system of three
= 2x10"^ Nm.
charges,
W=U
f L/. =2.304x10"® J. (ii) W = pE{cOS - cos 02)
1 ^-<h = 2 X 10“® X 10^{cos 0°-cosl80°)
9. U = 4- + + + +

4rcSo ^^2a ■Jla = 2xlO"^(l+l) = 4xlO‘^J.

w
a a a n

9xl(f[ (1)(3)
(l)(-2) + + (l)(2) + (-2)(3) 2.13 CONDUCTORS AND INSULATORS
1.0 V2

Flo
(-2)x(2) 23. What are co7idi{ctor$ and insulators ? Why were
+ + (3)(2) xl0"‘®J
V2 insulators called dielectrics and cottductors non-electrics ?

ee
9xl0%10“’‘^ Conductors and insulators. On the basis of their
J =-6.36x10"^ J.
V2 behaviour in an external electric field, most of the

Fr
10. Suppose the charges + q, + 2q and Q are placed at materials can be broadly classified into two categories :

the comers A, B and Cof an equilateral AABCof side 1. Co«d«cfors. These are the substances zuhich allozv

for
ur
a. Then large scale physical movement of electric charges through
1 q x2q q xQ 2q xQ them ivhen an external electric field is applied. For example,
= 0
a
+
a
+
a
silver, copper, aluminium, graphite, human body, acids,
ks
alkalies, etc.
.
Yo

or 2q+Q+2Q^0 or Q = -2^/3. 2. Insulators. These are the substances lohich do not


oo

n{-e){-e) i-e)i-e) allozv physical movement of electric charges through them zvhoz
11. L/ = 9xl0^ + 12
an external electric field is applied. For example, diamond,
eB

a Vlr?
glass, wood, mica, wax, distilled water, ebonite, etc.
+ 4
i-e){-e) + 8
{-e){2e)
VSfl The rubbed insulators were able to retain charges
y/3a/2
r

placed on them, so they were called dielectrics. The


ou

9xl0^x4xe^r 3 1 8'
ad

rubbed conductors (metals) could not retain charges


a “ V2''V3“V3. placed on them but immediately drained away the
Y

36 X 10^ e^ 10 charges, so they were called non-electrics.


[3+ 2.12-4.041=3.89x10 joule.
a a
2.14 FREE AND BOUND CHARGES
nd
Re

12. Here q = 2.0x 10"^ C, m = lOg = 10"^ kg ,


r = 10cm = 0.10 m 24. Discuss the various free and bound charges
Fi

present in conductors and msidators.


Let V be the speed of each particle at infinite
separation. By conservation of energy. Free and bound charges. Tlie difference between
the electrical behaviour of conductors and insulators
P.E. of two particles at the separation of 10 cm can be understood on the basis of free and bound
= K.E. of the two particles at infinite separation charges.
1 ^2 I 2 I
. 2LJ4 = _ niv^ + - mv
2 In metallic conductors, the electrons of the outer
47r£ 2 2
0
r
shells of the atoms are loosely bound to the nucleus.
or V
2 1 They get detached from the atoms and move almost freely
4n e 0 rm inside the metal. In an external electric field, these free
9 X10^ X 2.0 x 10"^ X 2.0 X 10"‘‘ electrons drift in the opposite direction of the electric
= 36x10'* field. The positive ions which consist of nuclei and
0.10x10"-
electrons of inner shells remain held in their fixed posi
V = 600 ms *. tions. These immobile charges constitute tine bowzd charges.
2.24 PHYSICS-XI!

In electrolytic conductors, both positive and charges are induced on the right end of the conductor.
negative ions act as charge carriers. However, their
The process continues till the electric field E-ind set up
movements are restricted by the external electric field
and the electrostatic forces between them. by the induced charges becomes equal and opposite to
In insulators, the electrons are tightly bound to the the field The net field E (= ext ^ind) inside the
nuclei and cannot be detached from the atoms, i.e., charges conductor will be zero.
in insulators are bound charges. Due to the absence of
2. Just outside the surface of a charged conductor,
free charges, insulators are poor conductors of electricity. electric field is normal to the surface. If the electric
field is not normal to the surface, it will have a
For Your Knowledge component tangential to the surface which will
A third important category of materials is the immediately cause the flow of charges, producing
surface currents. But no such currents can exist under
semiconductors which we shall discuss in chapter 14.
static conditions. Hence electric field is normal to the
In metallic conductors, electrons of outer shells of the
atoms are the free charges while the immobile
surface of the conductor at every point.
positive ions are the bound charges. 3. The net charge in the interior of a conductor is
In electrolytic conductors, both positive and negative zero and any excess charge resides at its surface. As
ions are the free charges. shown in Fig. 2.43, consider a conductor carrying an
excess charge q with no currents flowing in it. Choose a
In insulators, both electrons and the positive ions are
Gaussian surface inside the conductor just near its
the bound charges.
There is no clear cut distinction between conductors outer boundary. As the field E =0 at all points inside
and insulators - their electrical properties vary the conductor, the flux through the Gaussian surface
continuously within a very large range. For example, must be zero. According to Gauss's theorem,
the ratio of the electrical properties between a metal
and glass may be as high as 10^^. <^=h E.dS=
%
As
2.15 BEHAVIOUR OF CONDUCTORS IN <lfe=0. so
q=0
ELECTROSTATIC FIELDS ‘<E

25. State and prove the various electrostatic properties + +

shown by conductors placed in electrostatic fields.


Electrostatic properties of a conductor. When
placed in electrostatic fields, the conductors show the
following properties :
1. Net electrostatic field is zero in the interior of a
Gaussian
conductor. As shown in Fig. 2.42, when a conductor is surface
—^

placed in an electric field E^^^, its free electrons begin to


Fig. 2.43
move in the opposite direction of E^^^. Negative
charges are induced on the left end and positive Hence there can be no charge in the interior of the
conductor because the Gaussian surface lies just near
the outer boundary. The entire excess charge q must
reside at the surface of the conductor.
>■ +
^ind 4. Potential is constant within and on the surface
+

*■ + *■
of a conductor. Electric field at any point is equal to the
+
E =0
negative of the potential gradient,
+ dV
i.e.. E = -
+ dr
-►
+■
But inside a conductor £ = 0 and moreover, £ has
no tangential component on the surface, so
Conductor dV
= 0 or V = constant
dr
Fig. 2.42 Electric field inside a conductor is zero.
ELECTROSTATIC POTENTIAL AND CAPACITANCE 2.25

Hence electric potential is constant throughout the inside the cavity. Imagine a Gaussian surface inside the
volume of a conductor and has the same value (as conductor quite close to the cavity. Everywhere inside
inside) on its surface. Thus the surface of a conductor is an the conductor, E=0. By Gauss's theorem, charge
equipotential surface. enclosed by this Gaussian surface is zero (E=0 =>
If a conductor is charged, there exists an electric </=0). Consequently, the electric field must be zero at
field normal to its surface. This indicates that the every point inside the cavity (q =0=> E =0). The entire
potential on the surface will be different from the excess charge + q lies op its surface.
potential at a point just outside the surface.
5. Electric field at the surface of a charged Gaussian
conductor is proportional to the surface charge + +
surface

density. Consider a charged conductor of irregular +


+ +
shape. Let a be the surface charge density at any point + +

w
£- fJkV= constant J>Y v=v;0 ■►E
+ +

+ +

Flo
Gaussian £ = 0
+ +
surface +
+ +
+

Surface of

ee
[ conductor ■’£

Fr
Fig. 2.45 Electric field vanishes in the cavity of a conductor.

for
ur
Fig. 2.44 A small pill box as a Gaussian surface 2.16 ELECTROSTATIC SHIELDING
of a charged conductor,
26. What is electrostatic shielding ? Mention its few
ks
of its surface. To determine E at this point, we choose a applications.
Yo

short cylinder (pill box) as the Gaussian surface about Electrostatic shielding. Consider a conductor with a
oo

this point. The pill box lies partly inside and partly cavity, with no charges placed inside the cavity.
outside the conductor (Fig. 2.44). It has a cross-
eB

Whatever be the size and shape of the cavity and


sectional area AS and negligible height. whatever be the charge on the conductor and the
Electric field is zero inside the conductor and just external fields in which it might be placed, the electric
r

outside, it is normal to the surface. The contribution to field inside the cavity is zero, i.e., the cavity inside the
ou
ad

conductor remains shielded from outside electric


the total flux through the pill box comes only from its
outer cross-section. influence. This is known as electrostatic shielding.
Y

Such a field free region is called a Faraday cage.


^ ~ E AS
Charge enclosed by pill box, q = G AS The phenomenon of making a region free from any
nd
Re

electric field is called electrostatic shielding. It is based on


By Gauss's theorem.
the fact that electric field vanishes inside the cavity of a
Fi

hollow conductor.
^0
a AS a Applications of electrostatic shielding
EAS = or £ = —
^0 ^0
1. In a thunderstorm accompanied by lightning, it
is safest to sit inside a car, rather than near a tree
As £ points normally outward, so we write or on the open ground. The metallic body of the
car becomes an electrostatic shielding from
£ = — n
^0
lightning.
2. Sensitive components of electronic devices are pro
where n is a unit vector normal to the surface in the tected or shielded from external electric distur
outer direction. bances by placing metal shields around them.
6. Electric field is zero in the cavity of a hollow 3. In a coaxial cable, the outer conductor connec
charged conductor. As shown in Fig. 2.45, consider a ted to ground provides an electrical shield to
charged conductor having a cavity, with no charges the signals carried by the central conductor.
s 2.26

For Your Knowledge


PHYSICS-XII

> In the interior of a conductor, the electric field and the


28. Define the unit of capacitance for a conductor.
Give its dimensions.

Units of capacitance. The SI unit of capacitance is


volume charge density both vanish. Therefore,
charges in a conductor can only be at the surface. farad (F), named in the honour of Michael Faraday.
^ Electric field at the surface of a charged conductor The capacitance of conductor is 1 farad if the addition
must be normal to the surface at every point. ofa charge ofl coulomb to it, increases its potential by 1 volt.
> For a conductor without any surface charge, electric 1 coulomb 1C
field is zero even at the surface.
.●. 1 farad = or 1F = = 1CV'^
1 volt IV
^ The entire body of each conductor, including its
surface, is at a constant potential. One farad is a very large unit of capacitance. For
practical purposes, we use its following submultiples:
Ss If we have conductors of arbitrary size, shape and
charge configuration, then each conductor will have a 1 millifarad = 1 mF = 10“^ F
characteristic value of constant potential which may 1 microfarad = 1 pF = 10”^ F
differ from one conductor to another.

^ A cavity inside a conductor is shielded from outside


1 picofarad = 1 pF = 10’^^ F
electrical disturbances. However, the electrostatic
Dimensions of capacitance. The unit of capacitance is
shielding does not work the other way round. That is,
if we place charges inside the cavity, the exterior of 1 F = 1C_ 1C ^lC^_l(As)^
the conductor cannot be shielded from the electric IV IJ/C IJ ” INm
fields of the inside charges. .●. Dimensions of capacitance
A^T^
2.17 ELECTRICAL CAPACITANCE OF A — =[M“^L“^T^A^]
MLT"^
CONDUCTOR

27. Define electrical capacitance of a conductor. On 2.18 CAPACITANCE OF AN ISOLATED


which factors does it depend ? SPHERICAL CAPACITOR
Electrical capacitance of a conductor. The electrical 29. Obtain an expression for the capacitance of an
capacitance of a conductor is the measure of its ability to hold isolated spherical conductor of radius R.
electric charge. When an insulated conductor is given
some charge, it acquires a certain potential. If we Capacitance of an iso¬
increase the charge on a conductor, its potential also spherical conductor,
increases. If a charge Q put on an insulated conductor Consider an isolated sphe
rical conductor of radius R.
increases its potential by V, then
QocV or Q=CV The charge Q is uniformly
distributed over its entire
The proportionality constant C is called the surface. It can be assumed
capacitance of the conductor. Thus
to be concentrated at the
Charge centre of the sphere. The
Capacitance =
Potential
potential at any point on the
Hence the capacitance of a conductor may be defined surface of the spherical Fig. 2.46 Capacitance of a
as the charge required to increase the potential of the conductor conductor will be spherical conductor.
by unit amount. y =
1 Q
The capacitance of a conductor is the measure of its 4:71 S
0
R

capacity to hold a large amount of charge without running


a high potential. It depends upon the following factors : .●. Capacitance of the spherical conductor situated
. vacuum is
in
1. Size and shape of the conductor. Q Q
c = ^
2. Nature (permittivity) of the surroimding medium. V 1 Q
3. Presence of the other conductors in its neigh 47t e
0
R
bourhood.
C = 4tc £«0 R
It is worth-noting that the capacitance of a con or

ductor does not depend on the nature of its material Clearly, the capacitance of a spherical conductor is
and the amount of charge existing on the conductor. proportional to its radius.
ELECTROSTATIC POTENTIAL AND CAPACITANCE 2.27

Let us calculate the radius of the spherical Solution. Let r and R be the radii of the small and
conductor of capacitance 1 F. bigger drops, respectively.
1
R = .C=9xl0^ mF"^lF Volume of the bigger drop
471 e
0 = 27 X Volume of a small drop
= 9xl0V=9xl0'’’km i.e., inR^=27.i K]
.3

This radius is about 1500 times the radius of the


R =3r = 3 X 3 mm = 9 x 10”^
earth (~6 x 10^ km).So we conclude : or m

1. One farad is a very large unit of capacitance. Capacitance of the bigger drop is
1
2. It is not possible to have a single isolated
conductor of very large capacitance.
C = 47te„0 R =
9x 10
g.9x 10“^ F
= 10“'^ F = 1 pF

w
For Your Knowledge
Charge on bigger drop
> Tine formula : C = 4ti R is valid for both hollow and
£/ = 27 X Charge on a small drop
solid spherical conductors. = 27x 10“^^ C

Flo
C
As
®0 = 47t R .'. Potential of bigger drop is
-12

ee
So the SI unit of can be written as farad per metre ^.,^,27x10 -12
= 27 V.
(Fm”'). From Coulomb's law, the SI unit of comes C 10

Fr
out to beC^N~‘m"^. Both of these units are equivalent. Example 34. Eight identical spherical drops, each carrying
^ The farad (IF =1 CV"') is an enormously large unit a charge 1 nC are at a potential of900 Veach. All these drops
of capacitance because the coulomb is a very big unit combine together to form a single large drop. Calculate the
for
ur
of charge while the volt is the unit of potential having potential of this large drop. (Assume no wastage of any kind
reasonable size. and take the capacitance of a sphere of radius r as
proportional to r).
ks
[CBSE Sample Paper 15]
Yo

Solution.
Examples Based on
oo

Capacitance of Spherical Conductors Capacitance of each small drop, C cc r => C = kr


eB

Formulae Used
Charge on each small drop, q = CV =(/crx900)C
Charge on large drop, if =8q = 7200krC
1. Capacitance of a spherical conductor of radius R,
Volume of a large drop =Volume of 8 small drops
r

C = 47Te„0 R
inR^=8.inP R=8^^^r=2r
ou
ad

Charge c = -i.
2. Capacitance = or
Potential V Capacitance of large drop, C'= kR =2kr
Y

Units Used Hence, the potential of the large drop is


nd

Charge is in coulomb, potential in volt and = 3600 V.


Re

C 2kr
capacitancein farad (F).
Fi

Example 35. N drof>s of mercury of equal radii and


Example 32. An isolated sphere has a capacitance 50 pF. possessing equal charges combine to form a big drop. Compare
(i) Calculate its radius, (ii) Hoio much charge should be the charge, capacitance and potential of bigger drop with the
placed on it to raise its potential to 10** V ? corresponding quantities of individual drops. [Punjab 01]
Solution. Here C = 50 pF = 50 x 10“*^ F, V = 10^ V Solution. Let q be the charge on each small drop
(0 R = —^.
47T8
C = 9 X10^ mF"* x 50 x 10“*^ F and r its radius.
0
Capacitance of each small drop, C = 47c Eq r
= 45 X 10“^m = 45 cm.
Potential of each small drop, V = —^ 9
{ii) q = CV = 50x 10"*^ x 10^ = 5 x 10“^C = 0.5 pC. 4tc Eq r
Example 33. Twenty seven spherical drops of radius 3 mm If R is the radius of the bigger drop, then
and carrying 10"*^ C of charge are combined to form a single
drop. Find the capacitance and the potential of the bigger 1 nrr^ X N = - or R = r
3 3
drop. [Haryana 01]
PHYSIC5-XII

Charge on the bigger drop, neutralised or leaks away. For air, the breakdown point
(f = Nq or = N. occurs at fields of the order of 3 x 10^ Vm“ ^ This puts
the limit on the capacitance of a conductor. Moreover,
Capacitance of the bigger drop, if we tend to have a single conductor of large capaci
tance, it will have practically inconvenient large size.
C = 47te„0 R = 4ti6„0 r=N^^^ C
31. W/iy does the capacitance of a conductor increase,
or
C when an earthed connected conductor is placed near it ?
C Briefly explain.
Potential of the bigger drop. Principle of a capacitor. Consider a positively
charged metal plate A and place an uncharged plate B
y'= 1 cf _ 1 Nq
4 71 E R
close to it, as shown in Fig. 2.47. Due to induction, the
0 47t 8g r closer face of plate B acquires negative charge and its
1
,1= V farther face acquires a positive charge. The negative
4tc Eg r charge on plate B tends to reduce the potential on plate
V A while the positive charge on plate B tends to
or — =
increase the potential on A As the negative charge of
plate Bis closer to plate A than its positive charge, so
the net effect is that the potential of A decreases by a
roblems For Practice
small amount and hence its capacitance increases by a
small amount.
1. Find the capacitance of a conducting sphere of
A B A B
radius 10 cm situated in air. How much charge is + + + + +
required to raise it to a potential of 1000 volt ? +
+
+
+ +
+
+
+
+

(Ans. llpF, 1.1x10“® C) +


+
+
+
+
+
+
+
+
+
+ + + + +
2. Assuming the earth to be a spherical conductor of + + + + +
+ + + + +
radius 6400 km, calculate its capacitance. + + + + +
+ + + + +
(Ans, 711 ^F) +
+
+
+
+
+
+
+
+
+
+ + + + +
HINTS + + + + +
+ + + + +
1 + + + + +
1. C=4jib„R
'0
= 9
xO.lO + + + + +

9x10
-12
Fig. 2.47 Principle of a capacitor.
= 11x10 F=n pF.
Now if the positive face of plate B is earthed, its
= =11x10’^2Fx1000 V positive charge gets neutralised due to the flow of
= 1.1x10“® c. electrons from the earth to the plate B. The negative
charge on B is held in position due to the positive
I
2. C=47r£QR = 9 X 6.4x10® charge on A. The negative charge on B reduces the
9x10
potential of A considerably and hence increases its
= 0.711xl0“^F = 711pF. capacitance by a large amount.
Hence we see that the capacitance of an insulated
2.19 CONCEPT OF A CAPACITOR AND conductor is considerably increased when we place an
ITS PRINCIPLE
earthed connected conductor near it. Such a system of two
conductors is called a capacitor.
30. An isolated conductor cannot have a large 32. What is a capacitor ? Define capacitance of a
capacitance, why ? capacitor. On what factors does it depend ?
The capacitance of an isolated conductor is small. Capacitor. A capacitor is an arrangement of two
When a conductor holds a large amount of charge, its conductors separated by an insulating medium that is used
potential is also high. If the associated electric field to store electric charge and electric energy.
(E = a/£g) becomes high enough, the atoms or A capacitor, in general, consists of two conductors
molecules of the surrounding air get ionised. A of any size and shape carrying different potentials and
breakdown occurs in the insulaHon of the surrounding charges, and placed closed together in some definite
medium and the charge put on the conductor gets positions relative to one anotRer.
ELECTROSTATIC POTENTIAL AND CAPACITANCE 2.29 \

Pictorial representation of a capacitor. The pictorial Parallel plate capacitor. The simplest and the most
symbol for a capacitor with fixed capacitance is as widely used capacitor is the parallel plate capacitor. If
shown in Fig. 2.48(a) and for that with a variable capa- consists of two large plane parallel conducting plates,
citance is as shown in Fig. 2.48(&). separated by a small distance.
Let/4 = area of each plate,
(a) (b) d = distance between the two plates
Fig. 2.48 Symbols for a capacitor with ± a = uniform surface charge densities on the
(a) fixed, (b) variable capacitance. two plates
Capacitance of a capacitor. As shown in Fig. 2.49, ± Q = ± a /4 = total charge on each plate.
usually a capacitor consists of two conductors having Area = A £ = 0

charges + Q and - Q. The potential difference between ]


them is y = - V_. Here Q is called the charge on the
+ + + +
+●●— Charge

w
+

a density + a
capacitor. Note that tlie charge on capacitor does not d
E=^
^0
mean the total charge given to the capacitor which is
+ Q-Q=0. — Charge

Flo
E = 0
^ density - a

ee
Fig. 2.50 Parallel plate capacitor.

Fr
In the outer regions above the upper plate and
below the lower plate, the electric fields due to the two
charged plates cancel out. The net field is zero.
for
ur
a
£ = 51=0
Fig. 2.49 Two conductors separated by an Is.0 2s 0
insulator form a capacitor.
In the inner region between the two capacitor
ks
For a given capacitor, the charge Q on the capacitor
plates, the electric fields due to the two charged plates
Yo

is proportional to the potential difference V between


oo

the two conductors. Thus, add up. The net field is


a a a
£ =
eB

+
or Q = cy 2s 2e
0 0
The proportionality constant C is called the capa
The direction of the electric field is from the
citance of the capacitor. Clearly,
r

positive to the negative plate and the field is uniform


c=5
ou

throughout. For plates with finite area, the field lines


ad

V
bend at the edges. This effect is called fringing of the
Y

Charge on either conductor


or Capacitance® field. But for large plates separated by small distance
P.D. between the two conductors
(A » d^), the field is almost uniform in the regions far
nd

The capacitance of a capacitor may be defined as the charge from the edges. For a uniform electric field,
Re

required to be supplied to either of the conductors of the P.D. between the plates
Fi

capacitor so as to increase the potential difference between


them by unit amount. = Electric field x distance between the plates
ad
The capacitance of a given capacitor is a constant or V = Ed =

and depends on the geometric factors, such as the ^0


shapes, sizes and relative positions of the two cond Capacitance of the parallel plate capacitor is
uctors, and the nature of the medium between them. e.A
Q (jA 0
C =^ or C=
SI unit of capacitance is farad (F). A capacitor has a V a d I e0 d
capacitance of 1 farad if 1 coulomb of charge is transferred
from its one conductor to another on applying a potential Factors on which the capacitance of a parallel
difference of 1 volt across the two conductors. plate capacitor depends
2.20 PARALLEL PLATE CAPACITOR 1. Area of the plates (C « A).
33. What is a parallel plate capacitor ? Drive an 2. Distance between the plates (C oc 1 / d).
expression for its capacitance. On what factors does the 3. Permittivity of the medium between the plates
capacitance of a parallel plate capacitor depend ? (C oc 8).
PHYSICS-Xll

2.21 SPHERICAL CAPACITOR* Cylindrical capacitor. A cylindrical capacitor consists


of tivo coaxial conducting cylinders of inner and outer radii a
34. What is a spherical capacitor ? Derive an and b. Let the two cylinders have uniform linear charge
expression for its capacitance. densities of ± X Cm"^. The length Lof the capacitor is
Spherical capacitor. A spherical capacitor consists of so large (L» radii a or b) that the edge effect can be
two concentric spherical shells of inner and outer radii a and neglected. The electric field in the region between the
b. The two shells carry charges - Q and + Q two cylinders comes only from the inner cylinder, the
respectively. Since the electric field inside a hollow outer cylinder does not contribute due to shielding. To
conductor is zero, so E = 0 for r < fl. Also the field is calculate the electric field £ at any point P in between
the two cylinders at a distance r from the central axis,
zero outside the outer shell, i.e., E = 0 for r>b. A radial we consider a coaxial Gaussian cylinder of radius r.
Using Gauss's theorem, the flux through Gaussian
field £ exists in the region between the two shells due surface must be
to the charge on the inner shell only. XL X
or E.27trL = — => E =
To determine the electric field at any point P at dis ^0 2ji
0
r
tance r from the centre, consider a concentric sphere of
radius r as the Gaussian surface. b a

Using Gauss's theorem.


(L = £.47ir^=-^ or E =
Q
^'7-T
I+
% 4 71
0
I— Charge
+ 1^^ +
E-0 Charge + Q density + X
+ +
F . Charge
+ + density - X
\

I ' P
r + +
/
a
Gaussian
I ‘ E
+ + cylinder
E = / + r
\ P
/
\
V
Charge - Q + +

'\bt Gaussian
/-
/
+
surface

Fig. 2.51 Spherical capacitor.


The potential difference (caused by the inner Fig. 2.52
sphere alone) between the two shells will be
b b .●. Potential difference between the two cylinders is
Q
V = -f = Edr = j a
4;r
dr b

P = -f E.dr=f Edr
b

['.' ? and d r are in


a a

Q [1 1
a a
opposite directions]
47t 8
0 a 4tcSq L ^ -la
4ti e
0 >-
a b b
X I
dr = - dr
271 e«0 r 271 8 0 r
[●.● E points radially inward and dr points a a

X
outward so E .dr = Edr 180° = - Edr] [In & - In fl]
2 71 8 0 27T 8 0
The capacitance of the spherical capacitor is X
47tSr,0 ab or V = In^
c = fi = Q or C = 27C 8
0
a
V Q 1 1 b~a
47t 8 0 >- a b Total charge on each cylinder is Q = LX
Capacitance of cylindrical capacitor is
2.22 CYLINDRICAL CAPACITOR* c=Q LX
or C =
27T 6«L
0

X , b b
35. What is a cylindrical capacitor ? Derive an — In - In-
2tc 8 0
expression for its capacitance. a a
ELECTROSTATIC POTENTIAL AND CAPACITANCE 231

Examples based on When the potential difference is reduced by 120 V,


Capacitance of Air-Filled Capacitors i/ = C(V-120) = 120MC
V
Formulae Used - = ^=3 => l^ = 180V
V-120 120
1. Capacitance, C=—
V
c = q _360)iC
= 2pF.
2. Capacitance of a parallel plate capacitor, C =
8
— V~ .180V
d
(//) When the voltage is increased by 120 V,
3. P.D. between the two plates of a capacitor having
charges cf^ and q2, ({' = C(V + 120) = 2pFx(180 + 120) = 600pC
V = ^2 Example 38. A parallel plate capacitor has plate area of
2C
ab
25.0 OT^ and a separation of 2.0 mmhetween its plates. The

w
4. Capacitance of a spherical capacitor, C = 4tt eg capacitor is connected to 12 V battery, (i) Find the charge on
b- a
the capacitor, (ii) If the plate separation is decreased by
Here a and b are the radii of inner and outer shells 1.0 mm, what extra charge is given by the battery to the
of the spherical capacitor. positive plate ?

Flo
5. Capacitance of a cylindrical capacitor, Solution. A=25.0 cm^ =25 X 10“^
L

ee
= 2n e0 d =2.0 mm =2x10 ^m, V=12V
log,- Z303 log.jg - Sn0 A 8.85 X 10
-12
X 25 X lO"'^

Fr
C = = 1.1x10 -lip
Here a and b are the radii of inner and outer coaxial
d 2 X 10~^
cylinders and L is the length of the capacitor. (0 q^CV = 1.1x10
●11
x 12 = 1.32 X 10 -10 Q
for
ur
Units Used
(ii) Here d' =2.0 -1.0 =1.0 mm =lx 10“^ m

Capacitance C is in farad, charge q in coulomb, -12 -4


_ 8.85 X 10 X 25x10
ks
potential difference V in volt, thicknesses d and t =2.2xlO"^^F
1x10"^
Yo

in metre.
oo

£^ = C'V=2.2x10 xl2 =2.64x10 -10 Q


-11
Constant Used
Extra charge given by the battery to the positive
eB

Permittivity constant. Eg = 8.85 x 10“^^ C^N"^m"^


plate is
Example 36. When 1.0x10^^ electrons are transferred q' -q = {2.6A-1.32)xlO
-10
= 1.32 X 10
●10
C.
from one conductor to another of a capacitor, a potential
r
ou

difference of 10 V develops between the two conductors. Example 39. A capacitor of unknozvn capacitance is
ad

Calculate the capacitance of the capacitor. connected across a battery ofV volt. A charge of 120 jiC is
Y

12
Solution. Here q = ne = 1.0x10 X 1.6x10
-19
stored in it. When the potential across the capacitor is
reduced by 40 V, the charge stored in the capacitor becomes
= 1.6xl0”^C
40 jiC. Calculate V and the unknown capacitance. What
nd
Re

V=10V would have been charge in the capacitor if the voltage is


increased by 40 V ? (CBSE D 17C]
Fi

.-. C = ^
q 1.6x10”^ = 1.6 X 10"® F.
V 10 Solution. C=—= —
V,1
^2
Example 37. A capacitor of unknown capacitance is
connected across a battery of V volts. The charge stored in it 120 pC _ 40 pC
is 360 pC. When potential across the capacitor is reduced by V “V-40
120 V, the charge stored in it becomes 120 pC. Calculate : or 3 y-120 = V

(/) The potezitial V and the unknown capacitance C V = 60 V


{ii) What will be the charge stored in the capacitor, if
the voltage applied had increased by 120 V ? ^^^_120 pC = 2pF
[CBSE D 13] ^1 60 V

Solution, (f) Let C be the capacitance of the capa When the voltage is increased by 40 V,
citor and V the potential drop across the plates. Then q=C(V +40)
q=CV =360 pC = 2 pFx(60-f40)V=200pC
2.32 PHYSICS-XIl

Q
Example 40. A sphere of radius^Syi mis suspended within pigid ,^ue to surface 1 = , towards left
28.0
a hollow sphere of radius 0.05 m. If the inner sphere is
charged to a potential of 1500 volt and outer sphere is earthed, Q
find the capacitance and the charge on the inner sphere. Field due to surface 2 = towards right
2e„0 a'
Solution. Here a =0.03 m, b=0.05 m, V =1500 V
Q
The capacitance of the air-filled spherical capacitor is Field due to surface 3 = , towards left
28„0 A
ah 0.03 X 0.05
0
C =
b -a 9 X 10^ X (0.05-0.03) 20xl0'®C
Field due to surface 4 = , towards left
2e.0 .4
= 8.33 X 10 -12 F
= 8.33 pF As the point P lies inside the conductor, the field
here must be zero.
Charge, q=CV= 8.33 x 10"^^ x 1500 -8

= 1.25 X 10"® C. _Q Q ^ Q ^ Q-20X10 =0


2e„0 a 2e„0 A 2e„0 A 2e^A
Example 41. The thickness of air layer between the two -8
or 2Q-20xl0 =0
coatings of a spherical capacitor is 2 cm The capacitor has
the same capacitance as the sphere of 1.2 m diameter. Find Q = + 10xl0"®C
the radii of its surfaces.
47t£n0 ab Charge on surface 1 = -10 x 10"® C
Solution. Here = 471 e«0 R
b~a Charge on surface 2 = + 10 x 10"® C
or
ab
= R
Charge on surface 3 = -10 x 10"® C
b~a Charge on surface 4 = -10 x 10"® C.
1.2
Now b~a=2cm and 1? =

ab
2
m = 60 cm
7>. roblems For Practice
— =60
2 1. A parallel plate air capacitor consists of two circular
plates of diameter 8 cm. At what distance should
or ab = 120
the plates be held so as to have the same capa
{b + a)^ =(b-af‘ + iab citance as that of a sphere of diameter 20 cm ?
= 2^4-4x 120=484 (Ans. 4 mm)
or b + a = 22 2. A parallel-plate capacitor has plates of area 200 cm^
or 2 + a + a = 22 [v b~a=2 cm] and separation between the plates 1.0 mm. (/) What
fl = 10 cm and b = 12 cm. potential difference will be developed if a charge of
1.0 nC is given to the capacitor ? (ii) If the plate
Example 42. The negative plate of a parallel plate capacitor separation is now increased to 2.0 mm, what will
is given a charge of -20xl0"®C. Find the charges be the new potential difference ?
appearing on the four surfaces of the capacitor plates. (Ans. 5.65 V, 11.3 V)
Solution. As shown in Fig. 2.53, let the charge 3. Two metallic conductors have net charges of
appearing on the inner surface of the negative plate be -I- 70 pc and - 70 pC, which result in a potential
-Q- Then the charge on its outer surface will be difference of 20 V between them. What is the
Q-20x 10"® C capacitance of the system ? (Ans. 3.5 pF)
The induced cliarge on 1 n 2 3n4 4. A spherical capacitor has an inner sphere of radius
the inner surface of the 9 cm and an outer sphere of radius 10 cm. The outer
positive plate will be -i- Q and sphere is earthed and the inner sphere is charged.
that on the outer surface will What is the capacitance of the capacitor ?
be - Q, as the positive plate (Ans. 0.1 nF)
is electrically neutral. To find o
5. A charge of -i- 2.0 x 10"® C is placed on the positive
Q, we consider the electric o plate and a charge of -1.0 x 10“® C on the negative
field at a point P inside the I
plate of a parallel plate capacitor of capacitance
-qU+q -q oi
negative plate. 1.2x10"® |iF. Calculate the potential difference
developed between the plates. (Ans. 12.5 V)
Fig. 2.53
ELECTROSTATIC POTENTIAL AND CAPACITANCE

HINTS +Q -Q +Q
+ + +
1. = 4tt e«0 R or = 4nenjR
0
d id + + +

or ^ (0.08)^ + + +

16 K 16x0.10 + + +

= 4 X 10“^ m = 4 mm. C1 C3
H- V.1 ^2 ■m ^3 -H

2. C = Eq A _ 8.85 X 10"^^- X 200 x 10"“*


d 1x10"^ ■« V o-
(+) (-)
= 0.177 X 10"® F = 0.177 nF
InC Fig. 2.54 Capacitors in series.
(i) = = 5.65 V.
C 0.177 nF

w
The potential differences across the various
(//) When the plate separation increases from capacitors are
1.0 mm to 2.0 mm, the capacitance decreases by Q Q Q
a factor of 2. For the same charge, the potential

Flo
1 ^2 C3
difference will increase by a factor of 2.
V' = 2V=2x5.65 = 11.3V. For the series circuit, the sum of these potential diffe

ee
3. Charge on the capacitor, rences must be equal to the applied potential difference.

Fr
t; = 70pC = 70xl0"*^C .-. y = y,-H
1
V,2 + V,3 =-2.
r
+^
r
+^
r
70xlO"*^C L-i c-2 c,3
= 3.5 pF.
y 20 V Z = J_ J_ for J_
ur
or ...(1)
4. Here a = 9 cm = 0.09 m, = 10 cm = 0.10 m Q Cj C2 C3
Clearly, the combination can be regarded as an
^ _ iTie^ab _ 1 0.09 x 0.10 ^
ks
b-a “ 9x10®'(0.10-0.09) effective capacitor with charge Q and potential dif
Yo

ference V. If Cj is the equivalent capacitance of the


oo

0.01x0.10
xl0"®F series combination, then
0.01
eB

= 0.1xl0"®F = 0.1 nF.


y

5. y =
-<?2 _ 20x10'^+ 1.0x10"^ = 12.5 V.
r

2C 2 X 1.2 X10"® or ...(2)


ou

C. Q
ad

From equations (1) and (2), we get


Y

2.23 COMBINATION OF CAPACITORS IN


SERIES AND IN PARALLEL — = -L — —
r " r c c
nd

s '“i ^2 ^3
Re

36. A number of capacitors are connected in series.


Derive an expression for the equivalent capacitance of the For a series combination of n capacitors, we can
Fi

series combination. write


1
Capacitors in series. When the negative plate of one
c.
capacitor is connected to the positive plate of the second, and
the negative of the second to the positive of third and so on,
For series combination of capacitors
the capacitors are said to be connected in series.
1. Tlie reciprocal of equivalent capacitance is equal
Figure 2.54 shows three capacitors of capacitances to the sum of the reciprocals of the individual
Cj, C2 and C3 connected in series. A potential capacitances.
difference V is applied across the combination. Tliis
2. The equivalent capacitance is smaller than the
sets up charges ± Q on the two plates of each capacitor.
smallest individual capacitance.
What actually happens is, a charge + Q is given to the
3. The charge on each capacitors is same.
left plate of capacitor Cj during the charging process.
The charge + Q induces a charge - Q on the irght plate 4. The potential difference across any capacitor is
inversely proportional to its capacitance.
of Cj and a charge - Q on the left plate of C2, etc.
2.34 PHYSICS-XII

37. A number of capacitors are connected in parallel. Examples based on


Derive an expression for the equivalent capacitance of the Grouping of Capacitors
parallel coiribination.
Formulae Used
Capacitors in parallel. When the positive plates of all
capacitors are connected to one common point and the 1. In series combination. J_ = J_ J_ J_
negative plates to another common point, the capacitors are
said to be connected in parallel. 2. In parallel combination, + (^ + ...
Figure 2.55 shows three capacitors of capacitances 3. In series combination, charge on each capacitor is
Cj, C2 and C3 connected in parallel. A potential same (equal to the charge supplied by battery) but
difference V is applied across the combination. All the potential differences across the capacitors may be
capacitors have a common potential difference V but different.
different charges given by
4. In parallel combination, potential difference on
Qi = qy, Q2=C2V, q^=c^v each capacitor is same but the charges on the capa
citors may be different.
+ “Qi
+
Units Used
C,
+ Capacitances are in farad, potential differences in
+ volt and charges in coulomb.
+ Q2 - Q2
+ Example 43. Tzoo capacitors of capacitance of 6 pf and
4- 12 pF are connected in series with a battery. The voltage
+ Q across the 6 pF capacitor is 2 V. Compute the total battery
+ Q3 -Q3
voltage. [CBSE OD 06]
+
Solution. As the two capacitors are connected in
+ series, the charge on each capacitor must be same.
+
.●. Charge on 6 pF - Charge on 12 pF
C3
■o P o-
capacitor capacitor
(+) (-) or 6 pF X 2 volt = 12 pF X y volt
Fig. 2.55 Capacitors in parallel. 6x2
P.D. across 12 pF capacitor = = 1 volt
12
Total charge stored in the combination is
Battery voltage = V^ +V2=^2W+ IV
Q = Qi + Q2 + Qs^c, + C2 + c^)V ...(1)
Example 44. Two capacitors of capacitances 3pF and
If Cp is the equivalent capacitance of the parallel 6 pF, are charged to potentials of 2 V and 5V respectively.
combination, then
These tzoo charged capacitors are connected in scries. Find
Q = C„V ...(2)
the potential across each of the two capacitors nozo.
From equations (1) and (2), we get (CBSE SP 04)

Solution. Total charge on the two capacitors


or
Cp = Cl + C2 + C3 = Ci^i + C2V2 =(3 X 2 + 6 X 5) pC =36 pC
For a parallel combination of n capacitors, we can write
In series combination, charge is conserved.
c, = Ci + q + + C.
.●. Charge on either capacitor,
For parallel combination of capacitors (/ = 36 pC
1. The equivalent capacitance is equal to the sum 36 pC
of the individual capacitances. Potential on 3 pF capacitor = — = 12 V
C1 3pF
2. The equivalent capacitance is larger than the
36 pC
largest individual capacitance. Potential on 6 pF capacitor = — = = 6 V.

3. The potential difference across each capacitor is C2 6pF


same.
Example 45. Two capacitors have a capacitance of 5pF
4. The charge on each capacitor is proportional to when connected in parallel and 1.2 pF when connected in
its capacitance. series. Calculate their capacitances.
ELECTROSTATIC POTENTIAL AND CAPACITANCE 235

Solution. Let the two capacitances be Cj pF and C2 pF. Solution. The given arrangement is equivalent to
the arrangement shown in Fig. 2.58.
In parallel, + C2 = 5 pF
Ci=5nF
qc2
In series. = 1.2 pF
C,-.C2 +

or
Ci(5-C,) = 1.2
5 +

C2 = 6pF
or
Cj ^ - 5C, + 6 = 0
Hence, Cj = 2 or 3 pF
10 V
.●. The capacitances are of 2 pF and 3 pF.
Example 46. Three capacitors of equal capacitance, when Fig. 2.58

w
connected in series have net capacitance and when
connected in parallel have net capacitance €2- What is the Clearly, the two capacitors are connected in parallel.
value of C-^l Cj'.^ Their equivalent capacitance is
C = Cj + C2=5+6=llpF

Flo
Solution. Let C = capacitance of each capacitor.
For series combination, Charge supplied by the battery is

ee
J_ = l 1 1=1 or
(? = cv =11 pFx 10 V = no pC.
C~ C Example 49. Three capacitors Cy C2 and C^ are connected

Fr
For parallel combination, to a 6V battery, as shown in Fig. 2.59. Find the charges on
the three capacitors.
=1 1-=1
C2 = C + C+C=3C for
ur
C2 3’3C 9' C, =10pF C2=5pF

Example 47. In Fig. 2.56, each of the uncharged capacitors


ks
has a capacitance of 25 pF. What charge will flow through +
Yo

the meter M when the switch S is closed ? C3 = 5 pF


oo
eB

Fig. 2.59
4200 V
C c c
Solution. The given arrangement is equivalent to
r
ou

the arrangement shown in Fig. 2.60(a).


ad

Fig. 2.56
C2 = 5 pF
Y

Solution. As the tliree capacitors are connected in C, = 10 pF


parallel, their equivalent capacitance is
nd
Re

Cp = C+ C + C = 3C=3x25 pF=75pF
Fi

y = 4200 V C3 = 5pF
-6 6V
Charge, q= C^V =75x10 X 4200
hr
= 315xlO“^C = 315 mC +

(Cl)
Example 48. Calculate the charge supplied by the battery
in the arrangement shown in Fig. 2.57. C, = l0pF C = 10 pF

C,=5pF C2 = 6pF

+ +

+
\
+
\ 6 V

10 V (b)

Fig. 2.57 Fig. 2.60


;\
i':236 PHYSICS-XII

Clearly, and C3 are in parallel. Their equivalent Clearly, the three capacitors of 10 pF, 10 pF and
capacitance is 20 pF are in parallel. Their equivalent capacitance is
C = C2 + C3=5 + 5=10|iF C2 =10 + 10+20=40 pF
Now Cj and C form a series combination, as Now the circuit reduces to the equivalent circuit
shown in Fig. 2.58(fc). Their equivalent capacitance is shown in Fig. 2.62(b). We have two capacitors of 40 |iF
C1 C' 10x10 each connected in series. The equivalent capacitance
= 5pF between A and B is
C1 +C 10 +10
40x 40
Charge drawn from the battery, C = = 20 pF.
40 + 40
^=Cy = 5pFx6 V=30 pC
Given y= 100 V
Charge on the capacitor Cj = <^ = 30 pC
Charge on the parallel combination of C2 and .'. Charge, q = CV = 20 pF x 100 V
C^=q = 30 |iC = 2000 pC = 2mC
As Cj and C3 are equal, so the charge is shared Example 51. If C, =3 pF and C2 =2 pF, calculate the
equally by the two capacitors. ecjuivalent capacitance of the given network between points
30 A and B.
Charge on C2 = charge on C3 = — = 15 pC.
C, Cl
Example 50. Find the equivalent capacitance of the Ao-
2
combination of capacitors between the points A and B as 1

shown in Fig. 2.61. Also calculate the total charge that flows 5 3
c.
in the circuit when a 100 V battery is connected between the
points A and B. [CBSE D 02] 6 4
40 nF 60 nF BO-

A9- n c, C,

60 \xF
Fig. 2.63
ii-
Solution. Clearly, capacitors 2, 3 and 4 form a
series combination. Their total capacitance C' is
10
dp 10 ^F ^ 60mF given by
1 1 1 1 1 7

C'~C ^ C2
-—I _ — -f — -(— = —

Fig. 2.61
1 Cj 3 2 3 6
6 _
Solution. Here three capacitors of 60 pF each are C' = ;;7 pF
connected in series. Their equivalent capacitance is
given by The capacitance C forms a parallel combination
—=— i- -L=A=_L with capacitor 5, so their equivalent capacitance is
Cj 60 60 60 ~ 60 “ 20
or
C = 20 pF
C''=C'+C2=^ + 2=y pF 7

The given arrangement now reduces to the The capacitance C" forms a series combination with
equivalent circuit shown in Fig. 2.62(f?) capacitors 1 and 6. The equivalent capacitance C of the
40 hF
entire network is given by
1 1
^0-
1 J_ ^ 7 1
+ - + - = 1 61
(a)
C C" q C1 20 3 3 60

10 mF 10 nF 20 nF 60
C = — pF.
61
●OB

40 nF 40 pF Example 52. From the network shown in Fig. 2.64, find


ib)
AO- -OB
the value of the capacitance C if the equivalent capacitance
between points A and B is to be 1 pF. All the capacitances are
Fig. 2.62 in pf.
ELECTROSTATIC POTENTIAL AND CAPACITANCE 231

C C; Again, capacitors and form a series combi- ^


Ao-
nation of capacitance C^2 given by
Cl Cs 6
C4
_ Q 4
^12 -“ ^“7 ^ ^11 ^ ^ = - liF
+ C„ 1+8 9
12
Now CjQ and C^2 ^ parallel combination of
C2 2 C3 2 capacitance as shown in Fig. 2.65(c).
8 8 32 ^
■OB C,,13 - Cjo + Cj2 - 3 9 9
Fig. 2.64
o-

Solution. Capacitors C2 and C3 form a parallel com


8
(2
*-12 —L. 8r
bination of equivalent capacitance, Qo'

w
3 9

Cg = C2 + C3=2+2=4pF
Capacitors and C5 form a series combination of
capacitance Cg given by Fig. 2.65 (c)

Flo
Finally, the capacitors C and ^13 form a series com
Cg“C4''c5"l2'"6“l2“4

ee
bination of capacitance 1 pF as shown in Fig. 2.65(d).
c Cl3

Fr
C5 = 4pF
AO- ■OB
The equivalent circuit can be shown as in Fig. 2.65(a)
32
C C7
for
ur
9
Ao-

Fig. 2.65 (d)


C, 8
C, 4Q 4
11 9 r’ 32 _
ks
Yo
1 C 32 23
oo

Cg 4 Example 53. Connect three capacitors o/3pF,3)if


and 6 pF such that their equivalent capacitance is 5 pF.
eB

OB
Solution. Capacitors connected in parallel have
Fig. 2.65 (fl) maximum equivalent capacitance.
r

=3+3 + 6=12 pF
ou

Capacitors Cj and Cg form a series combination of


ad

capacitance C^g given by Capacitors connected in series have minimum


Y

8 X 4 _ 32 _ 8 equivalent capacitance.
^10 “ C1 + C8 8+^~12~ 3^^ 1 _1 1 1_5
C,„in ~ 3 ^ 3 6 " 6
nd
Re

Capacitors Cg and Cg form a parallel combination


min

of capacitance.
Fi

or C...
min -=1.2 pF
5
Cii — Cg + C^—4 + 4—8 pF
The given network reduces to the equivalent circuit The required equivalent capacitance of 5pF lies
between C max and C_:_. So
Fig. 2.65(b). mm

C C7 3x6
Ao
5pF = 3pF + 2pF=3pF + pF
3+6

So we should connect the series combination of 3 pF


8
and 6pF capacitors in parallel with the third capacitor
Qo 3
8 of3pF.
Example 54. Seven capacitors, each of capacitance! pF are
to be connected in a configuration to obtain an effective
■OB
capacitance of 10 /11 pF. Suggest a suitable combination to
achieve the desired result. [irr 90]
Fig. 2.65 (b)
PHYSICS-XII

‘●5
● Solution. Suppose a parallel combination of n Example 56. Four capacitors are connected as shown in
capacitors is connected in series with a series the Fig. 2.67. Calculate the equivalent capacitance between
combination of (7 - n) capacitors. the points X and Y. [CBSE D 2000]

Capacitance of parallel combination, Cj = 2 « pF


Capacitance of series combination, C2 = 7-n Xp-
A II T II II f If ■o Y

2pF 3pF 5pF 10 pF


As these two combinations are in series, so
10
Fig. 2.67
11

But
1 1 1
n_j_ 7j-» Solution. Clearly, the first plate of 2 pF capacitor,
q ' c/ q 10"2« 2 the second plate of 3 pF capacitor and the first plate of
5 pF capacitor are connected to the point A On the
Multiplying both sides by 10 n, we get
other hand, the second plate of 2 pF capacitor, the first
lln = 5 + 35w-5«^
plate of 3 pF capacitor and the second plate of 5 pF
or 5n^ -24w-5 = 0 capacitor are connected to the point R Thus the capa
(«-5)(5n + l) = 0 citors of 2 pF, 3 pF and 5 pF are connected in parallel
between points A and B, as shown in the equivalent
or n=5 [Rejecting -ve value]
circuit diagram of Fig. 2.68.
Hence parallel combination of 5 capacitors must be 2pF
connected in series with the other 2 capacitors.
Example 55. Find the equivalent capacitance between the 3|iF
points P and Q as shown in Fig. 2.66. Given C = 18 pf and XO-
q=12 pf. A B

C 5pF 10 pF

Po-

Fig. 2.68

Total capacitance of the parallel combination of


Q capacitances 2 pF, 3 pF and 5 pF is
E B G
C C C=2+3 + 5=10pF
Fig. 2.66 As shown in Fig. 2.69, this parallel combination is
in series with capacitance of 10 pF.
Equivalent capacitance between points F and B is
18x18 XO- ■OY
+18 = 27 pF
18-H18 10 pF 10 pF

Equivalent capacitance between points A and B is Fig. 2.69


18x27
12 -H = 12 -Hl0.8=22B-23 pF Equivalent capacitance between X and Y
18 + 27 10x10
= 5pF.
Equivalent capacitance between points A and £ is 10 + 10
23x18
+ 18=28 pF Example 57. Five capacitors of capacitance 10 pf each are
23 + 18
connected with each other, as shown in Fig. 2.70. Calculate
Equivalent capacitance between points D and E is the total capacitance between the points A and C.
28x18
+ 12=23 pF C4
28 + 18

Equivalent capacitance between points D and Q is C, B


C3
23x 18 AO- -oc
+18 = 28 pF D
23 + 18

Equivalent capacitance between points P and Q is


28x 18 C2
= 11 pF.
28 + 18 Fig. 2.70
ELECTROSTATIC POTENTIAL AND CAPACITANCE 2.39

Solution. The given circuit can be redrawn in the Example 58. How would you connect 8, 12 and 24 jaF
form of a wheatstone bridge as shown in Fig. 2.71. capacitors to obtain (i) minimum capacitance (ii) maximum
capacitance ? If a potential difference of 100 volt is applied
across the system, what would be the charges on the
capacitors in each case ?
Solution.
AO- -oc
(/) For minimum capacitance, the three capacitors
must be connected in series. Then

1 _1 J_ or C_.
min = 4 pF.
C_:_ ~8 12 24 4
^
min

(ii) For maximum capacitance, the three capacitors


As r =r =r =r
must be connected in parallel.
'-1 '-2 '“4 ^5'

w
C
1 _ C4 = 8 + 12-1-24 = 44 pF.
Therefore,
q q (m) In series combination, charge is same on all
capacitors.

Flo
Thus the given circuit is a balanced wheatstone
^ = CV = 4pFxl00V = 4pC
bridge. So the potential difference across the ends of
capacitor is zero. Capacitance C3 is ineffective. The In parallel combination, charges on the

ee
given circuit reduces to the equivalent circuit shown in capacitors are

Fr
Fig. 2.72(a). =qV=8pFxl00 V = 800 pC
= C^V =12 pF X 100 V = 1200 pC
% = ^2^ =24 pF X 100 V = 2400 pC.
for
ur
Example 59. Find the equivalent capacitor of the ladder
AO- ■oc
(Fig. 2.74) between points A and B.
ks
2pF 2pF 2pF
Yo
oo

AO-
eB

Fig. 2.72 (a)


IpF 1 pF IpF 1 pF
Capacitors and €3 form a series combination of
equivalent capacitance Cg given by
r

Bo-
ou
ad

X _ 10 X 10
= 5pF Fig. 2.74
Cj + C2 ~ 10 + 10
Y

Solution. Let C be the equivalent capacitance of the


Similarly, and Cg form a series combination of infinite network. It consists of repeating units of two
nd

equivalent capacitance Cj given by


Re

capacitors of 1 pF and 2 pF. The addition of one such


C4 X Cg 10 X 10 more unit will not affect the equivalent capacitance.
Fi

^7 = = 5pF
C4 + Cg “1^+10 But then the network will take shape as shown in
Fig. 2.75.
As shown in Fig. 2.72(1?), Cg and Cy form a parallel
2pF
combination. Hence the equivalent capacitance of the
network is given by AO-

C = Cg + C7 = 5 + 5 = 10 pF.
C6 1 pF C

Ao- -OC Bo-

Fig. 2.75
C7
The equivalent capacitance of the new arrangement
Fig. 2.72 (b) must be equal to C.
PHYSICS-XII

2x C
C = l + Now Cj and C' form a series combination, there
2 + C fore, the equivalent capacitance of the entire network is
or C^-C-2 =0 C =
CC 3x6
=2 pF
or C = 2 pF or -1 pF C+C'~3 + 6
As the capacitance cannot be negative, so the The charge on the equivalent capacitor is
equivalent capacitance of the ladder is 2 pF. g = CV =2 X 10“ ^ X 1200 C = 2.4 X 10“ ^ C
Example 60. Given =20 pF, C2 =30 pF and C3 =15 pF jhis must be equal to the charge on and also the
and the insulated plate ofC^ be at a potential of 90 V, one sum of the charges on C2 and C3. Thus
plate of C3 being earthed. Wliaf is the potential difference 2.4 X 10'^
between the plates 0/C2, three capacitors being connected in = 800 V
series ? [CBSE OD 15] q 3 X10"^
Solution. Here q = 20pF, C2=30pF, C3=15pF, = 1200 V
V=90 V

C, Q
Vg= 1200-800 =400 V
o- Hence - Vg = 400 - 0 = 400 V
^2 = - Vg) = 4x10'^ X 400 C = 1.6 X lO'^C
*?3 = q (y^ - yg) =2 X10“^ X 400 c = o.s x io“^c
Fig. 2.76 =<7 = 2.4 X 10“^c
Example 62. A network of four 10 pF capacitors is
The equivalent capacitance C of the series com
connected to a 500 V supply as shown in Fig. 2.78. Deter
bination is given by
mine (a) the equivalent capacitance of the network, (b) the
i = _L J_ J_ = _L JL J_ = i_ charge on each capacitor. [NCERT]
C “ Cj C2 ^ C3 “ 20 30 ^ 15 " 20 + q ~q
^ 20 „ +
+
B + C
+

Total potential difference = 90 - 0 = 90 V C2

.●. Total charge, -q +++++«?


Cl C3
20 X10"^ + q ++ ++
~-q
q=CV = .90 =600 X 10'^ C
3 +q' -q'
+

P.D. between the plates of capacitor C2 is A


+
+
D
600 X 10“^ C +

y^2 =-^
/- = 20 V. C4
q 30 X10'^ F

Example 61. In the circuit shown in Fig. 2.77, ifthepoint 500 V

C is earthed and point A is given a potential of+1200 V,find


the charge on each capacitor and the potential at the point B. Fig. 2.78

C2 Solution. (<?) In the given network, q, C2 and C3 are


connected in series. Their equivalent capacitance C' is
C,
1200 V 4pF V=0 given by
-o- -o-
A
3pF
B C3 c i- = J_ J- X = J_=A
c' “ c. ^ q q " 10 10 10 " lo
1

2mF 10
or c'=
Fig. 2.77

NowC' and q form a parallel combination. There


Solution. Capacitors C2 and C3 form a parallel com fore, the equivalent capacitance of the whole network is
bination. Their equivalent capacitance is 40
C = C' + q = — +10 =
C = C2 + C3=(4+2)pF=6pF ^ 3 -^pF = 13.3 pF.
ELECTROSTATIC POTENTIAL AND CAPACITANCE

(b) It is clear from Fig. 2.78 that the charge on each ■■■ VA-VB=iV^-VB)-(VQ-V^)
of the capacitors C2 and C3 is same. Let it be q. Let
the charge on be q'.
Putting the values of q^ and q2> we get
P.D. across AB,
’ c, VC,

P.D. across BC, V2=^


=
C2C3 C^C^
(q + C2)(C3 + C4)_
P.D. across CD, ^3 = ^
C3 For
But

q*“3 qq - 0 or ^=£3
q q-

w
= 500
c1 q C3
Example 64. Five identical capacitor plates, each of area A
are arranged such that the adjacent plates are at distance d
or q —+ —+ — =500

Flo
Iq q q apart. The plates are connected to a source of emf V, as
shown in Fig. 2.80. Find the charges on the various plates.
- = 500 [IIT]

ee
or
^'C

Fr
10 5000
.-. g = 500 X C = 500 X
— pC= — xlO’^C B

3 3

= 1.7x 10“^ C
for
ur
1 2 3 4 5
Also, P.D. across AD = —=500 V +
q
ks
.-. ij'=500x C^= 500x10pC
Yo

= 5000 X 10"^ C = 5 X 10“^ C. A


oo

Example 63 Determine the potential difference -Vg


eB

between points A and B of the circuit shown in Fig. 2.79. Fig. 2.80
Under what condition is it equal to zero ? Solution. As shown in Fig. 2.81, the given network
C, C2
is equivalent to three parallel-plate capacitors con
r

A
«-
ou

nected in parallel. 3
ad

B 2
Their capacitances are
Y

Po- OQ A 3 B

C3 C4 4
8^ A 2 8(5 A and
8r.0 A
5
nd

V
~d~
Re

d
V

The p.d. across each


Fi

Fig. 2.79
capacitor is V. ip
Solution. Suppose the charge flows in the upper
As Fig. 2.81
branch and q2 in the lower branch. Then
Charge= Capacitancex p.d.
V-q,
c/q So charges on various plates are
vqq 2 8.0 AV
or
h =
Cl + C2
£/l - + 1^,
d
,2=- d

17 1 1 2 8. AV 2e„0 AV
Also,
V C,3 Cij 93= + —^' % = d

8.0 AV
or
v^qq % =+
‘?2 = d
q-^q
PHYSICS-XII

Example 65. Por the network shoivn in Fig. 2.82, find the 2. Two capacitors of equal capacitance when
potential difference between points A and B, and that bet- connected iii series have net capacitance C\, and
zueen B and C in the steady state. when connected in parallel have net capacitance C2.
3mF 1 nF What is the value of Q / ? IJEE Main Feb. 211
B
● o (Ans. / Cj = 1/ 4)
3. Three capacitors of capacity 1, 2 and 3|iF are
3pF 1 pF connected such that second and third are in series
and the first one in parallel. Calculate the resultant
u ■4 4.
capacity. (Ans. 2.2 pF)
IpF 4. The capacities of three capacitors are in the ratio
ion
1:2:3. Their equivalent capacity in parallel is
20 fi
greater than the equivalent capacity in series by
AM r|* I ic 60/11 pF. Calculate the individual capacitances.
100 V
(Ans. 1 pF, 2 pF, 3 pF)
Fig. 2.82 5. The equivalent capacitance of the combination
between A and B in Fig. 2.84 is 4 pF.
Solution. The two capacitors of 3 pF and 3 pF on the
20 pF
left side of the network are in parallel, their equivalent
capacitance = 6 pF A
0- -o
B
The two capacitors of 1 pF and 1 pF on the other side
of the network are also in parallel, their equivalent Fig. 2.84
capacitance = 2 pF. So the given network reduces to the
(1) Calculate capacitance of the capacitor C.
equivalent circuit shown in Fig. 2.83.
(ji) Calculate charge on each capacitor if a 12 V
«
B battery is connected across terminals A and B.
6 pF 2pF
(in) What will be the potential drop across each
capacitor ? [CBSE D 091
4i [Ans. (0 5 pF (ii) 48 pC (m) 2.4 V, 9.6 V]
1 pF 6. Calculate the capacitance of the capacitor in Fig. 2.85,
lOQ.
100 V
if the equivalent capacitance of the combination bet-
20 n
ween A and B is 15 pF. [CBSE D 94]
0—VvV iC
A (Ans. 60 pF)
10 pF
Fig. 2.83
In the steady state, when all the capacitors are C

charged, there is no current in the circuit. So there is no A^

potential drop across any resistance. Hence


p.d. across 1 pF capacitor 10 pF

= p.d. between points A and C = 100 V


Fig. 2.85
As 6 pF and 2 pF capacitances are in series, the p.d.
of 100 V is divided between them in the inverse ratio of 7. In the combination of four identical capacitors shown
their capacitances i.e., in the ratio 2 : 6 or 1 : 3. in Fig. 2.86, the equivalent capacitance between
points P and Q is 1 pF. Find the value of each
= p.d. across 6 pF= — x 100= 25 V
AB
^ 4 separate capacitance.
3
V,BC = p. d. across 2 pF = — x 100 = 75 V.
^ 4

'roblems For Practice

1. Two capacitors have a capacitance of 5pF when


connected in parallel and 1.2 pF when connected in
Fig. 2.86
series. Calculate their capacitances.
(Ans.2pF, 3pF) (Ans.4pF)
ELECTROSTATIC POTENTIAL AND CAPACITANCE 2A3

8. Find the equivalentcapacitanceof the combination 13. Calculate the equivalent capacitance between the
shown in Fig. 2.87 between the points A and B. points A and B of the circuit given below.
c, Q [CBSE F 95]
2qq
Ans. q + AO- 28
q + q (Ans. — mF)
4^F 4nF

Cl q C6^4hF C3 .2 |jF
C3 C4
AO- -OB

C, q q: 2mF 4^iF

-OB
Fig. 2.87
Fig. 2.92
9. For the network shown in Fig. 2.88, calculate the

w
equivalent capacitance between points A and B. 14. A network of six identical capacitors, each of value
3pF Cis made, as shown in Fig. 2.93. Find the equivalent
(Ans. 6 pF)
A B
o 0 capacitance between the points A and B.
(Ans. 4C/3)

Flo
AO-

9pF 9pF

ee
9pF

Fr
-OB

Fig. 2.88
Fig. 2.93
10. Calculate the capacitance of the capacitor C in
for
15. Find the equivalent capacitance between the points
ur
Fig. 2.89. The equivalent capacitance of the combi A and B of the network of capacitors shown in
nation between P and Q is 30 pF. [CBSE OD 951
(Ans. liiF)
Fig. 2.94.
(Ans. 60 pF)
ks
20 pF 2pF 2pF 2pF 2pF
Yo

-0 A
oo

C 20 pF
eB

Po- 4^
2pF IpF IpF 1 pF
R S

20 pF
r
ou

Fig. 2.94
ad

Fig. 2.89
16. Find the capacitance between the points A and B
Y

11. Calculate the equivalent capacitance between points of the assembly shown in Fig. 2.95.
A and B of the combination shown in Fig. 2.90. 1 pF 2pF (Ans. 2.25 pF)
IpF Iff (Ans.0.5pF)
nd

-●-
Re
Fi

A%- -eB Aa ' I 5 pF =“ I1-

1 pF

1 pF 1 pF
2pF 6 pF Fig. 2.95
Fig. 2.90
12. Find the equivalent capacitance between points A 17. Find the resultant capacitance between the points X
and B for the network shown in Fig. 2.91. and V of the combination of capacitors shown in
C, C: 8 (Ans. 2.5 pF)
Ans. -
Fig. 2.96. 5pF
-●-
'v J
IpF IpF
10 pF 5 pF
B
Ae 1, Cj^lpF 1, Xo- -«y

q ^ 5pF 5pF
-< -

2pF 5pF

Fig. 2.91 Fig. 2.96


PHYS1C5-XII

18. Find the total charge stored in the network of 22. Find the potential difference between the points A
capacitors connectedbetween A and B as shown in and B of the arrangement shown in Fig. 2.101.
Fig. 2.97. [CBSE OD 20) (Ans. - 8 V)

1
3V

Fig. 2.97 23. A variable capacitor has n plates and the distance
19. In Fig. 2.98, q = IpF, q = 2 pF and q = 3 pR Find between two successive plates is d. Determine its
the equivalent capacitance between points A and 6. capacitance. Ans. C -
(»-l)gQ A'
d
(Ans. 6 pF)
24. A network of four capacitors each of 12 pF
capacitance is connected to a 500 V supply as
shown in Fig. 2.102. Determine
AO- ■OB
(a) equivalent capacitance of the network, and
{b) charge on each capacitor. [CBSE OD 10]

[Ans. (a) 16 pF (if) =^2 =^3 =2000pC,


- 6000 pQ

20. Four capacitors of equal capacitances are con q


nected in series with a battery of 10 V, as shown in
Fig. 2.99. The middle point B is connected to the
earth. What will be the potentials of the points A q q
and C ?
(Ans. = + 5V,y(- = -5V)
lOV
-i- 500 V
I I-

Fig. 2.102

A
B 25. For the network shown in Fig. 2.103, compute
3pF 3pF 3pF
e c
ao-

Fig. 2.99
2pF 2pF 3pF
21. Determine the potential difference across the plates
of each capacitor of the network shown in bo-
Fig. 2.100. Take q > q. 3pF 3pF
d
3pF
_(q-A)5
Ans. q =
q + q q +q Fig. 2.103
C,
-q (i) the equivalent capacitance between points a
and b.

D (ii) the charge on each of the capacitors nearest to a


and b when V|,j, = 900 V.
+ (j -q
(m) , when = 900 V.
C2
[Ans. (I) 1 pF (if) 900 pC (iff) 100 V]
Fig. 2.100
ELECTROSTATIC POTENTIAL AND CAPACITANCE

HINTS
11. i I_ 1 C=0.5mF.
1. Proceed as in Example 45 on page 2.34.
C 1+ l"^ 1+ 1
2, Proceed as in Example 46 on page 2.35. 12. C\ and are in parallel between points A and D. So
2x3
3. C=l + = 2.2mF. the equivalent capacitance between A and D is
2+3
c'=q + q = i+i = 2nF
4. Let the capacitances be C, 2 C and 3 C. Then The given network now reduces to the equivalent
circuit shown in Fig. 2.104. Between points A and B,
Cp = C+2C+3C=6C C,
6C
J_-I A. J_-il
or q = D
q ” C'*' 2C’*’ 3C~6C 11
ImF

Given Cp-q = ^pF


c 2mF
C4 B

A
6C 60 -

w
or 6C- = -G PF 2^F
11 11

or
C = IpF Fig. 2.104

Flo
So the individual capacitances are 1 pF, 2 pF and now C and C, are in series and C^ in parallel. Hence
3pF. the equivalent capacitance between A and B is
5. (/’) As 20 |iF capacitor and capacitor C are in series, eg 2x1 8

ee
C = + 2 =
their equivalent capacitance is c'+ q 2+1 3FF.

Fr
Cx20
c AB ~
13. Capacitors q, q and are connected in series,
C+20
their equivalent capacitance q is given by
20 c
J_-_L 1 for 1 1-1
ur
or
4pF =
C+20 c.“q''q'"q“4'’2^4“i
4C+80 = 20C
or
q=ipF
ks
or C = 5 pF. Also, C5 and C^^ are in series, the equivalent capa
Yo

citance is
oo

{ii) Charge on each capacitor,


q = = 4 pF X12 V = 48 pC. qxq_2x4^4 pF
eB

C5 + q 2+ 4 3
q _ 48 pC = 2.4V
(m) P.D. on 20 pF capacitor = q and q form a parallel combination of capacitance,
20pF“20pF
r
ou

P.D. on capacitor C = — = ——
ad

= 9.6 V,
C 5 pF Now q and q form a series combination. The equi
Y

6. The combined capacitance of the parallel com valent capacitance C between A and B is given by
bination of two 10 pF capacitors is 20 pF. Tlris com I-J 1-1 1-11 28
or C = — pF.
nd

bination is connected in series with capacitance C. C“ q q “4 7 “ 28


Re

19

J_ 2-J_ 1
__ 1 ^4-3_ 1 14. The equivalent network is shown in Fig. 2.105.
Fi

or
20 C ~ 15 C 15 26 60 60
C C
or
C= 60 pF. AO' ■o B

7. All capacitors are in series.


C c
4 1
or C = 4 pF.
C IpF
C c

8. C= + q + + 2q_q
q + q q +q q + q
Fig. 2.105
1
= -9 + 19 + 19 = 1,
1
9.
C' 3
C' = 3pF Clearly, the equivalent capacitance
C = 3 pF + C’ = 3 pF + 3 pF = 6 pF. = [2 C and C in series] 11 (C and 2 C in series]
2CxC Cx2C 4C
10. l-C + -l
60
= Ji
30
C=60pF. 2C+ C+ 2C“"3
● 2.46 PHYSICS-XII

15. Two 2ijF capacitors at the left side of the network


are in series. Their equivalent capacitance is or
^a + ^c=2V^b=0
2x2
But v^-y^ = iov
= UiF
2+2
V^ = + 5V and V^ = -5V.
The capacitance and the next capacitor of 1 pF are 21. Let charge q flow across the capacitor plates until
in parallel. Their equivalent capacitance is the current stops. In a closed circuit,
C,,=l+l=2pF E Ay = 0

Proceeding in this way, we finally get two 2jaF or +


capacitors connected in series. q
Equivalent capacitance between A and B
or
2x2
= UiF. qq
2+2
or
i('. The given arrangement is a balanced wheatstone q + q
bridge. Proceed as in Example 57 on page 2.38.
17. The arrangement between the points A and B is a P.D. across plates of q = q q +q
^
balanced wheatstone bridge. Proceeding as in
Example 57, we find that the equivalent capacitance (E^-E^)q
between A and B is P.D. across plates of q ■= — =
q +q
C' = 5
22. The given arrangement is equivalent to the circuit
Now the capacitor C and the left out capacitor of shown in Fig. 2.107.
5|iF are in series. The equivalent capacitance C, = 2^F
between points X and Y will be
A B
C = C' X 5 _ 5x5 = 2.5pF.
C' + 5 “ 5+1 £,
J
12V 24 V
4 pF _ 6 pF
18. Clearly
Fig. 2.107 q=4pF
4 pF 2 pF
Proceeding as in the above problem 21, we get
A B (q-£i)qq
6pF 3pF ^ q+q
P.D. across the plates of q,
3V (q-q)q_(i2v-24V)4pF
1
q q-^q 2 pF + 4 pF

Fig. 2.106 23. The given arrangement is equivalent to (fi-1)


capacitors joined in parallel.
The given circuit (Fig. 2.97) is a balanced wheatstone C =
(»-1)Eo/1
bridge. The middle arm is ineffective. The d
equivalent circuit is shown in Fig. 2.106.
4x2 6x3 10
24. (fl) q 23 “ lM
3
= 4pF
C
nj
i + 2.
4+2 6+3 3
q^ = q23+Q=4+i2=i6pF.
Total charge stored in the capacitors.
10 (&) = q23 V = 4 pF X 500V = 2000 pC
Q=q/=yFFx3V=10pJ. 74 = q V = 12 pF X 500 V = 6000 pC.
14. The three capacitors are connected in parallel 25. (/) Tliree 3pF capacitors in series have equivalent
between points A and B. capacitance = 1 pF. The combination is in
c=q + q + q = i+2 + 3 = 6pR parallel with 2pF capacitor.
20. Here Vg = 0. As the capacitances are equal on the .'. Equivalent capacitance between c and d
two sides of point B, = l+2 = 3pF
H4'
p
● A

ELECTROSTATIC POTENTIAL AND CAPACITANCE ZA7 4

The situation is repeated for points e and .f Suppose now a small additional charge dQ be trans
Hence there are three 3 )aF capacitors in series ferred from plate 2 to plate 1. The work done will be
between points a and b. Equivalent capacitance
between a and b=lpF.
dW = V'.dQ=^.dQ
C
(n) Potential drop of 900 V across a and b is equally The total work done in transferring a charge Q from
shared by three 3 pF capacitors. plate 2 to plate 1 [Fig. 2.108(fc)] will be
Hence charge on each capacitor nearest to n
Q ,21Q
and b
W= =
O' Q'^ i ^
= 300x3 = 900pC 2C 2 ■ C
0 J 0
(in) Potential drop of 300 V across e and/is equally
shared by 3 pF capacitors. This work done is stored as electrical potential
Hence V,.
cd
= 100 V. energy U of the capacitor.

w
1

2.24 ENERGY STORED IN A CAPACITOR 2 C 2 .CV^-^QV [V Q = CV]


38. How does a capacitor store energy ? Derive an 39. If several capacitors are connected in series or

Flo
expression for the energy stored in a capacitor. parallel, shozv that the energy stored would be additive in
either case.
Energy stored in a capacitor. A capacitor is a device

ee
EniT ●● stored in.. ..I’.uilors.
to store energy. The process of charging up a capacitor
●● mii'i.

Fr
involves the transferring of electric charges from its For a series combination, Q = constant
one plate to another. The work done in charging the Total energy.
capacitor is stored as its electrical potential energy. This 1 1 1
energy is supplied by the battery at the expense of its u =
for
ur
— + — + — +...

stored chemical energy and can be recovered by 2 ■C 2 [C 1 ^3 J


allowing the capacitor to discharge.
+...
ks
Expression for the energy stored in a capacitor. 2q 2C2 2C3
Yo

Consider a capacitor of capacitance C. Initially, its two


oo

plates are uncharged. Suppose the positive charge is


or
U = Uj + U2 + U3 +...
transferred from plate 2 to plate 1 bit by bit. In this
eB

Encrg. ● M-hi
process, external work has to be done because at any '.ipacilor^ For a parallel combination, V = constant
stage plate 1 is at higher potential than the plate 2.
Suppose at any instant the plates 1 and 2 have charges Total energy.
r

l[c, + q + C3 + ...n/^
ou

and - Q respectively, as shown in Fig. 2.108(a). Then


ad

2
the potential difference between the two plates will be
Y

V =^ = ir v2 + ic,v2 + icv/2 + .„
c
nd

or
U = Uj + U2 + 1/3 +...
Re

Q' -Q'-rfQ' Q -Q

Hence total energy is additive both in series and parallel


+ +
Fi

+ +
+ +
combinations of capacitors.
+ +
dQ'
+ 2.25 ENERGY DENSITY OF
+ <■ TE
+
AN ELECTRIC FIELD
+ +
+

+
+
+
40. Where is the energy stored in a capacitor ? Derive
+ +
an expression for the energy density of an electric field.
Encrg\’ )f When a

1 (a) 2 1 ih) 2 capacitor is charged, an electric field is set up in the


region between its two plates. We can say that the
Fig. 2.108(a) Work done in transferring charge dQ' from work done in the charging process has been used in
plate 2 to plate 1. (i>) Total work done in charging the creating the electric field. Thus the presence of an
capacitor may be considered as the energy stored electric field implies stored energy or the energy is stored
in the electric field between the plates. in the electric field.
1^. 2.48 PHYSiCS-XII

Consider a parallel plate capacitor, having plate from the conductor at higher potential to that at lower
area A and plate separation d. Capacitance of the potential till their potentials become equal. Thus the
parallel plate capacitor is given by charges are redistributed. But the total charge still
_ e«A
0 remains Qj + Q2.
C =
d If the capacitance of the thin connecting wire is
If a is the surface charge density on the capacitor negligible and the conductors are a sufficient distance
plates, then electric field between the capacitor plates apart so that do not exert mutual electric forces, then
will be their combined capacitance will be + C2-
£ = —
a
or a = 8rt£ Total charge
0 Common potential =
Total capacitance
Charge on either plate of capacitor is Ql + 02 _ + C2V,
or

0 = a A = Eq£A q + C2~ + C2
●. Energy stored in the capacitor is
If after redistribution charges on Aand Bare Qj and
0' (gp Q2 respectively, then
U = ^ = - E. Ad
2C 8,,A 2 °
2.-0 and
d

But Ad = volume of the capacitor between its two


plates. Therefore, the energy stored per unit volume or the 02 ^2
energy density of the electric field is given by Thus, after redistribution, the charges on the two
) f
1 P2 conductors are in the ratio of their capacitances.
0
Ad
42. When tivo charged conductors having different
Although we have derived the above equation for a capacities and different potentials are joined together,
parallel plate capacitor, it is true for electric field due to show that there is always a loss of energy.
any charge configuration. In general, we can say that Loss of energy in redistribution of charges. Let
an electric field E can be regarded ns a seat of energy with and C2be the capacitances and and fhe poten
energy density equal to — Eq £". Similarly, energy is also tials of the two conductors before they are connected
together. Potential energy before cormection is
associated with a magnetic field.
IJ =— C + — c
2.26 REDISTRIBUTION OF CHARGES 2 2

41. Iftzoo charged conductors are touched mutually After connection, let V be their common potential.
Tlren
and then separated, prove that the charges on them ivill be
divided in the ratio of their capacitances. y = Total charge _ 0i + 02 _ ^1^1 ^2^2
Redistribution of charges. Consider two insulated Total capacitance + C2 q^C2
conductors A and Bof capacitances and C2, and Potential energy after connection is
carrying charges 0^ and 02 respectively. Let and
V2 be their respective potentials. Then u
2
0^=qV'i and 02 = C2V2 n2

CjVl^
|(Ci + q) Cj + C2

1 (qv, +
2 ' (q 4- C2)
Loss in energy.
u = u.-u
Fig. 2.109 Redistribution of charges. /

Now, if the two conductors are joined by a thin = iqq2,l


2 1 1 2 2 2
1
2■
+
(c, + q)
conducting wire, then the positive charge will flow
ELECTROSTATIC POTENTIAL AND CAPACITANCE

1 f

(//) Initial energy stored in the capacitor,


2(q + C2)
U=-CV-=-x3x X (200)^ =6 X 10"^° J
+ C/ - Cl' - ^2 ^2' -2 C1C2 VjVJ 2 2

1 qq Final energy stored in the capacitor


2 (q + q) U' = - C'V’^ = - X 10"^^ X (600f =18 X 10
-10
J
2 2

1 ClC2(^1 -^7/ Increase in energy = U - U = 12 x 10


-10
J-
2
q +q Example 67. Two identical capacitors of 12 pF each are
This is always positive whether q > q or q < q. connected in series across a battery of 50 V. Hozo much
So when two charged conductors are connected, electrostatic energy is stored in the combination ? If these
charges flow from higher potential side to lower were connected in parallel across the same battery, how

w
potential side till the potentials of the two conductors much energy luill he stored in the combination noiv ?
get equalised. In doing so, there is always some loss of Also ifnd the charge drawn from the battery in each case.
potential energy in the form of heat due to the flow of [CBSE D 17 ; OD 19]
charges in connecting wires.

Flo
Solution. Equivalent capacitance of series combination,
Examples based on 12x12
= 6 pF

ee
Energy Stored in Capacitors q +q 12 + 12

Fr
12 pF 12 pF
Formulae Used Energy stored in the series
combination,
1. Energy stored in a capacitor,

lcV^=-.^ = -qV for


ur
U =
2 2 C 2 ‘
2. Energy stored per unit volume or the energy
= ix6xl0“^^x(50)^J
ks
50 V
density of the electric field of a capacitor.
Yo

1
Fig. 2.110(a)
= 75x10“^*’]
oo

u =
2^0
Charge drawn from the battery by series combination,
eB

a
3. Electric field between capacitor plates, £ =
^0
cj^ = qv=6xlO“^^x50C = 3xlO“^°C
Units Used
r

Capacitance is in farad, charge in coulomb, Equivalent capacitance of the parallel combination,


ou
ad

electric field in NC*^ or Vm energy in joule and Cp = Ci + C2=12+12=24pF


energy density in Jm“^.
Y

Energy stored in the parallel 12 pF


combination,
Example 66. Two parallel plates, separated by 2 mm of air,
nd
Re

have a capacitance 0/ 3 x 10"^“^ F and are charged to a 12 pF


potential of 200 V. Then without touching the plates, they
Fi

-12
are moved apart till the separation is 6 mm. (i) What is the
= -x24xl0
2
x(50)^J
potential difference between the plates ? (ii) What is the 8
h
= 3xlO"°J
change in energy 7 50 V
Solution. Charge, (/ = Cy =3x 10“**^ x200=6 x IQ-’^ C Charge drawn from'the battery
Fig. 2.110(b)
When the separation increases from 2 mm to 6 mm, by the parallel combination,
the capacitance becomes = Cfd = 24 X 10“^^ X 50 C = 1.2 X 10“^ C.
C' = -.C=-x3xlO"^‘*=10 -14 p Example 68. A 12 pf capacitor is connected to a 50 V
d' 6
battery. Hozo much electrostatic energy is stored in the
(/) P.D. between the plates becomes capacitor ? If another capacitor of 6 pF is connected in series
-12 with it zoith the same battery connected across the
? _6xl0 = 600 V. combination, find the charge stored and potential difference
-14
C 10 across each capacitor. (CBSE D 171

■■

«x
2.30 PHYSICS-XIl

Solution. Energy stored in 12 pF capacitor, Energy stored.


Q~ _{6xlQ-^fx7 J = 21xlO'^J = 21pJ.
U=-C,V^=~xl2x
2 1 2
10”^^X(50)^J = 1.5 X10"® J U =
2C 2x6x10"^
eq

Equivalent capacitance of 12 pF and 6 pF capacitors. Example 70. Three identical capacitors C,, and C3 of
in series, will be capacitance 6 pf each are connected to a 12 V battery as
12x6 shown. Find:
C = pF =4 pF
q + Cj 12+6 (i) charge on each capacitor,
Charge stored in each capacitor of the series (ii) equivalent
combination. capacitance of the Cl
netzooj'k.
q=CV=4x 10"’^ X 50C = 2 X 10"’°C 12 V C3
(Hi) energy stored in C2
P.D. across 12 pF capacitor, the network of
-10
q _ 2x10 50 capacitors.
^i=- -12
V =
3
V
(CBSE D 09] Fig. 2.113
12x10
Solution, (i) and Cj are connected in series across
P.D. across 6 pF capacitor,
-10
100 12 battery while C3 is in parallel with this combination.
V - ^ v = V.
-12 Equivalent capacitance of Cj and C2 is
“ C2 6x10 3
C CjC2 _6x6
= 3pF
Example 69. 'O) Find equivalent capacitance between A
12
6+6
and B in the combination given below (Fig. 2.111). Each
Charge on either of the capacitors Cj and C2 is same.
capacitor is oflpF capacitance.
= C12V =3 pFx 12 V =36pC
C2 C3 C4
^ S ^
C5 Charge on C3, =6pFx 12 V = 72 pC
AO OB
R T
(ii) Equivalent capacitance of the network,
Fig. 2.111 ■ C = C^2 C^ = 3pF + 6pF = 9 pF.
(ii) Ifadc source of7V is connected across AB, how much (Hi) Energy stored in the network,
charge is drawn from the source and what is the energy
stored in the network ? [COSE D 17]
L/ = icV^=-x9xl0"^
2 2
x(12)“
^ ^
=6.48x10"^ J.^
Solution. 0) The equivalent network of the capa- Example 71. In Fig. 2.114, the energy stored in C,^is27 J.
citors is shown in Fig. 2.112. Calculate the total energy stored in the system.
C, 2nF

Cl C3 C5 1 laF C2
Ao _ -0-- -o - - -OB 3nF
P/S IR/T o- -o-

C4 p
C1
Q C3 R
Fig. 2.112 6^F
C4
Capacitors C2, C3 and C^ are in parallel.
Fig. 2.114
^234 “ ^2 ^3 Q ^ ^ ^ ^^
Solution. Energy stored in C^ is
Now, Cy C234 and C5 form a series combination.
1 1 1
+ —+ —=
1 1 7 U^=jC^V^=27]
C
cq Cl c
234 C5 2 6 2 6 or
jx6xl0~^x V^=27
27x2
c or = 9x10^
eq 6x10'^
(ii) Charge drawn from the 7 V source. Energy stored in C2,
Q=q,,V' = |^Fx7V=6nC. L/2=^x2x10“^x9x10®=9 J
ELECTROSTATIC POTENTIAL AND CAPACITANCE

Energy stored in C3 , Common potential.


●10
(Jg = -i X 3 X 10“^ X 9 X 10^ = 13.5 J v =
qVi + qVs _ 8x10'
-10
xioo+o
-10
= 50 V
q +q 8x10 + 8x10
Energy stored in C2, C3 and
U
= U2 + U3 + 1/4 =9 +13.5 +27 = 49.5 ]
Equivalent capacitance of C2, C3 and C^ connected -10
)x(50)^=2xl0"^J.
10
= -(8x10 + 8x10
in parallel 2'
= 2 +3 + 5=llpF
Example 74
= 49.5 J U = (r) A 900 pF capacitor is charged by a 100 V battery.
2x11x10"^ 2C
How much electrostatic energy is stored by the
capacitor ?

w
Energy stored in C^,
(n) The capacitor is disconnected from the battery and
_49.5x2x 11x10"^ = 544.5 J connected to another 900 pF capacitor. What is the
2C 2x1x10"^ electrostatic energy stored by the system ?

Flo
1

Total energy stored in the arrangement


{Hi) Where has the remainder of the energy gone ?
[NCERT ; CBSE OD 90)
= 544.5 + 49.5 = 594.0 J.

ee
Solution, (j) The charge on the capacitor ic
Example 72. In Fig. 2.115, find the

Fr
ij = cy =900x 10
-12
Fxl00V=9xl0‘^C
{a) equivalent capacitance of the network between points
A and B, The energy stored by the capacitor is

for iflV=ix9xl0"“Cxl00V
ur
Given Cj = Cg = 4pF, C2 = C3 = =2pF U=-CV^
2 2 ^ 2
(b) maximum charge supplied by the battery, and
(c) total energy stored in the network. [CBSE OD 2U| = 4.5 X 10'^ J.
ks
+ (j -q 4
Yo
+
Cl Cj Cg C4 C5
+
+ 2 + 2
oo

A B
+ +
+ +
eB

4
100 V 2
+ -o o—
+
h +
r

5 V
ou
ad

Fig. 2.116
Fig. 2.115
Y

(n) In the steady situation, the two capacitors have


Solution, (i) Combinations of and C2 as well as their positive plates at the same potential, and their
negative plates at the same potential. Let the common
nd

C. and C- have been shorted


Re

potential difference be V. The charge on eadi capacitor


■■■ C,^=C3=2hF. is then q' = CV. By charge conservation, (f = ql2.
Fi

("■) 9 max = C^y =2^Fx5V = 10nC Total energy of the system


eq

= 2x

(ra)l/=iq,v2=lK2>=10-^>c(5fj
1 i! 1 1/ 1 1 t/T = — and ~ = V
= 2.5x 10"^J. 4' C 4 ' 22' C

Example 73. A 800 pF capacitor is charged by a 100 V = 2i X 4.5 X 10"^ J = 2.25 x 10"® J.
battery. After some time the battery is disconnected. The
capacitor is then connected to another 800 pF capacitor. (in') There is a transient period before the system
What is the electrostatic energy stored ? [CBSE F 091 settles to the situation (ii). During this period, a
Solution. Here Cj = C2 = 800 pF =8 x 10"^“ F, transient current flows from the first capacitor to the
second. Energy is lost during this time in the form of
q = 100 V, q = 0 heat and electromagnetic radiation.
2.52 PHYSICS-Xil

Example 75. A capacitor of capacitance q is charged to a qq


From (i), 0.045 = -X
4 X (100)2
potential while another capacitor of capacitance q is 2 0.5x10"
charged to a potential difference q* T'/ie capacitors are now or
qq =0.045x10"'
disconnected from their respective charging batteries and
connected in parallel to each other. Now (q-q)2 = (q + q)2-4qq
(/) Find the total energy stored in the huo capacitors = (0.5 X 10"‘^)2 - 4 X 0.045 X10"^
before they are connected. = (0.25-0.180)x lO"** =0.07x 10"
(//) Find the total energy stored in the parallel combination q-q = Voo7x 10"^ =026x 10"^ ...{Hi)
of two capacitors.
On solving (//) and (Hi), we get
(Hi) Explain the reason for the difference of energy in
parallel combinafioji in comparison to the total q = 0.38 X 10"^ F and q = 0.12 x 10"^ F
energy before they are connected. [CBSE 18C1 Charges on capacitors and C2 in parallel combi
nation are :
Solution, (i) Total energy stored in the two
capacitors before they are connected, Qj = q = 0.38 X10"^ x 100 c = 0.38 x lO"^ C
1 9 1 7
Q2 = C2V = 0.12 X10“^ X100 C = 0.12 xl0"2 C
Example 77. A capacitor of capacitance 6 pF is charged to
(n) After the two capacitors are connected in a potential of 150 V. Its potential falls to 90 V, when another
parallel, the common potential is capacitor is connected to it. Find the capacitance of the second
V^ = Total charge capacitor and the amount of energy lost due to the connection.
Total capacitance q + q q-nq Solution. Here =6pF, q =150 V, q=o.
Total energy stored in the parallel combination. V=90V, C^ = ?
^2
Common potential.
u
/■ l(q + q)v^=l(q + q) q +q v =
6 X 10“^ X 150+0
or 90 V =
q+q 6 X 10"^ + C2
^ijqq + qq^ 6 X 10"'^ X 150
2 q+q or
C2 + 6 X 10"^ = 10x10"^
90
(Hi) Clearly, < Uy. Thus the total energy of the
parallel combination is less than the sum of the
or
C2 = 4 X 10"^ F = 4 pF.
energies stored in the two capacitors before they are Initial energy stored,
connected. During sharing of charges, some energy is 1 C V^
lost as heat due to the flow of charges in connecting
U.= U 1 “ IX 6 X 10"^ X (150)^
wires.
= 6.75 X 10"^ J
Example 76. Two capacitors of unknown capacitances Final energy stored,
and C2 are connected first in series and then in parallel
across a battery of 100 V. If the energy stored in the two
U
-^(q +C2)V^=i(6 + 4)xl0"^x(90)2
/ ~

combinations is 0.045 J and 0.25 J respectively, determine = 4.05 X 10“^ J


the values of and C2. Also calculate the charge on each Tlie loss of energy on connecting the two capacitors,
capacitor in parallel combination. (CBSE D 15]
AiJ = U- (6.75 - 4.05) x 10"^
Solution. For series combination, we have
= 2.7 X 10"^ J = 0.027 J.
1 qq
u = -
2 Cj + C2 Example 78. In the follozoing arrangement of capacitors
(Fig. 2.117), the energy stored in the6pF capacitor is E.Find
0.045 = 1-^1^ X (100)2 ...(/) the value of the following :
2 q + C2 (/) Energy stored in
I 1

For parallel combination, we have 12 pF capacitor,


6 pF
Li = i(q + q)t/2 (ii) Energy stored in
3 pF capacitor.
0.25 = i(q + C2)x (100)2 (Hi) Total energy 3pF
12 pF
-4 drawn from the
or
C^+C2 =0.5x10 ...(H)
battery. Fig. 2.117 [CBSE F 16]
ELECTROSTATIC POTENTIAL AND CAPACITANCE 2.53

Solution. (0 £ = i ^ X 6 X10'^ = 3 X10"^ 4. A battery of 10 V is connected to a capacitor of capa


city 0.1 F. The battery is now removed and this
capacitor is connected to a second uncharged
3x10"^ capacitor. If the charge distributes equally on these
Energy stored in 12 |iF capacitor two capacitors, find the total energy stored in the
two capacitors. Further, compare this energy with
= 2-xl2xl0"^x—L^=2E.
3x10"^
the initial energy stored in the first capacitor.
IRoorkee 961 (Ans. 2.5 J, 1:2)
(ii) Capacitance of the parallel combination 5. Find the total energy stored in the capacitors in the
network shown below. [CBSE D 04 ; SP 20]
= 6 + 12 =18pF
Charge on parallel combination, (Ans. 3.6x10'^ J)
Q = CV=18xlO"^y

w
Charge on 3pF capacitor = Q =3x 10
3x 10'^ y,1 =18x10“'^ F => = 6V.
6V“^ IpF

Flo
Energy stored in 3pF capacitor

= -CV'/
^ = ^x3xl0‘^x(6y)^

ee
2

Fr
36 £ Fig. 2.119
= -x3xl0"^x -6
= 18 E.
2 3x10
6. A lOfiF capacitor is charged by a 30 V d.c. supply
{in) Total energy drawn = £ + 2 £ +18 £ = 21E. and then connected across an uncharged 50 nF
for
ur
capacitor. Calculate (0 the final potential difference

7^. roblems for Practice


across the combination, and (//) the initial and final
energies. How will you account for the difference in
ks
1. For flash pictures, a photographer uses a capacitor energy ? [CBSE OD 04]
Yo

of 30 nF and a charger that supplies 3 x 10^ V. Find [Ans. (i) 5V, {ii) U. = 4.5 x 10^^ J,
oo

the charge and energy expended in joule for each U, =0.75x10"^ J]


flash. (Ans. 9 x 10"^ C, 135 J)
eB

7. Net capacitance of three identical capacitors in


2. In the circuit shown in Fig. 2.118, the charge on the series is 1 pF. What will be their net capacitance if
capacitor of 4pF is 16 pC. Calculate the energy connected in parallel ?
r

stored in the capacitor of 12 pF capacitance. Find the ratio of energy stored in the two configu
ou
ad

[CBSE 19C]
rations if they are both connected to the same source.
(Ans. 384 pj) [CBSE OD 11] (Ans. 9 pF, 1 : 9)
Y

20 pF
8. Figure 2,120 shows a network of five capacitors
connected to a 100 V supply. Calculate the total
nd

12 pF
Re

charge and energy stored in the network.


[CBSE OD 19]
Fi

(Ans. 4xl0~‘C, 0.02 J)


4 pF
3pF
I F
12 V
3pF 1 pF 2pF
Fig. 2.118

3. (rt) How many electrons must be added to one


2pF
plate and removed from the other so as to store
25.0 J of energy in a 5.0 nF parallel plate
capacitor ? I F
(6) How would you modify this capacitor so that 100 V
it can store 50.0 J of energy without changing
the charge on its plates ? [CBSE SP18]
Fig. 2.120
. '*

2.54 PHYSICS-XII
\ '

Find the ratio of the potential differences that must Finally, l^F capacitance is in parallel with the left
be applied across the parallel and the series out l)i,F capacitor. The equivalent capacitance is
combination of two capacitors and with their C=l+ l = 2pF = 2xlO“^F
capacitances in the ratio 1 : 2 so that the energy U = 6V
stored in the two cases, becomes the same.
1 1
[CBSE F 10 ; OD 161 (Ans. V2 :3) U=2 = -x2xl0‘® x(6)^ =3.6x10'^ J.
HINTS 2

1. Here C = SOpF = 3xlO‘^F, F = 3xlO^V (X Hereq =10pF = 10xl0"^F, q = 30 V,


Charge, q=CV = 3^ 10“^ x 3 x 10^ C = 9 xlO"^ C q =50pF = 50xl0“^F, q=0
Energy, = ^ ^ x3 xl0“^ x9 xlO^ =135 J. (/) Common potential.
qq + qq _ x lO'^ x 30 + o
V = = 5 V.
q _ 16pC ~6
2. P.D. across 4pF capacitor, V = =4 V q + q (10 + 50) X 10
C~ 4pF
{if) Initial electrostatic energy of 10 pF capacitor,
P.D. across 12 pF capacitor = 12 -4 = 8 V
a.=
Energy stored in 12 pF capacitor,
H = ICV^ = i X12 X10“^ X 8^ = 3.84 X 10~^ J = 384 pj. -X
2
10x10"^ X(30)^ =4.5x10"^ J
(a) U = -3~
2C
Final electrostatic energy of the combination,

.-. q~ = 2UC = 2 X 25J x5.0 x 10‘^C = 25 x 10 ^(10+50)xl0‘^x(5)^ = 0.75x10“^ J


-8
u
/ ~
or ij = 5xlO^C
If n is the number of electrons added or removed,
Loss in energy = q ~ ^ 10”^J
The difference in energ)' is lost in the form of heat
then </ = = 5 X 10~'*C
5x10
-4 and electromagnetic radiation as the charge flows
-19
= 3.125 xlO^® electrons. from first capacitor to second capacitor.
1.6x10

(b) Without changing the charge on the plates, we can ' ● Here C = — = 1 pF
3
0
make C half. As C = , we should either double the C=3pF
d
plate separation d or make the plate area A half. C^ = 3C = 9pF
4. Initial energy stored in the first capacitor is
-S-1-
LI..=
lcU^=|x0.1x(10)^=5.0J S 9
= 1:9.

When the first capacitor is connected to the second


uncharged capacitor, the charge distributes equally. The equivalent circuit diagram for the given
network is shown below :
This implies Oiat the capacitance of second capacitor is
also C. The voltage across each capacitor is now Vj2. 3pF 1 pF
The final total energy stored in the two capacitors is
\2 \2

(J.-/ =
-Cf-
2 [2J 2 Uj
= -CV^^=2.5J
4
2pF
U 3pF 2pF
1
-=1:2.
II: ~ 5.0 “ 2
100 V
V The two 2pF capacitors on the right side are in
2x2 I I-
series, their equivalent capacitance = = lpF
2+2
Fig. 2.121
This IpF capacitance is in parallel with the central
Two 3 pF capacitors in parallel. The equivalent
IpF capacitor. Their equivalent capacitance
capacitance.
= 1+ l = 2pF
q =3+3 = 6pF
This 2pF capacitance is in series with the 2pF
capacitor at the bottom. Their equivalent capacitance The IpF capacitor and a 2pF capacitor are in
2x2 parallel. Their equivalent capacitance,
= lpF
2+2 q = l+2 = 3pF
ELECTROSTATIC POTENTIAL AND CAPACITANCE 2.55

Difference in the behaviour of a conductor and a


Then C, and forni a series combination of equi
valent capacitance. dielectric in the presence of an external electric field.
6x3 Dielectrics have negligibly small number of charge
^2 ~ 6+3
= 2pF carriers as compared to conductors.
This combination is in parallel with the fifth capa In a conductor, the external field Eq moves the free
citor of 2 pF. charge carriers inducing field ^ind in the opposite
direction of Eq. The process continues until the two
Net capacitance, C = 2+2 = 4pF fields cancel each other and the net electric field in the
Total charge, q-CV-4>^lO x 100 = 4x10 C conductor becomes zero.

Total energy stored, +


^0 +

- = - X 4 X 10“^ X (100)^ = 0.02 J.


+
u = _ °ftee ^'free E.
+
2 2 +
^ind +

w
9. Given
C,
q 2 Conductor ^ind
Now

Flo
C2
or -c v^ = -cv^ a
+

2 pp 2 ® ®

ee
[A
^ind +

V-

Fr
or
p _ ^ ^ Cl c. Dielectric ^ind ^0

Fig. 2.122 Difference in the behaviour of a conductor and


_ qq for
a dielectric in an external electric field.
ur
h
In a dielectric, the external field £q induces dipole
q 1 moment by stretching or re-orienting the molecules of
ks
q 2 2 the dielectric. The induced dipole moment sets up an
Yo

\2 -
n +1 9
electric field £ind which opposes £q but does not
oo

q +1 exactly cancel this field. It only reduces it.


Polar and non-polar dielectrics. A dielectric may
eB

= ^^:3 consist of either polar or non-polar molecules. A


molecule in which the centre of mass of positive charges
(protons) lioes not coincide with the centre of mass of
r

2.27 DIELECTRICS AND THEIR POLARIZATION negative charges (electiwis) is called a polar molecule.
ou
ad

43. What are dielectrics ? Explain the difference in the The dielectrics made of polar molecules are called
Y

behaviour of a conductor and a dielectric in the presence polar dielectrics. The polar molecules have unsym-
an
external electric field. Distinguish between polar and rnetrical shapes. They h^e permanent dipole mo
nd

ments of the order of 10 ^ Cm. For example, a water


Re

non-polar dielectrics.
molecule has a bent shape with its two O - H bonds
Dielectrics. In insulators, the electrons remain
inclined at an angle of 105° as shown in Fig. 2.123. It has
Fi

attached to the individual atoms or molecules.


However, these electrons can suffer small movements
a very large dipole moment of 6.1 x 10"^^ Cm. Some
within the atoms or molecules under the influence of
other polar molecules are HCl, NH^, CO, CFl^OH etc.
an external electric field. The net effect of these micro Non-polar f H-
scopic movements gives rise to some important electric
properties to such materials. In view of these electrical
properties, insulators are called dielectrics.
A dielectric is a substance which docs not allow the flow
of charges through it but permits them to exert electrostatic
^rces on one another through it. A dielectric is essentialli/ an
insulator which can be polarised through small localised
displacements of its charges.
Examples. Glass, wax, water, air, wood, rubber,
stone, plastic, etc. Fig. 2.123 Some polar and non-polar molecules.
2.56 PHYStCS-Xll

A molecule in which the centre of mass of positive agitation in the material, as shown in Fig. 2.12^b){i). So
charges coincides loith the centre of mass of negative charges the total dipole moment is zero. When an external field
is called a non-polar molecule. The dielectrics made of is applied, the dipole moments of different molecules
non-polar molecules are called non-polar dielectrics, tend to align with the field. As a result, there is a net
Non-polar molecules have symmetrical shapes. They dipole moment in the direction of the field, as shown in
have normally zero dipole moment. Examples of Fig. 2.!24{b){ii). The extent of polarisation of a dielectric
non-polar molecules are H^, N2, O2, CO2, CH^, etc. depends on relative values of two opposing energies:
44. Hozu does a dielectric develop a net dipole moment 1. The potential energy of the dipoles in the external
in an external electric field zvhen it has (i) non-polar field which tends to align tlie dipoles with the field.
molecules and (ii) polar molecules ? 2. Thermal energy of the dipoles which tends to
Polarization of a non-polar dielectric in an external disrupt the alignment of the dipoles.
electric field. In the absence of any electric field, the Hence both polar and non-polar dielectrics develop a
centres of positive and negative charges of the net dipole moment in the presence of an external electric
molecules of a non-polar dielectric coincide, as shown field. This fact is called polarization of the dielectric.
in Fig. 2.124(fl)(/). The dipole moment of each molecule
is zero. In the presence of an external electric field £,0' The polarization P is defined as the dipole moment per
the centres of positive charges are displaced in the unit volume and its magnitude is usually referred to as
direction of external field while the centres of negative
the polarization density. The direction of P is same as
charges are displaced in the opposite direction. The dis
placement of the charges stops when the force exerted on that of the external field ^ O’
them by tlie external field is balanced by the restoring
force due to the internal fields in the molecules. This 45. Explain zohy the polarization of dielectric reduces the
induces dipole moment in each molecule i.e., each electric field inside the dielectric. Hence define dielectric
constant.
non-polar molecule becomes an induced dipole. The
induced dipole moments of different molecules add up Reduction of electric field by the polarization of a
giving a net dipole moment to the dielectric in the dielectric. Consider a rectangular dielectric slab placed
direction of the external field, as shown in Fig. 2.124(fl)(n).
in a uniform electric field Eg acting parallel to two of its
Ey =0 £q*0
faces, as shown in Fig. 2.125(a). Its molecular dipoles
© © Gi) G±) Dielectric slab 1—1

© +

(ES) O G±) GS)


© © © G+) G±) G±) +
G±) G±) GE)
©
+
Gi) («)

Gb G±), G±) GE)


+
('> {ii)
+

Fig. 2.124 (a) Polarization of a non-polai dielectric


Gi5 GE) GE) GE)
-I-

in an external electric field.

Polarization of a polar dielectric in an external ’G' Region of zero charge density


electric field. The molecules of a polar dielectric have
permanent dipole moments. In the absence of any I—I -tr, + I—

external electric field, the dipole moments of different +


+

molecules are randomly oriented due to thermal +


£,.0
+
-H
+
(b)
eP +
.F
\J +
-h

-I-
cGP ●E,
+
+

(0 00

Fig. 2.124 (i?) Polarization of a polar dielectric Fig. 2.125 (a) Polarization of a dielectric.
in an external electric field.
(6) Reduced field in a dielectric, E = Eq- E^.
ELECTROSTATIC POTENTIAL AND CAPACITANCE 2.57

moment per unit volume or the polarisation density


align themselves in the direction of Eq. This results in will be
uniform polarization of the dielectric, i.e., every small
volume of the slab has a dipole moment in the
p _ dipole moment of dielectric
volume of dielectric

direction of Eq. The positive charges of the dipoles of


= a
first vertical column cancel the negative chutges of the Ad A P

dipoles of second column and so on. Thus the volume t, „ i ● ! ■ j x. i j c j xj i..,


, . . , . . , , , , . Thus the polarisation densih/may be denned as the charge
charge density in the interior of the slab is zero. ■. r
° , . , induced per unit surface area.
However, there is a net uncancelied negative charge on
the left face and uncancelled positive charge on the Obviously, a uniformly polarised dielectric with
right face of the slab uniform polarisation density P can be replaced by two
The uncancelled charges are the induced surface surface layers (perpendicular to P) of surface charge
densities ± , and zero charge density in the interior.

w
charges due to the external field Eq. Since the slab as a
47. Define electric susceptibility. Deduce the relation
whole remains electrically neutral, the magnitude of
betiveen dielectric constant and electric susceptibility.
the positive induced surface charge is equal to that of

Flo
—»

the negative induced surface charge. Electric susceptibilit)'. If the field E is not large,
Thus the polarized dielectric is equivalent to two then the polarisation P is proportional to the resultant

ee
charged surfaces loith induced surface charge densities ±
field £ existing in the dielectric, i.e.,

Fr
Reduced field inside a dielectric and dielectric
constant. In case of a homogeneous and isotropic —>

dielectric, the induced surface charges set up an


P oc £ or
P=.r„l E

for
where x (chi) is a proportionality constant called
ur
electric field (field due to polarization) inside the
electric susceptibility. The multiplicative factor Sq is used
dielectric in a direction opposite to that of external to keep X dimensionless. Clearly,
●>
ks
field £q , thus tending to reduce the original field in the -»

P
Yo

X =
oo

dielectric. The resultant ifeld E in the dielectric will be ->

—► —^ —> Eq £
equal to Eq - £^, and directed in the direction of Eq.
eB

Thus the ratio of the polarisation to Eq times the electric


field is called the electric susceptibility of the dielectric. Like
The ratio of the original field Eq and the reduced p ilso describes the electrical behaviour of a dielec-
r

field ,£0
tri he dielectrics with constant % are called linear
- £p in the dielectric is called dielectric constant
ou
ad

f-'i tries.
(k) or relative permittivity (s^). Thus Relation between k and x- The net electric field in
Y

a
£. £ ised dielectric is
0 _ 0
K = —^
nd

£ = ,£0
Re

a
Fi

46. Define polarisation density. How is it related to But


the induced surface charge density ? ^0 ^0
P
Polarisation density. The induced dipole moment E = £
0
developed per unit volume of a dielectric zvhen placed in an ^0
external electric field is called polarisation densih/. It is
denoted by P. Suppose a dielectric slab of surface area A F ^0^ ^
and thickness d acquires a surface charge density ±
or £ =
^0 [P =6q X £]
%
due to its polarisation in the electric field and its two
faces acquire charges ± Q^,. Tlien Dividing both sides by £, we get
£
G
P
1=^ -X
A
£
We can consider the whole dielectric slab as a large
or l = K~x or K =l+ x
dipole having dipole moment equal to d. Tlie dipole
2.58 PHYSICS-XII

2.28 DIELECTRIC STRENGTH + —+ Q


i ^ i ^ i
+

48. What do you mean by dielectric strength of a 0 ,,

dielectric ?
●— Dielectric
,, E = Eo-£p
Dielectric strength. V\Tien a dielectric is placed in a f
slab

very high electric field, the outer electrons may get


detached from their parent atoms. The dielectric then
i
behaves like a conductor. This phenomenon is called
dielectric breakdown. Fig. 2.126 A dielectric slab placed in a
The maximum electric field that can exist in a dielectric parallel plate capacitor.
loithout causing the breakdown of its insulating property is + Qp on the lower surface of the dielectric. These
called dielectric strength of the material. induced charges set up a field £ inside the dielectric in
The unit of dielectric strength is same as that of tlie opposite direction of Eq. The induced field is given by
electric field i.c., Vm" ^ But the more common practical
unit is kV mm"'.
— =— =— = P, polarisation density]
6q Sq a
Table 2.1 Dielectric constants and dielectric
The net field inside the dielectric is
strengths of some common dielectrics.
E
0
£ = £0 = K
Dielectric
Dielectric Dielectric strength K
constant in kV mm“^
Vacuum 1.00000 CO where k is the dielectric constant of the slab. So

Air 1.00054 0.8


between the capacitor plates, the field £ exists over a
distance t and field £q exists over the remaining
Water 81
distance {d -1). Hence the potential difference
Paper 3.5 14 between the capacitor plates is
£ E0
Pyrex glass 4.5 13
V = £q (d - f) + Ef = Ep (if - 0 + — t = K
K E
Mica 5.4 160

Porcelain 6.5
t Q t
4 = £.0 d-t + - d-t + ~
Kj e«0 A K

2.29 CAPACITANCE OF A PARALLEL PLATE The capacitance of the capacitor on introduction of


dielectric slab becomes
CAPACITOR V/ITH A DIELECTRIC SLAB
Q 8,,0 A
49. Deduce the expression for the capacitance of a
V t ■
parallel plate capacitor when a dielectric slab is inserted d-t-i-
between its plates. Assume the slab thickness less than the K

plate separation.
Special Case If the dielectric fills the entire space
Capacitance of a parallel plate capacitor with a between the plates, then f = d, and we get
dielectric slab. The capacitance of a parallel plate £«0 A
capacitor of plate area A and plate separation d with C = . K = K C0
d
vacuum between its plates is given by
e.,A
Thus the capacitance of a parallel plate capacitor increases
Co =
0
K limes when its entire space isfilled with a dielectric material.
d C
Clearly, K =

Suppose initially the charges on the capacitor plates Co


are ± Q. Tlien the uniform electric field set up between Dielectric constant
the capacitor plates is
_ Capacitance with dielectric between two plates
o Q
As
Capacitance with vacuum between two plates
^0 0
Thus the dielectric constant of a dielectric material
When a dielectric slab of thickness t <d is placed
may be defined as the ratio of the capacitance of a capacitor
between the plates, the field Eq polarises the dielectric. completely filled with that material to the capacitance of the
Tliis induces charge - on the upper surface and
same capacitor zoith vacuum between its plates.
ELECTROSTATIC POTENTIAL AND CAPACITANCE

2.30 CAPACITANCE OF A PARALLEL PLATE 2.31 USES OF CAPACITORS


CAPACITOR WITH A CONDUCTING SLAB 51. Mention some important uses of capacitors.
50. Deduce the expression for the capacitance of a Uses of capacitors. Capacitors are very useful
parallel plate capacitor when a conducting slab is inserted circuit elements in any of the electric and electronic
betzueen its plates. Assume the slab thickness less than the circuits. Some of their uses are
plate separation. 1. To produce electric fields of desired patterns,
Capacitance of a parallel plate capacitor with a e.g., for Millikan's experiment.
conducting slab. Consider a parallel plate capacitor of 2. In radio circuits for tuning.
plate area A and plate separation d. If the space bet
3. In power supplies for smoothing the rectified
ween the plates is vacuum, its capacitance is given by current.
r
d 4. For producing rotating magnetic fields in
induction motors.

w
Suppose initially the charges on the capacitor plates 5. In the tank circuit of oscillators.
are ± Q. Then the uniform electric field set up between
the capacitor plates is 6. They store not only charge, but also energy in
the electric field between their plates.

Flo
a Q
Ae
^0 0
2.32 EFFECT OF DIELECTRIC ON

ee
where cr is the surface charge density. The potential VARIOUS PARAMETERS
difference between the capacitor plates will be

Fr
Qd 52. A parallel-plate capacitor is charged by a battery
V.0 = E.d
0
= which is then disconnected. A dielectric slab is then
Ae 0
inserted to fill the space betzveen the plates. Explain the
for
ur
When a conducting slab of thickness t <d is placed changes, if any, that occur in the values of (i) charge on
between the capacitor plates, free electrons flow inside the plates, (ii) electric field between the plates, (Hi) p.d.
it so as to reduce the field to zero inside the slab, as
between the plates, (iv) capacitance and (v) energy stored
ks
shown in Fig. 2.127. Charges - Q and + Q appear on in the capacitor.
Yo

the upper and lower faces of the slab. Now the electric
Effect of dielectric when the battery is kept
oo

field exists only in the vacuum regions between the


plates of the capacitor on the either side of the slab, i.e., disconnected from the capacitor. Let , Cq , Vq , Eq
eB

the field exists only in thickness d -1, therefore, potential and Uq be the charge, capacitance, potential difference,
difference between the plates of the capacitor is electric field and energy stored respectively before the
dielectric slab is inserted. Then
Q
V=E^{d-t) =
r

(d-t) V.
A £
ou

Qb=W,Eo=^, U
ad

0
0 “
— c
2
Y

{/) Charge. The charge on the capacitor plates


+ + + +

i u remains because the battery has been disconnected


nd

before the insertion of the dielectric slab.


Re

d t £ = 0
+ +
JTTq Conducting
slab
(ii) Electric field. When the dielectric slab is
i - i i -
Fi

-Q inserted between the plates, the induced surface


charge on the dielectric reduces the field to a new value
given by
Fig. 2.127 A conducting slab placed in a £
parallel plate capacitor.
K

Capacitance of the capacitor in the presence of (Hi) Potential difference. The reduction in the
conducting slab becomes electric field results in the decrease in potential difference.
C =^
Q
or C =
r d .c0 V = Ed =
y (d-t) d 'd-t d-t K K

(iv) Capacitance. As a result of the decrease in


Clearly, C > Cq. Thus the introduction of a conducting potential difference, the capacitance increases k times.
slab of thickness t in a parallel plate capacitor increases its
c = 9o._Qo Qo = K C0
capacitance by a factor of
= K

d-t y vjk
0
y.0
s 2.60

(i?) Energy stored. The energy stored decreases by


factor of K.
\2
1 u
PHYSICS-XII

a For Your Knowledge

^ Capacitance of a parallel plate capacitor with


U = ±CV^ = ● ^0^0
0

2 K 2 K compound dielectric.

53. A parallel plate capacitor is charged hy a battery. A. Series type arrangement. If a capacitor is filled with
When battery remains connected, a dielectric slab is inserted » dielectric slabs of thicknesses f^, as shown
between the plates. Explain what changes, if any, occur in in Fig. 2.128(a), then this arrangement is equivalent to n
capacitors connected in series.
the values of(i) p.d. between the plates, (U) electric ifeld
between the plates, (Hi) capacitance, (iv) charge on the With a single dielectric slab,
plates and (v) energy stored in the capacitor ? C =
0
A
t
Effect of dielectric when battery remains con d - t +
K
nected across the capacitor. Let Q^, Cq,Vq, Eq and Uq
be the charge, capacitance, potential difference, electric Capacitance with n dielectric slabs will be
field and energy stored respectively, before the C =
£«0 A
introduction of the dielectric slab. Then t
I
+
^2 + ... +
t
n

n = r V F =-^ U C V K
CM ^^2 K

'“O 0' 0 (f ' ^ 2 “


n J

But
ri — f| + f2 + ^3 + + t
n

(/) Potential difference. As the battery remains e«0 A


connected across the capacitor, so the potential difference C =
t
1 h t
n

remains constant at Vq even after the introduction of


+ + +
K K K
1 2
dielectric slab. n

(ii) Electric field. As the potential difference 9 A


9 A

remains unchanged, so the electric field Eq between the


capacitor plates remains unchanged. K1 t 1

E = —d = ^d = E^ 0 ^2 t-,

«3 ^3 K1 ^2 K3 K. d
[Hi) Capacitance. The capacitance inaeascs from Cg to C.
C = K C.0

(iu) Charge. The charge on the capacitor plates increases K, t,

from Qg to Q.
A] Aj A3 K
Q=ci^ = KCg.yg = K(3g. OB OB

(i>) Energy stored. The energy stored in the capacitor Fig. 2.128(a) Fig. 2.128(b)
increases k times.

Icv^ B. Parallel type arrangement, The arrangement shown


2 ^0^0 “ ^0'
U = = K .
2 in Fig. 2.128(b) consists of n capacitors in parallel,
having plate areas Aj , A^ , , A , and plate
Toble‘2.2 Effect of dielectric on various porameters.
U

separation d.

Battery disconnected Battery kept connected The equivalent capacitance of the parallel arrange
ment will be
from the capacitor across the capacitor
Q= (constant) Q=kQ) c=q + q + + c,

V. \
V=-^ V = Vg (constant) + ... + 5l!sA
K d d d

%
E=-^ £ = q (constant) or C =
-^(K^ A, + Kj A2 + ...+ K„ A„)
K
A
If Aj = Aj = = A„ = —
n
, then
n

u u = K a0 8„ A
U = ^ C =
0
K
d >1
(>h + Kj +
ELECTROSTATIC POTENTIAL AND CAPACITANCE 2.61

Examples based on is filled with a medium of dielectric constant 5. Calculate the


Capacitors Filled with Dielectrics value of capacitance of the capacitor in the second case.
[CBSE OD 06]
and Conductors
Formulae Used
Solution. Capacitance of the capacitor with air
between its plates.
1. Capacitance of a parallel plate capacitor filled
with a dielectric of dielectric constant k, ^=8pF
d ^
When the capacitor is filled with dielectric (k =5)
between its plates and the distance between the plates
2. Capacitance of a parallel plate capacitor with a dielec is reduced by half, capacitance becomes
tric slab of thickness t (< d) in between its plates. eQK/1 £q X 5 X A
C = EqA C =
U2~ Jn = 10 Co=10x8=80 pF.
d-t(l- ^

w
k; Example 81. Figure 2.129 shows two identical capacitors,
3. Capacitance of a parallel plate capacitor with a con Cj and C2, each of 1 pF capacitance connected to a batten/ of
ducting slab of thickness t {<d)in between its plates, 6V. Initial!}/ szvitch 'S' is closed. After some time 'S' is left

Flo
open and dielectric slabs of dielectric constant k =3 are
C =
d-t

ee
4. Capacitance of spherical capacitor filled with a

Fr
dielectric. 6 V— 1 pF C 1^F C2
ab
C = 4n 8«0 K.
b - a Fig. 2.129

for
ur
5. Capacitance of a cylindrical capacitor filled with a inserted to fdl completely the space between the plates of the
dielectric. hvo capacitors. How will the (i) charge and (ii) potential
C = 2rc £q K / difference between the plates of the capacitors be affected
ks
after the slabs are inserted ? [CBSE Dll]
2.303 log,
Yo

a
Solution. With switch S closed, V'.j = V2 " ^ ^
oo

6. Effect of dielectric with battery disconnected from 6V =6 pC


eB

the capacitor.
When dielectric slabs (k =3) are inserted, capaci
Q=a,V=^,£ = -^,C=KC,,U = -^
K K K
tance of each capacitor becomes 3 pF.
P.D. across C V W1 = 6 V
r

7. Effect of dielectric with battery connected across


ou
ad

the capacitor, Charge, (/i =3pFx6V = 18 pC


Q=kQ,,P = Po,£=^,C = Q,U = Kli,
Y

With switch S open, the p.d. on C2 attains a new


Units Used
value but charge ^2 ^
Capacitance C is in farad, charge q in coulomb,
nd
Re

potential difference V in volt, area A in m^, ^ 3pF


= 2 V.

thicknesses d and t in metre.


Fi

Constant Used Example 82. A parallel plate capacitor, of capacitance


20pF, is connected to a 100 V supply. After sometime the
Permittivity constant, Sq = 8.85xlO"’^C^N"^m"^. battery is disconnected, and the space, between the plates of
the capacitor is iflled with a dielectric, of dielectric constant
Example 79. In a parallel plate capacitor, the capacitance 5. Calculate the energy stored in the capacitor (i) before
increases from 4 pF to 80 pF, on introducing a dielectric (U) after the dielectric has been put in between its plates.
medium between the plates. What is the dielectric constant of [CBSEOD16C]
the medium ?
Solution. Here Cq =20pF, Vq =100V, tc = 5
Solution.
_ Capacitance with dielectric _80pF = 20.
= Cq Vq = 20 pF X100 V = 2000 pC
Capacitance without dielectric 4 pF Capacitance after the dielectric has been put,
Example 80. A parallel plate capacitor with air between C = kCq = 5x20pF = 100pF
the plates has a capacitance of 8 pF. The separation between As the battery has been disconnected, charge
the plates is now reduced by half and the space between them remains same.
PHYSICS-XIl

(i) Energy stored before the dielectric is put, (Hi) When dielectric slab (k = 2) is introduced, the
_l _ 1(2000x10“^)^ p.d. becomes
U J = 0.1 J. SxlO'^xO.Ol
V = EJd -1) + ^ t =1200 +
0 “ ● -6
2 C0 2 20x10 =1350 V.
K 2
(//) Energy stored after the dielectric is put,
Example 85. A parallel plate capacitor is charged to a
2 ji^l(2000xloy 0 certain potential difference. When a 3.0 mm thick slab is
C 2 100x10"^ slipped between the capacitor plates, then to maintain the
X
same p.d. between the plates, the plate separation is to be
Example 83. '^'wo parallel plate increased by 2.4 mm Find the dielectric constant of the slab.
capacitors, X and Y, have the same
area of plates and same separation Solution. Let £q be the electric field between the
+ -
between them. X has air between ja fi
capacitor plates before the introduction of the slab.
ls V
the plates while Y contains a Then, the p.d. between the plates is
dielectric medium of =4. Fig. 2.130

(0 Calculate capacitance of each capacitor if equivalent Suppose the separation between the plates is
capacitance of the combination is 4 pF. increased by fr' to maintain the same p.d. after the
introduction of the slab of thickness t. Then
(n) Calculate the potential difference between the plates
of X and Y. £

(Hi) Estimate the ratio of electrostatic energy stored in X V^=E^{d + d'-t) + ^.t K
and Y. [CBSE D 04, 09, 16]

Solution, (i) Let Cj, = C Then Cy = e^C = 4 C £p (rf + fr' - 0 + — .t=E.d


K
0

Now X and Y are connected in series. t 3.0 mm


or K = = 5.
C.4C t-d 3.0 mm -2.4 mm
C
eq
C;^+Cy C+4C
Example 86. The area of parallel plates of an air-filled
or
4pF=|C or C = 5pF capacitor is 0.20 irP' and the distance between them is 0.01 m.
The p.d. across the plates is 3000 V. When a 0.01^« thick
Hence Cj,=C = 5pF and Cy = 4C = 4x 5 =20 pF. dielectric sheet is placed between the plates, the p.d. decreases
(ii) Total charge, Q = CV = 4pF x 15 V = 60pC to 1000 V. Determine (i) capacitance of the capacitor before
placing the sheet (ii) charge on each plate (in) dielectric
Q _60pC
V'.
X ~
= 12 V constant of the material (iv) capacitance of the capacitor after
5pF
placing the dielectric (v) permittivity of the dielectric. Given
-12
Fm-\
Q ^60pC = 3 V. Ep =8.85x10
Cy 20pF Solution. (/) Capacitance of air-filled capacitor is
Energy stored in X 4C(12)^ 4 = 4:1. C0
e^A 8.85 X 10'^^ X 0.20 = 1.77x 10
-10
F.
(ill) ^ ^ =- d oE
Energy stored in V f 4C(3r ^
(ii) Charge on each plate,
Example 84. An electric field Eq = 3 x 101/w”’ is (j = Cq Vq =1.77 X 10"^” X 3000 = 5.31 x 10“^ C.
established between the plates, 0.05 m apart, of a parallel
plate capacitor. After removing the charging battery, an (m) Dielectric constant of the material is
uncharged metal plate of thickness t=0.01m is inserted K =
C _q/V _Vq _3000
between the capacitor plates. Find the p.d. across the q/V^~ V ^1000 ”
capacitor (i) before, (ii) after the introduction of the plate.
(Hi) What would be the p.d. if a dielectric slab (k = 2) were (iv) Capacitance after the dielectric sheet is introduced,
introduced in place of metal plate ? C = K Cq =3 X 1.77 X 10“^“ = 5.31 x 10"^° F.
Solution. (/) The p.d. across the capacitor plates (lO Permittivity of the dielectric is
before metal plate is inserted, £ = K Eg = 3 X 8.85 X 10'^^ = 2.65 x 10“^^ Fm
Vq = £q (i =3 X lO'* X 0.05 = 1500 V. Example 87. The capacitance of a parallel plate capacitor is
(ii) As no electric field exists in metal plate, so the 50 pF and the distance between the plates is 4 mm It is
p.d. after the introduction of metal plate is charged to 200 V and then the charging battery is removed.
V = Eq (d - f) =3 X lO'^ X (0.05 -0.01) = 1200 V. Now a dielectric slab (k = 4) of thickness 2 mm is placed.
ELECTROSTATIC POTENTIAL AND CAPACITANCE
2.63

Determine (i) final charge on each plate (ii) ifnal potential has a thickness 3d / 4, where d is the separation of the plates,
difference between the plates (Hi) final energy in the How is the capacitance changed when the slab is mserted
capacitor and (iv) energit loss. between the plates ? [NCERT; CBSE F 17 ; lEE Mam 21, 22)
Solution.
Solution. Capacitance of air-filled capacitor,
r ...(1)
0" d C, C2
Capacitance with dielectric slab of thickness t {< d) is o- ■o

e«0 A K

C = ...(2)
d-t + t/K
d 3d
(i) The charge on capacitor plates, when 200 V p.d. 4 4
is applied, becomes
8«A
(j = Cq Vo = 50xl0 X 200 =10“® C
-12
_ ‘'0

w
Even after the battery is removed, the charge of
10 ® C on the capacitor plates remains the same. and C2 are in series.
(ii) On placing the dielectric slab, suppose the capa . 3d/4

Flo
citance becomes C and potential difference V. Then C CI ^2 0
KEnA
0

q = C,V, = CV

ee
1 3 1 S']
1+-
d-t + 11K

Fr
y.0 4C 0 4kC.0 4C 0 Kj
or y =
C
^0 = d
4k
[Using (1) and (2)] C = Co
4-2 +2/4
for
K +3
ur
X 200 = 125 V.
4 Example 90.
(Hi) Final energy in the capacitor is (fl) Find the ratio of the capacitances of a capacitor filled
ks
with two dielectrics of same dimensions but of dielectric
lqV=^x\0^x 125 = 6.25X 10“ ^ J.
Yo
U =
constants and K2 , respectively,
oo

(iv) Energy loss (b) A capacitor is filled with tioo dielectrics of the same
eB

dimensions but of dielectric constants K.1 =2 and


= U0 -U=iq(V„-V) K
2=3. Find the ratio of capacities in tivo possible
arrangements.
= ^xl0“®x(200-125) = 3.75x 10“^ J.
r
ou
ad

Example 88. A parallel plate capacitor is formed by two


plates, each of area 100 cn^, separated by a distance of 1
Y

mm.

A dielectric of dielectric constant 5 and dielectric strength Kj


Kj
1.9xl0^yw“’ is filled betzueen the plates. Find the
nd
Re

maximum charge that can be stored on the capacitor without


causing any dielectric breakdozon.
Fi

Solution. Electric field between capacitor plates is Fig. 2.131 (a) (h)

given by Solution, (a) The two possible arrangements of the


G q
£ = two dielectrics are shown in Figs. 2.131(fl) and (1;).
K e K e„0 A
(2) The arrangement (ii) can be supposed to be a
0

As the electric field should not exceed 1.9 x 10^ Vm“\ parallel combination of two capacitors, each with plate
so the maximum charge that can be stored is area A/2 and separation d. Therefore, the total capa-
citance is
q = K Eq AE
= 5 X 8.85 X 10“^^ X 100 X 10“^ X 1.9 x 10^ £n(^/2)K 1 ep(A/2)K2
= 8.4 X 10“® C.
c = q + C2= -° d d

Example 89. A slab of material of dielectric constant k has Sq A(k^ + K2)


the same area as the plates of a parallel plate capacitor but 2d
2.64 PHYSICS-XII

(//) The arrangement {b) can be supposed to be a citance of the capacitor becomes 2/3rd of its original
series combination of two capacitors, each with plate value. What is the new distance between the
area A and separation fr/2. Therefore, the total plates ? [CBSE OD 08C1 (Ans. 8 mm)
capacitance C' is given by
7. The distance between the parallel plates of a charged
— = 2l 1 1
+
1
capacitor is 5 cm and the intensity of electric field is
c' c:1 300 V cm A slab of dielectric constant 5 and thick-
d/2 d/2 ness 1 cm is inserted parallel to the plates. Determine
d the potential difference between the plates, before and
2e^A
— + —
after the slab is inserted ? (Ans. 1500 V, 1260 V)
0
8. A parallel plate capacitor with plate separation
or C' =
2e„0 A 5 mm is charged by a battery. It is found that on
d K,
1
+ K
2>
introducing a mica sheet 2 mm thick, while keeping
the battery connections intact, the capacitor draws
Ratio of the capacitances in tlie two arrangements is 25% more energy from the battery than before. Find
the dielectric constant of mica. (Ans. 2)
C _ gp A(Kj K^) d(K^+K^)
C 2d
2e„0 Ak^K2 4k,K 9. Figure 2.132 shows a parallel plate capacitor of
plate area A and plate separation d. Its entire space
(1j) Here k.j=2,K2=3 is filled with three different dielectric slabs of same

thickness. Find the equivalent capacitance of the


C _ (2+3)^ _25 arrangement. 3 Eq A Kj K2 K3
C 4 X 2 X 3 ~ 24 Ans. C =
d (Kj Kj + K2 K3 + K3 Kj)
9 A 9 A
roblems For Practice
T

1. A parallel-plate capacitor having plate area 100 cm- d/3

and separation 1.0 mm holds a charge of 0.12 )iC d/3 K1 K, K3


when connected to a 120 V battery. Find the
d/3
dielectric constant of the material filling the gap. ±.

(Ans. 11.3)
9B
2. Find the length of the paper used in a capacitor of 9B

capacitance 2pF, if the dielectric constant of the Fig. 2.132 Fig. 2.133
paper is 2.5 and its width and thickness are 50 mm
and 0.05 mm, respectively. (Ans. 90 m)
10. The space between the plates of a parallel plate
capacitor of capacitance C is filled with three
3. A parallel-plate capacitor consists of 26 metal strips, dielectric slabs of equal thickness, as shown in
each of 3 cm x4cm, separated by mica sheets of Fig. 2.133. If the dielectric constants of the three
dielectric constant 6 and uniform thickness 0.2 mm.
slabs are K|, K2 and k^, find the new capacitance.
Find the capacitance. (Ans. 7.97 X 10“ ^F) C
[Ans. C' = -(K, + K2 + K3)]
4. A parallel-plate capacitor of capacity 0.5 pF is to be
constructed using paper sheets of thickness 0.04 mm
11. A slab of material of dielectric constant k has the
as dielectric. Find, how many circular metal foils same area as the plates of a parallel plate capacitor
of diameter 0.1 m will have to be used. Take the
but has tliickness fr / 2, where d is the separation
dielectric constant of paper used as 4. (Ans. 73) between the plates. Find the expression for the
5. When a slab of insulating material 4 mm thick is capacitance when the slab is inserted between the
plates. [CBSE F 10; OD 131 2k
introduced between the plates of a parallel plate Ans. C0
capacitor, it is found that the distance between the K+1

plates has to be increased by 3.2 mm to restore the HINTS


capacitance to the original value. Calculate the
dielectric constant of the material. q 0.12 pC = 10"^F
(Ans. 5) 1. Capacitance, C =
V “ 120 V
6. The two plates of a parallel plate capacitor are

4 mm apart. A slab of dielectric constant 3 and If K is the dielectric constant, then


thickness 3 mm is introduced between the plates KSp A _ kx8.85x10~*^x100x1Q~* = lO'^F
with its faces parallel to them. The distance d 1.0 X 10"^
between the plates is so adjusted that the capa- K = 11.3.
ELECTROSTATIC POTENTIAL AND CAPACITANCE

3- Arrangement of n metal plates separated by dielectric For charge to remain constant.


acts as a parallel combination of (/i - 1) capacitors. 0) V'o = cy
C = (« - 1) K 6q a 25 0,
d Cq X 1500 = 21 xV or y=1260V.
-12
25 x6x8.85x10 x3x4x]0“‘^
8. As the battery connections are intact (V = constant)
0.2 X 10"^ and the capacitor draws 25% more charge, so the
= 7.97x10'^ F. capacitance also Increases by 25%. That is
125
C = C0
4. As ^ (>?-l)Keo A 100

d
or
Sq ^ —^ ^ ^
.-. 0.5xl0"^F \ ~
4‘ d
-12
d-t \ l- ^
(»-l)x4x8.85xl0 x3.14x(0.05)-

w
0.04 X 10'^ 1 4d ■ n d
or d-t 1-- or t 1
K 5 K 5
0.5x0.04x10^
or n-\ = = 71.97-72
4x8.85x3.14x(0.05)^ or or k = 2.

Flo
K 5f 5x2 2
or n = 73.
4. Tine given arrangement is equivalent to three

ee
So A capacitors connected in series. Each such capacitor
5. Capacitance without dielectric, C =™
has plate area A and plate separation d.

Fr
When dielectric is introduced. C =
So A Kj Sq A 3 Kj
_ __ _
A _

1 d
d'-t 1- ●●
Kj 3 K, £n A 3 K3 Eg A
for
ur
As the capacitance remains same in both cases, so q= and q= ^
The equivalent capacity Cof the given arrangement
Sq ^ ^ £q ^ is given by
ks
1 1 1 1 d ( 1 1 1
Yo

+ + +
k;
oo

C C2 C3 3soAI^Kj K2 K
3>
1
d = d'-f 1-- or d'-d=t 1- C = 3 ^ A Kj K2 K3
eB

or or
k; K
d (Kj K2 + K2 Kg + Kg K^ )
But fr'-d = 3.2mm, t=4mm 8„0 A
1 10. Original capacitance, C=
d
r

3.2 = 4 1 - -
ou

ic; Tine new arrangement is equivalent to three capaci


ad

3.2
= 0.8 or
tors connected in parallel. Each such capacitor has
or - = 0.2 or K = 5.
Y

K 4 K plate area A / 3 and plate separation d. The new


●y
capacitance is
(1. ^ X Capacitance with air - Capacitance with dielectric c'=q + q + C3
nd
Re

KjSqTI/S K2 Bq a / 3 _ Kg SgA/S
or 2£q-^_ +
Fi

t d d d
3 d
t + - Sn0 A
K
3d
(Kj +.Kt + Kg)
or Hd- -3+- 3^1
=d = 4mm or d'=8mm C
3V 3J or C' = -(Kj +K2 + K3).
7. P.D. before the dielectric slab is inserted,
-1
11. Without dielectric, =~
V^ = Ef)d=^ 300 V cm X 5 cm = 1500 V.

So A_Sq A farad With dielectric,


K
0.05
or En0 A = 0.05 C 0 V=E^.-+
^ 2 £.-2 ^'2 K '2
Capacitance with dielectric slab, _ qd (k+ 1) _ Vo(^+
2 2k
C = hA 0.05 C 0 25 C
0
K

d -t +
f
0.05-0.01+ ●●
0.01 21
or C= — =
iKqQ 2k
K 5 y Vo(k+1)”k7i
2.66 PHYSICS-XII

2.33 DISCHARGING ACTION OF SHARP Total potential on the outer sphere,


POINTS : CORONA DISCHARGE = Potential due to its own charge Q
54. Briefly explain discharging action of sharp points + Potential due to the charge q on
the inner sphere
or corona discharge.
Discharging action of sharp points : Corona _ 1 Q^l Insulating
4?: R R_
discharge. When a spherical conductor of radius r suspension
carries a charge q, its surface charge density is Potential on the inner
sphere due to its own charge is
o = —
A 47t
^1 =
Electric field on the surface is
4tc8q r
a ‘I
As the potential at every
£ = — point inside a charged sphere
^0 4k e„0
is the same as tliat on its
surface, so potential on the
< 0 © ► Fig. 2.135 Small charged
inner sphere due to charge Q
sphere suspended inside a
<—G ©—► on outer sphere is
Low E +1 +
charged spherical shell.
4—G ©—► 1 Q
^2 =
+
High £ .4 O © ► 4ne
0
R

Total potential on inner sphere


E ] 1

4k8q [r R
Fig. 2.134 Corona discharge.
Hence the potential difference is
The pointed end of a conductor is highly curved 1 11
and its radius of curvature r very small. If the con
4K8oLr R
ductor is given a charge q, then the charge density o at
the pointed end will be very high. Consequently, the So if q is positive, the potential of the inner sphere
will always be higher than that of the outer sphere,
electric field near the pointed end will be very high
which may cause the ionisation or electrical break- Now if the two spheres are connected by a conducting
down of the surrounding air. The oppositely charged wire, the charge 7 will flow entirely to the outer sphere,
ions neutralise the pointed end while the similarly irrespective of the charge Q already present on the
charged ions are repelled away. Fresh air molecules outer sphere. In fact this is true for conductors of any
come near the pointed end and take away its charge, shape,
setting up a kind of electric wind. This process by which
the charge at the pointed end of a conductor gets discharged 2.35 VAN DE GRAAFF GENERATOR*
is called corona discharge. The discharge is often 56. Explain the basic principle, construction and
accompanied by a visible glow near the pointed end. working of Van de Graaff generator.
Van de Graaff generator. It is an electrostatic
2.34 COLLECTING ACTION OF A generator capable of building up high potential differences of
HOLLOW CONDUCTOR the order of 10^ volts.
55. A small sphere of radius rand charge q is enclosed Principle. The working of a Van de Graaff gene-
by a spherical shell of radius R and charge Q. Show that if j-ator is based on following two electrostatic phenomena:
q is positive, charge q will necessarily flow from the {i) Discharging action of sharp points (corona
sphere to the shell (when the two are connected by a wire) discharge) i.e., electric discharge takes place in air or
no matter what the charge Q on the shell is. [NCERTI
gases readily at the pointed ends of conductors,
Collecting action of a hollow sphere. Consider a (ii) If a charged conductor is brought into internal
small sphere of radius r placed inside a large spherical contact with a hollow conductor, all of its charge
shell of radius R. Let the spheres carry charges q and Q, transfers to the hollow conductor, howsoever high the
respectively. potential of the latter may be.
ELECTROSTATIC POTENTIAL AND CAPACITANCE

Construction. A large spherical conducting shell (of


few metres radius) is supported at a height several + Metal

metres above the ground on an insulating coluirm. A shell, S

long narrow belt of insulating material, like rubber or


silk, is wound around two pulleys, at ground level
and ?2 at the centre of the shell.
This belt is kept continuously moving by an electric o +

motor attached to the lower pulley Py Near the bottom +


and the top of its run, the belt passes close to two
Collector
P2 / Ion
sharply pointed brass combs Bj and pointing + +
source
comb +

towards the belt. The comb By called spray comb is Insulating belt to carry
given a positive potential of 10 kV with respect to the + and deliver charge
earth by means of a battery ; while the comb B2’ called

w
collecting comb, is connected to the spherical shell S. + ●—Insulating pillar
Working. Due to the high electric field at the +
pointed ends of comb By the air of the neighbourhood ●Discharge tube

Flo
B, +
gets ionised and its positive charge repelled or sprayed
on to the belt, which moves up into the shell S. As it Spray ► Grounded metal

ee
passes close to comb it induces a negative charge at comb ► + base

the pointed ends of comb B^ and a positive charge on ► ^

Fr
the shell S. The positive charge spreads uniformly on
the outer surface of the shell S. The high electric field at
the pointed ends of comb ^ ionises the air there and H.T.B. '
for r
ur
repels the negative charges on to the belt which Target
neutralise its positive charge. This process continues.
As more and more positive charge is given to the shell,
ks
its potential continues to rise. In this way, a high
Yo

potential of 6 to 8 million volts can be built upon the


oo

sphere.
eB

Fig. 2.136
A discharge tube is placed with its upper end
inside the hollow sphere and lower end earthed. The
ion source is placed at the upper end of the tube. The Use. The high potential difference set up in a
high potential on the sphere repels the charged Van de Graaff generator is used to accelerate charged
r
ou

particles downward with large acceleration, where particles like protons, deutrons, a-particles, etc. to high
ad

they hit the target atoms to bring about the nuclear energies of about 10 MeV, needed for experiments to
Y

disintegration. probe the small scale structure of matter.


nd
Re
Fi

Very Short Answer Conceptual Problems

Problem 1. Is the electrostatic potential necessarily Examples. (/) Electric field inside a charged con-
zero at a point where the electric field strength is zero ? ducting sphere is zero but potential at any inside pomt is
the same as that on the surface of the sphere.
Give an example to illustrate your answer.
Solution. No. We know that the electric field is equal (ii) Electric field at the midpoint of the line joining two
to the negative of potential gradient: equal and similar charges is zero, but potential at this
dV
point is twice of that due to a single charge.
E = Problem 2. Can electrostatic potential at a point be
dr
zero, while electric field at that point is not zero ?
This implies that even if the electric field at a point is Solution. Yes. For example, the potential at any point
zero, the potential may have some non-zero constant on
the perpendicular bisector of a dipole axis is zero,
value at that point. white electric field is not zero.
2.68 PHYSICS-XII

Problem 3. The electric potential is constant in a Solution. The points /land Care at the same potential.
given region. What can you say about the electric field
there ?
[Haryana 01)
Solution. We know that £=
dV Hence, the magnitude of the work done in taking
dr charge + q from /I to B or from S to C will be same i.e.,

As V is constant, so electric field £is zero. W, = W^.


Problem 4. In a certain 0.1 of space, electric Problem 10. A uniform electric field E exists bet
ween two charged plates as shown in the figure. What
potential is found to be 5 V throughout. What is the would be the work done in
electric field in this region ? [ICSE 97]
dV
moving a charge 'q' along +

Solution. We know that £=


dr the closed rectangular path
ABCDA? [CBSE D OICJ + A\ *—I B
As electric potential is 5 V throughout i.e., constant, so +
●»>

Solution. Zero. Electric I.

the electric field is zero in the given region. +


' ’

Problem 5. When kept in an electric field, does a field is a conservative field, Dl—< IC
so
* no work is done in moving
+
proton move from lower to higher or from higher to
lower potential region ? -® charge q along a closed path
in a uniform electric field. Fig. 2.139
Solution. Proton is a positively charged particle. In an
electric field, it will move from higher to lower potential Problem 11. What is the work done in moving a
region so as to reduce its potential energy. 2 microcoulomb point charge from comer A to comer B
Problem 6. Can there be a potential difference ® square ABCD shown in
between two neighbouring conductors carrying equal Fig-2.140,whenalOjiCcharge A B

positive charges ? exists at the centre of the


Solution. Yes. The potential of a conductor depends square ?
not only on the net charge carried by it, but also on its
geometrical shape and size. So two conductors of Solution. As the points A 10 pC
and B are at the same
different size and shape will have different potentials
even if they carry equal charges. distance from the charge of D C
10 pC at the centre, so 10 cm
Problem 7. A positive charge +qis located at a point.
~^B- Herice the work
What is the work done if a unit positive charge is carried
once round this charge along a circle of radius r about done in moving a charge of Fig. 2.140

this point ? 2 pC from A to B will be zero.

Solution. Work done is zero. Force on the unit positive Problem 12. Two protons A and B are placed bet
charge is along the radius and direction of motion is
ween two parallel plates having a potential difference V,
perpendicular to it. W = FS cos 90° = 0.
as shown in Fig. 2.143. Will
Problem 8. What would be the work done if a point these protons experience +

charge -hq, is taken from a point A to the point B on the equal or unequal force ?

+

circumference of a circle with [CBSE D 98) +

another point charge +q at the Solution.


+
The electric
centre ? [CBSE D OICJ +
field is uniform in the space ●A
Solution. The points A and B ● between the two plates.
+

are at same distance from the M- V-


Hence the protons A and B -M

charge + q at the centre, so


will experience equal force. Fig. 2.141
= Vp. Hence the work done in
taking another charge from point Problem 13. A point charge<7 is placed atO, as shown
A to B will be zero.
in Fig. 2.142. Is - Vg positive, negative or zero, if ^ is a
Problem 9. In Fig. 2.138, charge
+Q is placed at the centre of a (i) positive. Hi) negative charge ?
[CBSE D 01C 16;OD 06]
dashed circle. Work done in taking '
another charge +q from A to B is W^ ;
'I
and from B toCis Wj. Which one of \ +
O
the following is correct : > ,W2' A B

Wj = Wj and Wj < Wj ? [CBSE SP 18]


Fig. 2.142
ELECTROSTATIC POTENTIAL AND CAPACITANCE 2.69

Solution. Clearly, Problem 19. Can two equipotential surfaces intersect


1 1 1 each other ? Give reason. [CBSE D 09]
4jr Eg OA 4k£q OB 4n [ OA OB Solution. No. If two equipotential surfaces intersect,
then there would be two values of electric potential at the
As OA < OB, so the quantity within brackets is positive. point of intersection, which is not possible.
(/) If is a positive charge, then - Vg is positive, Problem 20. For any charge configuration,
(ii) If ^ is a negative charge, then - Vg is negative. equipotential surface through a point is normal to the
Problem 14. A charged particle (+q) in a uniform electric field". Justify. [CBSE D14 ; OD 16]
Solution. If the electric field were not normal to the
electric field E moves in the opposite direction of E .
equipotential surface, it would have a non-zero component
What will be the effect on the electrostatic potential along the surface. To move a test charge against this
energy during its motion ? [CBSE OD 20]
component, work would have to be done. But no work is
Solution. In the opposite direction of E, the charged needed to move a test charge on an equipotential surface.

w
Hence, electric field must be normal to the equipotential
particle moves from low potential region to high potential
surface at every point.
region. So its electrostatic P.E. energy increases.
Problem 21. Why must electrostatic field be normal
Problem 15. Express the unit of electric potential in

Flo
terms of the base units of SI. [ISCE 02]
to the surface at every point of a charged conductor ?
[CBSE D 12]
Solution. Electric potential

ee
Work done ML^ Solution. In electrostatic situation, Ehas to ensure that
= ML^T-^A"’
the free charges on the surface do not experience any force

Fr
Charge AT
or there are no surface currents. So £ must not have any
SI unit of electric potential = kg m^s"^A"'.
tangential component.
Problem 16. A metallic sphere is placed in a uniform
for
ur
Problem 22. Why should electrostatic field be zero
electric field. Which path is followed by lines of force inside a conductor ? [CBSE D 12]
shown in Fig. 2.143. [IIT 96 ; CBSE F 2010]
Solution. If any electric field exists inside a conductor,
ks
it would produce ordered motion of electrons without the
Yo

2
expense of any external energy. This violates the law of
oo

3 > conservation of energy.


Problem 23. Why is electrostatic potential constant
eB

4 throughout the volume of the conductor and has the same


value (as inside) on its surface ? [CBSE D 12; OD 19]
Fig. 2.143
Solution. Electric field is zero throughout the volume
r
ou

Solution. Path 4. This is because lines of force start and of the conductor.
ad

end normally at the surface of a conductor and do not So £=-


dV
= 0 ^ V = constant.
Y

exist inside it. dr

Problem 17. The work done in moving a charged Moreover, the potential just inside the conductor has
particle between two points in a uniform electric field
nd

to be equal to that on its surface.


Re

does not depend on the path followed by the particle.


Why? [CBSE OD 20]
Problem 24. No work is done in moving a test charge
Fi

Solution. Electrostatic force is a conservative force.


over an equipotentialsurface. Why ? [Punjab 01]
Solution. The potential difference. between any two
The work done in moving a charged particle in a uniform
points on an equipotential surface, AV = 0.
electric field does not depend on the path followed by the
particle. Work done, V/ -% AV ~ 0.
Problem 18. No work is done in taking a positive Problem 25. Does the charge given to a metallic
charge from one point to another inside a positively sphere depend on whether it is hollow or solid ? Give
charged metallic shell, while outside the shell work has reason for your answer. [CBSE D 17]
to be done in taking the charge from one point to the Solution. No, because the charge resides only on the
other towards the shell. Why ? surface of the conductor. Both hollow and solid spheres
Solution. All points inside the metallic shell are at the will have same capacitance (C = 4Tt£gR) if they have the
same potential. So no work is done in moving any charge same radii. When charged to same potential, they will
inside the charged shell. While outside the shell, there is a have the same charge.
potential gradient and electric field. Work has to be done Problem 26. Can a metal sphere of radius 1 cm hold a
in moving the charge against this field. charge of 1 coulomb ? Justify your answer. [Punjab 95C]
s 2.10

Solution. Here l? = lcm = 10 ^m, q = \C


Electric field on the surface of the sphere,
1
PHYSICS-Xll

By daily standards, the SI unit of charge, the coulomb,


is very large and the SI unit of energy, the joule, is a
reasonable unit of energy. As a result, the SI unit of
= 9 X 10% = 9 X 10^^ Vm -1 potential difference, the volt (IV = 1 JC"^)is a very small
4n 0
■ r2 (10-2)2 unit. So the farad, the ratio of the coulomb to the volt, is
The field is much greater than the dielectric strength doubly large. In other words, the largeness of the farad
of air (3 x 10^ Vm- ^). It causes ionisation of the surroun goes as the square of the largeness of the coulomb.
ding air and the charge of the metal sphere leaks into the Problem 32. A metal plate is + Q -Q
surrounding air. Thus a metal sphere of 1cm radius introduced between the plates
cannot hold a charge of 1C. of a charged parallel plate >
Problem 27. What is the justification of choosing the capacitor. Sketch the electric
earth as the zero of potential in practice ? lines of force between the
>
Solution. If we consider earth to be a conducting plates,
sphere surrounded by air, its capacitance will be equal to Solution. Inside the metal
4n 8q R. Since the radius Rof the earth is very large, so its plate, electric field is zero, so no Metal plate
capacitance is also very large. A charge q supplied to the lines of force exist inside the metal
earth will increase its potential by amount, Fig. 2.144
plate, as shown in Fig. 2.144.
V=1 Problem 33. What is the dielectric constant of a metal ?
C
Solution. The dielectric constant of a metal is infinity.
As C is very large, therefore V = —
C
-> 0 for all finite The electric field inside a conductor is zero so the
charges. dielectric constant, which is the ratio of applied electric
field to the reduced electric field, will be infinite for the
Problem 28. Can we take the potential of the earth as
metallic conductor.
+100 V ? What effect would such an assumption have on
the measured values of (a) potentials at various positions Problem 34. Is there any kind of material which
and (f>) potential difference between two given points ? when placed between the plates of a capacitor reduces
its capacitance ?
Solution. All potentials are measured relative to a Solution. No. The dielectric constant of a material is
reference position, to which we assign a zero value of
potential. Earth is assigned zero potential value as it is always greater than 1.
generally taken as a reference. Q
As K =
and K > 1 .'. > Cj,
C,
However, earth can be assigned a value of + 100 V.
This only increases the potential values by 100 V. But the
the capacitance with dielectric between the plates is
potential difference between two points would still greater than that with vacuum between the plates. So
remain the same. there is no such material which when placed between the
Problem 29. Can we give any desired charge to a plates of a capacitor will reduce its capacitance,
capacitor ? Problem 35. For a given medium, the dielectric
Solution. No, as we increase the charge on the plates of constant is unity. What is its permittivity ?-12
a capacitor, the potential difference between them also Solution. Permittivity, e = EqK = 8.85 x 10 xl

increases. A stage is reached when the electric field bet = 8.85x10"’2 c2N"%-2.
ween the two plates attains the breakdown value of air. Problem 36. Why do ordinary capacitors have
The surrounding air gets ionised and the charge begins to
leak into air. capacities of the order of microfarads ?
Problem30. What happens if the plates of a charged Solution. Farad is a very large unit of capacitance. So
capacitor are suddenly connected by a conducting wire ? capacitors having smaller capacitances of the order of
l|iF (= 10 ®F) are more common.
Solution. The capacitor plates will get discharged
Problem 37. Why does the electric field inside a
immediately. The energy stored in the capacitor changes
into heat energy. dielectric decrease when it is placed in an external
electric field ? [CBSE D 05]
Problem 31. Why is the farad an inconveniently large
Or
unit of capacitance ?
Solution. We know that How is the electric field due to a charged parallel
plate capacitor affected when a dielectric slab is inserted
1F =
1C _1C2 1 V =
IJ between the plates fully occupying the intervening
Tv ij 1C region ? [CBSE F 10]
ELECTROSTATIC POTENTIAL AND CAPACITANCE 2.71

Solution. Due to polarisation of the dielectric, an Solution. As capacitance C is independent of charge Q


electric field is induced inside the dielectric in the
so graph between C and Q is a straight line parallel to the
opposite direction of external electric field. Thus the net charge axis, as shown in Fig. 2.146.
electric field decreases inside the dielectric.
Problem 38. The introduction of dielectric slab between
C = constant
the capacitor plates increases the capacitance. Why ?
Solution. A dielectric slab of dielectric constant k
u
reduces the electric field from E to £/ k. This reduces the
potential difference from V to V / k. Hence the capacitance
increases from C to kC.
Q —
Problem 39. A thin metal sheet is placed in the
middle of a parallel plate capacitor. What will be the Fig. 2.146
effect on the capacitance?

w
Solution. No effect. When the metal sheet is placed in Problem 44. Two plates are placed side by side. How
the middle, the new arrangement is equivalent to a series many capacitors are formed ?
combination of two capacitors, each of plate separation Solution. Three, first between distant earthed bodies

Flo
d / 2 and hence capacitance 2 C. and the first face of the first plate, the second between the
2Cx2C two plates and the third between the second face of the

ee
= C.
2C + 2C second plate and the distant earthed objects. The
capacitances of the first and third capacitors are negligibly

Fr
Problem 40. Suppose a charge +Qj is given to the small than capacitance of the second capacitor.
positive plate and a charge -Q2 negative plate of a Problem 45. Draw the lines of force between the
plates of a charged parallel plate capacitor.
capacitor. What is the charge on the capacitor ? for
ur
[CBSE Sample Paper 03]
Solution. The charge on a capacitor is equal to tlie
Solution. The electric lines of force between the plates
charge on its positive plate. So the charge on the given
of a charged parallel plate capacitor are shown in Fig. 2.147.
ks
capacitor is + Q,.
Yo

*■
Problem 41. Is the capacitance C of a capacitor pro
oo

portional to the charge Q ?


eB

*■
Solution. No. As the charge Qincreases, the potential
difference V also increases in the same proportion, so the
capacitance C=Q/V, remains unaffected i.e., C is
r

independent of Q.
ou
ad

Problem 42. Sketch a graph to show how the charge


Q acquired by a capacitor of capacitance C varies with
Y

>
increase in potential difference between its plates.
Solution. As Qoc V, the graph between Q and y is a
nd
Re

straight line with slope Q/V=C, as shown in Fig. 2.145.


Fi

Fig. 2.147

Problem 46. The space between the plates of a


t parallel plate capacitor is filled consecutively with two
a
dielectric layers 1 and 2 having the thicknesses dj anddj
and the relative permittivities e, and £2 respectively. The
area of each plate is equal to A. What is the capacitance
of the capacitor ?
Solution. The effective separation between the plates
is 1/ e times the geometrical separation. So the capacitance
of the given parallel plate capacitor is
Sq
Problem 43. Sketch a graph to show how the capa C =
citance C of a capacitor varies with the charge Q given
to it. Si 82
2.72 PHYSICS-XII

Problem 47. Two insulated charged spheres of radii Solution. When the plastic part is removed, the
7 cm and 13 cm and having the same charge are con- capacitance of the metal die decreases but the charge
nected by a conductor and then they are separated, (produced by friction) remains unchanged and so the voltage
Which of the two spheres will carry greater charge ? increases in accordance with the relation : (2= CV.
Solution. The sphere of 13 cm radius has greater Problem 53. What happens to the stored energy if
capacitance than the sphere of 7 cm radius (C = 4n Eq R). after disconnecting the battery, the plates of the charged
So the sphere of 13 cm radius will carry more charge, capacitor are drawn apart ?
Q=CV. Both spheres attain same potential V when
Solution. As C ccl/ d, so when the plates are drawn
connected by a conductor.
apart, the capacitance decreases. After disconnecting the
In fact, y _ Q _ Qz battery, the charge on plates remains constant. Hence the
Cl energy stored in the capacitor, U = q^ 12C increases.
Problem 48. A spherical shell with radius a and Problem 54. What happens to the energy stored in a
chargeQ is expanded to radius b. What is the work done capacitor, if the plates of a charged capacitor are drawn
apart, the battery remaining connected ?
by the electrostatic force in this process ?
Solution. Work done by the electrostatic force Solution. When the plates are drawn apart, the
capacitance decreases. As the battery remains connected,
= Initial stored energy - Final stored energy the potential difference remains constant.
=1 -i 1 (f
2C^ 2 C2 2 4nEQfl 2 in SQb Hence energy stored, U = ^CV ^ decreases.
(fill Problem 55. When a capacitor is charged and then
8n \_a b_ discharged repeatedly, its dielectric gets heated. Why ?
Solution. The energy consumed during the polari
Problem 49. By what factor does the capacitance of a
sation of a dielectric is not completely recovered during
metal sphere increase if its volume is tripled ? the process of depolarization. Some energy is lost during
Solution. If and V2 are the initial and final volumes, the charging and discharging of the capacitor. This energy
then
\l/3 appears as heat.

V,1;
= (3)^^^ =1.44 Problem 56. n identical capacitors are joined in series
and the combination is given a potential difference V. If
or
q = i.44q these capacitors be disconnected and joined in parallel,
what potential difference will be obtained across the
Thus the capacitance increases 1.44 times. combination ?

Problem 50. How would you connect two capacitors Solution. In series combination, each capacitor will
across a battery, in series or parallel, so that they store have a potential difference V / n, which will remain same
greater (i) total charge and (lO total energy ? when capacitors are joined in parallel. Hence potential
Solution. Total charge. q=CV difference across the parallel combination will heV ! ti

Total energy, u = -cv^ Problem 57. n identical capacitors are joined in


2 parallel and the combination is given a potential dif
As V is constant and > q, so the capacitors must be ference y. If these capacitors be disconnected and joined
connected in parallel for storing greater charge and in series, what potential difference will be obtained
greater energy. across the combination ?

Problem 51. Why should a circuit containing capa Solution. In parallel combination, each capacitor will
citors be handled cautiously, even when there is no have a potential difference V. When the capacitors are dis
current ? connected and joined in series, the potential differences
Solution. Even if there is no current in the circuit, a on n capacitors get added. Hence the p.d. across the series
combination will be nV.
capacitor may have charge. When such a circuit is
touched, a discharge current is produced in the body and Problem 58. How can the whole charge of a-con-
so the man touching it may receive a severe shock. This ductor be transferred to another insulated conductor ?

can be avoided by wearing shoes with rubber soles. Solution. When the charged conductor is placed in
Problem 52. When moulded plastic parts are removed internal contact with the hollow insulated conductor, its
from metal dies, they develop a high voltage. Why ? whole charge is transferred to the hollow conductor.
ELECTROSTATIC POTENTIAL AND CAPACITANCE 2.73

Problem 59. Why is it that a man sitting in an Problem 63. Why is there loss of energy when two
insulated metal cage does not receive any shock when it charged conductors at different potentials are connected
is connected to a high voltage supply ? by a conducting wire ?
Solution. The charge in the cage goes to its surface. Solution. As tine charges flow in connecting wires.
The inside of the cage is equipotential. There is no some potential energy of the charges changes into heat
potential difference between the man and the cage. So the energy which is lost into the surroundings,
man does not receive any shock. Problem 64. During lightning, it is safest to sit inside
Problem 60. A large hollow metallic sphere A is a car rather than near a tree. Why ?
charged positively to a potential of 100 volt and a small Solution. The metallic body of the car provides an
sphere B to a potential of 50 volt. Now B is placed inside electrostatic shielding from lightning. When we stand
A and they are connected by a wire. In which direction near a tree, our body provides an easy path,
will the charge flow ? Problem 65. Is there some way of producing high
Solution. The charge will flow from B to A till no voltage on your body without getting a shock ?

w
charge is left on B. Inside A, B will acquire potential of A Solution. Yes. If we stand on an insulating surface and
and its own potential will become 150 V which is higher touch the live wire of a high power supply, a high
than that of A (100 W <V^ < Vg). Hence charge starts potential is developed on our body, without causing any

Flo
flowing from B to A. shock.
Problem 61. Why should the radius of the sphere of a Problem 66. Two metallic spheres of radii R and 2R
Van de Graaff generator be sufficiently large ?

ee
are charged so that both of these have same surface
Solution. Potential on a spherical shell is given by charge density a. If they are connected to each other with

Fr
V = 1 <7 a conducting wire, in which direction will the charge
4tC£
0
R flow and why ? [CBSE OD 16]
Solution. The potential inside and on the surface of
Larger the radius R of the spherical shell, the more charge for a
ur
it can collect without being raised to a high potential. charged metallic sphere of radius r is given by
1
Problem 62. Why is Van de Graaff generator enclosed V = —i = => Veer
ks
inside an earth connected steel tank filled with air under 4t:Sq r 4rcSp r
Yo

pressure ?
Hence, the bigger sphere will be at a higher potential
oo

Solution. It prevents the leakage of charge due to and the charge will flow from bigger sphere to smaller
ionisation. In air under pressure, as soon as free ions are sphere when the two spheres are connected by a
eB

formed, they recombine to form neutral air molecules. conducting wire.


r

Short Answer Conceptual Problems


ou
ad
Y

Problem 1. Figures 2.148(<?) and 2.148(b) show the (ill) Give the sign of the work done by the field in
field lines of a single positive and negative charge moving a small positive charge fromQ to P.
nd

respectively.
Re

[NCERT] iiv) Give the sign of the work done by an external


agency in moving a small negative charge from B
Fi

to A.

(u) Does the kinetic energy of a small negative


charge increase or decrease in going from B to A ?
Solution, (i) Potentials at botli the points Pand Qare
positive. Pis nearer to the source charge than Q. Also, we
know that the electrostatic potential at a point is inversely
proportional to the distance of the point from the charge
i.e. V eel/ r, therefore

Vp >Vq or Vp - Vq > 0
(i) Give the signs of the potential differences,
Vp -Vq ; Vg-V^. Potentials at both the points A and B are negative.
Hi) Give the sign of the potential energy difference Point B is farther from the charge than the point A. So,
of a small negative charge between the points O potential at B is less negative than at A.
and P; A and B. V.B >V A
2.74 PHYSICS-Xll

(ii) P.E. of two point charges, Problem 4. The electric field due to a point charge at
U =
a distance r depends according to inverse square law
4n £q r a:— . State how the following quantities depend
r )
The P.E. of a positive charge and a negative charge is upon r :
negative. The potential energy of a negative charge will be
(/) potential due to a point charge
more negative at Pthan at Q so
(li) potential at a distance r from the centre of a
(P.E.)q-(F.E.)p>0 charged metallic sphere of radius R(r < R).
The P.E. of two negative charges is positive. So the
Solution. (0 Potential due to a point charge,
P.E. of a negative charge will be more positive at A than at 1 ? ;
B, hence V = i.e., V X-.
4k n,0 r r

(P.E.)^-(RE.)g>0
(ii) In case of a charged metallic sphere,
(Hi) As Vp > Vq , if a small positive charge is moved by
an external agency from Q to P, the work done will be V:irrside = Vsurface —-— = constant

positive. The work done by the electric field in moving a 4k £q R


small positive charge from Qto Pwill be uegatwe. Potential V does not depend on r.
(iv) The negative charge will experience less repulsion Problem 5. Justify that the electrostatic potential is
at B than at A so the work done by the external agency in constant throughout the volume of a charged conductor
moving a negative charge from B to A is positive. and has the same value on its surface as inside it.
(i>) As the negative charge moves from B to A, it [CBSE Sample Paper 2011]
experiences more repulsion, its velocity decreases, and so Solution. Since electric field inside the conductor is
its kinetic energy decreases. zero and has no tangential component on the surface, no
Problem 2. Answer carefully : work is done in moving a small test charge within the
ii) A comb run through one's dry hair attracts small conductor or on its surface.
bits of paper. Why ? What happens if the hair is Tlnis means that, there is no potential difference
wet or if it is a rainy day ? between any two points inside or on the surface of the
(ii) Ordinary rubber is an insulator. But the special conductor. Hence the potential is constant throughout the
rubber tyres, of aircrafts are made slightly volume of the conductor and has the same value on its
conducting. Why is this necessary ? surface.

(m) Vehicles carrying inflammable materials usually Problem 6. Is it possible to create an electric field in
have metallic ropes touching the ground during which all the lines of force are parallel lines and whose
motion. Why ? density increases gradually in a direction perpendicular to
iiv) A bird perches on a bare high power line and the lines of force, as shown in Fig. 2.49(a) ?
nothing happens to the bird. A man standing on >■
A
>

the ground touches the same line and gets a fatal >■
shock. Why ? [NCERTl
>
Solution. (/) Refer answer to Problem 12 on page 1.62. >■

(ij) Refer answer to Problem 13 on page 1.62. *■

(Hi) Refer answer to Problem 14 on page 1.63. D

*■
(iv) Bird's whole body is at same potential. No charge
flows and no shock is produced. The man touching the Fig. 2.149 (a) (b)
ground maintains a potential difference between different
parts of his body. A large current flows, which electrolysis Solution. No. This is not possible because the work
blood and causes death. done in carrying a test charge along a closed path ABCDA,
as shown in Fig. 2.H9(b), will not be zero. More work is
Problem 3. Why electric potential in the field of a
negative charge is lower at near points and higher at done along CD, less along AB, zero along BC and DA. But
in an electric field, work done is essentially zero as it is a
distant points ?
conservative field.
Solution. For a negative charge the electric potential is
Problem 7. Figure 2.150 shows lines of constant
V = -
4Keg r
potential in a region in which electric field exists. The
values of the potential are indicated. Out of the points
For near points, ris less and V is more negative (lower). A, B and C, which will be of greatest electric field
For distant pointe, r is more and V is less negative (higher). strength ? Give reason. [irr 84]
ELECTROSTATIC POTENTIAL AND CAPACITANCE 2.75

Problem 10. A lest charge 'q' is moved without


acceleration from A to C along the path from A to B
and then from B to C in electric field E as shown in
Fig. 2.154. (j) Calculate the potential difference
between A and C. (if) At which point (of the two) is
the electric potential more and why ? [CBSE OD 12]

(2,3)
►E

Solution. Electric field is the rate of fall of potential i.e.,


dV (2,0)
E =- C
dr

w
For constant dV, Eccll dr. The stronger the field, the
Fig. 2.154
closer the equipotential surfaces. As the equipotential
surfaces are closed in the neighbourhood of B, so the field Solution. (0 P.D. does not depend on the path along

Flo
is greatest at B. which the test charge is moved. Therefore,
Problem 8. The equipotential surfaces of certain field dV
ZczZa-ViZa

ee
£=-
are shown in Fig. 2.151. It is given that > Vj. Draw the dx 2-6 4

Fr
corresponding lines of force for this pattern. Also state
the region in which the electric field intensity is highest.
or
V^-V^=AE^ V^>V^
i.
(i7) The direction of the electric field is in the
for
ur
direction of decreasing potential, so
Problem 11. Two uniformly large parallel thin
plates having charge densities +a and -a are kept in
ks
the X-Z plane at a distance 'ci' apart. Sketch an
Yo

equipotential surface due to electric field between


oo

the plates. If a particle of mass m and charge '-cf


eB

remains stationary between plates, what is the


Fig. 2.151 Fig. 2.152 magnitude and direction of this field ? [CBSE Dll]

Solution. Two parallel plates having charge


r

Solution. As shown in Fig. 2.152, the lines of force are


ou

densities + a and -a kept in the X-Z plane are shown in


ad

perpendiailar to the equipotential surfaces and directed


from higher potential to lower potential. The electric field Fig. 2.155. Also, a plane equipotential surface is shown.
Y

intensity is highest in the lower left region where the The field E acts in the -Y direction. The upward
equipotential surfaces are closest to each other.
electric force on charge -q balances its weight mg. For
nd
Re

Problem 9. Suggest a configuration of three point the stationary particle,


charges separated by finite distances that has zero mg
qE = mg or E=
Fi

electric potential energy.


Solution. The configuration of three charges - q, + q Y
and + q shown in Fig. 2.153 has zero potential energy.

Z r + CT
7
d i /*— Equipotential
surface
E

z i7
-►X

Fig. 2.155 Zjf


s 2.76

Problem 12. Find the P.E. associated with a charge


Y if it were present at the point P with respect to the
'set-up' of two charged spheres, arranged as shown in
PHYSICS-XII

(i7) Tlie capacitance becomes twice the original


capacitance because d becomes d /Z
Problem 14. The graph [Fig. 2.158(n)I shows the
Fig. 2.156. Here O is the mid-point of the line 0^02* variation of voltage V across the plates of two capacitors
(CBSE Sample Paper 13] A and B versus increase of charge Q stored on them.
Which of the capacitors has higher capacitance ? Give
I
reason for your answer. [CBSE D 04]

V
B
VB
1
I
I A
r
2(a + b) Of >P
I
Va
I

Fig. 2.158(a)
2

Solution. From Fig. 2.158(b),


Fig. 2.156
C ^ and C B
Q
Solution. = C\P = ^r^ + {2a + bf A “ ~
B

h ~ C^P= + (n + Ihf' But < Vg, therefore, > Cg


Potential at point P due to and Q2 is Thus capacitor A has a higher capacitance.
y = ^ Q I Problem 15. The given graph shows the variation of
4ji8 0
r, r2 ^
charge q versus potential difference V for two capacitors
P.E. associated with charge q at point P, C-y andCj. The two capacitors have same plate separation
Q Q2
^ “ 4^[[r^ + (2a + bff^ [r^ + (a + 4

Problem 13. A sheet of aluminium foil of negligible


thickness is placed between the plates of a capacitor,
as shown in Fig. 2.157. What effect has it on the capa
citance if (z) the foil is electrically insulated, and (zz) the
foil is connected to the upper plate with a conducting
wire ? [CBSE F 09]

Fig. 2.159

Foil
but the plate area ofCj is double than that ofCj. Which
of the lines in the graph correspond to and Cj and
why ? [CBSE OD 06, I4C]

Fig. 2.157 Solution. As C = -^ and graph A has a larger slope


than B, so the graph A represents a capacitor of larger
Solution, (z) The arrangement is equivalent to two capacitance.
capacitors connected in series. Each such capacitor has
plate separation d/2 and hence capacitance 2C. Total Also, i.e., C oc A
capacitance is d
2C x2C
= C
2C+ 2C As the plate area of is double of that of so C2 has
a larger capacitance. Hence the line A of the graph
i.e., the capacitance remains unaffected. corresponds to Cj and line B corresponds to Q.
ELECTROSTATIC POTENTIAL AND CAPACITANCE 2.77

Problem 16. As shown in Fig. 2.160, a dielectric of relative permittivity Kj and Sj. The permittivity of
material of dielectric constant k is inserted in half por free space is s^. [Roorkee 87]
tion between the plates of a
Solution. The given system is equivalent to two
parallel-plate capacitor. If its
initial capacitance is C, what
parallel-plate capacitors connected in series. Their
capacitances are
I

is the new capacitance ? 1 K

I _£j_eg_A and C2 - ^2 ^
Solution, The new
^2
arrangement is equivalent to
If C, is the equivalent capacitance of the series
two capacitors connected in
Fig. 2.160 combination, then
parallel.
J_ = J_ +J_ = 1 dj ^2 1 S2 + e, ^2
r_%An K Cn0 All
q q q a ^ ^2 S«0 A
d ' d

w
_ SgA
Sq ^ K Eq A or q =
q, = q + q = Id Id 4+q £2 + Ej ^2
£« A C e, £2

Flo
_ ‘'0
(K+1) = -=(K+1).
2d
Problem 19. Figure 2.162
Problem 17. Two identical capacitors of plate shows two capacitors

ee
dimensions / xb and plate separation d have dielectric joined in series, the rigid

Fr
slabs filled in between the space of the plates as shown central section of length b
in Figs. 2.161(a) and {b). Obtain the relation between the being movable. Prove that
dielectric constants k,k.j and K2. [CBSE OD 13C] the equivalent capacitance
of the combination
for is
ur
I I
independent of the position
of the central section.

if~
I I
Fig. 2.162
T 1
ks
d y d /
I £a0 A £n A
and q=-7“
Yo

Solution. Here q =
IL
K
111 K
1 ●^2
q
oo

I—//2—H //2H
(«) ib)
The equivalent capacitance of the series combination,
eB

Fig. 2.161 En A
Q qq_^ _ ‘'0

Solution. Capacitance of the capacitor shown in q +q ^ +q a - b


r
ou

Fig. 2.161(n) is dj ^2
ad

^ KEgA kEq(/ xb) Clearly, the equivalent capacitance of the series


Y

d d
combination does not depend on q and q i.e., it is
Capacitor shown in Fig. 2.161(b) is equivalent to a parallel independent of the position of the central section.
nd

Problem 20. Two charged spherical conductors of


Re

combination of two capacitors, each of plate area - xb


\2
radii Rj and Rj when connected by a conducting wire
Fi

and plate separation d. Their equivalent capacitance is acquire charges and q^ respectively. Find the ratio of
(I xb (\ xb their surface charge densities in terms of their radii.
K,Eq K2£q
u 2 ) (lC^ + K2)£Q(/xb) [CBSE D 14, 15C)
a d Id
Solution. The charges will flow between the two
As the two capacitors are identical, so spherical conductors till their potentials become equal.
q =q That is.
kc^ _ kq2 or ^=A
KSQ(/xb) _ (K^ +K2)£o(/xb) R, R2 ^2 q
d Id The ratio of the surface charge densities on the two
conductors will be
K =
1 <h

Problem 18. Find the capacitance of three parallel


plates, each of area A metre^ and separated by dj andd2 a
2 qi q q
metre. The in-between spaces are filled with dielectrics
2.78 PHYSICS-Xli

2 2
Problem 21. Two parallel plate capacitors of
capacitances and such that = 2Cj are connected {Hi} U = = -^ = 2 L/(j i.e., the stored energy is doubled.
across a battery of V volts as shown in Fig. 2.163. Initially Problem 23. Capacitors P, Q and R have each a
the key (K) is kept closed to fully charge the capacitors.
capacity C. A battery can charge the capacitor P to a
The key is now thrown open and a dielectric slab of
potential difference V. If after charging P, the battery is
dielectric constant k is inserted in the two capacitors to disconnected from it and the charged capacitor P is
completely fill the gap between the plates. Find the ratio connected in following separate instances to Q and R :
of (z) the net capacitance and {ii) the energies stored in
the combination, before and after the introduction of the {i) to Q in parallel, and
dielectric slab. [CBSE D 14C] (ii) to R in series, then what will be the potential
Solution. (0 Net capa
K
differences between the plates of P in the two
citance before the intro instances ?

duction of the dielectric slab. Solution. (0 When Pis connected to Qin parallel, the
C,^
GInitial = q + C2 capacitors Pand Q share charges equally.
= 2q + q = 3q P.D. between the plate of P = q/2^ q
C 2C 2
Net capacitance after intr
Fig. 2.163 (ii) Wlien Pis connected to R in series, charges on both
oduction of the dielectric slab,
CFinal capacitors remain same. The capacitor P with the same
= Kq + Kq = 2Kq + Kq = 3Kq charge will have the same potential difference V across it.
Ratio of the net capacitance, Problem 24. An uncharged capacitor is connected to a
^Initial _ =2 =1 -K battery. Show that half the energy supplied by the
cFinal 3Kq K battery is lost as heat while charging the capacitor.
{ii) Energy stored in the combination before the Solution. Let capacitance of the capacitor = C
introduction of dielectric slab, emf of the battery = V
uInitial _ <1^ .'.Charge given to the capacitor, q = CV
3q Energy supplied by the battery
Energy stored in the combination after the = Work done by the battery = qV
introduction of dielectric slab
Energy stored in the capacitor = ~ CV^
U _

Final
3Kq Energy lost as heat = qV -^qV =~qV
u
Ratio of energy stored
Initial _
- = k:1. Thus, half the energy supplied by the battery is lost as
' UFinal 1
heat while charging the capacitor.
Problem 22. A parallel plate capacitor is charged to a Problem 25. A capacitor is connected across a battery.
potential difference 'V' by a d.c. source. The capacitor is ii) Why does each plate receive a charge of exactly the
then disconnected from the source. If the distance same magnitude ? {ii) Is this true even if the plates are of
different sizes ?
between the plates is doubled, state with reason how the
following will change : (0 electric field between the Solution, (i) This is because of conservation of charge.
plates, {ii) capacitance and {Hi) energy stored in the If q^ and q-, are the charges taken by the two plates, then
capacitor. [CBSE D 01 ;ODll, 16] (ji, + must be zero because the charge on the battery is
Solution. Here simply redistributed and not created or destroyed.
U
{ii) Yes, the charge.s will be of equal magnitude even if
E„0 A ' 0 “
2C the two plates have different sizes.
0
Problem 26. Explain briefly the process of charging a
On disconnecting the battery, the charge q on the parallel plate capacitor when it is connected across a d.c.
capacitor plates remains unchanged because no transfer battery.
of charge occurs after the disconnection. If the distance d
is doubled, then A capacitor of capacitance C' is charged to 'V' volts
by a battery. After some time the battery is disconnected
(0 E= = q le., tile electric field remains unchanged. and the distance between the plates is doubled. Now a
0
slab of dielectric constant, 1 < k < 2, is introduced to fill

{ii) C =
1 the space between the plates. How will the following be
2d
- q, i.e., the capacitance is halved. affected ?
ELECTROSTATIC POTENTIAL AND CAPACITANCE

(fl) The electric field between the plates of the Energy of


capacitor
U, =-r(K
(b) The energy stored in the capacitor 2 2
Justify your answer by writing the necessary However, there will be a net increase in the energy of
expressions. [CBSE OD 15. 19] the combination because more charge has been drawn by
Solution. Electrons start flowing from one plate of the the capacitors from the battery.
capacitor to the positive terminal of the battery and from Problem 28. A parallel plate capacitor is charged by a
the negative terminal to the other plate. The two plates battery. After some time the battery is disconnected
start acquiring positive and negative charges respectively. and a dielectric slab with its thickness equal to the
Tliis process of charging of the capacitor continues until plate separation is inserted between the plates. What
the potential across its plates becomes equal to the p.d. change, in any will take place in (i) charge on the plates
across the terminals of the battery.
(ii) electric field intensity between the plates {Hi) the
Effect of dielectric. When the battery is disconnected, capacitance of the capacitor, iiv) potential difference

w
charges on the capacitor plates remain unchanged, between the plates and (u) the energy stored in the
(f) Initial capacitance, Q = capacitor ? Justify your answer in each case.
ICBSE D 07,10; OD 09]

Flo
CT

Initial field, ^= Solution, (i) The charge </q on the capacitor plates
^0
remains the same because the battery has been

ee
Final field. disconnected, before placing the dielectric slab.
K E K

Fr
0
{ii) The surface charges induced on the dielectric slab
Thus, the electric field decreases. reduce electric field intensity to a new value given by

(ii) Final capacitance, C = k for


ur
2d 2

Initial energy, = (Hi) The reduction in the electric field reduces the
2Q
ks
potential difference
Yo

V = Ed = M-Yo
oo

Final energy. (J =
2':
2C 2'kCq k ° K

(lu) Due to the decrease in p.d., the capacitance


eB

As 1 < K < 2, the energy stored increases i.e., Ii > (J^,. increases k times
Problem 27. Two identical capacitors Cj and C2 are
P_ %^ % %
connected to a battery B, as shown in Fig. 2.164. A
- K
= xQ)
r

V V,/K 0
dielectric slab is slipped between the plates of Cy the
ou
ad

battery remaining connected. What happens to the (u) Energy stored decreases by a factor of k
Y

\2
charge, the capacitance, the potential difference and 1 u.0
U=-CV'^
stored energy of each capacitor ? 2 K 2
nd

Solution. On introducing ,
Re

the dielectric slab between the Problem 29. A parallel plate capacitor, each with
plates of capacitor q, its “T plate area A and separation d, is charged to a potential
Fi

capacitance increases, hence difference V. The battery used to charge it remains


the capacitance of the whole ~z~B connected. A dielectric slab of thickness d and dielectric
combination increases. As the ► constant k is now placed between the plates. What
battery remains connected, the I .k [_^ Cj change, if any, will take place in : (i) charge on plates
p.d. across the combination electric field intensity between the plates (Hi) capa-
remains constant, and so the citance of the capacitor ?
charge on the combination Fig. 2.164 , ● u r/Ticnnim
Justify your answer in each case. [CBSE D10)
will increase. As C, and q are ●*
in series, charges on them will be equal. As charge on Cj Solution. As the battery remains connected, the
has increased, its capacitance is same and so p.d. across C| potential difference V remains constant even after the
will increase. But ^1 + ^2= constant, so the potential introduction of the dielectric medium,
difference across q will decrease. (j) The capacitance increases from q to C
1 7
Energy of q, q = - Q q will increase. c= Kq
● - 2.80 PHYSiCS-XII

to a
iii) Charge on the capacitor plates increases from Q, Solution. As the battery remains connected, the
potential difference V remains unchanged,
Q=cv^Kq,v = KQ^ e A
(0 Original capacitance, Q = -2_
d
(Hi) As the potential difference V remains unchanged,
the electric field also remains unchanged. New capacitance.
C = kSqA lOxe^A 10
d' ~ 3d "3 O’
Problem 30. A parallel plate capacitor of capacitance
C is charged to a potential V by a battery. Without
(,V)Q = cy = |q,i/ = SQ 0 ●

disconnecting the battery, the distance between the (m) New field, £ = — = — = .^
plates is tripled and a dielectric medium of k=10 is d' 3d 3

introduced between the plates of the capacitor. Explain Original energy density, u0 = -e

giving reasons, how will the following be affected : 2 0 0


(z) capacitance of the capacitor New energy density,
n2
(ii) charge on the capacitor, and u = -eE^
1
10 1 ^2 10
= -Kg
(Hi) energy density of the capacitor. 2 2 ®I 3 g 9
0 ●

[CBSE OD 17C]

Problems on Higher Order Thinking Skills


Problem 1. The electric field
++++++++++++
a b distribution. From the points marked A, B and C, identify
inside a parallel plate capacitor is E. the point at which the electric field is :
Find the amount of work done in (/) zero (ii) maximum.
moving a charge q over a closed d

rectangular loop abed a. Explain your answer in each case. [CBSE SP 03]
[CBSE D 14] Solution. At any point, we have
Fig. 2.165
Solution. E± ab and El. dc, dV
E=-
= Negative slope of V-x graph
so
W,l,=0 and W^=0. dx
dV
Also,W,^=-W,^ At point A, = 0
dx

Total work done in moving charge q over the closed dV


loop abeda. At point B, <0
dx
dn dV
At point C, >0
= o-w,„+o+wda = 0. dx

Problem 2. Figure 2.166 shows the variation of Therefore, (i) £ is zero at point A
electrostatic potential V with distance 'x'for a given charge (ii) E is maximum at point R
Problem 3. Two identical plane metallic surfaces A and
Bare kept parallel to each other in air, separated by a distance
A B ofl cm, as shown in Fig. 2.167.
V A B
C

1 cm T
y- 2
X

Fig. 2.166
Fig. 2.167
ELECTROSTATIC POTENTIAL AND CAPACITANCE 2.81

Surface A is given a positive potential of 10 V, and the Problem 5. The two graphs drawn here, show the
outer surface of B is earthed. variation of electrostatic potential (V) with l/r (r being
(i) What is the magnitude and direction of the uniform distance of the ifeld point from the point charge) for two
electric field between points Y and Z ? point charges i/j and q^.
(ii) What is the work done in moving a charge of 20 fiC (0 What are the signs of the two charges ?
from point X to point Y. [CBSE SP 08] (»■) Which of the two charges has a larger magnitude and
dV 10 V why ? [CBSE SP 08]
Solution, (i) £ = -
dr 1 cm

10 V
= -1000 Vm~\
10“^ m

Magnitude of the uniform electric field between X

w
and y =1000 Vm~\
The direction of the electric field is from plate A to
plate R

Flo
(if) Zero. This is because the points X and Y are at
the same potential.

ee
Problem 4. The electric potential V as a function of Solution, (i) Charge is -ve while charge q2 is +ve.
(ii) As V = J—,1
1

Fr
distance X is shown in Fig. 2.168. Draw a graph of the electric
field strength Eas a function ofx. [CBSE OD 19 ; SP 23] 4tiSq r 4x60 r
V
.'. Slope of V vs. - graph = —-
for
ur
r 4xe
0

As the graph for q^ has a slope of larger magnitude


than that for so q^ has a larger magnitude than ^2-
ks
Yo

Problem 6. The following data loas obtained for the


oo

0 1 2 3 X

dependence of the magnilude of electric field, with distance,


Fig. 2.168
from a reference point O, zvithin the charge distribution in
eB

Solution. We know that the shaded region.


dV Field Point A B C A' B' C
r

£ = - = Negative slope of V-x graph


dx
ou

E E E E E
ad

Magnitude of £
For 0 < a: < 1, electric field 8 27 2 16 64
Y

dV
= +ve constant, so field = - E
dx (0 Identify the charge distribution a?id justify your
nd

answer.
Re

For 1< AT <2,


dV in) If the potential due to this charge distribution, has
= 0, so field =0
Fi

dx a value V at the point A, what is its value at the


point A' ? [CBSE SP 08]
For 2 <a: <3,
dV
= -ve constant, so field = + E
dx C
t
Consequently, we get the E-x graph as shown below:

+ E

A-
O
1 2 3
..I.
X

O A B C
-E I

Fig. 2.171
Fig. 2.169 f-x graph.
s 2.82

Solution. (0 From the given data, we observe that


(fl) The magnitude of electric field varies inversely as
the cube of the distance of the field point along the
PHYSICS-XII

(ii) When the consecutive charges have opposite


signs, potential at .r =0 is

horizontal line OC V = -^
4ns 1 2 4 8 "'
(b) The magnitudes of E at points A', B' and C on the 0 >-

vertical line OC are just half the magnitudes at the corres ,111
1 — + + ...
ponding equidistant points A, B and C. 47ts
0 >-
2 4 8
The above properties indicate that given charge distri 1
bution is a short electric dipole centred at the reference
4718 6718
point O. 0 1 0
2]
(/;■) As tlae point A' lies on the equatorial line of the
dipole, so the electric potential at point A' must be zero. Problem 9. A charge Q is distributed oi>er two
Problem 7. Draw 3 equipolcntiiil surfaces corres concentric hollow spheres of radii r and R, where R> r, such
ponding to a field that umforinly iiurenses hi magnitude but that the surface charge densities are equal. Find the potential
at the common centre. [IIT 81; CBSE OD 19]
remains constant along Z-direction. How are these surfaces
different from that of a constant electric field along Solution. Let and be the charges distributed
Z-direction ? ICBSE OD 09, D 19]
over the smaller and the larger spheres, respectively.
Solution. The equipotential surfaces corres Then
ponding to an electric field that uniformly increases in
magnitude but remains in a constant Z-direction are The surface charge densities will be
planes parallel to XY-plane shown in Fig. 2.172. But as Q Qr
field increases, such planes get closer. a =
_

4kR^ 4nr^
AZ

r or 9^-Jl or
Qr Qr _ ^
UJ QR QR R^

d2<d 1 z Equipotentials Q _r^ + R^ R^


or
R^ or Q^ = Q
QR

iz
J- -►y
Q
Similarly, =

V
Potential due to the charge on tlie smaller sphere is
Fig. 2.172
Q.r
Q =
In case of a constant electric field along Z-direction, 47tE«0 r 4nSQ r 47I8q(R^ + r^)
the equipotential surfaces are equidistant planes parallel
to XY-planes, as shown in Fig. 2.211 on page 2.102. Potential due to the charge on larger sphere is
Problem 8. An infinite number of charges, each equal to R^
QR 1 Q.R
q, are placed along the x-axis at x == I, x =2, x = 4.... and so ^R = 4ne„ Q =
on. (/) Find the potential at the point .y =Qdiie to this set of 0
R 47T8q R ^^ 4nsQ{R^ + i^)
charges, (ii) What zuill be the potential if in the above set up
the consecutive charges have opposite signs ? HIT] Total potential at the centre.
Solution, (i) The potential at y = 0 due to given set of Q(r+R)
charges is 47I8o(R^-H7^)'
V=^ ^ ^ q q Problem 10. Three concentric metallic shells A, B and C
4ns 1 2 4 8
0 L of radii, a, b and c(a <b <c) have surface charge densities
7 ,111 1 -r CT, - CT and + c respectively, as shown in Fig. 2.173.
1 + — + — + --I-... ^_s.
4ns
0 ■-
2 4 8
47T8/1-1 2ne
0
(i) Find potential of three shells A, B, C.
(ii) If shells A and C are at the same potential, obtain the
Sum of an infinite G.P. = — relation between radii a, b, c.
1-r [CBSE OD 19 ; IIT 90]
ELECTROSTATIC POTENTIAL AND CAPACITANCE

f -2
(j a a
(a-b + c) = — + c
8
0 ^0 V ^ c
{a-b){a + b)
or n-b-
c

+ a
or a+b = c.

Problem 11. In Fig. 2.174, the potentials at points A


ntid Bare and Vg respectively. Calculate - Vgfor the
given arrangement.
A B 4
+ H-

+ y AT H

Solution, (i) Charges on the three shells are

w
Fig. 2.174
q^=4:Tia^a, qg=-4itb^a, f]'^=4rcc^a
Solution. The net potential at A due to the two
The potential at every point inside a spherical shell
is constant and is equal to that on its surface. charges is

Flo
1
Potential of shell A. Any point on the surface of
shell A lies inside the shells B and C. oL^ x + i/J 47ceQ x(x + y)

ee
1 The net potential at B due to the two charges is

Fr
.^4 =
4n£Q\_ a b c 1 1
B
4 71 8
0 L X + y X 4tc 8
0 x{x + y)
1 4Tta^(j 4nb^G 4n(^a
for
ur
4718 0 a b c 2qy
V
A -Vg® = —
4ti 8 0 X {x + y)
o
or
V^=-{a-b+c).
ks
8
0 Problem 12. Figure 2.175 shows some equipotential
Yo

surfaces. What can you say about the magnitude and


oo

Potential of shell B. Any point on shell B lies direction of the electric ifeld ?
outside the shell A and inside the shell C. y(cm)
eB

10 V 20 V 30 V 40 V
1
Ha + ^ + Hc.
4tI8 b b c
0
r

30° 30° 30° 30°


0
ou

47tfl^a 47tl7^a 47rc^a ^40 X (cm)


ad

4ti:8 b b c
Y

2
a a
Fig. 2.175
nd

or
V3 = b+C .
Re

^oL b
Solution. As shown in Fig. 2.176, consider two
Fi

Potential of shell C. Any point on shell C lies consecutive equipotential surfaces. Electric field is normal
outside the shells A and B. to the equipotential surfaces and always directed from
higher potential to lower potential.
Vc^ = —
4718 oL c c C J

1 l^na^a 4nb^ g +
Atzc^g
4716 0 C C C

G b^
or + C Electnc
c
\ line of force

{ii) If the shells A and C are at the same potential,


then
Fig. 2.176
2.84 PHYSICS-XII

The normal distance between two consecutive Solution. As the electric field is a conservative field,
equipotential surfaces is so the work done depends only on the initial and final
dr = AB = 10 cm x cos 60° = 5 cm points.
Y
AB
= cos 60°
10 cm

Also frK = 10-20=-10 V


P

£ = - dV___ -lOV = 200 Vm"V S


dr 5x10" 2 m
X

R
Angle made by £ with positive X-axis is
0 = 180-60° = 120°. Fig. 2.177

Problem 13. The electric potential V at am/ point x, y, z


Displacement of charge q,
(nil in metres) in space is given byV = volts. Calculate
the electric field at the point (1 m,0,2 »/). [AIPMT 11] ^S = (0,0,0)-(fl, b,0)
Solution. Here V = 4x^
dV dV dV = (0-a)i +(0-b)j +{0-0)^ =-af J
dx = -8x,E^^- = 0,£,=- 8z
=0
Force on charge q,
A

-8a: I p ^qE=qEi

At point (1 m, 0, 2 m), W = F . PS = qE i .{-ai -bj ) = ~ qEa.


£=-8x1/ =-8?Vm"^. Problem 16. Tuio point charges -Q and +Q/V3 are
Problem 14. There are tzoo particles each of mass m and xy-plane at the origin (0,0) and a point (2,0),
carrying a charge Q. Initially one of them is at rest on a respectively, as shown in the Fig. 2.178(a). This results in an
smooth-horizontal plane and the other is projected from a equipiotential circle of radius R and potential V^=0 in the
long distance along the plane, directly towards the first xy-plane with its centre at (b,0). All lengths are measured in
particle with a velocity v. Find the closest distance of ^^tctrcs. Find the values of R and b. [JEE Main Adv. 21]
approach.

Solution. At the distance of closest approach ^r^


both particles acquire a common velocity Vq. By conser
vation of linear momentum, ●►ar

0 + mv= uWq + mv 0 or
Vq=v!2
By conservation of energy,
1 7 1 Q-Q .1 2 1 2
0 + — mv + - mvn + — mv, 0
2 4ti 8
0 ^0 2 0 2 '
/-.^2
Fig. 2.178(g)
1-*● 2 V
or - mv = + m
2 4718
0 ^0 u Solution. Take a point P{x,y) on the circle, as

shown in Fig. 2.178(1;). Then


or
1
— mv
2 1
4 4t: 8 H-Q) . kQlfB
0 'o
r.

or 1 4Q"
^0 =
4;r eg ’ mv^
Problem 15. A point charge q movesfrom a point P to a
''2

point S along the path PQRS in a uniform electric field ? -Q


■►a:

acting along the positive direction of X-axis. The coordinates


of the points P,Q R and S are (a, b, 0), {2a, 0,0), {a, - b, 0)
dtid (0, 0, 0) respectively. Find the work done by the field in

the process. [HT 92]


Fig. 2.178(1;)
ELECTROSTATIC POTENTIAL AND CAPACITANCE 2.85

kQ kQ
Problem 19. Capacitance of an isolated conducting
^/3^(x-2)2+y2 sphere of radius Rj becomes n times when it is enclosed bi/ a
concentric coiducting sphere of radius R~y connected to
3(j:^ -4^: + 4 + y^) =
earth. Find the ratio of their radii.
=>
2x^-12x + 12+2y^ = 0 R
1 J IJEE Main July 22]
x^-6x + 6 + y^ - 0
Solution. Capacitance of the isolated sphere.
(x^-6x + 9)-9 + 6 + y^ = 0
(x-3)'+y2=(V3)2 Cl = 47te(,Ri
Capacitance of the spherical capacitor so formed,
This is the equation of a circle with centre (3,0) and
radius fs. C _4Tt£„RiR^
R=VSm ' «2-«l
As centre {b, 0) is (3, 0), so b = 3 m.

w
q“ R^
Problem 17. Two electric dipoles of moments pj and p2 1 R
1
are in a straight line. Show that the potential energy of each

Flo
n n
1 Pi P2
in the presence of the other is - , tvhere r is the
2%z 0 r^ h- n

ee
R n-1
distance between the dipoles. (Assume r to be much greater 1

than the length of the dipole).

Fr
Problem 20. Assuming an expression for the potential
Pi P2
of an isolated conductor, show that the capacitance of such a
0---;----© 0---^---© sphere will be increased by a factor n if it is enclosed zuithin
for
ur
an earthed coiia'jjfr/c sphere, the ratio of the radii of the
spheres being n/ln-l).
Fig. 2.179
Solution. The capacitance of an isolated conduc
ks
Solution. Electric field due to the dipole of moment ting sphere of radius a is
Yo

Pi at the other dipole is C = 47ceQ a


oo

E = 1 _2pi When surrounded by an earthed sphere of radius b,


eB

471 0
r^ its capacitance becomes
ab
C = An&r,.
The potential energy of the other dipole of moment 0
b-a
r

P2 in the electric field E is


ou

C ab b 1 1
ad

1 1 P1P2 = n.
U = -Pi E = ~P2 ■

C a{b-a) b-a
47t8n0
Y

0
b n

Problem 18. Tzuo identical thin metal plates are given Problem 21. N spherical droplets, each of radius r, have
charges t/j and 1/2 {< t/j) respectwely. if they are noio brought
nd
Re

been charged to have a potential V each. If all these droplets


close together to form a parallel plate capacitor with zvere to coalesce to form a single large drop, what zoould be
capacitance C, what will be the potential difference between
Fi

the potential of this large drop ?


the plates ? [IIT 99 ; JEE Main July 22]
(It is gii’cn that the capacitance of a sphere of radius x
Solution. Let A be area of each plate. When the two
equals 4 TiEg/c.rJ. [CBSE SP 13]
plates are placed d distance apart, the capacitance of
the parallel plate capacitor so formed is Solution. Potential of each droplet = V
£„0 A
C = Capacitance of each droplet, C = 4nsQkr
d
‘h~l2 ^l“^2 Charge on each droplet, q = CV = insQkrV
E = ^h-5i 2 2 Total (initial) charge on all droplets,
2e.A
0 +
q' = Nq = 4neQkr NV
+
d
V=Ed = Si ^ + If R is the radius of the large drop, then
2 e.A
0
+
-tcR^ = Nx~Kr^
+ 3 3
_ ^^
2C R =
2.86 PHYSICS-XII

Capacitance of the large drop, Percentage change in the stored energy,


U'-U U' no ^ 100
C = 4tu£q/cR = xl00= —-1 xl00 = -1 xl00 =
U U I 9 9
Potential of the large drop,
= 11.11%
*?' 4Ti£/rNl/ = N
2/3
V
C 47ie.fcN^^^r Problem 24. Tivo identical parallel plate capacitors A and
0
Bare connected to a battery ofV volts ivith the switch Sclosed.
Problem 22. (/) Two circular metal plates, each of The switch is now opened and the free space between the plates
radius 10 cm, are kept parallel to each other at a distance of 1 of the capacitors is filled S
mm. What kind of capacitor do they make ? Mention one with a dielectric of dielectric o-

application of this capacitor. constant k. Find the ratio of


A B
(ii) If the radius of each of the plates is increased by a electrostatic energy
factor of and their distance of separation decreased to half stored in both capacitors
of its initial value, calculate the ratio of the capacitance in the before and after the intro-
two cases. duction of the dielectric. Fig. 2.180
[CBSE OD 171
(Hi) Suggest any one possible method by which the
capacitance in the second case be increased by n times. Solution. When the switch S is closed,
[CBSE SP 03] P.D. across A = P.D. across B=V

Solution, (i) The two plates form a parallel plate Initial energy stored in both the capacitors is
capacitor.
Application. Alongwith an inductor, a capacitor is u. = + lig = I+ - cv^ = cv^ 2
used in an oscillatory circuit. Wlten the dielectric is inserted, the capacitance of
(if) Original capacitance, C = d ^ 0

d
each capacitor becomes KCThe p.d. across A is still V
With switch Sopen, the p.d. on B attains a new value V
New capacitance, but charge {q = CV) does not change.

C =
£„ X 71
0 {r-Jlf 4 Zr.0 X
= 4C q = CV = KCxV or V = —
K
dll d

^=i= The final total energy stored in both capacitors is


C 4

(iff) The capacitance of a capacitor can be increased 1 9 1 1 1+K 2\


n times by any of the following methods : = -.kCx V^ + -.kCx CV^
2 2 V Ky 2 K

(fl) By inserting a dielectric of dielectric constant n


u,t CV^ 2k
between the capacitor plates. Then C' = nC. _

u 1 1 +K
2^
l+K^- .
(b) By decreasing the distance d between the / CV^
2
plates by a factor n, because C oc 1/d. K

(c) By increasing the area of the plates n times, Problem 25. A parallel-plate capacitor contains one
because C qc A
mica sheet of thickness dj = U) x 10”^ m and one fibre sheet
(d) By increasing the area of the plates ^fn times of thickness d^ =0.5 x 10“^ m The dielectric constants of
and simultaneously decreasing the distance mica and fibre are 8 and 2.5 respectively. Fibre breaks down
between the plates by a factor of in an electric ifeld of 6.4: x 10 ^ Vm~^. What maximum
Problem 23. Keeping the voltage of the charging source ‘voltage can be applied in the capacitor ?
constant, what would be the percentage change in the energy Solution. Let a be the surface density on the
stored in a parallel plate capacitor if the separation between capacitor plates. Electric fields in mica (Kj) of thickness
its plates were to be decreased by 10% ? [CBSE SP 08] dj and fibre (K2) of thickness d2 will be
CV^ = 1^
a
1
Solution. U -
2 2 d
i.e., L/ocid ●^1 ^0
and £2 =
K2 £q

When the separation between the plates is decreased


K

Ei = K 2 £
by 10%, the energy stored becomes U' such that, 1

U' _ d _ 100 or (i'=—U


The maximum permissible value for £2 is
U ~ d'~ 90 9 6.4x10^ Vm"^
ELECTROSTATIC POTENTIAL AND CAPACITANCE 2,87

Maximum permissible value for The equivalent capacitance of the series


2.5
X 6.4x10"^ =2.0x10^ Vm"^ combination of Cj and is
8
K1K2
C
Maximum voltage that can be applied to capacitor is eq
C1+C2 d +
V=E^d^+ E^d^ =2.0 X 10^ x 1.0 x 10”^
+ 6.4x10^x0.5x10"^ Now, capacitor C3 can be regarded as the series
= 2.0 X 10^ + 3.2 X 10^ = 5200 V. combination of capacitors and C2, each of plate area
A and separation d. Again,
Problem 26. The msulated plates of a parallel plate
capacitor have a charge density g. Shoiv that the work done KiEqA
C1 =
d ■
and C2 =
in changing the distance from d^ to dj is
C C
l/=- _ '-0

w
II q + c' d [K^+Ki)
Solution. The capacitances at the two separations Hence, = C3.
are

Flo
K e,,0 A K A
C,1 = and C2 =
0 Problem 28. Three identical parallel plate (air)
d
1 ^2 capacitors €^,€2X2 capacitartces C each. Tl\e space

ee
between their plates is no70 iflled ivith dielectrics as shown. If
The energies stored in the capacitor at the two
all the three capacitors, still have equal capacitances, obtain

Fr
separations are
the relation between the dielectric constants k.,,K2, K3
1 1
[CBSE SP 2011]
and U2 = and K^.
2 C1 2 C2
for
ur
q
Work done

1 1 >^3
ks
= ^2
2 C2 C1
Yo
K K
1 ^2
oo

K4
eB

2 K e,.0 A
d —●\ d
A rf/2 dH
id2-d^). q = G A]
2 K e
r

0
Fig. 2.182
ou
ad

Problem 27. Ihe capacitor-^ C[,C2. luur


lulution. New capacitance of
Y

area Aeach, are connected in series, as shown. Coinpiu.


capiacitance of this combination with the capacitor Cy again ke«A
0

having plates of area A each, but 'made up' as shown in


nd

d
Re

Fig. 2.181. [CBSE SP 13]


New capacitance of C2
Fi

I = Series combination of two capacitors

EqAT 2kjK2
<2 K K,
d [k, + kJ
K
1 1

New capacitance of C3
d d 2d
= Parallel combination of two capacitors
c, q q
^SpAr Kg + K^
d [ 2
Fig. 2.181
As q “ ^2 ” *“3'
Solution. Clearly,
k-BoA 2KjK2 _ K3 + K4
- KjBpA K =
and C2=-^-^
d + K2 2
2.88 PHYSICS-XII

Problem 29. Between the plates of a parallel-plate 9A


capacitor of area A, a copper plate sits on a dielectric slab of AH AH
k:=2, as siwiun in Fig. 2.183. Find the equivalent capa "2 Q d/3
citance of this arrangement.
K
1

2dH

C3 d/3
Cu C2 d/3

k =2 Cl d/3 Fig. 2.184

The equivalent capacitance of Cj and C3 connected


Fig. 2.183 in series is given by

Solution. The given arrangement is equivalent to c' ■ C/ C3


three parallel plate capacitors connected in series.
But K2 8q -^/2 _3 Kj Eq A
Capacitance of first capacitor, (k =2)
rf/3 2d
2 Bq A _ 6 A
~dl3 T~ and K3 £q A/2 _ 3 K3 £q a
^3 =
2 fr/3 4rf
Capacitance of second capacitor,
1 Id 4d
K 8g A _ 3 K 0 A
^2 = c
3 K2 £q A 3 K3 Eq A
~dI3 d

where k = co for copper metal. 2d


2- + —'
3 £«0 A
Capacitance of third capacitor,
K K
K‘^2

£3 A 3 Ef, A 3 £q A K2 K3
C3= 7/3
0 T~
or C' =
2d (K3 +2 ^2)
Equivalent capacitance of the series combination is
The equivalent capacitance between A and B is
given by given by
i = J- _L 3 £q A K2 K3 En0 K.1 A
C C1 ^2 ^ ^3 c = c'+q =
2d (K3 + 2 K2) 2d
d d d
+
_ Eq A 3 K2 K3
+
6 £n A 3 K En0 A 3 £n0 A + K
1
2d
K3+2K2
d d
+ 0 + [k = 00 for Cu]
6 £„0 A 3 £n0 A or c = Eq a 3 <2 K3 + K3 Kj +2 K2 Kj
2d
2e„A
K3+2K2
0
or C =
-12 -4
d 8.85 X 10 xlxlO

2 X 2 X 10”^
Problem 30. A parallel plate capacitor consists of th ree
dielectrics as shown in Fig. 2.184. It has plate area A = 1 aii^ 3x6x2+2x4+2x6x4
and plate separation d =2 mm //"Kj =4, K2 =6andK^ =2,
X

2+2x6
find the capacitance of this arrangement between points A
and B. 8.85x10’^^ X
92
4 14
Solution. This arrangement consists of capacitor
= 14.54 X 10"’^ F
connected in parallel with the series arrangement of C,
and C3. = 1.454 pF.
ELECTROSTATIC POTENTIAL AND CAPACITANCE

Problem 31. What is the capacitance of arrangement of Equivalent capacitance,


4 plates of area A at distance d in air in Fig. 2.185 ? 3e„A
0
ICPMT 891 c^ = Ci4-q + C3=3C = d

d Problem 33. A capacitor is made of a flat plate of area A


and a second pdate haznng a stair-like structure, as shown in
P
Q
Fig. 2.189. The loidth of each stair is a and the height is b.
Find the capacitance of the assembly.
a

Fig. 2.185 b

a
Solution. As shown in Fig. 2.186, suppose the point b
P is connected to the positive terminal and point Q to a
d
the negative terminal of a battery. Clearly, we have

w
two capacitors I and II with their positive plates 3fl

connected together and their negative plates connected


Fig. 2.189
together. So the two capacitors are in parallel.

Flo
+ +
! 1 + + + Solution. As shown in Fig. 2.190, the given arrange
ment is equivalent to a parallel combination of three

ee
p
capacitors of capacitances o

Fr
+ + + ^^ + + +
Cj, C2 and C3.
Here
Fig. 2.186
for
ur
d*2b
d*b
Equivalent capacitance, d

2e.A Eq A/3
Cp = Ci + C2=C+C=2C =
0
ks
d d + b '
Yo

Problem 32. What is the capacitance of arrangement of


oo

6
gp A/3
4 plates of area A at distance d in air in Fig. 2.187 ? d+2b Fig. 2.190
eB

[DPMI 87]
fin ^ —1
C=C+C + C,=-2- + _l 1

d 1 2 3 3 d d + h- d + 2b
r
ou

P
ad

_ jd + b){d-i-2b) + d{d + 2b) + d{d + b)


Q 3 d{d + b){d + 2b)
Y

A (3d^ + 6bd + 2b^)


nd

Fig. 2.187
Re

3d(d+b){d+2b)

Solution. As shown in Fig. 2.188, suppose the point Problem 34. Four point charges Q, q, Q and q are
Fi

P is connected to the positive terminal and point Q to placed at the corners of a square of side 'a' ns shozon in
the negative terminal of a battery. Clearly, we have Fig. 2.191.
three capacitors /, II and III. Their positive plates are
connected to the same point P while the negative Q
plates are connected to the same point Q. So the three
capacitors are in parallel.

+ + + + + + + +
I

P a
Q
+ + + + // + + + +

+ + + +
m + + + +
Q Fig. 2.191

Find the (a) resultant electric force on a charge Q, and


Fig. 2.188 (b) potential energy of this system. (CBSE 181
2.90 PHYSICS-XIl

■ Solution, (fl) Let us find the force on charge Q (&) Find out the nwount of the zuork done to separate the
located at point C. charges at inifnite distance. [CBSE18]
Q a
Solution, (fl) Force on charge q due to the charge -4q,
A >D

1 V
, along AB
a a
4tc£q f
Force on charge q due to the charge 2q,
C 1 2q^
a
Q ^2 = 47ISn f , along CA
Fi 0

F23
F3
Fig. 2.192

Force due to charge Q at point A,

F = 1 Q\ along AC
^ 4%Zq {flaf 4tc£q 2a^'
Force due to charge q at point B,

^2 “ ^
47t£ a
along BC The forces Fj and F2 are inclined to each other at an
0
angle of 120°.
Force due to charge q at point D, Hence, the resultant force on charge q is
^
^3 = 4716 a
along DC F + F^+2 FjF^cosl20°
0

Resultant of two perpendicular forces F2 and F^ is 2\2 r n2

1
1 4q +
2^' + 2. 4q^ 2q^f 1^
F
23 "
47X6 0
4716 0 2)

1 -j2qQ . _ 1
2
1 l^q^
47X6 0
.—
a
along AC .-^Vl6 + 4-8 = 47X6
47X6„ I
0 0
.2

Hence, the net force on the charge Q located at


point C, (B) Net potential energy of the system,
1 Q —+
Q 1 \qi-4q) _ {-4q){2q) qx2q
F - Fj + F23 - f2 q , along AC. U =
47I6q a^i2 47X6 0>- } I I

(b) Potential energy of the system


.-^[-4-8 + 2]=-^.-^
1
1 IQq ^ qQ ^ Qq ^ Qq ^ qq ^ QQ 47X6 0 I 4tX6 }
47X6oL^F BC CD AD BD AC
0

Work done in separating the charges to infinity


1
4iQ^4! Q' q'
4tx8
0
-J2a
a fZa 47xe„fl
0
^/2 f2 = ~U = 10 q"^ ^ 5q^
47X6,,0 / 27X6„0 I
Problem 35.
Problem 36. Two point charges q and -q are located at
(a) Three point charges q, -4q and 2q are placed at the (0,0,-fl) and (0,0, fl) respectively,
vertices of an equilateral triangle ABC of side T as (0 Find the electrostatic potential at (0,0,2) and (7x,y,0).
shown in Fig. 2.193. Obtain
(ii) Hoio much work is done in moving a small test charge
the expression for the
magnitude of the resultant from the point (5,0,0) to (-7,0,0) along the X-axis ?
electric force acting on the (in) How woidd your ansioer change if the path of the test
charge q. charge between the same points is not along the x-axis
Fig. 2.193 I I but along any other random path ?
ELECTROSTATIC POTENTIAL AND CAPACITANCE 2.91

(/i») If the above point charges are now placed in the same (Hi) No, the work done will not change. This is V
because electric field is a conservative field. Work done
positions in a uniform external electric field E, what
,,, , ,, .... . . against this field is path independent.
would be the potential energy of the charge system in its
orientation of unstable equilibrium ? Justify your (/f) The dipole will be in unstable equilibrium if hs
answer ill each case. ICBSE18C] dipole moment p is antiparallel to external field £.
Solution. (0 Potential at point P{0,0,2) due to Then its potential energy will be
charge +q{0,0,-a) is U^+pE
1 'I 1

4716q z~{-a) 4tt8q z + a Problem 37. (n) An electric dipole is kept first to the left
and then to the right of a negatively charged infinite plane
kZ sheet having a uniform surface charge density. The arrows
P{0, 0, 2)
and P2 show the directions of its electric dipole moment in

w
the two cases.

-q(0, 0, n)

Flo
> Y

Pi P2
+tj(0, 0, -fl)

ee
X

Fr
Fig. 2.195

Potential at point P(0,0,z)due to charge -g(0,0,fl)is


for
ur
1 -q Fig. 2.196(a)
47ie„0 z-a
Identify for each case, lohether the dipole is in stable or
ks
Total potential at point P(0,0,2) is unstable equilibrium. Justify each answer,
Yo

1 (b) Next, the dipole is kept in a similar way las shown in


oo

p = + K =
Ane^iz -a z-a Fig. 2.196(b)], near an inifnitely long straight wire having
eB

uniform negative linear charge density.


1 Iqa
4:cey z^-a^
r

1 P Pi P2
ou
ad

*■

4tcq z^-a^
Y

Potentials at point (x, y, 0) will be


1 <]
nd
Re

Fig. 2.196(b)
4jt£
0
yjx^+y^ + a^
Will the dipole be in equilibrium at these two positions ?
Fi

1
V Justify your answer. (CBSE SP 181
4nz 0
yjx^ + y^ + a^ Solution, (a) is in stable equilibrium
Total potential at point (:r, y,0) will be P2 is in unstable equilibrium.
The direction of electric field, on either side, is
V = V^ + V_=Q directed towards the negatively charged sheet and its
(n) Points (5, 0, 0) and (-7,0,0) are the points on the magnitude is independent of the distance of the field
X-axis i.e., these points lie on the perpendicular point from the sheet. p\ is parallel to E while ^ i: is

bisector of the dipole. Electric potential at each of these -*

points will be zero. antiparallel to £.


Work done in moving the test charge from point (b) The dipole will not be in equilibrium in any of
(5, 0, 0) to (7, 0, 0) is the two positions. The electric field due to infinite
charged wire is non-uniform (£ocl/r). Hence there
W^q{V^-V2) = q{0-0)^0 will be a net non-zero force on the dipole in each case.
2.92 PHYSIC5-XII

Guidelines To NCERT Exercises

2.1. Two charges 5x10 and -3x10“® C are located 2.2. A regular hexagon of side 10 cm has a charge of5 pCat
16 cm apart. At what point on the line joining the two charges is of its vertices. Calculate the potential at the centre of the
the electric potential zero ? Take the potential at infinih/ to be hexagon,
zero.
Ans. Clearly, distance of each charge from the centre
Ans. Zero of electric potential for two charges. As O is
shown in Fig. 2.197, suppose the two charges are placed on r = 10 cm = 0.10 m

X-axis with the positive charge located at the origin O.


5pC 5pC
ij,=5xl0 C (^2 = -3x10 C

o P A
X 0.16-X '9/
5pC 5pC
Fig. 2.197' c

Let the potential be zero at the point Pand OP = x. For


X < 0 (j.e., to the left of O), the potentials of the two charges 5\iC 10 cm 5 nC
cannot add up to zero. Clearly, x must be positive. If x lies
between 0 and A, then
Fig. 2.199
v^ + v^ = o
Magnitude of each charge is
1 ●h
— +
‘h = 0 (? = 5 ^C = 5 X 10“^C
4ji 6q X 0.16-X .●. Potential at the centre O is

-8 -8 1 V=6.
5 X 10 3x10
or 9x10^
X 0.16-X
= 0 4jie
o''’
6 X 9 X 10^ X 5 X 10^
5 3
or = 0 0.10
X 0.16-X
= 2.7x10® V.
or X = 0.10 m =10 cm
2.3. Two charges + 2pC and -2 pC are placed at points A
The other possibility is that x may also lie on OA and B, 6 cm apart, (i) Identify an equipotential surface of the
produced, as shown in Fig. 2.198. system (ii) What is the direction of the electric field at eveiy
point on the surface ?
^] = 5 X 10 **€ ^2=-3x10 C Ans. (0 The equipotential surface will be a plane
\
O A p normal to AB and passing through its midpoint O, as
0.16 + x-0.16 ■M shown in Fig. 2.200. It has zero potential everywhere.
X

Fig. 2.198
£

As
+ 2mC -2pC
o- o
-8 -8 A O
1 5x10 3x10 B
= 0
4n 8 X x-0.16
0 L
Equipotential
5 3 surface
or =0
X x-0.16

or X = 0.40 m = 40 cm
Fig. 2.200

Thus the electric potential is zero at 10 cm and 40 cm {ii) The direction of electric field is normal to the
away from the positive charge on the side of tine negative plane in the direction AB i.e., from positive to negative
charge. charge.
ELECTROSTATIC POTENTIAL AND CAPACITANCE 2.93

2.4. A spherical conductor of radius 12 cm has a charge of 2.8. In a parallel plate capacitor with air between the plates,
1.6xlQ~^C distributed uniformly on its surface. What is the each plate has an area o/6 x 10'^and the distance between
electric field the plates is 3 mm. Calculate the capacitance of the capacitor. If
(a) inside the sphere the capacitor is connected to a 100 V supply, what is the charge
(b) just outside the sphere on each plate of the capacitor ? [CBSE SP 23]

(c) at a point 18 cm from the centre of the sphere ? Ans. Capacitance of capacitor with air between its
Ans. Refer to the solution of Example 75 on page 1.60. plates is
-12
^ EqA 8.85x10 x6xl0“^
2.5. A parallel plate capacitor with air between the plates
has a capacitance of 8pF (IpF = 10"’^ F). Wlmt will be the
^~~T~ 3x10"^
-11
capacitance if the distance between the plates be reduced by hal,f = 1.8x10 F=18 pF,
the space between them is filled with a substance of dielectric Charge,
constant, k = 6 ? -11
q=CQV = 1.8x10 xlOO

w
Ans. Capacitance of the capacitor with air between its = 1.8x10"’ C.
plates.
2.9. Explain what would happen if in the capacitor given in
_ Eg A
^ = 8pF Exercise 2.8, a 3 mm thick mica sheet (of dielectric constant-6)

Flo
were inserted between the plates, (i) while the voltage supply
When the capacitor is filled with dielectric (k = 6) remains connected (ii) after the supply was disconnected.
between its plates and the distance between the plates is

ee
[CBSE SP 23]
reduced by half, capacitance becomes,
Ans. From the above question, we have

Fr
X 6 X A

d
0
C^ = 1.8xl0““ F = 18pF, £^0 = 1.8 X 10"’C
d/2
Also, K= 6
= 12-^ =12x8 = 96 pF. for
ur
(i) When the voltage supply remains connected, the
d
potential difference between capacitor plates remains
2.6. Three capacitors each of capacitance 9 pF are connected same i.e., 100 V. The capacitance increases k times.
ks
in scries, (a) What is the total capacitance of the combination ? C=Kqj=6xl8 = 108pF.
Yo

(b) What is the potential difference across each capacitor when The charge on the capacitor plates will be
oo

the combination is connected to a 120 V supply ? = CV = 108 X 10"’^ X 100 = 1.08 X10'® C
Ans. (a) If C is the equivalent capacitance of the series
eB

combination, then (ii) After the supply is disconnected, the charge on the
capacitor plates remains same i.e., = 18 x 10“^ C
1-1. J_ J_-i i 1-1-1 The capacitance increases k times.
c~q q q 9*^ 9”9~3
r

C=Kq3=108 pF.
ou
ad

or C=3 pF.
The potential difference between the capacitor plates
Y

(b) As all the capacitors have equal capacitance, so becomes


potential drop A V would be same across each capacitor. q _ 100
y = = 16.6 V.
V=A +Aq + Aq
nd

K 6
Re

=AV+AV+AV=3AV
2.10. A 12 pF capacitor is connected to a 50 V battery. How
q_ no
Fi

or A\^ = = 40 V. much electrostatic energy is stored in the capacitor ?


3 ” 3
Ans. Here C = 12 pF = 12 x 10"*^ F, V = 50 V
2.7. Tltree capacitors of capacitances 2 pF, 3 pF and 4 pF are
Energy stored,
connected in parallel, (a) What is the total capacitance of the
U =
combination ? (b) Determine the charge on each capacitor if the
combination is connected to a 100 V supply.
Ans. («) For the parallel combination, total capacitance |x12x10'^2 ^^50)2
is given by = 1.5xlO"®J.
c=q + q + q = 2+ 3 + 4 = 9pF. 2.11. A 600 pF capacitor is charged by a 200 V supply. It is
(b) When the combination is connected to 100 V then disconnected from the suppdy and is connected to another
supply, charges on the capacitors will be uncharged 600 pF capacitor. How much electrostatic energy is
g, = qv = 2xio X100 = 2x10 -*10 ^
-12
lost in the process ?
-12
= qi/ =3x10 xl00 = 3xl0"'®C Ans. Here q = 600 pF, q = 200 V,
q^ = qv =4x10
-12
xl00 = 4xl0
-10
C. q = 600 pF, q = 0
PHYSICS-XII

Common potential. 2.13. A cube of side b has a charge cj at each of its vertices.
-12 Determine the potential and electric field due to this charge
v = + _ 600x10 X200+0
= 100 V
-12 array at the centre of the cube.
q +q (600 + 600) X10
Ans. Length of longest diagonal of the cube
Initial energy stored,
= ^b^ + b^ + = Vs &
u- = q=-|qq2=lx60oxio
-12
x(200)^ = 12xl0"^J
Distance of each charge (placed at vertex) from the
centre of the cube is
Final energy stored,

=-i(q + q)i^^ =^(600+ 60o)xio


-12
x(lOO)^ r -
2

= 6xl0“^J Potential at the centre of the cube is

Electrostatic energy lost,


y = 8.
AL/=Ii.-[i, =12x10
-6
-6x10
-6
47T6o ■ r
= 6x10“^ J = 8x
1

2.12. A charge o/8 mC is located at the origin. Calculate 4ti€q VSfc


the work done in taking a small charge of-2 x IQ~'^Cfrom 4q
a point P (0, 0, 3 cm) to a point Q(0, 4 cm, 0) via a point R (0, Vs ttCq!)
6 cm, 9 cm).
Electric fields at the centre due to any pair of charges
Ans. As the work done in taking a charge from one at the opposite comers will be equal and opposite thus
point to another is independent of the path followed,
therefore
cancelling out in pairs. Hence resultant electric field at the
centre will be zero.

2.14. Two tiny spheres carrying charges 15 pC and 2.5 pC


^ = %[VQ-V,] = q,
are located 30 an apart. Find the potential and electric field (a) at
_ %<i 2 ^ the midpoint of the line joining the two charges, and (b) at a
point 10 an from this midpoint in a plane normal to the line and
4n £q f2
passing through the midpoint.
Ans. (fl)

£i £2
A B
+
15 cm O 15 cm
^,“1.5 pC ^2 = 2.5pC

Fig. 2.202 (a)

Electric potential at the midpoint O,

V.0 _“ 1 ^+
4ne,0 vi '2;
-6
1.5x10 2.5x10"®
= 9x10^ V
0.15 0.15

= 2.4 xlO® V.

Electric field at the midpoint O,


Here
q = S mC = 8 X 10 ^C, </q = 2xl0"^C
^ *?2 ?1
'i = 3cm = 3 X 10“^ m, r, = 4cm = 4xl0“^m
.-. W = -2xl0"^x8xl0"^x9xl0^
1 1 2.5x10^ 1.5xl0~^
= 9x10^ V
(0.15)^ (0.15)^
X

4x10“^ 3x10"^
= 1.2 J. = 4.0xl0^Vm"', from charge 2.5 pC to 1.5 pC
ELECTROSTATIC POTENTIAL AND CAPACITANCE

e
{b) PA=PB = Vl5^ + 10^ cm B+a-- = 90“
2
= V325cm 18 cm = 0.18 m
A

B = 90'=-a + - = 90°- 76.9°+ 56.3°= 69.4°


2

Hence, the resultant field £ makes an angle of 69.4°


with the line joining charge 2.5jiC to 1.5)aC.
2.15. A spherical conducting shell of inner radius r, and
outer radius T2 has a charge Q
(a) A charge is placed at the centre of the shell. What is the
surface charge density on the inner and outer surfaces of
the shell ?

(b) Is the electric field inside a cavity (with no charge) zero


Fig. 2.202 (b) even if the shell is not spherical, but has any irregular

w
shape ? Explain.
Electric potential at point P, Ans. (fl) The charge q placed at the centre of the shell
Vp = 1 /h ^2 induces a charge - ^ on the inner surface of the shell and
4n£Q PA PB charge + </ on its outer surface.

Flo
1.5x10"^ 2.5x10 -6 Surface charge density on the inner surface of the shell
= 9x10^ v=2xl0® V. _ charge <7

ee
0.18 0.18
surface area

Fr
Electric field at Pdue to charge q^,
Surface charge density on the outer surface of the shell
9x10^x1.5x10"^ Q+ q
= 0.42 xlO^Vm -1
AkZq' PA^ (0.18)^ 4tc^
for
ur
Electric field at Pdue to charge (&) Even if the shell is not spherical, the entire charge
resides on its outer surface. The net charge on the inner
_ 9x10^
xlSxlO"^
£, =-i— = 0.69xl0^Vm
-1
ks
surface enclosing the cavity is zero. From Gauss s theorem,
^ 4t:Eo'PB^ (0.18)^ electric field vanishes at all points inside the cavity. For a
Yo
oo

ZAPO = ZBOP = Q / 2 cavity of arbitrary shape, this is not enough to claim that
In right AAOP, electric field inside must be zero. The cavity surface may
eB

15 9 have positive and negative charges with total charge zero.


tan- = — = 1.5 => - = 56.3°
2 10 2
r

9=2x5.63°= 112.6°
ou
ad

The resultant field at P will be


Y

cos 9

-1

= ^(0.42)^+(0.69)^+2x0.42x0.67cos 112.6° xlO^ Vm


nd
Re

Fig. 2.203 Electric field vanishes inside a cavity of any shape.


= VO.1764 + 0.4761 - 0.2227 x 10*^ Vm
-1

To overrule this possibility, consider a closed loop


Fi

[cosll2.6°=-0.3843]
PQRSP, such that part PQR is inside the cavity along a
= V0.4298xlO‘^Vm
-1
-6.6 xlO® Vm"’ line of force and the part RSP is inside the conductor.
Since the field inside a conductor is zero, this gives a
Let the field E make angle a with E^. Then,
network done by the field (in part RSP) in carrying a test
tana =
^sin9 charge over a closed loop. But this is not possible for a
+ £jCOs9 conservative field like the electrostatic field. Hence there
0.69x10^ X 0.9239 0.688 are no lines of force (i.e., no field), and no charge on the
= 4.3
inner surface of the conductor, whatever be ils shape.
" 0.42x10^ +0.069 xl0^(-0.3843)~ 0.10
2.16. (a) Show that the normal component of electrostatic
[sinll2.6°= sin 67.4°= 0.9239]
field has a discoiJtinuity from one side of a charged surface to
= tan"’(4.3)=76.9°
a another given by
a
Let E make an angle p with the direction from B to
^0
A Then from right APOC,
2.96 PHYSICS-XII

where n is a unit vector normal to the surface at a point and c is Consider a rectangular loop ABCD with length I and
the surface charge density at that point. (The direction ofn is alorig the dosed
from,Mel to side 2) path/IBCD will be
Hence show that just outside a conductor, the electric field £.d/ = £|./-^./=0
is onl Eq. or
E^l cos 0] - cos 02 = 0
(b) Show that the tangential component of electrostatic field cos 0J - £2 cos 63) / = 0
is continuous from one side of a charged surface to another. (^'-£^)/=0
[Hint: For (fl). Use Gauss's law. For (b), use the fact that u r. j r-. >
work done by electrostatic field on a closed loop is zero.] ^ ^ tangenhal components of E, and
Ans. (fl) Electric field near a plane sheet of charge is respectively. Thus,
given by
(V /;^0)
£ = -^
2^0 Hence the tangential component of the electrostatic
field is continuous across the surface.
If « is a unit vector normal to the sheet from side 1 to
side 2, then electric field on side 2
2.17. A long charged cylinder of linear charged density X is
surrounded by a hollow co-axial conducting cylinder. What is
a
n
A
the electric field in the space between the two cylinders ?
2s 0
Ans. Refer answer to Q. 35 on page 2.30.
in the direction of the outward normal to the side 2. 2.18. In a hydrogen atom, the electron and proton are bound
Similarly, electric field on side 1 is at a distance of about 053 A
n (/) Estimate the potential energy of the system in eV,
2e 0 faking the zero of potential energy at infinite
in the direction of the outward normal to the side 1. separation of the electron from proton,
■■■ =
a a a (ii) What is the minimum work required to free the elec
2e
2so
tron, given that its kinetic energy in the orbit is half
0 %
the magnitude of potential energy obtained in (i) ?
As E^ and £3 act in opposite directions, tliere must be (m) What are the answers to (i) and (ii) above if the zero
discontinuity at the sheet of charge. Now electric field of potential energy is taken at 1.06 A separation ?
vanishes inside a conductor, therefore Ans. (0 -19
= -1.6x10 <?2 =+ 1.6x10 C,
£,=0 r = 0.53 A = 0.53 x 10"’° m

Hence outside the conductor, the electric field is P.E. of the electron-proton system will be
1
£ = £3 = — n U =
^0 4;i8

(b) Let XY be the charged surface of a dielectric and ^


-19

0.53x10"’°
and £3 be the electric fields on the two sides of the charged
surface as shown in Fig. 2.204. 9x1.6x1.6x10"’^
-19
eV=-27.2 eV.
0.53 X 1.6 X 10

(ii) K.E. of the electron in the orbit


1
A = -P.E.= -x27.2eV = 13.6eV
E'l =£] cos 0] D
2 2
X
' + + + + + + +"+*+"+"1
Total energy of the electron
B C
= P.E. + K.E.

= ( - 27.2 + 13.6) eV = -13.6 eV


Dielectric
As minimum energy of the free electron is zero, so
£2 = £2 cos 0j minimum work required to free the electron
= 0-(-13.6) =13.6 eV.
Fig. 2.204
ELECTROSTATIC POTENTIAL AND CAPACITANCE

(ill) When the zero of potential energy is not taken at (1.6x10"^^)^ x9xl0^ (-4] T
infinity, the potential energy of the system is 10
-10
" T ^
li = (1.6x10’^^)^ x9xl0’ x4 eV
4ttEo [ -19
=-19.2 eV
^2 1.6x10 x3
-19
= 9xl(f X (-1.6 X 10"^^) X 1.6 X10
-19
[v leV =1.6x10 J]
1 1 2.20. Two charged conductmg spheres of radii a and b are
J connected to each other by a wire. What is the ratio
X

0.53 X 10
-10
1.06x10
-10 of electric
fields at the surfaces of the two spheres ? Use the result obtained
9 X 10^ X 1.6 X 10"^^ X 1.6 X 10”^^ r 1 to explain why charge density on the sharp and pointed ends of a
1-- eV
1.6x10
-19
X 0.53 X10
-10
2 conductor is higher than that on its flatter portions.
9x1.6 Ans. The charges will flow between the two spheres

w
eV =-13.6eV till their potentials become equal. Then the charges on the
0.53x2
two spheres would be
This indicates that the K.E. of 13.6 eV of case (i) is used
up in increasing the P.E. from -27.2 eV to - 13.6 eV as the

Flo
Q2 q
electron is carried from 0.53 A to 1.06 A position. K.E. in
this situation should be zero. As the total energy in this But

ee
case is zero, therefore, minimum work required to free the q ^
electron

Fr
= 0-(-13.6eV)= 13.6 eV. 9x- a

Qi b
2.19. If one of the two electrons of a H2 molecule is removed,
for
ur
The ratio of the electric fields at the surface of the two
we get a hydrogen molecular ion (Hf). In the ground state of a
H2 ion, the two protons are separated by roughly 15 A, and the spheres will be
electron is roughly lA from each proton. Determine the 1 Q1
ks
potential energy of the system. Specify your choice of the zero of £, 4Tieo ' Q ^_a ^_b
Yo

potential energy. q _ 1 Qi Q2 b a'^ a


oo

Ans. The system of charges is shown in Fig. 2.205. 4n Eq b^


eB

Electron
Also,
=-4
q 02
r

f3 = 1 A
ou

''l = 1A
ad

a
Y

Thus the surface charge densities are inversely


q2 = + q 43 = + 4 proportional to the radii of the spheres. Since the flat
Proton rj = l.SA Proton
portion may be considered as a spherical surface of large
nd
Re

Fig. 2.205 radius and a pointed portion as that of small radius, that is
why, the surface charge density on the sharp and pointed
Fi

Charge on an electron, ends of a conductor is much higher than that on its flatter
% = - c = - 1.6 X10 -19 c portion.
Charge on each proton, 2.21. Two charges -q and + q are located at points (0,0,-u)
q2 = q^ = + e -+ 1.6 X 10“’^^ C and (0,0,d) respectively.
If the zero of potential energy is taken at infinity, then (;) What is the electrostatic potential at the points
potential energy of the system is (0,0,z)imd(Ar,i/,0) ? [CBSE D 19]
1
U — U^2 “t" U23 + U,~y
^1^2 +. ^2% , ^1% (i7) Obtain the dependence of potential on the distance r
13 =
4jie
0 L
.r
'1 ^3 J of a point from the origin when r/a»l
1
(ill) How much zvork is done in moving a small lest
(-9)? q.q (-9)9
-10 -10
+
-10 chargefrom the point (5, 0, 0) to{-7,0,0)along the
4jI£q 1 X 10 1.5 X 10 1x10
x-axis ?
1
-10
-1 + -1 [q = e]
Does the answer change if the path of the test charge
4Jle,^0 xlO 1.5 between the same points is not along the x-axis ?
. J

2.98 PHYSICS-XII

Ans. (i) When the point P lies closer to the charge + q (in) (5,0,0) and (- 7, 0,0) are the points on the X-axis
as shown in Fig. 2.206(a), the potential at this point P i.e., these points lie on the perpendicular bisector of the
will be dipole. Each point is at the same distance from the two
1
1.1 1
1 charges. Hence electric potential at each of these points is
V = zero.
4n Gq [ rj 4tt e 0 z - a z-(-a)
2fl
Work done in moving the test charge qQ from the point
(5, 0, 0) to (- 7, 0, 0) is
4ti Sq
W = q{V,-V2) = q{0-0) = 0.
1 P
or V = [●.● p = qx2a] No, the answer will not change if the path of the test
4n So0 2^ -
charge between the same two points is not along X-axis.
This is because the work done by the electrostatic field
Zi between two points is independent of the path connecting
the two points.
P (0,0, z)
2.22. Figure 2.207 below shows a charge array known as an
+ ^ (0,0,fl) electric quadrupole. For a point on the axis of the quadrupole,
obtain the dependence of potential on rfor r» a. Contrast your
► y («)
o result with that due to an electric dipole and an electric
monopole (i.e. a single charge).
- q (0, 0, - a)
a a
X
+ 4 -q -q ■rq P
r
Z.

Fig. 2.207

+q (0,0, a) Ans. Potential at point P is


1
O
Y (b) V =
4tc r - a r r r+ a
0 L

- q (0, 0, - a)
1 r(r + a)-2(r - a)(r + a)+ r(r ~ a)
. q
X P (0, 0, 2) 4neo r(r-a)(r+a)

1 r'^ + ar-2r^ + 2a^ + - ar


Fig. 2.206 = -q
4nSo r(r^ -a^)
When the point Plies closer to charge - q, as shown in 1 2q a^ 1 Q
Fig. 2.206(fc), it can be easily seen that 4neo ■ r(r^ - a
2) 4nGo‘rtr2-A2)
1 P
V = -
4k Eg 2^ - where Q=2q a^ is the quadrupole moment of the given
charge distribution. As r » a, so we can write
Again, any point (x, y, 0) lies in XY-plane which is 1 Q
perpendicular bisector of Z-axis. Such a point will be at 4it£n0 r ^
equal distances from the charges - q and + q. Hence
potential at point (x, y, 0) will be zero. Hence for large r, quadrupole potential varies as 1/ r^,
whereas dipole potential varies as 1/ r^ and monopole
(ii) If the distance of point Pfrom the origin Ois r, then potential varies as 1/ r.
from the results of part (i)/ we get 2.23. An electrical technician requires a capacitance of 2 pF
1 P
V = ± [Put 2 = r] in a circuit across a potential difference oflkV. A large number
4rt Eg ’ of 1 pF capacitors are available to him each of xuhich can
U r» a, we can neglect a compared to r , so withstand a potential difference of not more than 400 V.
1 Suggest a possible arrangement that requires a minimum
P
V = ± number of capacitors.
4k Eg
Ans. Let this arrangement require « capacitors of 1 pF
.-. For r » a, the dependenceof potential P on r is 1 / each in series and m such series combinations to be
type. connected in parallel.
ELECTROSTATIC POTENTIAL AND CAPACITANCE 239

P.D. across each capacitor of a series combination Ans. As C2 ^ in series, their equivalent capa-
1000 1000 citance 200 X 200
= 400 or n= = 2.5 = 100 pF
n 400 q + q 200 + 200
But number of capacitors cannot be a fraction, Series combination of C2 and C3 is in parallel with Cj,
n = 3 their equivalent capacitance
Equivalent capacitance of the combination is = 100 pF + lOOpF = 200pF
1
-. w = 2 or m =2n = 6 The combination of Q q and q is in series with C^,
n
equivalent capacitance of the network
200 X100 200
Total number of capacitors required pF = —pF
= 3x6 = 18 200 + 100

So six series combinations, each of three 1 pF capacitors, Total charge on the network is
should be connected in parallel as shown in Fig. 2.208. 200
xlO
-12
X 300 = 2 X10"® C

w
1 pF 1 pF 1 pF
This must be equal to charge on q and also to the sum
1 pF 1 pF 1 pF of the charges on the combination of Q q and q.

Flo
,, = <, = 2xl0-“C
2x10
V = 200 V
1 pF 1 pF 1 pF

ee
-12
C4 100 X10

Fr
P.D. between points A and B
IpF IpF IpF
= V -V^= (300 - 200) V = 100 V
Vj = 100 V
for
ur
1 pF 1 pF 1 pF
7] = qVj = 100 X10 X 100 = 10"® C
-12

Also the P.D. across the series combination of C2 and q


1 pF 1 pF 1 pF = 100 V
ks
Yo

Now since q = q
oo

100
= 50V
IkV
^2 = ^3=^
eB

-12

Fig. 2.208
and (^2 = % = 200 X 10 x50=10“® C.
2.26. T7ie plates of a parallel plate capacitor have an area of
are separated by 2.5 mtTL The capacitor is
2.24. What is the area of the plates ofalF parallel plate 90 or?- each and
r

charged by connecting it to a 400 V supply.


ou

capacitor ? Given that the separation between the plates is 03 an.


ad

Ans. Here C = 2F, d = 0.5 cm = 5 x 10"^ m (i) How much energy is stored by the capacitor ?
Y

As C = bA
(if) View this energy stored in the electrostatic field
d between the plates and obtain the energy per unit
nd

2 X 5 X10"® volume u. Hence arrive at a relation between u and


Re

m^
8.85 X10
-12 the magnitude of electric field E between the plates.
Ans. (0 Here A =90cm^ =90x10
-4
^=9xl0"^m^
Fi

m
- 1130x10^ m^ =1130 km^
d = 2.5 mm = 2.5 x 10”^m,
2.25. Obtain the equivalent capacitance of the network = 8.85xl0"’^Fm \
shown in Fig. 2.209. For a 300 V supply, determine the charge = 400 V
and voltage across each capacitor. [CBSE OD 081
Capacitance of the parallel plate capacitor is
C = £qA 8.85xl0~^^x9xl0~^ ^
100 pF d 2.5x10"^
200 pF 200 pF = 31.86x10"^^ F = 31.86 pF.
A B
Electrostatic energy stored by the capacitor,
C2 C3 6 +
300 Vp_ U = - CV ^ = - X 31.86 X10"^^ x(400)^ J
100 pF 2 2

Fig. 2.209 = 25488 X 10"® J = 2.55 xlO ® J.


C4
2.100 PHYSICS-XII

(ii) Energy stored per unit volume or energy density of If u be the energy stored per unit volume or the energy
the capacitor is density of the capacitor, then increase in potential energy
U 2.55 X 10“^ of the capacitor
9 X 10'^ X 2.5 X 10" jjm ^
M =
Ad = M X increase in volume =u. A. Ax
F.Ax = u.A.Ax
= 0.113 Jm"^.
The relation between u and £ can be arrived at as
or F= uA =
E^.A = ^(s^E)AE
follows :
= ^.aA.E = ^qE
U =
U _2-
Ad Ad
1 Sq a
2 d 'Ad~2
1 The physical origin of the factor | in the force formula
dj
lies in the fact that just inside the capacitor, field is E, and
or « - outside it is zero. So the average value £/ Zcontributes to
the force.

2.27. A4nF capacitor is charged by a 200 V supply. It is 2.29. A spherical capacitor consists of two concentric
then disconnected from the supply and is connected to another spherical conductors, held in position by suitable insulating
uncharged 2 pF capacitor. Flow much electrostatic energy of the supports (Fig. 2.210). Show that the capacitance of a spherical
first capacitor is lost in the form of heat and electromagnetic (^^^P^^citor is given by
radiation ?
(CBSE OD 05]
Ans. Initial electrostatic energy of the 4 capacitor is
U.=lcV ^ =^x4xlQ-^ X(200)^ = 8x10"^J where ^ and rj are the radii of outer and inner spheres, respectively.
Charge on 4 pF capacitor
= C V = 4 X 10“^ X 200 = 8 X 10“‘*C
When the 4 pF and 2 pF capacitors are connected
together, both attain a common potential V. Thus
V = Total charge 8xlO“^C 400
V
Total capacitance (4 + 2) x 10"^ F ~ 3
Final electrostatic energy of the combination,
1^400^^
^x(4 + 2)xl0
-6
u
f ~ Fig. 2.210
3 ) o

= 5.33 X 10"^ J Ans. Refer answer to Q, 34 on page 2.30.


Electrostatic energy of the first capacitor lost in the capacitor has an inner sphere of radius
form of heat and electromagnetic radiation is ^ outer sphere of radius 13 an. The outer sphere is
A/i = H - J7 - 7R _ ^ in-2 , S‘ven a charge of 25 pC. The
u. o.:i3) X w J space between the co-centric spheres is filled with a liquid of
= 2.67x10"^ J. dielectric constant 32. (a) Determine the capacitance of the
2.28. Show that the force on each plate of a parallel plate potential of the inner sphere ?
_ .. . ^ ,1 (0) Compare the capacitance of this capacitor zvith that of an
capaator has a magmiude eqiml to - qE, where q is the charge isolated sphere of radius 12 an. Explain why the latter is much
smaller.
on
the capacitor, and E is the magnitude of electric field betiveen
Ans. Here a = 12 cm = 12 x 10 ^ m
the plates. Explain the origin of the factor ^ .
,

b = 13cm = 13 X 10"^cm ,
Ans. Let A be the plate area and a, the surface charge q = 2.5 pC = 2.5 X 10“^C, 1C = 32
density of the capacitor. Then
(a) Capacitance of the spherical capacitor is
q = A ab
C = 4Tre,.0 K.
E=-^ b - a
% 32 12xl0"^xl3xl0"^
I - p
Suppose we increase the separation of the capacitor 9x10^' (13 - 12) X 10"^
plates by small distance Ax against the force F. Then work 32 X 12 X 13 -11
done by the extenial agency = F. Ax xlO F = 5.5x10"’ F.
9
ELECTROSTATIC POTENTIAL AND CAPACITANCE 2.101

(b) Potential of the inner sphere is 2.32. A cylindrical capacitor has two co-axial cylinders of
-6
y_q^2.5x 10 9 V = 0.45 X 10^ V =4.5x10^ V. length 15 cm and radii 1.5 cm and 1.4 an The outer cylinder is
C 5.5 X10" earthed and the inner cylinder is givai a charge of 3.5 pC
Determine the capacitance of the system and the potential of the
(c) Capacitance of the isolated sphere of radius 12 cm is
12x10"^ inner cylinder. Neglect end effects (i.e., bending offield lines at
the ends).
C = 4;ie«R
0
=
9x10
5-F =1.3x10"” F.
Ans. Here L = 15 cm =0.15 m,
When an earthed conductor is placed near a charged =3.5 pC =3.5 X 10" ^ C fl =1.4 cm =0.014 m,
conductor, the capacitance of the latter increases. The two & = 1.5 cm =0.015 m
conductors form a capacitor. But the capacitance of an
isolated conductor is always small. Capacitance of a cylindrical capacitor is given by
2.31. Answer carefully: 2t^_ L
. b ~
In 2 2.303 log -
(i) Two large conducting spheres carrying charges and 4neo
^ a

w
Q2 are brought close to each other. Is the magnitude of 0.15
el^twstatic force between them exactly given by 0015 ^
QQz where r is the distance between their cen tres ? 2 X 9 X 10® X 2.303 log
47t e.,0 ' aoi4

Flo
0.15 XIQ-^
(i7) If Coulomb's law involved Vr^ dependence (instead of ” 18x2.303x0.03
F
Vr^), would GflHss' law be still true ?

ee
= 0.1206 X10"^ F = 1.2 X10"^® F
(Hi) A small test charge is released at rest at a point in an

Fr
electrostatic field configuration. Will it travel along the Potential,
-6
line offorce passing through that point ? 1-
3.5x10
V=2.9xl0‘‘ V.
(iv) What is the zuork done by the field of a nucleus in a
-10
C 1.2x10

for
ur
complete circidar orbit of the electron ? What if the orbit
is elliptical ? A parallel plate capacitor is to be designed with a
voltage rating \kV, using a material of dielectric constant 3
{V) Wc know »ml ekctnc ifeld is dtsconhniioiis across the
ks
surface of a charged conductor. Is electric potential also like the^field never to exceed say 10% of the dielectric strength.
Yo

discontinuous there ?
What minimum area of the plates is required to have a
oo

(vi) What meaning would you give to the capacity of a single capacitance of 50 pF ? [CBSE OD 05]
conductor ?
eB

Ans. Maximum permissible voltage


(vH) Guess a possible reason why water has a much greater = 1 kV = 10^ V
dielectric constant (= 80) than say, mica (= 6).
Maximumpermissibleelectric field
Ans. (i) No. When the two spheres are brought close to
r

= 10%of 10^ Vm"^ = 10^ Vm“^


ou

each other, their charge distributions do not remain


ad

uniform and they will not act as point charges. .'. Minimum separation d required between the plates
Y

(ii) No. Gauss's law will not hold if Coulomb's law is given by
involved 1 / or any other power of r (except 2). In or d= — V _ 10^ = 10"^ m

that case the electric flux will depend upon ralso. d E ■ 10^
nd
Re

(Hi) Not necessarily. The small test charge will move Capacitance of a parallel plate capacitor is
K Cr.A
along the line of force only if it is a straight line. The
Fi

0
C =
line offorce gives the direction of acceleration, and not d
that of velocity, Cd 50x10"” xlQ-^
A = m
(iv) Zero. But when the orbit is elliptical, work is done in 3 X 8.85 X 10
,-12

moving the electron from one point to the other. How


= 18.8xl0~*m^=19 cm^
ever, net work done over a complete cycle is zero,
(u) No, potential is everywhere constant as it is a 234. Describe schematically the ecjuipotential surfaces corres
scalar quantity. ponding to
(vi) A single conductor is a capacitor with one plate at (0 a constant electric field in the Z-direction.
infinity. It also possesses capacitance. (ii) a field that uniformly increases in magnitude but
(vii) Because of its bent shape and the presence of remains in a constant (say, Z) directions.
two highly polar O - H bonds, a water molecule (Hi) a single positive charge at the origin,
possesses a large permanent dipole moment about
0.6x10"^^ Cm. Hence water has a large dielectric (iv) a uniform grid consisting of long equally spaced
constant. parallel charged wires in a plane.
2.102 PHYSICS-Xll

Ans. (0 For a constant electric field in Z-direction, 2.37. Answer the following :
equipotential surfaces will be planes parallel to XY-planes, (i) The top of the atmosphere is at about 400 kV with respect
as shown in Fig. 2.211. to the surface of the earth, corresponding to an electricfield that
kZ decreases with altitude. Near the surface of the earth, the field is
aboutlQO Vm~^. Whydo then wenotgetan electric shock as we
step out of our house into the open ? (Assume the house to be a
Z 7 steel cage so there is no field inside.)
" Z 7 (ii) A man fixes outside his house one evening a two metre
Equi potentials high insulating slab carrying on its top a large aluminium sheet
Z 7 of area 1 m^. Will he get an electric shock if he touches the metal
►r
sheet next morning ?
/ o
(iij) The discharging current in the atmosphere due to the
small conductivity of air is known to be 1800 A on an average
over the globe. Why then does the atmosphere not discharge
itself completely in due course and become electrically neutral ?
Fig. 2.211 In other words, what keeps the atmosphere charged ?
(iv) What are the forms of energy into which the electric
(ii) In this case also, the equipotential surfaces will be
planes parallel to XY-plane. However, as field increases, energy of the atmosphere is dissipated during a lightning ?
such planes will get closer. Ans. (f) Normally the equipotential surfaces are
(Hi) For a single positive charge at the origin, the parallel to the surface of the earth as shown in Fig. 2.212.
equipotential surfaces will be concentric spheres having Now our body is a good conductor. So as we step out into
origin as their common centre, as shown in Fig. 2.25. The the open, the original equipotential surfaces of open air
separation between the equipotentials differing by a get modified, but keeping our head and the ground at the
constant potential increases with increase in distance same potential and we do not get any electric shock.
from the origin. 300 V 300 V
(iv) Near the grid the equipotential surfaces will have
varying shapes. At far off distances, the equipotential 200 V 200 V
surfaces will be planes parallel to the grid.
2.35. In a Van de Graaff type generator, a spherical metal shell 100 V
is to be a 15 X 10^ V electrode. The dielectric strength of the gas
surrounding the electrode is 5 x Itf Vnf^. What is the mini
mum radius of the spherical shell required ? [CBSE OD 08]

Ans. Maximum permissible potential, V = 1.5 x 10^ V 77777777777777 77777777777777


For safety, the maximum permissible electric field is Ground Ground

£ = 10% of dielectric strength (a) ib)


= 10% of 5 x 10^ Vm”^ = 5 X 10^ Vm
-1

Fig. 2.212
Now for a spherical shell.
1 5
(ii) Yes. The aluminium sheet and the ground form a
4n 8
0
capacitor with insulating slab as dielectric. The dis
1
E = charging current in the atmosphere will charge the capa
4n Sq r citor steadily and raise its voltage. Next morning, if the
man touches the metal sheet, he will receive shock to the
.-. Minimum radius required is
V l.SxlO^V extent depending upon the capacitance of the capacitor
-1
= 3x10 m = 30 cm. formed.
''“'e“5x10^ Vm'^
(Hi) The atmosphere is continuously being charged by
2.36. A small sphere of radius r^ and charge is enclosed by tlumder storms and lightning bolts all over globe and
a spherical shell of radius and charge q^- Show that if q^ is maintains an equilibrium with the discharge of the
positive, charge will necessarily flow fivm the sphere to the shell atmosphere in ordinary weather conditions.
(when the two are connected by a wire) no matter what the (iv) The electrical energy is lost as (i) light energy
charge q2 on the shell is. involved in lightning (i7) heat and sound energy in the
Ans. Refer answer to Q. 55 on page 2.67. accompanying thunder.
ELECTROSTATIC POTENTIAL AND CAPACITANCE 2.103

OTQs : Objective Type Questions

^YPE A : Multiple Choice Questions

Based on Electric Potential due to a Point Charge, 8. In the case of a charged metallic sphere,
Group of Point Charges, Electric Dipole and potential (V) changes with respect to distance (r) from
the centre as
Charged Spherical Shell
1. The amount of work done in moving a unit («) {b)
positive charge (without acceleration) from infinity to

w
a point in an electric field against the electrostatic
force is

(fl) electric field (b) electric potential

Flo
(c) electric flux (b) electrostatic p.e.
2. The electric potential due to a point charge -q at

ee
a point at distance r from it is (rf)

Fr
kq kq
(&)-
r

kq kq for
ur
(b) t
(c)
r

3. The electric potential at the-13


surface of an atomic
ks
nucleus (Z = 50) of radius 9.0 x 10 cm is
Yo

(fl)9xl0^ volt (b)8xl0^ volt


oo

(c) 80 volt (d) 9 volt 9. The potential on the hollow sphere of radius 1 m
eB

4. At a point A, there is an electric field of 500 V/m is 1000 V, then potential at |m from the centre of the
and potential difference of 3000 V. The distance
sphere is
between the point charge and A is
r

(b) 500 V
ou

(fl) 1000 V
ad

(fl) 6 m (b) 36 m
(c) 250 V (d) zero V
Y

(b) 12 m (d) 144 m


10. Three charges +2q,-q and -q lie at vertices of a
5. The electric potential on the axis of an electric
triangle. The values of E and V at centroid of triangle
nd

dipole at a distance 'r' from it's centre is V. Then the will be


Re

[CBSE SP 22]
potential at a point at the same distance on its
(a) E 0 and V 0 (b) £=0 and V^ = 0
Fi

equatorial line will be [CBSE SP 23]

(fl)2V (b) -V (c) E 0 and V = 0 (d) E=0ar\dV^0


(c) VI2 (d) zero 11. A solid spherical conductor has charge + Q and
6. A spherical drop of mercury having a potential radius R. It is surrounded by a solid spherical shell
of 2.5 V is obtained as a result of merging 125 droplets.
The potential of a constituent droplet would be
(fl) 1.0 V (b) 0.5 V
(c) 0.2 V (d) 0.1 V
7. Electric field and electric potential inside a
charged spherical shell are
(rt) £=0, V=0 (b) £=0, V?^0
(c) E=^0,V=0 {d) E^0,V=^0
r

^ 2.104 PHYSICS-XII

with charge Q, inner radius 2 R, and outer radius 3/?. 17. An electron initially at rest, is accelerated
Which of the following statements is true ? [CBSE SP 22] through a potential difference of 200 volt, so that it
(fl) The electric potential has a maximum acquires a velocity 8.4 x 10^ m/s. The value of e/m of
magnitude at C and the electric field has a electron will be
maximum magnitude at A (fl) 1.76x10” C/kg (b) 2.76x10” C/kg
(b) The electric potential has a maximum (c) 0.76x10^^ C/kg (d) none of these.
magnitude at D and the electric field has a
maximum magnitude at R 18. The velocity v acquired by an electron starting
(c) The electric potential at A is zero and the electric
from rest and moving through potential difference V is
shown by which of the following graphs ?
field has a maximum magnitude at D.
(«) ib)
{d) Both the electric potential and electric field
achieve a maximum magnitude at B.
Bosed on Relation between Electric Field and
Electric Potential and on Equipotential Surfaces
V V
12. Electric potential at any point is
{c) id)
V = -5x + 3y + ^/^5z,
then the magnitude of the electric field is
(a) 3V2 (b) 4^2 (c) 5^I2 id) 7

13. The angle between the electric lines of force and 1/


the equipotential surface is
(«)0“
19. A positive point charge +q is placed at the origin.
(b) 45°
There is an electric field
(c) 90° (d) 180° (NEETJuly 22]
14. In a certain region, a uniform electric field exists
along X-direction. The equipotential surfaces associated
with this field will be
that accelerates the point charge along the x-axis.
(<?) equidistant planes parallel to VZ-plane Determine the energy of the charge when it reaches the
(b) equidistant planes parallel to XY-plane position X =2 d.
(c) equidistant planes parallel to X2-plane (a) 6 qdE0 ib) 12 q
(d) coaxial cylinders of increasing radii around the
X-axis (c) 12 qdE,0 (d)24qdE^0
15. Which of the following is NOT the property of 20. Three charges 1 pQ 2 pC 3 pC are kept at
equipotential surface ? [CBSE SP 22] vertices of an equilateral triangle of side 1 m. If they are
(fl) They do not cross each other. brought nearer, so that they now form an equilateral
(b) The rate of change of potential with distance on triangle of side 0.5 m, then work done is
them is zero, (<7)11J (C)O.OIJ (d)O.llJ
(c) For a uniform electric field they are concentric 21. Two charges q^ and q2 are placed 30 cm apart, as
spheres.
shown in the figure. A third charge q^ is moved
(d) They can be imaginary spheres. along the arc of a circle of radius 40 cm from C to D.
^3
Based on Electric P.E., P.E. of Point Charges and c
P.E. of a dipole in a Uniform Electric Field
16. Mass of a proton is 1840 times that of an
electron. It is accelerated through a potential difference u

of 1 kV, what is the kinetic energy of proton ? o

1
(fl) 1 keV ib)
1840 keV
^2

(c) 1840 keV (rf)1kV A 30 cm B D


ELECTROSTATIC POTENTIAL AND CAPACITANCE 2.105

The change in the potential energy of the system is 26. A free electron and a free proton are placed
k, where k is between two oppositely charged parallel plates. Both
47T£
0 are closer to the positive plate than the negative plate.
(a) 8q^ {b) (c) 6q^ (d) 6q^ + + ■i- + + + 4- + + +|

22. A dipole is placed in a uniform electric field. Its © 0


potential energy will be minimum when the angle
between its axis and field is
(fl) zero ]
(b) Ti

id)2n Which of the following statements is true ?


I. The force on the proton is greater than the force
23. A dipole is placed parallel to the electric field. If on the electron.
W is the work done in rotating the dipole by 60®; then

w
work done in rotating it by 180® is II. The potential energy of the proton is greater
than that of the electron.
(fl)2W (fe)3W
III. The potential energy of the proton and the

Flo
W electron is the same.
(c)4W
(i7) I only (b) II only

ee
24. An electric dipole of moment p is placed parallel
(c) III and I only (d) II and I orUy (CBSE SP 22]
to the uniform electric field. The amount of work done

Fr
in rotating the dipole by 90° is (CBSE SP 22] Based on Capacitance of Conductors
(a)2pE ib) pE 27. The potential to which a conductor is raised,
for
ur
(c) pE/2 (d) zero depends on
25. The electric potential ^ as a function of distance (fl) the amount of charge
X is shown in the below figure.
ks
[CBSE SP 23] (b) geometry and size of the conductor
Yo

(c) both (fl) and (b)


oo

(d) only on (a)


eB

28. The earth has volume V and surface area A, then


capacitance would be
A
r

V
(fl)47tSo — (b) 4k£q —
ou
ad

0 2 4 6 X
V A
Y

The graph of the magnitude of electric field V A

intensity E as a function of x is
(c) 12ti£q —
A
(d)127l£Q —
V
nd

{a) (b) 29. 8 drops of Hg are combined to form a bigger


Re

+E single drop. The capacitance of a single small drop and


Fi

+E
that of the single big drop will be in the ratio of
0 *■ 0
(«} 1 : 2 {b)l:8
2 4 6 2 4
X
(c)8:l (d) none of these
-E -E
30. Two charged spherical conductors of radius Kj
and R2 are connected by a wire. Then the ratio of
(c) surface charge densities of the spheres (Oj / 02) is
R
1
+£ +£ («) ib)
R
1
6 6
0 0
2 4l r}1
(C) (d)
-£ -£ [NEET21]
2J06 PHYSICS-Xll

Based on Capacitance of Capacitors 38. The equivalent capacitance of the combination


31. The distance between the two plates of a shown in tlie figure is
parallel plate capacitor is doubled and the area of each
plate is halved. If C is its initial capacitance, its final
capacitance is equal to c
{a)2C I
C
c
(c)4C (^)- ib)2C
[NEET Sept. 22] {a)3C
C 3C
32. Capacitors are used in electrical circuits where
[NEET 21]
appliances need more
(fl) current (&) voltage 39. Equivalent capacitance between A and B is
(c) watt (d) resistance

Based on Grouping of Capacitors


33. Across each of two capacitors of capacitance 1 pF
and 4 pF, a potential difference of 10 V is applied.
Then positive plate of one is connected to the negative
plate of the other, and negative plate of one is connected 4 pF

to the positive plate of the other. AO-


if -OB
After contact, 4pF 4pF
(a) charge on each is zero
(fl) 8 pF (b) 6 pF
(&) charge on each is same but non-zero 10
(c) charge on each is different but non-zero (c) 268 pF {d)
38 pF
(d) none of these
40. Four capacitors are connected in a circuit as
34. Tliree capacitors 2 pF, 3 pF and 6 pF are joined in shown in figure. Tlie effective capacitance in between
series with each other. The equivalent capacitance is P and Q will be
(fl)l/2pF (i;)lpF 2pF 12 pF
(c)2pF (d) 11 pF [CBSE SP 22,

35. The effective capacitances of two capacitors are po 2pF oQ


3 pF and 16 pF, when they are connected in series and 2pF

parallel respectively. The capacitances of two


capacitors are
{a) 10 pF {b) 5 pF
(fl) 10pF,6pF (1j) 8 pF,8 pF
(c)2pF (rf) 7.5 pF
(c) 12 pF, 4 pF (rf) 1.2 pF, 1.8 pF
[NEET Sept. 22] 41. Equivalent capacitance of the given combi
nation of five capacitors is
36. Flow many 1 pF capacitors must be connected in
4pF
parallel to store a charge of 1 C with a potential of
110 V across the capacitors ?
8pF 4pF
(fl) 990 (&) 900 B C

(c) 9090 (d) 909 /I D


4pF
37. Three capacitors of capacitances 1 pF, 2 pF and
3 pF are connected in series and a p.d. of 11V is applied 4)iF
across the combination. Then, the p.d. across the plates
of 1 pF capacitor is (a) 4 pF (b) 10 pF

(fl)2V {&)4V (c)lV (d)6V (c)8pF (d) 120 pF


ELECTROSTATIC POTENTIAL AND CAPACITANCE 2.107

42. For the given circuit the equivalent capacitance 48. If there are n capacitors in parallel connected to
between P and Q is V volt source, then the energy stored is equal to
{a)CV {b)-nCV^'
2
P Q
{€) CV^ {d)—CV^
o- -O

C
2n [AIEEE 02]
C C C C c

49. A capacitor of 20 pF is charged up to 500 V is


(fl)6C (f;)4C connected in parallel witli another capacitor of 10 |iF
3C 6C
which is charged up to 200 V. The common potential is
id) (a) 500 V (b) 300 V
11
(c) 400 V (d) 200 V
43. In tlie circuit shown in the figure, the potential

w
difference across the 4.5 jiF capacitor is 50. Two insulated metallic spheres of 3 |iF and 5 pF
3jiF
capacitances are charged to 300 V and 500 V respec
4.5 ^iF tively. The energy loss, when they are connected by a

Flo
wire, is
(fl) 0.012 J (b) 0.0375 J
6 jiF

ee
(c) 0.0218 J (d) 3.75 J

Fr
51. A capacitor is charged by connecting a battery
12 volt across its plates. It stores energy U. Now the battery is
disconnected and another identical capacitor is
(rt) 8/3 volt (b) 4 volt for
ur
connected across it, then the energy stored by both
(c) 6 volt (d) 8 volt capacitors of the system will be [CBSE PMT 2000]
U
(n)U
(^)j
ks
Based on Energ/ Stored in Capacitors and Loss of
Yo

Energy on Redistribution of Charges


oo

44. A capacitor of capacitance C has charge Q and (c)2U


(d)|u
eB

stored energy is W. If the charge is increased to 2 Q, the


stored energy will be 52. A parallel plate capacitor of capacitance C is
W w
connected to a battery and is charged to a potential
difference V. Another capacitor of capacitance 2 C is
r
ou

similarly charged to a potential difference 2 V. The


ad

(c)2W (d) 4W charging battery is then disconnected and the


Y

capacitors are connected in parallel to each other in


45. A 4 pF capacitor is charged to 400 V. If its plates
such a way that the positive terminal of one is
are joined through a resistance of 2 k£T, then heat connected to the negative terminal of the other.
nd
Re

produced in the resistance is


The final energy of the configuration is
(b) 0.32 J
Fi

(fl) 0.16 J
(c) 0.64 J (d)1.28J ICBSE 1994, 95] (<7) zero

46. If P.D. across a capacitor is changed from 15 V to 25


30 V, work done is W. What will be the work done CV^ (d)-cy^
2
when P.D. is changed from 30 V to 60 V ?
(fl) W (&)4W 53. Two parallel plate capacitors X and Y, have the
(c)3W (ri)2W same area of plates and same separation between
plates. X has air and Y with dielectric of constant 2,
47. If the potential of a capacitor having capacity 8 |iF
between its plates. They are connected in series to a
is increased from 10 V to 20 V, then increase in its battery of 12 V. The ratio of electrostatic energy stored
energy will be in X and Y is

(fl) 4xl0“^J (b) 12xlO“^J (rt) 4 : 1 (b) 1 : 4


(c) 4xl0‘^J (d)12xlO"^J (c) 2 : 1 (d) 1 : 2 [CBSE SP 22]
s 2.108

Based on Capacitors filled with Dielectrics


54. A parallel plate air capacitor is charged and then
isolated. When a dielectric material is inserted between
PHYSICS-XII

doubled and the interspace between the plates is filled


with wax. If the capacity is increased to 6 pF, the
dielectric constant of wax is

the plates of the capacitor, then which of the following (^7)2 (&)3
does not change ? (c)4 (d)6
(fl) Electric field between the plates 61. If a dielectric plate of thickness t is placed
(b) Potential difference across the plates between the plates of a parallel plate capacitor of plate
(c) Charge on the plates distance d, the capacitance becomes half of the original
value. The dielectric constant of the plate will be
(d) Energy stored in the capacitor
2t 2t
55. When a dielectric material is introduced («) {b)
2d + t 2d~t
between the plates of a charged condenser, then t t
electric field between the plates {c) (d)
d + t d-t
(fl) decreases (b) remains constant
(c) increases
62. The plates in a parallel plate capacitor are
(d) first (c) and then (a)
separated by a distance d with air as the medium
56. If the distance between the plates of parallel between the plates. In order to increase the capacity by
plate capacitor is halved and the dielectric constant is 66% a dielectric slab of dielectric constant 5 is
doubled, then its capacity will introduced between the plates. What is the thickness of
(a) increase by 16 times the dielectric slab ?

(b) increase by 4 times / \ d /} \ d , . 5d


{a) - (b) - (c) — id)d
4 2 8
(c) increase by 2 times
{d) remain the same 63. A parallel plate air capacitor has a capacitance
C. When it is half filled with a dielectric of dielectric
57. A parallel plate condenser with oil between the
plates (dielectric constant of oil k = 2) has a capacitance constant 5, the percentage increase in the capacitance
will be
C. If the oil is removed, then capacitance of the
capacitor becomes (fl) 400% (b) 66.6%
C (c) 33.3%
(fl)^y2C (b) (d) 200%
V2 64. If the circumference of a sphere is 2 m, then
C
(c)2C capacitance of sphere in water would be
(u) 2700 pF (b) 2760 pF
58. A parallel plate capacitor is charged by a battery (c) 2780 pF (d) 2800 pF
to a potential difference of 'V" volts. After the charging
battery is disconnected, a dielectric slab with dielectric 65. An uncharged capacitor with a solid dielectric is
connected to a similar air capacitor charged to a
constant 'k ' is inserted between its plates. The
potential difference across the plates of the capacitor potential of V'g.If the common potential after sharing of
will become charges becomes V, then the dielectric constant of the
[CBSE SP 22] dielectric must be
(a) zero (b)V/2 V, y
(c)V/k (d) kV (b)iT
V y
0

59. A copper plate of thickness b is placed inside a


parallel plate capacitor of plate distance d and area A as (V^Q-n
(C) id)
V y0
shown in figure. — fr ^

The capacitance of capacitor is 66. Two capacitors of capacitance C are connected


As 0 Ae 0 in series. If one of them is filled with dielectric
substance k, what is the effective capacitance ?
Ae 0 kC
(C) id) CO {«) (b) C(K + 1)
d-b b (1 + K)
2kC
60. An air filled parallel plate condenser has a ic) (d) none of these
capacity of 2 pF. The separation of the plates is 1-rK
ELECTROSTATIC POTENTIAL AND CAPACITANCE 2.109-

67. A parallel plate capacitor of plate area A and 70. A capacitor of capacitance 1 pF is filled with two
separation d is filled with dielectric as shown in the dielectrics of dielectric constants 4 and 6. What is the
new capacitance ?
K
I
d

<2
k,=
1 4 Kj= 6 d

figure. The dielectric constants are and K2* Net


capacitance is (a) 10 pF (b) 5 pF
£qA f Kj + K2''

w
(c)4pF (d)7pF
(«) (b)^ a
{ ^1^2 )
Based on Collecting Action of a Hollow Sphere
K1K2 2£qA f Kj H-Kj 71. Given two hollow spherical

Flo
(d)-jd conductors A and 6 as shown in the
a
^Kj+K2, KjK2
figure.

ee
68. A parallel plate capacitor has capacitance C. If it Points M and N can be connected
is equally filled with parallel layers of materials of by conductor wire.

Fr
dielectric constants Kj and K2, its capacity becomes C|.
The ratio of C,1 to C is
Initially (before the connection
of points M and N) conductor B is
KjK2
for
ur
(fl) Kj + K2 (b) neutral, while charge appeared on conductor A is q.
+ K2 After connecting the spheres, the potential of sphere B
2k,k
r"2 will be
ks
id)
K1K2 Kj + K2
Yo
oo

87t£Q(r+ R) 47ce„R
69. Two dielectrics of dielectric constants and K2 0
eB

are filled in the gap of parallel plate capacitor as shown ic)


9
(d) zero
4jc8Q(r+ R)
72. A hollow charged metal sphere has a radius r. If
r

the potential difference between its surface and a point


ou
ad

d
at distance 3r from the centre is V, then the electric
Y

intensity at distance 3r from the centre is


(a) V/6r (fc) V/4r
nd
Re

in the figure. The capacitor has plate each of area A and (c)VI3r {d)Vl2r
separation d. The capacitance of the capacitor is 73. The insulation property of air breaks down at
Fi

, /

(«)
SqA(k^+K2)
ib)
8q A Kj+K2 £=3x10^ V/m. The maximum charge that can be
2d 2d
k^<2 given to a sphere of diameter 5 m is approximately (in
coulomb)
£0 A
(c)-e-
K1K2 KjK2 (fl) 2 X10”^ {b) 2 X10”^
d
{K^ + K^J d K.1 + Kt.1 J
(c) 2 X10
-4
(d) 2 X10“^

Answers and Explanations


1. (b) By definition, 3. (b) Electric potential at the surface of an atomic
W=V. nucleus,
-19
1 50x1.6x10
fc(-9) _ .±1 =9xl0^x = 8xlO^V.
2. id)V = -15
r r 47t£o r 9x10
:ii6» PHYSICS-XII

4.{fl) £ = 7^4
47tEQ r
and V =
1
4ne„0 r
9 At A, £=0, 1^=0

At B, P_feQ cfQ = Maximum


V 30Q0V
= 6m. (2R)^
£ "SOOV/m
y = = Maximum
5. {d) The potential at any point on the equatorial R 2R
line of a dipole is zero.
At C, £=_^Q__JQ_ = 0
6. (d) Potential of big drop (2R)2 (2R)^
X Potential of a small drop
y = A2-A2 =0
25 = (125)^^^xy = 25y 2R 2R
or y = 0.1 V.
At D, net charge = 0
7. (£>) Inside a charged spherical shell, electric field £=0,V = 0.
is zero and potential at any point is constant.
8. (1?) Potential at any point inside the sphere is 12. (c/) y = -5x+Zy + VTSz
constant and is equal to that on the surface.
1 ? £ =-(-5f+3/ + Vl5fc)
y = A A A

47te«0 R
= 5i-3/-Vl5fc
At outside points.
y =
1 ? 1
y QC-.
I £ I = 75^ + (-3)2 + (-V15)2 = 7.
i.e..
47ts«0 r r
13. (fl) Electric field lines are always perpendicular
Hence option (b) is correct.
to the equipotential surface.
14. (a) Planes perpendicular to X-direction i.e.,
9. (fl) The potential at any point inside the charged
planes parallel to YZ-plane will be equipotential surfaces.
hollow metallic sphere is same as that on its surface.
15. (c) For a uniform electric field, equipotential
10. (c) Net electric field at centroid G, £ ?* 0
surfaces are parallel planes perpendicular to the
electric field lines. All other options are the properties
of equipotential surfaces.
16. (fl) K.E. of the proton=Work done on the proton
=gAy = exlkV=lkeV.
17. (fl) K.E. gained by an electron when accelerated
through a potential difference of V volts is

eV=^nw^
Net potential at G,
e _v^ _ (8.4x10^)^
y =
k(2q) kg kg m~2V~ 2x200 = 1.76 X 10^1 Ckg-* .
r r r

18. (b) K.E. gained by the electron,


11. (d) Both the electric potential and electric field
assume maximum values at point R =eV

Thus the graph between v and V must be a


parabola. Only option (1j) is correct.
f
19. (c) £W = £„2^+3^
d d^)
Potential at a: = 2d will be
2d/'
X X
y = £0 + dx
0
d d^
ELECTROSTATIC POTENTIAL AND CAPACITANCE 2.111

Id
o -..312-^^
j X Being closer to the positive plate, P.E. of proton is
= £
0 d 2 d
2
3
positive due to repulsion and that of electron is
0 Jo negative due to attraction.

_ r4d^ (P.E.)p>(P.E.),
= 12d£
0
\d d}
0
Energy of the point charge at x = 2d is
Thus the potential on a conductor depends on the
U = qV =^\lqdE^. amount of charge, geometry and size of the conductor.
20. (d) Initial P.E. of the three charges,
U:I =
1 ^2% 28. (c)V = jTiK^ and A = 4kR^
4ne r
3V
0
Y- = —

w
-12 -12 -12 A 3 A
1x2x10 + 2x3x10 +1x3x10
= 9x10^
1
C = 4ner.R - 4ne 0
3V^_127t£oV
0
A) A

Flo
=99xlO"^J
Final P.E. of the three charges, 29. (») —I tiR^ =8x—Trr^ 3

ee
-12 -12 -12
1x2x10 + 2 X 3x 10 +1x3x10 R=2r
U./ =9x10^
or

Fr
0.5
C(small drop) _ 47i£(jr _ r _ 1 _ ^ . 2
99x10"^ C(bigdrop) 4jrepR 2r 2
= 198xl0"^J
for
ur
0.5 30. (fj) After the two conductors are cormected by a
W = Uj-U-= (198-99)x 10“^ =99x 10'^ J wire.

, ^1 = ^2
ks
=0.099 J « 0.1 J.
Yo
oo

21. (fl) AU=U^-U,. R1


eB

1
or -^k = ^1^2 I^2% , ?1^3 5i = ^
4ne 47t£ 03 0.1 0.4
0 OL ^2 ^
I ^1^2 I^2% , ^1^3
r

o
1^ ^1 4iiR^
ou

4k£q _ 03 0.5 0.4


ad

02 47tRj^ ‘^2
Y

or

‘h'Kl Kj'
nd

k=8q^.
Re

or
£«A
0
22. (fl)P.E. of a dipole is minimum when its axis (or 31. (d) C =
Fi

d
dipole moment) is parallel to the electric field.
C =
£n(A/2) _ 1 EpA _ C
U = -p£cos0“=-p£. 2d 4 d "4
23. (c) =p£(cos0=’-cos60°)=|p£=W 32. (fl) Capacitor is a device to store charge. It is
used in appliances where more current is needed.
W2 = p£(cos0°-cosl80°) =2p£ = 4W.
33. (c)Charge on each capacitor becomes zero when
24. (6)Here 0^ =0° and 02 =90° two capacitors of equal capacitances are charged and
W = p£(cos0°-cos90®) = p£(l -0) = pE. then connected to opposite terminals. But capacitances
of the two capacitors are given to be different.
25. (fl)Refer to the solution of Problem 4 on page 2.81.
34.(b)^=^ + ^ + ^=i2 + i3 + l=l
1 1
6
26. {b) fp = Fg, because F = qE, and q and £ are same c, q1 C2 C3
for both proton and electron. q=ipF.
PHYSICS-Xil

35.{c)‘Cp=q + C2=16^F 40. (b) The equivalent circuit is


qq 2nF
=3^F 12 nF
q-.q
or qq=3{q + q)=3xl6=48^iF Po-
C,
2|jF OQ
q(i6-q)=48 2^iF

On solving, q = 12 jiF, Cj = 4 |iF. Q


36. (c) Q = Q: + Q2 + Q3 + -Q,rtiCxV C'=2 |iF + 2nF = 4 laF
Q 1C 10^
n =

CV IpFxllOV 110
= 9090. C' and Cj are in series with effective capacitance.
C'xC1 4x12 48
37. (d) C" = _
— = 3pF
C' + C1 4 + 12 16
ImF 2^F 3hF
+ ? I j-*? I 1-^ +4 1-4 Effective capacitance between P and Q,
V-1 V2 ^3
C = C"+C2=3 + 2 =5pF.
41. (fl) The equivalent circuit is a balanced Wheat
stone bridge.
B

11 V 4^F 4^F

A
o-
8 ^F^———o
V = -^ + -l + -l=llV
c
"1 q q 4nF 4nF
D
(1 1 1\
U 2 3 4 pF _ 4 pF
4 pF 4 pF
4^ = llxA=6pC
11 ^ The 8 pF capacitor is ineffective.
4x4
T/ __1_6fC = 6 V. r
'-ABC
=
= 2pF
4 + 4
' q I^F
4x4
38. (b) The capacitor on the right hand side gets
r -
^ADC = 2pF
4 + 4
shorted.
C + C
B
C
ABC ADC = 2+2=4pF.
42. (c)
o-
C o o ■OB
A B
P
Q
B o
C C B D E F
C c c c c
Now the two capacitors are connected in parallel.
=C + C = 2 C.
eq Tire capadtors across DE and EF are short
39. (fl) The two sets of 4 pF and 4 pF form series circuited. So the equivalent circuit is
C
combinations with equivalent capacitance,
4x’4 2(4F
C' = =2pF P=B C
4 +4
;
4pF c A = D = F C
The equivalent drcuit
then takes the form as shown,
3C C
2mF
q^ =2+4+2 =8pF. /lo- ■OB po-
I

ELECTROSTATIC POTENTIAL AND CAPACITANCE 2A13

1
C„ 3C C 3C
50. (b) Energy loss = —. (^1-^2)'
eif 2 Cj + C2' ●
3C 1 3xl0~^5xl0~^
C
^ 4
(500 -300)^
2 (3 + 5)10“^
43. (cf) Effective capacitance of 3 fiF and 6 |iF
15xl0-^x(200)^ J = 0.0375 J.
capacitors connected in parallel, 16
C'=3 + 6=9pF
51. (6) Initial energy stored in one capacitor.
Now C' and 4.5 pF capacitors are in series.
U =
C = 3pF 2C
^ 9 + 4.5
When the battery is disconnected, charge q remains

w
Charge through the circuit, the same ie., q= constant. The capacitors form a
<? = q^V=3xl2=36 pC parallel combination.
r =C+C=2C
P.D. across 4.5 pF capacitor,

Flo
eq

Y-^ - = 8 volt. Final energy stored by the combination.


C 4.5 pF

ee
.,2
^ 9 1
U' = = -U.
Q' 2C 2x2C 2

Fr
eq
44. (d) W =
2C
52. (b) Here Q^=CV,
for
ur
W' = = 4 W.
2C 2C
Q2=2Cx2V=4CV
As the two capacitors are connected with opposite
45. (b) Heat produced in the 2 kD resistor polarity, the common potential is
ks
= Energy stored in the charged capacitor _4CV-CV = v
Yo
oo

2
ix4xl0“^x(400)^J =0.32 J. C1 + C2 C + 2C
Equivalent capacitance,
eB

46. (b)W=^a/2 => W«:y2 C' = C + 2C=3C

\2 Final energy of the configuration is


W2 J AP2
x2
r

60-30 r 30
= 4
ou
ad

% 130-15 15 U'=-C'V'^ = -x3Cx V^=-CV^.


2 2 2
Y

or
W2 =4W^ =4W. 0
53. (c) = d '
47. (b)AU = U^-U^ =^C(V}-V^)
nd
Re

_2SqA
= 1x8x10“^ (20^-10^) = 2q
Fi

= 4xl0‘^x300J = 12xl0~*J. For capacitors connected in series.


48. {b) Energy is additive for a parallel combination u
Q' u
Q'
of capacitors. X ~
2C^'
X
Y ~
2Cy
U = nx Energy stored in one capacitor.
= nxlcV^=lnCV^. = 2:1.
Uy
49. (c) v = 54. (c) When the battery is disconnected, the charge
q + C2
on the capacitor plates remains the same.
20 pFx500 V+10pFx200 V
55. (a) Due to polarisation of the dielectric, an
20 pF + 10 pF electric field is induced in the opposite direction of the
12000
-V=400V. applied field. The net field between the capacitor
30 plates decreases.
2.114 PHYSICS-XII

Sr,0 A
63. (6) Original capacitance,
K
56. (b) C = K =K i.e., C oc —
0 ^ d
Sr,A
_ ‘*0
2k 4k
C cc or C’ cc
dll d
c Area = A
or — =4 or C' = 4C
C ♦

c c
d/2 C,
57.(^)C,=-=-
d/2
4
58. (c) The charge Q on the capacitor plates remains
same. The surface charges induced on the dielectric
slab reduce the electric field to a new value,
When the capacitor is half-filled with a dielectric, it
E'=^ becomes a series combination of two capacitances Cj
K
and Cj ●
This reduces the potential difference to a new value, r =
&r,A
0
. £« A
Ed V ‘“i =2 0 =2C0
V' = E'd = dll d
K K
Sr,0 A s„A
0
= K = 5x2 = 10C0
59. (c) Electric field inside the copper plate is zero. It dll d
exists only in the region of thickness d-~b.
Sr,A
0
Equivalent capacitance.
C =
d-b C = qq _2qxioq_5q0
Sr,0 A Cj + C2 2Co + 10q0 3

60. (rf) For air filled capacitor, Cq = ^ =2pF


Increase in capacitance
For wax filled capacitor.
SrxA K - 2x6 2x6 -c0 200
C = K
0
= — Cn = 6 pF or K= -_3 xl00 = =66.6%.
Id 1 ^ ^ C0 2 C0 3

Sr, A 64. (d) lnR=lm


61. (c) Without dielectric, Cq =-^ Cq = 47tSQi? = 4s 0
With dielectric,
Capacitance in water,
Sr,0 A
_ ^0 - 1 ^0 -12
C = C = kCq =80x4x8.85x10 F = 2800 pF.
d-t + ^ l id
65. (c) Common potential,
K

t
2d = d-t + - 0-f cy.0 cv.0 V.0
y = _
K
_

t f q +q kC+C C(1 + k) 1+k


or d+t= — or K =
K d+t V,0 0
K = -1 =
V y
Sr,A
62.(i,)C„=-^ 66. (fl) We now have two capacitors of capacitances
C and kC connected in series.
With a dielectric slab of k = 5,
CxkC kC
&„A
0
C
C = C+kC 1+k
t
d-t +
5 67. (c) The arrangement is equivalent to series
t 4f combination of two capacitors of plate area A and
C0 _
d — t + —5 100 5 separation dll.
or
C d 166 d 1=J_ 1 1 d
J_ +J_
166x —=66d
4t c = c/q 2sq/II^Ki K
2J
5 d/2 d/2

66dx5 d lSr,A
0 K^Kz
t = or C=
166x4 2 d K,I +K.,2J
V
ELECTROSTATIC POTENTIAL AND CAPACITANCE 2.115 A

71. (b) After the connection, the charge on inner


68. (rf) For original capacitor, C = -^ a
sphere transfers to outer sphere. Then the potential on
From the above problem, we have the outer sphere is

r
Vg=—L.

Cj _ 2KjK2 72. (fl) Given = y

C Kj + K2 4k£qU 3rj
1 1
69. (fl) The arrangement is equivalent to a parallel or ~^ = ,,
V or
combination of two capacitors, each wjth plate area 4t18
0
3r 47teQ r 2
All and separation d.
i 1=^ 1=X.

w
47tEQ (3r)^ 4::£g r 9r 2 9r 6r
(A/2)k^1 £q(A/2)k2 _ EqA
d d Id
(K1 + K2). 73. (b) £ = —.4=9x10^x4
47I£a r r

Flo
0

70. (b) From the above problem.


or
Er^
3 X 10^ X (2.^ c =2.08 X 1Q"^C
C ^ 9x10^ 9x10^

ee
(Ki+K2) = -^(Ki+K2)
0
C =
Id
q should be less than 2.08 x 10“^C.

Fr
i(4 + 6) = 5pF. So option (b) is correct.

for
ur
^YPE B : Case Study Based Questions
ks
PARAGRAPH 1 Electric Potential
Yo
oo

\
An electric potential is defined as the amount work required to move a unit positive charge from a reference
eB

point to a specific point in an electric field without producing an acceleration. Usually, the reference point
is taken as the earth or a point at infinity where the potential is taken as zero. The potential energy for a
positive increases when it moves against an electric field and decreases when it moves with the electric
r

unit charge, we can define the electric potential as the


ou

field. Like we define electric field as the force per


ad

potential energy per unit charge. So electric potential can be expressed in units of joule per coulomb or
Y

volt. The electric potential near an isolated positive charge is positive


because work has to be done by an external agent to push a positive fcQi ^2 ^^3
nd

charge in, from infinity. The electric potential near an isolated negative
— + — + —
Re

'■2 '■3
charge is negative because the positive test charge is attracted by
Fi

negative charge. The electric potential due to a charge q at its own


location is not defined—it is infinite. Because of arbitrary choice of the
reference point, the electric potential at a point is arbitrary to within an
additive constant. But it is immaterial because it is the potential
difference between two points which is physically significant.

QUESTIONS (Answer any four of the following questions)


1, Which of the following is not true ? (c) The electric dipole potential varies as 1/ r at large
distance
(fl) For a point charge, the electrostatic potential
varies as 1/ r
(d) For a point charge, the electrostatic field varies as
l/r2
(£j) For a dipole, the potential depends on the position
vector and dipole moment vector
2.116 PHYSICS-XII

2. The variation of electrostatic potential V and the 4. n small metal drops of same size are charged to V
electrostatic field E with distance r from a point charge volt each. If they coalesce to form a single large drop, then
q is correctly shown by graphs : its potential will be
(«) V
(fc) W-
n
(b) Vn
t T (c)
tu
(d)
li.' 5. A hollow conducting sphere is placed in an electric
field produced by a point charge placed at Pas shown in
the figure. Let be the potentials at points A, B
{C) (rf) and C respectively.

T T
UJ
A
-P
c

B
r -*■

Then

3. What is the elctric potential at ■^q -q


(a)V^>V, (b) Vg >
the centre C of the square ?
kq (^)^A = ^C
(a) zero (b)
a

a V2 C

kq
(c)^
a
(d) none of these -q ■^q

PARAGRAPH 2 Faraday Cage


,.4

A Faraday cage or Faraday shield is an enclosure made of a


conducting material. The fields within a conductor cancel out
with any external fields, so the electric field within the enclosure
is zero. These Faraday cages act as big hollow conductors you can

put things in to shield them from electrical fields. Any electrical I

shocks the cage receives, pass harmlessly around the outside of I


? I

the cage. [CBSE SP 21]

QUESTIONS (Answer any four of the following questions)


6. Which of the following material can be used to 9. An
isolated point charge +q is placed inside the
make a Faraday cage ?
Faraday cage.
(fl) Plastic (b) Glass
(c) Copper
Its surface must have charge equal to
(d) Wood
(d) zero (b) +q
7. Example of a real-world Faraday cage is
(a) car (c) -q (d) +2q
(b) plastic box
(c) lightning rod (d) metal rod 10. A point charge of 2 pC is placed at centre of
Faraday cage in the shape of cube with surface of 9 cm
8. What is the electrical force inside a Faraday cage
when it is struck by lightning ? edge. The number of electric field lines passing through
the cube normally will be
{a) The same as the lightning
(a) 1.9 xlO^ Nm^/C, entering the surface
(b) Half that of the lightning (b) 1.9 xlO^ Nm^/C, leaving the surface
(c) Zero
(c) 2.26 xlO^ Nm^/C, leaving the surface
(d) A quarter of the lightning
(d) 2.26 xlO^ Nm^/C, entering the surface
ELECTROSTATIC POTENTIAL AND CAPACITANCE 2.117

PARAGRAPH 3 Capacitor

An arrangement of two conductors separated by an insulating medium can be used to store electric charge
and electric energy. Such a system is called a capacitor. The more charge a capacitor can store, the greater is
its capacitance. Usually, a capacitor consists of two conductors having equal and opposite charge +Q and
-Q. Hence, there is a potential difference 7 between them. By the capacitance of a capacitor, we mean the
ratio of the charge Q to the potential difference V. By the charge Dielectric
on a capacitor we mean only the charge Q on the positive plate.
Total charge of the capacitor is zero. The capacitance of a
capacitor is a constant and depends on geometric factors, such
as the shapes, sizes and relative positions of the two
conductors, and the nature of the medium between them. The
+

unit of capacitance is farad (F), but the more convenient units ]

w
are nF and pF. A commonly used capacitor consists of two long
Plate 1 Plate 2
strips or metal foils, separated by two long strips of dielectrics,
rolled up into a small cylinder. Common dielectric materials are

Flo
plastics (such as polyesters and polycarbonates) and aluminium
oxide. Capacitors are widely used in radio, television, computer,

ee
and other electric circuits.

Fr
QUESTIONS (Answer any four of the following questions)

for
ur
11. A parallel plate capacitor C has a charge Q. The 14. Three capacitors of 2.0,3.0 and 6.0 pF are connected
actual charges on its plates are in series to a 10 V source. The charge on the 3.0 pF
(«)aQ (b)QI2,Qn capacitor is
ks
(0 a-Q {d)Ql2,-QI2 (a) 5pC (b) lOpC
Yo

12. A parallel plate capacitor is charged. If the plates


oo

(c) 12pC {d) 15pC


are pulled apart,
15. What is the potential difference across 2pF
eB

(a) the capacitance increases


capacitor in the circuit shown ?
(fc) the potential difference increases
(c) the total charge increases 6V
r

h
ou

(d) the charge and potential difference remain the


ad

same.
2^F ^ ^3jiF
Y

13. If « capacitors, each of capacitance C, are connected 16 V


in series, then the equivalent capacitance of the com
nd

bination will be
Re

(fl) nC (b) (fl) 12 V (b)4V


Fi

(c) C/n (d) C/t7 (c)6V (d) 18 V

Answers

1. (c) The electric dipole potential at large distance. 1 +q~q+q~q = 0.


3. (rt) V =
1 pcosO 4jre
OL
■Jla
i.e..
47C£
0
r2
4. (d) Volume of the big drop
2. (/;) For a point charge, = n X Volume of a small drop
V<c- and £ tx: -4- 4
r -TtR^ = nx —Ttr
3

3 3
Graphs of figure {b) correctly show the variations V
and E with r. R =
2.118 PHYSiCS-X!l

1 7 12. (h) V= Ed
Potential of a small drop, V-
47re„0 r
As £ remains the same, so V increases as distance
increases.
Potential of the big drop,
V' = 1 _ 1 nq 13. (0 — = — + — + — + n factors = —
4ti6„0 R 4jte„0 n 1/3^ q C C C C

5. (d) Potential is same at every point of the


■■■

conducting sphere. 1 1 1
14. ib) - => C„ =lpF
■■■ K.=^8 = ^C- eq
2 3 6 6 1 eq

6. (c) Copper, because it is a conductor. Charge on each capacitor is


7. (fl) Car, its metallic body becomes an electrostatic (, = Cy = lpFxlOV = 104C.
shielding from lightning.
8. (c) Zero 15. (0 e„^j=16-6=10V
2x3 6 ^
9. (b) + q, on the outer surface due to induction. C
eq
=-uF
2+3 5
2 X 10~^
10. {€) -12 -xlO=12pC
8.85x10 5

=2.26xlO^Nm^/C, leaving the surface. P.D. across 2 pF capacitor,


11. (c) The Q charge on a capacitor indicates that the V,=-!-
q _12pC = 6V.
1
charges on its plates are +Q and -Q. Cl 2pF

ifl'YPE C : Assertions and Reasons


DIRECTIONS

In the following questions {1-11), a statement of assertion (A) is followed by a statement of reason (R).
Mark the correct choice as :

(g) If both assertion and reason are true and reason is the correct explanation of the assertion.
(6) If both assertion and reason are true but reason is not the correct explanation of the assertion,
(c) If assertion is true but reason is false. (d) If both assertion and reason are false.

1. Assertion. Electric potential of earth is taken zero. 5. Assertion. When charges are shared between any
Reason. No electric field exists on earth surface. two bodies, no charge is really lost and some loss of
[AIIMS 2009] energy does occur.
2. Assertion. Work done in moving a charge between Reason. Some energy disappears in the form of heat,
any two points in a uniform electric field is independent sparking etc. [AIIMS 2014]
of the path followed by the charge, between these 6. Assertion. A spherical equipotential surface is not
points. possible for a point charge.
Reason. Electrostatic forces are not conservative.
Reason. A spherical equipotential surface is not
[AIIMS 13]
possible inside a spherical capacitor. [AIIMS 15]
3. Assertion. A metallic shield in the form of a hollow 7. Assertion Lines of force are perpendicular to
shell may be built to block an electric field. conductor surface.

Reason. In a hollow spherical shield, the electric field Reason. Generally electric field is perpendicular to
inside it is zero at every point. [ARMS 12] equipontential surface. [ARMS 16]
4. Assertion. Dielectric polarisation means formation 8. Assertion If a dielectric is placed in external field,
of positive and negative charges inside the dielectric. then field inside dielectric will be less than applied field.
Reason. Free electrons are formed in this process. Reason. Electric field will induce dipole moment
[AIIM 2014] opposite to field direction. [AIIMS 17]
ELECTROSTATIC POTENTIAL AND CAPACITANCE 2.119 M

9. Assertion Charge never flows from a condenser of Reason. In polar dielectrics, each molecule has a
higher capacity to the condenser of lower capacity. permanent dipole moment but these are randomly
Reason. Flow of charge between two bodies oriented in the absence of an externally applied electric
field. [AIIMS 18]
connected by a thin wire is determined by the charges on
them. (AUMS 18] 12. Assertion. An electron has a high potential energy
when it is at a location associated with a more negative
10. Assertion The force between the plates of a parallel
plate capacitor is proportional to charge on it. value of potential, and a low potential energy when at a
location associated with a more positive potential.
Reason. Electric force is equal to charge per unit area.
[AIIMS 18] Reason (R). Electrons move from a region of higher
potential to region of lower potential. (CBSE SP 22]
11. Assertion In the absence of an externally applied
electric field, the displacement per unit volume of a polar
dielectric material is always zero.

w
Answers T-7

Flo
1. (c) Electric potential of the earth is taken zero 7. (fl) Both assertion and reason are true and reason is
because its capacitance C is very large and so, correct explanation of the assertion.

ee
8. (c) External field induces an electric field in
V = — _> 0 for all finite charges.
C the dielectric in the opposite direction which sets up a

Fr
2. (c) Electrostatic forces are conservative. Work done dipole moment in the same direction of external field.
in moving a charge in a uniform electric field is path 9. (d) Charges always flow from higher potential
independent. Assertion is true but reason is false. for
to lower potential. Both assertion and reason are false.
ur
3. (fl) Both assertion and reason are true and reason is 10. (c) F=I-qE =5> F ccq
the correct explanation of the assertion.
ks
4. (rf) Polarisationinduces +ve and -ve charges on the F cannot be equal to charge per unit area.
Yo

two opposite faces of the dielectric. No free electrons are .●. Assertion is true but the reason is false.
oo

formed in the process. Both assertion and reason are 11. (fl) Both assertion and reason are true and the
eB

false.
reason is correct explanation of the assertion.
5. (<j) Charge is always conserved but some energy is 12. (c) Assertion is true but reason is false. An
lost in the form of heat. Both assertion and reason are true.
electron moves from a region of low potential to high
r

6. (d) Both assertion and reason are false. potential.


ou
ad
Y

Text Based Exercises


nd
Re
Fi

^YPE A : Very Short Answer Questions (i mark each)


1. Define electric potential. Is it a scalar or a vector 7. What do you mean by a potential difference of 1 volt ?
quantity ? [Punjab 01; CBSE OD 06] 8. Write the dimensional formula of potential difference.
2. Define the unit of electric potential. [Punjab 02] 9. 5 J of work is done in moving a positive charge of
3. Write down the relation between electric field and 0.5 C between two points. What is the potential
difference between these two points ? [ISCE 95]
electric potential at a point.
10. A charge of 2 C moves between two points main
4. Name the physical quantity whose SI, unit is JC*’. tained at a potential difference of 1 volt. What is the
Is it a scalar or a vector quantity ? [CBSE OD 2010]
energy acquired by the charge ? [CBSE D IOC]
5. Write the SI unit of potential gradient. 11. In a conductor, a point P is at a higher potential
6. Define electric potential difference between two than another point Q. In which direction do the
points. Is it scalar or vector ? [Punjab 01] electrons move ?
2.120 PHYSICS-Xll

i 12. Give two examples of conservative forces. 33. The middle point of a conductor is earthed and its
13. How much is the electric potential of a charge at a ends are maintained at a potential difference of
point at infinity ? 220 V. What is the potential at the ends and at the
14. What is the nature of symmetry of the potential of a middle point ?
point charge ? 34. Define capacitance of a conductor.
15. What are the points at which electric potential of a 35. Can there be a potential difference between two
dipole has maximum value ? conductors of same volume carrying equal positive
charges ?
16. What are the points at which electric potential of a
dipole has a minimum value ? 36. The capacitance of a conductor is 1 farad. What do
17. What is the nature of symmetry of a dipole you mean by this statement ?
potential ? 37. What is a capacitor ?
18. What is electrostatic potential energy ? Where does 38. Write the physical quantity that has its unit
it reside ? coulomb volf^ Is it a vector or scalar quantity ?
19. What is the value of the angle between the vectors ICBSE D 93C, 98]
39. Define capacitance. Give its SI unit. [CBSE D 93C)
p and E for which the potential energy of an electric
40. Define the SI unit of capacitance. [CBSE 20C]
dipole of dipole moment p, kept in an external 41. Write the dimensions of capacitance.
electric field £ , has maximum value ? 42. What is the net charge on a charged capacitor ?
(CBSE SP 151
43. On what factors does the capacitance of a capacitor
20. Write an expression for potential at point ) depend ?
due to two charges f^j and c\.^ located at positions 44. Write two applications of capacitors in electrical
r j and ^2 respectively. circuits.

45. In what form is the energy stored in a cliarged


21.
Define electron volt. How is it related to joule ? capacitor ?
22.
How many electron volts make up one joule ? 46. What is the basic purpose of using a capacitor ?
23.
Will there be any effect on the potential at a point if 47. Write different expressions for the energy stored in
the medium around this point is changed ? a capacitor.
24.
What work must be done in carrying an a-particle 48. Write down the expression for the capacitance of a
across a potential difference of 1 volt ? spherical capacitor.
25.
What is an equipotential surface ? Give an example. 49. The difference between the radii of the two spheres
[Punjab 2000, 02 ; CBSE D 03] of a spherical capacitor is increased. Will the capa
citance increase or decrease ?
26. Why are electric field lines perpendicular at a point
on an equipotential surface of a conductor ? 50. What is a dielectric ?

[CBSE OD ISC] 51. Define dielectric constant in terms of the capa


27. Can you say that the earth is an equipotential citance of a capacitor. [CBSE D 06]
surface ? 52. Write down the relation between dielectric constant
28. What is the geometrical shape of equipotential and electric susceptibility.
surfaces due to a single isolated charge ? 53. Write a relation for polarisation P of a dielectric
[CBSE D13]
material in the presence of an external electric field
29. What is the shape of the equipotential surfaces for a
uniform electric field ? £.
(CBSE OD 15]
30. How much work is done in moving a 500 pC charge 54. Define dielectric strength of a medium. What is its
between two points on an equipotential surface ? value for vacuum ?
[CBSE D 02] 55. Where is the knowledge of dielectric strength helpful ?
31. A charge of + 1C is placed at the centre of a spherical 56. What is the effect of temperature on dielectric
shell of radius 10 cm. What will be the work done in constant ?
moving a charge of +1 pC on its surface through a
distance of 5 cm ? 57. An air capacitor is given a charge of 2 pC raising its
potential to 200 V. If on inserting a dielectric
32. What is the optical analogue of an equipotential medium, its potential falls to 50 V, what is the
surface ?
dielectric constant of the medium ?
ELECTROSTATIC POTENTIAL AND CAPACITANCE 2A21

58. An uncharged insulated conductor A is brought 73. In which orientation, a dipole placed in a uniform
near a charged insulated conductor B. What happens electric field is in (i) stable, (//) unstable equili
to the charge and potential of B ? [CBSE OD OIC) brium ? [CBSE OD 08; D 10]
59. For a given potential difference, does a capacitor 74. Write the expression for the work done on an
store more or less charge with a dielectric than it
does without a dielectric ?
electric dipole of dipole moment ^ in turning it
from its position of stable equilibrium to a position
60. Can we place a parallel plate capacitor of 1 F
capacity in our house ? of unstable equilibrium in a uniform electric field £.
61. What is the basic difference between a capacitor [CBSE D13C]
and an electric cell ? 75. Draw equipotential surfaces due to an electric
62. Two capacitors of capacitances Q and are dipole. Or (CBSE D 20]
connected in parallel. A charge q is given to the Two charges 2 ^iC and -2 )iC are placed at points A
combination. What will be the potential difference and B, 5 cm apart. Depict an equipotential surface

w
across each capacitor ? of the system. [CBSE D13C]
63. What is the order of capacitances used in a radio 76. What is the equivalent capacitance, C, of the five
receiver ? capacitors, connected as shown in Fig. 2.214 ?

Flo
64. Is there any conductor which can take unlimited c, C3
charge ? tHHI-HI

ee
65. A parallel plate capacitor with air between the C4 C5

Fr
plates has a capacitance of 8 pF. What will be the
capacitance if the distance between the plates be h
reduced by half and the space between them is
for
ur
filled with a substance of dielectric constant k = 6 ?

[CBSE D 05)
Fig. 2.214 [CBSE SP2011]
A
66. A 500 |iC charge is at the centre of a square of side 71. A charge is moved
ks
10 cm. Find the work done in moving a charge of from a point A above a
Yo

10 pC between two diagonally opposite points on dipole of dipole moment


oo

the square. [CBSE D 08] 'p' to a point B below the -‘7


dipole in equatorial plane
eB

67. The graph of Fig. 2.213,


without acceleration.
shows the variation of the B
Find the work done in the
total energy (£) stored in a
process. [CBSE OD 16] Fig. 2.215
r

capacitor against the value t


ou

of the capacitance (Q itself. 78. What is the amount of work done in moving a
ad

UJ

Which of the two the charge charge Q around a circular arc of radius r at the
Y

on the capacitor or the centre of which another point charge q is located ?


potential used to charge it [CBSE OD13C 16]
nd

is kept constant for this 79. A point charge Q is placed at point O as shown in
Re

graph ? [CBSE SP 08] Fig. 2.213


the figure 2.216. The potential difference V^~V B
Fi

68. Define the term 'potential energy' of charge '</' at a positive. Is the charge Q negative or positive ?
distance V in an external electric field. [CBSE OD 09] Q0. -●
o A B
69. What is the work done in moving a test charge q
through a distance of 1 cm along the equatorial axis Fig. 2.216 [CBSE F 16]
of an electric dipole ? [CBSE OD 09]
80. A point charge Q is placed at point 'O' as shown in
70. What is the electrostatic potential due to an electric
dipole at an equatorial point ? (CBSE OD 09]
Fig. 2.216. Is the potential at point A, i.e., V^, greater,
smaller or equal to potential, Vg, at point B, when Q
71. A metal plate is introduced between the plates of a is (i) positive, and {ii) negative charge ? [CBSE F 17]
charged parallel plate capacitor. What is its effect on
81. An electron is accelerated through a potential
the capacitance of the capacitor ? [CBSE F 09]
difference V. Write the expression for its final
72. A hollow metal sphere of radius 5 cm is charged speed, if it was initially at rest. [CBSE 18C]
such that the potential on its surface is 10 V. What is ->

the potential at the centre of the sphere ? 82. State the SI unit of electric polarization vector P.
[CBSE OD 11} [CBSE SP 18]
s 2:122

Answers

1. Refer to point 3 of Glimpses.


PHYSICS-XII

25. Any surface which has same electric potential at


2. Refer to point 4 of Glimpses. every point is called an equipotential surface. The


dV surface of a charged conductor is an equipotential
3. £ =- surface.
dr

4. Electric potential or potential difference. It is a 26. If it were not so, the presence of a component of the
scalar quantity. field along the surface would destroy its equi
potential nature.
5. SI unit of potential gradient = Vm“\ 27. Yes. Earth is a conductor, so its surface is equi
6. Refer to point 1 of Glimpses. potential.
7, Refer to point 2 of Glimpses. 28. For a point charge, the equipotential surfaces are
8. Potential difference concentric spherical shells with their centre at the
work done ML^T point charge.
charge C
29. For a uniform electric field, the equipotential
ML^T'^ surfaces are parallel planes perpendicular to the
AT direction of the electric field.
W 5J 30. Zero. W = ijAV^=500pCxO = 0.
9. V^=—= = 10 V.
q 0.5 C 31. Zero. This is because the surface of the spherical
shell will be an equipotential su: ■hce.
10. Energy acquired by the charge
32. Wavefront.
= i/V=2CxlV = 2J.
11. From Q to P. 33. The potential at the middle point of the conductor
is zero and that at the ends + 110 V and - 110 V, so
12. (0 Electrostatic force, (a) Gravitational force.
that the p.d. at ends = 110 -(- 110) = 220 V.
13. Zero.
34. The capacitance of a conductor may be defined as
14. The potential of a point charge is spherically the charge required to raise its potential by unit amount.
symmetric. 35. Yes. Two conductors of same volume but of
15. At axial points, the electric potential of a dipole has different shapes will have different capacitances.
a maximum positive or negative value. 36. A conductor is said to have a capacitance of 1 farad,
16. At equatorial points, the electric potential of a if 1 coulomb of charge increases its electric potential
dipole is zero. through 1 volt.
17. The dipole potential is cylindrically symmetric. 37. A capacitor is a device to store electric charge. It
18. The electrostatic potential energy of a system of consists of two conducting plates separated by an
charges may be defined as the work required to be insulatingmedium.
done to bring the various charges to their respective 38. Capacitance has its unit coulomb volt“^ It is a
positions from infinity. scalar quantity.
19. P.E. =-p£cos6. Clearly, P.E. is maximum when 39. The capacitance of a capacitor may be defined as
cos6 = -l or 0 = 180°.
the charge required to be supplied to either of the
conductors so as to increase the potential difference
20. V^(?^) = T^ <h
between them by unit amount.
\r -r J |r -r ^ 40. The SI unit of capacitance is farad (F). A capacitor
has a capacitance of 1 F if 1 coulomb of charge is
21. Electron volt is the potential energy gained or lost
trai\sferred from its one plate to another on
by an electron in moving through a potential
difference of one volt.
applying a potential difference of 1 volt across the
,-19 two plates.
1 electron volt = leV = 1.6 x 10 ]
41. AslF =
1C _ 1C _ IC^ _ l(As)^
22. IJ =6.25xl0^^eV. IV IJ/C IJ INm
23. Yes. If the dielectric constant of the medium is A^T^
increased, the electric potential will decrease. [Capacitance] =
MLT"\
24. W = q^V=2eAV
= 3.2 X10
-19
CxlV=3.2xlO
-19
J. = [M-^L'^T^A^].
ELECTROSTATIC POTENTIAL AND CAPACITANCE 2.123

42. Zero, because the two plates have equal and 60. No. If d = 1 an = 10 ^ m, then area of such a capa
opposite charges. citor would be

43. The capacitance of a capacitor depends on the 1 F X 10"^ m


= 10'm2
geometry of the plates, distance between them and So as5 X10
-12
c2N"’m"2
the nature of the dielectric medium between them,
This is a plate about 30 km in length and breadth.
44. (i) Capacitors are used in radio circuits for tuning
61. A capacitor provides electrical energy stored in it.
purposes.
A cell provides electrical energy by converting
(//) Capacitors are used in power supplies for
chemical energy into electrical energy.
smoothening the rectified current.
62. lu parallel combination, potential difference is same
45. In a charged capacitor, energy is stored in the form
aaoss each capacitor.
of electrostatic potential energy in the electric field
between its plates. Net capacitance, C = (^ +
46. To store charge and electric energy.

w
.●. P.D. across each capacitor, V = — =
c q + q
47. a = ^cy2 63. In tlie power supply, it is 1 - 10 pF and for tuning

Flo
1 1
= -QC purposes, it is 100 pF.
2 V 2 ^ 64, Yes, the earth because of its large capacitance can
take unlimited charge.

ee
ab
48. C = 4toq. b -a
, where a and b are the radii of the
65 With air between the capacitor plates.

Fr
inner and outer spheres respectively.
49. The capacitance will increase.
With dielectric between the capacitor plates,
for
ur
50. A dielectric is essentially an insulator which allows
electric induction to take place through it but does C = K = 2 <q = 2x6x8 = 96 pF.
not permit the flow of charges through it. dl2
ks
51. The ratio of the capacitance (Q) of the capacitor 66. The work done in moving a charge of 10 pC
Yo

completely filled with the dielectric material to the between two diagonally opposite points on the
oo

capacitance (q,) of the same capacitor with vacuum square will be zero because these two points will be
between its plates is called dielectric constant. equipotential.
eB

67. Energy stored, ^ ^ "


1
Q
K = —
C 2 C
r

52 K = 1 + X/ where k is dielectric constant and x is When Q is constant, T oc , and we get a graph of


ou
ad

electric susceptibility. the type given in the question.


Y

53. P = EoX, £ ■ Hence the charge Qon the capacitor is kept constant.
54. The maximum value of electric field that can exist 68. The potential energy of a charge q is the work done
nd
Re

inside a dielectric without causing its electrical in bringing charge q from infinity to the position r*
breakdown is called its dielectric strength. The in the external electric field.
Fi

dielectric strength for vacuum is infinity.


55. The knowledge of dielectric strength helps in
designing a capacitor by determining the maximum 69. As potential at any point on the equatorial axis of
potential that can be applied aaoss the capacitor an electric dipole is zero, so
without causing its electrical breakdown. W=^AV=(?(0-0)= 0.
56. The value of dielectric constant deaeases with the
70. Zero.
inaease of temperature.
71. The introduction of a metal sheet of thickness t in a
57. K = vacuum ^ ,?00 ^ ^ parallel plate capacitor increases its capacitance by
V 50
’^dielectric a factor of
d-t
, where d is the plate separation of
58. The charge on the conductor B remains unchanged
but its potential gets lowered. the capacitor.
72. Potential at the centre = Potential at the surface
59. A capacitor with a dielectric has a higher capaci
= 10V.
tance and hence stores more charge.
2:124 PHYSICS-XII

73. (i) When the dipole moment p is parallel to the the 78. Zero, because all points of the circular arc will be at
the same potential.
electric field £(9 = 0°), the dipole is in stable
equilibrium, 79. = Q J L
4tcEo . OA OB,
(ii) When the dipole moment p is antiparallel to the
electric field £(9= 180°), the dipole is in unstable V^-V^>0 and _l
OA
l_
OB
>0,

equilibrium.
so the charge Q must be positive.
74. W = p£(cos0j -COS62) = p£(cos0°-cosl80°) 80. (i)V^>Vs {ii)V^<V^.
= p£(l+l) = 2pE. 81. Gain in K.E. = Work done on electron

75. See Fig. 2.26 on page 2.15. 1 2 =eV1/


-mv
76. C = (^, because the combinations of and Cj as 2

well as C4 and have been shorted. [2^


y =
77. No work is done. Potential at equatorial points of m

the dipole is zero. 82. Cm "2.


W = qVAB = </{0-0) = 0.

■ Type B : Short Answer Questions (2 marks each)


1. Define electric potential. Derive an expression for Write the two important properties concerning the
the electric potential at a point at distance r ft'om a relation between the electric field and electric
charge q. [CBSE D 16C] potentials. [CBSE D I4C 17C]
3. Draw a plot showing the variation of (i) electric 8. Show that the amount of work done in moving a
field (£) and (ii) electric potential (V') with distance r test charge over an equipotential surface is zero.
due to a point charge Q. [CBSE D 12] 9. Show that the direction of the electric field is normal

3. Derive an expression for the electric potential at to the equipotential surface at every point.
any point along the axial line of an electric dipole. 10. Show that the equipotential surfaces are closed
(CBSE OD 13C ; D 17,19) together in the regions of strong field and far apart
4. Show mathematically that the electric potential at in the region of weak field.
any equatorial point of an electric dipole is zero. 11. Two point charges and are located at (0,0,-a)
[CBSE OD Oil and (0, 0, a) respectively,
5. Derive the expression for the electric potential at (fl) Depict the equipotential surfaces due to this
any point P, at distance r from the centre of an arrangement,
electric dipole, making angle 9, with its axis. (b) Find the amount of work done in moving a
[CBSE OD 17C] small test charge i^Q from point (/, 0, 0) to (0,0,1).
6. Show that the electric field at any point is equal to [CBSE 20C]
the negative of the potential gradient at that point. 12. Deduce the expression for the potential energy of a
7. Two closely spaced equipotential surfaces A and B system of two charges q^ and ^2 located at r\ and
with potentials V and V + 5V, (where 5V is the
change in V), are kept 81 distance apart as shown in respectively in an external electric field.
[CBSE OD 15]
the figure. Deduce the relation between the electric
field and the potential gradient between them. 13. Obtain the expression for potential energy of an
electric dipole placed with its axis at an angle 9 to
an external field £ . What is the minimum value of
■V
the potential energy. [CBSE 19C]
14. Using Gauss's law, show that electric field inside a
conductor is zero. [CBSE D 2000]
5/
'A 15. Just outside a conductor electric field is perpen
dicular to the surface. Give reason.
Fig. 2.217
ELECTROSTATIC POTENTIAL AND CAPACITANCE

16. Show that the excess charge on a conductor resides 1.0 m, as shown in
only on its surface. Fig. 2.218. What is the
17. Show that the electric field at the surface of a value of charge
placed on the same
charged conductor is given by E = —n, where a is circle if the potential
%
at centre, =
the surface charge density and » is a unit vector [ISC 03]
normal to the surface in the outward direction.
28. Two thin concentric shells of radii q and (J2 > ^i)
Or [CBSE OD 10]
have charges and </2. Write the expression for the
Derive an expression for the electric field at the potential at the surface of inner and outer shells.
surface of a charged conductor. [CBSE OD 09] [CBSE OD 13 C]
18. Show that electric field is zero in the cavity of 29. If two charged conductors are touched mutually
hollow charged conductor.

w
and then separated, prove that the charges on them
19. What is electrostatic shielding ? Mention its two will be divided in the ratio of their capacitances.
applications. 30. Define 'dielectric constant' of a medium. Briefly

Flo
20. Define electrical capacitance of a conductor. On explain why the capacitance of a parallel plate capa
what factors does it depend ? citor increases, on introducing a dielectric medium
between the plates. [CBSE OD 06C]

ee
21. Show that the capacitance of a spherical conductor
is proportional to its radius. Hence justify that farad 31. What is meant by dielectric polarisation ? Hence

Fr
is a large unit of capacitance. establish the relation : k = 1 + x [Haiyana 01]
22. An isolated conductor cannot have a large capa 32. Two dielectric slabs of dielectric constants Kj and K2
citance. Why ? are filled in between the two plates, each of area A,
for
ur
23. Wiry does the capacitance of a conductor increase, of the parallel plate capacitor as shown in Fig. 2.219.
Find the net capacitance of the capacitor.
when an earth connected conductor is brought near
ks
it ? Briefly explain.
Yo

24. What is a capacitor ? Explain its principle.


oo

1 ^^2 d
(Punjab 031
eB

25. Derive an expression for the capacitance of a parallel Fig. 2.219


plate capacitor. [CBSE D 05C, 14 ; OD 03] IP. ■*ih IP

26. Distinguish between polar and non-polar dielec


r

trics. Give one example of each. 33. Draw equipotential surfaces for (/) an electric
ou
ad

dipole and (ii) two identical positive charges placed


27. The charges q^ =3 pF, i/., = 4 pF and ijg = - 7 pF are near each other. [CBSE SP 23]
Y

placed on the circumference of a circle of radius


nd
Re

Answers ▼
Fi

1. Refer answer to Q. 2 on page 2.2. 8. Refer answer to Q. 14 on page 2.14.


2. See Fig. 2.3 on page 2.2. 9. Refer answer to Q. 14 on page 2.14.
3. ● Refer answer to Q. 3 on page 2.2. 10. Refer answer to Q. 14 on page 2.14.
4. Refer answer to Q. 4 on page 2.3. 11. ((7) See Fig. 2.26 on page 2.15.
5. Refer answer to Q. 5 on page 2.3. (&) The test charge q^ moves along x-axis which is
6. Refer answer to Q. 10 on page 2.11. on equipotential line for the given charge
system, so AV = 0
7. Work done in moving a unit positive charge
through distance S/, W = q^AV = 0.

* Ex5l = Vy^-Vg = V-iV + bV) = -5V


12. Refer answer to Q. 20 on page 2.17.
8V
13. Refer answer to Q. 22 on page 2.18.
E = -
5/
14. Refer answer to Q. 25 on page 2.24.
For properties, refer answer to Q. 10 on page 2.11.
15. Refer answer to Q. 25 on page 2.24.
s 2,126

16.

17.

18.
Refer
Refer
answer to
answer to
Q.
Q.
25
25
on
on
page
page
2.24.
2.24.
PHYSICS-XII

Potential at the surface of outer shell

= Potential due to charge on inner shell


Refer answer to Q. 25 on page 2.24. + Potential due to charge ^2 on outer shell
19.
Refer answer to Q. 26 on page 2.25.
20. Refer answer to Q. 27 on page 2.26.
L k+h
21.
4nSol^r2 ^2j
Refer answer to Q. 29 on page 2.26.
29. Refer answer to Q. 41 on page 2.48.
22.
Refer answer to Q. 30 on page 2.28.
30. Refer to point 35 of Glimpses.
23.
Refer answer to Q. 31 on page 2.28.
31. Refer answer to Q. 47 on page 2.57.
24.
Refer answer to Q. 31 on page 2.28.
32. The given arrangement is equivalent to a parallel
25. Refer answer to Q. 33 on page 2.29.
26. combination of two capacitors each with area A/2
Refer answer to Q. 43 on page 2.55.
27.
and plate separation d.
As = 0
Hence the net capacitance of the resulting capacitor
^ k + k + k + k. = 0
IS

47180 [ r r r r
c=q + q
or

or
3+4—7+<y^ = 0 or
<74=0-
^eo(A/2)iq ^ 8q(A/2)k2
d d
28. Potential at the surface of inner shell,
_ Sq A(k^ +K2)
q = Potential due to its own charge q^ d
+ Potential due to charge q2 on outer shell 33. (i) See Fig. 2.26 on page 2.15.
k+k (if) See Fig. 2.27 on page 2.15.
47180 I ^2)

^YPE C : Long Answer Questions I (3 marks each)


1. 5.
(fl) Obtain an expression for the electric potential Define the term electric dipole moment. Derive an
at a point due to group of N point charges. expression for the total work done in rotating
(b) Give two differences between the nature of the dipole through an angle 0 in uniform electric
electric potentials of a single point charge and field £.

an electric dipole, 6.
Derive an expression for the potential energy of an
2.
(fl) Derive an expression for the electric potential electric dipole placed in a uniform electric field.
at any point along the axial line of an electric Hence discuss the conditions of its stable and
dipole. unstable equilibrium. [CBSE D 08j
7. An electric dipole is held in a uniform electric field
(b) Draw the equipotential surfaces corresponding
to a uniform electric field in the 2-direction.
E. (a) Show that the net force acting on it is zero.
(CBSE D 19]
3.
(b) The dipole is aligned with its dipole moment ^
Define an equipotential surface. Draw equipoten
tial surfaces : (tj) in the case of a single point charge, parallel to the field £.
and {b) in a constant electric field in Z-direction. Find : (/) the work done in turning the dipole till its
Why the equipotential surfaces about a single
dipole moment points in the direction opposite to E
charge are not equidistant ? (c) Can electric field
.

exist tangential to an equipotential surface ? Give (ii) the orientation of the dipole for which the torque
reason. [CBSE OD 16] acting on it becomes maximum. [CBSE OD 12,14C]
4.
8. Sketch equipotentialsurfaces for
(zz) Two point charges q^ and q^ are kept at a
(/) a positive point charge, [CBSE D 2000]
distance of ^r2 ^ Deduce the expression for
the electrostatic potential energy of this system. (iz) a negative point charge. [CBSE D 01]
(z7z) two equal and opposite charges separated by a
(?7) If an external electric field (£) is applied on the small distance.
system, write the expression for the total
{iv) two equal and positive charges separated by a
energy of this system. [CBSE D 20]
small distance. [CBSE OD 15]
ELECTROSTATIC POTENTIAL AND CAPACITANCE 2.121

9. A charge Qis distributed uniformly over a metallic 14. (a) Explain briefly, using a proper diagram, the
sphere of radius R. Obtain the expressions for the difference in behaviour of a conductor and a
electric field (E) and electric potential (K) at a point dielectric in the presence of external electric
0<x < R. Show on a plot the variation of £ and V field,
with xtor 0<x <2R. [CBSE F 17] (b) Define the term polarization of a dielectric and
10. A long charged cylinder of linear charge density write the expression for a linear isotropic
+ Xj is surrounded by a hollow coaxial conducting dielectric in terms of electric field.
cylinder of linear charge density -X2. Use [CBSE D 15 ; OD 19]
Gauss's law to obtain expression for the electric 15. Deduce the expression for the electrostatic energy
field at a point (i) in the space between the stored in a capacitor of capacitance 'C' and
cylinders, and (n) outside the larger cylinder. having charge 'Q'. How will the (i) energy stored and
[CBSE F 171 (ii) the electric field inside the capacitor be affected
11. Obtain the expressions for the resultant capacitance when it is completely filled with a dielectric

w
when the three capacitors C^, and are material of dielectric constant 'td ?
connected (1) in series and then {ii) in parallel. [CBSE D 14 ; OD 08,12]
[CBSE OD 19C] 16. A capacitor is charged with a battery and then its

Flo
12. Two capacitors with capacity Q and are charged plate separation is increased without disconnecting
to potentials Vj and respectively and then the battery.

ee
connected in parallel. Calculate the common What will be the change in
potential across the combination, the charge on

Fr
(fl) charge stored in the capacitor ?
each capacitor, the electrostatic energy stored in the {b) energy stored in the capacitor ?
system and the change in the electrostatic energy
(c) potential difference across the plates of the
from its initial value. [CBSE Sample Paper 08] for
ur
capacitor ?
13. Explain how the polarization of a dielectric reduces (d) electric field between the plates of the
the electric field inside the dielectric. Hence define [CBSE SPll]
capacitor ?
ks
dielectric constant. [CBSE D 99]
Yo
oo

Answers T
eB

1. (fl) Refer answer to Q. 7 on page 2.4. 6. Refer answer to Q. 22 on page 2.18.


7. (rt) Refer answer to Q. 40 on page 1.40 of chapter 1.
r

(b) Refer answer to Q. 6 on page 2.3,


ou

ju n
ad

2. («) Refer answer to Q. 3 on page 2.2. (1?) (/) W = rd0 = p£sin 0 = p£[-cos 0] q = -2pE.
(b) See Fig. 2.26 on page 2.15.
Y

0 0

3. {a) See Fig. 2.25 on page 2.15. (ii) As X = p£sin 0, so x is maximum when 0 = 90°.
(b) See Fig. 2.211 on page 2.102. 8. Refer answer to Q. 15 on page 2.15.
nd
Re

dV dV
As E =- or dr = - 9. By Gauss's theorem,
Fi

dr E
■£ “
E.dS=^ Gaussian
surface S
1 2
For constant dV, dr<x—ccr s
E

Hence the spacing between the equipotential


surfaces will increase with the increase in distance
from the point charge,
(c) No. If the field lines are tangential, work will be
done in moving a test charge on the surface. But no
work is needed in moving a test charge on an
equipotential surface.
4. (fl) Refer answer to Q. 18 on page 2.16, Surface charge
density cr
(b) Refer answer to Q. 20 on page 2.17.
5. Refer answer to Q. 22 on page 2.18. Fig. 2.220
.128 PHYSICS-XII

For Q<x < R, q = 0


£ = 0
or
^ X 27ir2/ =
^0
dV
But £ =
dr
2ner2 ^
.●. V = constant =
1 Q
47T6n0 R 11. (/) Refer answer to Q. 36 on page 2.33.
(n) Refer answer to Q. 37 on page 2.34.
£, V,
V = constant
12. Refer answer to Q. 42 on page 2,48.
13. Refer answer to Q. 45 on page 2.56.
Vccl 14. Refer answer to Q. 43 on page 2.55.
V r

Eli
1
Polarisation of a dielectric. The induced dipole
£ = 0 moment set up per unit volume of a dielectric when
O R 2R r O R IR r placed in an external electric field is called
polarisation. For linear isotropic dielectrics,
Fig. 2.221

10. By Gauss's law, (fig = >£.dS = — where is the electric susceptibility of the dielectric
^0 medium.

(0 In the space between the two cylinders, field is 15. Refer answer to Q. 38 on page 2.47.
due to charge on inner cylinder only. When the capacitor is completely filled with a
+ 1 — Xt dielectric material and for constant charge Q,
'£ = E^.ds

Si C = kCJ, and V = Vq/ k
N
/

or ^ x27irj/ = fv.V
(0 u = tcv'2=i(Kq,)
t 0
s

So K
^ h '
+ X.J A
K a0
1
iTKn,T
O'i K
+ +
Gaussian

+
surface S 1
+
(//) H=
K
+ +

-Gaussian
4 4 16. C= . When d is increased, C decreases.
surface Sj d

(fl) q = CV decreases due to the decrease in the


\
■'I 1
I
S
●s / value of C.

Fig. 2.222
{h) U = ^CV^ decreases due to the decrease in the
(n) In the region outside the larger cylinder, value of C

(c) V remains unchanged because the battery


remains connected.
S2 ^0
(d) E=V / d decreases due to the increase in the
value of d.

^^YPE D : Long Answer Questions II (5 marks each)

1. Find the expression for the electric field intensity 2. (1?) Obtain the expression for the potential due to a
and the electric potential, due to a dipole at a point point charge.
on the equatorial line. Would the electric field be (b) Use the above expression to show that the
necessarily zero at a point where the electric potential, due to an electric dipole (length 2a),
potential is zero ? Give an example to illustrate varies as the 'inverse square' of the distance r
your answer. [CBSE SP2011] of the 'field point' from the centre of the dipole
forr»fl. (CBSED16C]
ELECTROSTATIC POTENTIAL AND CAPACITANCE 2.129

3. (a) Two point charges and f/j are r distance {ii) If two similar large plates, each of area A having
apart in a uniform external electric field E. surface charge densities +crand -a are separated
by a distance d in air, find the expressions for
Find the amount of work done in assembling (fl) field at points between the two plates and on
this system of charges. outer side of the plates. Specify the direction of the
{b) A cube of side 20 cm is kept in a region as field in each case {b) the potential difference
shown in Fig. 2.223. An electric field E exists in between the plates (c) the capacitance of the
capacitor so formed.
the region such that the potential at a point is
given hy V = lOx + 5, 8. Obtain the expression for the capacitance of a
where V is in volt
parallel plate capacitor. Three capacitors of
and X is in m.
capacitances C^, Cj and are connected (/) in
Find the (/) electric series, (») in parallel. Show that the energy stored
in the series combination is the same as that in the

w
field £, and (jj) total
parallel combination. (CBSE OD 03]
electric
the cube.
flux through ^ 9. (fl) Derive the expression for the energy stored in
Fig. 2.223 a parallel plate capacitor. Hence obtain the

Flo
[CBSE OD 20]
4. {a) Derive an expression for the potential energy expression for the energy density of the electric
field.
of an electric dipole in a uniform electric field.

ee
Explain conditions for stable and unstable (b) A fully charged parallel plate capacitor is

Fr
equilibrium. connected across an uncharged identical
(b) Is the electrostatic potential necessarily zero at capacitor. Show that the energy stored in the
a point where the electric field is zero ? Give an combination is less than that stored initially in
for
ur
example to support your answer. [CBSE OD 19) the single capacitor. [CBSE OD 1.5]

5. (fl) Explain why, for any charge configuration, the 10, Define the terms (i) capacitance of a capacitor
equipotential surface through a point is normal (ii) dielectric strength of a dielectric. When a dielectric
ks
to the electric field at that point. is inserted between the plates of a charged parallel
Yo

Draw a sketch of equipotential surfaces due to plate capacitor, fully occupying the intervening
oo

a single charge (-(/), depicting the electric field region, how does the polarization of the dielectric
eB

lines due to the charge, ‘1 medium affect the net electric field ? For linear

(b) Obtain an expression dielectrics, show that the introduction of a


for the work done to dielectric increases its capacitance by a factor
r

dissociate the system characteristic of the dielectric. [CBSE D 08C]


ou
ad

of three charges placed 11. Find the expression for the capacitance of a para‘ el
at the vertices of an /
plate capacitor of area A and plate separation d if
Y

equilateral triangle of _4^ + 2^ (i) a dielectric slab of thickness t and {ii) a metallic
side 'a' as shown here. slab of thickness f, where {t < d) are introduced one
nd

Fig. 2.224 by one between the plates of the capacitor. In which


Re

[CBSE OD16]
6. {a) Define the capacitance of a capacitor. Obtain case would the capacitance be more and why ?
Fi

the expression for the capacitance of a parallel [CBSE SP 2011]


plate capacitor in vacuum in terms of plate 12. What is a dielectric ? A dielectric slab of thickness f
area A and separation d between the plates, is kept between the plates of a parallel plate
(b) A slab of material of dielectric constant k has capacitor separated by distance d. Derive the
the same area as the plates of a parallel plate expression for the capacitance of the capacitor for
3d t « d. [Himachal 02 ; CBSE D 93 ; OD OIC]
capacitor but has a thickness — . Find the ratio
13. (fl) A small sphere, of radius 'a', carrying a positive
of the capacitance with dielectric inside it to its charge q, is placed concentrically inside a larger
capacitance without the dielectric. [CBSE F 17] hollow conducting shell of radius b(b > a). This
7. (0 Distinguish, with the help of a suitable diagram, outer shell has a charge Qon it. Show that if these
the difference in the behaviour of a conductor and a spheres are connected by a conducting wire, charge
dielectric placed in an external electric field. How will always flow from the inner sphere to the outer
does polarised dielectric modify the original sphere, irrespective of the magnitude of the two
external field ? charges. [CBSE F 15]
[CBSE OD 16]
.130 PHYSICS-XII

(b) Name the machine which makes use of this (b) A capacitor of capacitance 1 pF is charged by
principle. Draw a simple labelled line diagram of this connecting a battery of negligible internal
machine. What 'practical difficulty' puts on upper resistance and emf 10 V across it. Calculate the
limit on tine maximum potential difference which amount of charge supplied by the battery in
this machine can built up ? [CBSE D 09C; OD 14] charging the capacitor fully. [CBSE F 20]
14. (a) Describe briefly the process of transferring the 16. (d) Consider a system of two parallel metal plates
charge between the two plates of a parallel of area 'A', each placed at a separation'd ' in
plate capacitor when connected to a battery, air. Derive the expression for the capacitance of
(fc) Derive an expression for the energy stored in a this parallel plate capacitor.
capacitor, (1>) If the two plates of the capacitor have + q and
(c) A parallel plate capacitor is charged by a -q charges, respectively, find the force expe
battery to a potential difference V. It is rienced by the negative plate due to the
disconnected from battery and then connected positive plate,
to another uncharged capacitor of the same (c) A network of four C3
capacitance. Calculate the ratio of the energy capacitors each of ~
stored in the combination to the initial energy capacitance 12 pF is c,
on the single capacitor. [CBSE OD 14; D 19] connected to a battery
15. (fl) Derive an expression for the energy stored in a as shown in the figure. 11
parallel plate capacitor of capacitance C when Find the total charge 100 V
charged up to voltage V. How is this energy stored in the network.
stored in the capacitor ? [CBSE 20C] Fig. 2.225

Answers

1. For derivation of electric field intensity at 4. (a) Refer answer to Q. 22 on page 2.18.
equatorial point of a dipole, refer answer to Q. 38 (1j) Refer answer to Q. 1 above.
on page 1.39 of chapter 1.
5. (a) Refer answer to
P_
Q. 14 on page 2.14. The
equipotential surfaces
For derivation of electric potential at an equatorial and the field lines of a — TT H
T7 Field line

point of a dipole, refer answer to Q. 4 on page 2.3. point charge -q are Equipotential
V.
equa
= 0 shown in the figure, surface

(fc) Work required to


No, the electric field may not be necessarily zero at
a point where the electric potential is zero. For dissociate the system of Fig. 2.226
three charges
example, the electric potential at an equatorial point of
a dipole is zero, while electric field is not zero. = - P.E. of the system
2. {a) Refer answer to Q. 2 on page 2.2.
1 t?x(-4<7) , i-^)x2q
{b) Refer answer to Q. 3 on page 2.2.
47180 - a a

V
We get.
4ji£o r^-a^ 1
10^
[-4q^ + 2q^-8q^] = + 47X8nrt
Ip T/ 1 47x80 0
For r»a, V = => V OC-y
4ti£o 6. (fl) Refer answer to Q. 33 on page 2.29.
3. (t?) Refer answer to Q. 20 on page 2.17. (b) Refer to the solution of Example 89 on page 2.63.
(b) (0 7 = 10x + 5 7. ((') See Fig. 2.122 on page 2.55.
dV
E =-
dx
= -10/. In a conductor, the induced field is exactly
equal and opposite to the external field The
(//) Total flux through the cube, net field inside the conductor is zero.

= £l-AS(-i ) + E^.ASl In a dielectric, the induced field opposes


but does not exactly cancel it. It only
= -10/ .(0.20)^(-/ ) -10/ .(0.20)2 '> ^ Q reduces it.
ELECTROSTATIC POTENTIAL AND CAPACITANCE 2.1S1

a a
(c) Initial energy stored in the single capacitor
(") («) = 2e 0 ^0
2 2 C
^ =0
Outside Capacitance of the combined (parallel) system
= C+C = 2C

As the total charge cj remains the same, so


+o -o

Final energy stored in the


1
^
^2 E2 £2- £i combined system
P 2 2C
P El
Final energy _ -1 = 1: 2.
Initial energy 2
Plate 1 Plate 2

15. (fl) Refer to answer to Q. 38 on page 2.47. When a


Fig. 2.227 capacitor is charged, electric charges are

w
od
transferred from its one plate to another. The
{b)V = Ed= — work done in charging the capacitor is stored
So as its electrical potential energy. This energy is

Flo
aA supplied by the battery,
0
d (fc) Amount of charge supplied by the battery,

ee
8. Refer answer to Q. 33 on page 2.29 and Q. 39 on q = cy = lixV^10W =10 ^iC.

Fr
page 2.47. 16. (a) Refer answer to Q. 33 on page 2.29.
9. (fl) Refer answer to Q. 38 and Q. 40 on page 2.47. (6) Electric field due to the positive plate at the
(b) Refer to the solution of Example 75 on page 2.52. location of negative plate,
for
ur
10. Refer answer to Q. 45 on page 2.56 and Q. 49 on E= —= ^
page 2.58. 2eq 2eqA
11, For derivation, refer answers to Q. 49 on page 2.58
Force experienced by the negative plate due to
ks
and Q. 50 on page 2.59.
positive plate.
Yo

0
oo

c
^dielectric
~
t F =
iT
d —t + ^ 2e.A
0
eB

Cmetal [For metal, k = 00] The negative sign shows that the force is
d-t attractive,
r

are connected in
Clearly, ^metal ^ Qielectcic (c) Capacitors C2' ^ ^4
ou
ad

12. Refer answer to Q. 49 on page 2.58. series.

13. (fl) Refer answer to Q. 55 on page 2.66. 1 _ 1 1 1^1 = 1


Y

(b) A Van-de-Graaff generator works on this ^34 ^ ^ ^4■ 12 12 12 4

principle. See Fig. 2.136. The potential on the outer


nd

q34 = 4 nF
Re

surface of its metallic shell cannot exceed the break


down field of air (-3 xlO^Vm"’) because then the q,=q + q34 = i2+4 = i6^xF
Fi

charges begin to leak into air. This puts the limit on

Total charge stored in the network,


the potential difference which the machine can
built up. Q= =16|4Fx100V
14. (11) Refer to the solution of Problem 26 on = 1600 |iC.
page 2.78.
(b) Refer answer to Q. 38 on page 2.47.
COMPETITION SECTION

EleclTosfaNc PofenHal and CapacIfancE


GLIMPSES

1. Potential difference. The potential difference 5. Electric potential due to a point charge. The
between two points is defined as the amount of
electric potential of a point charge q at distance r
work done in bringing a unit positive charge from it is given by
from one point to another against the electro 1
static forces. i.e.. Vcc-
Work done 47tSq r r
Potential difference =
Charge It is spherically symmetric.
W
or 6. Electric potential due to a dipole. Electric potential
at a point having position vector t, due to a
Potential difference is a scalar quantity. dipole of moment p at the origin is given by
2. SI unit of potential difference is volt (V). The
1 1
potential difference between two points in an V = p .r p cos 0
electric field is said to be 1 volt if 1 joule of work 4jreQ ' 4718
0

has to be done in moving a positive charge of 1


coulomb from one point to the other against the At points on the axial line of the dipole
electrostatic forces.
(0=0'= or 180°),
1 P
1 V +
1 V=lJC-^=lNmCT axial
4jt8o V
3. Electric-potential. It is defined as the amount of
At points on the equatorial line of the dipole
work done in bringing a unit positive charge (0=90°),
from infinity to the observation point against
the electrostatic forces. V
equa
= 0.

Electric potential =
Work done
7. Electric potential due to a group of N point
Charge charges. If are the distances of N
W point charges from the observation point, then
or y=—
1
y = - + +.... +
4718
Electric potential is a scalar quantity. 0 . h h ^3

4. SI unit of electric potential is volt. The electric 1 N n


potential at a point in an electric field is said to be 4718
I ^ r.
0 «=l
1 volt if one joule of work has to be done in
moving a positive charge of 1 coulomb from 8. Determination of electric field from electric
infinity to that point against the electrostatic potential. The rate of change of potential with
forces.
distance is called potential gradient. Electric
(2.132)
ELECTROSTATIC POTENTIAL AND CAPACITANCE (Competition Section) 2J33

field at any point is equal to the negative of the 13. Potential energy of a system of three point
potential gradient at that point charges. It is given by
dV 1
£=- U = ^2 , ^2 % . %
dr
471EQ ^ Tj2 ^23 '31
-I
SI unit of electric field = Vm
->
14. Potential energy of N point charges. It is given
by
The direction of £ is in the direction of steepest
1
decrease of potential. L/ = z
4^e
0 All pairs '/y
9. Determination of electric potential from electric
field. The electric potential at a point having 1 1
I
jV

position vector r* is given by

w
2 4k8., i= l y= i
0
I*)

V^ = - £ .dr 15. Potential energy of a dipole in a uniform electric

Flo
GO
field. It is equal to the amount of work done in
turning the dipole from orientation 0^ to Gj in
10. Equipotential surface. Any surface that has

ee
the field £.
same electric potential at every point on it is
U = - pE (cos 02 - cos 0.J)

Fr
called an equipotential surface. The surface of a
charged conductor is an equipotential surface. If initially the dipole is perpendicular to the
Some of the important properties of equi field £, 0^ =90° and = 0 (say), then
for
ur
potential surface are as follows :
U = ~ pE cos 0 = - ^ . £
(/) No work is done in moving a test charge
ks
over an equipotential surface. When 0=0°, U = ~ pE i.e., the potential energy
Yo

of the dipole is minimum. Tlie dipole is in stable


(ii) Electric field is always normal to the
oo

equilibrium.
equipotential surface at every point.
eB

When 0=90°, U=0


{Hi) Equipotential surfaces are close together in
When 0=180°, U=+pE
the regions of strong field and farther apart
in the regions of weak field. i.e., the potential energy of the dipole is
r
ou

maximum. The dipole is in unstable equilibrium.


ad

(iv) No two equipotential surfaces can intersect


each other. 16. Conductors and insulators. Conductors are the
Y

11. Electric potential energy. The electric potential substances which allow large scale physical
energy of a system of point charges is defined as movement of electric charges through them
nd
Re

the amount of work done in assembling the when an external electric field is applied. They
contain a large number of free electrons.
Fi

charges at their locations by bringing them in,


Insulators are the substances which do not
from infinity.
allow physical movement of electric charges
P.E. of a charge = Charge x Electric potential
through them when an external electric field is
at the given point
applied. Tliey contain a negligibly small
It is measured in joule (J) or electron volt (eV). number of free charge carriers.
-19
1 eV «1.6 X 10 J
17. Electrostatic properties of a conductor. When
12. Potential energy of a system of two point placed in an electrostatic field, a conductor
charges. If two point charges q^ and q^ are shows the following properties :
separated by distance rj2, then their potential (/) Net electrostatic field is zero in the interior
energy is of a conductor.
1 ^2
u = (ii) Just outside the surface of a conductor,
4k6 n
0 12 electric field is normal to the surface.
.134 PHYSICS-Xll

(in') The net charge in the interior of a conductor 24. Parallel plate capacitor. It consists of two large
is zero and any excess charge resides on its parallel conducting plates, each of area A, and
surface. separated by a small distance d. Its capacitance
IS
(ii>) Potential is constant within and on the Ea0 a
surface of a conductor, C =
d
(n) Electric field at the surface of a charged
conductor is proportional to the surface 25. Spherical capacitor. It consists of two concentric
charge density, spherical conducting shells of inner and outer
radii a and b.
(ni) Electric field is zero in the cavity of a hollow
charged conductor. 4 ne^0 ab
C =
b-iJ
18. Electrostatic shielding. The phenomenon of
making a region free from any electric field is
26. Cylindrical capacitor. It consists of two coaxial
called electrostatic shielding. It is based on the
conducting cylinders of inner and outer radii
fact that electric field vanishes inside the cavity
a and b and of common length /.
of a hollow conductor.
I I
19. Capacitance of a conductor. It is the charge C=2 7t£ 0 = 2 ne
0

required to increase the potential of a conductor


by unit amount.
1log. ^
‘ a 2303 log,„ ^
Charge
27. Capacitors in series. The equivalent capa
Capacitance = Potential citance Cj of number of capacitors connected in
series is given by
or c=3 1 1 1 1
— - — + — + — + ...

c, q C3
20. Capacitance of a spherical conductor. It is pro
portional to the radius R of the spherical In a series combination of capacitors, the charge
conductor. on each capacitor is same but the potential
C=47t£y R difference across any capacitor is inversely
proportional to its capacitance.
21. Capacitor. It is an arrangement of two con
ductors separated by an insulating medium 28. Capacitors in parallel. The equivalent capa
that is used to store electric charge and electric citance of a number of capacitors connected in
energy. parallel is given by
22. Capacitance of a capacitor. The capacitance of a
capacitor is the charge required to be supplied
to one of its conductors so as to increase the In a parallel combination of capacitors, the
potential difference across each capacitor is
potential difference between two conductors by
unit amount.
same but the charge on each capacitor is
7
proportional to its capacitance.
C = -i-
y 29. Energy stored in a capacitor. The energy stored
in a capacitor of capacitance C and charge q with
23. Farad. It is the SI unit of capacitance. The
voltage V is
capacitance of a capacitor is 1 farad (F) if 1
1
coulomb of charge is transferred from its one u = - cy ^ —
plate to another on applying a potential dif 2 2 C

ference of 1 volt across the two plates. 30. Energy density. The electrical energy stored per
1 coulomb 1C unit volume or energy density in a region with
1 farad = or 1F =
1 volt IV electric field £ is
u =
lmF = 10'^F,lp F = 10"^F, lpF = 10”'-R 2
ELECTROSTATIC POTENTIAL AND CAPACITANCE (Competition Section) 2.135

31. Common potential. If a number of conductors of dielectric as the medium to its capacitance (C^)
capacitances C^, C^, Cy ...., at potentials Vy Vy when conductors are in vacuum.

Vy having charges —● respectively K =


C
are placed in contact, their common potential V is C0
given by
Total charge _ 'll + *72 ^3 ● It is also equal to the ratio of the applied electric
V' =
Total capacitance Cj + C2 + C3 + field (£q) to the reduced value of electric field
(£) on inserting the dielectric slab between the
plates of the capacitor.
Cl + C2 + C3 -f-.... K = ^0 _ ^0
32. Loss of energy on sharing charges. If two £ £„-£'■
conductors of capacitances Cj and C2 at Here £' is the field set up due to polarisation of

w
potentials and q respectively are connected the dielectric in the opposite direction of £q.
together, a loss of energy takes place which is
37. Capacitance of a parallel plate capacitor filled
given by

Flo
with a dielectric.
AU=--£fi- (q-q)'- C = K C.=-2
Er.K A
2 q +q 0

ee
d

33. Dielectric. A dielectric is a substance which does


38. Capacitance of a parallel plate capacitor with a

Fr
not allow the flow of charges through it but dielectric slab between its plates. If f is the
permits them to exert electrostatic forces on one thickness of the dielectric slab and t < d, then
another. It is essentially an insulator which can for
ur
0
C =
be polarised through small localised displace
d-t( 1-^
\ ●

ments of its charges. K )


ks
34. Polar and non-polar dielectrics. Tlie dielectrics
Yo

39. Capacitance of a parallel plate capacitor with


made of polar molecules (such as HCl, NH3,
oo

conducting slab between its plates. For t < d,


H2O, CH3OH etc.) are called polar dielectrics. In
eB

a polar molecule, the centre of mass of positive C = ^ d Yo^-( " C.0 .


charges does not coincide with the centre of d-tj d \d~t,
mass of negative charges.
40. Capacitance of a spherical capacitor filled with
r

a
ou

The dielectrics made of non-polar molecules are


ad

dielectric.
called non-polar dielectrics. In a non-polar C = 4 K Er,0 K
ab
Y

molecule, the centre of mass of positive charges b-a


coincides with the centre of mass of negative
41. Capacitance of a cylindrical capacitor filled with
nd
Re

charges e.g., O2, CO2, CH^, etc. a dielectric


35. Polarisation of dielectric. If the medium
Fi

2 Tie..0 K I
between the plates of a capacitor is filled with a C =

dielectric, the electric field due to the charged 2.303 logjo ^


plates induces a net dipole moment in the
dielectric. Tlnis effect is called polarisation which 42. Van de Graaff generator. It is an electrostatic
induces a field in the opposite direction. Tlie net generator capable of building up high potential
electric field inside the dielectric and hence the
differences of the order of 10^ volts. It is based
on the principle that when a charged conductor
potential difference between the plates are
is brought into internal contact with a hollow
reduced. Consequently, the capacitance C increases
conductor, it transfers whole of its charge to the
from its value Cq when there is vacuum. hollow conductor, howsoever high tlie potential
C = K Cq . of the latter may be. Also, it uses discharging
36. Dielectric constant. It is the ratio of the action of sharp points. It is used for accelerating
capacitance (C) of the capacitor with the charged particles.
2.1^6 PHYSICS-Xll

Muliipl ' v-iioico Questions with 2. Tlie electric potential V at any point x, \j, z (all the
orr- oci i'*r
coordinates are in metres) in space is given by V = 4x^
'. Consider a thin spherical shell of radius R with volt. The electric field at the point (lm,0,2 m) in volt
its centre at the origin, carrying uniform positive metre”^ is
surface charge density. The variation of the magnitude (a) 8 along negative X-axis.
of the electric field | E (r)| and the electric potential V{r) (&) 8 along positive X-axis,
with the distance r from the centre, is best represented (c) 16 along negative X-axis,
by which graph ? (ri) 16 along positive X-axis. [111

(a) lS:r)| V{r) 3. A uniform electric field pointing in positive


X-direction exists in a region. Let A be the origin, Bbe
the point on the X-axis at .y = -i-l cm and C be the point
/ on the Y-axis at i/ = -i-l cm. Then the potential at points
A, B and C satisfy.
/
/
/
/
/
/
/
id) > Vc [IIT 2001
O r
4. Two equal charges are fixed at y = and x -+a
on the X-axis. Another point charge Q is placed at the
(b) |£{r)| V{r)
origin. The change in the electrical potential energy of
\ Q, when it is displaced by a small distance y along the
\
X-axis, is approximately proportional to
(a) x (b)
1
(C) Y^ id)
Y
(IIT 2002]

O
i
R r
Three charges Q, +q and + q
are placed at the vertices of a right
angled isosceles triangle as shown.
ic) m\ V(r)
The net electrostatic energy of the
configuration is zero, if Q is equal +

to
/
'\
-(} -2q
(«) (b)
/ s l + ^/2 2 + ^2
/

/
/
(C) ~2q (d) +q illT 2i)i'I : IPUEE J31
t

o R
Consider a system of y
r
2 c/ B
' three charges -, - and -
3 3 3
(d) |£(r)| V(r) placed at points A, Band C, C
X

\
respectively, as shown in
s
the figure. Take O to be the
centre of the circle of radius
R and angle CAB = 60°.
(f?) Tlie electric field at point O is —~ _ , directed
8ne„B^
0

O R r along the negative Y-axis.


ELECTROSTATIC POTENTIAL AND CAPACITANCE (Competition Section) 2./-;;

(Jj) The potential energy of the system is zero, 12. Two identical capacitors have the same capaci
(c) The magnitude of the force between the charges tance C One of them is charged to potential and the
at C and B is
r other to V^. The negative ends of tlae capacitors are con
54k£qR^’ nected, the decrease in energy of the combi i. d system is

(d) The potential at point O is


7
4 ^ (b)~C(V^ + V^)
12 ne.R
0

7. A metal foil of negligible thickness is introduced ilii


between two plates of a capacitor at the centre. The
capacitance of capacitor will be
i A parallel plate capacitor of area A, plate sepa
ration d and capacitance C is filled with three different
(fl) same (b) double
dielectric materials having dielectric constants k V

w
(c) half (rf) K times ji'! '● .‘■.n ii ii '; A/2 A/2
T
8. A solid sphere and a hollow sphere of equal *^2 d/2
diameters are raised to the same potential. Then, d 1

Flo
{a) hollow sphere has more charge. ●^3

(b) both have equal charge,

ee
(c) only hollow sphere has charge. and K3 as shown. If a single dielectric material is to be

Fr
used to have the same capacitance C in this capacitor,
(d) solid sphere has more charge.
then its dielectric constant k is given by
9. Two capacitors of capacitances Cj and C, are , , 1 1 1 1 1 1 1
for (b)- =
ur
connected in parallel. If a charge q is given to the K K
i K2 2k^ K
k^ + K2 2kj
assembly, the charge gets shared. The ratio of the
K2K3
charge on the capacitor to the charge that on C2 is (C) K = + 2K3 (d) K =
ks
Kj -I- K2 K1 + K3 K2-hK3
c
Yo

(b)
oo

Q c1
14. Consider the situation shown in the figure. The
eB

1
(C) C,C2 (d) capacitor A has charge q on it, whereas B is uncharged.
qc2 ii I .
r

10. Two identical metal plates are given positive +


ou
ad

charges q^ and £/2(<^?]) respectively. If they are now s


brought close together to form a parallel plate
Y

capacitor with capacitance C, the potential difference


A B
between them is
nd
Re

The charge appearing on the capacitor B a long time


(«) (b) after the switch S is closed, is
Fi

2C C
(a) zero ib)q/2
(d)
Wl~?2) (c) q (d)2q
C 2C
Mil I‘-
I t. For the circuit shown in figure, which of the
11. In the given circuit, a o following statements is true ? [III
charge of -1-8O pC is given to the + 80(iC
l',I = 3U V 1^2 = A) v

upper plate of the 4 pF capacitor. 4pF +

Then in the steady state, the C, = 2pF C2 = 3 pF


charge on the upper plate of the
3 pF capacitor is 2pF 3pF {a) With Sj closed, =15 V, =20V.
(fl) +32 pC {b) +40 pC {(?) With S3 closed, = V^=25V.
(c)+48pC (d) +80 pC
IIIT 2012) I (c)
(rf)
With
with
S3 and ^ closed, =1/2=0.
S3 and S3 closed, V3 =30V, V2 =20V.
2.138 PHYSICS-Xll

16. A parallel plate capacitor C with plates of unit fixedat each of the points x x = 4Xq, x =6Xq, ... to
area and separation c/is filled with a liquid of dielectric co. Here Xq is a positive constant. Take the electric
constant k =2. The level of I i potential at a point due to a charge Q at a distance r
liquid is — initially. Suppose the c 1 Q
from it to be
3 d R r
■* 3
liquid level decreases at a
Tlien, the potential at the origin due to the above
constant speed v, the time
constant as a function of time t is system of the charges is
{a)0 (b) CO

0 (15rf + 9i'f)egK
(«) q\n2
5d + 3vt 2d^-3dvf-9v^t^ (d)
8n6Q.Tgln2 (irr 1998)
6e.R
0
id)
(15d-9uf)8oR
5d -3vt 2d^ + 3dvf-9v^t^ 20. A solid conducting sphere having a charge Q is
(IIT 20081 surrounded by an uncharged concentric conducting
17. A 2 pF capacitor hollow spherical shell. Let the potential difference
1 2 between the surfaces of the solid sphere and that of the
is charged as shown in outer surface of the hollow shell be V. If the shell is
the figure, The
now given a charge -3Q. the new potential difference
percentage of its stored v'nir- 2 nF 8pF between the same two surfaces is
energy dissipated after {a)V (b)2V
the switch S is turned to
(c)4V {d)-2V (IIT 1989)
position 2 is
(«)0% (10 20% (c)75% (rf)80% [11X20111 21, An ellipsoidal cavity is carved within a perfect
con- ductor as shown in figure. A positive charge q is
18. A thin spherical insulating shell of radius R
carries a uniformly distributed charge such that the
potential at its surface is Vq. A hole with a small area
a47tR'(a«l) is made on the shell without affecting
the rest of the shell. Which one of the following
statements is correct ?
placed at the centre of the cavity. The points A and B
(fi) The potential at the centre of the shell is reduced are on the cavity surface. Then,
by 2aVQ. (a) electric field near A in the cavity
(I?) The magnitude of electric field at the centre of
aV.0 = electric field near B in the cavity
the shell is reduced by
2R (t>) charge density at A = charge density at B
(c) The magnitude of electric field at a point, (c) potential at A = potential at B
located on a line passing through the hole and (d) total electric field flux through the surface of the
shell's centre, on a distance 2 R from the centre cavity is q/ Cq. [IIT 1999]
aV,0
of the spherical shell will be reduced by 2R
22. A spherically symmetric charge system is
centred at origin. Given that
(d) The ratio of the potential at the centre of the
I Electric potential.
shell to that of the point at — R from centre Q ^5.
y = r<Ro
towards the hole will be
1-a 47reoR,) '
l-2a Q
[JEE Adv. 2019J r
r>Ro O

Multiple Choice Questions with one or more (a) Within r = 2Rp, total enclosed net charge is Q.
than one correct answers
(b) Electric field is discontinuous at r = Rq.
19. A charge +qis fixed at each of the points .t = Xq, (c) Charge is only present at r = Rq.
Xq =3x^;x = 5Xq,... to Qc on the X-axis and a charge~qis (d) Electrostatic energy is zero for r<R^. [IIT 2006)
ELECTROSTATIC POTENTIAL AND CAPACITANCE (Competition Section) 2.139

electrostatic charge density varies with the radial


23. Two identical thin rings, each of radius R metre
are coaxially placed at distance R metre apart. If Qj
distance r from the centre as = kr, where k is
and Q2 coulomb are respectively the charges positive. In the spherical shell B
uniformly spread on two rings, the work done in of outer radius the electro-
moving a charge q from the centre of one ring to that static charge density vanes as
of the other is 2k
Pg =—.Assume that dimensions
(a) zero ib)
?(Qi-Q2)(V2-1)
A-JIuSqR are taken care of. All physical
quantities are in their SI units.
(C)
?^/2(Ql + Q2) (d)
^(Qi + Q2)(^ Which of the following statement(s) is/(are) correct ?
4TC8,^R
0
4^/2 TiSnR
0 i!3
(rt) If r„ = j-, then the electric field is zero every
24. Which of the following statement(s) is/are correct ? V2

w
where outside B.
(r?) if the electric field due to a point charge varies
3
as instead of r, then the Gauss law will still
(fc) If Tg = -, then the electric potential just outside B
be valid

Flo
. k
(b) the Gauss law can be used to calculate the field is —.

^0
distribution around an electric dipole

ee
(c) if the electric field between two point charges is (c) If rg=2, then the total charge of the

Fr
zero somewhere, then the sign of the two configuration is I5nk.
charges is the same 5
(d) If Tp = -, then the magnitude of the electric field
(d) the work done by the external force in moving a for
ur
unit positive charge from point A at potential just outside B is
to point B at the potential Vg is (Vg - ). ^0 [[EE Adv 22)
ks
I
27. In the circuit shown in the figure, there are two
Yo

25- Six charges are placed parallel plate capacitors each of capacitance C. The
oo

around a regular hexagon of switch Sj is pressed first to fully charge the capacitor
eB

side length a as shown in the ●T


.-,90°
●<? Si / Sj/
figure. Five of them have I tr o—●—a 0—f—cf 0 1

charge q, and the remaining one O* C.


£2
r

±-V.0
has charge x. Tlie perpendicular 2V.0 X
ou
ad

from each charge to the nearest ● q ●<?


hexagon side passes through
Y

the centre O of the hexagon and *.r


and then released. The switch Sj is then pressed to
is bisected by the side. charge the capacitor C2. After some time, $2 is released
nd
Re

Which of the following statement(s) is (are) correct and then S3 is pressed. After some time,
in SI units ?
Fi

(<7) The charge on the upper plate of Cj is 2.CVq


(a) When x = q, the magnitude of the electric field at (b) The charge on the upper plates of Cj is
O is zero.
(c) The charge on the upper plates of C2 is 0
(&) When X = ~q, the magnitude of the electric field
9 (rf) The charge on the upper plate of C2 is -CVq.
at 0 is
IJEE Adv. 13]
0

7q 28. A medium having dielectric constant X >1 fills


(c) When x = 2q, the potential at O is the space between the plates of a parallel plate
0
capacitor. The plates have large area, and the distance
3q
{d) When .t = -3q, the potential at O is between them is d. The capacitor is connected to a
4>/37iSQfl battery of voltage V, as shown in Fig. (a). Now, both
IJEE Adv 22]
the plates are moved by a distance of ~ from their
26. In the figure, the inner (shaded) region A
represents a sphere of radius =1, within which the original positions, as shown in Fig. (b).
.140 PHYSICS-XII

32. A parallel plate capacitor has a dielectric slab of


dielectric constant /C between its plates that covers 1/3
7^‘^*7 of the area of its plates, as shown in the figure. Tl-ie
2 2
total capacitance of the capacitor is C while that of the
V portion with dielectric in between is C^. When the
(«) (/') capacitor is charged, the plate area covered by the
dielectric gets charge Qj and the rest
In the process of going from the configuration of the area gets charge Qj. The electric
depicted in Fig. (j7) to that in Fig. {b), which of the Qi £1
field in the dielectric is £j and that in
following statement(s) is(are) correct ? the other portion is £2- Choose the
(fz) The electric field inside the dielectric material is correct option/options, ignoring edge Qi
reduced by a factor of 2K. effects.

(«)l^ = 1 {b)^=~
1
(b) The capacitance is decreased by a factor of K+1
£2 £2 K
(c) Tlie voltage between the capacitor plates is 9l=1. C 2 + K
(c) (d)
increased by a factor of (K + 1). Q2 K C
1
K
!i:i- AJ\. 14)
(rf) The work done in the process DOES NOT
depend on the presence of the dielectric 33. A parallel plate capacitor having plates of area S
material. [IFilAJv 2:1 and plate separation d, has capacitance in air.
2^}. A parallel plate air capacitor is connected to a When two dielectrics of different relative permittivities
battery. The quantities charge, voltage, electric field (8^=2 and 82=4) are dll
introduced between the
and energy associated with capacitor are given by Q,, two plates as shown in
Vq , £q and Uq respectively. A dielectric slab is now
the figure, the capaci
introduced to fill the space between the plates with SI2
battery still in connection. The corresponding tance becomes C2. £2
C o-

quantities now given by Q, V, £ and U are related to Tlie ratio ~ is’ +

C
the previous ones as 1 S/2

(«)Q>Q, {b)V>V^ (fl) 6/5 (/j) 5/3


(c)E>£„0 (d)U>U 0 |j| r !9s=;| (c) 7/5 (d) 7/3
30. A dielectric slab of thickness d is inserted in a .^dv. 1=^1

parallel plate capacitor, whose negative plate is at .v = 0 34. A point charge +Q is placed just outside an
and positive plate is at x=3d. The slab is equidistant imaginary hemispherical surface of radius R as shown
from the plates. Tlie capacitor is given some charge. As in the figure. Which of the following statements is/are
a: goes from 0 to 3 fr, correct ?

{a) the magnitude of the electric field remains tlie same,


(h) the direction of the electric field remains the same.

(c) the electric potential increases continuously.


(d) the electric potential increases at first, then
decreases and again increases. i 111 1 [ (a) Total flux through the airved and the flat
surfaces is —.
31. The magnitude of electric field £ in the angular
^0
region of a charged cylindrical capacitor
(rt) is same throughout. (b) The component of the electric field normal to
the flat surface is constant over the surface.
(b) is higher near the outer cylinder than near the
inner cylinder, (c) The circumference of the flat surface is an
equipotential.
(c) varies as-, where ris the distance from the axis.
r
(rf) The electric flux passing through the curved
Q O
(d) varies as , where ris the distance from the axis. surface of the hemisphere is - 1-

I in 19961
28 0 ^ V2/
ELECTROSTATIC POTENTIAL AND CAPACITANCE

35. An electric dipoie with dipole moment iket' - .f it^ ’

pi A A
+ i )is held fixed at the origin Oin the presence of Paragraph for Questions 37-38
a uniform electric field of magnitude £q. If the potential Consider a simple RC circuit as shown in Fig. 1.
Process 1. In the circuit the switch S is closed at t = 0
and the capacitor is fully charged to voltage (i.e.,
charging continues for time T » KQ. In the process
some dissipation (E^) occurs across the resistance R.
■►X The amount of energy finally stored in the fully
R
charged capacitor is .
V

w
Process 1
AW 0

is constant on a circle of radius R centred at the origin R

as shown in the figure, then the correct statement(s) V© C


Process 2. r»RC

Flo
is/are :

(Sq is permittivity of free space, R » dipole size) T 2T t

ee
(a) Total electric field at point B is Eg =0

Fr
Figure 1 Figure 2

(&) Total electric field at point A is E^ =-^/2Eg(i + j ) Process 2. In a different process the voltage is first
set to — and maintained for a charging time T » RC.
n1/3
for
ur
Po 3
(c)K = 2V,
47I£oEoy Then the voltage is raised to —2.
3
without discliarging
ks
(d) The magnitude of total electric field on any two the capacitor and again maintained for a time T » RC.
Yo

points of the circle will be same. The process is repeated one more time by raising the
oo

Ui voltage to Vq and the capacitor is charged to the same


eB

final voltage as in Process 1.


Reasoning Type
These two processes are depicted in Fig. 2.
Instructions. This question contains statement -1
r

In Process 1, the energy stored in the capacitor


ou

(assertion) and statement - 2 (reason). Of these statements


ad

and heat dissipated across resistance E^ are related


mark correct choice if by
Y

(a) statements - 1 and 2 are true and statement - 2 1e D (b)^=2E^


is a correct explanation for statement - 1.
nd
Re

(fc) statements - 1 and 2 are true and statement - 2 (c) ^ = Ej) (d) ^ =£pln2
Fi

is not a correct explanation for statement - 1


(c) statement - 1 is true, statement - 2 is false In Process 2, total energy dissipated across the
resistance E,, is
(d) statement - 1 is false, statement - 2 is true. D

36. Statement - 1 : For practical purposes, the earth


(«)£D=3ficV„^l \1 J
(b)Eo=l cv}z
0

is used as a reference at zero potential in electrical Ifl 9


circuits. (c)Ep=3CV2
Statement - 2 : The electrical potential of a sphere
of radius R with charge Q uniformly distributed on the Int Ansv.
Q
surface is given by A point charge q of mass m is suspended
0

vertically by a string of length 1. A point dipole of


2.N2 PHYSICS-XII

dipole moment p is now brought towards q from The cell electromotive force (emf) Vq = 8 V. First the
infinity so that the charge moves away. The final switch Sj is closed while the switch ^ is kept open.
When the capacitor Cj is fully charged, Sj is opened
equilibrium position of the system including the and $2 is closed simultaneously. When all the capa
direction of the dipole, the angles and distances is
citors reach equilibrium, the charge on C3 is found to
shown in the figure below. If the work done in be 5pC The value of =
q. 11. A parallel plate capacitor of capacitance C has
I
spacing d between two plates having area A Then
region between the plates is filled with N dielectric
I layers, parallel to its plates, each with thickness 5 = -^.
N'
th
The dielectric constant of the m laver is

h
K
HI =K 1+— . For a very large N{>10 ), the capaci-
n "J
0
P. tance C is a . The value of a will be . [8 0
dln2

bringing the dipole to this position is Nx {mgh), where is the permittivity of free space] [J. I
^ is the acceleration due to gravity, then the value of N ●n. Two capacitors with capacitance values
. (Note that for three c^lanar forces keeping
IS
Cj =2000 ± 10pF and Cj = 3000 ± 15pF are connected in
a point mass in equilibrium, is the same for all series. The voltage applied across this combination is
sinG
V= 5.00 ±0.02 V. The percentage error in the
forces, where F is any one of the forces and 0 is
calculation of the energy stored in this combination of
the angle between the other two forces) [1: ! . u, 'n j
capacitors is
K'umerical Value Ljj)1o Socoik; . In the following circuit, Cj = 12 pF, C2 = 4pF and
!o. Three identical capacitors C2 and C3 have a = C. =2 jiF. The charge stored in C3 is pC.
capacitance of 1.0 pF each and they are uncharged
initially, They are

connected in a circuit as
0 T S2\
shown in the figure and Cj 6V- 2V
is then filled completely Cl C3
with a dielectric material C2
of relative permittivity e^..
il K.'.^iv ^2)

Aii.su^er.s ;iiid l‘>.\p]an;il


1. uh Electric field at point (lm,0,2m) is
£ V kQ =-8xl = ~8Vm’^ M’til V - I in
Vjj, = — - constant
R
\ 3.; Electric potential decreases in the direction of
kQ
the electric field, so > Vg. Dotted lines represent
^ out J. equipotential surfaces, so = V^.
£|n»0 y

o >
r O r

>
C
2. i.'; Given V = 4x^ volt
i.e., electric potential changes only along X-axis. ●X
A B
dV
dx
= ~(4x^)
dx
= ~8x
ELECTROSTATIC POTENTIAL AND CAPACITANCE . / t J

4. ((') Potential at point O,


Q <7 Q <] V = 1 r ? I? ^
x = -a x = 0 X = + fl x = -a x=x x=+a 4ti£q 3R 3R 3R
Initial configuration Final configuration Option (d) is wrong.

1 2Qq 7. The arrangement is equivalent to two capacitors


U- =
I
4ne 0 a
connected in series. Each such capacitor has plate separa
tion d /2 and hence capacitance2 C. Total capacitance is
u r 1 1 2Cx2C
= C
^ 4Tte 0 a+x a-x Cs = 2C + 2C

1 IQcjx^ i.e., the capacitance remains same.


/ I
4xs 0
3

w
a
8. For a spherical conductor,
AUocxrl C = 4kSqR i.e., CccR
5. 1 The net electrostatic energy. As both the spheres have equal radii, they have the

Flo
same capacitance. When charged to the same potential,
1 Q-(l
U = = 0 they will have the same charge.
-Jla

ee
4:16 a a
0
9. For the two capacitors connected in parallel,
ij->/2 + Q + ->/Q - 0

Fr
or

2q q=V2 or or

or Q =- ‘?T ^2 ^2 ^
^/2-^l 2+^'
for
ur
6 A
6. I ,Electric field at O, 10. Capacitance, C = -2—
d

£ = ^x 1
ks
3 47te«R^ 67tEr.R^ Electric field between the plates,
Yo
0 0
E= - £2
oo

Option (fl) is wrong.


1
eB

2s 2s 0 2s
(0^-02) = 2
0 0 0

P.D. difference between the plates.


r
ou

V= £d =
ad

0
Y

11. Net charge on the isolated system = 0.


The capacitors of 2pF and 3 pF are in parallel.
nd
Re

Potential energy of system, a

+ 80 ^C
+ ix r-2q r-2‘7'1 q 1
Fi

1 q q 1 1
U = — X - X +
I, 3 j 3 AC
r
47te
0 L 3 3 2R 3 3 j BC -80 nC 4nF

AC = 2Rcos60“ = R
2mF 80-X X 3pF
BC = 2Rsin60°=V3R

u =
1 r 2q^ 2q^ ^0

Option (6) is wrong.


4jte«0 12 R 9^/3R 9R
T
P.D. across each of them must be same.
Electric force, between the charges at B and C,
1 --^q ^ 2q 80-x _ X
F = ~2 3
4t:So 3 3(2 R sin60'*)^ 54ti£oR^
x=+48pC.
Option (c) is correct.
2.144 PHYSICS-XII

12. (rt) AL/ = a-U Id d


I
f + vt - -vt
J-=J- J_ +3

2
S " c/ q ^0 28
0

6s 0
or C
eg
3vf + 5rf
6e„R
0
Time constant, t = RC
13. (i^l The given arrangement can be represented eg
3i;f+ 5rf
as follows :
17. I.-'J
A!2 AH A
« 2 1 2
T
dll K
I K
2
u
f
20 q +C2 ~2 ■ 2+8 “T
K d
d/2 K3 <3
i.
- 4V^
Loss of energy = V^ - — D 5
In parallel
41/2
For each portion, we use
E«/l
0
% loss = -^xl00=-xl00=80%
5
C =
d-t
1 f2 + -^
ICj
+ -?K2 18. ■ Surface charge density on the shell, a =
Q
4nR^
As
c = q + C2 Charge on the small area (ct4rtR^), dq = aQ
kSq a ^/l/2 g,)A/2
d , d d d d . d d d d
d + — + — d~ -- + +
2 2 2k 1
2K3 2 2 2k2 2K3
A
K Sn0 A e«0 A 8n0 A
d d- + —
d d d
— H
K
1 K2 K3
Potential on the surface before removing small
K = kQ
area, VL =
K1+K3 '^2 + '^3 R

14. (<0 When switch S is closed, there is no transfer = K-y. =\/_


kaQ
centre 0 dq 0
R
= W-«)
of charge from capacitor A to capacitor B because the
charge -q of A is held by its charge + q. AV
centre

15. (k) With the closing of switch S3, the potential Option (fl) is incorrect.
drops across Cj and Cj would become identical and P, kaQ
R/2 = ^B = ^0 RI2
= W-2a)
equal to the average of q and V.
centre 1-a
30V+20V => Option (rf) is correct.
I.C., =25V. V, l-2a
2 R/2

kqa _ aV,0
16. At any instant t, level of the liquid =--vt
3
^2R ~ ^A~ ^0~^Bg 'W~~~R
aV,0
The arrangement can be regarded as a series Electric field at A reduces by R
combination of two capacitances with A = unit area.
«0 => Option (c) is incorrect.
■■■ =
2d
+ vt £ al/g _ ctV,0
centre
3 2d R R
2e 0 c_i_ _ 5 -^ 3 aV.0
and Cj X-2 ^
^ - vt
3
Electric field at C increases by R
--vt
3
=> Option (b) is incorrect.
ELECTROSTATIC POTENTIAL AND CAPACITANCE (Competition Section) 2.145

19. rj\ Potential at the origin will be 23.il,)W = q{V^-Vg)


1
<7 JL + A i +... Q1 Qa
4 Tie IXr.0 3x^0 4x 0 sf2R 42R
0 .^0
R R
9 i-i+1 —+...
1 ^—ln2. A R
2 3 4
47180^0
20. ( The potential at any point inside the shell
will be equal to that on its surface. When the shell is 1
given a charge -3Q, the potential on its surface 47CE R yl2R
0 L
changes by same amount as that inside. Hence the
potential difference V between the surfaces of solid 1
9z.^3 1

w
sphere and hollow sphere remains unchanged. 4tI£
0 L
R -J2R
21. (i 1 and (,;i Under electrostatic condition, all
points lying on the conductor are at the same potential. W =
?(Ql-Q2)(^/2-l)
4-«^7r6r,K

Flo
Therefore, potential at A = potential at B. Hence option 0

(c) is correct. From Gauss's theorem, total flux through


24. (n, \ih Gauss law is applicable only to that

ee
the surface of the cavity = — . electric field which obeys inverse square law of
^0
distance. So option {a) is incorrect.

Fr
22. ur, ' and !.,'i The given V-r graph is for a
As the field of an electric dipole is not symmetric,
charged solid or hollow metallic sphere.
so the integral Ii E.dS cannot be evaluated. Option (&)
for
ur
is incorrect.

In case of similar charges, null point lies between


ks
the two charges. Option (c) is correct.
Yo
O
r
oo

W
ext -q{VB~V^). So option (d) is correct for a unit
positive charge.
eB

25. („■), (/ i, (c )
When X =q, the charge
r

distribution is symmetric.
ou

+
ad

^=0
Y

t
tjj => Option {«) is correct.
When X = ~q, fields due to
nd
Re

£ = 0 ' diagonally opposite charges q


R0 and -q get added at O.
Fi

1
(/j) Option (rt) is correct because the entire charge Q
lies within a sphere of radius r=2R^. 47t£Q (^/3fl)^ 67r8gfl^
dV
(b) For t<Rq, I£ I= =0, => Option (b) is correct.
dr
When X = 2 q,
dV Q
For r> Rq, |£ = 5x ^ I 7q
dr
4718^?^ 0
47tS
0^ 4Sa y/Sa) 4V37i8Qfl
.'. Electric field is discontinuous at r=
=> Option (c) is correct.
The option (b) is correct,
When X = -3q,
(c) As V changes continuously for r> Rq, the
1 ( 5x J7 q
option (c) is wrong, ^0 =
(rf) For r < £g, E =0. Hence there is no electrostatic
4716
0^ ■JSa -Jda) 2-s/3 7t8Qfl
energy for r < Rg. The option (d) is correct. ^ Option (rf) is incorrect.
.' .J i () PHYSICS-Xll

26.{-- 28. :
K K
r^4
^/l = krx4nr^dr = ATxk —
£i £() £2 £q

2 2
h
V V
(«) (b)

= 4bI:(;^-1)
In Fig. (fl), £, =^, q = a a

In Fig. (b), V=EqX^+E^xcI+EqX^


Val =3kK ^KE^d+E^d [q = icq]
Electric field just outside Bis = {K + l)E^d
V _
£ = 1 3nk ^ k ^ 0 Option (fl) is incorrect. £2 =
47t8rt 3/2 2e 0 d{K + l)~ K + 1
Electric field inside the dielectric reduces by a
W’ii=|' “(total -6nk
factor of . So option (a) is incorrect.
K+1
Potential just outside Bis Sr,A
0 KsqA _ q
1 6;dc
V = — Option (b) is correct.
47t£n0 3/2 K
1
=12n/c Option (c) is incorrect. ^ Capacitance decreases by a factor of
K+1
= 227tA:
“2 ' Option (b) is correct.
Electric field just outside B is Voltage between the plates remains V.
£ = 1 227tfc ^ 22k .'. Option (d) is incorrect. Option (c) is incorrect.
47r£n '25/4 ”25e 0 w = a-u^=J(c,-C2)y"
I

27.1.'
/ ^2y Ke„A/.
0
1-
1 K\AV^
When Sj is
—O O y g >
d V K+TJ~2~“ 2d(K + l)
+ 2CVo C,
pressed and 2V.0
£^2 => Work done depends on K. Option (d) is incorrect.
X 0
released. 29.' ' and '

—c
✓ o—●—o
^2y 0 ●
^3y
q o-

When $2 is cv. cv.0


pressed and 0+ Cl + ^0
2P,0 X C2 i-V,0
released. P.D. = K0 P.D. = ,V0

Capacitance = Cq Capacitance = K Cq
Q) ” Q = ^CqVq

When S3 is CV.0
U=IkCqVq2
cv,0
pressed. V,
2V.0 X Ci = C2 +
d^V.0
£=-!i
cv0 d

Options (fl) and (d) are correct.


/ “
ELECTROSTATIC POTENTIAL AND CAPACITANCE ; Section) 2.1 “7 /

30.' ') and ! ; The situation is shown in the figure. Now, C, =


K8f,A/3
d
Dielectric

£q +
_8,2A/3
+ + d
+
E net
+ {K + 2)e^A
c = q-.q =
+

= Eo-£,- + 3d
+ +

+ C fC+2
Option (d) is correct.
C K
x = 0 x-d x = 2d x = 3d 1

fCe.A
0
V
Due to polarisation of the dielectric, a field E. is Again, Q.j =Cj\/ = 3d

w
induced in the opposite direction of original field Eg. 2e.A
Q
Q2 = C^V =
-* -*

But net field Enet = Eg- E. is in the direction of Eg . 3d

Flo
Hence the direction of the field remains the same from
=
=> Option (c) is incorrect.
+ve plate to -ve plate. Q2 2

ee
The electric potential increases from -ve plate to 33. I,.’!
+ve plate. Due to the weaker electric field, the increase

Fr
C'
in potential is less inside the dielectric slab. The d/2
C h
variations of electric field £ and potential V with
distance x are shown below :
for
ur
C C
S/2
E
^2
ks

o- ■o o- ■O
+
0 d 2d 3d +
Yo

t t
oo

S/2
C
^1.
c
eB

d
V
r

EnS 2eg-2 _ 2e«S


c ^^
ou

0 0
= 2C
ad

Ci = d ' d
1
Y

2
S
4e
2_ 4e„S
0
nd

0
C = = 4C
Re

^ d d
1'

2
Fi

31. ((1 In the annular region of a charged cylindrical


capacitor, 2s.-^
0
2
X C =
£ = d ■ d
2 Tte«0 r
CC 4 7
+ C1 = -c,+ c =-c 1
i.e.. Eoc- 3 ^ ^3
r
C +C 2q + 4q
where X is the charge per unit length of the capacitor. ^=7
Hence,
32. (rt), id) q 3
As both parts have the same potential difference. 34. irv <.;)

=1 In option («?). As the charge Q lies outside the


hemisphere, the net flux through it is zero.
Option (fl) is correct and option (b) is incorrect. .●. Option (fl) is incorrect.
PHY5ICS-XII

In option (b).
R
cos 9 = — 0

Eq is in the direction of p so that


^_kQ ^kQcos^Q E„ =£ 0
eq
^3 ^0
\l/3 n1/3
E,1 = E cos 0 =
kQcos^Q kp,0 Eo
R =
R^
4neo£oJ
As we move away from the centre 0 increases, cos 0
=> Option (c) is correct.
decreases and Ej_ decreases. That is the component of
£ normal to the flat surface is not constant.
= ^eq + ^0 =
kpQ^SkpQ
r3 p3 ^3
Option (b) is incorrect.
In option (c). Potential at any point on the Option (b) is incorrect.
circumference of the flat surface, Eq is uniform and electric field of the dipole is
V = — --
Q Q different at different points. So option (d) is incorrect.
= constant
4^+R^ 47rEo(^/2K) 36. !,:●) As radius of the earth is very large, so the
potential on the earth can be taken equal to 2ero.
=> Circumference of the flat surface is equipotential
Option (c) is correct.
37. ) Work done by the battery.
In option id). Solid angle subtended by the flat
surface at the location of the charge Q, Energy stored in the capacitor, ^ ^
±Q
Energy dissipated across the resistor,
45'

^ - £^ => Option (c) is correct.


y.
38. (ii) When voltage is set to charge supplied
n =27c(1-cos0) = 2tu(1~cos45°)=2ji; 1—^ CW
^/2 by the battery =“^-
Now, (f)curved surface flat surface
=0 2V
When voltage is raised to additional charge
Iturved surface ’flat surface — x27lfl- 1
2CV„ 0
% 4% supplied =■"
- 3 3
Q 1 ^
1- When voltage is raised to y„ additional charge
2e
0^ ^/2
supplied - CVq
2CVq_CV^ 0 “ ^
3 3
=> Option (rf) is correct.
35. ,,^i.
Total work done by the battery,
=> \p\ = Po CK,0 2K CK0 CV,0
X Ox + y,x _

As potential is constant on the circle, ,£langentia] ^ 3 3 3 3


0
3 3 ®
should be zero at all points on the circle. Final charge on the capacitor = CVq
—>

£n =0 => Option (a) is correct.


Energy stored in the capacitor, ^ ^ CVq
ELECTROSTATIC POTENTIAL AND CAPACITANCE (Comi)eLition Section)

Energy dissipated across the resistor, By KVL,


3
=2
cv}0 re/i
3
= -=1.50.
Option (d) is correct. 2
39. -■!
41.
d
dC

Area = A
K
m

w
Flo
m _X
For m dielectric layers
' N~d
K (^1 + —
A
/

ee
m
K._ =K 1 + = ^ dC=-
U r = mgh +
d) dx
in
N

Fr
W= Li
The dielectric layers are connected in series.
/ U. = mgh +
1 r 1 i dx ^ 1 i dx
Applying law of sines for forces on charge q, for
ur
d)
qE mg
id dln2
sin(180°-a) sin 90®+ —
d d
ks
[ln2-lnl] =
2) ^/Jo kc«A KEr.A
kSqA
Yo
0 0
a a
oo

2 sin — cos K£«A


0
mg sina _ mg 2 2 C = a =1.00.
dln2
eB

£ a £ a
cos cos—
2 2 42. 1.30
2000x3000
fngr^ 2 sin a C =1200 pF
r

eq 5000
2kp 2 q + c^
ou
ad

AC
1 1 1 ACj AC^
m
gi^ sin 0— As
Y

kp 1 ● « cleij C?1 r2
A- = mgh + mgrsin— ‘"2
W = mgh + 2
r

AC1 AC2 10 15
nd

= (1200)^
Re

, . a . a
= mgh + mg 2/sin—Ism— C?1 r2
‘-2 (2000)^ (3000)^ _
Fi

f 2 9 4
= lOx — +15x = 6nF
= mgh + mg 21 sin — = mgh + mgh 25 25
2j
. a h
sm— =
2 2/sina/2
ALL AC 2AV^
eq
W =2mgh => N=2. xl00 = xl00 + xlOO
a C V
cq
40. i.50 6x100 2x0.02x100
= 0.5 + 0.8=1.30.
+ 3mC 1200 5
1 X E, + 5pC 43. S.UO
-3nC
+ 8pC 1 pF
V.0 = 8W 1 pF _+3 jiC - 5 nC Potential difference across the terminals of C3 is 2 V
-8pC 1 tiF (constant).
-3mC

Only Sj is closed Sj is opened and S2 closed q^ = CjV = 4pFx2W = S[iC.


2J50 PHYSICS-XII

JEEMain

Based on Electric Potential due to a Point 5. A charge Q is uniformly distributed over a long
Charge, Group of Point Charges, Electric rod AS of length Las shown in the figure. The electric
Dipole and Charged Spherical Shell
o A B
i- Two points P and Q are maintained at the
potentials of 10 V and -4 V respectively. The work potential at the point O lying at distance Lfrom the
done in moving 100 electrons from P to Q is endA is
(fl) -19xlO‘^^J (&)9.60xl0“^^J Q 3Q
(«) (b)
87T£«L 4718^1
(c)-2.24xl0“‘^J (d) 2.24 X10"^^ J {AIEEE 0 0

2. An electric charge 10"^p C is placed at the origin (c)


Q
{d)
Qln2
4ne„Lln2 4718,,0 L
(0,0) of X-Y co-ordinate system. Two points A and B are 0 [)EE Main 13]
situated at (-v^,%/2) and (2,0) respectively. The potential 6. The two thin coaxial rings, each of radius 'a' and
difference between the points A and B will be having charges +Q and -Q respectively are separated
(a) 4.5 volt (b) 9 volt by a distance of'sThe potential difference between
(c) zero (d) 2 volt [AIEEE 2007] the centres of the two rings is
3. Charges are placed on the q {«)
Q 1 1
(b)
Q 1 1
vertices of a square as shown. Let A B 2 718
0
a

£ be the electric field and V the


potential at the centre. If the Q 1 1 Q 1 1
(C) - +
id)
2 718 a 47t8
charges on A and B are D C
0 0
a

intercharged with those on Dand -Cj PEE Main .Aug 21]


C respectively, then
7. A point dipole p =~PqX is kept at the origin. The
(a) E changes, V remains unchanged.
—> potential and electric fields due to this dipole on the
(b) £ remains unchanged, V changes, y-axis at a distance d are, respectively
(c) both £ and V change. (Take V =0 at infinity)

(d) E and V remain unchanged. (AJEEE 2007]


IP P -p
(«) (&)0
47T8nd^' 4ne„d^ 47T8,^d^
4. Ten charges are placed on the circumference of a 0 0 0

circle of radius R with constant angular separation IP -P


(c)
between successive charges. Alternate charges 1, 3, 5, 4ns^d^'
0
47ce„d^
0
47re„d‘^
0
7, 9 have charge (+f|) each, while 2, 4, 6, 8, 10 have
[JEE M.iin April 19]
charge (-<^) each. The potential V and the electric field £
at the centre of the circle are respectively (Take = 0 at 8. The electric dipoles. A, B with respective dipole
A —> A
infinity) moments d
= ~4qai and d^ =-2qai are placed on the
A

(fl)V=0; £=0 AT-axis with a separation R, as shown in the figure.


R

{b)V = 10? . p lOq A B


> X

4t18„K
0
' 47C8a£^
0
The distance from A at which both of them
Wq produced the same potential is
(c)V = E=0
4t:8aR
0
" ■J2R ■J2R
(«) (b)
J2-1 -J2+1
lOq
(d)V=0; £ = R R
4n8nR^ ic) (d)
0
pEE Main Sept. 20] J2-1 J2+1 pF L Main Jan 19]
ELECTROSTATIC POTENTIAL AND CAPACITANCE (Competition Section) 2.131

The given graph shows variation {with distance r 13. Concentric metallic hollow spheres of radii R
from centre) of and 4K hold charges and Q2 respectively. Given that
surface charge densities of the concentric spheres are
equal, the potential difference V(R)-V(4R) is
3Q2 3Qi
ip) ib)
4mr.R 167te«R
0
0

3Q1
ic) id)
> 4ne„R 4ne^R
''0
0 0 IJti; M.iin Sc'pl. 20]
14. Three concentric metal shells A, B and C of
(fl) Potential of a uniformly charged spherical shell
respective radii a, b and c{a<b< c) have surface charge
{b) Electric field of a uniformly charged sphere
densities +a, -a and +a respectively. The potential of

w
(c) Electric field of uniformly charged spherical shell B is [][.[■ Main 18]
shell
o a^-b^ a a^-b^
(li) Potential of a uniformly charged sphere («)- ib) + c

Flo
+ c
a b
I i M.iiu [an is’l ^0 .
I” Thin spherical conducting shell of radius R has

ee
o b^-c^
a charge q. Another charge Q is placed at the centre of ic) + a id)- + n

Fr
^0 c
^oL J
the shell. The electrostatic potential at a point P at a
distance R/2 from the centre of the shell is 15. Consider two charged metallic spheres Sj and $2
2Q 2Q 2q of radii Rj and R^, respectively. The electric fields
for
ur
(fl) ib)
47t8rtR
0
4k£«R
0
4Tce„R
0 (on Sj) and (on $2) on their surfaces are such that
/ £2 = Rj / R2- Then the ratio Vj(on ) / ^2(0^1 ^2) of
2Q
3— id) iq+Q)2
ks
ic) the electrostatic potentials on each sphere is
4jtSQR AkEqR
Yo

0
R
oo

1
(«) ib)
11 - A charge Q is distributed over two concentric
eB

conducting thin spherical shells of radii rand R(R > r).


If the surface charge densities on R1
(c) id)
the two shells are equal, the
[in Main Jan 20]
r

electric potential at the common


ou
ad

centre is 1 (1. Two uniformly charged spherical conductors A


and B of radii 5 mm and 10 mm are separated by a
Y

1 (R + r) 1 (R + 2r)Q
(«) Q ib) distance of 2 cm. If the spheres are connected by a
4neo(R^ + r^) 4Tteo2(R^ + r^) conducting wire, then in equilibrium condition, the
nd
Re

1 (R + r) 1 (2R + r) ratio of the magnitude of the electric fields at the


ic) Q id) Q
47iEQ2(R^ + r^) 4kEq (R^ + r^) surface of the spheres A and B will be
Fi

(fl) 1 : 2 ib)2:l
(rf) 1 : 4 M.in, [iiK T?
(c) 1 : 1
12. A charge Qis distributed over three concentric
17. Two isolated conducting spheres Sj and of
spherical shells of radii a, b, c(a < b <c) such that their
surface charge densities are equal to one another. The radius — R and -R have 12 pC and -3pC charges,
3
total potential at a point at distance r from their 3
common centre, where r < a, would be respectively, and are at a large distance from each
other. They are now connected by a conducting wire. A
Q(fl + b + c) Q
ia) ib) long time after this is done the charges on and Sj are
Ans^ia^ + b^+ <^) 47T8Q(fl + l;+C) respectively
Q(a^ + b^ + c^) id):
Q nb + bc+ca (n) 4.5 pc on both (b) + 4.5 pC and - 4.5 pC
ic)
4718^(^7^+ &^ + C^) 12718 0 abc
(c) 6 pc and 3 pC (d) 3 pC and 6 pC
[lEE M.tin '-L'pt. 2U]
2.152 PHYSICS-XII

18, A solid conducting sphere, having a charge Q, is 23. The electric field in a region is given by
surrounded by an uncharged conducting hollow E ={Ax+B)h where E is inNC
-1
and X is in metres.
spherical shell. Let the potential difference between the -ru i c *. .. ● . r,
f j The valuesofconstantsare A=20SIunitsand B = 10SI
surface of the solid sphere and that of the outer surface
^ i-u u 11 u 11 u 17 .1 1- „ ■ units. If the potential at r=l IS V, and that at :r =-5 s
u. u ^ i.- i *. i

of the hollow shell be V. If the shell is now given a y y _y ’


charge of-4 Q, the new potential difference between ^ ^
the same two surfaces is («) 180 V (&) -520 V
(fl)-2V {b)V (c) 320 V (d)-48 V [liU, M.ii:l Ap,i! A-l
(c)2V (rf)4V 24. The electrostatic potential inside a charged
lll'l' M.iin April l^'l
19. A uniformly charged solid sphere of radius R spherical ball is given by ^ = ar^ + b, where r is the
has potential (measured with respect to co) on its distance from the centre ; a, b are constants. Then the
surface. For this sphere the equipotential surfaces with charge density inside the ball is
3K 5V„0 3K0
potentials and
have radii R^,R^,R^ (a) -24 Tu (b) ~6a&Qr
4 ' 4
and respectively. Then (c) -24n(ie 0 (d) ~6as 0 [AIEEi:::;ll|

(a) R.^ =0and R^>{R^~R^) 25. The potential (in volts) of a charge distribution
is given by
(b) R^ ^0and{R^-R^)>{R^~R^)
(c) Kj =0 and R^ <(K^ - R^) V{z) =30 -5z^ for |z| < 1 m
(d) 2 R < R^ f!l i' M.iin 1 ->] V(2)=35-10|z| for jzl 2:1m
Based on Relation between Electric Field and V(z) does not depend on 2 and y. If this potential is
Electric Potential and an Equipotential Surfaces generated by a constant charge per unit volume pg (in
20. The potential at a point .r (measured in p m) due units of Eg) which is spread over certain region, then
choose the correct statement.
to some charges situated on the X-axis is given by
20 (a) Pg =40£q in the entire region
V{x) = volts
U'-4) (b) pg =20 Eg in the entire region
The electric field £ at 2 = 4pm is given by (c) pg =20eg for|2j in <lm and pg =0 elsewhere
10
(fl) — volt / pm and in the +ve x-direction. (d) pg =10eq for|2| in <lm and pg =0 elsewhere
del; M.iin !(.]

(b) ^ volt / pm and in the -ve x-direction. Based on Electric P.E., P.E. of Point Charges
and P.E. of a Dipole in a Uniform Electric Field
(c) ^ volt/pm and in the +ve x-direction. 26. The charges Q, +q and +q are placed at the vertices

(d) ^ volt / pm and in the -ve 2-direction. [AIFl-T :ai171


of a right-angle isosceles triangle as shown below.
Q
21. Assume that an electric field £ =302^ t exits in
space. Then the potential difference V^-V^, where Vg
is the potential at the origin and the potential at
2 =2m is

(fl)120 V (b) -120 V


(c) -80 V (d) 80 V [JFl M.iin Mi The net electrostatic energy of the configuration is
zero, if the value of Q is
22. An electric field £ =(25f+3o/)NC“^ exists i .. in a

22a
region of space. If the potential at the origin is taken to {a) - {h) -2q
V2+1
be zero, then the potential at2=2m,i/ =2m is
(a) -llOJ (b) -140 J (c) L
I + V2
(d) +q
(c) -120J (d) -130 J (JI ^ M.uii I-l [IL.i. Main j.in A'j
ELECTROSTATIC POTENTIAL AND CAPACITANCE (Competition Section) 2.153

27. Two point charges and -q are fixed on the 31. Four equal point charges Q each are placed in
x-axis at :r = — and a: = respectively. If a third point the xi/-plane at (0,2 ),(4,2 ),(4, -2) and {0, -2). The work
2
required to put a fifth charge Q at the origin of the
charge 'q' is taken from the origin to .r = fr along the coordinate system will be
Q'
2 /
Q 1 ^
(^) 1 +
‘I 2yj2-Ke 0 47te
0^ V3

Q'
2 /
4q -T Q 1 'l
I
(c) (d) 1 +
_d 47T6
0
4ne O'- V5
2 2
IIU Mtiin l.in in|

semicircle as shown in the figure, the energy of the 32, A uniformly charged ring of radius 3fl and total
charge will charge q is placed in .ry-plane centred at origin. A point

w
(a) decrease by —-—
2q^ charge q is moving towards the ring along die 2-axis
47T£«d
{b) increase by and has speed u at z = 4a. The minimum value of v such
0
3jrs„d
0
that it crosses the origin is

Flo
4q^ 3q^ \l/2 n1/2
(c) decrease by
3;ce«d
0
(d) increase by
47rEQd («) jlf 1 q^ (b)
2 1

ee
\m 15 4nSna m 5 47te«fl
(JM. .Main Si |>l, 2«\ n1/2 n1/2

Fr
2H. . X 2 4 2 2
Hydrogen ion and singly ionized helium atom (c) _ _ (d).- -
m 15 4Ke«fl \ m 15 4:rer,fl
are accelerated, from rest, through the same potential 0

difference. The ratio of final speeds of hydrogen and for


ur
\}t\- M.un April
helium ions is close to
33. A solid sphere of radius R carries a charge Q+q
(«) 2 : 1 (b)l:2
distributed uniformly over its volume. A very small
ks
(c) 10 : 7 (d)5:7 ||!'i: M.iiii S'pt. 2i'l
point like piece of it of mass m gets detached from the
Yo
oo

29. A charged particle q is shot towards another bottom of the sphere and falls down vertically under
gravity. This piece carries charge q. If it acquires a
eB

charged particle Q which is fixed, with a speed v. It speed V when it has fallen through a vertical height y
approaches upto a closest distance r and then returns. (see figure), then (assume the remaining portion to be
spherical)
r

Q
ou
ad

<? <
gQ
(a) v^- +g
> H—^ —*1
^ 4iieQR{R + y)m Q
Y

If q was given a speed 2v, the closest distance of qQR


(b) v^=2y
nd

+8
Re

approach would be
4n£Q(R + yfm q, m

(a) r (b) 2r I
Fi

W = +s
(.●MU'! 2ni'H

30. In free space, a particle A of charge 1 pC is held


c}Q
fixed at a point P. Another particle Bof the same charge (d) v^=2y +8
and mass 4 mg is kept at a distance of 1 mm from P. If B 4mQR{R + y)m lllliMam S,'pl. 20]
is released, then its velocity at a distance of 9 mm from
34. An electric field of 1000 Vm“^ is applied to an
P is Take = 9 x lO^Nm^C electric dipole at an angle of 45°. The value of electric
47160 dipole moment is 10"^^ Cm. What is the potential
(a) 2.0 X 10“^ m/s (b) 3.0 X 10^ m/s energy of the electric dipole ?
(fl) -9xlO"^‘^J (b) -7xlO'^^J
(c) 1.5 X 10^ m/s (d) 1.0 m/s
(c) -lOxlO'^^J
-13
lllii: \l.iin April M| (d) -20x10 J iJl.r.MainJnn ly]
\

'>2^:154 PHYSICS-XI!

35. Two identical electric point dipoles have dipole Tills question contains statement-1 and
—» A —^ A

moments = pi and P2 =-pi and are held on the statement-2. Of the four choices given after the
statements, choose the one that best describes the two
;c-axis at distance 'a' from each other. When released,
statements.
they move along the A^-axis with the direction of their
dipole moments remaining unchanged. If the mass of Statement 1 : For a charged particle moving from
each dipole is 'in', their speed when they are infinitely point P to point Q, the net work done by an electro
far apart is static field on the particle is independent of the path
1 connecting point P to point Q.
a
’^InEgWa a ^ KE^ma Statement 2 : The net work done by a conser
vative force on the object moving along a closed loop
P I i I 3~ is zero.
(c) —
a nsQina a
P nz^ma
0
IJEH M, J ● 'I
{a) Statement-1 is true, Statement-2 is false
36. A system of three charges is placed as shown in
(fc) Statement-1 is true, Statement-2 is true ;
the figure :
Statement-2 is the correct explanation of
D
Statement-1
d Q
(c) Statement-1 is true, Statement-2 is true ;
If D » d, the potential energy of the system is best Statement-2 is not the correct explanation of
given by Statement-1

(«)
1
^ 3Q£ (rf) Statement-1 is false, Statement-2 is true
4 TIE 0 d 47tE
0
■rf 'AM.

39. This question has statement-1 and statement-2.


+
ijQd Of the four choices given after the statements, choose
4nz 0 d Am
0 d 2D^ the one that best describes the two statements.

An insulating solid sphere of radius R has a


(ICE Ma!^.^p|'il 19]
uniformly positive charge density p. As a result of this
37. There is a uniform spherically symmetric uniform charge distribution, there is a finite value of
surface charge density at a distance Rq from the origin. electric potential at the centre of the sphere, at the
The charge distribution is initially at rest and starts surface of the sphere and also at a point outside the
expanding because of mutual repulsion. The figure sphere. The electric potential at infinity is zero.
that represents the best speed i>(R(f)) of the distri
bution as a function of its instantaneous radius R(f)is Statement 1. When a charge 'q' is taken from the
(a) (b) (JCE ,Ad\. i.;M 1?| centre of the surface of the sphere, its potential energy
-p(R(0) -v(R(/)) «?P
changes by
3e
0

Statement 2. The electric field at a distance r{r < R)


pr
from the centre of the sphere is 3e
0

Ra R(0 RD R(0
(rt) Statement-1 is true, Statement-2 is true ;
(^) (d) Statement-2 is not the correct explanation
of statement-1.
‘-viRit)) “v(R(0)
(b) Statement-1 is true Statement-2 is false,
(c) Statement-1 is false Statement-2 is true,
(rf) Statement-1 is true, Statement-2 is true,
Statement-2 is the correct explanation of
Statement-1.

R0 m Ro m I.AICCH ]2
ELECTROSTATIC POTENTIAL AND CAPACITANCE (Competition Section)

Based on Capacitance of Capocitors The capacitance will be close to


40. If 'C and 'V' represent capacity and voltage an an
1 1 +—
respectively, then what are the dimensions of X, where a 2d 0 V d)
C/V = X7
2/ 2/ Sow
(0^
an
(a) {b) 1-— 1-
a 4dJ a 2d
(c) (d) [IvT^rtV]
IJEE Main Jan 20]
[JIT Main Feb 21)
46. The charge on
41. If the capacitance of a nanocapacitor is
a capacitor plate in a 6
measured in terms of a unit 'u' made by combining the
circuit, as a function 3

electronic charge 'e', Bohr radius 'a^', Planck's constant of time, is shown in 4 ■
'h' and speed of light 'c', then 3 ■
e^h eh the figure.

w
2 ■
(fl) w = (b) u = What is the value
ea
0
ha
0
of current at f = 4 s ? 0
he eh.0 2 4 6 8
(c) u = -^ id) » = («)2pA

Flo
f(s)
e a he
0 [JEE Mflinl?) (b) zero
42. A parallel plate capacitor has 1 pF capacitance.

ee
(c)3pA
One of its two plates is given +2pC charge and the (ri) 1.5 pA [JEE Main Jan 19]

Fr
other plate, +4pC charge. The potential difference
47. A condenser of 2pF capacitance is charged
developed across the capacitor is
(fl)5 V (b)lV steadily from 0 to 5 C. Which of the following graphs
for
ur
represents correctly the variation of potential
(c)3V {d)2V
[JEE Moin April J9| {difference (V) across its plates with respect to the
43. A force of 10 N acts on a charged particle placed charge (Q) on the condenser ?
ks
between two plates of a charged capacitor. If one plate (fl) ib)
Yo

of capacitor is removed, then the force acting on that ^ t


oo

particle will be O o
eB

(fl) 5 N ib) 10 N ^5

(c) 20 N (rf) zero [JEF Main June 22]


2.5 ' 2.5
44. An electron with kinetic energy enters
r

between parallel plates of a capacitor at an angle 'a'


ou
ad

i » >
O
with the plates. It leaves the plates at angle 'p' with 5 Q(C) o
5 Q(C)
Y

kinetic energy K^. Then the ratio of kinetic energies (c (d)


Kj:K2willbe ^0 > +
O o
cosp
nd
Re

ib) >< 5 2S-5


sma cos a Is.
Fi

ic)
cos^p id)
sin^P 2.5 2.5
2
cos a cos^a [JEE Main Feb 21]
X
45. A capacitor is made of two square plates each of O
5 Q(Q o
5 Q(Q
side 'a' making a very small angle a between them, as
[JEE Main July 22]
shown in the figure.
Based on Grouping of Capacitors
48. A parallel plate capacitor is made by stacking n
equally spaced plates connected alternatively. If the
capacitance between any two adjacent plates is C, then
the resultant capacitance is
ia) («-l)C ib)in + l)C
2
a ic)C (d) nC [AIEEE 2003]
2.156 PHYSICS-XIl

●19. Figure shows 32. A10 pF capacitor is fully charged to a potential


charge (q) versus voltage difference of 50 V. After removing the source voltage it
(K) graph for series and is connected to an uncharged capacitor in parallel.
parallel combination of Now the potential difference across them becomes
two given capacitors. 20 V. The capacitance of the second capacitor is
The capacitances are (fl) 20 pF (b) 10 pF
(fl) 40 pF and 10 pF (c) 15 pF (d) 30 pF |MT S'p'
>
(b) 20 pF and 30 pF K(Volt) 53. In the circuit shown in the figure, the total charge
(c) 60 pF and 40 pF is 750 pC and tire voltage across capacitor Cj is 20 V.
(d) 50 pF and 30 pF lJi;i; M.iin ]u| C2
50. In the circuit shown, find C if the effective Cl = 15 nF
capacitance of the whole circuit is to be 0.5 pF. All the
values are in pF.
c 2
C3 = 8mF
Ag¬
io
+ -
2 2 1 ●o o-

V
11
2 2 2 Then the charge on capacitor C2 is
(c)|mF (a) 650 pC
(c) 160 pC
(b) 590 pC
(d) 450 pC IIH-: M.nn 2l'l
(d) 4 pF ■OB
54. In the given circuit, the charge on 4 pF capacitor
(JM. VKiiii l.m iy|
will be
51. Seven capacitors, each of capacitance 2 pF, are to IpF
be connected in a configuration to obtain an effective 4pF
fe \
capacitance of pF. Which of the combinations. 5pF
13

shown in the figures below, will achieve the desired


value ? (|M M.nn |,m P'| 3pF

(«) lOV

■o
(a) 5.4 pC (b) 9.6 pC
(c) 13.4 pC (d) 24 pC ||H- M.iin 19)
55. In the circuit shown, 2pF 4pF
charge on the 5 pF
capacitor is
(b) (fl) 16.36 pC 5pF
o- -o (b) 18.00 pC
(c) 5.45 pC (d) 10.90 pC
6V 6V
M.iin jn]

56. In the figure shown below, the charge on the left


(c) plate of the 10 pF capacitor is -30 pC. The charge on the
o- -o right plate of the 6 pF capacitor is il I M.iin [.111 I'ij
6 pF
(fl) -FlSpC
(b) -12 pC
(d) -o
(c) +12 pC
10 pF 2pF
(d) -18pC
4pF
A

ELECTROSTATIC POTENTIAL AND CAPACITANCE (Competition Section) 2.157 4W

57. In the given circuit, charge Q2 on the 2pF Based on Energy Stored in Capacitors and
capacitor changes as C is varied from 1 pF to 3 pF. Q2 as Loss of Energy in Redistribution of Charges
a function of C is given properly (figures are drawn 60. If the charge on a capacitor is increased by 2 C,
schematically and are not to scale) by []EE Main 15| the energy stored in it increases by 44%. The original
1 (iF charge on the capacitor (in C) is
(a) 10 (b) 20

C
(c) 30 (d) 40 [JEE Main June 22]
2mF 61. Effective capacitance of parallel combination of
F two capacitors Cj and C2 is 10 pF. When these
6
capacitors are individually connected to a voltage
source of 1 V, the energy stored in the capacitor C2 is
(fl) (h) Charge
4 times that of C^. If these capacitors are connected in

w
series, their effective capacitance will be
Qi Qi (fl)1.6pF ib) 4.2 pF

Flo
(c) 8.4 pF id) 3.2 pF [JEE Main )an 20]

C 62. Two capacitors Cj and C2 are charged to 120 V

ee
1 pF 3 pF 1 pF 3pF
and 200 V respectively. It is found that by connecting

Fr
(c) id) them together the potential on each one can be made
Charge Charge zero. Then

(«)5Cj=3C2 (b)3C, =5C2


for
ur
Q2 Q2
(d)9q=4C2
(c)3C,=5C2 PE Main 13]
63. Two capacitors of capacitances C and 2C are
ks
C
charged to potential differences V and 2V, respectively.
Yo

IpF 3pF 1 pF 3 pF These are then connected in parallel in such a manner


oo

that the positive terminal of one is connected to the


58. A combination of capacitors is set up as shown negative terminal of the other. The final energy of this
eB

in the figure. The magnitude of the electric field, due to configuration is


a point charge Q (having a charge equal to the sum of
the charges on the 4pF and 3pF
(a)~CV^
2 (b)^CV^
r

4pF
9pF capacitors), at a point
ou
ad

distant 30 m from it, would (c) zero (d)-CV^


6 [JEE Main Sepl. 20]
Y

equal 9pF 64. A capacitor C is fully charged with voltage Vq.


2pF
(a) 360 N/C
After disconnecting the voltage source, it is connected
nd
Re

(b) 420 N/C in parallel with another uncharged capacitor of


(c) 480 N/C F capacitance —. The energy loss in the process after the
Fi

(d) 240 N/C |]EE Main 16] 8V


charge is distributed between the two capacitors is
59. In the figure, is shown a system of four
capacitors connected across a 10 V battery. Charge that
will flow from switch S when it is closed is

(a) 5 pC from bto a 2pF 3pF [JEE Main Sept. 20]


{b) 20 pC from ato b 65. A battery is used to charge a parallel plate
(c) zero 5 capacitor till the potential difference between the
plates becomes equal to the electromotive force of the
(d) 5 pC from a to b
battery. The ratio of the energy stored in the capacitor
[JEE Main 15] 3pF 2pF
and the work done by the battery will be
F (a) 1/2 {b)l
lOV (c)2 id) 1/4 [AIEEE 2007]
V \

2.158 PHYSICS-XII

66. A fully charged capacitor has a capacitance C. It {n)K =


(Kj +K^)(K3 + K4)
is discharged through a small coil of resistance wire 2(k^ + Kj + K3 + K^)
embedded in a thermally insulated block of specific
(k^+K^)(K3 + K^)
heat capacity s and mass m. If the temperature of the {b)K=,
(KJ + K2 + K3 + K4)
block is raised by AT, then the potential difference V
across tiie capacitor is (k^ + K2)(K3 + K^) K1K2 K3K4
(c) K = {d)K =
12 mCAT mCAT K, +K2+K3 + K4 K^+K2 K3 + K4
(fl) (b)
s s

msAT 12 ms AT
71. Two identical parallel plate capacitors, of
(c) id) capacitance C each, have plates of area A, separated by
C V C [AIEEE 2liil5| a distance d. The space between the plates of the two
Based on Capacitors filled with Dielectrics capacitors, is filled with three dielectrics, of equal
thickness and dielectric constants ic,, 1C2 and K3. The
67. A parallel plate capacitor is formed by two firet capacitor is filled as shown in Fig. I, and the
plates each of area 30jrcm^ separated by 1 mm. A second one is filled as shown in Fig. II.
material of dielectric strength 3.6xlO^Vm”^ is filled
between the plates. If the maximum charge that can be
stored on the capacitor without causing any dielectric
K,
breakdown is 7x the value of dielectric constant
1
of the material is Use: =9xlO^Nm^C”^
4ti£ I
0 II

(fl) 1.66 (b) 1.75 If these two modified capacitors are charged by the
(c) 2.25 (d) 2.33 flEK Main June 22[ same potential V, the ratio of the energy stored in the
68. A parallel plate capacitor filled with a medium two, would be [ refers to capacitor (I) and £2 to
capacitor (II)]
of dielectric constant 10, is connected across a battery
E
and is charged. The dielectric slab is replaced by («)
1 _ 9KJK2K3
another slab of dielectric constant 15. Then the energy £2 (Kj + K2 + K3)(K2K3 + K3KJ + K|K2)
of capacitor will
(fl) increase by 50% (b) decrease by 15%
£2 (Kj +K2 + K3)(K2K3 + K3Kj + KJK2)
(c) increase by 25% (rf) increase by 33%
1)1:1 Main lime 22[

69. A parallel plate capacitor of capacitance 90 pF is K,K2K3

connected to a battery of emf 20 V. If a dielectric


5
material of dielectric constant k = — is inserted between
(rf) A = +
3
^2 9KJK2K3
li'l M.^m 1
the plates, the magnitude of the induced charge will be
(a) 1.2 nC (b) 0.3 nC 72. A parallel plate capacitor with plate area A has
separation d between the plates. Two dielectric slabs of
(c) 2.4 nC (d) 0.9 nC IJEE Main ISl
dielectric constant K, and of same area A/2 and
70. A parallel plate capacitor with square plates is thickness fr/2 are inserted in the space between the
filled with four dielectrics of dielectric constants k
1/^2' plates. The capacitance of the 9
K
3, K4 arranged as shown in the figure. The effective capacitor will be given by I+ + + + + + + + + +Q
dielectric constant k will be

I
{«)
0
1
- + va' K1

d 2 i d
I
I Kj L/2
o- ●o

I
U2 ib)
0
1
—+
K,fC2 [
K3 >^4 -Q
I rf 1^2 2(X, + i<2)J
6
ELECTROSTATIC POTENTIAL AND CAPACITANCE (Competition Section) 2.159

with a dielectric which has a dielectric


(c)
d 2
J coiistant that varies as k{x) - ^C(l + cor)
where 'x' is the distance measured from
one of the plates. If (oif)«l, the total
.id)
capacitance of the system is best given by
tJEE Main Aug 21]
the expression
73. In the reported figure, a capacitor is formed by . . AKz 0 aV
placing a compound dielectric between the plates of i^) (1 + ad) 1 +
2
d
parallel plate capacitor. The expression for the capacity
of the said capacitor will be AK& 0 ad'' fad^
2 A

(Given area of plate = A) Cl C2 C3 1+ 1 +


2 a 2 ')
Q
(«)

w
34 d 6 d IJEE Main Jan 20]
K 3k 5k
78. A parallel plate capacitor is made of two square
6 « 6 fl plates of side ‘aseparated by a distance d {d « a). The

Flo
d 2d 3d

IJEE Main July 21] lower triangular portion is filled with a dielectric of
dielectric constant k as shown in the figure.
74. The parallel combination of two air filled

ee
parallel plate capacitors of capacitance C and nC is

Fr
connected to a battery of voltage, V. When the
capacitors arc fully charged, the battery is removed
and after that a dielectric material of dielectric constant
for
ur
K is placed between the two plates of the first capacitor.
The new potential difference of the combined system is
V
ks
(«) (b) Capacitance of this capacitor is
2
Yo

(k + «) K + «
K8„fl
0
oo

(b) InK
(d)
hV ^ ^2 d d
(c)V
[JEE Main April 19]
eB

K + U 2
K6nfll
0
(c) InK (d)
75. A parallel plate capacitor having capacitance d(K-l) 2rf(K + l) [JEE Main Jan 19]
12 pF is charged by a battery to a potential difference of
79. If qj. is the free charge on the capacitor plates
r

10 V between its plates. The charging battery is now


ou
ad

disconnected and a porcelain slab of dielectric constant and is the bound charge on the dielectric slab of
6.5 is slipped between the plates the work done by the dielectric constant k placed between the capacitor
Y

capacitor on the slab is plates, then bound charge can be expressed as


{«) 692 pJ (b) 60 pj
nd

1 ^ 1
Re

(c) 508 p] (d) 560 pJ [JEE Main Jan 19] ' V Vk , (b)%=‘qf 1--
K
Fi

76. A parallel plate capacitor has plate of length 'I', n


width 'zv' and separation of plates is 'dIt is connected
1.4 id) % = ‘If 1 + -
Vk , k;
to a battery of emf V. A dielectric slab of the same
thickness'd' and of dielectric constant k = 4 is being [JEE Main July 21]
inserted between the plates of the capacitor. At what 80. A parallel plate capacitor with plate area 'A'
length of the slab inside plates, will the energy stored and distance of separation'd' is filled with a dielectric,
in the capacitor be two times the initial energy stored ? What is the capacity of the capacitor when permittivity
I I of the dielectric varies as
(^>2 f d\
&{x)-eQ + kx, for 0 < a: < - ?
21 / D
<">3 [JEE Main Sept. 20] /
d
^x<d ?
77. A parallel plate capacitor has plates of area A z{x) = ZQ + k{d--x).lox ~
separated by distance 'd' between them. It is filled
PHYSICS-Xll

n2/M
U Cl(^2-Vl)
kA -1
(^) (b) tan
2e^ + kd
21n ●● 0 mg (Cj+q)(rf-0
2e 0

(c) tan ’ X.
kA 2e 0 (q + q)((f-o
(c)0 {d)~\n
2s kd
\ 0
- ^ q(v, + q)
(rf) tan'^ -^x
!JEENTainJuly2ij mg (Cj+C2)(rf-0 [lEE M>rn luK :;!
81. A simple pendulum of mass 'mlength 7' and
charge '+q' is suspended in the electric field produced Numerical Value Type Questions
///////////////// 82. A 5 gF capacitor is charged fully by a 220 V
supply. It is then disconnected from the supply and is
\
connected in series to another uncharged 2.5 pF
Medium
capacitor. If the energy change during the charge
(K) X
redistribution is J, then value of X to the nearest
+V2©- m
o-V, 100
+<?
integer is .
Air
83. A parallel plate capacitor is A
I
made up of stair like structure b
A
with a plate area A of each stair
and that is connected with a wire
by two conducting parallel plates as shown. The
value of deflection of pendulum in equilibrium of length h, as shown in the figure. b
The capacitance of the A
position will be :r £nA h
0
arrangement is — . The value A
- X
(/i) tan ' — q(V2-^i) 15 b b
of X is
mg (q + q)(rf-/) II.I' M.iin luiK' 22| A

Answers and Explanations


Q = 100c 1 ‘1 1 10“^ X10'^
V.B =
= -100x1.6x10
-19
= -1.6x10 -17 c 4rcep r2 47iSq 2

AV= q-Vp =-4-10 = -14V ■■■ v,-q = o.


17 -16
.-. W = QAV = 14xl.6xlO = 2.24x10 J. 3.IW) A unit positive charge placed at the centre will be
repelled by charges at A and B, and attracted by charges at
2. (() The situation is shown in the figure. Cand D. The net field will be in the downward direction
when the charges are interchanged, the direction of the
-4(42,^) field is reversed. Electric potential, being scalar, remains
zero whatever be the position of the charges.
.'. £ changes while V remains unchanged.
i7=10'V 4. (,;)

r, = OA = =2
= 06 = 2-0=2

1 lO'^xlO"^
4ke 47t8 2
0 'i 0
ELECTROSTATICPOTENTIAL AND CAPACITANCE (Competition Section) 2.161

V4 kq
fc , cf V=-^
centre
x0=0 9. (/JJ For a spherical shell. ^0
R R
ka
V = — = constant, r ^ 1
Five equal electric field vectors are equally inclined Vx —
r
about the centre.
kq 1
^centre = ° and V = —i => V cc - , r > K0 ¥
r r ^0 r
5. (‘ /)
}.
10. {i) Potential at point Pdue to the charge q on the
o A dx B spherical shell is
X 1
V,1 =
2L 2L 4Tte R
1 dq _ I Xdx 0

4K8q X 4718^ ^ X Potential at point P due to the charge Q at the centre

w
of shell is
I

4to
[ln;c]f = 4K8 L 47re„L
1 Q - 2Q
0 0 0 ^2 = 4718
R/2 47r8()R

Flo
0
6.
Net potential at point P is

ee
+Q 12 -Q
p = y.+ \A =^— ^ . 2Q
R 4ti:8„R
4nsn

Fr
0
-4^
s B
A
11. 1.?) Let {/j and q2 be the charges on the inner and
\J outer shells respectively.
for
ur
kQ kQ Then, q^+q2~Q
^2
Also, a =
ks
47cr^ 4tcR^ r2
Yo

^B = -
v?+? R^
oo

(7

^1 = Q and 2/2 = Q
2fcQ 2kQ Q 1 1 r^+R^
eB

P. -V„ =
V?+^
A
a 2tU8
f)
a Electric potential at the common centre.
1 r+ R
7. !'●) p = Q.
47t8QLr^+ R^
r

fy 4tc8qL r R
ou
ad

■►E
12. '.<)
Q = Qi + Q2 + Q3
Y

Q = 47:(fl^ + l7^ + c:^)a


nd

- - -4 - - ►x

1 Qi , Q2, Q3
Re

o
v =
4tcs 27 /) c
OL
Fi

Point A is an equatorial point of the dipole. 1 r47W^ 47ib^ 47tc^


4ne a b c
OL
£ = -
kp p
and V =0.
d^ 4718.4^ Q
0 —(fl + ij + c)x47ix
8. i.n
4tc8
0 47T(f7^ + b^ + C^)
Q(27 + 17+C)
k X 4qa _ ki<2qa X
yp = 4mQ{a^ + b^ + (P')
~{x~ Rf R
■►X

2{x-Rf = x‘^ A B P
13. iM a, -02
J2(x-R)^x Q. Q2 Rl
{j2-l)x = ^R 4tcR^ 47r(4R)^ Q1
■J2R
X = -j=—.
V2-I
2.162 PHYSiCS-Xll

^Qi 18. {b)


V{R)-V(4R)r=
R
Shell

3kQ (uncharged)
4 R iQ

14. (ft) -Q/+Q

4::eQ [I? c 1 1
sphere -Khoii^'^Q = 7
1 4rai^a 4nb^a
4tC£ b b
OL
c
When shell is given charge -4Q, potential both in
its inside and on its surface changes by amount,
a a^-b^ _ k(-3Q)
+ c V.0 “
^0 b
^2
P
R,1
15. (c) Given —i1 _
sphere
+ .V
0
^2
v^'
kQ
shell ”
+ K0
^2
V, R^R, ^2)
1 1
16. (b) At equilibrium, ^ihere-V'sliell = kQ -- = V
L^'i ^2
Due to the presence of charge on outer shell, the
_ kq^
potential at every point inside and on the surface of the
Ra RB shell changes by the same amount. Hence, potential
=> difference between sphere and shell remains unchanged.
% Rr.B 10 2 kQ
19. (c), (d) Off the surface (r = R), =
R
E and £
kq B
rIA B ~ kQ
RlB Inside the sphere {r < R)^ V j(3R'-r^)
2R

Ea. = 5a = 2 :1. At the centre (r =0),


E!^=Ev => R,1 =0
17. (c) 2 R 2 °
At r= R^,
kQ
2R^ 4 R

5=3-^R
2
R

kQ
3—9 pC Outside the sphere (r > R),
r

3V,0 4
^1=^2 2R~ R At r= R., — R.,=~ R
^ 4 ^3 ^3 ^ 3
3 3

^1 “^'/2 At r=R,, -i Rj=4R>2R


^ 4- ^4 ^4
2 ^2'*■^2=^1*'^
= 3 pC R,-R. =4R iR=?-R>R,
4 3 3 3 2
and = 6 pC. Hence, options (c) and (d) are correct.
ELECTROSTATIC POTENTIAL AND CAPACITANCE (Competition Section) 2.163

20
20. f.'I Given V(x) = volts This implies that the source is an infinitely large
x^-4 non-conducting thick plate of thickness 2 m.
dV
£ = -
dx
40.T
= 20{x^-4)‘^x2x = Pq j IOgq p„ = 0
(x--4)2 Po = 0
Atx = 4pm,

£ = 40x4 ^160
(16-4)^ ~ 144 z = -1 m z = 0 Z-+1 m

= ^ V /f.im, in the +ve x-direction.


For\z\ >lm.
a
£ =

w
2e
21. h ) dV = -E.dx 0

Volume charge density x thickness


2e
dV = -{3Qx^dx

Flo
0

^0 0 -£o^ = 10
2e

ee
0
or
V^-Vo = -[10a;3]^=-80V.

Fr
22. (lO Potential atX =2m, y=2mis 26. lu-i
x =2 y=2 2 2

j£^.rfx- Eydif = -jl5dx - 30dy for


ur
V = -
0 0 0

= ~[25x]l -[30y= -50 -60 = -110 V.


ks
Yo

->
oo

23. i.'i dV = -E.dr =-{Ax+ B)dx


eB

dV = - {Ax+B)dx
^2 -5
r

[x^T
ou

1
ad

-B[x]i-
2
J-5
J2q
Q =-
Y

20
J2+I'
= -y[l-25]-10[l + 5] 27. (() Potential due to charge -q is same both at
nd
Re

= 240-60=180. initial and final positions of charge q. It does not change


P.E.
24. i. /1 Potential, ^ = ar^ + b
Fi

d(j) P.E. decreases due to charge 4q due to the decrease


Electric field, £ = = -2*7r
dt in its potential.
Using Gauss' law. kx4q cfx4(/]_ 4q^
4 3 d/2 ~3d/2 37te„rf
0
-itr^
-2arx 4kt^ = =i> p=-6fl6Q. 28. (h)
^0
Gain in K.E. = lmv^=qV
2
25. i./i

For |z| < Im, =


2qV
-1
V(2)=30-5z^ => £=10zVm m

For lz| >lm. e 4m


= 2:1.
-1
V(z)=35-10iz| => E=10Vm V
Ht
V m e
2,164 PHYSICS-XII

29. (d) WJten the charge q is shot with a speed v. Work required to put a charge Q at the origin O,
At the distance of closest approach r.
2 /
Q 1
W = QV = 1 +
Initial K.E. of charge q = P.E. of cliarges q and Q 4 TIE
0^ ^^5J'
1
-mv 32. id)
2 r
q

When the charge q is shot with a speed 2v.


4
Let / be the distance of closest approach. +
- -►x
+ 4a V q
Then +' '+
2 )’ -p +
+

or or
r 4 By conservation of energy.
30. (rt)
9 mm kq^ 1X
+ -mv
2 _kq^
= +0
I mC
1 mm
oImC 4l6a^+9a~ 2 3a
po -o- ■►y
B 8'
1
mv
By conservation of energy. 2 <7 V3 5 15a

n1/2
AB AF 2
i; =
2 2 *7^
m 15 47te„fl
1 i_- o*V

77/ AB AB'.
33. (d) Total energy of m at highest point
2x9xl0^xl0~^^r 1 1 = Total energy of 7?/at lowest point
4x10'^ LlxlO"^ 9x10"^ kQq kQq 1 2
+ 777^^ = -—^ + -77/17'^
-6 R R+y 2
9x10 8 -6
X- =4x10
2 9
i;2=:2gy +
i? = 2x 10~^m /s. 77/R(R + y)

31. id)
=2y ●

4Y 47t8gR(R + y)77i
(4,2) 34. (f7) U = -y£cos0
(0,2) n
Q Q -29
10 X1000 X cos 45°
1 -26
= -10 X0.707J =-7xlO“^^J.
>X
0(0,0) 1 2 3 4 2p
35. (fl) U = -p.£, £ =
47re,.77^
0
-1

Pi = P‘ P2^-pi
Q Qj > <■ - -►x
(0,-2)
(4,-2) a

Potential at the origin O, K. + U. = K^ + U /


I^Q.kQ kQ kQ 2p^cosl80° ^ 1
0- —=2x- 7711’^+ 0
2 2 .^"2 + 42 ^2^ + 4^ 4TCE„r?
0
2

1 Q 1
= 1 + 1 + 1
45 J 4718 o'-
17 =
V5 av 2ns„ma
0
ELECTROSTATIC POTENTIAL AND CAPACITANCE (Competition Section) 2.765

4-2
36. (li) 42. (/>) 0 —

cap
U - P.E. dipole + P.E. due to interaction
of charge Q with the dipole = 1 V.
C
kq'^ + QV
d 2 |iC 4

kp kqd
+

+
dipole tP- o- ■o

kq^ kqQd _ 1
+

U=- +

d 47IE 0 d 3 pC -1 pC 1 pC + 3 pc

37. ((?) Total energy of the charge distribution 43. i.n In between the two plates of the capacitor,
remains constant at every instant. p = (^E = q—=lON

w
^0
U^ + Kj = U. + -K I

-^ + -nw =-^ +0
When one plate is removed, F =qB =q 2e = 5N.

Flo
0
2R 2
2^ 44. lo

.^2

ee
+ + + + + +
V =
I

Fr
t V
1 E I

As R increases, v increases and attains a finite value. I


I

38. (/’) Work done by conservative force does not


for
ur
depend on the path. Electrostatic force is a conser Velocity component perpendicular to E i.e.,
vative force.
horizontal component will remain unchanged.
39. (t‘) Electric field at r < R is
iJjCosa = U2COSP
ks

(br. = Ex 4nr^ =~ - —x — rtr^p


Yo

^ E Ecfg 3J K cos^p
oo

1 _
bg
~ “ 2
, E = i^ Statement 2 is true.
V2 cos a
eB

3e
0 45. (^0 The capacitance of the element of thickness
When charge q taken from the centre to the surface dx will be
of the sphere, the work done is EnOdx
r

0
ou
ad

R R -i« d + ax lix
Pr qp ?
W = qEdr = q dr=^ rdr =
The elemental capaci
Y

-T tan a = xa
3e 3e 3e 2 6e itt
0 0 0; 0 0 0 Jo 0
tances are connected in X

qpR^ parallel.
nd

ALi=W = ^ Statement 1 is false.


Re

d
6e
0 C = |rfC
Fi

40. {/’) X=- C^QT^^_Q^_Q\2 _Q^ _OTf a


£n(ldX
0
a

V V rw W^ (Fs)2 0
d+ax

Q) E^a d+cm
[ITf = [M-V‘*lVl.
0

a
[ln(rf + ox)]g =-^ln a d
[MlV^]' 2_2
aa a a

41. id)[u]=[er[aj’[hncf a
In 1 +
d } a d 2d^

x^
-c-dA‘^ ] ln(l + ,v) = .T- —+., , for X «1

(7=2, b=l r = -l, fr = -l


e a
2 Er.P
0
■ GUI
0
u =
d 2d
he
2.166 PHYSICS-XII

46. (/;) At^ = 4s,


51. (./) Ceq — HF
dq 13
/ =—
dt
= Slope of q-t graph = 0
Three capacitors must be in paraUel to get 6 in .
47. Q = CV
o- o

=> V-Q graph is a straight line with,


1 1 1 1 1 2.
C..eq 3C C C C'*’C
+ —+ —+
1 1
slope = — = 5x10^
C 2xl0~^
3C
C
Q1 _
eq
13 13
r = 2.5x lO^V.
^ C 2x10"^ 52.

48. (a) When the alternate plates of capacitor V Total charge


consisting of n plates are connected, it gives rise to a
c»mmon

Total capacitance
parallel combination of («-l) capacitors, each of 10xl0"®x50
capacitance C. 20 =
-6
(10 + C2)xl0
Resultant capacitance =(n-l)C.
10 + C2 = 25 => C2=15pF.
49. (ii) From graph A for parallel combination,
53. fh '?3 = C3x20V
500pC
Cj + C2 - lOV
= 50pF
= 8pFx20V = 160 irC

From graph B for series combination, qi = q-q^ =750—160 = 590 pC


^ qC2 _ 80 = 8pF 54. I ’i

C, + C2 ~ 50pF “ 10 4x6
4 + 6
= 2.4 mF

CiC2 = 400 4pF 6pF

On solving, q = 40 pF, Cj = 10 pF.


50. ((?)

C 2 2
3pF

Ag¬ I-
io V
2
Q4MF = Q6^F=2.4pFxl0V=24pC
4
55.
OB V,0
152.
2pF 4pF
11
+ q
Ao OB
5nF
c 7/3

1
OV
3 1 6V 6 V

q = q^+qj
3

14C = 7 + 3C
5q=2(6-Vo)+4(6-y,)
36
K0 = —V
11
C = —pF
11 36,. 180
q-5 pFx V = pC = 16.36 pC.
11 11
ELECTROSTATIC POTENTIAL AND CAPACITANCE (Competition Section) 2.Z67

56. (r?) Total charge on parallel combination of 6 pF 59. 1,(1 Before switch S is closed
and 4 pF capacitors =30 pC. C,1 =
2x3 6 „

6 pF 2+3 5

30 jiC + -12jiC +12 pC


a
+ +
-o-
+
+
10 pF 2pF 6V 4V
10 V
o- -o
4pF
4V 6V

Q=CV => Q°cC -o-

-12 pC ^ +12 pC
Charge on 6 pF capacitor = — x30 pC= 18pC
6 +4 3x2
6 i:
^2 = 3 + 5

w
5
Charge on the right plate is +18 pC.
57. ib)Ceq
3C
3 +C
q = q = |nF

Flo
3C Cj and C2 are in parallel. Charge on each (Cj,C2)
Total charge, Q= e will remain same.
\3+ C,

ee
Charge on 2 pF capacitor.
(y = CjV = C2V=-pFxlOV=12pC
5

Fr
2 3C€ 2C6 2g After switch S is closed
Q2 = —Q
2+1
=
3
— X

3 + C 3+C 3
1 + C, =2+3=5 pF C2=3+2=5pF
for
ur
C
-10 pC +15 pC
dQ d^Q a

f<o
■0-
Clearly, ->0 and
dC dC
ks
10 V
o- 5V 5 V ■o
Yo

=> Q2 - C curve has a deaeasing slope.


oo

Hence, option (b) is correct. ■O' J.


eB

58. {b) -15 pC ^ +10 pC


3pF
4pF 2pF 12 pF
C^ and C2 are in series,
r

5x5
C = = 2.5 pF
ou

o- -0=0- .0
ad

5+5
6 V 2V
Charges on combinations Cj and C2 will be same.
Y

9pF

q=CV=2.5 pFxl0V=25pC
Charge on 4pF capacitor
nd

These charges split on individual capacitors in


Re

Qj =4pFx6V=24pC
proportion to their individual capacitances as shown.
Fi

Charge on 12 pF capacitor = =24pC 5 pC charge will flow from b to a.


9
Charge on 9pF capacitor = —
12
x24pC 60. u(i U-I =
2C
Q2 = 18pC (^ + 2f
U
f~ 2C
Total charge on 4pF and 9pF capacitors,
But LZ, = U:I + 44% of U: = 1.44 U..I
Q=Qi+Q2=42pC
Electric field at a distance of 30 m, = ].44x
*7^
2C 2C

q+2=fTMq = \.2q
9x10% 42x10"^ => 2=0.2q
= 420N/C.
30x30 q = IOC
2.168 PHYSICS-XII

61. (<0
Cp = q + q = io^iF 66. 1‘0 Energy released by the capacitor
= Heat gained by the block
Given 4 -Cxl^ 4C,=q
U ^ j 2^ '2 msAT
or -cv^ - msAT or V^ =
C| = 2 (iF, C2 =8 |iF 2 C

2x8 67. (d)C = -^ = KC0


= 1.6pF. V
q + C2 8+2
0
= K
62. (lO For potential to be made 120 c 1 Ed d
+
zero, after connection 7x10"^
K =
1
120Cj =200C2 e.TlE
0
gx307txl0“^x3.6xl0^
471x9x10
3Cj =5C2- 200
+
7x36
= 2.33.
30x3.6
63. ((0

C
Ci}
=C+2C=3C CV
+
68. f.M
U-=^K,CV^ ; Uf=^K,CV^
_ Total charge Percentage increase in energy,
common
c
eq

+
AU
xl00 =
j(K^-K,)CV^ xlOO
4CV-CP
3C
= v
2Cx27=4CV
U.
I
\k^cv^
15-10
xl00 = X100 = 50%.
U^=~{3C)xV^=~CV^.
2
K
1
10

69. U)
64. (c ) Qo = CoVo
C C

t=> Q„duced = Q-Q) = '^q''ofi-r KJ

5 3^
0
C = -x90xl0~’^x20x 1- C
2 3 5J
=:1.2xlO*^C=1.2 nC.
Q) = <^^o 2 2
70. l.')
K-
>^2
u I
I
1C / 2x3C I
1 Kj LI2
o ■o O o-
Cl C2 ■o
2 I C3 C4
AL/= (i,-U
I
1-- ●«3 I >^4 U2
2C. 3 6C
●^4

6 C 6 O’ C
aj
q +q q+q
65. i') Let V be the emf of the battery. Work done by
the battery. EoA/2
q = KjC, where C =
dll
w = QV =
Energy stored in the capacitor. ^2 — ^2^C, — K^G
u=-cv^ £rsA\
0 K3K4 &.AH
0
K
1 eq
d J Kj+K2 K3 + K4J d/1
1
II
__2 CV^ 1 K
K1K2
W CV'^ ~2 eq
Kj + K2 K3 + K4
ELECTROSTATIC POTENTIAL AND CAPACITANCE (Competition Section) 2J6V

71.1,0 74. (,o After full charging, battery is disconnected.


o

C.V nC,V

K2 ^2 Total charge = CV + nCV


I
^3
After the insertion of the dielectric.
6 o
I II
kC, Vc nC,
1 1 1 1
6«A 8„A
Cl

w
0 0 0
K K K
^d/3 ^d/3 ^rf/3 ^total _(»-^l)CV _(n + l)V
C kC + nC K + JI
'eff
d 1 1 1
75. (c) Charge on the capacitor plates remains same

Flo
— +
3e.A
0 _KIi K2 K3 after the battery is disconnected.
1 Qo' 1 Qo

ee
3e.A
0 w = u.-u
Ci = I
^ 2 C0 2 kC 0
d KjK2 + <2*^3

Fr
1
ZnAI3 SnA/3 BnAI3 = 1r 1- [Q. = CoV^o]
Cz = K -y +K^~y + K.,-^ 2 ^0^0 ^ Kj
d ^ d ^ d
for
ur
1
= -xl2xl00 1-
2 6.5
^IpJ
[k^ +
_

3d U
ks
= 600 X — = 507.69 pj 508 pj.
1 13
Yo

E
CV^
oo

1
1=1 76. (,/)
£2 2ic2V^ (k^ +K2 + K3)(k^K2 + K2K3 + KjK3) T
eB

l-x

o- ■o I o- ■o

72. i.M 0 T
K X
r

C,1 = W r _L
ou

d ' ^2
_
ad

d ’ AH
AH Cl d/2 1
Y

c =
0 u =
d C
2d A/2 2
^2 ah u. c
f f
nd

Ciq
Re

=2
c + c
etj aI c/
C1+C2
Fi

6
KEnWX
0 SQiy(/->r)
e„A £„A 1 d
0 _ ‘'0
—+ => ^ =2
{K-^ + K^)d 2d d 2 zJw
0

73. (,0 J_=J„ + 1 +


]
=> 4x + {l-x) = 2l [k=4]
C
'“m
~ C
'“I C2 C3 I
eq
X =

d 2d 3d 3
X dx
K£.,A 3K£r.A 5k8.,A
0 0 0 77. k{x)~ Kfl + ox)

d

r,] +-+-
2 3 d
—^x —
34
dC = K(l + ojOsqA A
K8.,A
0 3 5J K8„A
0 15 dx O' ■o

15ks^A
0
All such capacitors are
C connected in series.
eij 34d
2.! 70 PHYSICS-XIl

d
1 dx 79. (/') Net field inside a dielectric.
C K(l+ ca)sQA
c<i 0
^ ^ ^ind
+<T
1 1
d.T ^=E..-E 0 ind +

KsqA'1 + ax K
+ +

1 +

[In(l + -ar)]g E.w = Eol Ky


4-
£ £ ind
4-
Ksr,Aa
0 4-

1 a, /
1
4-
/
In(l + cd) 1-- 4-
Ksr.Aa
0 Eq^. Kj
Using expansion. 1
5t
ln(l + .t)=.T- As^0 As 0 Kj %=y 1
2
+... for X «1 and taking x = ad, K

80. i/'i
\^d/2^d/2~M
1 1 .
ad- —adf d
1
ad Consider an element of
C Ks^Aa 2 J Ke.A 2 J thickness dx at distance x
eq 0 0

from left. All such elements ®- ●o


-1

C ^^0^ ad ^
1 +
ad are in series.
eq d 2 d 2
1 r dx
mi-
78. I. I C ^ As dx
eq

d/2
dx dx

0 A(&Q-i-kx) dl2 A[e^ + k(d-x)]


1 d/2 1

Ak
[In(ep + cfr)]o Ak \n[sQ + k{d-x)^f^^
X a a 1 r, f kd) -Ins
d
dx=—dy 4* I ” 2 J 0
V d
/
kd\
s^adx KE^adx —^ Ins 0 -In s„+
i)
C1 =-2 ^2 =
0 Ak 2J
d-y ' V
2
In Sq + kdf2
1 1 y Ak
^0
dC s„adx
0
K
kA
C
eq \ ●
Sr^adx
0
2s.0 + kd
dC 2ln

d + yf---l
etj
2s
0
VK
81. l i)
a
C dC.. =e^a—
0
eq eq 0
d^ 1 T T cos 0
0 d+y -1
t
K
I
e
t
2
E^a
0 1 ><j£
In d + y —1 T sin 0

4^ VK
1
K
0

mg

^Infl
2
K£.,fl
0
InK. o- ■o
d(l-K) d(K-l) +V2 -VI

C2 C,
ELECTROSTATICPOTENTIAL AND CAPACITANCE ^' rro^^cHtion Section) 2J7i

Consider empty region of thickness (d-t) of 82. 04.00


C = 5^F
capacitance Cj and that with dielectric with In first case :
capacitance C^. Let £ be tlie electric field in the empty
region.
Then in equilibrium position, U:I = Vn = 220 V
2C
T sin 0 = i^£
In second case:
T COS0 = mg
C = 50mF

tan0 =
qE eq 2 2

mg
U
Charge on each capacitor. f 2x3C
- = 2.5 uF

w
2

Q = [q-(-q)]
qq AU=U:-U
I
^ 2C 3C

qq _ieg _i (cv,f

Flo
6 C 6' C
q +q
= lry2 -x5xl0"^x220 x 220J

ee
P.D. across C 6 0 6
1'

Fr
V,
_ Q q(q + q) =s> j=4xio~q X = 4.

' q (q + q) 100

83.
^1 _ q(q+q) for
ur
£ =
Tliree capacitors form a parallel combination.
d-t (q + C2)(d-f)
^eq “ q q q
q(q + q)
ks
tan0 = u
Yo

(c^ + q)(d-f) 5b 3b b
oo

3 + 5 + 15^eQ/l _23 e^A


e = tan"’ 2Lx q(q + q) 15 J b ~15‘ b
eB

(q+q)(i^-0 x = 23.
r
ou
ad
Y

Based on Electric Potential due to a dipole, situated on a line making an angle of 60® with
Point Charge, Group of Point Charges, the dipole axis is
nd
Re

Electric Dipole and 1


= 9xlO^NmqC^
Fi

Charged Spherical Shell 4716 0

1. Four point charges -Q, -q,2qai\d 2 Q are placed, (fl) 200 V (^)O
one at each comer of the square. The relation between (c) 400 V (d) 50 V [NEET201
Q and q for which the potential at the centre of the 3. A conducting sphere of radius R is given a charge
square is zero is Q. The electric potential and the electric field at the
i<^)Q = -q ib)Q = -~ centre of the sphere respectively are
Q Q
(«) and
4tc8„R 47C6^R^
{c)Q-q (d)Q = ~ 0 0

[ '.ip\n Q
(Ij) both zero (c) zero and
2. A short electric dipole has a dipole moment of 47te„K^
0

16 X10“^Cm. The electric potential due to the dipole at (d)


Q and zero
a point at a distance of 0.6 m from the centre of the 4jc8„R
0 [AIPMT i41
2.172 PHYSICS-Xll

4. The variation of electrostatic potential with radial The electric potential is


distance r from the centre of a positively charged (fl) maximum at A (b) maximum at B
metallic thin shell of radius R is given by the graph (c) maximum at C
(«) ib)
V“
(d) same at all the three points A, B and C.
V/i.
[NEKT 13]

9. The mean free path of electrons in a metal is


-8
4x10 m. The electric field which can give on an
average 2 eV energy to an electron in the metal will be
in units of V/m
11 -n
{a) 5x10 (f))8xl0
(c)5xl0^ (d)8xl0^ ICBSE PMT 2009)
V 10. The electric potential at a point (a:,i/,2) is given
2 3 ->
by V = -2 1/ + 4. The electric field E at that point is

(fl) £ -xlx\j + ] {x^ + i/)+fc (3xz-i/^)


A o A A 1
(b) E = I z + j xyz + k z~
l\i i 1 :i'|
(c) £ = 1 {2xy-z^) +} xu^ + ic 'dz^x
5. Three concentric spherical shells have radii a, b
and c(a <b <c) and have surface charge densities o> -a (d) £ = f (2 xy + ^ 3xz^ ICBSE PMT 2009]
and a respectively. If V^, and denote the 11. In a region, the potential is represented by
potentials of the three shells, then for c-n + b, we have
y(^:,y,z)=6i'-8A:i/-8i/ + 6i/2, where V is in volts and
{b)Vc^VB:^V^ x,y,z are in metres. The electric force experienced by a
(c)y^=Vg = y^ ICBSE charge of 2 coulomb situated at point (1,1,1) is
6. Two hollow conducting spheres of radii and (fl) 24 N (&) 4V35N
R^i R| » i?2) equal charges. The potential would be (c)6V5N (d) 30 N [AlPMT 14]
(a) more on bigger sphere
12. If potential (in volts) in a region is expressed as
(1?) more on smaller sphere V(a:,y,2)=6A:y-i/+2y2, the electric field (in N/C) at
(c) equal on both the spheres point (1, 1, 0) is
(d) dependent on the material property of the
(a) -(6 /’ + 9 ; + A: ) (/;) -(3i'+5/ +3fc )
sphere IXEETJulv 221

Based on relation between Electric Field and


(c) -{6('+5/+2fc ) (d) ~{2i+3j+k)

Electric Potential and on Equipotential Surfaces lAIPMT Re 15]

7. In a certain region of space with volume 0.2 m^, 13. The diagrams below show regions of
equipotentials.
the electric potential is found to be 5 V throughout. The
magnitude of electric field in tliis region is 20V 40V 20V 40V lOV 30V
40 V-

(a) 0.5 N/C (b) 1 N/C 20 V

(c) 5 N/C (d) zero |\l ! 1 2"i A


8. A, B and C are three points in a uniform electric A B A B /t' B A B
field.

» A 10 V
B 30V
£ lOV 30V lOV 30V 20V 40V
(«) ib) {c) (d)
●C
ELECTROSTATIC POTENTIAL AND CAPACITANCE (Competition Section) 2.173

A positive charge is moved from A to B in each


diagram.
(«) Maximum v^ork is required to move q in
figure (c).
A
(!;) In all the four cases the work done is the same.
(c) Minimum work is required to move q in midpoint between A and B. The work done in moving
figure (fl). a charge +Q along the semicircle CRD is
(d) Maximum work is required to move q in qQ
(a) ib)
figure (b). (NEET17) 2 718/,0 L 6;i8„0 L

Based on Electric P.E., P.E. of Point Charges


(c)- id)
and P.E. of a Dipole in a Uniform Electric Field

w
67t8/,L
0
4 718/,0 L [CBSE 2007]
14. Six charges + q,-q, + q,-q, + q and -q are fixed at 18. Four electric charges + ‘l -4
the corners of a hexagon of side d as shown in the + q,+q, -qand -qare placed at

Flo
figure. The work done in +q -q the corners of a square of side
--A
bringing a charge to the 2L (see figure). The electric

ee
centre of the hexagon from potential at point 4 midway

Fr
infinity is (Sq = permittivity of ●qo +q
between the two charges + q +,J -q
free space) d
and + q is

(<i) zero (b) -


-q 1 2q 1 Iq 1
(^) (1 + V5)
for (b) 1 +
ur
4jt8/,d
0 4718/,0 L 4718/,0 L VsJ
1 ^ 1 ^ 1 2q 1
ic) - 3—r id) - 3— (d) zero
ks
(0 1-
47t8/,ri 4Tt8„d
0
J2 4718/,0 L Vs [CBSE 2011)
Yo
0
oo

[NEET Sepl. 22]


19. Three charges, each
15. In bringing an electron towards another are placed at the corners
eB

electron, the electrostatic potential energy of the of an isosceles triangle ABC


system of sides BC and AC, each
equal to 2fl. D and £ are the
r

(ti) increases (b) decreases


ou

midpoints of BC and CA
ad

(c) remains unchanged (d) becomes zero


[CBSE 991
The work done in taking a
Y

charge Q from D to E is
16. A point charge +qis placed at the origin O as 3qQ
(a) zero ib)
nd

shown in the figure. Work done in taking another 47t8/,rt


Re

point charge -Q from the point A(0,rt) to another point 3tjQ


(c) id)
Fi

B((7,0) along the straight path AB is 87l8rtfl


0 0 [CBSE 2011]

(«) 20. Identical charges {-q) are placed at each corner


^4tiCo of a cube of side b. Then, the electrostatic potential
energy of charge (+q) placed at the centre of the cube
(b) -
47t8Q
^ -^'V27I
a~
will be

Aj2q^ 8j2q~
(a)- (b)
1 ^q1 a
718
0 TCCpb
ic)
_47t8„ «2 V2
id)
8V2q^
ic) -
[CBSE 2005]
V3 7I8/,b 47re„b
0 [CBSE 2002]
(d) zero 0

17. Charges +q and -q are placed at points A and B 21. An electric dipole of moment ‘p' is placed in an
respectively which are a distance 2 L apart, C is the electric field of intensity £ . The dipole acquires a
2.174 PHYSICS-XII

position such that the axis of the dipole makes an angle A parallel plate air capacitor is charged to a
0 with the direction of the field. Assuming that the potential difference of V volt. After disconnecting the
potential energy of the dipole to be zero when 0 = 90®, charging battery, the distance between the plates of the
the torque and the potential energy of the dipole will capacitor is increased using an insulating handle. As a
respectively be result, the potential difference between the plates
(a) p£sin0,-pEcos0 (1j) p£sin0,-2p£cos0 (fl) decreases (b)increases
(c) p£sin0,2p£cos0 (d) p£cos0,-p£cos0 (c) does not change (e) becomes zero

22. A dipole is placed in an electric field as shown.


In which direction will it move ? Three capacitors each of capacitance 4 pFare to
(fl) Towards the left as its ;r
be connected in such a way that the effective capaci
potential energy will ^ tance is 6 pF. This can be done by connecting
increase. '' (fl) all of them in series, (b) all of them in parallel,
(&} Towards the right as ^ ' ● ● (c) two in parallel and one in series,
-*■£
its potential energy (d) two in series and one in parallel.
will decrease. " ^ A network of four capacitors of capacitances
(c) Towards the left as its equal to q=C, C2=2C, C^=3C and Q=4C are
'A
potential energy will
decrease,

(d) Towards the right as its potential energy will


increase. C3 C,

Based on Capacitance of Copocitors


23. Two metallic spheres of radii 1 cm and 3 cm are h

given charges of -lx 10”^C and 5 x lO'^C, respectively. V

If these are connected by a conducting wire, the final connected to a battery as shown in the figure. The ratio
charge on the bigger sphere is
of the charges on C2 and is
{rt)2xl0“^C (b)3xlO~^C / ^ 4 ,,, 3 , , 7 ,^,22
(fl) - (b) — (c) - (d) —
(c)4xlO‘^C (d)lxlO"^C 7 ^ ^ 22 4 ^ ^ 3
24. The electrostatic force between the metal plates Three capacitors each of capacitance C and of
of an isolated parallel capacitor C having a charge Q breakdown voltage V are joined in series. The capaci-
and area A, is
tance and breakdown voltage of the combination will be
(«) independent of the distance between the plates c
(^)3C,- W-,3V
(b) inversely proportional to the distance between
the plates
(c) 3 C, 3
3 3
(c) proportional to the square root of the distance
between the plates Four capacitors of 25 pF each are connected as
(d) linearly proportional to the distance between shown in the figure. If the d.c. voltmeter reads 200 V,
the plates charge on each plate of the capacitor is
25. A parallel plate air capacitor has capacity 'C', (r7)2xl0'^C
distance of separation between plates is 'd' and
(b) 5xlO“^C
potential difference 'V is applied between the plates.
Force of attraction between the plates of the parallel (c)2xlO“^C
plate air capacitor is
cv^ cv^
(d) 5xlO~^C
(«) ib) (c) (^)-
2d^ 2d 2d
ELECTROSTATIC POTENTIAL AND CAPACITANCE (CjinpeLitior, Section) 2.175

Based on Energy Stored in Capacitors and TO. Two condensers, one of capacity C and the other
Loss of Energy on Redistribution of Charges of capacitor C/2 are connected to a K volt battery, as
31. A capacitor of capacitance Cj is charged upto
potential V and then connected in parallel to an
C C/2
uncharged capacitor of capacitance €2- The final
potential difference across each capacitor will be
cv
1
(«) ib)
Ci + C, c, + q shown. Tire work done in charging fully both the
( c condensers is
(c) 1 + ^ V 1/
C C
i> IUT-! (b)-CV^
2

w
32. The dimension of where e0 IS

permittivity of free space and £ is electric field, is (C)|CV^ (rf) 2 CV'^ [CB5I‘

(fl) ML^T”^ {b) T*^ 37. A series combination of capacitors, each of

Flo
ii. lisi i’:. value Cj, is charged by a source of potential difference
-1
(c) (rf) MLT

ee
4 V. When another parallel combination of
capacitors,
33. In a parallel plate capacitor, the distance
between the plates is d and potential difference across each of value C2, is charged by a source of potential

Fr
difference V, it has the same (total) energy stored in it,
plates is V. Energy stored per unit volume between the
as the first combination has. The value of C2, in terms
plates of capacitor is
for
ur
1 v'^ of Cj, is then
2V^ 2q (b)
«2
(«)
ks
n
”l"2 1
1
Yo

(c)z 16C 1
«2 C
oo

2
0 (02 1 id)
”1 «1«2 ICBSEi’v' ; ;
34. A parallel plate capacitor has a uniform electric
eB

field £ in the space between the plates. If the distance 3«. A capacitor is charged by a battery. Tire battery
between the plates is d and area of each plate is A, the is removed and another identical uncharged capacitor
energy stored in the capacitor is is connected in parallel. The total electrostatic energy
r
ou
ad

of resulting system
(0) To£' (b) E^Adle 0
(rt) increases by a factor of 4
Y

(1j) decreases by a factor of 2


{c)jC„E^Ad (d) Sf^EAd
(c) remains the same
nd
Re

35. A capacitor of capacitance C = 900 pF is charged (d) increases by a factor of 2 1\ki:t 171
Fi

fully by 100 V battery B as shown in Fig. (p). Tlien it is 3". A capacitor of 2pF is charged as shown in the
disconnected from the battery and connected to
another uncharged capacitor of capacitance C =900 pF diagram. When the switch S is turned to position 2, the
as shown in Fig. (q). The electrostatic energy stored by percentage of its stored energy dissipated is
the system (q) is 1 2

(a) A3xlQ-^] ?S

{b) 3.25x10"^] + 2pF 8 nF


C
(c) 2.25 X 10"^ J
100 V
(d) 1.5xlO“^J —o o-

(NEETJuly 22] (a) 0% (1?) 20%


B
(c) 75% (d) 80% \\VA:\ 1--I
(P) (4)
2.176 PHYSICS-XII

Based on Capacitors filled with Dielectrics 44. A parallel plate air capacitor of capacitance C is
40. Polar molecules are the molecules connected to a cell of emf V and then disconnected

(rt) having zero dipole moment from it. A dielectric slab of dielectric constant k, which
can just fill the air gap of the capacitor, is now inserted
(&) acquire a dipole moment only in the presence of
in it. Which of the following is incorrect ?
electric field due to displacement of charges
(c) acquire a dipole moment only when magnetic (n) The energy stored in the capacitor decreases k
times.
field is absent
1
(d) having a permanent electric dipole moment (b) The change in energy stored is -1
V.K
[NHCT211
41. A parallel plate capacitor with oil between the (c) The charge on the capacitor is not conserved.
plates (dielectric constant of oil, k=2) has a (d) The potential difference between the plates
capacitance C If the oil is removed, then capacitance of decreases k times. [AH’Mr 15]
the capacitor becomes
C
45. A parallel plate capacitor having cross-sectional
C
(a) -J2C {b)2C (c) area A and separation d has air in between the plates.
[CBSE iy‘w| Now an insulating slab of same d H

42. The capacitance of a parallel plate capacitor area but thickness d/2 is inserted
with air as medium is 6 pF. With the introduction of a between the plates as shown in
dielectric medium, the capacitance becomes 30 pF. The *be figure having dielectric K ■O

permittivity of the medium is constant k(= 4). The ratio of new


(Eq =8.55x10 capacitance to its original
-12
capacitance will be
(a) 1.77x10 C^N“’m'- H-d/2H
(rt) 4 : 1 (&) 2 : 1
(b) O.44xl0'’®C2N'’m"“ (c) 5.00 C^N
-1
m
-2
(c) 8 ; 5 (d) 6 : 5 INEET 20]

(d) 0.44x10-^3<~2jsj-1jj^-2 INEET 201 46. A parallel plate capacitor of area A, plate
43. Two thin dielectric slabs of dielectric constants ^^P^^'^bon d and capacitance C is filled with four
Kj and K2(k^ <><2) are inserted between plates of a dielectric materials having dielectric constants k 1' ^2'
parallel plate capacitor, as shown in K3 and as shown in the figure below. If a single
Q
the figure. The variation of electric + A/3 A/3 A/3
+
field £ between the plates with +
+
distance d as measured from plate P +
+ d/1
is correctly shown bj' +
+

£ {b) £

dielectric material is to be used to have the same


0 d 0 d
capacitance C in this capacitor, then its dielectric
ic) (d) constant k is given by
3 1
£ £ (n)-
K K] +K2 -I- IC3
1 1 1 3
(b)- — + — + — +
K K
1
K
2 ^3 ^^4
(C) K = + K2 + K3-I-3k^
0 d d
2
(d) K = -(K, +K2 + K3) + 2k4
[AIPMT 14]
INEET 16 1I|
ELECTROSTATIC POTENTIAL AND CAPACITANCE (Competition Section) 2.177

Answers and explanations


1. (n) Potential at the centre O is 6. (fc)
47re„i) R

V = k -Q..^ -Cl 2^ , 2Q =0 1
a!^ alyfl a/y/2 n/>j2 For same q, V ■.x —
R

-Q~q + lq + 2Q=^0 As Rj » R,, so V2 > Vj


Potential would be more on smaller sphere.
-Q 2Q
7. (d) As the electric potential is constant
' -«/j2 / throughout the volume.
\ /a/j2 dV
17

w
£ = - = 0.
fl/j2/0\ dr
/ rt/j2\
8. (!?) Electric field acts in the direction of decreasing
~q a
2q
potential.

Flo
=> Q+q=0 => Q = -q- V,>V^>V^.

ee
1 pcosB 2
2. (n) V= 9. (c) E = ~

Fr
2 -8
4tis 0 1 d 4x10

9xl0^xl6xl0‘^xcos60° 9x16 = 0.5x10^=5x10^ Vm"^


V = V
(0.6)2 2x0.36
for
ur
dv 6v c- ay A ev A
= 200 V. 10. (d) t i - —I k
dr dx d\j dz
ks
3. (d) At the centre of the conducting sphere,
Q \{2xy-¥z^)i+x^f +3xz2^].
Yo

y = and £ = 0.
oo

47T8,,R
0 11. (/’) V =6.Y-8.Y]/-8y+ 6yz
eB

q
4. (c) ,Vinside -— = constant (r<R) At the point (1, 1, 1), we have
47ie„R
0 dV
E = -(6-8y)=2
1 .r
CX
r

V => Vx~ (r>R)


outside
47ie,.r
ou

r
ad

0 dV
= -(-8y-8 + 6z) = 10
Option (c) is correct.
Y

5. (d) From the solution of Problem 10 on page 2.82, £. = -


dV
= -6y - -6
we get dz
nd
Re

Va = {a-b+ c) £ =
^E;+E^+E^ =V4 + 100+36 =Vi^
Fi

^0

CT
^
a
2 = 2^/^NC -1
^8=- b +c
F^qE==2x2^/^=4^/35N.
a a^-b^ „ , ai/A avA
12. (c) £ =-—i —)
ev^k
+ c
dx dz
£oL J
£ =-(6y)/-{6Y-l+22)/-(2i/)^:
+ c

^0 - c
At the point (1, 1, 0),
a
{a-b+ c) I'.' a+b-c]
£ =-6/-5/-2£ =-(6i'+5;+2A')NC“\
^0
13. (b) W = q(V^ = -10) =30g,
is same in each case.
2.178 PHYSICS-XII
'●■JS*

14. («) (I
20. ( , ) Half diagonal of the tube,
Net potential at the centre, ■yjb^ + b^ + b^ Sh
^0 = 0 / \
--X-- d--V<?
r =
2 2
Work done in bringing the
\ fr' 'r.f / P.E. of the charge +q at the centre due to eight
charge from infinity to the
-q
charges (each = -q) at the corners of the cube is
centre.
1
W=q,V,=0. U=Sx
47t6g r
15. (fl) Work has to be done against the force of
repulsion in bringing an electron towards another = -8x
1

electron. Tlnis increases the electrostatic potential ATZZQ-l3bll -JSneQb


energy of the system.
1 q 21. [,;) T = p£sin0
16. {d) A
4jre« a 0 li = -pE cos 0.

W=-Q(Pg-y^)=0. 22.,'i
17. (c)

P
«
► Decreasing
+q, ^ £
'●vf F

Field lines are closer around charge +q, so £


decreases towards right.
=> Net force acts toward right
U = -p£cosl80° =+p£
Dipole moves towards irght due to decrease in P.E.

Work done in moving the charge +Q along the 23- ..^0<;,„„,=4xlO-2c


semicircle CRD is
Charge on a sphere oc Capacitance oc Radius
qQ
W= + Q(V'n-K-)= + Q 0 ^mall ■ %ig ^ ^
0
6jts«0 L
‘Ibi■8 = -x4xlO'^C = 3xlO"^C.
T, 1 1 1 1 4
18. (c) ^ ~ —^ ^
47r8gLf. f- VSL V5L 24.:,; F=QE = Q
a
=Q
Q ^
2e
0
2 Ae.0 2 Ae 0

47reg£L
1--^ . f is independent of the distance between the
plates.
19. (-:)
4^
V L ~V D _ 4 '_L _i. j_vrj_ _L j_' 25.,,, F —qE = qy —-
~
47iCg [, EA EB£cj ..DA'^ DB'^ DC, IZ.A
0
lZr.A
0

0
But q = CV and C = => Er.A = Cd
_L J_ J_"' 1 1 1
=0 d
0

4Tce EA^ £B^ EC Veb'^ Ea'^ EC


CV^
f =
W = Q(V^-l/o) = 0- 2Cd 2if
ELECTROSTATIC POTENTIAL AND CAPACITANCE (Competition Section) 2A79‘

26. (/?) As the battery is disconnected, q = constant. 33. (/)) Energy stored per unit volume in a capacitor,
Ef, A 1 y ^2 1
New capacitance, C'=-^ u =
2 °
— Ef,
2 \d

2 '
But d' >d, so C'< C
1 { EnA"l
34. ^CV^ = E^d^=-^.E^Ad.
Now V' =-^ 11
2l d 2»
c
As capacitance decreases, so potential difference 35. Common potential.
increases. -12

y =
+ ^2^2 _ 900x10 xlOO + 0
= 50 V
27. (, The series combination of two capacitors 1800x10
-12

should be connected in parallel with the third


-12
capacitor. Then U X 1800x10 x{50)“

w
4x4
c + C3 = 4 + 4 + 4=2 + 4 = 6pF.
= 2.25xlO'®J.
q +q
28. (^)Th^ series combination of C^, C2 and C3 is in 36. The equivalent capacitance of the two con

Flo
densers connected in parallel is
parallel with C^. The equivalent capacitance Cj23 of the
series combination is given by c=c.^=^

ee
J__J_ 1 + 1 = —+ 1 1 1 11 2 2

c.."c/q

Fr
123 C3 C 2C 3C 6C The work done in charging both the capacitors is
or c
6C stored as potential energy.
123 ”
11
-c
for =-f — =-cv^.
ur
w = u =
2V 2 4
Let ^y|,^y2/43 sod be the charges on the respective 2

capacitors. Since in a series combination, charge on all


37. (f/)
ks
the capacitors is same.
Yo

6cy
^123 ^ ^ “
oo

1 c
11 or
2 n1
^(4y)^
eB

Also, the charge on the capacitor q,


16 q
^4 = qy = 4cy or
<^2 =
«1«2
r

6cy/ii ^ 3
ou

38. (^0
ad

^4 4Cy 22' After connection


Before connection
Y

C
29. f/i) = y+y+y=3y
1 1 1 1 c
= —+ —+ => ceff “ ^ ● c
nd
Re

Ceff c c c 3

I-
Charge on each plate of the capacitor,
Fi

30.
C

Q = Cy = 25 X10'^ F X 200 V = 5 X 10"^ C. Initial energy, U.


1 2

31. (h> The common potential difference across the


After connection,
parallel combination of two capacitors,
_ 9i+*?2 _ ^ + 0 _ ^
y' = qviqqq ^c =
q +q Cj + C2 C+C 2C 2
●. y' =
qv Final energy.
But q=y, y2=o \2
q +q Li =-icy2
1 71 f 2 [2) 2 u 4

32. .■;) — = energy density 1 2


Loss of energy = LI.-Uj =— CV
ML^T"^
= [ML-^T"^1.
l3 The total energy decreases by a factor of 2.
2.180 PHYSiCS-Xll

39.{d) 45. (c) Original capacitance.


Initial energy stored =~{2\iF)V^ e„A
C - "^0
"" d
Loss of energy after connection,
After inserting the dielectric,
1 *^1*^2 ^.2._ 1^2x8 _1 d/2 . d/2
2C^+C^ 2U+8
d d 5d
% loss of energy 100 = 80% 2s„A
h
8s,^A 8e^A
0 0 0

40. (d) In polar molecules, the centres of positive C


8e.A
0

and negative charges are separated by certain distance. eij 5d


So these molecules have permanent electric dipole C
0 _
moment. - = 8 : 5.
C 5
c cq
med
41. (rf) fC =
C vac 46. (rt)
c
med „ '~-
Tire given arrangement can be redrawn as shown in
cvac
K 2 the figure below.
42. (&) 0

C £ A/3 A/3 A/3


med _ med
c.
air

30 '^2 ^2 d/2

^med g
-12
= 44.25x10 ci0.44xl0"’'^C^N"^m-'^.

Q
43. (fj) In a region of vacuum, E = d/2
£,^A
0

Q
In a region where dielectric is present, £ =
ke^A
0

^ r X —1
£
K
(d/2) 3
As Kj < K2, decrease in E is greater for K2 than for e^A
K 0
r
C2 = K

44. (c) Charge on the capacitor plates remains


{d/2) ^
constant, Q = CV
i = J- J;
V' = C C
1
K K

d 1 1
c = Q_ Q = kC
V V/k s„Ak
0 fo A 8„A
0
-.-(KJ + K2 + K3)
(d/2) 3 d/2 ^
K

W = -CV'^
1
= -kCx
(V^
2 2 V K d 2 3 1

K1 + K2+K3
llci/^ = u K
'<4
k2 K

U'-U=U ri-i 1 n -1 .
Ik ; 2 VK

Hence, only option (c) is incorrect.


C H APTER

w
Currenf Elecfricify
' '7'

Flo
ee
Fr
If the current is steady i.e., the rate of flow of charge
3.1 CURRENT ELECTRICITY
does not change with time, then
for
ur
1. What is current electricity ? Electric charge or i=^ Q
Electric current =
Current electricity. In chapters 1 and 2, we studied Time t
die phenomena associated with the electric charges at
ks
rest. The physics of charges at rest is called where Q is the charge that flows across the given area
Yo

electrostatics or static electricity. We shall now study in time t.


oo

the motion or dynamics of charges. As the term current A conductor may have different areas of cross-
eB

implies some sort of motion, so the motion of electric section at different points along its length. However,
charges constitutes an electric current. the current I will be same for all cross-sections of the
The study of electric charges in motion is called current conductor. This is a consequence of the law of
r

electricity. conservation of charge.


ou
ad

Lightning, which is the flow of electric charge


3.2 ELECTRIC CURRENT between two clouds or from a cloud to the earth, is
Y

an

2. Define electric current. example of a transient current (a current of short


duration). But the charges flow in a steady manner in
nd

Electric current. If two bodies charged to different


Re

devices like a torch, cell-driven clock, transistor radios,


potentials are connected together by means of a
hearing aids, etc.
conducting wire, charges begin to flow from one body
Fi

to another. The charges continue to flow till the 3. Give the SI unit of current.
potentials of the two bodies become equal. SI unit of current is ampere. If one coulomb of charge
Thefloiv of electric charges through a conductor constitutes crosses an area in one second, then the current through that
an electric current. Quantitatively, electric current in a area is one ampere (A).
conductor across an area held perpendicular to the direction 1 coulomb
1 A = lCs”^
1 ampere = or
offlow of charge is deifned as the amount of charge f owing 1 second
across that area per unit time.
Ampere is one basic SI unit. We shall formally define it
If a charge AQ passes through an area in time t to in chapter 4 in terms of magnetic effect of current.
t + Af, then the current I at time t is given by Smaller currents are expressed in following units :
/ = lim
Af ^0 At dt
1 milliampere = l mA = 10”^A
1 microampere =1 pA ^lO'^A
(3.1)
3.2 PHYSICS-XII

The orders of magnitude of some electric currents Examples based on


we come across in daily life are as follows : Definitions of Electric Current
Current in a domestic appliance -1 A Formulae Used
Current carried by a lightning -10^ A
1. Electric current = or I = -
Current in our nerves -10"^A = l)iA Time t
tie

4. Distinguish between conventional and electronic 2. As q = ne, so J = —


t
currents.
3. In case of an electron revolving in a circle of radius
Conventional and electronic airrents. By con r with speed v, period of revolution of the electron is
vention, the direction of motion of positive charges is j,^2nr
taken as the direction of electric current. However, a V

negative charge moving in one direction is equivalent to 1 V


Frequency of revolution, v =
an equal positive charge moving in the opposite T 2nr
direction, as shoum in Fig. 3.1. As the electrons are Current at any point of the orbit is
negatively charged particles, so the direction of electronic
I - Charge flowing in 1 revolution
current {i.e., the current constituted by the flow of xNo. of revolutions per second
electrons) is opposite to that of the conventional ev
or j =e \ =
current.
iTcr

Units Used
Conventional current Electronic current
Electric charge is in coulomb (C), time in second
0—► 0-* ©—* ©~* -© -G -0 -0 (s), and current in ampere (A)
Constant Used
I *■ I
Charge on an electron, e = 1.6 x 10”'^C.
Fig. 3.1 Flow of negative charge is equivalent to the
flow of positive charge in the opposite direction. Example 1. 10^° electrons, each having a charge of
1.6x 10"^^C pass from a point A toxvards another point B in
0.1 s. What is the current in ampere ? What is its direction ?
5. /s electric current a scalar or vector quantity ?
Solution. Here n =10^®, e = 1.6x 10 f =0.1 s
Electric current is a scalar quantity. Although
electric current has both magnitude and direction, yet Current,
it is a scalar quantity. This is because the laws of j-l-HE- 10^°xl,6x 10“’^C = 160 A
ordinary algebra are used to add electric currents and ~ t 0.1s
the laws of vector addition are not applicable to The direction of current is from B to A.
the addition of electric currents. For example, in
Fig. 3.2, two different currentsof 3 A and 4 A flowing Example 2. Show that one ampere is equivalent to a flozo of
in two mutually perpendicular wires AO and BO meet x 10^® elementary charges per second. |CBSE D 92C]
at the junction O and then flow along wire OC. Solution. Here /=1A, f=ls, t’=1.6xl0
-19
C
The current in wire OC is 7 A which is the scalar
As 1 = ^=^
addition of 3 A and 4 A and not 5 A as required by t t
vector addition.
.’. Number of electrons.
It 1x1
A 11 = —
-19
= 6.25x 10^®.
3A e 1.6x10

Example 3. How many electrons pass through a lamp in


C
90°
O 7 A
one minute, if the current is 300 mA ?
(Himachal 95 ; Punjab 02]
4 A
Solution. / =300 mA =300x 10 ^A,
B
f =1 minute =60 s, e = 1.6 X 10

Fig. 3.2 Addition of electric currents is scalar. As j=l=


f t
CURRENT ELECTRICITY

Number of electrons. Solution. Amount of charge that flows in 10 s


It 300 X10"^ X 60 = Area under the I-t graph
n = — = 1.125x10^®.
= |x 5x5+ (10-5)5 =37.5 C
-19
e 1.6x10

Example 4. How many electrons per second flow through a Example 8. The amount of charge passing through cross¬

filainent of a 120 V and 60 W electric bulb ? Given electric section of a wire is q{t) = at^ + bt + c
power is the product of voltage and current. (j) Write the dimensional formulae for a, b and c.
Solution. Here 120 V, P = 60W, f = ls
(ii) If the values of a, b and c in SI units are 5, 3 and 1
As P = VI, therefore, I = —
p 60
= 0.5 A respectively,find the value of current att = 5 second.
V 120
Solution, (i) Given q{t) = at^ + bt + c
Number of electrons, AT
Dimension of a = X = AT
-1

It 0.5 X 1 t^ j2
= 3.125 X 10^®.

w
n = —
-19
e 1.6x10
Dimension oib = —
q _ AT = A
t T
Example 5. In the Bohr model of hydrogen atom, the
electron revolves around the nucleus in a circular path of Dimension oi c = [q]= AT

Flo
m at a frequency of 6.S x 10^^ revolutions
-11
radius 5.1 10

per second. Calculate the equivalent current.


(I'O Current, I = ^=
dt
—dt + bt + c)=2at + b

ee
Solution. Here r = 5.1 X 10"^'m, At f = 5 s, / =2 X 5 X 5 + 3 = 53 A.

Fr
V =6.8 X 10^^ rps, e
Current,
= 1.6x10 -19 c
7^. roblems For Practice
for
1. One billion electrons pass from a point P towards
ur
[ = e V = 1.6 X 10“^^ X 6.8 X 10^^ = 1.088 x 10“^ A.
another point Q in 10'^ s . What is the current in
Example 6. In a hydrogen atom, an electron moves in an ampere ? What is its direction ?
orbit of radius 5.0 x 10“^^ m with a speed of 2.2 x 10^ ms~^.
ks
-19
(Ans. 1-6 X 10"^ A, direction of
Yo

find the equivalent current. (Electronic charge = 1.6 x 10 current is from Q to P)


oo

coulomb). [Roorkee 84]


-11 2 If 2.25 X 10^^^ electrons pass through a wire in one
eB

Solution. Here r = 5.0x10 m.


minute, find the magnitude of the current flowing
11=2.2x10^ ms"\ e = 1.6xlO"^^C through the wire. [Punjab 02] (Ans. 0.6 A)
Period of revolution of electron, 3. A solution of sodium chloride discharges
r

-11 6.1 X10^^ Na* ions and 4.6 x 10^^ Cl" ions in 2 s. Find
2nr 2tix 5.Qx 10 ^
ou
ad

T =
V 2.2x10^ the current passing through the solution.
(Ans. 8.56 x 10 ^ A)
Y

1 2.2 X 10^
Frequency, V = —
-11 4. An electric current of 2.0 pA exists in a discharge tube.
T 2tcx 5.0 X 10
How much charge flows across a cross-section of
nd
Re

2.2x7x10
17 the tube ill 5 minutes ? (Ans. 6.0 x 10 C)
= 7x10^^
Fi

2 X 22 X 5 5. In a hydrogen atom, the electron makes about


Current, Z = ev = 1.6 X 10
-19
x7xl0
15 0.6 X 10^^ revolutions per second around the nucleus.
Determine the average current at any point on the
= 1.12 X 10"^ A. orbit of the electron. (Ans. 0.96 mA)
Example 7. figure 3.3 shows a plot of current I through 6. An electron moves in a circular orbit of radius 10 cm
the cross-section of a wire over a time interval of 10 s. find with a constant speed of 4.0 x 10^ ms Determine
the amount of /(X) the electric current at a point on the orbit.
-12
charge that flows (Ans. 1.02 X10 A)
through the wire 5
7. In a hydrogen discharge tube, the number of
during this time
protons drifting across a cross-section per second is
period. 1.1 x 10^®, while the number of electrons drifting in
[CBSE OD 15C1 the opposite direction across another cross-section
O 5 10 l(s)
is 3.1 X10^® per second. Find the current flowing in
Fig. 3.3
the tube. (Ans. 0.672 A)
3.4 PHYSICS-XII

HINTS
A steady flow of electric current in a conductor is
ne 10^ X 1.6x10'^^ maintained in a similar way. As shown in Fig. 3.5,
1. / = = 1.6x10'^ A
t 10"^ positive charge flows spontaneously in a conductor
ne 2.25 X10^° X 1.6x10' -19 from higher potential (A) to lower potential (S) i.e., in
2. /=
t 60
= 0.6 A the direction of the electric field. To maintain steady
current through the conductor, some external device
3. / = /cations
{«■* + « )e must do work at a steady rate to take positive charge
anions
t
from lower potential (6) to the higher potential (A).
_ (6.1xitf^ + 4.6xl0^6) X 1.6 X 10“ Such a device is the source of electromotive force (emf)
19

= 8.56x10“^ A
2 which may be an electrochemical cell or an electric-
4. = =2.0x10"^ x5x60 = 6.0xl0"‘‘C generator. A source of emf transfers positive charge
form lower potential to higher potential i.e., in the
5. / = ve = 0.6 x 10^^ X 1.6x10'^^ opposite direction of the electric field. Clearly, a charge
= 0.96 X lO'^A = 0.96 mA flow circuit is analogous to the water flow circuit.
6. T=
2itr 2nx0.10 1 4x10^
S-'
V ~ 4x10'r® r”2nx0.10 + Source of emf

4x10^x1.6x10"'^ B -{Chargepump)
I =ve = = 1.02x10"'^ A. R
2kx0.\0
AAV
('V + ^
t Fig. 3.5 A closed charge flow circuit.
(1.1x10'®-h3.1x10'®)x1.6x10 -19

= 0.672 A 3.4 ELECTROMOTIVE FORCE : EMF


I
7. Define emf of a batterif. Is it really aforce ? When is
3.3 AAAINTENANCE OF STEADY CURRENT the emf of a battery equal to the potential difference
IN A CIRCUIT between its terminals ? Defineemfofl volt.
Electromotive force. A battery is a device which
6. With the help of a mechanical analogy, explain how maintains a potential difference between its two ter-
thefloiv of electric avrent is maintained in an electric ciraiit. minals A and R
Maintenance of steady current in an electric circuit.
F.
The flow of electric current in a circuit is analogous to
the flow of water in a pipe. As shown in Fig. 3.4, *■
suppose we wish to maintain a steady flow of water in A B

a horizontal pipe from A to R As pressure at A is


higher than that at B, so water flows spontaneously Fig. 3.6 A schematic diagram of a battery.
from the upper tank to the lower tank. To maintain a

steady flow, a water pump must do work at a steady Figure 3.6 shows a schematic diagram of a battery,
rate to pump water back from the lower tank to the certain chemical reactions, a force (of non¬
upper tank. Obviously, the water pump makes water electrostatic origin) is exerted on the charges of the
flow from lower to liigher pressure. It helps to maintain electrolyte. This force drives positive charges towards
the pressure difference between A and R terminal A and negative charges towards terminal R
Suppose the force on a positive charge q is As the
●U
● n \
charges build up on the two terminals A and B, a
potential difference is set up between them. An electric
field E is set up in the electrolyte from A to R This field
Water
h
pump 0 exerts a force F^,=qE on the charge q, in the opposite
A B direction of In the steady state, the charges stop
accumulating further and F^^ = F^ .
The work done by the non-electrostatic force during
the displacement of a charge q from R to A is
W = ,F,d
Fig. 3.4 A closed water flow circuit.
where d is the distance between the terminals A and R.
CURRENT ELECTRICITY 3.5

The work done per unit charge is In case of a closed circuit, we can define emf in
another way as follows :
7 (J Vie emf of a source may be defined as the energy supplied
by the source in taking a unit positive charge once round the
The quantity t=W/qis called the electromotive force complete circuit. Again, we note that
or cm/of the battery or any other source. . Work done W
emf = 6 =
The electromotive force of a source may be defined as or

Charge
the work done by the source in taking a unit positive charge
from lower to the higher potential. Literally, emf means the force which causes the
flow of charges in a circuit. However, the term emf is a
If the two terminals of the battery are not connected misnomer. The emf is not a force at all. It is a special
externally, then case of potential difference, so it has the nature of work
done per unit charge.

w
FJ=F^d = qEd = qV SI unit of emf is volt. If an electrochemical cell supplies
an energy of 1 joule for the flow of 1 coidomb of charge
where V = Ed is the p.d. between the two terminals. Thus, through the whole circuit (including the cell), then its emf is
said to be one volt.

Flo
€ = = y
7 (J
3.5 EMF VS. POTENTIAL DIFFERENCE

ee
Hence the emfof a source is equal to the maximum potential 8. Give important points of differences between
difference between its terminals when it is in the open circuit electromotive force and potential difference.

Fr
i.e., when it is not sending any current in the circuit. Differences between electromotive force and
Basically, an electrochemical cell consists of two potential difference
electrodes P and N immersed in an electrolyte, as
for
ur
Electromotive force Potential difference
shown in Fig. 3.7
1. It is the work done by a It is the amount of work
C / R I D
source in taking a unit done in taking a unit
A/A—^—?
ks
I »-

charge once round the charge from one point


Yo

complete circuit. of a circuit to another.


I P N I
oo

2. It is equal to the maxi Potential difference


R

A/W mum potential diffe may exist between any


eB

rence between the two two points of a closed


terminals of a source when circuit.
+ it is in an open circuit.
V
r

A B 3. It exists even when the It exists only when the


Cell symbol
ou
ad

circuit is not closed. circuit is closed.


4. It has non-electrostatic It originates from the
Y

Electrolyte
origin. electrostatic field set up
by the charges accumu
lated on the two termi
nd
Re

Fig. 3.7 An electrochemical cell connected to an external nals of the source.


resistance and the symbolic representation. 5. It is a cause. When emf It is an effect.
Fi

HerePp >0andl^^-Vg =-V_ <0. is applied in a circuit,


potential difference is
caused.
The two electrodes exchange charges with the
6. It is equal to the sum of Every circuit
electrolyte. Consequently, the positive electrode P potential differences component has its own
develops a positive potential >0) with respect to across all the compo potential difference
its adjacent electrolyte marked A Tlie negative nents of a circuit inclu across its ends.

electrode N develops a negative potential - V_ (V_ >0) ding the p.d. required
to send current through
with respect to the adjacent electrolyte B. When no the cell itself.
current flows through the cell, the electrolyte has the
7. It is independent of : It depends on :
same potential throughout, so that the potential dif (f) resistance of the circuit (i) resistance
ference between the two electrodes P and N is
(/i) internal resistance of (ii) internal resistance of
Vh -(- V )= v; -H V =S, the emf. the cell the cell
(Hi) current in the circuit (Hi) current in the circuit
Obviously, -i- V_ >0.
3.6 PHYSICS-Xll

3.6 OHM'S LAW : RESISTANCE Fixed or ●Wv—


resistor

9. State Ohm's law. D(^ne resistance and state its SI unit. Variable or

Ohm's law. On the basis of his experimental resistor 7"

observations, a German physicist George Simon Ohm Potential or


wv—
derived a relationship between electric current and divider
potential difference in 1828. This relationship is known I
as Ohm's law and can be stated as follows :
Meters —^ —
The current flowing through a conductor is directly Voltmeter Ammeter Galvanometer
proportional to the potential difference applied across its
ends, provided the temperature and other physical Fig. 3.9 Symbols for resistors and meters.
conditions remain unchanged.
Thus, Potential difference cc Current 10. Briefly explain how can we measure the resistance
Vocl
of a wire.
Measurement of resistance. Fig. 3.10 shows a simple
V = Ri
or
circuit for measuring the resistance of a wire. Here the
battery and ammeter are connected in series with the
The proportionality constant R is called the resis
tance of the conductor. Its value is independent of V wire and the voltmeter in parallel with it. The ratio of
and I but depends on the nature of the conductor, its the voltmeter reading (V) and the ammeter reading (/)
gives the resistance (R) of the wire.
length and area of cross-section and physical con
ditions like temperature, etc. Ohm's law may also be Battery Ammeter

<S>
+ +
expressed as I t
V'
— = R
I / '■

R
The graph between the
'VW-
potential difference V ' ’

applied across a conductor v-» +

to the current I flowing 0


through it is a straight line, Fig. 3.8 V-I graph for an Voltmeter

ohmic conductor.
as shown in Fig. 3.8. Fig. 3.10 To measure resistance of wire.
Resistance. The resistance of a conductor is the property
FACTORS AFFECTING THE
by virtue of which it opposes the flow of charges through it. 3.7
The more the resistance, the less is the current I for a RESISTANCE : RESISTIVITY

given potential difference. It is equal to the ratio of the poten 11. What are the factors on which the resistance of a
tial difference applied across the conductor to the current conductor depends ? Define resistivity and state its SI unit.
flowing through it. Thus Factors affecting the resistance. At a constant
R = temperature, the resistance of a conductor depends on
1
the following factors:
SI unit of resistance is ohm (Q). If the potential 1. Length. The resistance R of a conductor is directly
difference (V) is 1 volt and current (7) is 1 ampere, then proportional to its length i.e.,
the resistance (R) is 1 ohm. Razl
1 volt
1 ohm = 2. Area of cross-section. The resistance R of a uniform
1 ampere
conductor is inversely proportional to its area of cross-
or m = i VA"^ section A, i.e.,
I
Roc —
Thus, the resistance of a conductor is said to be 1 ohm if A
a current of 1 ampere flows through it on applying a 3. Nature of the material. The resistance of a
potential difference ofl volt across its ends.
conductor also d«=‘pends on the nature of its material.
Any material that has some resistance is called a resistor. For example, the resistance of a nichrome wire is
Pictorial symbols for resistors and meters are given in 60 times that of a copper wire of equal length and area
Fig. 3.9. of cross-section.
CURRENT ELECTRICITY 3.1

Area =y4
Combining the above factors, we get
/ I
Roc — or R=p
A A
A
>1,
where p is the constant of proportionality called resistiviti/
or speciifc resistance of the material of the conductor. It A
depends on the nature of the material of the conductor («) (b)
and on the physical conditions like temperature and Fig. 3.11 Current density.
pressure but it is independent of its size or shape.
Resistivity or specific resistance. If in the above Current density.
equation, we take I I
/ =
I = 1 unit and A = 1 square unit A^i A cos 0
then R=p

w
or
I = jA cos 0 = / .A
Thus, the resistivity or specific resistance of a material
may be defined as the resistance of a conductor of that This equation again shows that electric current,
material, having unit length and unit area of cross-section. being scalar product of two vectors, is a scalar quantity.

Flo
Or, it is the resistance offered by the unit cube of the material The SI unit of current density is ampere per square
of a conductor. metre (Am"^) and its dimensions are [AL"^].

ee
SI unit of resistivity. We can write
Note The current / through a particular surface Sin

Fr
Rx A
P =
/ a conductor is the flux of j through that surface and is
ohmx metre^ given by the surface integral
SI unit of p = for
ur
metre
7= j.dS
= ohm meter (D m) S
ks
Thus, the SI unit of resistivity is ohm metre (O m). where dS is a small element of the given surface area.
Yo
oo

3.8 CURRENT DENSITY, CONDUCTANCE Conductance. The conductance of a conductor is the


AND CONDUCTIVITY ease with which electric charges flow through it. It is equal to
eB

the reciprocal of its resistance and is denoted by G.


12. Define the terms current density, conductance Thus,
and conductivity. Write their SI units. Express Ohm's 1
r

Conductance =
law in vector form.
ou

Resistance
ad

Current density. The current density at any point


inside a conductor is defined as the amount of chargeflowing or G=i
Y

R
per second through a unit area held normal to the direction of -1
The SI unit of conductance is ohm or mho or
the flow of charge at that point. It is a vector quantity
nd
Re

siemens (S)
having the same direction as that of the motion of the
positive charge. It is a characteristic property of any Conductivity. The reciprocal of the resistivity of a
Fi

point inside the conductor and is denoted by j . material is called its conductivity and is denoted by a.
Thus,
As shown in Fig. 3.11(fl), if a current I is flowing 1
uniformly and normally through an area of cross- Conductivity =
section A of a conductor, then the magnitude of current Resistivity
density at any point of this cross-section will be 1
or a = —
q/t 1 P

The SI unit of conductivity is ohm ^ormhom ^


If the area A is not perpendicular to the direction of or Sm”'.
current and normal to this area makes angle ©with the Vector form of Ohm's Law. If £ is the magnitude of
direction of current as shown in Fig. 3.11(b), then the electric field in a conductor of length /, then the
component of A normal to the direction of current
flow will be
potential difference across its ends is
V= El
\ = A cos 0
s 3.8

Also from Ohm's law, we can write

V= IR =
Ipl
A
PHYSICS-XII

Table 3.1 Eledri ca resistivities of some substances

Temperature coeffi No. of


ifesistivity cient of resistivity valence
/ Material at 1 ; electrons
El = — pl at (rc, a = I
A OCpfiim) p ' dT I per unit
('C ■) cell
or
E = jp
A. Conductors
As the direction of current density j is same as -s
Silver 1.6x10 0.0041 1
that of electric field E, we can write the above
Copper 1.7 X 10"** 0.0068 1
equation as
Aluminium 2.7 X 10"' 0.0043 3
E =p j -8
Tungsten 5.6x10 0.0045 6

or j =G E Iron 10x10
-8
0.0065 8

-8
The above equation is the vector form of Ohm's Platinum 11x10 0.0039 10

law. It is equivalent to the scalar form V = Rl. Mercury 98x10


-8
0,0009 2

Nichrome 100x10"' 0.0004


3.9 CLASSIFICATION OF AAATERIALS IN
(alloy of
TERMS OF RESISTIVITY Ni, Fe, Cr)
-8
13. How ca?i we classify solids on the basis of their Manganin 48x10 0.002 X 10"^
resistivity values ? (alloy of Cu,
Ni, Fe, Mn)
Classification of solids on the basis of their
B. Semiconductors
resistivity values. The electrical resistivity of sub
Carbon 3.5x10"® - 0.0005 4
stances varies over a very wide range, as shown in
I'able 3.1. Various substances can be classified into (graphite)
three categories : Germanium 0.46 -0.05 4

1. Conductors. The materials which conduct electric Silicon 2300 -0.07 4

current fairly well are called conductors. Metals are good C. Insulators
conductors. They have low resistivities in the range of
Pure water 2.5 X 10®
10" n m to 10 Q m. Copper and aluminium have
10
the lowest resistivities of all the metals, so their wires
14
Class 10*^-10

are used for transporting electric current over large Hard Rubber 10’® -10,16
distances without the appreciable loss of energy. On 14
NaCl -10 8
the other hand nichrome has a resistivity of about 60
.16
times that of copper. It is used in the elements of Fused quartz -10

electric heater and electric iron.

2. Insulators. The materials which do not conduct 14. What are the two common varieties of commercial
resistors ?
electric current are called insulators. Tliey have high
resistivity, more than 10"^ H m. Insulators like glass, Common commercial resistors. The commercial
mica, bakelite and hard rubber have very high resistors are of two major types :
resistivities in the range lO^"* Q m to 10^^ Q m. So they 1. Wire-boimd resistors. These are made by winding
are used for blocking electric current between two the wires of an alloy like manganin, constantan or
points. nichrome on an insulating base. The advantage of
3. Semiconductors. These are the materials whos,e using these alloys is that they are relatively insensitive
resistivities lie in betzuecn those of conductors and insulators to temperature. But inconveniently large length is
i.e., between 10"^ Qm to lO^flm. Germanium and required for making a high resistance.
silicon are typical semiconductors. For moderately high 2. Carbon resistors. They are made from mixture of
resistances in the range of kQ, resistors made of carbon black, clay and resin binder which are pressed
carbon (graphite) or some semiconducting material are and then moulded into cylindrical rods by heating. The
used. rods are enclosed in a ceramic or plastic jacket.
CURRENT ELECTRICITY

The carbon resistors are widely used in electronic First significant figure
circuits of radio receivers, amplifiers, etc. They have — Second significant figure
the following advantages :
Decimal multiplier
(i) They can be made with resistance values rang Tolerance
ing from few ohms to several million ohms.

in
(ii) They are quite cheap and compact. n
(Hi) They are good enough for many purposes.
u
Fig. 3.12 Meanings of four bands.
3.10 COLOUR CODE FOR CARBON RESISTORS
15. Describe the colour code used for carbon resistors. Illustrations : 1. In Fig. 3.13, the colours of the four
bands are red, red, red and silver; the resistance value is
Colour code for resistors. A colour code is used to
indicate the resistance value of a carbon resistor and its Red Red Red Silver

w
4. i i
percentage accuracy. The colour code used throughout
the world is shown in Table 3.2. 2 2 2 ± 10%

Table 3.2 K= 22x10^0 ±10%.

Flo
Resistor colour code
Red Red
Letter as Red
Toler

ee
Num- Multi- Silver
Colour an aid to Colour
ber plier ance
memory
A

Fr
0
Black B 0 10 Gold 5%

Brown B 1 10^ Silver 10%

10^ Fig. 3.13


Red R 2 No fourth 20%
for
ur
band

Orange O 3 10^ 2. In Fig. 3.14, the colours of the four bands are
10' yellow, violet, brown and gold; the resistance value is
ks
Yellow Y 4
Yo

Green G 5 10^ Yellow Violet Brown Gold


oo

i i i
Blue B 6 10^
4 7 1 ±5%
eB

Violet V 7 10^
10** K =47x10^ a±5%.
Grey G 8

White W 9- 10^ Violet Brown


r

Yellow- Gold
1 I
ou
ad

How to remember colour code :


A
1
Y

B B R O Y of Great Britain had Very Good Wife U


4 ●i* 4.
Fig. 3.14
nd

0 1 2 3 4 5 6 7 8 9
Re

3. When there are only three coloured bands


There are two systems of marking the colour codes:
Fi

printed on a resistor and there is no gold or silver


First system. A set of coloured co-axial rings or band, the tolerance is 20%. In Fig. 3.15, there are only
bands is printed on the resistor which reveals the three bands of green, violet and red colours ; the
following facts : resistance value is

1. The first band indicates the first significant figure. Green Violet Red No 4th band
2. The second band indicates the second significant 4 4 4 4
figure. 5 7 2 ± 20%

3. The third band indicates the power of ten with K = 57x 10^n±20%.
which the above two significant figures must be
Violet Red
multiplied to get the resistance value in ohms.

%
Green
4. The fourth band indicates the tolerance or possible
variation in percent of the indicated value. If the A
fourth band is absent, it implies a tolerance of U
±20%.
Fig. 3.15
.10 PHYSICS-XII

Second System : Example 9. o discharge tube, the number of hydrogen


1. The colour of the body gives the first significant ions (i.e., protons) drifting across a cross-section per second
figure. is 1.0 X 10^®, while the number of electrons drifting in the
opposite direction across another cross-section is 2.7 x 10^®
y—Body Ring
per second. If the supply voltage is 230 V, what is the
effective resistance of the tube ? INCERT]
End End
Solution. The current carried by a negatively charged
electron is equivalent to the current carried by a proton
Dot
in the opposite direction, therefore, total current in the
Fig. 3.16 direction of protons is

2. The colour of the end gives the second signi I = Total charge flowing per second = (n^ + n^) e
ficant figure. = [2.7X lO^** +1.0X 10^® ]X 1.6 X 10“^^
3. The colour of the dot gives the number of zeroes = 3.7x 1.6x 10“^ =0.592 A
to be placed after the second figure. Effective resistance,
4. The colour of the ring gives the tolerance or
jj _ ^ _ 230 n =388.5 Q-3.9 X 10^ a.
percent accuracy of the indicated value. ~ / ~ 0.592
Illustration. Suppose for a given resistor, the body Example 10. A 10 V battery of negligible internal
colour is yellow, end colour is violet, dot colour is resistance is connected across a 200 V battery and a resis
orange and the ring colour is silver. tance of 38 53! as shown in Fig. 3.17. Find the value of the
current in circuit. [CBSE 18]
Body End Dot Ring
Yellow Violet Orange Silver 10 V
i I i 4
4 7 3 ± 10%

38 n
.-. K = 47 X 10^ Q ± 10% = 47 kn ± 10%. VS^
200 V
Fig. 3.17
Examples based on
Ohm's law. Resistance, Resistivity, Solution. The two cells are in opposition, so the
net emf =200-10=190V
Conductance, Conductivity, Current Density and
Colour Code of Carbon Resistors ^
Current,, 7, = —
V=190V = 5 A.
R 38n
Formulae Used

1. Ohm's law, R= — or V = IR Example 11. A copper loire of radius 0.1 mm and


I resistance IkQis connected across a power supply of 20 V.
2. Resistance of a uniform conductor, R =
I (i) How many electrons are transferred per second between
the supply and the wire at one end ? (ii) Write dozon the
RA current density in the wire.
3. Resistivity or specific resistance, p = / Solution. Here r =0.1 mm =0.1 x 10“^ m.
1
4. Conductance = —
R R=lkQ=10^ a V=20V
1
5. Conductivity = or 1
a = - J_
(0 Current, /=— =—^ = 0.02 A
Resistivity p RA R 10^
Current I
6. Current density = or No. of electrons.
Area

7. Colour code of carbon resistors. Refer to Table 3.2 =


q _ It _ 0.02 X 1
-=■ = 1.25 X lO'^.
-19
e e 1.6x10
Units Used

Potential difference V is in volt (V), current I in (n) Current density,


ampere (A), resistance R in ohm (Q), resistivity p . 7 7 0.02
in Qm, conductance in ohm " ’ or mho or siemens 2
A m
3.14 X (0.1x10“^)^
(S), conductivity inQ“Vi“' orSm'^ and current
density j in Am"^. = 6.37x10^ Am‘I
CURRENT ELECTRICITY

Example 12. Current flows through a constricted con Solution. Resistance of the arc lamp is
ductor, as shoion in Fig. 3.18. The diameter Dj =2.0 mm K = ^=^=sn
and the current density to the left of the constriction is 1 10
j =1.27 X10^ Am~^. (i) What current flows into the In order to use arc lamp with a source of 240 V, a
constriction ? (ii) If the current density is doubled as it resistance R' should be connected in series with it so

emerges from the right side of the constriction, what is that current through the circuit does not exceed 10 A.
diameter Dj ? Then

A I{R+ R')=V 10(8+ R')=240


or

Di
J\ 7TT ^2 A or R'=24-8=16n.
I

i iy Example 15. Calculate the resistivity of a material of a


Vi wire 10 m long, 0.4 mm in diameter and having a resistance
of 2.0 n. [Haryana 02]

w
Fig. 3.18
Solution. Here / =10 m, r=0.2 mm = 0.2 x 10“^ m,
Solution. Here =2.0 mm, /j =1.27xlO^Am“^, R=2Q

Flo
h it Resistivity,
(i) Current flowing into the constriction. RA Rxnr^

ee
P =
I I
DI
2 X 3.14 X (0.2x10“^)^

Fr
X 7C
8
2 ) = 2.513 X 10"°Qm.
10
= 1.27 X 10^ X 3.14 x(lxl0"^r =3.987 A.
Example 16.The external diameter of a 5 metre long
for
ur
(ii) For a steady flow of current, hollow tube is 10 an and the thickness of its wall is 5 mm If
^ h the specific resistance of copper be 1.7 x 10“® ohm-metre,
then determine its resistance.
ks
or

Solution. The cross-sectional area of the tube is


Yo

f f = ;,■
x2
^2
oo

or X 7t — X TC —
n 2 J2 [ 2
eB

= 3.14 X [{5 X 10“^ - (4.5 X 10“^)^ ]


or
h X Tt
2 )
X 71 f ^2? = 14.9x10^'*

p=1.7xlO”®Qm,/ = 5m
r

Also,
ou

1
ad

or
^2 = ■J2 Dj =0.707 Dj .●. Resistance,
I 1.7xl0‘®x5
Y

= 0.707 X 2.0 mm = 1.414 mm. R = p- -4


14.9x10
Example 13. A current of 2 niA is passed through a = 5.7xl0'®n.
nd
Re

colour coded carbon resistor with first, second and third


rings of yellow, green and orange colours. What is the Example 17. Fmd the resistivity of a conductor in which a
Fi

voltage drop across the resistor ? current density of 2.5 Am~^ is found to exist, when an
Solution. electric field of 15 Vnf ^ is applied on it. [ISCE 981

Solution. Here j =2.5 Am~^, £ =15 Vm~^


Yellow Green I Orange
4- 4- I 4.
Resistivity, p =
^_V A
4 5 I 3
I ~ I' I
R = 45 X 10^ n V/l E 15
= 6 m.
Given 7 = 2 mA=2 X 10“^ A HA J 2.5

y = K7 = 45x10^ x2x10'A = 90 V. Example 18. A 2 w long copper wire carries a current of


Example 14. An arc lamp operates at 80 V, 10 A Suggest 1 A. If the cross-section of the wire is 2.0 mn? and the
a method to use it with a 240 V d.c. source. Calculate the resistivity of copper is 1.7 x 10~® Elm, find the force
value of the electric component required for this purpose. experienced by moving electron in the wire.
[CBSE F 94] (Charge on electron = 1.6 x lO'^^O [JEE Main July 22]
3.12 PHYSICS-XII

c w
Solution. r c
F = eE= e— = — = — x — =V elR el p! elp Example 22. The resistance of a wire is R ohm. What will
l I ! A A be its nezv resistance if it is stretched to n times its original
-19 -8
1.6x10 xlxl.7xl0 length ?
N
2.0x10"^ Solution. In both cases, volume of the wire is same.
-23
= 136 X 10 N. V= Al = Al'

Example 19. A wire of resistance 4 Q is used to xvind a coil or A=L = n [V /'=«/]


of radius 1 cm. The wire has a diameter of 1.4 mm and the /I' /

specific resistance of its material is 2 x 10~^ .Qm Find the /'


number of turns in the coil. R' /' 71 2
- ^ - — = n.n = n
Solution. Let n be the number of turns in the coil. R / / K

Then total length of wire used ^ A


= 2k Rx n =27tX 7x 10“^x n metre or R' = R.

Total resistance. Example 23. A cylindrical wire is stretched to increase its


R =
^7
/
or ^^_2x10~^x2tcx7x10~^ X n length by 10%. Calculate the percentage increase in
resistance.
71(0.7x10“^)^
Solution. New length, /' = / +10% of /
.-. n = 70.
= 1 + 0.11=1.11
Example 20. A wire of 10 ohm resistance is stretched to /'
thrice its original length. What zvill be its (i) nezv resistivity, or - = 1.1
I -
and (ii) new resistance ? [CBSE D 98C)
Al= AT'
Solution. (/) Resistivity p remains unchanged
A=L
because it is the property of the material of the wire. or
71' /
(ii) In both cases, volume of wire is same. So r I'
/
V^A'r=Al
R~ A A'~ 1
= (l.lf =1.21
or — = 1=1=1 [v /'=/ + 2/=3/J The percentage increase in resistance,
71 r 31 3
R’-R
xl00 =
( R'
/' -1 xl00=(1.21-l)xl00=21%.
R R
R' - —^
— P'X' =— X
71
= 3-X 3^
-=9
R / 71' 1 1 Example 24. Tioo zvires A and B of equal mass and of the
^7 same metal arc taken. The diameter of the wire A is half the
Hence R'=9R=9xl0 = 90n. diameter of wire B. If the resistance of wire A is 24 Q,
calculate the resistance of wire B.
Example 21. A wire has a resistance of 16 O. It is melted and
drawn into a wire of half its length. Calculate the resistance of Solution. Mass of wire = volume x density
the new wire. WJzat is the percentage change in its resistance ? = area of cross - section x length x density
Solution. In both cases, volume of the wire is same. m

rj
V = AT' = Al
or
;B _ 1/2 f 1
or
A r ii 1
I'bJ V 1 4

2 /B
P
r R
B _
Tz r
B
■A.
rA =—1 X riV 1
A'_L A-1. 1-^ I
u r„
\ B J
4 1) 16
R / I A 2 2 4
^A ^'4
1 1
or R'=lK=-xl6=4n. or R
B ~
16
R. =—x24a=1.5n.
16
4 4

Change in resistance Example 25. A piece of silver has a resistance of IQ. What
R-R'
will be the resistance of a constantan wire of one-third length
12
xl00= —xl00=75%. and one-half diameter, if the specific resistance of constantan
R 16 is 30 times that of silver ?
CURRENT ELECTRICITY 3.13
_Q ,V

Solution. For silver, 4. A metal wire of specific resistance 64 x 10 Q m >

4p I and length 1.98 m has a resistance of 7 O. Find its


=ioa radius. (Ans. 2.4 x 10'“* m)
nd'^
5. Calculate the resistance of a 2 m long nichrome wire
For constantan,
I of radius 0.321 mm. Resistivity of nichrome is

R' =
4p-/' 4x30px- 15xl0"'’O m. If a potential difference of 10 V is
Kd
,2
df applied across this wire, what will be the current in
71 the wire ? (Ans. 9.260,1.08 A)
2)
40 X 4p / 6. An electron beam has an aperture of 1.0 mm^. A
= 40R = 40xl=40a. 16
total of 6 X10 electrons flow through any
nd^
perpendicular cross-section per second. Calculate
Example 26. On applying the same potential difference (/) the current and (a) the current density in the
between the ends of wires of iron and copper of the same electron beam.

w
length, the same current flows in them. Compare their radii. [Ans. (/) 9.6 X10"^ A (a) 9.6 x 10^ Am
Specific resistances of iron and copper are respectively 7. Calailate the electric field in a copper wire of
1.0 X 10” ^ and 1.6 x 10” ® Om Can their current-densities
cross-sectional area 2.6 mm^ carrying a current of

Flo
be made equal by taking appropriate radii ?
1 A. The resistivity of copper = 1.7 x 10"^ Qm.
Solution. On applying same potential difference, (Ans. 0.85x10“^ Vm"^)

ee
same current flows in the two wires. Hence the
resistances of the two wires should be equal. 8. A given copper wire is stretched to reduce its

Fr
/ I
diameter to half its previous value. What would be
its new resistance ? ICBSE D 92C]
But
K = P —=P^
A Tir
(Ans. R' = 16 R)
for
ur
For the two wires of same length /, we have 9. What will be the change in resistance of a
/ constantan wire when its radius is made half and
and
Ri=Pi 7?1 length reduced to one-fourth of its original length ?
ks
71
TC r2
(Ans. No change)
Yo

As Ri = R2
oo

10. A wire of resistance 5 Q is uniformly stretched until


Pi = P2 £i its new length becomes 4 times the original length.
eB

or

^2 \P2 Find its new resistance. (Ans. 80 Q)

r Piiron l.OxlQ-^ 11. A metallic wire of length 1 m is stretched to double


r

iron
= 2.5.
its length. Calculate the ratio of its initial and final
^1.6x 10
-8
ou
ad

r
copper ^ Pcopper resistances assuming that there is no change in its
density on stretching. [CBSE D 94]
No, current densities cannot be equal because they
Y

depend on nature of the metals. (Ans. 1 : 4)


12. A wire of certain resistance R is stretched so that its
nd

7>.
Re

roblems For Practice radius decreases by a factor Calculate its new


resistance. (Ans. R)
Fi

1. A voltage of 30 V is applied across a colour coded 13. A wire 1 m long and 0.13 mm in diameter has a
carbon resistor with first, second and third rings of resistance of 4.2 H. Calculate the resistance of
blue, black and yellow colours. What is the current another wire of the same material whose length is
flowing through the resistor ? ICBSE D 051
1.5 m and diameter 0.155 mm. (Ans. 4.4H)
(Ans. 0.5x10"'‘A) 14. A rheostat has 100 turns of a wire of radius 0.4 mm
2. A potential difference of 10 V is applied across a having resistivity 4.2xl0“’'nm. The diameter of
conductor of resistance IkQ. Find the number of each turn is 3 cm. What is the maximum value of
electrons flowing through the conductor in resistance that it can introduce ? (Ans. 7.875 Q)
5 minutes. (Ans. 1.875 x 10^^) u

15. Given that resistivity of copper is 1.68 x 10 Qm.


3. What length of a copper wire of cross-sectional area
Calculate the amount of copper required to draw a
O.Olmm^would be required to obtain a resistance
wire 10 km long having resistance of 10 n. The density
of 1 kfl ? Resistivity of copper = 1.7xl0“^Om.
of copper is 8.9 xlO^kgm”^. (Ans. 1495.2 kg)
(Ans. 588.2 m)
3.14 PHYSiCS~XII

/ /
16. The size of a carbon block is 1.0 cm xl.O cm x50 cm.
9. l? = p
A
Find its resistance (f) between the opposite
square faces (//) between the opposite rectangular //4 /
= R.
faces of the block. The resistivity of carbon is ^ Tz{rIlf ^ %-3
3.5xl0"-‘^ncm. (Ans. 0.175a 7.0xl0“^n) ]
17. Two wires A and B of the same material have their 10. R = p — = 5Q
^ A
lengths in the ratio 1 : 5 and diameters in the ratio 41 I
3 : 2. If the resistance of the wire B is 180 a find the R'=p = 16p — = 16 R = 16x5 = 80Q.
resistance of the wire A. A/4 A
(Ans. 16 n)
/
18. A uniform wire is cut into four segments. Each 11. R = p
A
segment is twice as long as the earlier segment. If 21 I
the shortest segment has a resistance of 4 a find the R' = p = 4p = 4 R
resistance of the original wire. (Ans. 60 a
A/2 ^ A

.-. R: R' = 1: 4
19. Calculate the conductance and conductivity of a
wire of resistance 0.01 Q, area of cross-section 12. V = A’/'= A/
10"‘^m^ and length 0.1 m. [Haryana 2000]
(Ans. 100 S, 10^ Sm"’) or V = 71 r = nr^ I or 1' = ti I
\nj
HINTS
r i
1. R = 60xl0^a V^=30V .-. R'=p = }fp 2 R.
30
Tcr ^Tz(r/nf nr

= 0.5x10"* A.
R 60 X10"* h
13. R2 = Ri
y lov lov L^JL^2J
2. 1 = = 10'^A -|2
R IkQ looon 1.5 0.13 X 10"^
= 4.2 = 4.4 Q.
q _It _10"^x5x60 1 0.155x10"^
-19
= 1.875 xlO^^
e e 1.6x10
14. Length of the wire used, / = IOOtt D
4. As R = ^ = -BL / 100 tiD 100 pD
A Ttr^ R=p

^ _ pi 64x10^^x1.98x7 = 5.76xl0"®m^ 100x4.2x10-^x3x10"^


tiR 22 X 7 = 7.875 n.
(0.4x10"^)^
or r = 2.4x10 m. /
15. As R =
/ V ^A
5. Use R = p 2 and i = — .
Ttr R
.'.
, pi 1.68 X 10"® X 10x10^
A = — - = 1.68xlO"V^
ne 6x10*^x16x10 -19 R 10
6. (,) /=at t 1 Mass of copper required,
= 9.6x10"^ A. m = Volume x density = Al x density
(j'i) Current density, = 1.68 X10“^ X10 X10^ X 8.9 x 10^
/ _9.6xl0"^ = 9.6xlO^Am"^ = 1495.2 kg.
A~1.0xl0 -6
I 3.5x10"® X 50x10"^
-8 16. (i)R = p-L = = 0.175 a.
7. £ = X. = -l£l -ZP. - ^^1-7x10 A 1.0x10”^ xl.OxlO"^
I ~ I ~ IA~ A 20x10"^
/ 3.5x10"® xl.OX10"^
= 0.85 xl0"^Vm"*. (if) R = p-- = = 7.oxio”®n.
A 1.0x10"^ xSOxlO"^
8. When the diameter of the wire is reduced to its half
p
value, area of cross-section becomes one-fourth and
17. Ia B
=IX ±
the length increases to four times the original RB / I d 5^3/ 45
B V'AJ
length.
V 41 I
.-. R' = = 16p—=16R.
A
A -xl80=16a
B -
4 45 45
CURRENT ELECTRICITY 3A5

IB- Let the lengths of the four segments be /, 21, 4/ and flowing steadily in circular loop. Their observations
8/. Then their corresponding resistances will be R, indicated that
2R, 4K and 8R 1. The sign of the charges is negative.
Given R = 4 fi
2. The ratio ej mot the charges is equal to that mea
Resistance of the original wire sured for the electrons in other experiments.
= R+ 2R+4R+ 8R=15R=15x4 = 60n.
It was thus established directly that current in
1
19- Conductance, G = — — =100 a metals is carried by negatively charged electrons.
R 0.01

Conductivity, 3.12 MECHANISM OF CURRENT FLOW IN A


1 / 0.1 CONDUCTOR : DRIFT VELOCITY AND
o = — = 10® Sm'L
P RA 0.01x10“^ RELAXATION TIME

18. Explain the mechanism of the flow of current in a

w
3.11 CARRIERS OF CURRENT metallic conductor. Hence define the terms drift velocity
and relaxation time. Deduce a relation between them.
16. Mention different types of charge carriers in
Mechanism of the flow of electric charges in a
solids, liquids and gases.

Flo
metallic conductor : Concepts of drift velocity and
Carriers of current. The charged particles which by
relaxation time. Metals have a large number of free
flowing in a definite direction set up an electric current are

ee
called current carriers. The different types of current
electrons, nearly 10^® per cubic metre. In the absence of
any electric field, these electrons are in a state of

Fr
carriers are as follows :
continuous random motion due to thermal energy. At
1. In solids. In metallic conductors, electrons are room temperature, they move with velocities of the
the charge carriers. The electric current is due to the 10^ms“^ However, these velocities are
for
ur
drift of electrons from low to high potential regions. In distributed randomly in all directions. There is no
tt-type semi-conductors, electrons are the majority preferred direction of motion. On the average, the
charge carriers while in p-type semiconductors, holes number of electrons travelling in any direction will be
ks
are the majority charge carriers. A hole is a vacant state
equal to number of electrons travelling in the opposite
Yo

from which an electron has been removed and it acts as


oo

a positive charge carrier. direction. If Uy ...., are the random velocities of N


eB

2. In liquids. In electrolytic liquids, the charge free electron;?, then average velocity of electrons will be
carriers are positively and negatively charged ions. For
example, CuSO^ solution has Cu^"^ and SO^ ions. _ tq + «2 + ... + u^ =0
r

N
which act as the charge carriers.
ou
ad

3. In gases. In ionised gases, positive and negative Thus, there is no net flow of charge in any direction.
Y

ions and electrons are the charge carriers.


4. In vacuum tubes. In vacuum tubes like radio In the presence of an external field E, each electron
nd

valves, cathode ray oscilloscope, picture tube etc ; free experiences a force - e £ in the opposite direction of E
Re

electrons emitted by the heated cathode act as charge (since an electron has negative charge) and undergoes
Fi

carriers.
an acceleration a given by
17. Why is it that electrons carry current in metals ?
-* Force e E
Metallic conduction, in metals, the atoms are a =

closely packed. The valence electrons of one atom are Mass


m

close to the neighbouring atoms and experience electrical where m is the mass of an electron. As the electrons
forces due to them. So they do not remain attached to a accelerate, they frequently collide with the positive
particular atom, but can hop from one atom to another metal ions or other electrons of the metal. Between two
and are free to move throughout the lattice. These free successive collisions, an electron gains a velocity
electrons are responsible for conduction in metals. component (in addition to its random velocity) in a
The fact, that the negatively charged electrons carry direction opposite to £. However, the gain in velocity
current in metals, was first experimentally confirmed lasts for a short time and is lost in the next collision. At
by the American physicists Tohnan and Stewart in 1917. each collision, the electron starts afresh with a random
TTiey measured the angular momentum of the charges thermal velocity.
v<

3.16 PHYSICS-XII
\'V

If an electron having random thermal velocity as the average time t between two successive collisions
^ is small, an electron slowly and steadily drifts in the
accelerates for time Tj (before it suffers next collision),
then it will attain a velocity, opposite direction of E, as shown in Fig. 3.19.

v^ = u^ + a

Similarly, the velocities of the other electrons will be


V2 = U2+a X2,

^= ●● ..y

V^=ti^ + a-: N

The average velocity of all the N electrons will be Fig. 3.19 Slow and steady drift of an electron in the opposite
-*
direction of E. The solid lines represent the path in the
V
_ + ^2 + 1^3 + + i; hi
d ~
N absence of E and dashed lines in the presence of £.

+ +^T2) + -+(»iv +^^iv) 3.13 RELATION BETWEEN ELECTRIC CURRENT


N

AND DRIFT VELOCITY : DERIVATION OF
Ti+T2+- + T^ OHM'S LAW
+ a
N N
19. Derive relation between electric current and drift
= 0 +fl T velocity. Hence deduce Ohm's law. Also write the
expression for resistivity in terms of number density offree
where t =(Xj + T2 + + x^)/N is the average time electrons and relaxation time.
between two successive collisions. The average time that
elapses between tzvo successive collisions of an electron is Relation between electric current and drift velocity.
called relaxation time. For most conductors, it is of the Suppose a potential difference V is applied across a con
order of 10
-14
ductor of length / and of uniform cross-section A The
s. The velocity gained by an electron
during this time is electric field E set up inside the conductor is given by
e £ T
= a z = - I
d
m -*

Under the influence of field E, the free electrons


The parameter v^ is called drift velocity of begin to drift in the opposite direction £ with an
electrons. It may be deifned as the average velocity gained average drift velocity v^.
by the free electrons of a conductor in the opposite direction Let the number of electrons per unit volume or
of the externally applied electric field. electron density = n
It may be noted that although the electric field Charge on an electron = e
accelerates an electron between two collisions, yet it
does not produce any net acceleration. This is because M- / >1

the electron keeps colliding with the positive metal Area = A £ Free electron
ions. The velocity gained by it due to the electric field is o ■e ■e
lost in next collision. As a result, it acquires a constant o ■e
average velocity in the opposite direction of £. The yy-Q e
motion of the electron is similar to that of a small
Conventional
spherical metal ball rolling down a long flight of stairs. current
Electronic
current
As the ball falls from one stair to the next, it acquires
acceleration due to the force of gravity. The moment it
collides with the stair, it gets decelerated. The net effect
Battery
is that after falling through a number of steps, the ball
begins to roll down the stairs with zero average Fig. 3.20 Drift of electrons and electric field inside a conductor.
acceleration i.e., at constant average speed. Moreover,
CURRENT ELECTRICITY

ml
Number of electrons in length / of the conductor But R =
ne^ \A
= ft X volume of the conductor = n Al
where x is the relaxation time. Comparing the above
Total charge contained in length / of the conductor is
two equations, we get
q = en Al m
All the electrons which enter the conductor at the p = 2
ne X
right end will pass through the conductor at the left
end in time. Obviously, p is independent of the dimensions of
t =
distance / the conductor but depends on its two parameters :
velocity 1. Number of free electrons per unit volume or
^ . IJ q enAl electron density of the conductor.
Current, = -= or
1 = enAv^ 2. The relaxation time x, the average time between
t l/v^
two successive collisions of an electron.

w
This equation relates the current I with the drift
velocity v^. 20. Write relation between quantities j , <3 and E.
The current density '/' is given by Relation between j , o and E . For an electron.

Flo
I
) = = e7Wj
A

ee
e Ex
In vector form I and V

Fr
d ~
m

The above equation is valid for both positive and 2


-*
eEx') ne X X*
negative values of q. j =nqv^i=n(-e) ~ E

for m m
ur
Deduction of Ohm's law. When a potential 2
difference V is applied across a conductor of length /, But
ne X 1
— = o, conductivity of the conductor
the drift velocity in terms of V is given by m p
ks
eEx eVx
V
Yo
d ~
m ml / =a£ or E =p j
oo

If the area of cross-section of the conductor is A and This is Ohm's law in terms of vector quantities like
eB

the number of electrons per unit volume or the electron


—* —*

current density j and electric field E.


density of the conductor is n, then the current through
the conductor will be 21. What causes resistance in a conductor ?
r

eVx Cause of resistance. Collisions are the basic cause of


I = enAv^ - enA.
ou
ad

ml resistance. When a potential difference is applied


V ml across a conductor, its free electrons get accelerated.
Y

or
I ne^ xA On their way, they frequently collide with the positive
metal ions i.e., their motion is opposed and this
nd

At a fixed temperature, the quantities n\ /, n, e, x and


Re

opposition to the flow of electrons is called resistance.


A all have constant values for a given conductor.
Larger the number of collisions per second, smaller is
Fi

Therefore,
the relaxation time x, and larger will be the resistivity
— = a constant, R (p = m / ne^ t).
1
The number of collisions that the electrons make with
This proves Ohm's law for a conductor and here the atoms/ions depends on the arrangement of atoms
ml or ions in a conductor. So the resistance depends on the nature
R =
xA of the material (copper, silver, etc.) of the conductor.
is the resistance of the conductor.
The resistance of a conductor depends on its length. A
long wire offers more resistance than short wire
Resistivity in terms of electron density and relaxation because there will be more collisions in the longer wire.
time. The resistance K of a conductor of length /, area of
77ie resistance of conductor depends on its area of cross-
cross-section A and resistivity p is given by section. A thick wire offers less resistance than a thin
/ wire because in a thick wire, more area of cross-section
R = p-
is available for the flow of electrons.
3.18 PHYSICS-XII

22. Alloys ofvietals have greater resistivity than their Example 27. A particle of charge 2 pC and mass 1.6 g is
constituent metals. Why ? . ,
moving with a velocity 41 ms . At t=0 the particle enters in
High resistivity of nichrome. In an alloy, e.g.r —* _i '' ''
nichrome (Ni-Cr alloy), Ni^"^ and ions have a region having an electric field E{inNC )=80i+60j.
different charge and size. They occupy random locations Find the velocity of the particle at t = 5s. [CBSE OD 201
relative to each other, though their ionic sites form a Solution.
regular crystalline lattice. An electron, therefore, passes -*

through a very random medium and is very frequently «=4ims'\ i7 = 2pC=2xl0~^C,


deflected. So there is a small relaxation time and hence
large resistivity. In general, alloys have more resistivity m=1.6 g =1.6 xlO’^kg, E =80i+60;NC"^
than that of their constituent metals.
->_F _f?£ _2xl0~^(80f + 60y ) -2
23. Explain the cause of instantaneous current in an m m 1.6x10'^
ms

electric circuit.

= (1002-+75/)xl0"^
-2
Cause of instantaneous current. Although the drift ms

speed of electrons is very small, typically 1 mm/s, yet -»

an electric bulb lights up as soon as we turn the switch V = u + a t=[Ai +(100/ +75/ )x 10 ^x5]ms ^
on. This is because electrons are present everywhere in
an electric circuit. When a potential difference is = (4.5/+0.375})ms“\
applied to the circuit, an electric field is set up through- Example 28. A copper wire has a resistance oflO Q and an
out the circuit, almost with the speed of light. Electrons area of cross-section 1 mn?. A potential difference of 10 V
m
every part of the circuit begin to drift under the exists across the wire. Calculate the drift speed of electrons if
influence of this electric field and a current begins to the number of electrons per cubic metre in copper is
flow in the circuit almost immediately. 8 x 10^ electrons. (CBSE D 96]
The above situation is analogous to the flow of Solution. Here R= 10 Q, A = 1 mm^ = 10“^m^,
water in a long pipe. As soon as the pressure is applied V =10 V, «=8x 10^® electrons/m^
at one end of the water filled pipe, a pressure wave is
transmitted along the pipe with a speed of about Now I =en A v^
1400 ms" \ When this wave reaches the other end, V
water starts flowing out. But water inside pipe moves —
R
= enAv^
forward with a much smaller speed.
or V
d ~
enAR
Examples based on
10
Drift Velocity -19
1.6x10 X 8 X 10^® X 10"^ X 10
Formulae Used
= 0.078 X 10“^ms“* = 0.078 mm s”
1 ● Current in terms of drift velocity (v^)is I = en A
2. Current density, j = env^ Example 29. (a) Estimate the average drift speed of
conduction electrons in a copper wire of cross-sectional area
3. No. of atoms in one gram atomic mass of an
element, N = Avogadro's number = 6.023 x 10^. l.OxlO-^ii^, carrying a current of 1.5 A Assume that each
copper atom contributes roughly one conduction electron.
4. In terms of relaxation time t,
ml
The density of copper is 9.0 x 10^ kg nf^, and its atomic
R =
ne^ T A and p = —5— mass /s63.5 u. Take Avogadro's number = 6.0 x 10^^ mol'^.
ne T
{b) Compare the drift speed obtained above with
5. Relation between current density and electric field, (i) thermal speeds of copper atoms at ordinary temperatures,
/ = CT £ or
£ = p/ (ii) speeds of electrons carrying the current and (Hi) speed of
Units Used propagation of electric field along the conductor which
causes the drift motion. [NCERT]
Drift velocity is in ms"^ free-electron density in Solution. Mass of 1 m^ of Cu
m"^, cross-sectional area A in m^, current density
j in Am"^, all resistances in Q. = 9.0x10^ kg=9xl0^g
Constants Used
Since Avogadro's number is 6.0 x 10^ and atomic
e= 1.6x10
-19
C and = 6.023x10^ mol \ mass of Cu is 63.5 u, therefore, 63.5 g of Cu contains
6.0 X 10^^ atoms.
CURRENT ELECTRICITY 3.19

So 9 X 10^ g of Cu contains (in) An electric field propagates along a conductor


6.0 X 10
23 with the speed of an electromagnetic wave i.e.,
X 9 X 10^ atoms =8.5 x 10^® atoms 3x10® ms“^
63.5
(electron) 1.1x10”®
8
Number of conduction electrons. speed of propagation of electric field 3x10

28
n = number of Cu atoms =8.5 x 10

Now 1=1.5 A, A = 10”^m^ e = 1.6xl0”^^C Example 30. Calculate the electricfield in a copper wireof
cross-sectional area 2.0 mn? carrying a current ofl A The
I 1.5
V conductivity of copper -6.25 x lO^Sm”^
enA 1.6xl0"^^x8.5xl0^®xl0”^ Solution. Here A =2.0 mm^ =2.0xl0”^m^,
15 -1
= l.lx 10”®ms”\ 1 = 1 A, a =6.25X lO^Sm
16 X 85 X 10

w
As )=—
A
= a£
(b) (i) At any temperature T, the thermal speed of a
I 1
copper atom of mass M is given by E =
Aa 2.0 X10”^ X 6.25x10^

Flo
3k.T
V
rms
B
= 8x10”® Vm”^
V M

ee
Example 31. A potential difference o/lOO V is applied to
But ordinary temperature, T-300 K, the ends of a copper wire one metre long. Calculate the

Fr
Boltzmann constant. fcg=1.38xlO”^JK”\ average drift velocity of the electrons. Compare it with the
thermal velocity at 27° C. Given conductivity of copper,
Mass of a copper atom. - = 5.81 X 10^ O” ^ m~ ^ and number density of conduction
for
ur
63.5 63.5 X 10"® electrons, «=8.5xl0^®m ®. [NCERT]
M=
23 g = 23 kg
6.0 X 10 6.0 X 10 Solution. Electric field,
ks
-23 23
Yo

3x1.38x10 X 300 X 6.0 X 10 / 1 m


oo

V
63.5x10"®
rms

As j = cE = env^
eB

= V117354.33 =342.57 ms”^ .●. Drift speed.


gE 5.81x10^x100
From part (fl), drift speed of electrons, V
d ~
1.6x10
-19
X 8.5 X10
28
r

e n

^ =l.lx 10”® ms"^


ou
ad

V
= 0.43 ms"\
(electrons) _ 1.1 x 10”®
Y

Now, fcg=1.38xlO"2®JK■^
V
rms (Cu atoms) 342.57
T=27 + 273=300K
nd

= 3.21x 10”^
m^= 9.1x10”®^ kg
Re

1 2
Thermal velocity of electron at27°C,
Fi

(ii) The maximum kinetic energy — mv^ of electron


-23
in copper corresponds to a temperature, V
3fcgT _ 3x1.38x10 x300
rms -31
9.1x10
T.0 = 10® K V
= 1.17x 10®ms”\
^ 2 K ^
— mvp = kg I V 0.43
d -
= 3.67x10”^
V
rms
1.17x10®
YkTf
B
2 X 1.38 X 10”^® X 10®
or V
m 9.1x10
-31 Example 32. Find the time of relaxation between collision
and free path of electrons in copper at room temperature.
= 1.74x 10^ ms”\ Given resistivity of copper = 1.7 xl0~^ Cim, number density
of electrons in copper =8.5xl0^®m”®, charge on electron
v^ (electron) _ 1.1x10”® -10”’. = 1.6x10
-19
C, mass of electron =9.1x10
-31
kg and drift
Dg (electron) 1.74x10^ velocity of free electrons = 1.6 x 10"^ ms”\
3.20 PHYSICS-XII

Solution. Here p =1.7x10 ®nm,« = 8.5x10 (ii) Current density.


-19
C, =9.1x10“^'kg, =1.6 x 10“‘^
-1
e = 1.6xl0 ms . I 1.0

ak A 1.0x10"^
As resistivity, P = 2
ne T
= 1.0x 10^ Am"^
Relaxation time,
-31 (Hi) Free-electron density.
9.1x10
T = -
dx N 8.9 X 10^ X 6.02 x 10^^
np {1.6 x 10“^^)^ x 8.5 x 10^® x 1.7 x 10“^ n -
M 63.5
= 2.5xlO“^S = 8.4x10^%"^
Mean free path of electron
Drift velocity.
= i?^t = 1.6x10^x2.5x10
-14

-18 ) 1.0 X10^


= 4.0x10 m. V
d ~ -19 28
en 1.6x10 X 8.4x10
Example 33. The current density in a cylindrical wire of
radius r =4.0 mm is 1.0 x 10^ A/n3. If the current through = 7.4 X 10"^ ms -I
the outer portion of the wire between radial distances k and r roblems For Practice
is xkA, then find x. [JEE Main June 22]
Solution. 1. The free electrons of a copper wire of cross-
sectional area 10“^m^ acquire a drift velocity of
10"^m/s when a certain potential difference is
4 mm
7r^ >
applied across the wire. Find the current flowing in
the wire if the density of free electrons in copper is
/= 1.0x10 Am
-2
8.5 X 10^® electrons/m^. (Ans. 1.36 A)
2 mm
2. Estimate the average drift speed of conduction
electrons in a copper wire of cross-sectional area
Current = Current density x Area 2.5xl0~^m^ carrying a current of 2.7 A. Assume
n2
. 3kR^ the density of conduction electrons to be
l = jxn R^-\ — 9 X10^^ m ICBSE OD 141
[ U (Ans. 0.75 mms"’)
_ 10^x37rx(4xlQ-^)^^^ 3. A current of 1.8 A flows through a wire of cross-
4
sectional area 0.5 mm^. Find the current density in
= 12nA. the wire. If the number density of conduction
electrons in the wire is 8.8 x 10^®m'^, find the drift
a: = 12.
speed of electrons.
Example 34. A current of 1.0 ampere is flowing through a (Ans. 3.6xl0^Am”^,2.56xl0^ms”^)
copper wire of length 0.1 metre
and cross-section 4. When 5 V potential difference is applied across a
1.0xl0~^ }t^. (i) If the specific resistance of copper be wire of length 0.1 m, the drift speed of electrons is
1.7xlO~®Q/fi calculate the potential difference across the 2.5xl0“^m /s. If the electron density in the wire is
ends of the wire, (ii) Determine current density in the wire. 8 X 10^®m“^, calculate the resistivity of the material
(Hi) If there be one free electron per atom in copper, then of wire. [CBSE OD 161
determine the drift velocity of electrons. Given : density of (Ans. 1.56 xlO'^Om)
copper = 8.9 X10^kg m~^, atomic weight = 63.5, 5. A copper wire of diameter 1.0 mm carries a current
A/=6.02 X 10^^ per 28
of 0.2 A. Copper has 8.4 x 10 atoms per cubic
Solution. Here /=1.0A, / =0.1 m, A =1.0 x 10"^m^, metre. Find the drift velocity of electrons, assuming
that one charge carrier of 1.6 x 10"'^C is associated
p = 1.7x 10“^Qm, d =8.9 X 10^ kg m“^ with each atom of the metal. IISCE 97]
(/) Resistance of wire is (Ans. 1.895 x 10“^ ms
j,_p/_1.7xl0~^x0.1 = 1.7xlO'^Q 6. A current of 2 A is flowing through a wire of length
4 m and cross-sectional area 1 mm^. If each cubic
A 1.0x10“^
metre of the wire contains 10^^ free electrons, find
.'. Potential difference,
the average time taken by an electron to cross the
V =IR =1.0 X 1.7 X 10“^ = 1.7 X 10'^ V. length of the wire. (Ans. 3.2 X lO^s)
CURRENT ELECTRICITY

I
7. A 10 C of charge flows through a wire in 5 minutes. 4. R = p— and I=enAv^f
The radius of the wire is 1 mm. It contains 5 x 10^ A
/
electrons per centimetre^. Calculate the current V = RI=p
A
.enAvj
and drift velocity. y
(Ans. 3.33 x 10' ^ A, 1.326 x 10" ^ ms ’) or P = ;
euv^J
8. A copper wire of diameter 0.16 cm is connected in 5
series to an aluminium wire of diameter 0.25 cm. A
1.6 X10'^^ X 8 X10^^ X 2.5 X10“^ X 0.1
current of 10 A is passed through them. Find
(/) current density in the copper wire (i7) drift = 1.56x 10'^ Qm.
velocity of free electrons in the aluminium wire. 5. Diameter of wire, D = 1.0 mm = 10 ^m
Tlie number of free electrons per unit volume of Area of cross-section,
29 -3
aluminium wire is 10
tiD^ 7ix(10"^)^ = 7.854xl0“^m^
m

(Ans. 4.976X10*^Am'^ 1.28x10 '^ms ’) A =

w
4 4

9. A current of 30 ampere is flowing through a wire of I 0.2


cross-sectional area 2 mm^. Calculate the drift velo enA 1.6x10'^’x8.4xl0^®x7.854xl0'^
city of electrons. Assuming the temperature of the

Flo
= 1.895 X 10'® ms'\
wire to be 27°C, also calculate the rms velocity at
6. Drift velocity,
this temperature. Which velocity is larger ? Given

ee
1 2
that Boltzman's constant =1.38xlO'^JK \ den
enA 1.6xl0'^^xl0^^xlxl0“®
sity of copper 8.9 g cm'^ atomic mass of copper

Fr
= 63. (Ans. l.lxl0'^ms■^ 1.17x10^ms = 1.25xl0'^ms'^
4
10. What is the drift velocity of electrons in silver wire of = 3.2x10'* s.
Required time, t = —
lengtli 1 m, having cross-sectional area S.HxlO'^’m^ for Vj 1.25x10"*
ur
and carrying a current of 10 A ? Given atomic mass IOC
of silver = 108, density of silver = 10.5 xlO^kg m 7. f=i = = 3.33x10'^ A
t 5x60s
ks
charge on electron = 1.6 x 10'^^ C and Avogadro's I I
number = 6.023 x 10^^ per kg-atom.
Yo

V
(ni^)
oo

enA
(Ans. 3.399 xlO'**ms'*)
en

3.33 X 10'^
11. When a potential difference of 1.5 V is applied
eB

across a wire of length 0.2 m and area of cross-


1.6x10
-19
x5xl0“ xl0^x3.14x(10'^)^
section 0.3 mm^, a current of 2.4 A flows through = 1.326x10'^ ms'*.
the wire. If the number density of free electrons
r

8. As the two wires are connected in series, so current


ou

in the wire is 8.4xl0^m"®, calailate the average


ad

through each wire, I = 10 A.


relaxation lime. Given that mass of electron
(/) Current density in copper wire,
Y

= 9.1x10'®’kg and charge on electron


I 10x4
= 1.6xl0'*^C. (Ans. 4.51x10'*S) ; =
^^^74'3.I4x(0.16xl0'^)^
nd
Re

HINTS = 4.976 xlO^Am'^.


Fi

1. ; = cnAv^i = 1.6 X10'*^ X 8.5 x lo“ x 10'*’ x lO^* (ii) Area of cross-section of aluminium wire,
=1.36 A. TtD^ 3.14x(0.25xl0'^)^ = 4.9xl0"^m^
A =
4 4
I 2.7 1
2, ms
1 10
enA 1.6x10'*^ x9xl0^ x2.5xl0'^
enA 1.6xl0'*%10’*’x4,9xl0'^
= 0.75x10'® ms"’ =0.75 mms'*.
= 1.28x10'* ms'*.
/ 1.8 A 23

3. Current density, j = — 9. No. of atoms in 63 gram of copper = 6.023 x 10


A 0.5xl0"^m^
No. of atoms in 8.9 gram or 1 cm-’' of copper
= 3.6 xl0®Am'^. 6.023 X 10^® X 8.9
Drift speed. 63
3.6x10^
No. of atoms per m® of copper
en 1.6x10
-19
x8.8xl0
28
6.023 X10^® X 8.9 x 10*^
63
= 2.56x10"* ms'*.
3.22 PHYSICS-XII

Electron density, The mobility of a charge carrier is the drift velocity


6.023 X 10
23
X 8.9 X 10^ acquired by it in a unit electric field. It is given by
n - = 8.48xl0^«m'^
63
£
Also / = 30 A, A = 2mm^ =2xl0“^m^,
e=1.6xlO'-^’C qE T
As drift velocity, =
Drift velocity. m

1 30
V
d ~
enA 1.6 X10"^^ X 8.48 X10^® X 2 X10"*^ m

= 1.1x10"® ms"^ For an electron, =


The rms velocity of electrons at 27°C (= 300 K) is %
given by et
h
For a hole,
-23
3k.B T 3x1.38x10
V.
rms -31
m 9x10 The mobilities of both electrons and holes are

= 1.17xl0®ms"^ positive; although their drift velocities are opposite to


each other.
The rms velocity is about 10® times the dnft velocity.
10. Mass of silver wire, SI unit of mobility =m^V"’s"^
m - Alp = 3.14 X10"* X1X 10.5 x 10® Practical unit of mobility = cm^ V^s"^
1 =10“* cm^ V"*s“*
No. of electrons per unit volume of silver,
6.023 X 10^® 3.14 X 10.5 x 10'® Relation between electric current and mobility for a
n = conductor
108 3.14x10"® xl
In a metallic conductor, the electric current is due to

= 5.8557x10
28
its free electrons and is given by
1 I - enAv^
enA But
Vj =H, E .●. / = e«ApgE
10
-19
This is the relation between electric current and
1.6x10 X 5.8557 xl0“ X 3.14x10"® electron mobility.
3.399x10^ ms"^ Relation between electric current and mobility for a
semiconductor
11. £- V _ 1.5 V = 7.5 Vm"‘.
/ ”0.2m The conductivity of a semiconductor is both due to
electrons and holes. So electric current in a semi
Current density. conductor is given by
7 2.4
= 8x10® Am l=f + I,^=enAv^ + epAv,^
'=7 0.3x10"®
2
= enA)x^E +
ne X
As j = <jE = E = eA£(np^ +
m
where n and p are the electron and hole densities of the
_ m■ / _ 9.1xlQ~ X 8 x 10® semiconductor.
~ ne^E ~ M X 10^ x(1.6 x 10"*^)^ x 7.5 Conductivity of a semiconductor. According to
= 4.51xl0"’®s. Ohm's law.
_ El _ £A
●●■(»)
3.14 MOBILITY OF CHARGE CARRIERS R pi 1A p

24. Define mobility of charge carrier. Write relations From equations (i) and (i7), we get
between electric current and mobility for (i) a conductor EA
— = eA£(Hp^ + ppj^)
and (ii) a semiconductor. Hence write an expression for
the conductivity of a semiconductor.
Mobility. The conductivity of any material is due to
or
^ = e{np^ + pp^)
its mobile charge carriers. These may be electrons in
But 1 / p is the electrical conductivity a. Therefore,
metals, positive and negative ions in electrolytes; and
electrons and holes in semiconductors. a
= e(np^ + pp,^)
CURRENT ELECTRICITY

Table 3.3 Mobilities in some materials at room


2 *-l -1 tration
temperature, in cm V s
Find its conductivity. Given : electron mobility
Materials Electrons Holes = 0.135 and hole mobility =0.048 ;
Diamond 1800 1200 e = 1.6xl0 -19' coulomb.

Silicon 1350 480 Solution. Here h =0.45x lO^^m"^ p = 5x 10


Germanium 3600 1800 =0.135 =0.048

InSb 800 450 Conductivity of the semiconductor is


GaAs 8000 300 o = e(n\i^ +
(0.45 X 10^^ X 0.135
- 19
= 1.6x 10
-1
Examples based on + 5x 10^® X 0.048) Sm
Mobility of Charge Carriers = 1.6 X10'^ (0.06075 + 0.24 x 10^) Sm ^

w
Formulae Used = 1.6xl0"^x0.24xl0^Sm“^ =3.84 Sm“\

1. Mobility, = -|- 7^.

Flo
m roblems For Practice
2. Electric current, / = enAv^ - enA pE
1. A potential difference of 4.5 V is applied across a

ee
3. Conductivity of metallic conductor, o = nen ^ conductor of length 0.1 m. If the drift velocity of
4. Conductivity of a semiconductor, a = ne]i ^ + pcfx ^ electrons is 1.5xl0"''ms"’, find the electron

Fr
Units Used
mobility. (Ans. 3.33xlO'-^m^V-’s-’)
2. The number density of electrons in copper is
Conductivity a is in Sm“^ and mobility p in 8.5xl0^®m“^. A current of 1 A flows through a
for
ur
m^V-Vl
copper wire of length 0.24 m and area of
cross-section 1.2 mm^, when connected to a battery
Example 35. Apotential difference of 6 V is applied across of 3 V. Find the electron mobility.
ks
a conductor of length 0.12 ni Calculate the drift velocity of
(Ans. ^gxlO-'^m^V-^s-’)
Yo

electrons, if the electron mobility is 5.6 x 10”^ n^V~^s~^.


oo

3. Mobilities of electrons and holes in a sample of


Solution. Here V =6 V, / =0.12 m,
intrinsic germanium at room temperature are
eB

p =5.6x10“^ 0.54 m^vV’ and 0.18 m^V"’s"^ respectively. If the


Drift velocity, electron and hole densities are equal to
3.6xl0*^m~^, calculate the germanium conductivity.
^ V 5.6xl0'^x6
r

-1

i;^=p£=p.y =
ms
(Ans. 4.147Sm-')
ou

0.12
ad

= 2.8x 10“^ms“^ HINTS


Y

V 4.5 V
Example 36. The number density of electrons in copper is 1. E= = 45 Vm"^
I 0.1m
8.5xl0^m“^. Determine the current flowing through a
nd
Re

copper wire of length 0.2 m, area of cross-section 1 mn?,


-4 -1
V 1.5x10 ms
p =
d _ = 3.33xlO'^m^V"^s"^
when connected to a battery of 3 V. Given the electron £ 45 Vm'^
Fi

mobility -19
= 4.5 x 10"^ and charge on electron V
= 1.6 X 10 C.
2. I enApE = eriAp .—
I
Solution. Here n =8.5 x lO^^m / =0.2 m, n 1 X 0.24

A = lmm^=10“^ml V=3V, ■■ ^ enAV 1.6 x 10'^^ x 8.5 x 10^ x 1.2 xlO"^ x 3


= 4.5x10"^ e = 1.6xlO-^^C. = 4.9xlO'*m2V"^s'^
Electric field set up in the copper wire, = 0.18m^vV\
3, Herepg = 0.54 m^V“^s”\ =

£ = —=A=15 Vm ●1
n = p=3.6xl0^^m"^
/ 0.2
Current, Conductivity,

I = enApE a=c(«p^,+ pp,,) = cn(p^. + p;,)


= 1.6x10
-19
X 8.5x 10^® X10”^ X 4.5 X 10"^ x 15 = 1.6x10
-19
x3.6xl0^^(0,54+0.18)
= 0.918 X = 4.147 Sm"\
3.24 PHYSICS-XII

3.15 TEMPERATURE DEPENDENCE OF Alloys have high resistivity. The resistivity of nich-
RESISTIVITY rome has weak temperature dependence [Fig. 3.21(b)]
while that of manganin is almost independent of
25. Explain the variation of resistivity of metals, temperature. At absolute zero, a pure metal has negli-
semiconductors, insulators and electrolytes with the gib,y ersisKvity while an alloy (like nichrome)
change in temperature. Define temperature coefficient of has some residual resisHvity. This fact can be used to
resistivity. distinguish a pure metal from an alloy.
Temperature dependence of resistivity. The resisti As K = p-
I
i.e., R ocp
vity of any material depends on the number density n A
of free electrons and the mean collision time z.
m
Thus equation (1) can be written in terms of resis
tances as
’’ ne^z
^ (1 Cl 0
1. Metals. For metals, the number density n of free where R.t = the resistance at PC
electrons is almost independent of temperature. As
temperature increases, the thermal speed of free Rq = the resistance at 0‘^C and
electrons increases and also the amplitude of vibration t = the rise in temperature.
of the metal ions increases. Consequently, the free 2. Semiconductors and insulators. In case of insu
electrons collide more frequently with the metal ions. lators and semiconductors, the relaxation time x does
The mean collision time t decreases. Hence the resistivity not change with temperature but the number density
of a metal (p x 1 / x) increases and the conductivity decreases of free electrons increases exponentially with the
with the increase in temperature. increase in temperature. Consequently, the conductivity
For most of the metals, resistivity increases linearly increases or resistivity decreases exponentially with
with the increase in temperature, around and above the increase in temperature,
the room temperature. In such cases, resistivity p at
The number density of electrons at temperature T is
any temperature T is given by given by
p = Po [l + a(r-(Tj)l ...(1) n(,T) = n^e-'^eV
where p^ is the resistivity at a lower reference
where cfg is the Boltzmann constant and is the
temperature Tq (usually 20° Q and a is the coefficient
energy gap (positive energy) between conduction and
of resistivity. Obviously, valence bands of the substance.
a =
P-Pq 1 dp ^ 1
Asp X — , so we can write
Po(^-'^o) Po n

1 1 -£„ /koT
Thus, the temperature coefficient of resistivity a
may be defined as the increase in resistivity per unit P Po ^
resistivity per degree rise in temperature. E, /k^T
-1
or
P(^) = Po
The unit of a is °C . For metals a is positive. For
many metallic elements, a is nearly 4xlO"^°C"\ For This equation implies that the resistivity of semicon
such conductors, the temperature dependence of p at ductors and insulators rapidly increases with the decrease
low temperatures is non-linear. At low temperatures, in temperature, becoming infinitely large as T -> 0.
the resistivity of a pure metal increases as a higher power At room temperature, k^T =0.03 eV. Whether the
of temperature, as shown for copper in Fig. 3.21(a). non-conducting substance is an insulator or a semi
? conductor, depends on the size of the energy gap, E :
'o
-0.4 a
3 -1,20
(i) If < 1 eV, the resistivity at room temperature
o.
a
is not very high and the substance is a
semiconductor.
●I' -0.2 ●i.ia

(«) If > 1 eV, the resistivity at room temperature


X 1.00 is very high (~ 10^ O m) and the substance is an
0 50 100 150 200 400 600 800 insulator.
(a) Temperature T(K) Temperature T(K)-» (b) The coefficient of resistivity (a) is negative for
Fig. 3.21 (a) Variation of resistivity p of copper with carbon and semiconductors i.e., their resistivity
temperature, (b) Variation of resistivity p of decreases with temperature, as shown in
nichrome with temperature. Fig. 3.22.
CURRENT ELECTRICITY

1 1
t = — = 256®C
a 3.9x10"^
t Thus the resistance of copper conductor becomes
P double at256“C

(n) Since a does not depend on size and shape of


the conductor, so the above result holds for all copper
conductors.

Fig. 3.22 Resistivity of a semiconductor decreases Example 39. The resistance of the platinum wire of a
rapidly with temperature. platinum resistance thermometer at the ice point is 5 Q and
at steam point is 5.23 O. When the thermometer is inserted
3. Electrolytes. As the temperature increases, the ^ resistance of the platinum wire is 5.795 Q.
and Calculate the temperature of the bath.
interionic attractions (solute-solute, solvent-solute [NCERT]
solvent-solvent types) decrease and also the viscous

w
3 ^ 5 23 n, R, = 5.795 n
forces decrease, the ions move more freely. Hence ^ ‘
conductivity increases or the resistivity decreases as As K^ = Ro(l+a()
the temperature of an electrolytic solution increases. R, - Rg = Rq af ●●●{0

Flo
26. Why alloys like constantan or manganin are used ...(ii)
for making standard resistors ?

ee
On dividing (i) by (ii), we get
Use of alloys in making standard resistors. Alloys

Fr
like constantan or manganin are used for making stan
dard resistance coils because of the following reasons;
^100 “ ^
{i) These alloys have high value of resistivity. or t = xlOO
for
ur
{ii) They have very small temperature coefficient. ^100 ^
5.795 - 5 0.795
So their resistance does not change appreciably xl00 = X 100= 345.65° C
even for several degrees rise of temperature. 5.23-5 0.23
ks

(Hi) They are least affected by atmospheric


Yo

Example 40. A nichrome heating element connected to a


oo

conditions like air, moisture, etc.


220 V supply draws an initial current of 2.2 A which settles
(it?) Their contact potential with copper is small. down after a few seconds to a steady value of 2.0 A. Find the
eB

steady temperature of the heating element. The room


Examples based on temperature is 30° C and the average temperature coefficient
Temperature Variation oF Resistance of resistance of nichrome is 1.7 x 10"^ per°C.
r
ou
ad

Formulae Used Solution. Here V=220V, 1^ =2.2 A, l2 =2.0 A,


= 1.7x 10^°C"^
Y

Temperature coefficient of resistance a

a =
Resistance at room temperature of 30°C,
V 220
(^2 h)
nd

=100 n
Re

^1 = I 2.2
Iffj =0°C and <2 =f°C, then 1
Fi

Resistance at steady temperature,


a - — or
R,=R,(l+af) V 220
R„xf = 110Q
h 2.0
Units Used

Resistances are in Q, temperatures in °C or K. As a =

R,{t^-t,)
Example 38. (i) At what temperature would the resistance _hz3i- 110-100
= 588°C
^2-^1 100x1.7x10“^
of a copper conductor be double its resistance at 0°C ? R,a
1

(ii) Does this temperature hold for all copper conductors


Steady temperature.
regardless of shape and size ?
Given a/or Cu =3.9 x 10'^ ^2 = 588 + = 588 + 30 = 618°C.
Example 41 .An electric toaster uses nichrome (an alloy of
Solution, (i) a = rtickel and chromium) for its heating element. When a
R,(f2-f,) Ro(f-O) t negligibly small current passes through it, its resistance at
3.26 PHYSICS-XII

3.75-3.15
room temperature (27.0°C) is found to be 75.3 Q. When the a =

toaster is connected to a 230 V supply, the current settles (3.15 X 100)-(3.75 x 20)
after a few seconds to a steady value of 2.68 A. Wltat is the 0.60 ^0.60 = 0.0025°C”^
steady temperature of the nichrome element ? The tem 315-75 “240
perature coefficient of resistance of nichrome averaged over
the temperature range involved is 1.70 x 10"^°C“\ R
t 3.15
= 3.on.
fNCERT] 1 + af^ 1 + 0.0025x20
Solution. Here R^ =75.3 0, t^ =27°C
Example 44. A standard coil marked 2 Q is found to have
230
= 85.8 0,
a resistance of 2.11S Q «^30®C. Calculate the temperature at
2.68 ivhich the marking is correct. The temperature coefficient of
85.8-75.3 the resistance of the material of the coil is 0.0042®C~^
= 820°C
R,1 a 75.3 X 1.70 X 10"'^ Solution. Rj = Rg (1 + a ) and R^ = Rg (1 + a 12)
R 1 + a
1 _
Steady temperature.
R2 1 + at2
^2 = 820 + fj =820 + 27 = 847°C.
At the steady temperature, the heating effect due to Here, R^=2Q, R^ =2.118 0, t2=30°C, t^=2
the current equals heat loss to the surroundings. 2 _ 1 + 0.0042 X _ 1 + 0.0042 x fj
Example 42. The resistance of a tungsten filament at 2.118 “1 + 0.0042 x30 1.126

150°C is 133 ohm. What will be its resistance at 500°C ?


or 1 + 0.0042 = 2 X 1.126 _ 2.252
The temperature coefifcient of resistance of tungsten is 2.118 “ 2.118
0.0045 pcr°C.
Solution. Here R 150 = 1330, a =0.0045®C R = ? .-. t 1 [2.252 -1
0.104
-IS^C
0.0042 L2.II8
500 1 “
0.0042 X 2.118
Now
Rj = Rg (1 + a f)
R i.e., the marking will be correct at 15°C
150 = Rg (1 + a X 150)
Example 45. A potential difference of 200 V is applied to a
or
133 = Rg (1 + 0.0045 X 150) ...(1)
coil at a temperature ofl5°C and the current is 10 A. What
and R
500 = Rg (1 + a X 500)
will be the mean temperature of the coil when the current has
R
500 = Rg (1 + 0.0045 X 500) ...(2) fallen to 5 A, the applied voltage being same as before ?
or

1 o C"' flto^c.
Dividing (2) by (1), we get Given a =
234
R 1 + 0.0045x 500 3.25
500 _
Solution. In the second case, the current decreases
133 1 + 0.0045x150 1.675
due to the increase in resistance on heating.
3.25
or R 500 X133 = 258 n. V 200
“ Now R = 20 a
1.675 15 ~ T
1 ~ 10
Example 43. The resistance of a conductor at 20®C is Let t be the temperature at which current falls to
3.15 Q and at 100®C is 3.75 fl. Determine the temperature 5 A. Then
coefficient of resistance of the conductor. What will be the 200
resistance of the conductor at0°C ? R.I = = 400
5
Solution. Rj = Rg (1 + a f^) and R2 = Rg (1 + a f2) As
= RQ(l+a.t)
On dividing.
Rj _ 1 + ct fj R,. = R.0 1 +
15 ^ 20 =
Rgx249
15
or ...(1)
234 234
R2 1 + a f.,
t 234+ f
or R.J (1 + a f2) = R2 (1 + cc fj) R.t = R.0 1 + or 40= R
0 ...(2)
234 234
or a =
^2 ~ ^1
Rl #2 - fl Dividing (2) by (1),

Here =20°C, Rj =3.15 n,


3_234+f
249
f2=100°C R2 =3.75 0 or f = 498-234 = 264“C
CURRENT ELECTRICITY 5.27

Example 46. The temperature coefficient of resistivity, for 2. The resistances of iron and copper wires at 20° C are ?
two materials A and k are 0.0031°C~^ and 0.0068°C~^, 3.9 n and 4.1 £2 respectively. At what temperature
respectively. Two resistors, Rj and R2, made from materials will the resistances be equal ? Temperature
A and 8 respectively, have resistances of200 O and 100 O coefficient of resistivity for iron is 5.0 x 10~^K”^ and
at 0°C Show on a diagram, the ‘colour code', of a carbon for copper it is 4.0 x 10"^ Neglect any thermal
resistor, that would have a resistance equal to the series expansion. (Ans.84.5° C)
combination of R^ and R2, at a temperature of 100°C. 3. The resistance of a silver wire at 0°C is 1.25 Q. Upto
(Neglect the ring corresponding to the tolerance of the what temperature it must be heated so that its
carbon resistor) ICBSE D 16C] resistance is doubled ? The temperature coefficient
Solution. Here = 0.0031°C"\ a
2 = 0.0068° of resistance of silver is 0.00375 °C“^ Will the
temperature be same for all silver conductors of all
At0°C R^ =200 0, R^ =100 0 shapes ? (Ans.267° C Yes)
At 100°C the resistances will be 4. The resistance of a coil used in a platinum-resis

w
R; = R^(1-f ttjAf) =200(1 + 0.0031 X 100)0 = 262 O tance thermometer at 0°C is 3.000 and at 100°C is
3.75 O. Its resistance at an unknown temperature is
R:^ = R^{1+ a^Af) = 100(1 + 0.0068 x 100)0 = 168 O measured as 3.150. Calculate the unknown

Flo
Total resistance of the series combination at 100°C, temperature. (Ans.20°Q
R' = R; + R^ =262 +168 = 4300 = 43 X lO’O 5. The temperature coefficient of a resistance wire is

ee
r\ 0.0012.5°C”*. At 300 K, its resistance is lO. At what
temperature the resistance of the wire will be 2 O ?

Fr
\1 HIT 80]
Yellow J
Brown (Ans.1127 K)
(4) Orange 1

for
6. The temperature coefficient of resistivity of copper
ur
(3) (10) Fig. 3.23
is 0.004°C"^ Find the resistance of a 5 m long
Example 47. A metal wire of diameter 2 mm and length copper wire of diameter 0.2 mm at 100°C, if the
100 m has a resistance of 0.5475 O at 20° C and 0.805 O at resistivity of copper at 0°C is 1.7 x 10~® Om.
ks
150°C. Find (i) the temperature coefficient of resistance
Yo

(Ans.3.8 0)
(ii) resistance at 0°C (lii) resistivities at 0° and 20°C.
oo

Solution. Here r=lmm=10 m. / = 100m, HINTS


eB

fj =20°C Rj =0.5475 0, f2=150°C, R2 =0.8050 1. a =


Ri-Rq _ 20-10 2
R2-R1 _ 0.805-0.5475 Rq xt 10x273 273
(0 a =
r

Rj(f2-fj) ~ 0.5475 (150 - 20) 2. Let resistance of iron wire at t°C


ou
ad

= 3.6 X
= Resistance of copper wire at t °C
Y

R
1
0.5475 0.5475 R20 [1 + a (f - 20)] = R^o [1 + a' (f - 20)]
(”) ^ = 1 + af, l+3.6xl0“^x20 1.072 3.9[1+ 5.0 X 10^^(f - 20)] = 4.1 [1 + 4.0 x 10“^ (f - 20)]
nd

1
[3.9x5-4.1x4] xlO"^ x(f -20) = 4.1-3.9
Re

= 0.51070.
0.2
t - 20 = = 64.5
Fi

X 71
r^ 0.5107x3.14x(10~^)^ 3.1x10'^
{in) Pq = I I 100 t = 64.5+ 20 = 84.5° C.
8
= 1.60x10"° Om. 3. Proceed as in Example 38 on page 3.25.
Resistivity at 20°C is 4. R, = Rfl(l+ af)
P20=Po(l+“^) R^oo = Rq (1+ a xlOO)
(l + 3.6xl0"^x20)
-8
= 1.60x10 3.75 = 3.00(1+ a x 100)
-8
= 1.60 X10"° X 1.072 = 1.72 x 10 Om. 3.75
-1=100a
3
/ TOblems For Practice 0.75
a = = 0.0025°C"^
1. A platinum wire has a resistance of 10 O at 0°C and 3x100

of 20 O at 273°C. Find its temperature coefficient of


R, - Rq 3.15 - 3.00
resistance. (Ang..^oC"^)
t = = 20°C.
X a 3.00 X 0.0025
PHYSICS-Xil

300 K = 300 - 273 = 27®C


given temperature, rather it is current dependent.
R2^= J?o(l+ax27) = ia Non-ohmic situations may be of the following types :
and Kj = Rp(l+axi) = 2Q (0 The straight line V-I graph does not pass
1+at 2 through the origin.
■ ■ 1 + 27a “ : (ii) V-I relationship is non-linear.
or 1+ = 2 + 54a
(in) V-I relationship depends on the sign of V for
the same absolute value of V, and
1+ 54 a 1+ 54x0.00125
or t = = 854°C. (it;) V-I relationship is non-unique.
a 0.00125
Examples of non-ohmic conductors. (/) Metallic
= 854+273 ==1127 K.
conductor. For small currents, a metallic conductor
6. Pioo =Po(l+ a() = 1-7 xl0“®(l+ 0.004x100) obeys Ohm's law and its V-I graph is a straight line. But
= 2.38 X 10'®nm when large currents are passed through the same con
I 138xl0~® x5 ductor, it gets heated up and its resistance increases. V-I
R=p -3.80. graph no longer remains linear, i.e., conductor becomes
71
3.14x(0.1x10"^)2 non-ohmic at higher currents, as shown in Fig. 3.25(a).
Non-ohmic
3.16 LIMITATIONS OF OHM'S LAW : OHMIC behaviour
AND NON-OHMIC CONDUCTORS
27. State the conditions under which Ohm’s law is
not obeyed in a conductor. What are ohmic and
non-ohmic conductors ? Give examples of each type.
Limitations of Ohm's law. Ohm's law is obeyed by
o
many substances under certain conditions but it is not 7

a fundamental law of nature.


Fig. 3.25 (a) V-I graph for a metallic conductor.
Ohmic conductors. The conductors which obey Ohm's
law are called Ohmic conductors. For these conductors, (ii) Water voltameter. Here a back e.m.f. is set up
the linear relationship between voltage and current due to the liberation of hydrogen at the cathode and
(Vcc/) holds good. The resistance {R = V/I) is oxygen at the anode. No current flows through the
independent of the current I through the conductor. In voltameter until the
7.
these conductors, the current I gets reversed in direc applied p.d. exceeds the
tion when the p.d. V is reversed, but the magnitude of back e.m.f. Vq (1.67 V for
current changes linearly with voltage. Thus the V-I water voltameter). So V-1
graph for ohmic conductors is a straight line passing through graph is a straight line
the origin. A metallic conductor for small currents and but not passing through
the electrolyte like copper sulphate solution with the origin, as shown in
7,0 y
copper electrodes are ohmic conductors, as shown in Fig. 3.25(&). Hence the
Figs. 3.24(a) and (b) respectively. electrolyte (water acidi Fig. 3.25 (b) V-I graph for a
+ LI fied with dil. H2SOJ is a water voltameter.
Pure CuSOj solution non-ohmic conductor.
metal with Cu electrodes
(Hi) p-n junction diode. It consists of a junction of
4
77-type and n-type semiconductors (For details, refer to
-V O + V o + v
+ 7

-h'

-V
(a) (b) O + v
Fig. 3.24 Ohmic conductors.

Non-ohmic conductors. The conductors which do not


obey Ohm's law are called non-ohmic conductors. The -7

resistance of such conductors is not constant even at a


Fig. 3.26 V-J graph for a junction diode.
CURRENT ELECTRICITY 3.29

chapter 14 Vol. II). A voltage V is applied across the 3.17 SUPERCONDUCTIVITY


junction. The resulting current I is shown in Fig. 3.26. 28. What is superconductivity ? What is its cause ?
Obviously, I is not proportional to V. Further, very
little current flows for fairly high negative voltage Superconductivity. In 1911, Prof Kamerlingh Onnes at
(called negative bias) and a current begins to flow for the University of Leiden (Holland), observed that the
much smaller positive (forward) bias. Thus Uie junction resisHvity of mercury suddenly drops to zero at a tempe-
diode allows current to flow only in one direction i.e., it arhtre of about 4.2 K and it becomes a superconductor,
acts as a rectifier (converts a.c. into d.c.).
(iv) Thyristor. It consists of four alternate layers of p 0.16 -
t
and «-type semiconductors. Its V-I relationship is as
a
shown in Fig. 3.27(a). It can be easily seen that (a) the 5^ 0.08
V-1 relation is non-linear, (b) V-I relationship is
different for positive and negative values of V, and 1/

w
i
(c) in certain portions, there are two or more values of 0 2 4 6 8

current for the same value of voltage, i.e., the V-I r(K)^
relationship is not unique. The region ABis interesting

Flo
because the current carried by the device increases as Rg. 3.28 Mercury loses complete resistance at 4.2 K.
the voltage decreases, i.e., a is negative in this region.

ee
The phenomenon of complete loss of resistivify afy
certain metals and alloys when they are cooled below

Fr
a certain temperature is called superconductivity.

A
The temperature at which a substance undergoes a
transition from normal conductor to superconductor in
B
for
ur
a zero magnetic field is called transition or critical
>
O temperature (TJ.
-y ^ + v
ks
A current once set up in a superconductor persists
Yo

for a very long time without any apparent change in its


oo

magnitude.
eB

W-I Cause of superconductivity. It is believed that near

the transition temperature, a weak attractive force acts


Fig. 3.27 (a) V-I curve for a thyristor, on the electrons which brings them closer to form
r

coupled pairs. Such coupled pairs are not deflected by


ou
ad

(u) Gallium arsenide. Fig. 3.27(b) shows the V-I ionic vibrations and so move without collisions.
graph for the semiconductor GaAs. It exhibits 29. What is Meissetier effect in superconductors ?
Y

non-linear behaviour. Moreover, after a certain Meissener effect. In 1933, Meissner and Ochsenfeld
voltage, the current decreases as the voltage increases. observed that if a conductor is cooled in a magnetic
nd
Re

That is, if is positive then Al is negative and hence field to a temperature below the transition tempe
the effective resistance (= AV j AI) is negative. rature, then at this temperature, the lines of magnetic
Fi

induction B are pushed out of the specimen, as shown


, I

t i c
in Fig. 3.29. Thus B becomes zero inside a supercon
2
.2
c
» 5 « 2
00 ^
ducting specimen.
1C
M
OJ 4) 'K SJ
o *- 2 ^ M I. I. >>
Z
I

(mA)T

B
U

Voltage V (V) ->


T>T(- T<Tc

Fig. 3.27 (fc) V-I graph for GaAs. Fig. 3.29 Meissner effect in a superconductor.
3.30 PHYSICS-XII

The expulsion of the magnetic flux from a supercon is no change in the current in the circuit and in the potential
ducting material when it is cooled to a temperature difference between the terminals of the combination, then the
below the critical temperature in a magnetic field is single resistance is called the equivalent resistance of the
called Meissner effect. combination.

Meissner effect indicates that as the supercon 32. When are the resistances said to be connected in
ductivity appears in a material, it becomes perfectly series ? Find an expression for the equivalent resistance of
diamagnetic. a number of resistances connected in series.
30. What is high superconductivity ? Mention Resistances in series. If a nwnber of resistances are
important applications of superconductors. connected end to end so that the same current flows through
High 7^ superconductivity. A current once set up in each one of them in succession, then they are said to be
a superconducting loop can persist for years without series. Fig. 3.30 shows three resistances Ry
any applied emf. This important property of super- ^ connected in series. When a potential dif-
conductors can have important practical applications, ^ applied across the combination, the same
A serious difficulty in their use is the very low tern- ‘^^■^^ent 1 flows through each resistance,
perature at which they must be kept. Scientists all over R, ^3
the world are busy to construct alloys which would be »—vw
superconducting at room temperature. Superconduc + ^2 + ^3
tivity at around 125 K has already been achieved and
efforts are being made to improve upon this temperature.
h

Tobie 3.4 Critical temperatures of some


superconducting materials Fig. 3.30 Resistances in series.
Material
kM By Ohm's law, the potential drops across the three
Hg 4.2 resistances are

Pb2Au 7.0
V.^IRy V, = IR^, V^ = IR^
YBa2Cu307 90 If Rj is the equivalent resistance of the series
combination, then we must have
Tl2Ca 2Ba 2CU3OJQ 120

Applications of superconductors. The possible But V = Sum of the potential drops across the
applications of superconductors are individual resistances
1. For producing high magnetic fields required for or
K = Vj + 1^2 +
research work in high energy physics. or
IR^ = IR^ + IR^ + IRj
2. For storage of memory in high speed computers.
3. In the construction of very sensitive galvano
or
Rg = Rj + /?2 + R3
meters.
The equivalent resistance of n resistances connected
4. In levitation transportation (trains which move in series will be
without rails).
Rg — R2 + R2 + Rg + R.
5. In long distance power transmission without Thus when a number of resistances are connected in
any wastage of power.
series, their equivalent resistance is equal to the sum of the
individual resistances.
3.18 RESISTANCES IN SERIES AND PARALLEL
Laws of resistances in series
31. What do you mean by equivalent resistance of a
combination of resistances ? (/) Current through each resistance is same,
Equivalent resistance of a combination of resis {ii) Total potential drop = Sum of the potential drops
across the individual resistances.
tances. Sometimes, a number of resistances are
connected in a circuit in order to get a desired value of {Hi) Individual potential drops are directly proportional
current in the circuit. Resistances can be connected m
to individual resistances,

series, in parallel or their mixed combination can be used. (iv) Equivalent resistance - Sum of the individual resistances.
If a combination of two or more resistances in any electric (i?) Equivalent resistance is larger than the largest
circuit can be replaced by a single resistance such that there individual resistance.
CURRENT ELECTRICITY iJl

33. When are the resistances said to be connected in Laws yf resistaiiafs in parallel
parallel ? Find the equivalent resistance of a number of (/) Potential drop across each resistance is same,
resistances connected in parallel. (ii) Total current = Sum of the currents through
Resistances in parallel. If a number of resistances are individual resistances.
connected in between two common points so that each of (Hi) Individual currents are inversely proportional to the
them provides a separate path for current, then they are said individual resistances,
to be connected in parallel. Fig. 3.31 shows three resis
{iv) Reciprocal of equivalent resistance = Sum of the
tances Ry R2 and R^ connected in parallel between reciprocals of the individual resistances.
points A and B. Let V be the potential difference
applied across the combination. {i») Equivalent resistance is less than the smallest
individual resistance.
R1
I1 wv Examples based on
A R, B Combinatioiror Resistances in

w
WV
Series and Parallel
/3
WV Formulae Used
v/

Flo
1. The equivalent resistance of a number of resis
tances connected in series is given by

ee
V
Rg = R^ + R2+R^'^-"

Fr
Fig. 3.31 Resistance in parallel. 2. The equivalent resistance Rp of a number of resis
tances connected in parallel is given by
Let ly I2 and Z3 be the currents through the resis J_ J_ -I- ...

for
ur
tances Ry R2 and R^ respectively. Then the current in
the main circuit must be /= -h + ^3
3. For two resistances in parallel.
Since all the resistances have been connected
Currents through the two resistors will be
ks
between the same two points A and B, therefore, poten
Yo

tial drop V is same across each of them. By Ohm's law, Z _

and I2 =
oo

1 “
Rj + i?2 R^ + R2
the currents through the individual resistances will be
eB

y y Units Used
Z
1 “
R,1 ' h~
R2' R3 All resistances are in ohm (f2).

If Rp is the equivalent resistance of the parallel Example 48. A wire of resistance 4 R is bent in the form of
r
ou

combination, then we must have


ad

a
circle (Fig. 3.32). What is the effective resistance between
y [CBSE D 1(1]
the ends of the diameter ?
Y

Z =
2R
2R
AW
nd

Z - Z^ -I- Zj Zg
Re

But AO oB AO OB
2R

y _ y y y AW
Fi

2R
or

^ ^ Fig. 3.32 Fig. 3.33

or
j_ _ j_ ^ j_ Solution. As shown in Fig. 3.33, the two resistances
R^ R, R^ R3 of value 2 R each are in parallel with each other. So the
resistance between the ends A and Bof a diameter is
The equivalent resistance Rp of n resistances 2Rx2R
connected in parallel is given by R' =
2R+2R A

_L = J_ J_ J_ -I- -I-
1
= R.
R, 3on 30 n
-4t2V
Example 49. Find the value
Thus when a nwnber of resistances are connected in
of current I in the circuit AW ●
parallel, the reciprocal of the equivalent resistance of the shown in Fig. 3.34. B son c
parallel combination is equal to the sum of the reciprocals of [CBSE F 1)3. IIT 83j
the individual resistances. Fig. 3.34
■332 PHYSICS-Xil

Solution. In the given circuit, the resistance of arm


ACB (30 + 30 =60 Q) is in parallel with the resistance of
arm /4B(=30n).
Hence the effective resistance of the circuit is
30x60
R = =20 n
30+60
V 2
Current, / = — = — = 0.1 A.
R 20

Example 50. Determine the voltage drop across the


resistor in the circuit given below with 6= 60 V, Now resistances Rj, R and R^ form a series com
Rj =18Q, =10a bination. So resistance between the ends A and £ is
Solution. As the resistances R^ and R^ are in series, R' =10 +
20
+ 10=26.670.
their equivalent resistance 3

= 5 + 10=150.
Example 52. A set of n identical resistors, each of resis
R, = 18Q R3 = 5n tance RO, when connected in series have an effective
I Wv—f _VvV resistance XQ and when the resistors are connected in
parallel, their effective resistance is Y O, Find the product of
-i- S = 60 V R., = 10 Q R4 = 10Q X and y. [CBSE OD 19)
Solution. The effective resistance of the n resistors
connected in series is
Fig. 3.35 X=R+R+R+ n terms = nR

The series combination of R^ and R^ is in parallel The effective resistance Y of the n resistors
with R^. Their equivalent resistance is connected in parallel is given by
10x15 ^150 = 60 1 = 1 1 1 + ....n terms = —
n

10 + 15 25 y~ r"^ r"*" R R

R
The combination R' is in series with Rj. or y = —
.'. Total resistance of the circuit. n

R=6 + 18=240 R
xy = nR . - = r2.
€ 60 n
Current, — =2.5 A
R 24
Example 53. Aparallel combination of three resistors takes
.●. Voltage drop across Rj a current of 7.5 A from a 30 V supply. If the two resistors
are 10 O and 12 O, find the third one.
= IR^ =2.5x 18 V =45 V.
Example 51. A letter A consists of a uniform wire of Solution. Here R
^
=—
I 7.5
= 40
resistance 1 ohm per cm. The sides of the letter are each 20 cm
long and the cross-piece in the middle is 10 cm long while the But
1 1 1
+
1
apex angle is 60°. Find the resistance of the letter between the R
1
tioo ends of the legs.
1 1 1 1
Solution. Clearly, or

AB= BC = CD =D£=BD =10 cm 4 10 12 R3


R, = R2 = R3 = R^ = R5 = 10 O or
.-. R3=150.
As R2 and R3 are in series, their combined resis R3 ~ 4 60 “ 15
tance = 10 + 10 = 20 0. This combination is in parallel
with R5(=100). Example 54. When a current of 0.5 A is passed through
two resistances in series, the potential difference betzveen the
Hence resistance between points B and D is ends of the series arrangement is 12.5 V. On connecting
given by
them in parallel and passing a current of 1.5 A, the potential
1_J_ J__3^ or
20
R= —O difference between their ends is 6 V. Calculate the two
R~20 10 "20 3 resistances.
CURRENT ELECTRICITY 333
●K

Solution. For series cornbination, V =12.5 V, / = 0.5 A Example 57. Two metallic rods, each of length L, areas of
12.5 cross-section and A^, having resistivities pj and p2 are
= 25.0n ...(1)
0.5 connected in parallel across a d.c. battery. Obtain the
For parallel combination, y =6.0 V, / = 1.5 A
expression for the effective resistivity of this combination.
(CBSE D 201
V
or
Rj ^2 _ 6.0 = 4.0 Solution. For the parallel combination,
I
R, + R2 1.5 1 1 1
or
Rj R2 = 4(R^ + R2) = 4x25 =100 R
eff
R
]

= (25)^-4x100=225 = A+A
p^L P2L
Rj - R2 = 15 ...(2)

w
Solving (1) and (2), R^ = 20 Q, = 5 n. L

Pi
Example 55. In the circuit given below, find the voltmeter
reading across a 4H resistor. [CBSE OD 19] P2 ^2

Flo
20. 2Q
A
AW Fig. 3.38

ee
8n 4n
16V-=. V
(^ +A - Apz +

Fr
P1P2
-VW
20 B 20
(Aj + A^)p^P2
for
ur
Fig. 3.37 Peff-
AjP2+ A2P1
Solution. Resistance between A and B on the right
8x8 Example 58. A wire of uniform cross-section and length I
ks
hand side of the circuit = = 40.
8+8 has a resistance of 16 Q. It is cut into four equal parts. Each
Yo

part is stretched unifonnly to length I and all the four stretched


oo

Equivalent resistance of the circuit,


parts are connected in parallel. Calculate the total resistance
R=2+4+2=8Q
eB

16 V
of the combination so formed. Assume that stretching of wire
Current drawn from the battery, I = = 2 A does not cause any change in the density of its material.
8Q
Solution. Resistance of each of the four parts of
r

At the junction A, the current of 2 A is divided length//4 = 4 Q. When each part is stretched to length/,
ou
ad

equally between the 8fl resistance and the remaining volume remains same,
circuit of 80 resistance.
Y

V = A'l' = Al
.'. Current through the 40 resistance = 1 A
A' _ I _l/4 _1
nd

Voltmeter reading = 40 x 1A = 4 V.
Re

or
A~ V~ I 4
Example 56. Three conductors of conductances C2and
Fi

R / A' 1 1 1
G3 are connected in series. Find their equivalent conductance.
K V A 4 4 16
Solution. As conductance is reciprocal of resis
tance, therefore or R'=16x R=16x4=640
R1 “
i.e., resistance of each stretched part is 640. When
these four parts are connected in parallel, the total
For the series combination, R = R^ + R2 + Rg resistance R of the combination is given by
1
— = — + — +
1 1 1 1_
G Gj G) Gg GjGjG, R 64 64 64 64

or equivalent conductance. = 3= —
“ 64 " 16
G =
G2G3 + GjGj + Gj(^ or R=16 O.
3.34 PHYSICS-XII

Example 59. Find the equivalent resistance of the given Similarly, resistance between A and E
circuit between the terminals A and B. (JEE Main Aug 21) 1
2a 20
A/s/sr ■OA
1
+ -
^=3Q
3+3 6
50 2 0 3o: 30
Resistance between A and F

1
■OB =30
1 1
+ -
Fig. 3.39 3 +3 6
Solution. 5 Q resistance is shorted.
20 Finally, resistance between A and B
■o A 1
= 20
1 1
+ -
20 20 30 30
3+3 3

o B
Thus the effective resistance between A and B is
2 0.

20
Example 61. Find the effective resistance of the network
AVvr ■o A
shown in Fig. 3.41 between the points A and B when (i) the
switch S is open (ii) switch S is closed.
>
lO 30 30 60 12 0
-Vvv
o
-O 6
o- ■o
A B

■oA ■vw
120 60

30 30 30
Fig. 3.41

OB
Solution, (i) When the switch S is open, the resis
_J_ = i 1 i-1 R^e=10. tances of 6 O and 12 O in upper portion are in series,
the equivalent resistance is 18 O. Similarly, resistances
in the lower portion have equivalent resistance of 18 O.
Example 60. Find the effective resistance between points A
and B for the network shown in Fig. 3.40. Now the two resistances of 18 O are in parallel between
points A and B.
.'. Effective resistance between points A and B
18x18
= 9a
18 + 18

{ii) When the switch S is closed, the resistances of


6 Q and 12 Q on the left are in parallel.
Their equivalent resistance is
Fig. 3.40 6x12
= 4n
6 + 12
Solution. At points A and D, a series combination
of 3 Q, 3 n resistances (along AC and CD) is in parallel Similarly, the resistances on the right have
with 60 resistance (along AD), therefore, resistance equivalent resistance of 4 0. Now the two resistances
between A and D of 40 are in series.

1 -—^0=30
+ ~
Effective resistance between points A and B
3+3 6 = 4 + 4 = 80.
CURRENT ELECTRICITY 3J5

Example 62. Calculate the current shown by the ammeter Solution. In the steady state (when the capacitor is f"'
A in the circuit shown in Fig. 3.42. [CBSE OD 2000] fully charged), no current flows through the arm BE. In
5Q the loop ABCDEFA,
vw
C^, Net emf
■Q ^ / =
Total resistance
Q-
●Q
+
(12 -6)V = 2 A
5Q
(2+i)n
Now, V = ,VBE
AF

or 6-1x2
lOV

Fig. 3.42 v^=zy

w
Solution. The equivalent circuit is shown in Fig. 3.43. Charge stored in the capacitor,
I WV 1 Q= CK = 5pFx2V = 10
lOO 50 10 O
■WV

Flo
Example 64. An ideal cell of emf 10 V is connected in
10 o 10 o circuit as shown in Fig. 3.45. Each resistance is oflCl.Find
WV-i WV the potential difference across the capacitor when it is fully

ee
+
charged. [JEE Main Sept. 20]
AW

Fr
5Q
AW
c

for Ri
ur
lOV

Fig. 3.43
i?4
ks
For the two 10 Q resistances connected in parallel,
Yo

10x10
oo

10 V
equivalent resistance = = 50
10 + 10
eB

For two such combinations connected in series, Fig. 3.45

equivalent resistance =5 + 5=100 Solution. In the steady state (when the capacitor is
Now we have resistances of 50,10 0 and 5 0 fully charged), no current flows through C and Ry The
r

effective circuit for the current flow is as shown in


ou

connected in parallel, so
ad

Fig. 3.46.
1=1 J_ 1=1
Y

R 5 10 5 2 2Q A
AMr
or K = 20 ' ■ 1 A
nd
Re

Also V^= 10 V A 2n
2Q
i^AWn^
Fi

.'. Current, I- = —
^=“=5A. AW
R 2 2A B 2Q

Example 63. In the given circuit (Fig. 3.44), with steady


current, calculate the potential difference across the 10 V

capacitor and the charge stored in it. [CBSE F 17]


6V Fig. 3.46
in
F
A 1 AW 4x2 10
R + 2 = —O
4 +2 3
6V C = 5pF
e 1 £ 10x3
+ / = = 3A
10
12 V 2Q
V = VABC ~ ^AB ^BC
C AW D capacitor
= lx2+3x2 =8 V.
Fig. 3.44
PHYSICS-XII

Example 65. A battery of emf 10 V is connected to Example 67. In the circuit shown in Fig. 3.50, =4 0,
resistances as shown in Fig. 3.47. Find the potential R2 = -^3 = 15 O, =30 O S = 10 V. Work out the ecjui-
difference between the points A and B. valent resistance of the circuit and the current in each resistor.
1 LI 3n (CBSED2011I
/I
AAA/- AAV

c D
3Q in
B 3
VAA AAV

10 V

Fig. 3.47 Solution. The resistances R^ and are in parallel.


4x4
Solution. Total resistance, R = = 20 Their equivalent resistance R' is given by
4 + 4
J_ 1 1 L=Jl a A-A-i
V 10 V
R' R, 1?3 R, 15 15 30 "30 "6
Current, I = — = 5 A
r" 20
or R'=60
As each of the two parallel branches has same Tlie resistance Rj is in series with R'. Hence total
resistance (4 0), so the current of 5 A is divided resistance of the ciraiit is
equally through them. R= R,1 + R' =4+6=100
Current through each branch = 5/2 =2.5 A
Now =2.5xl=2.5 V The current is the current sent by the cell 6 in the
whole circuit.
and
-Vg=2.5x3=7.5 V € 10
— =1 A
R 10
= 7.5-2.5 = 5.0 V.
Potential drop between A and B,
Example 66. Find the equivalent resistance between points V= fR' =lx6=6V
A and B of the circuit shozon in Fig. 3.48. (irr 97\
This is Ihe potential drop across each of the resis
tances R^, R3 and R^ in parallel. Therefore, currents
2R 2R R
through these resistances are
AO- AAV VW -VW ■OB
y y
C D
h ~ — = 0.4 A ; h- = — = 0.4 A
A R3 15

Fig. 3.48 y
and = — =0.2 A.
R4 30
Solution. Obviously, the points A and D are
equipotential points. Also, the points B and C are equal Example 68. Find the equivalent resistance between the
potential points. So the given network of resistances points A and B of the network of resistors shown in Fig. 3.51.
reduces to the equivalent circuit shown in Fig. 3.49. Solution. The resistors ^2 = 30
2R -VA-
VW Rj and R2 are in series. O' ●r‘
"5
Their equivalent resistance
2R V
R3 = 3Q
o- VW ■o
=3+3=60
A D C B ●B
R 5n
AAV The 6 O resistance is in
3n
parallel with Ry so that 30 '3 0
Fig. 3.49 their equivalent resistance
30
The three resistances form a parallel combination. 6x3
=2 0
Their equivalent resistance R^^ is given by 6+3 Fig. 3.51
1 1 1 1 + 2
R 2R
+
2R"^R 2R
=—
R
or R„ =R/2.
eq Now the 2 O resistance is in series with R^. So the
eq total resistance of the upper portion =2 +3 = 50.
CURRENT ELECTRICITY 331'

Similarly, total resistance of the lower portion Solution. As shown in Fig. 3.55, the given hexagon
= 5Q has a line of symmetry Cj C Cj. So all points on this
Now we have three 5f^ resistors connected in line have the same potential i.e., potential at Cj =

parallel between the points A and B. Hence the equi potential at C = potential at Hence the points C
valent resistance R of the entire network is given by and C2 can be made to coincide with each other.
i i=i or R = -n. r/2 Irl2
R 5 3 D -VA- £
C,
Example 69. Find the effective resistance between points A
and B of the network of resistors shown in Fig. 3.52. r r
r r‘

Solution. By r r

symmetry, the potential


o-
A
Vv\ C-AA/V B
.0

drops across GC and GD

w
r. r r.
are equal, so no current r

flows in the arm CD. C,

Similarly, no current V/v- F

Flo
flows in the arm DE. r/2 1xH
Hence the resistances in

ee
the arms CD and DE are Fig. 3.55

ineffective. The given

Fr
After this is done, the circuit splits into identical parts,
circuit reduces to the
joined in series between the points A and B. One such
equivalent circuit shown
part between A and C is shown in Fig. 3.56(a) which, in
in Fig. 3.53.
for
ur
r
turn, is equivalent to the circuit shown in Fig. 3.56(b).
■vw -o- VA r D r/2 4r/3
r r 7VW oc i aa/v oc 1
ks
r-Wv
I
Yo
AO- r r r r
D r
oo

o- VA C o- A/W c
r r A A
T
F6 VW VA
eB

o
£ r 4r/3
AAAv wv C2
Fig. 3.53 G r/2
r

Resistance of arm GH = r+r=2r


(a) (b)
ou
ad

rx r rx r
Fig. 3.56
Resistance of arm IJ = + = r
Y

r+ r r+ r
From Fig. 3.56(b), the equivalent resistance R'
Resistance of arm FK =r + r =2r between the points A and C is given by
nd
Re

nie above three resistances are in parallel between 1 3 1 3 10


or
4r
R’=—=0.4r
points A and B and their equivalent resistance R is R’ 4r r 4r 4r 10
Fi

given by As two identical parts AC and CB are joined in


1-J- 1 -L=- R=0.5r. series, hence the equivalent resistance of the entire
R 2r r 2r r
circuit between points A and B is
Example 70. A regular hexagon with diagonals is made of R = R'+ R' =0.4r+0.4r = 0.8r.
identical wires, each having same resistance r, as shown in
Fig. 3.54. Find the equivalent resistance betiveen the points
A and B. Example 71. Find the
equivalent resistance of
the circuit shown in
Fig. 3.57 between the
points A and B. Each
resistor has a resistance r.

Fig. 3.57
3.38 PHYSICS-XII

4 Solution. By symmetry, potential drops across AC Effective resistance between points P and Q
and AD are equal. So resistance in arm CD is ineffec
tive. The given circuit reduces to the equivalent circuit ~7+{rl2)~3'
shown in Fig. 3.58. Clearly the equivalent resistance R
between points A and B is given by / Toblems For Practice
1 = J_ _L
R 2r 2r r 2r r 1. Given the resistances of 1Q, 2 O and 3 fi. How will
you combine them to get an equivalent resistance of
11 11
or K = - = 0.5 r. (0 — n and (i/) — Q ?
2 3 5 [CBSE F 2015]
r
C ^
AAfV -o—VvV [Ans. (i) parallel combination of IQ and 2Q in
series with 3Q (n) parallel combination of
r D r 2 n and 3 Q in series with 1Q
t Vv\—o \A/v *■
2. Given three resistances of 30 Q each. How can they
r be connected to give a total resistance of (z) 90 Q
o- ■4—o {ii) 10 Q (m)45Q?
/I B
[Ans. (z) in series (ii) in parallel {Hi) two
Fig. 3.58 resistances in parallel and one in series]
Example 72. Find the equivalent resistance of the circuit 3. A 5 Q resistor is connected in series with a parallel
shown in Fig. 3.59 between the points P and Q. Each resistor combination of n resistors of 6 Q each. The equi
has a resistance r. valent resistance is 7 Q. Find n. (Ans. 3)
4. A uniform wire of resistance 2.20 Q has a length of
2 m. Find the length of the similar wire which
connected in parallel with the 2 m long wire, will
give a resistance of 2.0 Q. (Ans. 20 m)
5. A wire of 15 Q resistance is gradually stretched to
double its original length. It is then cut into two
equal parts. These parts are then connected in
parallel across a 3.0 volt battery. Find the current
drawn from the battery. ICBSE OD 091

(Ans.0.2 A)
Fig. 3.59
6. The total resistance of two resistors when connec
Solution. Two resistances along each side of ted in series is 9 Q and when connected in parallel,
triangle are in parallel. their total resistance becomes 2Q. Calculate the
The equivalent resistance of each side value of each resistance. (Ans.6Q, 3Q)
_rx r r
7. The reading of the (ideal) ammeter, in the circuit
r+r 2
shown in Fig. 3.61, equals :
The given network of resistances reduces to the (/) I when key is closed but key is open.
equivalent circuit shown in Fig. 3.60. (»)
^ when both keys and are closed.
Find the expression for the resistance of X in terms
of the resistances of R and S. [CBSE D 16C]

S P
^2
O— Ans.X =
RS

R-S,
R
O'
P
A^/V -©-vw B

A ^
Fig. 3.60
The resistances in arms BA and AC are in series. +

K1 C
Their equivalent resistance =r/2 + r/2=r. This
resistance is in parallel with the resistance r 12 along BC. Fig. 3.61
CURRENT ELECTRICITY 3.39

S. What equal length of an iron wire and a copper- 14. Find the current through the 5Q resistor in the
nickel alloy wire, each of 2 mm diameter, be circuit shown in Fig. 3.65, when the switch S is
connected in parallel to give an equivalent (z) open and (zz) closed.
resistance of 3 ? (Given resistivities of iron and sn ion

copper-nickel alloy wire are 12 cm and 51 pQ cm AMr .A^


respectively) [JEE Main Aug. 21]
(Ans. 97 m) S

9. Two resistors are in the ratio 1 : 4. If these are 3V

connected in parallel, their total resistance becomes h


20 Q. Find the value of each resistance.
Fig. 3.65
(Ans. 25n, 100 Q)
10. Five resistors are connected as shown in Fig. 3.62. [Ans. (z) 0.2 A, (zz) 0.6 A]

w
Find the equivalent resistance between the points B 15. The letter A consists of a uniform wire of resistance
and C (Ans. 70/190)
IQ cm"\ The sides of the letter are 40 cm long and
9n
A B
the crosspiece 10 cm long divides the sides in the

Flo
ratio 1 : 3 from the apex. Find the resistance of the
10 Q letter between the two ends of the legs.
3fi 5Q

ee
(Ans. 66.67 Q)
16. Calculate the equivalent resistance between

Fr
A/W
D 7Cl c points A and B in each of the following networks of
resistors:
Fig. 3.62
for [Ans. (a) 12Q{b) 40/3Q(c)2Q
ur
11. A uniform wire is cut into three parts with their
id) 10/3Q(e) 16Q(/)5Q]
lengths in the ratio 2:3:6. The ends of each of
these three parts are connected across an ideal
ks
battery of 10 V. If a current of 5 A is drawn from the
Yo

battery, find the initial resistance of the wire.


oo

[CBSE OD 201 (Ans. 22 Q)


eB

12. If the reading of the ammeter A^ in Fig. 3.63 is 2.4 A,


what will the ammeters A2 and A^ read ? Neglect the
resistances of the ammeters. (Ans. 1.6 A, 4.0 A)
r

/, 20n _
ou
ad

* \AAr-0-
Y

ion
A/W
nd
Re

k 30 n
—WV
Fi

Fig. 3.63
B
13. The resistance of the rheostat shown in Fig. 3.64 is
30 Q. Neglecting the meter resistance, find the id)
minimum and maximum current through the
ammeter as the resistance of the rheostat is varied. D 4Q c
(Ai^s. 0.18 A, 1.5 A) T AAA—^
6V
ion

t—©■ |-AAAr-% 2n en
7n

ion ion ion


sn
^ ■
o—WV—4-'^Ar-*-AAAr-*—o VvA—
A B A ion B
30 n
1-AAV («■) (/)
20 n

Fig. '3.64 Fig. 3.66


3.40 PHYSICS-Xn

^ 17. Calculate the resistance between points A and B for 19. Find the potential difference between the points A
the following networks : and B for the network shown in Fig. 3.69.
(Ans. 8.0 V)

wv
2A 2,5
2Q
o- ^AA<—●
A B

Fig. 3.69

20. In the circuit diagram shown in Fig. 3.70, a volt


meter reads 30 V when connected across 400 H resis¬
tance. Calculate what the same voltmeter reads when
it is connected across 300 Q resistance. HIT 901

(Ans. 22.5 V)

O
30 V

AA/V—●—VsA,-—'
300 Q 400 n
A
_o O-

60 V
(c)
4H 4Q 4f2 Fig. 3.70
AO—VVV—f—VVV—●—'VVv—●
o
21. Find the potential difference between points A and
3n 3Q 20
B i.e., {V^ - Vg) in the network shown in Fig. 3.71.
Bo- 1 (Ans. 1 V)
22. In the circuit shown in Fig. 3.72, - 4n, 5Q,
id) = lOfl and 6 = 6 V. Work out the equivalent resis -
tance of the circuit and the current in each resistor.
Fig. 3.67
[CBSE D 11] (Ans.6Q, 1 A, 0.4 A, 0.2 A)
18. Find the equivalent resistance of the networks
shown in Fig. 3.68 between the points A and B. /1 A

IAns.(<j)^r W- (c)r]
3 4 %
4-AMr-*-VW
^^3
r

r
I1 B
«-AW-4AVV-0 A . /.
4—AA/V-*'
Fig. 3.72 Fig. 3.73

23. Find the equivalent resistance between points A


and B in Fig. 3.73.
(Ans. 7.5 Q)
24. Letter A as shown in
Fig. 3.74 has resistances on
each side of arm. Calculate the
total resistance between two
ends of the legs.
(Ans. 28.750)
CURRENT ELECTRICITY 3.41

25. Find the resistance between the points (/) A and B 29. In the circuit shown in Fig. 3.79, the battery has an
and (n) A and C of the network shown in Fig. 3.75. emf of 12.0 V and an internal resistance of 5 Jl/ 11. If

(Ans. (0 27.5 n(i7) 30 a] the ammeter reads 2.0 A, what is the value of B ?

10 Q ion ion (Ans.6Q)


/lo—^—*6
il2V

O
t
ion ion I
l-A^V
5R/11

Do—W\> ● Wv ■W^c
ion ion ion
R 2R 3R
Fig. 3.75 pi—VSA»—VVVtAAA/—®Q
26. A combination of four resistances is shown in Fig. 3.76.
Calculate the potential difference between the

w
Fig. 3.79
points P and Q, and the values of currents flowing
in the different resistances. 30. Find the ammeter reading in tine circuit shown in
Fig. 3.80. (Ans. 3 A)
(Ans. 14.4 V, 0.8 A, 1.6 A)

Flo
11 V
!. I in ion 1 n Q
2.4 A —VvV-»— O I— wv

ee
p 4n

Fr
''
5n 7n 6n
9n
AA/v
Q
for 8n
ur
Fig. 3.76
Fig. 3.80
27. In Fig. 3.77, X, Y and Z are ammeters and Y reads
ks
0.5 A. (i) What are the readings in ammeters X and HINTS
Yo

Z ? (ii) What is the total resistance of the circuit ?


oo

1. (i) When parallel combination of IQ and 2Q


[Ans. ()■) 1.5 A, 1.0 A (II) 4 Q] resistors is connected in series with 3 Q resistor,
eB

6n 05 A
the equivalent resistance is
rVvV-©-| in

r@ <-Wv-i R1 + R2
r

3n _
ou

l-Wv— 1x2
+ 3 = 1+3 = ~Q.
ad

''
1+2 3 3
Y

^Ill- (li) When parallel combination of 2Q and 3Q


resistors is connected in series with 1Q resistor,
nd

the equivalent resistance is


Re

Fig. 3.77
2x3
28. Calculate the value of the resistance R in the circuit + R.=
Fi

shown in Fig. 3.78 so that the current in the circuit is R2+R3 2+3

0.2 A. What would be the potential difference 3. Total resistance = 5 + - = 7 Q, so II = 3.


between points A and B ? (CBSE OD 12] n

(Ans.4Q, 1.0 V) 4. Resistance per unit length of the wire


2.2 -1
0.2 A C = llQm
2
6V Let R'be the resistance that should be connected in
ion
n
parallel to resistance, R = 2.20 Q, so that effective
15 n sn resistance, = 2.0 Q. Then
2V-^ 1 1
J. R' = 22Q
— ^ R' R.: 2.2 ^ 2.2 " 22
0.2 A R A D
22
Length of the wire needed = 1.1
= 20 m.
Fig. 3.78
3.42 PHYSICS-XII

5. When the wire of 15 Q resistance is stretdned to / = 3nr^x!V!l£l


double its original length, its resistance becomes P1P2

_ 3x22x(lQ-^)^ _
s
R' = }?R={lf x\5 = eOQ. 63x10
-16
m = 97 m
7 12x51x10
Resistance of each half part = 60/2 = 30 Q
9. Let the two resistances be R and 4 R. Tlnen
When the two parts are connected in parallel, their Rx4R
30x30 R = 20a or -R = 20Q
equivalent resistance = = 15n
R+ 4R 5
30+30

Current drawn from 3.0 V battery. R = 25ii and 4 R =1000.


V 3.0
f = — = 0.2 A. 10. Resistance in branch/IDC
R 15
Rj =3+7=100
6.
R^ + Ry=9a ...(1) This resistance is in parallel with the 10 O resistance
in branch AC.
Ml = 2
R, + Rj Their total resistance is

10x10
or
R,R2-2(Rj + R2) = 2x9= 18 R. = :i- = 50
10+ 10
.-. R, R,=
This 50 resi.stance is in series with the 90 resis
tance in branch AB.
= pl-72=3Q ...(2)

On solving (1) and (2), Their equivalent resistance is


R^ = 5+9=140
R|=60, R2=30
This 140 resistance is in parallel with the 50
7. When Rj is closed and K2 is open, the airrcmt resistance in branch BC. Hence the equivalent
drawn from the battery is resistance between B and C is
6
/ =
R = 14 x5 _ 70 O.
R+X
T4+"5'19
As Sand X are in parallel, total resistance is 11. Let the resistances of the three parts be 2R, 3R and
R’= R +
SX 6R respectively.
S+X
K _ lOV = 20
When both and Ko are closed, the current drawn I

from the battery is Ai "I 1 1


L-A.-1
Also, — = —- +

6(S+X) 2R 3R 6R~ 6R~ R


r=^ 6
SX
R' R(S+X)+SX
R + R =R=20
/’
S+X
Initial resistance of the wire.
The reading of the ammeter will be
R'= 2R + 3R +6R= 11x2 = 220.
1 X
r = -^x 6(S+X)
2 S+X S+X R(S+X)+SX 12. P.D. across 20 O = P.D. across 30 O

or
€s or /, X 20 = /^ X 30
2(R+X) RS+RX + SX 20 20
or L1 =—x2.4 = 1.6A
or 2RS+2SX = RS+ RX + SX 30 30

and
or RS= RX-SX /= /, + /2=2.4+ 1.6 = 4.0 A
RS 13. Equivalent resistance of the 50 and 200 resis
Hence, X =
R-S 5x20
tances connected in parallel = = 40. This
._Mi 5+20
8.R^, /^ + R2
= 30
re.sistance is connected in series with the rheostat

f^xl = 30 whose minimum and maximum resistances are OO


Pi +P2 A and 300.
CURRENT ELECTRICITY 3A3

When the rheostat is adjusted at the minimum (ii) All the three resistances are connected in ‘
resistance of OH, current will be maximum. parallel between points A and B.
6V
I max = 1.5 A 1 _ J_ J_ J 3
— or R =
10
n.
4n R" 10 ^10 "^10 10 3

When the rheostat is adjusted at the maximum (e) The given netw’ork is equivalent to the net
resistance of 30 Q, current will be minimum. work shown in Fig. 3.82.
6V 10x15
U
mm
= 0.18 A .-. R = 10 + =i6n.
(4+ 30) Q 10+ 15

14. (j) When switch S is open, resistances of 5Q and 10 f2

10 Q are in series.
3V 10 10 Q 5Q
Current, I = = 0.2 A
(5+ 10) n
o- ●-WV -VA VW

w
A B

(ii) When switch 5 is closed, no current flows


Fig. 3.82
through 10 £1 resistance.
3V (/) Resistance in branch ADC=2+4=6£1. This

Flo
.'. Current, / = = 0.6 A
5Q resistance is in parallel with 6Q resistance in
arm AC.

ee
15. Refer to Fig. 3.81. Clearly Their equivalent resistance
BC=CD= SD=10cm

Fr
6x6
= 3Q
AB = DE - 30 cm
6+6
R2 == R3 = R5 = ion The series combination of this 3Q resistance
for
ur
and
R^ = =30a and 7£T resistance in arm BC is in parallel with
ion resistance in arm AB.

10x10
ks
R = = sn.
Yo
10+ 10
oo

17. The corresponding equivalent circuit diagrams are


given below :
eB

AAV

2n 2n
r

—WV—WV—'
ou
ad

AO-
2Q 2Q
Y

-VA-AVv-h
in (a)
AW
nd
Re

4n
Series combination of R2 and R3 is in parallel with
Fi

WV
Rg. Their equivalent resistance 4f2 4n

AW AAA—i
^ (10. lOjxM ^200 ^20^^6.670
(10+ 10)+ 10 30 3 o-
A
4Q

AW
n 4Q
WV
B

This resistance is in series with Rj and R^. So the net


4£1
resistance is ib)
AW
R= 30+ 6.67+ 30 = 66.67 n.
R
8x8
16. (a) R = 8 + = i2n. AO AW ■OB
8+8
R
10x5 40
AAV
(b) R = 5 + + 5 = n.
10+5 3
R
(C)
(3 + 3) 3 AW
(c)R = = 2n.
(3+3)+ 3
Fig. 3.83
3A4 PHYSICS-XII

18, (fi) The equivalent network, for 3.68(i?) is slnown in 21. Current through each branch = 2/ 2 = 1 A
Fig. 3.84.
n r 4
R = r+ - =-r. V„-V^^ = 1x3 = 3V
3 3
Vs)-{Vc-V^) = 3-2 = lY.
r

r -vw 22. 1 1 + 1 1 1 i -I
A^V r R
/IB R, ^3 ^4 5 5 10 ” 10 ” 2
r r Vs^
o—wv AAV -o o- r ■o
A B /\ B
AW
R= l^ + R^p=4+2 = 6n
Y

AW r

AW 6 6V
1 = 1 A
1

Fig. 3.84 Fig. 3.85

(B) The equivalent network for 3.68(f)) is shown in


AB “ ^AB =lx2=2V

2
Fig. 3.85. - A = 0.4 A and — = 0.2 A.
10
1 1 1 1 1 4 r
— = -H i !-- = or R = - .
R r r r r r 4 23. The equivalent circuit is shown in Fig. 3.87. The
effective resistance between points C and D
(c) The current divides symmetrically in the two
3x6 .
upper and the two lower resistances. So the - + 8 = lOQ
resistances in the vertical arm are ineffective. The 3+6
3n
given network reduces
I—WV—VA—1 AW-i
to the equivalent net r r
C D
o ■o
work shown in o- ■o 6n
A 8fl B
A B
Fig. 3.86. r r AW
I—VSA—W\—*
2rx2r 30 a
R = = r.
2r+ 2r Fig. 3.86 AW

3x3 Fig. 3.87


19. R = + 2.5 = 1.5+2.5 = 4.00
3+3 Now the ion and 300 resistances are in parallel.
V = Rf = 4.0 X 2 = 8.0 V, The equivalent resistance between points A and B
10x30
= 7.5 0.
20. P.D. across 4000 resistance = 30 V 10+30
P.D. across 3000 resistance = 60 - 30 = 30 V
24. Proceed as in Example 51 on page 3.32.
This shows that potential drop is same across both 25. (0 The equivalent circuit is shown in Fig. 3.88.
resistances.

Let R bo the resistance of the voltmeter. Then ion £ ion H 10^


e—VW ● 9—AAV—9— ■OB
equivalent resistance of R and 4000 connected in
parallel should also be 3000.
10 n ion
Rx400
= 300 or R= 12000
R+ 400
o— i
When the voltmeter is connected across the 3000 f ion G
resistance, their equivalent resistance is given by
Fig. 3.88
_ 1200 X 300 = 2400 Resistance of the arm EFGH = 10 + 10 + 10 = 300
^ 1200 + 300
This resistance is parallel to the lOO resistance of
Total resistance in the circuit = 240 + 400 = 640£J arm EH.
60
Current in the circuit, I = Equivalent resistance between points Eand H
640 32
10x30
= 7.50
Reading of the voltmeter “10+30
= /R' = - X 240 = 22.5 V. Hence total resistance between points A and B
32
= 10+7.5+ 10 = 27.50
CURRENT ELECTRICITY 3.45

(7/) The equivalent circuit is shown in Fig. 3.89. Total resistance of the circuit = eR + 5_R = R
11 11
ion £ ion H
o— —9 VA—9 Resistance,
n ^ 12 = 6n.

ion 10 n 30. The resistances of (5 + 7) = 12Q, 6£T and 8il are in


parallel between points P and Q. Their equivalent
o—"WV—o—^—o resistance R' is given by
p ion G ion ^ J _J_ 2 1^3 or R' = -a
R' ~ 12 6 8 8 3
Fig. 3.89
R' is in series with IQ resistance.
Resistance of arm EHG =10+10=200 8 11
.-. Total resistance = - + 1 = O
Resistance of arm £FG = 10+ 10 = 200 3 3

These two 20 O resistances are in parallel. Current, / = — ^ = JJ_ = 3 A.

w
R 11/3
Effective resistance between points E and G
20x20
= 100 3.19 INTERNAL RESISTANCE OF A CELL
20+ 20

Flo
34. Wliat is internal resistance of a cell ? On what
Hence total resistance between points A and C
= 10+10+ 10 = 30 0.
factors does it depend ?
Internal resistance. When the terminals of a cell are

ee
26. The resistances of 40, lOO and 40 are in series.
connected by a wire, an electric current flows in the

Fr
Their equivalent resistance =180. This is in parallel wire from positive terminal of the cell towards the
with 90 resistance.
negative terminal. But inside the electrolyte of the cell,
Equivalent resistance between P and Q, the positive ions flow from the lower to the higher
for
ur
R = potential (or negative ions from the higher to the lower
18+9 potential) against the background of other ions and
P.D. between P and Q = /R = 2.4 x 6 = 14.4 V neutral atoms of the electrolyte. So the electrolyte offers
ks
some resistance to the flow of current inside the cell.
- ^ _ 14.4
Yo

= 1.6 A
- R~~T~ The resistance offered by the electrolyte of a cell to the
oo

and L1 =2.4-1.6 = 0.8 A. flow of current between its electrodes is called internal
eB

resistance of the cell.


27. P.D. across 60 = P.D. across 30
Hie internal resistance of a cell depends on following
6 X 0.5 = 3 X /, factors :
r

Current through Z, 1. Nature of the electrolyte.


ou
ad

/2=1.0A 2. It is directly proportional to the concentration of


Current through X = 0.5 + 1.0 =1.5 A
Y

the electrolyte.
6x3
Total resistance = + 2 = 40. 3. It is directly proportional to the distance
6+3
between the two electrodes.
nd
Re

28. 1 J_ 1 _ 6 _ 1 4, It varies inversely as the common area of the


15 "^30 "^5+5 ”30 5 electrodes immersed in the electrolyte.
Fi

R
BA

By Ohm's law, 5. It increases with the decrease in temperature of


6-2 the electrolyte.
0.2A = i_A
R+10+5 R+15 The internal resistance of a freslily prepared cell is
or R+ 15 = — = 20 or R = 50
usually low but its value increases as we draw more
0.2 and more current from it.

= 0.2 X R^g = 0.2 X 5 = l.O V. 3.20 RELATION BETWEEN INTERNAL


29. The resistances R, 2 R and 3 R are in parallel RESISTANCE, EMF AND TERMINAL
between the points P and Q. Their equivalent POTENTIAL DIFFERENCE OF A CELL
resistance R' is given by
J__J. 1 1 11 6R 35. Define terminal potential difference of a cell. Derive
R' =
R'~ R^ 2 R
+
3R 6R
or
11
a relation between the internal resistance, emfand terminal
potential difference of a cell. Draw (/) 6 vs. R (//) V vs. R
Now 6 R /11 and 5 R / 11 are in series. (Hi) V vs. I graphs for a cell and explain their significance.
-3A6 PHYSICS-Xli

Terminal potential difference. The potential drop Thus the potential difference across the terminals of the
across the terminals of a cell when a current is being drawn cell is equal to its ernf when no current is being drawn from
from it is called its terminal potential difference {V). the cell

Relation between r, 6 and V. Consider a cell of emf S (ii) A real cell has always some internal resistance r,
and internal resistance r connected to an external so when current is being drawn from cell, we have
resistance R, as shown in Fig. 3.90. Suppose a constant V<&
current I flows through this circuit.
Thus the potential difference across the terminals of the
^VW cell in a closed circuit is always less than its emf.
R
! Characteristic curves for a cell. When a cell of emf S
^1 + - B and internal resistance r is connected across a variable

6
load resistance R,its functioning can be represented by
Cell
the following three graphs :
(0 i versus R graph. The emf of a cell is equal to the
Fig. 3.90 Cell of emf S and internal resistance r. terminal p.d. of the cell when no current is drawn from
By definition of emf, it. Hence emf 6 is independent of R and 6-R graph is a
straight line, as shown in Fig. 3.91(c)
£ = Work done by the cell in carrying a unit charge
along the closed circuit
= Work done in carrying a unit charge from .A to B
against external resistance R iso

+ Work done in carrying a unit charge from B to


A against internal resistance r r = R R-^
or ^=V+V'
Fig. 3.91 (fl) 8 vs. R graph for a cell. (6) V vs. R graph for a cell.
By Ohm's law.
V = IR and V = Ir (ii) V versus R graph. In a closed circuit, the
terminal p.d of the cell is
g=/R+/r=/(R + r)
Hence the current in the circuit is V=^IR =
( € R =
£
R + r
£
/ = R
R+r As R increases, V also increases.
Thus to determine the current in the circuit, the When R -> 0, V = 0
internal resistance r combines in series with external When R = r, v = tn
resistance R.
When R CO,
The terminal p.d. of the cell that sends current 1
Hence V-R graph is as shown in Fig. 3.91(b).
through the external resistance R is given by
(ill) V versus I graph. As V =^-Ir
6R
V=IR = => l^ = -ri + £ o y = mx + c
R + r
Hence, the graph between V and i is a straight line
Also V = E-V'=^-Ir with a -ve slope, as shown in Fig. 3.91(c)
For point A, 7 = 0
or terminal p.d. = emf - potential drop across the Hence,
internal resistance

Again, from the above equation, we get


= intercept on the y-axis
g-y £-y ^-V^ _ For point B, V =0
r = R.
I V/R V
g = 7„r
B
Special Cases g
Hence, r = — Fig. 3.91 (c) V vs. / graph
(i) When cell is on open circuit, i.e., 7 =0, we have
h for a cell.
V =£
open
= negative of the slope of V-I graph.
CURRENT ELECTRICITY

Examples based on The p.d. across the voltmeter is


EMF, Internal Resistance, Terminal Potential V= IR

Difference and Grouping of Cells = 2 X 10"^ X 998 =1.996 V

Formulae Used
The sa' Ae will be the p.d. across the terminals of the
cell. The voltmeter used to measure the emf of the cell
W
1. EMF of a cell, € = — will read 1.996 volt. Hence the percentage error is
<?
E-V 2-1.996
2. For a cell of interna! resistance r, the emf is xl00 = X 100=0.2%.
6 2
t = V + lr= I{R+ r)
tR Example 75. the 1<V>
3. Terminal p.d. of a cell, V = IR = R+r circuit shown in Fig. 3.93,
the voltmeter rends 1.5 V, 1
4. Terminal p.d. when a current is being drawn from

w
when the ke\/ is open. When
the cell.
the key is closed, the R
V =^-Ir voltmeter reads 1.35 V and «
K
a

ammeter reads 1.5 A Find

Flo
5. Terminal p.d. when the cell is being charged,
V =6+ Ir the emf and the internal Fig. 3.93
6-V resistance of the ceil.

ee
6. Internal resistance of a cell, r=R
V Solution. When the key is open, the voltmeter

Fr
reads almost the emf of the cell,
Units Used g = 1.5 V
EMF E and terminal p.d. V are in volt (V), internal When the key is closed, voltmeter reads the
for
ur
resistance r and external resistance R in Q and
terminal potential difference V.
current I in ampere (A).
V = 1.35V, /=1.5A, r=?
ks
Example 73. For driving a current of 3 AfdrSmwutes in E-V 1.5-1.35
= o.in.
Yo
r =
an electric circuit, 900 / ofivork is to be done. Find the emf of / 1.5
oo

the source in the circuit.


Example 76. A battery of emf 12 V and internal resistance
eB

Solution. The amount of charge that flows through 4H is connected to an external resistance R. If the current in
the circuit in 5 minutes is
the resista?ice is 0.5 A, calculate the value of (a) R,and (b) the
q = I x f=3x5x60= 900 C terminal ifoltage of the battery. ICBSE F 201
r

As emf is the work done in flowing a unit charge in 6


ou
ad

Solution, (rt) As / =
the closed circuit, therefore r+ R
Y

W
€=— _ 900 J = 1.0 V. .-. 0.5 =
12
or i?=24-4 = 20n.
q ‘900 C 4+ R
nd
Re

Example 74. A voltmeter of resistance 998 Q is connected (b) V=6-/r = 12-0.5x 4 = 10 V.


across a cell of emf 2 V and internal resistance! H. Find the Example 77. The reading on a
+
Fi

<v>
p.d. across the voltmeter, that across the terminals of the cell high resistance voltmeter lohen a
and percentage error in the reading of the voltmeter. cell is connected across it is
[CBSE OD 19] 2.2 V. When the terminals of the +

Solution. Here 8=2 V, r=2 Cl cell are also connected to a

Resistance of voltmeter, Cell


resistance of 511, the voltmeter
2 V rending drops to 1.8 V. Fitid the R = 5Q K
R=998n
l-A/W internal resistance of the cell.
Current in the circuit is 2D [CBSE OD 10] Fig. 3.94
8 /w
1 = Voltmeter Solution. Here8=2.2 V, R = 5n, V =l.8W
R + r
Internal resistance,
2 V

(998+2)Q R=998n _ft-V ^22 -18^


r= R 5 a = 1.1 a
= 2xlO"^A Fig. 3.92 I ^ 1.8
3.48 PHYSICS-XI!

V
Example 78. A battery of (b) After long use, the internal resistance of the storage
etnf 12 V and internal <2>
battery increases to 5000. What maximum current can be
resistance 2Q is connected
to a 40 resistor as shown in
12 V,2a drawn from the battery ? Assume the emfof the battery to
remain unchanged,
Fig. 3.95. Show that a volt
(c) If the discharged battery is charged by an external emf
meter when placed across the 4Q A
source, is the terminal voltage of the battery during charging
cell and across the resistor, W.—0 greater or less than its emf 12 V ? [NCERT]
in turn, gives the same
reading. [CBSE OD 16) Fig. 3.95 Solution. (<7) HereS=12 V, 1=90 A, r=5.0xl0"^0

Solution. R,^ =R + r = 4 + 2 =60 Terminal voltage, V = 8 - /r = 12 - 4.5 = 7.5 V.


g 12 V (b) The maximum current can be drawn from a
/ = = 2A
R 60 battery by shorting it.
eq
Then V =0 and /max 8 _ 12 A = 24 mA.
Terminal p.d. across the cell, r ”500
V = €-/r = 12 -2x2=8V
Clearly, the battery is useless for starting the car
P.D. across 40 resistance =40x2A = 8V and must be charged again.
Hence, the voltmeter gives the same reading in the (c) During charging of the accumulator, the current
two cases.
inside the cells (of the accumulator) is opposite to what
Example 79. A battery ofe.m..f and internal resistance it is when the accumulator discharges. That is, during
y, gives a current of 0.5 A with an external resistor of charging, current flows from the + ve to - ve terminal
12 ohm and a current of 0.25 A with an external resistor Consequently, during charging
of 25 ohm. Calculate (i) internal resistance of the cell and V = 8 + fr
(ii) emf of the cell. [CBSE D 02 ; OD 13C) Hence V must be greater than 12 V during charging.
Solution. EMF of the cell, 8 = / (R + r) Example 82. A battery of emf 12.0 V and internal resis
In first case, 8 = 0.5 (12 + r) tance 0.5 O is to be charged by a battery charger which
in second case. 8=0.25 (25+ r) suppliesllO V d.c. Hoxp much resistance must be connected
in series with the battery to limit the charging current to
0.5(12 + r) = 0.25(25 + r) 5.0 A ? What will be the p.d. across the terminals of the
On solving, we get r = 1D battery during charging ?
Hence 8=0.5(12 +1) = 6.5 V. Solution. For charging, the positive terminal of the
Example 80. A battery of emf 3 volt and internal resis- charger is connected to the positive terminal of the battery.
tance r is connected in series with a resistor of 55 Q through .-. Net emf 8'=110-12.0 =98 V
an ammeter of resistance 1Q. The ammeter reads 50 inA.
Draw the circuit diagram and calculate the value of r. Battery charger
[CBSE D 95 ; Haryana 02] flip V I
Solution. Total resistance
5.0 Av
= 55 + 1 + rD =56 + rD 12.0 V 0.5 fi R
Current +
-II HAW-l-AVv
= 50 mA 55 Q
MAr
= 50x 10“^ A R
Fig. 3.97
emf =3 V ©in
Resistance =
emf 3 V
If R is the series resistor, then the charging current
l-AW will be
Current
3 8 98
56 + r = 1 = A
r=60 R + r
50x10"^ R+0.5
Fig. 3.96
r = 60 - 56 = 4n. Given / = 5.0 A, therefore
98
Example 81. (a) A car has a fresh storage battery of emf R+0.5
= 5.0 or R=19.1Q

12 V and internal resistance 5.0 x 10”^ Q. If the starter


motor draws a current of 90 A what is the terminal voltage The terminal p.d. of the battery during charging is
of the battery when the starter is on ? V=E+ Ir = 12.0 + 5.0 X 0.5 = 14.5 V
CURRENT ELECTRICITY

If the series resistor R were not included in the (ii) maximum current obtained from the cell, and
charging circuit, the charging current would be 98/0.5 (Hi) internal resistance of the cell. [CBSEDllC]
= 196 A, which is dangerously high. Solution. (0 The potential difference corresponding
Example 83. A cell o/cm/1.5 V and internal resistance to zero current equals the emf of the cell.
0.5 n is connected to a (non-linear) conductor whose V-I
.-. EMF of the cell, € = 1.4 V.
graph is shown in Fig. 3.98(a). Obtain graphically the
current draum from the cell and its terminal voltage. (ii) Maximum current is drawn from the cell when
the terminal potential difference is zero.
1 l‘‘
I = 0.28 A.
A max

3 € 1.4 V
= 5Q.
(hi) Internal resistance, r = I 0.28 A
max
2

Example 85. Find the current drawn from a cell of emf 1 V

w
1 and internal resistance 2/3Q connected to the network
B given below. [CBSE D OlCl
1 X
y A B
1.5

Flo
(b) in

in in
Fig. 3.98

ee
1 n in

Solution. Here 8 = 1.5 V, r = 0.5 Q 1 n

Fr
AAAr
Terminal voltage of the cell, V = 6 - /r D c

For different currents, terminal voltages can be


1
for
ur
determined as follows :

f=0, V = 1.5-0=1.5 V

J = 1 A, V = 1.5-lx0.5=1.0V Fig. 3.100 (a)


ks
1=2 A, V = 1.5-2 X 0.5 =0.5 V Solution. The equivalent circuit is shown below.
Yo
oo

in
I =3 A, V = 1.5-3x0.5=0 A B
AAAr
The V-I graph for the cell is a straight line AB, as
eB

1 n in

shown in Fig. 3.98(fe). This straight line graph intersects VyV “Wv
P Q
the given non-linear V-I graph at current =1A and at 1 n in
voltage = 1 V.
r

-wv
ou
ad

.'. Current drawn from the cell =1 A in

Terminal voltage of the cell = 1 V. AA/V


Y

D c

Example 84. Potential differences across terminals of a cell IV

were measured (in volt) against differeyit currents (in ampere) I AAV
nd
Re

flowing through the cell. A graph was drawn which was a


straight line ABC, as shown in Fig. 3.99. Determine from Fig. 3.100 (b)
Fi

the graph Resistance in arm AB = lfl


(i) emf of the cell
Resistance in arm PQ =
1x1
=1 + 1=1Q.
2.0 - 1+1 1+1 2 2

1.6 - Resistance in arm DC = 1 ii


t
These three resistances are connected in parallel.
^ 1.2- --
> Their equivalent resistance R is given by
1
0.8 - 1=1 1 1=1 R = -n
r“i^i’^i“i
or
3
0.4 -
Current drawn from the cell.

€ IV
o 1 = = 1 A.

/ (ampere) R + r ri^2' n
3 3,
Fig. 3.99
s ■3 JO

Example 86. A uniform wire of resistance 12 is cut into


three pieces in the ratio 1:2 :3 and the three pieces are
connected to form a triangle. A cell ofemfS V and internal
PHYSICS-XIl

5. The potential difference between the terminals of a


battery of emf 6.0 V and internal resistance IQ
drops to 5.8 V when connected across an external ●
resistance IQ is connected across the highest of the three resistor. Find the resistance of the external resistor.
resistors. Calcidate the current through each part of the (Ans. 29 n)
circuit. [CBSE OD 13C)
6. The potential difference between the terminals of a
Solution. In Fig. 3.101, R AB = 2 0, R BC = 40 and 6.0 V battery is 7.2 V when it is being charged by a
R
AC
= 60. B current of 2.0 A. What is the internal resistance of
The series combi 2Q
the battery ? (Ans. 0.60)
4Q

nation of 2 O and 4 O (of 7. A battery of emf 2 V and internal resistance 0.5 O is


connected across a resistance of 9.5 O. How many
equivalent resistance 6 O) 6Q
is in parallel with the 6 O A vw *■ C
electrons pass through a cross-section of the
resistance. The equi
resistance in 1 second ? (Ans. 1.25 xltf®)
I
valent resistance is 8 V
"I
8. A cell of emf 6 and internal resistance r is connected
r=in
6x6
across a variable load resistor 11 It is found that when
I—vw
R = =30 R = 40, the current is lA and when R is increased
6 +6
Fig. 3.101 to 9 O, the current reduces to 0.5 A. Find the values
6 8 V of the emf S and internal resistance r. [CBSE D 15]
Current, 1= =2 A
R+r (3 + 1)0 (Ans. 5 V, 10 )
The resistances R and R 9. The emf of a battery is 4.0 V and its internal resis
BAC BC of the par9llel
branches are equal. tance is 1.5 O. Its potential difference is measured
by a voltmeter of resistance 1000 O. Calculate the
1
ABC - Uc = 1 A.
percentage error in the reading of emf shown by
voltmeter. (Ans. 0.15 %)
5^. roblems For Practice 10. The emf of a battery is 6 V and its internal resistance
is 0.6 O. A wire of resistance 2.40 is connected to
1. A cell of emf 4 V and internal resistance lO is the two ends of the battery, calculate (fl) current in
connected to a d.c. source of 10 V through a resistor the circuit and (b) the potential difference between
of 5 O. Calculate the terminal voltage across the cell the two terminals of the battery in closed circuit.
during charging. [CBSE OD 17C) (Ans. 5 V) (Ans. 2 A, 4.8 V)
2. The potential difference across a cell is 1.8 V when a 11. A cell of emf 2 V and internal resistance 0.10 is
current of 0.5 A is drawn from it. The p.d. falls to 1.6 V connected to a 3.9 O external resistance. What will
when a current of 1.0 A is drawn. Find tine emf and tine be the p.d. across the terminals of the cell ?
internal resistance of the cell. [CBSE D 01 Cl (Ans. 1.95 V)
(Ans. 2.0 V, 0.4 n)
3. A 9 V battery is connected in series with a resistor. 12. The potential difference across the terminals of a
The terminal voltage is found to be 8 V. Current battery is 8.5 V, when a current of 3 A flows
through the circuit is measured as 5 A. What is the through it from its negative terminal to the positive
internal resistance of the battery ? [CBSE SP18] terminal. When a current of 2 A flows through it in
the opposite directipn, the terminal potential
(Ans. 0.2 n) difference is 11V. Find the internal resistance of the
4. In the circuit shovm in Fig. 3.102, the resistance of battery and its emf. (Ans. 0.5 a 10 V)
the ammeter A is negligible and that of the 13. In the given circuit, the internal resistance of the
voltmeter V is very high. When the switch S is open, 18 V cell is negligible. If 1^ = 400 Q, = lOOfi and
the reading of voltmeter is 1.53 V. On closing the iR^ = 500 Q and
switch S, the reading of ammeter is 1.00 A and that the reading of an ^3 R,
of the voltmeter drops to ideal voltmeter >VV\—^
1.03 V. Calculate : (i) emf I
across R_^ is 5 V, R1
^2
of the cell (//) value of R then find the vw
(m7) internal resistance of value of R,.
the cell. [JEE Main Jan 19]
I
[Ans. (/) 1.53 V VvVj:(A)J (Ans. 300 O) 18 V
(»■) 1.03n(m) 0.50 Q]
Fig. 3.102 Fig. 3.103
CURRENT ELECTRICITY 3.51

HINTS 13. /j 100 n 500 0


400 O ;
Net emf {10-4)V VA-H h; ^3 ^4
1. Charging current, / = = 1A
R+r (5+i)n R1
>

y=S+/r = 4+lxl = 5V.


2. EMF of a cell, 6 = V + fr
When/ = 0.5 A, 1/ = 1.8 V .-. £ = 18+0.5r ...(1) 18 V

When/ = 1.0 A, V = 1.6 V .'. S = 1.6+1.0r ...(2) Fig. 3.104


Solving (1) and (2), we get V4 = 5V
6 = 2.0V and r = 0.4n.
L =-^= — = 0.01 A
8 -V 9-8 1 ^ 500
3. r= = -a = o.2 n.

w
I 5 5
^3 = 'i Kg =0.01 X 100=1 V
4. (/) & = P.D. measured in open circuit =1.53 V.
Vz = ^3 + n=6V
V _ 1.03

Flo
(n) R = = 1.030. K1 =18-6=12V
/ ” 1.00
V 12
1 _
1.53-1.OS'! = 0.03 A

ee
I =
(m) r = R = 1.03 = 0.50 a. 400
V 1.03

Fr
h = i-f^=0.02A
rV 1 X 5.8 _ 5.8
5. R =
8-V“ 6.0-18” 02
= 29D.
R-=^ = —=300n.
^ L2 0.02
for
ur
6. During charging, V = 8 + Ir
7.2 = 6.0 + 2 xr or r = 0.6Q. 3.21 COMBINATIONS OF CELLS IN SERIES
ks
8 2 AND PARALLEL
7. 1 = = 0.2 A
Yo

R+r 9.5 + 0.5 36. Why do we often use a combination of cells ?


oo

It 0.2x1 Combination of cells. A single cell provides a feeble


= 1.25x10^®.
eB

n = —
e 1.6x10
-19 current. In order to get a higher current in a circuit, we
often use a combination of cells, two or more cells. A
8. Proceed as in Example 79 on page 3.48. combination of cells is called a battery. The battery cells
r

9. Proceed as in Example 74 on page 3.47. are used in torches, transistor sets, automobiles, etc.
ou
ad

8 6 Cells can be joined in series, parallel or in a mixed way.


in. Current, / = = 2 A.
37. What do you mean by a series combination of cells ?
Y

R+r 2.4 + 0.6

P.D. between the two terminals of the battery is Two cells of different emfs and internal resis- tnnces are
connected in series. Find expressions for the equivalent
nd

V = /R = 2 X 2.4 V = 4.8 V.
Re

emf and equivalent internal resistance of the combination.


8 2
Cells in series. When the negative terminal of one cell is
Fi

11. / = = 0.5 A
R+r 3.9+ 0.1
connected to the positive terminal of the other cell and so on,
V =/R = 0.5 X 3.9 = 1.95 V. the cells are said to be connected in series.

12. When current flows through the cell from its As shown in Fig. 3.105, suppose two cells of emfs 8.j
negative to positive terminal, and 82 and internal resistances r^ and are connected
V = 8 - Jr in series between points A and C. Let I be the current
or 8.5=8-3r ...(/) flowing through the series combination.
When current flows through the cell from its positive I
81 I ^2 I I 8., I
to negative terminal, p.d. across r adds to its emf. So o- I ■o—<- -0 =0- I— o
A B C A C
V = 8+ Jr
-VvV -Wv- ^vW
or ll=8+2r ...(H) ''2 ’'eq

On solving equations (i) and (ii), we get Fig. 3.105 A series combination of two cells is equivalent to a
r = 0.Sn and 8=10V.
single cell of emf 6^^ and internal resistance r^^
3.52 PHYSICS-XIl

Let V^, Vg and be the potentials at points A, B €1


and C respectively. The potential differences across the /1 /I
terminals of the two cells will be 6 *^9
I “VW I I I
V
o—< «—0 = 0—^1—II ■o
AB ~^A ~ A B 1
r,
82 c A
h h
and
BC = V,-Vc=t2-Ir,
Thus the potential difference between the terminals ^VVS/_
A and C of the series combination is ^2

Fig. 3.106 A parallel combination of two cells is equivalent to

= (6j -/r^) + {g2-/r2) a single cell of emf eq and internal resistance r^.
or
,
V = (€ + 6,) - / (r + r ) As the two cells are corrected in parallel between
If we wish to replace the se'ries'combination by a difference
single cell of emf g,, and internal' resistance r,,, then The potential difference between the terminals of
-E Ir
''AC ^eq -eq first cell is
Comparing the last two equations, we get V = W6 -W
1 82
K-q=^i + ^2 and ^cq = ^1+^2 L-V
1
I
1 “
We can extend the above rule to a series combi h
nation of any number of cells :
The potential difference between the terminals of
1. The equivalent emf of a series combination of n
^2 is
cells is equal to the sum of their indiWdual emfs.
+ € II
V=V,
81 ^62 “ ^2 hh
2. The equivalent internal resistance of a series h~
combination of n cells is equal to the sum of
their individual internal resistances.
Hence
/ - /j + /2 - t,-v J2-V
''eq=>i +^2 + '3+ + '■« ^1 ^2
3. Tlie above expression for is valid when the n
cells assist each other i.e., the current leaves each I +
Ey'] -y (1_L + _L1 'l
_2
cell from the positive terminal. However, if one I'i h) hj
cell of emf say, is turned around 'in ^1+^2 -I
opposition' to other cells, then or
nr-
V2 J ¥2
^eq + II ■ 2'1
nr
1'2
or V = -I
38. What do i/ou mean hi/ a parallel combination of rj + r2
cells ? Two cells of different emfs and internal resistances
are connected in parallel with one another. Find the If we wish to replace the parallel combination by a
expressions for the equivalent emf and equivalent internal single cell of emf E^^ and internal resistance , then
resistance of the combination.
Cells in parallel. When the positive terminals of all cells Comparing the last two equations, we get
are connected to one point and all their negative terminals to + 62rj
another point, the cells are said to be connected in parallel. 6..=^ and
cq ^eq
^1+^2
As shown in Fig. 3.106, suppose two cells of emfs
6j and 62 2nd internal resistances q and Tj are connec We can express the above results in a simpler way
ted in parallel between two points. Suppose the as follows :

currents f and I2 from the positive terminals of the E E


ClJ _ 2
two cells flow towards the junction 6j, and current I +

flows out. Since as much charge flows in as flows out, 'eq "1 ^2
we have 1 1 1
and
CURRENT ELECTRICITY 3.53'

For a parallel combination of n cells, we can write 40. Derive the condition for obtaining maximum
£ £ II
current through an external resistance connected to a
cq
parallel combination of cells.
r
r
eq II
Condition for maximum current from a parallel
1 1 1 1
combination of cells. As shown in Fig. 3.108, suppose m
and — = — H + + —. cells each of emf £ and internal resistance r be connec
r
eq 'l '2 II
ted in parallel between points A and R Let R be the
external resistance.
39. Derive the condition for obtaining maximum
current through an external resistance connected across a
series combination of cells.
Condition for maximum current from a series
combination of cells. As shown in Fig. 3.107(fl), suppose

w
n similar cells each of emf £ and internal resistance r be
} I
connected in series. Let R be the external resistance.

n cells

Flo
n- h Fig. 3.108 A parallel combination of m cells.

ee
I I Since all the m internal resistances are connected in
parallel, their equivalent resistance R' is given by

Fr
R
AAAr 1 1 1 1 m
— = - + - + - + m terms = —
R' r r r r

Fig. 3.107 (a) A series combination of n cells. for


ur
or
m
Total emf of n cells in series r

Total resistance in the £


ks
= Sum of emfs of all cells - «£
m

circuit
Yo

Total interna! resistance of n cells in series


oo

= R+R' = R + — I'' .J
= r + r + r + n terms - nr m
R
eB

Total resistance in the circuit = R + nr


As the only effect of joining VvV

The current in the circuit is £ m cells in parallel is to get a


}}
nr Fig. 3.109Equivalent circuit.
single cell of larger size witli
r

Total emf l-W^


the same chemical materials, so
ou

/ =
ad

Total resistance
,J total emf of parallel combination
Y

«£
AAAr
R
= emf due to single cell = £
R + ur .-. Tire current in the circuit is
nd
Re

Fig. 3.107 (b) Equivalent £ m£


Special Cases circuit. 1 =
R + rIm mR + r
Fi

(i) If R » nr, then


«£ Special Cases
/ =
R (i) If R «-^, then m
= n times the current (£ / R) that can
be drawn from one cell. / = = /7i times the current due to a single cell.
r

(ii) If R « nr, then


(ii) l( R»—, then
m
1 =
nr r £
/ = — = the current given by a single cell.
= the current given by a single cell R

Thus, when external resistance is much higher than the Thus, when external resistance is much smaller than the
total internal resistance, the cells should be connected in net internal resistance, the cells should be connected in
series to get maximum current. parallel to get maxhmnn current.
3.54 PHYSICS-XII

41. Derive the condition for obtaining maximum Now


current through an external resistance connected across a
mixed grouping of cells. mR + nr = + {■Jnr'f
Mixed grouping of cells. In this combination, a certain = + {4mf -2 -JmR nr

number of identical cells are joined in series, and all such = {4mR -yfnrf' + 2yjmnRr
rows are then connected in parallel with each other. As the perfect square cannot be negative, so
As shown in Fig. 3.110, suppose n cells, each of emf will be minimum if
t and internal resistance r, are connected in series in i
i.e., 4mR -V«r = 0
each row and m such rows are connected in parallel or mR - nr
across the external resistance R.
nr
or R = ~
« cells m
1 1 h
or External resistance
II h
= Total internal resistance of the cells.
m rows
Thus, in a mixed grouping of cells, the current through
i 1 h the external resistance will be maximum if the external
I
resistance is equal to the total internal resistance of the cells.
II
Examples based on
R
Grouping of Cells
Formulae Used
Fig. 3.110 Mixed grouping of cells.
1. For n cells in series. / =
Total number of cells R+ nr
r»g
nr
= mn m n g
2. For n cells in parallel, / =
Net emf of eacli row of n nR+ r
cells in series = ng I mnB
,.I
As m such rows are 3. For mixed grouping, /=
R mR + nr
connected in parallel, so net A/Vv
emf of the combination = nB where n = no. of cells in series in one row,
Fig. 3.111 Equivalent circuit.
Net internal resistance of m = no. of rows of cells in parallel.
each row of n cells = nr 4. For maximum current, the external resistance
must be equal to the total internal resistance.
As m such rows are connected in parallel, so the nr
total internal resistance r' of the combination is given I.C., — ^R
m
by
1 1 1 1 or nr - mR
m
= — + — + — + m terms =
r' nr nr nr nr Units Used
nr
or
EMF and terminal p.d. are in volt (V), internal
m
resistance (r) and external resistance R in Q,
Total resistance of the circuit current in ampere (A).
=R+/=R+—
Example 87. (a) Three cells of emf 2.0 V, 1.8 V and 1.5 V
m

The current through the external resistance R, are connected in series. Their internal resistances are
Total emf nB 0.05 n, 0.7 Q and 10 respectively. If the battery is connec
Total resistance R + nr/m ted to an external resistor of 40. via a very low resistance
mnB ammeter, what would be the reading in the ammeter ?
mR + nr (b) If the three cells above were joined in parallel, would
they be characterised by a definite emf and internal
Clearly, the current I will be maximum if the resistance (independent of external circuit) ? If not, how will
denominator i.e., (mR + nr) is minimum. you obtain currents in different branches ? [NCERT]
CURRENT ELECTRICITY

Solution, (a) Tlie circuit diagram is shown in , Solution.

Fig. 3.112, 20 0
W/
0.05 0 0.7 0 1 o
l-VW 20 0 10 o
h-WV l-\W
^A^ MAr
2V 1.8 V 1.5 V
1.5 V
/ r 1.5 V
I—V^ I—vw
40 1.5 V r
r
VvV I—WSH

Fig. 3.112
<£>-^
h
1.2 V
As the three cells have been connected in series to
an external resistor of 4 H, therefore Fig. 3.113

w
Total emf = (2.0 + 1.8 + 1.5)V=5.3 V 1.5 3
/ =
Total resistance 20+r
10+

Flo
2
= (0.05 + 0.7 + 1 + 4)0 = 5.75 O
emf V _1.2V Aa

ee
Current, / = Also, / =
resistance i^'lOO 25

Fr
5.3
A = 0.92 A. 3 _ 3
5.75
20+r 25
{b) No, there is no formula for emf and internal
for
ur
20 + /● = 25
resistance of non-similar cells, joined in parallel. For
this situation, we must use Kirchhoff's laws. r = 50.
ks
Example 88. A cell of em/1.1 V ami inteniai resistance Example 90. Four identical cells, each of emf 2 V, are
Yo

03Clisconnectedtoawireofresistance0.5^1.Anothercell yo/ned in parallel providing siipplp of current to external


oo

of the same emf is connected in series but the current in the circuit consisting o/hwl50 resistors joined in parallel,
of the
eB

wire remains the same. Fmd the internal resistance


The terminal voltage of the cells, as read by an ideal
second cell. voltmeter is 1.6 volt. Calculate the internal resistance of each
Solution. In first case : cell. [CBSE D 02]
r

Total emf, 8=1.1 V Solution. As shown in Fig. 3.114, four cell are
ou
ad

connected in parallel to the parallel combination of two


Total resistance. R=0.5+0.5=in
15 Cl resistors.
Y

.●. Current, / 8 _U
=- = 1.1 A
R~ 1
nd
Re

In second case :
Fi

Total emf.
8=1.1 + 1.1=2.2 V
I I

Total resistance.
WV
R=0.5+0.5 + r = (l + r)a 15 n

where r is the internal resistance of the second cell.


2.2
.●. Current, I= = 1.1 or r=in. Fig. 3.114
1+ r
Here 8=2 V, V^ = 1.6V
Example 89. Two identical cells each of emf 2.5 V are
The external resistance provided by two 15 f2
connected in parallel across a parallel combination of two
resistors connected in parallel is
resistors each of resistance 20 H. A voltmeter connected in
the circuit measures 1.2 V. Calculate the internal resistance 15x15
R = = 7.5Q
of each cell. [JEE Main June 22] 15+15
PHYSICS-XII

If r' is the total internal resistance of the four cells This emf sends circuit 1 in the anticlockwise
connected in parallel, then direction.
f^-V (2 -1.6 15
t' = R = 7.5 =—n Total resistance = K + rj + r2 = 5 + 1 + 2= 80
V 1.6 8 Current in the circuit

If r is the internal resistance of each cell, then Net emf


1 1 1 1 1 4 - = 0.25 A.
Total resistance 8
r r r r r r

or r = 4?^ = 4 X — = 7.5 n.
15 (i) Current inside the cell i-2 flows from -ve to +ve
8 terminal, so the terminal p.d. of this cell is
Example 91. In the circuit
E, K ~^b~ ^2 ~
1 = 4.0-0.25x2.0 = 3.5 V.
diagram given in Fig. 3.115,
4 V, 0.5 n 8V, lO
the cells £j and have enifs (ii) Current inside the cell flows from +ve to -ve
4 V and 8 V and internal ^ terminal. Hence the terminal p.d. of this cell is
AAV AMr
resistances 0.5 O and 1.0 O 4.5 n 3Q

respectively. Calculate the


AAV = 2.0+0.25x 1.0=2.25 V.
current in each resistance.
6a
[CBSE D 15C] Example 93. In the two electric circuits shown in
Fig. 3.115
Fig. 3.117, determine the readings of ideal ammeter (A) and
Solution. Effective emf of the circuit the ideal voltmeter (V). [CBSE D 15C]
= 62-^1 =8-4 = 4V +

O
+

Total resistance of the circuit <v>


3x6 I
6V
I I I
= 1 + 0.5 + 4.50 + = 80 I I
6V
I I
3 + 6 I- hAW
I 1
l-VW
.●. Current in the circuit, 1 = 48 = 0.5 A la I I
la
I I I I

(A) 0
Current through 4.5 O resistance = / = 0.5 A I
9V
I I
9V
I
I I I I
P.D. across the parallel combination of 3 O and 6 O H -AAV +
HVW
resistances is I
la
I I
la
I
I I I I
3x6
V = R'I = x0.5=l V
3+6
Fig. 3.117 (a) (b)
IV
Current through 3 O resistance = Aa Solution. In the circuit (a)
30 3
IV Total emf = 15 V, Total resistance =20
Current through 60 resistance .= 1a. 15V
60 6 Current, 1 = = 7.5 A
20
Example 92. In Fig. 3.116, and 62 are respectively As the current / flows from -ve to +ve terminal
2.0 V and 4.0 V and the resistances and R are
respectively 1.0 0,2.0 O and 5.0 O. Calculate the current in inside the cell of 6 V, the terminal p.d. of the cells is
the circuit. Also calculate (i) potential difference between the V = g-Ir = 6-7.5xl=-1.5 V
points b and a, (ii) potential difference between a and c. .●. Reading of ammeter = 7.5 A,
r
€ a
Reading of voltmeter = -1,5 V.
AAVH -o- l-AW In the circuit (b)
'●2
Net emf =9-6 =3V, Total resistance =20
r' .J
3V
Current, I = = 1.5 A
I R 20
AAA
As the current I flows from +ve to -ve terminal
Fig. 3.116
inside the 6 V cell, so the terminal p.d. of the cell is
Solution. As emfs Sj and £2 opposing each V = g+/r = 6 + 1.5xl = 7.5 V
other and 62 > so .'. Reading of ammeter = 1.5 A,
Net emf = 62 -Sj =4-2 =2 V. Reading of voltmeter = 7.5 V.
CURRENT ELECTRICITY 331

Example 94. A network of resistances is connected to a Example 95. A20V battery of internal resistance IQ is
16 V battery iuit}7 internal resistance of 1Q, as shown in connected to three coils of 12 Q, 6 Q and 4 Q in parallel, a
Fig. 3.118. resistor of 5 Q and a reversed battery (emf = 8 V and internal
(fl) Compute equivaletit resistance of the network, resistance =2Q), as shown in Fig. 3.119. Calculate the
(b) Obtain the current in each resistor, and current in each resistor and the terminal potential difference
across each battery. fCBSE OD OIC]
(c) Obtain the voltage drops Vg^ and
lO 20 V
[NCERT;CBSE SP 18]
12 a
■VWH I >

h
in
I

D
6n sn G G G
CM ■rr

8V, 2n

w
‘d r
16V

1 n i l^WV

Flo
Fig. 3.119
Fig. 3.118
Solution. Equivalent resistance R' of 12 Q,6Q,4Q

ee
Solution, (fl) As the two 4 Q resistances are in resistances connected in parallel is given by

Fr
parallel, their equivalent resistance is 1
4x4
Ri = = 2Q K' “ 12 6 4 “ 12 “ 2
4 + 4
for
ur
or R'=2Q
Also, the 12 Q and 6Q resistances are in parallel, Total resistance = l + 5+ 2+ 2=10Q
their equivalent resistance is Net emf =20-8 = 12
ks
12x6
= 4Q 12
Yo

12+6 Current in the circuit, I = — = 1.2 A


oo

10

Now the resistances Kj, R, and IQ are in series. So the current through each battery and 5 Q resistor
eB

Hence the equivalent resistance of the network is is 1.2 A.

R= + R2 + 1=2 + 4 + l = 7Q. P.D. across the parallel combination of three


{b) The total current in the circuit is resistors is
r
ou

S
ad

16 P' = IR' =1.2 X 2 =2.4 V


/ = = 2A
R+r 7+1 2.4
Y

Current in 2 Q coil = = 0.2 A


12
● Tlie potential difference between A and B is
2.4
nd
Re

Current in 6 Q coil = = 0,4 A


6
h =h
Fi

2.4
But f + 1^ = 1 =2 A Current in 4 Q coil = = 0.6 A.
4
1^=1 A
The potential difference between C and D is Terminal p.d. across 20 V battery,
K = 7r = 20-1.2 X 1 = 18.8 V
Yqq =12 13 =6 i.e., l_^=2I^
Terminal p.d. across 8 V battery,
But 13+/^ = 7 =2 A
y' = E' + 7 =8 + 1.2 X 2 = 10.4 V.
or
/3 +2 73 =2A
Example 96.36 cells each of internal resistance 0.5 Q and
=t
and -A.
emf 1.5 V each are used to send current through an external
circuit of 2 .0 resistance. Find the best mode of grouping
(c) V
AB = 4x 7j=4xl = 4V,
them and the current through the external circuit.
BC = lx 7=1x2 =2 V,
Solution. Here6=1.5 V, /-=0.5Q, R=2Q
KCD = 12 X 73 = 12x- = 8 V.
3 Total number of cells, mn =36 ...(1)
PHYSICS-Xll

For maximum current in the mixed grouping, Solution. In the steady state (when the capacitor is
nr Kx 0.5 fully charged), no current flows through the branch BE.
— = R or ...(2)
m m Net emf=2V-V = V

Multiplying equations (1) and (2), we get Net resistance = 2R+R=3R


0.5 n^=72 or «^=144 y
Current in the circuit, / =
n = 12 and m = 36
— =3^ 3R
12 Potential difference across BE
Thus for maximum current there should be three V 4
=2V-Ix2R=2V- x2R=-V
rows in parallel, each contairung 12 cells in series. 3R 3
Maximum current 4 V
(0 Potential difference across C = — V-V = ~.
3 3
mnt 36x1.5
= 4.5 A. V cv
mR + nr 3 x 2 +12 x 0.5 (ii) Charge on the capacitor, Q = Cx — = —
3 3
Example 97. 22 cells, each of emf 1.5 V and internal (in) Energy stored in the capacitor
resistance of 0.5 H, are arranged in m rows each containing 1 V
n2
CV^
n cells connected in series, as shown. Calculate the values of = -c —
n and m for which this combination would send maximum 2 ) 18
current through an external resistance of 1.5 Q. -\oblems For Practice
(CBSE Sample Paper 08]
R = 1.5Q 1. Three identical cells, each of emf 2 V and internal
resistance 0.2 H are connected in series to an
U- 1 external resistor of 7.4 Q. Calculate the current in
I the circuit. (Ans. 0.75 A)
I
4 m rows 2. Three identical cells each of emf 2 V and unknown
I

I internal resistance are connected in parallel. This


--II- |. _ I combination is connected to a 5 Q resistor. If the

(n cells in each row) terminal voltage across the cells is 1.5 V, what is the
internal resistance of each cell ? (Ans. 5Q)
Fig. 3.120
3. Two cells of emfs 1.5 V and 2.0 V having internal
Solution. For maximum current through the resistances 0.2 Q and 0.3 respectively are
external resistance. connected in parallel. Calculate the emf and
External resistance internal resistance of the equivalent cell.
[CBSE D 16]
= Total internal resistance of the cells
nr (Ans. 1.7 V, 0.12 n)
or R = —
m 4. When 10 cells in series are connected to the ends of
n X 0.5 a resistance of 59 Q, the current is found to be
1.5 = [mn = 12]
12 0.25 A, but when the same cells after being
n connected in parallel are joined to the ends of a
0.05 Q, the current is 25 A. Calculate the internal
or 36 = or n =6 and m = l.
resistant and emf of each cell. (Ans. O.IQ, 1.5 V)
Example 98. Iw the given circuit in the steady state, obtain 5. Find the minimum number of cells required to
the expressions afr (i) the potential drop (ii) the charge and produce an electric current of 1.5 A through a
(Hi) the energy stored in the capacitor, C. [CBSE F 15] resistance of 30 fl. Given that the emf of each cell is
Y R 1.5 V and internal resistance 10 Q.
F
A 4 -vw
(Ans. 120 cells, 60 cells in one row
V C and two rows in parallel)
BU 1
11 J.£ 6. Two identical (»lls, whether joined together in series
or in parallel give the same current, when connec
2V 2R ted to an external resistance of 1 Q. Find the inter
c 4 -VW D nal resistance of each cell. [ISCE 95)

(Ans. 1^1)
Fig. 3.121
CURRENT ELECTRICITY

When the cells are g


7. A set of 4 cells, each of cmf 2 V and internal resis y

tance 1.5 are cormected across an external load of connected in i

ion with 2 rows, 2 cells in each branch. Calculate parallel (Fig. 3.123), -A/W-
the current in each branch and potential difference the current in the '6
across 10 H. (Ans. 0.175 A, 3.5 V) circuit is I
, Jp
g
HINTS Txr
r

1-H
ng 3x2
r+ r
-Wv-
1. 7 = = - = 0.75 A. R=ia
R+ nr 7.4 + 3 X 0.2 8
2g
2. Here g = 2V,y = 1.5V,R = 5Q Fig. 3.123
2+ r
If r' is the total internal resistance of the three cells
2g 2g
connected in parallel, then Given 1^ = 1^ 1 + 2r 2+ r

w
2-1.5 5
r'= K = -n or l-i-2r = 2+r or r = in.
V 1.5 3
7. The circuit diagram is shown below.
1 1 1
3^3 g

Flo
But- = - + + - or r = sn. g
f r r r 5 r I

_ 1.5x0.3+2.0xQ.2 -VA~ -V/v

ee
3. 6 V r r
fl? 0.2+0.3
'i+^2 g g

Fr
I
0.45+0.40 I
V = 1.7 V -VA
0.5 r r

for
ur
_ hh _0.2x0.3 = 0.12 a
-AW-
0.2+0.3 R = ]0fl

4. For series combination. The current is Fig. 3.124


ks
log
Yo

= 0.25 A Here g = 2V, r = 15Q, R = 10n, n = 2, m = 2


oo

59 + 10 r
ng 2x2
7 = — = 0.35 A.
For parallel combination. The current is 2x1.5 11.5
eB

nr
R + 10 +
10 g m 2
= 25 A
10 X 0.05 + r
The two branches are identical.
r

On solving the above two equations, .-. Current in each branch =


0.35
= 0.175 A.
ou
ad

r = 0.1 Cl and g = 1.5 V. 2

n X 1 Potential difference across R


Y

nr
5. As — = R = 30 or tt = 30 m
m m = 7R = 0.35x10 = 3.5 V.
nd

n X 1.5
Re

1.5 = or « = 60
2R
or
2x30
3.22 HEATING EFFECT OF CURRENT
Fi

m = 60 / 30 = 2 and mn = 120. 42. What is heating effect of current ? Explain the


cause of heating effect of current.
6. When the cells are connected in series, (Fig. 3.122),
the current in the circuit is Heating effect uf current. Consider a purely
resistive circuit i.e., a circuit which consists of only
2g
some resistors and a source of emf. The energy of the
1+ 2r
source gets dissipated entirely in the form of heat
g g produced in the resistors. The phenomenon of the produc
I tion of heat in a resistor by the flow of an electric current
through it is called heating effect of current or Joule heating.
/s ’ ●
r r 7. Cause of heating effect o! current.When a potential
difference is applied across the ends of a conductor, its
R = in
-AA/V
free electrons get accelerated in the opposite direction
of the applied field. But the speed of the electrons does
Hg. 3.122 not increase beyond a constant drift speed. This is
3.60 PHYSICS-XII

because during the course of their motion, the but with a steady drift velocity. This is because of the
electrons collide frequently with the positive metal collisions of electrons with ions and atoms during the
ions. The kinetic energy gained by the electrons during course of their motion. The kinetic energy gained by
the intervals of free acceleration between collisions is the electrons is shared with the metal ions. These ions
transferred to the metal ions at the time of collision.
vibrate more vigorously and the conductor gets heated
The metal ions begin to vibrate about their mean up. The amount of energy dissipated as heat in
positions more and more violently. The average kinetic conductor in time t is
energy of the ions increases. This increases the
temperature of the conductor. Thus the conductor gets

H = Vn joule = I^Rt joule = R


joule
heated due to the flow of current. Obviously, the
electrical energy supplied by the source of cmf is Vlt I^Rt V^t
converted into heat. or H = cal = cal = cal'
4.18 4.18 4.18 R

3.23 HEAT PRODUCED BY ELECTRIC The above equations are known as Joule's law
CURRENT : JOULE'S LAW
of heating. According to this law, the heat produced in a
resistor is
43. Obtain an expression for the heat developed in a
resistor by the passage of an electric current through it. 1. directly proportional to the square of current for a
Hence state Joule's law of heating. given R,
Heat produced in a resistor. Consider a conductor 2. directly proportional to the resistance RJbra given I,
AB of resistance R, shown in Fig. 3.125. A source of emf 3. inversely proportional to the resistance Rfor a given
maintains a potential difference V between its ends A V, and
and Band sends a steady current I from Ato B. Clearly, 4. directly proportional to the time for which the
> Vg and the potential difference across AB is current flows through the resistor.
\/ = K KB >0

I
-H
I
For Your Knowledge
> o—VVv—^
A R B > Tine equation :W = Vlt is applicable to the conversion of
electrical energy into any otlier form, but the equation:
I I
H = l^Rt is applicable only to the conversion of electrical
energy into heat energy in an ohmic resistor.
Ml- > Joule's law of heating holds good even for a.c. circuits.
Only current and voltage have to be replaced by their
Fig. 3.125 Heat produced in a resistor. rms values.

The amount of charge that flows from A to B in If the circuit is purely resistive, the energy expended
time t is by the source entirely appears as heat. But if the
circuit has an active element like a motor, then a part
q=If
of the energy supplied by the source goes to do useful
As the charge q moves through a decrease of work and the rest appears as heat.
potential of magnitude V, its potential energy
decreases by the amount,
3.24 ELECTRIC POWER
L/= Final P. E at B - Initial P.E. at A
44. Define the term electric power and state its SI
= ^yB-^^A=-n(VA-Vs) = -qV<0 unit.
If the charges move through the conductor without
suffering collisions, their kinetic energy would change so Electric power. The rate at zohich work is done by a
that the total energy is unchanged. By conservation of source of emf in maintaining an electric current through a
energy, the change in kinetic energy must be circuit is called electric poxner of the circuit. Or, the rate at
zvhich an appliance converts electric energy into other forms
K = -U=qV=ltxV = Vlt>0
of energy is called its electric power.
Thus, in case, charges were moving freely through
the conductor under the action of the electric field, If a current 1 flows through a circuit for time t at a
their kinetic energy would increase as they move. constant potential difference V, then the work done or
However, we know that on the average, the charge energy consumed is given by
carriers or electrons do not move with any acceleration W = VJt joule
CURRENT ELECTRICITY 3.61

Electric power. 3.26 POWER RATING


w ■)
p = ll = W = /^R = 46. What is meant by the power rating of a circuit
t R element ? Briefly explain hozv can we measure the electric
or Electric power = current x voltage.
power of an electric lamp ?
SI unit of electric power. The SI unit of electric
Power rating. The power rating of an electrical appliance
is the electrical energy consumed per second by the appliance
power is watt (W). The power of an appliance is one xvatt if when connected across the marked voltage of the mams. If a
it consumes energy at the rate ofl joule per second. Or, the
power of a circuit is one watt if 1 ampere of current voltage V applied across a circuit element of resistance
R sends current I through it, then power rating of the
flows through it on applying a potential difference of 1 element will be
volt across it.
1 coulomb
1 watt = 1 joule _ 1 joule ^ P = —=/^R = Wwatt
1 second 1 coulomb 1 second R

w
or 1 watt = 1 volt X 1 ampere Measurement of electric power. To measure the
electric power of an appliance, say an electric lamp, we
The bigger units of electric power are kilowatt connect a battery and an ammeter in series with the

Flo
(kW) and megawatt (MW). electric lamp and a voltmeter in parallel with it, as
1 kW = 1000 W and 1 MW = 10“^ W shown in Fig. 3.126. Suppose the voltmeter reads V

ee
The commercial unit of power is horse power (hp) volts and the ammeter reads I amperes, then power
rating of the electric lamp will be

Fr
lhp=746 W.
P = VI watt
3.25 ELECTRIC ENERGY Battery
for
ur
45. Deifne the term electric energy. State its SI and 11-
commercial units.
/" ,J
Electric energy. The total work done (or the energy
ks
supplied) by the source of emf in maintaining an electric
Yo

current in a circuit for a given time is called electric energy


oo

consumed in the circuit. It depends upon the power of


the appliance used in the circuit and the time for which T<v>
eB

this power is maintained.


Fig. 3.126 To measure electric power of
Electric energy. an electric lamp.
r

W = P.t = Vlt joule = I^Rf joule


ou
ad

The SI unit of electric energy is joule (J). 3 27 POWER CONSUMPTION IN A


Y

1 joule = 1 volt X 1 ampere x 1 second COMBINATION OF APPLIANCES


= 1 watt X 1 second 47. Prove that the reciprocal of the total power con-
nd

sumed by a series combination of appliances is equal to the


Commercial unit of electric energy. The commercial
Re

unit of electric energy is kilowatt hour or Board of sum of the reciprocals of the individual powers of the
Fi

Trade (B.O.T.) unit. One kilowatt hour is defined as the appliances,


electric energy consumed by an appliance of 1 kilowatt in one Power consumed by a series combination of
hour.
appliances. As shown in Fig. 3.127, consider a series
1 kilowatt hour = 1 kilowatt x 1 hour combination of three bulbs of powers P,, and P3 ;
= 1000 watt x 3600 s which have been manufactured for working on the
same voltage V.
= 3,600,000 joules
or 1 kWh = 3.6 x 10*^ J P„V PyV

The electric metres installed in our houses mea


sure the electrical energy consumed in kilowatt hours.
Another common unit of electric energy is watt
hour. It is the electric energy consumed by an appliance of ■oVo-
one watt in one hour.
1 watt hour * 1 watt x 1 hour = 3.6 x 10^ J Fig. 3.127 Series combination of bulbs.
3.62 PHYSICS-Xll

The resistances of the three bulbs will be The resistances of the three bulbs will be

R
1 “
p Ri =
M ' p
^2 ' p
'3 P,' ^2 ^3
As the bulbs are connected in series, so their As the bulbs are connected in parallel, their
equivalent resistance is effective resistance R is given by
R = Rj + R2 + R^ 1 1
— + — + —
1 1

If Pis the effective power of the combination, then R


Pj i?2 P3
_V^ ^ Multiplying both sides by V^, we get
V'^
1 = J_ i J_
+
or ~R~.11 ^ R3
P P P P
M h ^3
or
P = Pj + + 1^
Thus for a series combination of appliances, the
reciprocal of the ef^ctive power is equal to the sum of the Thus afr a parallel combination of appliances, the elective
reciprocals of the individual powers of the appliances. power is equal to the sum of the powers of the individual
appliances.
Clearly, when N bulbs of same power P are

connected in series, If N bulbs, each of power P, are connected in


parallel, then
P = NP
^f~N
The brightness of the three bulbs will be
As die bulbs are connected in series, the current I
through each bulb will be same. 1^2
R1 = ,P-= —, P,= —.
P
7 1
1 =

Pj + ^2 + ^3 As the resistance of the highest wattage (power)


The brightness of the three bulbs will be minimum, it will glow with maximum bright-
P'=fR P'=/^P, P'-fR^ ness, if the current in the circuit exceeds the safety
1 1' 2 “ 3“ H limit, the bulb with maximum wattage will be fused
As P Qc —, the bulb of lowest wattage (power) will this reason, the appliances in houses are
^ connected in parallel,
have maximum resistance and it will glow with maxi-

mum brightness. When the current in the circuit exceeds 3.28 EFFICIENCY OF A SOURCE OF EMF
die safety limit, the bulb of lowest wattage will be 49. Define efficiency of a source of emf Write an
fused first.
expression for it.
48. Prove that when electrical appliances are Efficiency of a source of emf. The efficiency of a source
connected in parallel, the total power consumed is equal
of emf is defined as the ratio of the output power to the input
to the sum of the powers of the individual appliances. power. Suppose a source of emf € and internal
Power consumed by a parallel combination of resistance r is connected to an external resistance P.
appliances. As shown in Fig. 3.128, consider a parallel Then its efficiency will be
combination of three bulbs of powers P^, Pj and Py
which have been manufactured for working on the _ Output power _VI _V IR
same voltage V. Input power El € /(P+r)
P
or n =
p + r

50. (fl) A battery of emf t and internal resistance r is


connected across a pure resistive device (e.g., an electric
heater or an electric bulb) of resistance R. Show that the
power output of the device is maximum when there is a
perfect 'matching' between the external resistance and the
source resistance (i.e., where R = r). Determine the
Fig. 3.128 Parallel combination of bulbs. maximum power output.
CURRENT ELECTRICITY

VI V IR
(b) What is power output of the source above if the Efficiency Tj = -^
battery is shorted ? What is the power dissipation inside Sf € J(R + r) R+r
the battery in that case ? [NCERT] When the source delivers maximum power, R = r
Maximum power theorem. It states that the output -I-=-=50%
Ti =
power of a source of emf is maximum when the external r+r 2
resistance in the circuit is equal to the internal resistance of
the source. Thus the efficiency of a source of emf is just 50%
=6
when it is delivering maximum power.
Let emf of the battery
Internal resistance = r 3.29 EFFICIENCY OF AN ELECTRIC DEVICE
Resistance of the device = R 52. Define efficiency of an electric device. Write an
Current through the V expression for the efficiency of an electric motor.
Efficiency of an electric device. The efficiency of an

w
device. «—'VW
R
Total emf ^ 1 electric device is defined as the ratio of the output power to
J = the input power
Total resistance y
Output power

Flo
e -n =
Input power
R+r Fig. 3.129 For an electric motor, we can write

ee
Power output of the resistive device will be Output mechanical power

Fr
Tl =
8
Input electric power
P=/^R = R
Here, input electrical power
R + r
for
ur
= Output mechanical power + Power lost as heat
e^R
53. {a) An electric motor runs on a d.c. source of emf ^
(R + r)" {R-rf+ 4Rr and internal resistance r. Show that the power output of
ks

Obviously, the power output will be maximum the source is maximum when the current drawn by the
Yo

motor isttlr.
oo

when

R-r = 0 or R=r (b) Shaw that power output of electric motor is maximum
eB

Thus, the power output of the device is maximum when the back emf is one-half the source emf provided the
when there is a perfect matching between the external resistance of the windings of the motor is negligible.
resistance and the resistance of the source, i.e., when (c) Compare and contrast carefully the situation in this
r

exercise with that in Q.50(fl) above.


ou

[NCERT]
R = r. This proves maximum power theorem.
ad

Maximum power output of the source is (fl) Output power from a source connected to an
Y

electric motor. Let the current drawn by the motor be 1.


P = — [Putting R = r in Eq. (/)] Then
{r+rf
max
4r
nd

Power output of the source, P = €/-/^r


Re

(b) When the battery is shorted, R becomes zero,


Fi

u n
P IS maximum when — = U
therefore, power output =0. In this case, entire power dl
of the battery is dissipated as heat inside the battery 8
due to its internal resistance. or 6-2/r=0 or l =—
2r
Power dissipation inside the battery
Hence the power output of the source is maximum when
= /2r=
/
8f
-
the current drawn by the motor is 8/2 r.
rj r
(b) Here, emf of source =6

Internal resistance of source = r


51. Show that the efficiency of a battery when
delivering maximum power is only 50%. Back emf of motor =6'

Maximum efficiency of a source of emf. For a Resistance of motor = R~0

source of emf. As the external resistance R is negligible, therefore


=8/ 8-8
Input power current in the circuit =
Output power = VI
r
PHYSICS-Xll

And power output of the motor of it appears as light. A bulb gives nearly 1 candela of
= Power output of the source light energy for the consumption of every watt of
= g/-/2r electric power.
From part {a), this is maximum when 3. Electric fuse. It is a safety device used to protect
6 S-6' 6
electrical appliances from strong currents. A fuse wire
/ = —
2r
or or must have high resistivity and low melting point. It is
2r 2
usually made from an alloy of tin (63%) and lead
Hence the power output of electric motor is maximum (37%). It is put in series with the live wire of the circuit.

when the back emfis one-half the source emf When the current exceeds the safety limit, the fuse
MTU breaks the circuit. The electric
■ “Edition m Q. 50(a) .s for a passive resistor jnstallaHons are thus saved from getting damaged,
m which the entire electric energy is converted into o o o
heat while the condition in Q. 53(a) is for a non-passive The fuse wire of suitable current rating (1 A, 2 A,
3 A, 5 A, 10 A etc.) should be used in the circuit
resistor (e.g., electric motor) in which the supplied
depending on the load in the circuit. For example,
electric energy changes partly into heat and partly into when we use an electric iron of 1 kW electric
mechanical work. So the former is a special case of the power
latter. with electric mains of 220 V, a current of (1000/220) A
i.e., 4.54 A flows in the circuit. This requires a fuse of
3.30 APPLICATIONS OF HEATING 5 A rating.
EFFECT OF CURRENT 4. Electric arc. It consists of two carbon rods with a

54. Discuss some practical applications of the heating ‘heir pointed ends. When a high
effect of current. potential difference of 40 - 60 V is applied between the
Applications of heating effect of current. Some of
two rods, very intense light is emitted by the gap. We
know that E — — dV / dr. Clearly, £ will be large if the
the important applications of Joule heating are as follows: gap is small. When the electric field exceeds the
1. Household heating appliances. Many electrical dielectric strength of air, ionisation of air occurs. This
appliances used in daily life are based the heating
on
causes a big spark to pass across the gap.
effect of current such as room heater, electric toaster, 5. Other devices. Many other devices are based on
electnc iron, electric oven, electric kettle, geyser, etc. the heating effect of current such as electric welding.
The designing of these devices requires the selection of thermionic valves, hotwire ammeters and voltmeters
‘'■‘●nsmitted at high
resistance so that most of the electric power is voltages and low currents to distant places.
converted into heat. In most of the household heating
appliances, nichrome element is used because of the High voltage power transmission. Electric power is
following reasons : transmitted from power stations to homes and
factories through transmission cables. These cables
(i) Its melting point is high have resistance. Power is wasted in them as heat. Let
(n) Its resistivity is large us see how can we minimise this power loss.
(Hi) It is tensile, i.e., it can be easily drawn into wires, Suppose power P is delivered to a load R via
(iy) It is not easily oxidised by the oxygen of the air transmission cables of resistance R,. If V is the voltage
when heated.
across load R and I the current through it, then
2. Incandescent electric bulb. It is an important P= W
application of Joule heating in producing light. It The power wasted in transmission cables is
consists of a filament of fine metallic wire enclosed in a

glass bulb filled with chemically inactive gases like P^R,


P, = I^R, =
nitrogen and argon. The filament material should
have high resistivity and high melting point. Thus the power wasted in the transmission cables
Therefore, tungsten (melting point 3380°Q is used for is inversely proportional to the square of voltage.
-■ bulb filament. When current is passed through the Hence to minimise the power loss, electric power is
filament, it gets heated to a high temperature and transmitted to distant places at high voltages and low
emits light. Most of the power consumed by the currents. These voltages are stepped down by
filament is converted into heat and only a small part transformers before supplying to homes and factories.
A

CURRENT ELECTRICITY 3.65 A

For Your Knowledge Example 99. An electric current o/4.0 Afloivs through a
12 Q resistor. What is the rate at which heat energy is
> The emission of light by a substance wlien heated to a
produced in the resistor ? [NCERT]
high temperature is called incandescence.
Solution. Here / = 4 A, K=12a
^ A heater wire is made from a material of large resis
tivity and high melting point while a fuse wire is made Rate of production of heat energy,
from a material of large resistivity and low melting point. P=l^R=4^xl2=192 W.
^ The load in an electric circuit refers to the current
Example TOO. Hozo many electrons flow through the
drawn by the circuit from the supply line. If the
current in a circuit exceeds the safe value, we say that
iflament of a 120 -19
V and 60 W electric lamp per second ?
Given e-1.6x 10 c.
the circuit is overloaded.
Solution. Here P =60 W, V = 120 V, f = 1 s
> The temperature upto which a wire gets heated (i.e.,
steady state temperature 6) is directly proportional to /=p 60
= 0.5 A

w
the square of the current and is inversely proportional V 120
to the cube of its radius but is independentof its length. But
q _ ne
/ = -i-
t ~ t
0 « “7

Flo
.'. No. of electrons flowing per second is
> When the resistances are connected in series, the
// 0.5 X 1
current I through each resistance is same. Consequently, = 3.125 X 10^®.

ee
n- —
-19
e 1.6x10
PxR tP = I^R)

Fr
and V ccR (V V = /R) Example 101. A heating element is marked 210 V,630 W.
Hence in a series combination of resistances, the What is the current drawn by the element when connected to
potential difference, power consumed and hence heat a 210 Vd.c. mains ? What is the resistance of the element ?
for
ur
produced will be larger in the higher resistance. [NCERT]
> Wlien the resistances are connected in parallel, the Solution. Here R = 630 W, \/ = 210 V
potential difference V is same across each resistance.
P ^630
ks
Consequently, Current drawn, I = = 3 A.
Yo

V 210
oo

I
Poz- 210
= 70 n.
R R
Resistance of the element, ^ = y 3
eB

1 1/ ^
and 7 oc —
R R ) Example 102. A10 V storage battery of negligible internal
resistance is connected across a 50 Q resistor made of alloy
Hence in a parallel combination of resistances, the
r

current, power consumed and hence heat produced


manganin. How much heat energy is produced in the resistor
ou
ad

will be larger in the smaller resistance. in Ih? What is the source of this energy 7 [NCERT]
Solution. Here V = 10 V, R = 50Q, f=l h = 3600 s
Y

Examples based on Heat energy produced in 1 h is


Heating Effect of Current, Electric Vh 10x10x3600
nd
Re

Power and Electric Energy H = = 7200 J.


R 50
Fi

Formulae Used The source of this energy is the chemical energy


1. Heat produced by electric current, stored in the battery.
fRt Example ^ 03. An electric motor operates on a 50 V supply
H = I^Rt joule - cal
4.18 and draws a current of 12 A If the motor yields a mecha
or H = VJt joule =
Vlt
cal nical power of 150 W, what is the percentage effciency of the
4.18 motor ? [NCERT]
W 1 Solution. Input power ■W'l = 50 x 12 =600 W
2. Electric power, P = y = V7 = 7^R = R
Output power = 150 W
3. Electric energy, W = Pt =VIt = I^Rt Efficiency of motor
Units Used
Output power 150 X100
xl00 =
Current 7 is in ampere, resistance R in ohm, time t 600
Input power
in second, power P in watt, electric energy in joule
or in kWh. = 25%.
3M PHYSICS-XIl

Example 104. An electric motor operating on a 50 V d.c. Rate of heat dissipation at 200 V is
supply draws a current of 12 A. If the efficiency of the motor = /2k ={5 X 10^)2r =25 X 10^° R watt
is30%,estimate the resistance of the windings of the motor.
[NCERT] Clearly, Pj <
Hence there will be lesser power wastage when the
^Solution. Here y = 50 V, / = 12 A, r\= 30% power is transmitted at 20,000 V.
As the efficiency of electric motor is 30%, therefore,
Example 107. Two ribbons are given with the following
power dissipated as heat is
70
particulars:
P = 70% of V7 = X 50 X 12 W = 420 W
100 Ribbon A B

But power dissipated as heat, P = I^R Alloy Constantan Nichrome


fR = 420
Length (m) 8.456 4.235
or K = 420 ^420 = 2.9n. Width (mm) 1.0 2.0
/2 “l44
Thickness (mm) 0.03 0.06
Example 105. (a) A nichrome heating element across
230 V supply consumes 1.5 kW of power and heats up to a Temp, coefficient of Negligible Negligible
resistivity (°C~^)
temperature of 750® C. A tungsten bulb across the same supply
operates at a much higher temperature o/1600®C in order to Resistivity (Qm) 4.9 X10”^ 1.1 xlO
-6

be able to emit light. Does it mean that the tungsten bulb


necessarily consumes greater power ? (b) Which of the two For a fixed voltage supply, which of the two irbbons corres
ponds to a greater rate of heat production ? [NCERT]
has greater resistance : a 1 kW heater or a 100 W tungsten /
bulb, both marked for 230 V ? (NCERT] Solution. Since R =
<'a
Solution, {a) No, the steady temperature acquired
by a resistor depends not only on the power consumed .●. Resistance of constantan ribbon,
but also its characteristics such as surface area, emissi- 4.9 x 10”^ X 8.456
R n =138.10
vity, etc., which determine its power loss due to radiation, I “
1.0 X 10"^ X 0.03x10
(b) Here V = 230 V, P^ = 1 kW = 1000 W, P^ = 100 W
Let V be the fixed supply voltage. Then the rate of
230x230
«i = Q =52.90 production of heat in constantan ribbon,
Pi 1000
y2
230x230
Pl = R 138.1
watt
1
0=5290
100
Resistance of nichrome ribbon.
Thus the 100 W bulb has a greater resistance. l.lx 10
-6
X 4.235
«2 =
Example 106. An electric power station (100 MW) 2.0 X 10'^ X 0.06 X 10 :jO =38.80
transmits power to a distant load through long and thin
cables. Which of the two modes of transmission would result Rate of production of heat in nichrome ribbon,
in lesser power wastage: power transmission of: (i) 20))00 V
or (ii) 200 Y ? P2 = watt
[NCERT]
R2 ~ 38.8
Solution. Let R be the resistance of transmission cables.
Here P = 100MW=100xl0^W Clearly nichrome ribbon has greater rate of
produc- tion of heat because of its lesser resistance.
(i) Vj =20OT V Example 108. A heater coil is rated 100 W, 200 V. It is cut
P 100x10^ into two identical parts. Both parts are connected together in
.'. Current, =— = 5000 A
Vi 20,000 parallel, to the same source of200 V. Calculate the energy
liberated per second in the new combination.
Rate of heat dissipation at 20,000 V is [CBSE OD 2000]
P^ = I^R={5000fR =25x lO^R watt. Solution. Resistance of heater coil,
(ii) V^=200W R =
200 X 200
=4oon
100x10^ p 100
.●. Current, = = 5x10^ A
200 Resistance of either half part =2000
CURRENT ELECTRICITY 5,67

Equivalent resistance when both parts are connected Example 111. The maximum power rating of a 20 Cl
in parallel. resistor is 2.0 kW. (That is, this is the maximum power the
R' =
200 X 200
=ioon
resistor can dissipate (as heat) without melting or changing
200+200 in some other undesirable way). Would you connect this
resistor directly across a 300 V d.c. source of negligible internal
Energy liberated per second when combination is resistance ? Explain your answer. [NCERT]
connected to a source of 200 V
Solution. Maximum power rating of the given 20 O
200x200
= 400 J. resistor.
K 100 P' = 2.0kW

Example 109. An electric bulb is markedlOO W,230 V. If When connected to 300 V d.c. supply, the power
the supply voltage drops to 115 V, what is the heat and light consumption or rate of production of heat would be
energy produced by the bulb in 20 min ? Calculate the 300 X 300
W = 4500 W= 4.5 kW

w
current flowing through it. [NCERT; CBSE F 94) R 20
Solution. If the resistance of the bulb be R, then This power consumption exceeds the maximum
Rate of production of heat and light energy. power rating of the resistor. Hence the 20 resistor

Flo
must not be connected directly across the 300 V d.c.
R
source. For doing so, a small resistance of 10 O should
be connected in series with it.

ee
230 X 230
R = = 5290
Example 112. Two electric bulbs rated as 100 W, 220 V

Fr
P 100
and 25 W, 220 V are connected in series across 220 V line.
When the voltage drops to V =115 V, the total Calculate (i) current through (ii) potential difference across
heat and light energy produced by the bulb in 20 min actual powers consumed in filament of each bulb.
for
ur
will be
Solution. Resistances of the two bulbs are
H= Px f = X t 220 x 220
R, = — = = 4840
ks
R
^ Pi 100
Yo

115x115
oo

X 20x60=30^)00 J = 30 kj. _ 220 x 220


= 1936 O
529
25
^2
eB

Current, 1=^ 115


=Aa. = Ri + R2 = 484 +1936 = 24200
R 529 23
y 220
Example 110. An electric bulb rated for 500 W at 100 V is (0 1 = = —A.
r

R^ 2420 11
ou

used in a circuit having a 200 V supply. Calculate the


ad

resistance R that must be put in series with the bulb, so that V.1 =R,/ = 484x —= 44 V.
(»■)
Y

1
the bulb delivers 500 W. [UT 87]
Solution. Resistance of the bulb, 1
K = R,/ = 1936 x — = 176 V
2 ^ 11
nd
Re

100 X 100
R = = 20 0
P 500 ^ 1
m p, = /"R,= x484 = 4 W.
Fi

11
Current through the bulb,
V 100
R “ 20
= 5A P,2 = J^R,^ =(—
In
X1936 = 16 W.

For the same power dissipation, the current Example 113. Two heaters are marked 200 V, 300 W and
through bulb must be 5 A. 200 V, 600 W. If the heaters are combined in series and the
When the bulb is cormected to 200 V supply, the combination connected to a 200 V d.c. supply, which heater
safe resistance of the circuit should be will produce more heat ? (NCERT)
Solution. Resistances of the two heaters are
r 200
R' = = 40 y2 200x200 400
I ~ 5 R Q.
3
Resistance required to be put in series with the Pi 300

bulb is y2 200 X 200 200


— = a
R'-R = 40-20=20a. 600 3
^2
PHYSICS-Xll

For series combination. Solution. Total resistance between the points P


600 and Q,
=200 n 4x4
R = + 4=2 +4=6n
4 + 4
...Current, ; = !R = 200 =1 A
Current in the circuit, / = 1 A

Power dissipations in the two heaters are Power dissipated in the circuit,
400 400 p= fR=l^x6=6W.
p; = fR,=i^x ^ 3
w
Example 116. The current is drawn from a cell of emft
and internal resistance r connected to the netxvork of
P2' = /2 3
W
resistors each of resistance r as shown in Fig. 3.132(a).
Obtain the expression for (i) the current drawn from the cell
p;=2P' and (ii) the power consumed in the network. [CBSE D 17]
The first heater (of 300 W) produces more heat than
the second heater. tVa-t
A e
Example 114. In a part of the circuit shown in the r
c
r

Fig. 3.130, the rate of heat dissipation in 4Q resistor is


iWV
100 / / s. Calculate the heat dissipated in the 3 Q resistor in r
10 seconds. (CBSE SP 03]
e.r
R1 R2 Fig. 3.132 (a)
—vw
4Q 2fl Solution. The equivalent circuit is shown in
Fig. 3.132(&).

AAAr 7 r

rAW
A B
r C
Fig. 3.130 r

Solution. Let /j be the current through the series


combination of and R^ and be the current through
1
I
1
l-AW T
R,. ' € I
L —

P.D. across (Pj + R^) = P.D. across R^ Fig. 3.132 (b)

(4+2)/j =3/2 or 12=21-1 R(middle arm) = R AC + A-r = - + “ -r


XB 2 2
Rate of heat dissipation in 4Q resistor
= /j^Rj =/fx 4=100 = l+l+
R r r r r
AB
100
^1 = = ^/^ = 5 A r
4 R
AB-3
and
/2=2/^=10A This resistance is in series with the internal
Heat dissipated in 3 H resistor in lO s resistance r of the source.
= 1^ R.2^= (10)^ X 3 X 10 = 3000 J. r 4r

Example 115. The resistaiice of each of the three wires, ^eg-3+''-3


shown in Fig. 3.131, is 4D. This co7nbination of resistors is Current drawn from the cell,
connected to a source of 4Q

emfE. The ammeter shozus i—VA—I 4Q


S _ g ^36
1 =
a reading of IK Calculate ^ An Wv iQ
R-,
etj
4r/3 4r
the power dissipated in the
circuit. g Power consumed.
[CBSE F 03] h
P = I^R f36f
eq
4r I 3 4r
Fig. 3.131
CURRENT ELECTRICITY 3,69

Example 117. There are two electric bulbs rated 60 W, Example 119. thin metallic wire of resistance 100 Q is
110 Vfl?2rfl00 W, 110 V. They are connected in serieszuith a immersed in a calorimeter contaitiing250 gof water flnO°C
220 V d.c. supply. Will any bulb fuse ? What will happen if and a current of 0.5 ampere is passed through it for half an
they are connected in parallel zuith the same supply ? hour. If the water equivalent of the calorimeter is 10 g, find
the rise of temperature.
Solution. Currents required by the two bulbs for
Solution. Here w = 250g, 1=0.5 A,
the normal glowness are
f =30 min =1800 s, w=10g
/ - =
= 0.55 A
1 “
V 110 Heat produced
and _ P2 ^ 100 = 0.91 A
= 1^ Rt= (0.5)^ X 100 X 1800 J = 45000 J
” V 110 Heat gained by water and calorimeter
The resistances of the two bulbs are =(m+ zy)c0 ={250 + 10)x lx 0 cal
V 110
=260 X 4.2 0 joule

w
= 202 0
^i=-
I 0.55
1
260x 4.2 x 0=45000
V 110
and = 1210 45000
^2=7- Rise in temperature, 0 = 260 = 41.2°C

Flo
0.91
h X 4.2

When the bulbs are connected in series across the


Example 120. Two coils require 20 minutes and 60

ee
220 V supply, the current through each bulb will be minutes respectively to produce same aznount of heat energy
220

Fr
V
/ = when connected separately to the same source. If they are
= 0.68 A
R^ + R2 202 + 121 connected in parallel arrangement to the same source; ifnd
As f < I and I2 > 1, so that 60 W bulb will fuse the time required to produce same amount of heat by the
for
ur
[JEE Main June 22]
combination of coils.
while the 100 W bulb will light up dim.
Solution.
When the bulbs are joined in parallel, their
equivalent resistance is
ks
H = ^1 =
, R, K, 202 X121 H
Yo
R
R' = — = = 760 1
oo

R, + R2 202 +121
H = =>
eB

Current drawn from the 220 V supply will be R, H


V 220
r=— -3 A For parallel combination,
R' 76
r

In the two bulbs of resistances R.,(-202O) and J__J_ J_


ou
ad

R2(=120 0), the current of 3 A will split up into


roughly 1A and 2 A respectively. Hence both the bulbs
Y

3L = JL JL
will fuse.
v^t v\ ^ v\
Example 118. The resistance of a 240 V and 200 W
nd
Re

electric bulb zvhen hot is 10 times the resistance when cold. 1 = 1+1
t t
Find its resistance at room temperature. If the working 1 ^2
Fi

temperature of the filament is 2000°C, find the temperature t =


tJ
_
20x60
= 15 min.
coefficient of the filament. tj + f 20 + 60
Solution. Resistance of the hot bulb is given by
240 x 240 Example 121. A coil of enamelled copper wire of resis
R = ~ = 288 0 tance 50 0 is embedded in a block of ice and a potential
P 200
difference of 210 V applied across it. Calcidate the rate at
Resistance of bulb at room temperature. zohich ice melts. Latent heat of ice is 80 cal per gram.
R’ 288
R = — = 28.8 O Solution. Here R = 500, V=210V, t=ls,
10 10
L = 80 calg-^
Since R'= R(l+ af)
Heat produced.
288=28.8(1+a x2000)
t 210 X 210 X 1
H = = 210 cal
or a = — ®C"^= 4.5x10"^ 4.2 R 4.2 X 50
2000
PHYSiCS-Xli

Suppose m gram of ice melts per second. Then Example 123. The heater coil of an electric kettle is rated
mL=H flf 2000 W, 200 V. How much time will it take in raising the
H _m temperature of\ litre of water from 20°C to 100®C, assuming
or m = —
L~ 80
= 2.62g s"^
that only 80% of the total heat energy produced by the heater
Example 122. An electric kettle has two heating coils, in raising the temperature of water. Density of
when one of the coils is switched on, the kettle begins to boil cm" ^ and specific heat of water = lcalg~^'^C~^.
in 6 minutes and when the other is switched on, the boiling Solution. Here P =2000 W,
begins in 8 minutes. In what time will the boiling begin if
Volume of water =1 litre = 1000 cm^
both the coils are switched on simultaneously (i) in series
and (ii) in parallel ? [nr] Mass of water.
Solution. Let Rj and R2 be the resistances of the m = Volume x density
two coils, V the supply voltage and H, the heat required = 1000 cm^xlgcm ^=1000g
to boil the water.
Rise in temperature.
For the first coil. H = V\ _ V ^ X 6 X 60 cal
4.2 R 0 = 02 - 0^=100-20 =80°C
1
Heat gained by water
For the second coil, H = V\ _ V 2 X 8 X 60 cal
4.2 1?2 = me 0 = 1000 X1X 80 = 80OT cal
V ^ X 6 X 60 V ^ X 8 X 60 Let t be the time taken to increase the temperature
4.2 R
from 20® to 100°C
1 4.2 1?2
or ^ =^=^ Then total heat produced by heating coil
Rj 6 3 = Pt =2000 t joule
(i) When the coils are connected in series. Useful heat produced
80 X 2000 t
effective resistance = R^ + R2. = 80% 2000 f =
100
Let the boiling occur in time min.
80 X 2000 t
Then cal
100 X 4.2
V 1
X 60 V ^ X 6 X 60
= H =
4.2(Rj + R2) 4.2 R
1
Useful heat produced = Heat gained by water
80 X 2000 t
= 80000
or t =6 1 + 100 X 4.2
I «, J
1 ~
R
80000 X100 X 4.2
or t = = 210 s.
f 4^ 80 X 2000
= 6 1 + — min = 14 min.
3;
Example 124. One kilowatt electric heater is to be used
(ii) When the two coils are connected in parallel, with 220 V d.c. supply, (i) What is the current in the heater ?
effective resistance = (ii) What is its resistance ? (Hi) What is the power dissipated
R] + Rj in the heater ? (iv) How much heat in calories is produced
Let the boiling occur in time t2 min. Then per second ? (v) How many grams of water at W0°Cwill be
converted per minute into steam at 100° C, with the heater ?
V\ X 60 = H =-
X 6 X 60
Assume that the heat losses due to radiation are negligible.
4.2 M2_] 4.2 R
1
Latent heat of steam = 540 cal per gram iim
R, + R^^ SoluUon. Here P = 1 kW =1000 W, V =220 V
or
f2 = 6x
M2 =6x-
1

(R, + R,)R, R,^


1 (0 Current, j_P _ 1000 = 4.55 A.
1 + V 220

1 220 x 220
= 6x- (ii) Resistance, = 48.4 a
1 + -
^ min = 3.43 min. 1000

4 (Hi) Power dissipated in heater = 1000 W.


CURRENT ELECTRICITY 3Jl

(iv) Heat produced per second, Example 127. A dry cell of emf 1.6 V and internal
resistance 0.10 ohmis connected to a resistor of resistance R
Vlt _ P.f _ 1000x1 = 240 cal s"^
H =
ohm. If the current drawn from the cell is 2 A, then
T~T“ 4.2
(i) what is the voltage drop across R 7
(u) Heat produced per minute, (ii) what is the energy dissipation in the resistor ?
H= 240x60 =14400 cal Solution. Here E =1.6 V, r=0.10Q, /=2.0A
We know that 540 cal of heat convert 1 g water at R + r-~ = — = 0.80
100°Cinto steam atl00°C I 2.0
.'.Mass of water converted into steam R = 0.8-0.10 =0.70 n

14400 (i) Voltage drop across R,


= 26.67 g.
540 V= /R =2x0.70 = 1.4 V.

w
Example 125. The walls of a closed cubical box of edge (zj) Rate of energy dissipation inside the resistor
50 cm are made of a material of thickness 1 mm and thermal = W =1.4 X 2.0 = 2.8 W.
conductivity 4 x 10~^ cal s~^ cm~^°C~^. The interior of the Example 128. A dry cell of emf 1.5 V and internal
box maintained at 100^C above the outside temperature by a

Flo
resistanceO.lO Cl is connected across a resistor in series with
heater placed inside the box and connected across a 400 V
d.c. source. Calculate the resistance of the heater. (IIT)
a
very low resistance ammeter. When the circuit is switched

ee
on. the ammeter reading settles to a steady value of 2.0 A
Solution. Here, K =4 x IQ-^cal s”^cm“^°C-\ What is the steady

Fr
02-01 =100°C d=lmm=0.1cm (a) rate of chemical energy consumption of the cell,
Surface area of the six faces of the cubical box, (b) rate of energy dissipation inside the cell,
for
ur
A = 6 X (50 X 50) = 15000 cm^ (c) rate of energy dissipation inside the resistor,
The amount of heat conducted out per second (d) power output of the source ? [NCERT]
Solution. Here E = 1.5 V, r =0.10 £1, / =2.0 A
ks
through the walls of the cubical box is
Yo

K A(02 -0i) 4x10^x15000x100 (fl) Rate of chemical energy consumption of the cell
oo

«l = d 0.1 = E/ = 1.5Vx2.0 A = 3.0 W.


eB

= 6000 cal =6000 x 4.2 J (&) Rate of energy dissipation inside the cell
If R is the resistance of the heater, then heat produced = /^ r = (2 f X 0.10 W = 0.40 W.
per second
r

(c) Rate of energy dissipation inside the resistor


V^ _ (400)^
ou
ad

H2 = I^Rf = R R
[f=ls] = E / - fV = 3.0 - 0.40 = 2.6 W.
Y

(d) Power output of the source


Temperature inside the box will be maintained by
the heater if = Power input to the external circuit
nd

(400)^ = E/-/V = 2.6 W.


Re

= 6000 X 4.2
Hj = H2 or R
Example 129. A series battery of 10 lead accumulators,
Fi

400 X 400
or R = = 6.35 a each of emf 2 V and internal resistance 0.75 ohm is charged
6000 X 4.2
by a 720 V d.c. mains. To limit the charging current, a
resistance of 47.5 ohm is used in series in the charging
Example 126. A 10 V battery of negligible internal circuit. What is (a) the power supplied by the mains and
resistance is charged by a 200 V d.c. supply. If the resistance
in the charging circuit is 38 what is the value of charging (b) power dissipated as heat ? Account for the difference of
current ? (NCERT) power in (a) and (b). (CBSE SP 98]
Solution, emf of the battery =10 x 2 =20 V
Solution. As the battery emf opposes the charging
emf, therefore. Internal resistance of the battery
net emf =200-10 =190 V =10x0.25=2.50

Charging current. Total resistance = r + R = 2.5 + 47.5 = 50.0 O


Net emf 200 -10
= 5 A.
As the battery emf opposes the charging emf,
1 =
Resistance 38 .●. Effective emf = E-V=220-20=200 V
PHYSICS-XII

Effective emf 200


Charging current = =4 A Solution, emf of the battery,
Total resistance 50
g^=2.0x8V=16V
(fl) Power supplied by the mains d.c. supply voltage, = 64 V
= VI =220 X 4 = 880 W.
Internal resistance of the battery,
(b) Power dissipated as heat
r = -x80=lQ
= /^(R + r)=4^x 50=800W. 8

The difference of power = 880 - 800 = 80 W, is Resistance of motor, R = 7.0 O


stored in the battery in the form of chemical energy. Let back emf of motor = 8 m
Example 130. A series battery of 6 lead accumulators each
of emf 2S) V and internal resistarrce 0.50 Q is charged by a Both the back emf 8^^ of the motor and the emf of
100 V d.c. supply. What series resistance should be used in the battery act in the opposite direction of the supply emf
the charging circuit in order to limit the current to 8.0 A ? 8^ Therefore, net current in the circuit must be
Using the required resistor, obtain (a) the power supplied by Net emf 8^.-8,-8
1 = III

the d.c. source (b) the power supplied by the d.c. energy Net resistance r+ R
stored in the battery in 15 min. [NCERT]
64-16-8 III
Solution. Here 8= 2.0 V, r =0.50 0, V=100V, or 3.5 =
1=8.0 A

As the battery emf opposes the charging emf,


or
8,„ =48-28=20 V.
.●. Effective emf = 100 -2.0 x 6 =88 V (a) Mechanical energy yielded by motor in 1 h

Let the required series resistance be of R O. = 8,„ . =20 X 3.5 X 3600 J = 252000 J.
Then (b) Chemical energy stored in the battery in 1 h
total resistance = (0.50 x 6 + R)Q =(3 + R)0 = 8^. /f = 16 X 3.5 X 3600 J = 201600 J.
Total emf
Now / = Example 132. A 24 V batterr/ of internal resistance 4.0 .Q
Total resistance is connected to a variable resistor. At what value of the
8 =
88 current drazon from the battery is the rate of heat produced
3+ R in the resistor maximum ? [NCERT]
64 Solution. Here 8 =24 V, r = 4.0 H
or 24 + 8R=88 or R = —n=8n.
8 Let the variable resistor be R. The rate of heat
(fl) Power supplied by d.c. source produced in the resistor will be maximum when
= W=100Vx8 A = 800 W. External resistance = internal resistance

(b) Power dissipated as heat or R = 4a

.’. Required current,


= /^(R + r)=8^(8+0.50x6)W emf 24
= 64xllW=704 W. 7 = A = 3.0 A.
resistance 4 + 4
(c) Power supplied by the d.c. energy stored in the
battery in 15 min Example 133. 4 cells of identical emf 8, internal
resistance r, are connected in series to a variable resistor. The
= {800-704)Wxl5 min
following graph shows the variation of terminal voltage of
= 96 W X 900 s = 86400 J. the combination with the current output. [CBSE OD 06C]
Example 131. Power from a 64V d.c. supply goes to (7) What is the emf of
charge a battery of 8 lead accumidators each of emf 2.0 V each cell used ? 5,6

t
and internal resistance 1/SQ. The charging current also (ii) For what current from — 4.2
runs an electric motor placed in series with the battery. If the the cells, does maxi- > 2.8
resistance of the windings of the motor is 7.0 Q and the mum power dissi- v.
steady supply current is 3.5 A obtain pation occur in the
1.4

(a) the mechanical energy yielded by the motor, circuit ? 0


0.5 1.0 1.5 2.0
{b) the chemical energy, stored in the battery during (Hi) Calculate the mternal 7 (ampere) -»
charging in 1 h. [NCERT] resistance of each cell.
Fig. 3.133
CURRENT ELECTRICITY 3.13-

Solution. When / =0, Example 135. Two wires made of tinned copper having
identical cross-section ( =10"^m^) and lengths 10 an and
total emf = tem\inal voltage
15 an are to be used as fuses. Show that the fuses will melt at
4E = 5.6V
the same value of current in each case. [NCERT]
or S = 1.4 V
Solution. The temperature of the wire increases up
2.8 to a certain temperature 0 where the heat produced per
When 7 = 1.0 A. y = — =0.7 V
4 second by the current equals heat lost (by radiation)
Internal resistance per second.
^-V 1.4-0.7 But heat produced by the current
r = =0.7 n
I 1.0 I jV
= 72R = /2p_!_
A
=
The output power is maximum, when K

w
external resistance = internal resistance = 4 r
If h is heat lost per second per unit surface area of
Total emf 4E
1 the wire and if we ignore the heat loss from the end
max
Total resistance 4r + 4r faces of the wire, then heat loss per second by the wire

Flo
6 1,4 = 7i X curved surface area of the wire
= 1 A.
^ “ 2 X 0.7 = 7i X Inrl

ee
Example 134. Two batteries, each of ernfi- and internal When the steady state temperature is attained,

Fr
resistance r, are connected in parallel. If we take current fpl
h x2nrl =
from this combination in an external resistance R, then for K
what value of R maximum power will be obtained ? What
for
ur
will be this power ? or h = -(0
Solution. The situation is shown in Fig. 3.134. 27tV
Now h is independent of I and the values of r and p
ks
g
are same for both wires, hence steady state tempe
Yo

I WV
oo

A B
€ rature 0 will depend only on I i.e., the two fuses will
■VA—' melt at the same values of current.
eB

/V

R
Example 136. A fuse with a circular cross-sectional radius
AAAr of 0.15 mm blows at 15 A What should be the radius of
cross-section of a fuse made of the same material lohich will
r
ou

Fig. 3.134 blow at 30 A? [NCERT]


ad

Net emf of the parallel combination of two cells = 6 Solution. Here r^ =0.15 mm, 7^ =15 A, r2 =?
Y

Total resistance in the circuit 72 =30 A


rx r
From Eq. (i), the heat lost per second per unit
nd

+R=-+R
Re

r + r 2 surface area of the wire is


Fi

Hence current in the circuit is


h =
E 26 2rcV
7 =
r + 2R
- + R :. For a fuse wire of the given material and the
2
given value of h,
Power dissipated in the resistance R is
P= 7^R = (2 6)^ 7^
46^R
(r + 2i?)2~(r-2R)^+8rR or
r,\l^
Power P will be maximum when the denominator
has a minimum value. This happens when 3 .3 ^30']^ X (0.15)^
X .r
or
^2
{r-2Rf=0 or R=^ i3
1
1
U5
2

(2 6)^r/2 _6^ z= (4)^^^^ X 0.15 mm


P
= 1.5874 X 0.15 mm = 0.24 mm.
{r+rf
max
2r
PHYSICS-Xli

roblems For Practice 11. Three equal resistances connected in series across a
source of emf consume 20 W. If the same resistances

1. Calculate the current flowing through a heater are connected in parallel across the same source of
rated at 2 kW when connected to a 300 V d.c. emf, what will be the power dissipated ?
supply. [CBSE F 94 C] (Ans. 180 W)
(Ans. 6.67 A) 12. An electric heater consists of 20 m length of manganin
2. Calculate the amount of heat produced per second wire of 0.23 m^ cross-sectional area. Calculate the
(in calories), when a bulb of 100 W - 220 V glows wattage of the heater when a potential difference of
assuming tliat only 20% of electric energy is 200 V is applied across it. Resistivity of manganin
converted into light. / =4.2J cal"^ [Haryana 01] - 4.6 X 10~^Qm. (Ans. 10^ W)
(Ans. 19.05 cal) 13. A line having a total resistance of 0.2 Q delivers
3. An electric heating element to dissipate 480 watts 10 kW at 220 V to a small factory. Calculate tine
on 240 V mains is to be made from nichrome ribbon efficiency of the transmission. (Ans. 96%)
1 mm wide and thickness 0.05 mm. Calculate the 14. A motor operating on 120 V draws a current of 2 A.
length of the ribbon required if the resistivity of If the heat is developed in tine motor at the rate of
nichrome is 1.1 x 10~ ^ Qm. (Ans. 5.45 m) 9 cal s'’^ what is its efficiency ? (Ans. 84.425%)
4. 100 W, 220 V bulb is connected to 110 V source. 15. A 500 W electric heater is designed to work with a
Calculate the power consumed by the bulb. 200 V line. If the voltage of the line drops to 160 V,
(Ans. 25 W)
then what will be the percentage loss of the heat
developed ? (Ans. 36%)
5. How many electrons flow per second through an 16. A 50 W bulb is connected in a 200 V line. Determine
electric bulb rated 220 V, 100 W ?
the current flowing in it and its resistance. If 10% of
(Ans. 2.84 x lO'^) the total power is converted into light, then what will
6. An ammeter reads a current of 30 A when it is be tine rate of production of heat ?
-1
connected across the terminals of a cell of emf 1.5 V. Take / = 4.2 J cal
Neglecting the meter resistance, find the amount of (Ans. 0.25 A, 800 Q, 10.7 cal s"^)
heat produced in the battery in 10 seconds ? 17. Two bulbs rated 25 W, 220 V and 100 W, 220 V are
(Ans. 107.14 cal) coimected in series to a 440 V supply, (i) Show with
7. A coil of resistance 100 Q is connected across a necessary calculations which bulb if any will fuse.
battery of emf 6.0 V. Assume that the heat deve (//) What will happen if the two bulbs are connected
loped in the coil is used to raise its temperature. If in parallel to the same supply ?
the thermal capacity of coil is 4.0JK"\ how long [Ans. (0 25 W bulb will fuse
would it take to raise the temperature of the coil by (ii) Both the bulbs will fuse]
15°C? (Ans. 2.8 min)
18. A servo voltage stabiliser restricts the voltage
8. A generator is supplying power to a factory by output to 220 V ± 1%. If an electric bulb rated at
cables of resistance 20 D. If the generator is 220 V, 100 W is connected to it, what will be the
generating 50 kW power at 5000 V, what is the minimum and maximum power consumed by it ?
power received by the factory ? [Punjab 96 Cl
(Ans. 98.01 W, 102.01 W)
(Ans. 48 kW)
19. A room is lighted by 200 W, 124 V incandescent
9, Two bulbs are marked 220 V, 100 W and 220 V, lamps fed by a generator whose output voltage is
50 W respectively. They are connected in series to 130 V. The connecting wires from the generator to
220 V mains. Find the ratio of heats generated in the user are made of aluminium wire of total length
them. (Ans. 1 : 2) 150 m and cross-sectional area 15 mm“. How many
10. In a house having 220 V line, the following applia such lamps can be installed ? What is the total
nces are working: (i) a 60 W bulb (ii) a 1000 W heater power consumed by the user ? Specific resistance of
aluminium = 2.9 x 10~® Hm. (Ans. 12, 2.4 kW)
(in) a 40 W radio. Calculate (a) the current drawn by
heater and (6) the current passing through the fuse 20. Two wires A and B of same material and mass, have
line. their lengths in the ratio 1 : 2. On connecting them,
one at a time to the same source of emf, the rate of
50
Ans. (a) A (!j)5A heat dissipation in B is found to be 5 W. What is the
11
rate of heat dissipation in A ? (Ans. 20 W)
CURRENT ELECTRICITY 3.15 ■

21. An electric heater and an electric bulb are rated shown in Fig. 3.137. Also find the power dissipated
500 W, 220 V and 100 W, 220 V respectively. Both in the 6 Q resistor. HIT]
are connected in series to a 220 V d.c. mains. I-L 7Q
7 2n
Calculate the power consumed by (/) the heater and *—
71
(li) electric bulb. ICBSE D 97]
15 V,
[Ans. (i) 13.8 W(i/) 69.89 W]
22. The heater coil of an electric kettle is rated as 2000 W
0.5 Q

- J
3 60
1
in

at 200 V. How much time will it take to heat one AAV


7-7 1
80 10 O
litre of water from 20°C to 100°C, assuming that
entire electric energy liberated from the heater coil Hg. 3.137
is utilised for heating water ? Also calculate the (Ans. 1.0 A, 3.375 W)
resistance of the coil. Density of water is 1 g cm”^.
28. In the circuit shown in Fig. 3.138, the heat produced

w
(Ans.l68s, 20Q)
by 4 Q resistance due to current flowing through it
23. An electric kettle was marked 500 W, 230 V and was is 40 cal s”^ Find the rate at which heat is produced
found to raise 1 kg of water at 15° C to the boiling in 2 D resistance. (Ans.SOcals

Flo
point in 15 minutes. Calculate tire heat efficiency of 20 30
the kettle. (Ans. 79.3%) Wv—AW

ee
24. A 30 V storage battery is being charged by 120 V d.c.
supply. A resistor has been connected in series with

Fr
40 60
the battery to limit the charging current to 15 A.
AAA—VvV
Find the rate at which energy is dissipated in the
resistor. If the total heat produced could be made for
ur
Fig. 3.138
available for heating water, how long would it take
to bring 1 kg of water from 15°C to tire boiling 29. The 2.0 Q resistor shown in Fig. 3.139 is dipped into
point ? Specific heat of water = lcalg"^°C“^ and a calorimeter containing water. The heat capacity of
ks

(Ans. 1350 Js'\ 2644s) the calorimeter together with water is 2000 JK"^.
Yo

leal = 4.2 J.
(fl) If the circuit is active for 30 minutes, what would
oo

25. In the circuit shown in Fig. 3.135, each of the three be the rise in the temperature of the water ?
resistors of 4 Q can have a maximum power of 20 W
eB

(1?) Suppose the 6.0 D resistor gets burnt. What


(otherwise it will melt). What maximum power can would be the rise in the temperature of the water in
the whole circuit take ? (Ans. 30 W) the next 30 minutes ? (Ans.5.8°C,7.2°C)
r

6V in
4Q
ou
ad

iWV I AW
4f2
1 —^VVV—OB
Y

A9 ►
4n 6Q-
AW AAV
nd
Re

Fig. 3.135
AW
Fi

26. Find the heat produced per minute in each of the resis 20

tors shown in Fig. 3.136. (Ans. 360 J, 720 J, 540 J)

I, 60 Fig. 3.139
»—^^V
30. Three resistors I^, R, and each of 240 Q are
h 30 connected across a 120 V supply, as shown in
AAV
7;.
Fig. 3.140. Find (/) the potential difference across
9V 10
each resistor and (ii) the total heat developed across
the three resistors R1
-I AW
in 1 minute. AAV

Fig. 3.136 [Ans. (01^1= 80 V, 120 V R2


(ii) = V3 = 40 V
27. Calculate the current drawn from the battery of emf
(Hi) 2400 J]
15 V and internal resistance 0.5 Q in the circuit
Fig. 3.140
3J6 PHYSiCS-Xll

31. A heating coil is connected in series with a 4. Here P= 100 W, V = 220 V

resistance R The coil is dipped in a liquid of mass 220 x 220


Resistance of bulb, R = — = 484a
2 kg and specific heat O.Scal A potential P 100
difference of 200 V is applied and the temperature When the bulb is connected to 110 V source, the
of the liquid is found to increase by 60°C in 20
minutes. If R is removed, the same rise in
power consumed by the bulb is
temperature is reached in 15 minutes. Find the _ V'2 110x110
P'= = 25 W.
value of R. (Ans. 22.140) R 484
100x1
32. A series battery of 6 lead accumulators, each of emf 5. »= — = Z84xl0'*.
2.0 V and internal resistance 0.250 is charged by a e ~ Ve 220x1.6x10 -19
230 V d.c. mains. To limit the charging current, a 6. [f r is the internal resistance of the cell, then
series resistance of 530 is used in the charging s € 1.5
circuit. What is (/) power supplied by the mains / = or = 0.050
r I 30
(if) power dissipated as heat ? Account for the
difference in the two cases. INCERTl H =
rt (30)^ X 0.05 X 10 = 107.14 cal.
(Ans. 920 W, 872 W) / 4.2

33. A storage battery of emf 8 V, internal resistance 10, is 7. Heat required by the coil = Thermal capacity
being charged by a 120 V d.c. source, using a 150 X rise in temperature
resistor in series in the circuit. Calculate (/) the = 4.0x15 = 60)
current in the circuit, (n) terminal voltage across the Rate of prodxiction of heat,
battery during charging, and (Hi) chemical energy 6x6 -1
stored in the battery in 5 minutes. [CBSE 01, 08] = 0.36Js
R 100
[Ans. (/) 7 A, (;7) 15 V, (m) 16800 J] 60 J 60
.'. Required time = min
34. The following graph shows the variation of terminal 0.36 Js"^ 0.36x60
potentialdifferenceV, across a combinationof three
= 2.8 min.
cells in series to a resistor,
versus the current, i: 8. Here P= 50 kW =50x10'-'W, U= 5000 V
(i) Calculate the emf of Current supplied by generator,
each cell P 50x10^
= 10 A
(n) For what current i, will y 5000

the power dissipation Power wasted as heat dtiring transmission by


of the circuit be maxi cables of 20 Q resistance,
mum ? [CBSE OD 081
P'=!^R = {10f x20 = 2000W =2kW
(Ans. 2.0 V, 1.0 A)
Power received by the factory
HINTS = P'-P=50-2 = 48 kW.
220 X 220 220 X 220
] /-Z- _ 2000 W = 6.67 A. 9. R,= = 4840, R^ = = 9680
~ y ~ 300 V ~ 300 V 100 50

2. Power of bulb, P = 100 W


Ratio of heats produced when connected in series,

.●.Electric energy consumed per second = 100 J = 1:2


^2 ^ 968
Amount of heat produced per second p
1 _
1000 50
flO
10. (rt) Current drawn by heater = —
= 80% of 100 J = 80 J = -^ cal = 19.05 cal. y 220 ” 11
4.2

y2 y- 240 x 240 Current drawn by bulb = A - 220 111a


3. Power, P = .-. R= = 120 0
R P 480 P 40
Current drawn by radio = -2. - 1A
Area of cross-section of the ribbon, y 220 11
A = 0.05 mm“ = 0.05 x 10‘*'m“ (b) Current passing through fuse for the line
Required length. 50 3 2 ^ ^
11 11 11
RA 120x0.05 x10'^
/ = m = 5.45 m.
n X 10'*’ 11. Let Rbe the resistanceof each resistor and € the emf
P
of the source.
CURRENT ELECTRICITY 3.11

19. Resistance of aluminium wire,


For series combination : R^ = R+R+R = 3R
= 60 W.
_p/ 2.9x l0~^ xl50 = 0.29n
P = or 20 =
3R R
15x10"^
R.S
130 -124
For parallel combinatio)i : R^, = R / 3 Current from the main line =
0.29
= 20.69 A

200
F = = 3 x60 = 180 W. = 1.613 A
r73"~F Current through each lamp = 124
20.69
I = 12.83.
12. First find R = p — and then P = .-.No. of bulbs which can be used =
R
1.613
A
No. of bulbs that should be installed =12.
13. Let P be the power loss in the transmission line in
the form of heat. Then Power consumed = 12 x 200 = 2400 W = 2.4 kW.
10,000 20. As the Iw'o wires are of same material and mass,
P= I^R = R = xO.2

w
Vj 220 their volumes must be equal.
= 413.2 W = 0.4132 kW or rj| X1 = ^2 ^ 2/ or a^ = 2a2
If € is the emf of the source, then rate of heat
Efficiency of transmission.

Flo
Power delivered by line dissipation in wire B is
£- £2
^ Power supplied to line = 5 - o

ee
or

Power delivered 10 P ^2 / ^2

Fr
Power delivered + Power loss 10 + 0.4132
il2 = 10
or = 5 or Iv 12 = 2/1
= 0.96 = 96% p. 2/ / ^2 pi

14. Power supplied to line = V/ = 120 x 2 = 240 W for


Rate of heat dissipation in wire A is
ur
Power loss in the form of heat
_e-
= 9 cal s
-1
= 9 X 4.2 Js“‘ = 37.8 W —. 2^2 = 2 X10 = 20 W.
Rj p//rt, p/
ks
Power delivered by line = 240 - 37.8 = 202.2 W Proceed as in Example 112 on page 3.67.
Yo
21
oo

Power delivered by line _ 202.2 22. / =


P 2000
= 10A
Efficiency, r\ = 240 V ^ 200
Power supplied to line
eB

Vlt 200xl0xf
cal
= 0.8425 = 84.25%. Heat produced in time t = / 4.2

15. Here P = 500W, P = 200V


Heat gained by water = »ic0 = 1000 x 1 x 80 cal
r
ou

200x200
ad

R = = 8on 2000 f
500 = 1000x80
P
4.2
Y

When the voltage drops to 160 V, rate of heat 1000x80x4.2


= 168 s.
or t - -
production is 2000
nd

f2
V 160x160
Re

= 320W
F =
R 80 R =
V- _ 200x200- = 20Q.
T~ ~2000
Fi

% Drop in heat production


P-F 180 X100 23. Heat absorbed by water
xl00 = = 36%.
P 500 = 1 X 4200 X (100 - 15) = 4200 x 85 J
17. Proceed as in Example 117 on page 3.69. Heat produced by electric kettle
(220)^ = Pf = 500 X 15 X 60 ]
18. Resistance of the bulb, R= = 484n
100 4200 X 85
X 100 = 79.3%.
Heat efficiency = 500 X 15 X 60
Variation in voltage = ± 1% of 220 V - ± 2.2 V
= 220 - 2.2 = 217.8 V 120-30
Minimum voltage = 15
24. Charging current, / = R
(217.8)^ = 98.01 W. 90
Minimum power 484 R = = 6H
.-. Series resistor.
15
Maximum voltage = 220 + 2.2 = 222.2 V Rate of energy dissipation in the resistor,
(222.2)^ = 102.01 W.
Maximum power 484 P=;^R=(15)^ x6 = 1350 Js"\
PHYSICS-XII

Heat produced in resistor in time t - Heat absorbed Current flowing through the lower arm.
by water i x5 _ /
1350 X f = 1 X 4200 X (100 - 15) h ~ 5+10”3
4200 X 85
or i ~
1350
= 264.4 s. Heat produced per second in 2 n resistor, ^Pcclf x2
Heat produced per second in 4 Q resistor, cc x4
25, Let / be the current through a resistance of maximum
power 20 W. Then
_ l^x2^(2II3fx2
I^R = 20 or /^x4 = 20 or = o
P2 /|x4 (l/3)x4
Effective resistance between A and C,
4x4
or
i] =2^2 =2x40 = 80 cal
R' = + 4=2+ 4=6n 6x2
4 + 4 29. (a) Total resistance in the circuit = + 1=
6+2 2
The maximum power that can be dissipated by the 6V
circuit. Total current, 1=
P= f^R' = 5x6 = 30W. (5/2)n 5

26. The equivalent resistance of the circuit is Current through 2Q resistance


6x3 12 6
R = + 1=2+ l = 3f2 = X = 1.8 A
6+3 5 6+2

Current drawn from tlie battery is Heat produced in 2Q resistance in 30 minutes


9V
/ =
3Q
= 3A = (1.8)^ x2x30x60= 11664J
Rise in temperature
As the current through 1Q resistor is 3 A, so heat 11664J
produced in this resistor in 1 minute (or 60 s) is 2000JK
= 5.8Kor 5.8° C.

H = /^ Rf = 3^ X 1 X 60 = 540 J
Current through 6fi resistor, (b) When the 6Q resistor gets burnt.
6V
/,1 =
-^x3=lA
6+3
Current =
(2+l)a
= 2A

Heat produced in 2Q resistor in 30 minutes


Heat produced in 6Q resistor
= (2)“ x2 x30x 60= 14400 J
= 1^ X 6 X 60 = 360 J
Rise in temperature
Current through 30 resistor,
14400J
/2=f~/i=3-l = 2A -1 = 7.2Kor 7.2° C.
2000JK
Heat produced in 30 resistor 30.
(0 Total resistance of the circuit.
= 2“ x3x60 = 720 J. 240 X 240
27. The distribution of current is shown in Fig. 3.137. R=R^ + = 240 + = 3600
R^ + R^ 240 + 240
Applying Kirchhoff's second law to the loops 1 and
2, we get Current drawn from the battery,
(/-i,)x(7+ 1+ 10)- /j x6 = 0 / =
and R 360 3
x6+ / x(8+ 0.5+ 2) = 15
On solving the above two equations, we get P.D. across ^ = R, f = 240 x ^ = 80 V.
/, =0.75A and /=1.0A As R2 = R^,
Power dissipated in the 60 resistor which carries so current through each of these resistors
current /,1 is
-Ma
P= I^R=(0.75f X 6= 3.375 W. 2 3 6

28. Resistance of the upper arm = 2+3 = 50 P.D. across R^or R^ ,


Resistance of the lower arm = 4+6=100 V^2 = V3 = 240 X i = 40 V.
Let I be the total current in the circuit. Tlnen current (h) Total heat developed in three resistors in
flowing through the upper arm will be 1 minute.
fxlO 2/
i
1 “
H=l^Rt = -
n'
5+ lO'T X
360 X 60 = 2400 J.
CURRENT ELECTRICITY 5.79

The difference : 920 - 872 = 48 W, is the power


31. Here/» = 2kg =2000g,c = 0.5calg *“C ^0 = 6O‘’C
stored in the accumulator in the form of chemical
/j = 20 min, = 15 m, K = ?
energy of its contents.
.'. Heat gained by liquid 33. Totalemf=I20-8=112V
H = me 0 = 2000 x 0.5 x 60 = 6 x lO"* cal
Total resistance = 1 + 15 = 16 Q
= 6x10'* X 4.2 J =2.52x10^ J Total omf 112
= 7 A.
Let r be the resistance of the healing coil. In the first (i) Current, I = Total resistance 16
case, the resistance R is in the ciraiit.
V (/;■) Terminal voltage during charging,
Current, / =
R + r V =t+ /r = 8+ 7x1 = 15 V.

V
(iii) Chemical energy stored in the battery in
5 minutes
Heat dissipated in time f,. H,= rf
1
(R^r_ = €/f =8 x7 x(5x60) = 16800 J.

w
34. (i) Total emf the three cells in series
In the second case, the resistance R is removed. = P.D. corresponding to zero current = 6.0 V
y
Current, / = EMF of each cell = 6.0

Flo
r
(ji) When ( = 1.0 A, V = 3.0 / 3 = 1.0 V
V ^2
E-K 2.0-1.0
Heat dissipated in time fo/ ''^2 “ = l.OQ

ee
r =
rj r
/ 1.0

Fr
As the liquid is raised to same temperature in both The output power is maximum, when
cases, so external resistance = internal resistance = 3r
H = Hj = H2 Total emf 3E I
/
for
ur
/ \2
max
Total resistance 3r + 3r 2r
V
or rt
i “ 2.0
R+ r = 1.0 A.
\ /
2x 1.0
ks
r'
Yo
or
{R + r)‘ ~ f.t ” 20 4
oo

3.31 KIRCHHOFF'S LAWS


r V3 R+r 2
Introductory' concepts. In 1842, a German physicist
eB

or or
R+r 2 r 'V3
Kirchhoff extended Ohm’s law to complicated circuits
or
R
+ 1=4 and gave two laws, which enable us to determine
s
current in any part of such a circuit. Before under
r
r

R
ou

R
standing these laws, we first define a few terms.
ad

or ■- = 1.155- 1 = 0.155 0,,=--


r
1. Electric network. The term electric netiuork is used
Y

As H= H2 for n compticoted system of electrical conductors.


2.52x10^ =
(200)“ X 15 X 60 X 0.155 2. Junction. Any point in im electric circuit where two
nd
Re

R or more conductors are joined together is a junction.


4 X 10^ X 15 X 60 X 0.155 3. Loop or Mesh. Any closed conducting path in an
Fi

or R = = 22.14 n.
2.52x10^ electric netivork is called a loop or mesh.
32. EMF of the battery = 6 x 2.0 = 12 V 4. Branch. A branch is any part of the netzoork that lies
Internal resistance of the battery = 6 x 0.25 = 1.5 Q belzveen "two junctions.
Total resistance = 1.5 + 53 = 54.5Q 56. State the tzvo Kirchhoffs laws for electrical
circuits and explain them gizjing suitable illustrations.
Charging current
230 -12
Also state the sign conventions used.
Effective emf
54.5
= 4.0 A Kirchhoff's first law or junction rule. In an electric
Total resistance
circuit, the algebraic sum of currents at any junction is zero.

(/) Power supplied by the mains Or, the sum of currents entering a junction is equal to the
= VI = 230 X 4.0 = 920 W. sum of currents leaving that junction.
(ii) Power dissipated as heat Mathematically, this law may be expressed as
I f=0
= 1^ {R+r) =(4)“ X (53 + 1.5) = 872 W.
■'3.80 PHYSICS-XII

Sign convention for applying junction rule : I +


WV-*”
1. The currents flowing towards the junction are
\^= + !R
taken as positive.
2. The currents flowing away from the junction Fig. 3.145 Positive potential drop across a resistor.
are taken as negative. 5. The IR product is taken as negative if the
resistor is traversed in the opposite direction of
Figure 3.142 represents a assumed current.
junction / in a circuit where I + I
four currents meet. The —*—Wv-^
currents and flowing .^1 / V=-IR
towards the junction are ►rWV-
h h
positive, while the currents ,. Fig. 3.146 Negative potential drop across a resistor.
and flowing away from Illustration. Let us consider the circuit shown in
i
the junction are negative, Fig. 3.147.
therefore, by junction rule : ^^3
Fig. 3.142 Junction rule :
D WV £
S /= 0
or
- ^3 - =0
/j + Fg =-fs + ^4.
kg
0 ^3
or
/j + ^2 = ^3 + h
c ^AA>—*— I F
i.e., Incoming current = Outgoing current
First law is also called Kirchhoff s current law (KCL).
Justification. This law is based on the law of k,
0 1

conservation of charge. When currents in a circuit are B' VVV 11 'A


steady, charges cannot accumulate or originate at any Fig. 3.147 An electrical circuit.
point of the circuit. So whatever charge flows towards
In Fig. 3.147, traversing in the clockwise direction
the junction in any time interval, an equal charge must around the loop ABCFA we find that:
flow away from that junction in the same time interval.
Kirchhoff's second law or loop rule. Around any Algebraic sum of current resistance products
closed loop of a network, the algebraic sum of changes in = /jKj - J2R2
potential must be zero. Or, the algebraic sum of the einfa in Algebraic sum of emfs =6j -€2
any loop of a circuit is ecjual to the sum of the products of Applying KirchhofPs loop rule to closed path ABCFA,
currents and resistances in it.
we get =
Mathematically, the loop rule may be expressed as
Similarly, applying Kirchhoff's second rule to mesh
2 Ay = 0 or I^ = Z IR CDEFC, we get
Sign convention for applying loop rule : S2 = /2i?2 + (/, + J,)K3
1. We can take any direction (clockwise or anti- Second law is also called Kirchhoff's voltage law (KVL).
clockwise) as the direction of traversal. Justification. This law is based on the law of
2. The emf of cell
is taken as positive if the conservation of energy. As the electrostatic force is a
direction of traversal is from its negative to the conservative force, so the work done by it along any
positive terminal (through the electrolyte). closed path must be zero.
i
Examples based on
Kirchhoff's Laws
v=-^

Fig. 3.143 Positive emf. Formulae Used


Fig. 3.144 Negative emf.
1. 2 1 = 0
(Junction rule)
3. The emf of a cell is taken as negative if the
direction of traversal is from its positive to the or Total incoming current = Total outgoing current
2. 2 €=2 IR
negative terminal (Fig. 3.144). (Loop rule)
4. The current-resistance (IR) product is taken as Units Used
positive if the resistor is traversed in the same
direction of assumed current. Current / is in ampere, resistance k in ohm and
emf t in volt.
CURRENT ELECTRICITY

Example 137. Network PQRS (Fig. 3.148) is made as Solving equations (1), (2) and (3), we get
under : PQ has a battery of 4V and negligible resistance
with positive terminal connected to P, QR has a resistance of 1 -^A / - 18 A, /3 = 66 A.
31 31 31
60 fi. PS has a battery of5V and negligible resistance with
positive terminal connected to P, RS has a resistance of Example 139. Find the potential difference across each cell
200 Cl. If a milliammeter, of 20 Cl resistance is connected and the rate of energy dissipation in R. [Fig. 3.150(a)].
between P and R, calculate the reading of the milliammeter. [CBSE SP 11]
(NCERT)
200 n gi = 12V r^ = 2Q
s I R
wv-
I
5V R=4n
WV "
6on

w
62 = 6 V r, = m
P I
4V
Q 1 V/v '

Flo
Fig. 3.148 Fig. 3.150 (a)

Solution. Applying Kirchhoff's second law to the Solution. Applying Kirchhoff's laws.

ee
loop PRQP, we get For dosed loop ADCBA

Fr
20/^ +60/ = 4 ...(0 12 =4(I^ + ...(0
Similarly, from the loop PSRP, we get For dosed loop ADEFA,
200(/-i^)-20/i =-5 for
ur
6 = 4(I^ + I^)+I^=4I^+51^ ...(»)
or
40/-44/^ =-l ●●●(»)
Multiplying (i) by 2 and (it) by 3, we get ei = 12V ri=20
ks
120J+40/j =8 ...(nO B 1 c
Yo

I1
oo

and 1207-132 =-3 ...(iv)


R=4a
Subtracting (iz;) from {in), we get
eB

A -Wr- D

1727j =11
11 = r2=in
or
^1 = = 0.064 A
1 MV E
r

172
h
ou
ad

Thus the milliammeter of 20 Q will read 0.064 A.

Example 138. Using Kirchlvff'slaws in the electrical network


Y

Fig. 3.150 (b)


shown in Fig. 3.149, calculate the values of f, I2 and ly
[CBSE D 2000C] Solving (i) and (ii), we get
nd
Re

18
A
o-
B
—A and L = ^A
7 ^ 7
Fi

I'O 0
a P.D. across R = V
5fl
a h CO h
= (J, + /2)R
C>1

12 V

I 18-6^1 48
X 4 volt = — volt
F E D 7 J 7
48
Fig. 3.149 P.D. across each cell = P.D. across R = V
7
Solution. Applying Kirchhoff's first law at junction B,
Energy dissipated in R = 4 O resistor
Ij + iz = 73 ...(1)
^2
12
Applying Kirchhoff's second law to loops ABEFA — x4J
and BCDEB, we get 7 J
2/3 + 57, = 12 ...{2) 576
J = 11.75 J.
-2l3-37z = -6 ...{3) 49
3.82 PHYSICS-XII

'I Example 140. Two cells of emfs 1.5 V and 2.0 V and ^|-2V rj = 4n
internal resistances 1Q and 2 H respectively are connected t ■WV—►
in parallel so as to send current in the same direction
through an external resistance of 5 H. ICBSE OD 05] I, ^2=1V r2 = 3Q
t AVV—►
{/) Draw the circuit diagram.
(») Using Kirchhoff's laws, calculate
(a) current through each branch of the circuit,
J3 ^3 = 4V ^3=20
<■ AVv
3
(b) p.d. across the 5 Q resistance.
Fig. 3.152
Solution. (/) The circuit diagram is shown in Fig. 3.151.
Solution. By Kirchhoff's junction rule,
5fi
/, + /2
E9- AW D ^3 ^ ^ ^2 ...(/;
From upper loop.

1 n
0 3/2-4/^ =2-1 = 1 ...{ii]
l-WW < >C From lower loop,
Ii !1
1.5 v = E i
3/2+213=4-1=3 ...{Hi]
2Q 0 On solving equations (i), (//) and (Hi), we get
AO * -l-ww <—oe
2 7 9
2.0 V = 62 /,=—A, L= — K L=~A.
’ 13 ^13 ^13

Fig. 3.151 Example 142. Apply Kirchhojfs rules to the loops ACBPA
and ACBQA to write the expression for the currents /j, /j
(ii) (a) Let f and be the currents as shown in and in the network shown in Fig. 3.253. [CBSE OD 10]
Fig. 3.151. Using Kirchhoff's second law for the loop Solution. By Kirchhoff's €,=6V
AFCBA, we get
junction rule,
2/2 -1/, =62 -^1 =2 -1.5 ...(/) B
or
2 /2 - /j = 0.5 ...(1)
From loop AQBPA
For loop CFEDC, we have
1/j +5(/j + = =1.5 0.5/j-/2=6-10 = -4 ^3
62 = 10V
...(ii)
or
5/2 +6/j =1.5 ...(2) C

Solving equations (1) and (2), we get From loop ACBPA R = i2n

/
12/3+0.5/j =6
1 “ -A,
34
/2=—A
2 34 ...(Hi) Fig. 3.153

.'. Current through branch BA, On solving equations (/), (ii) and (Hi), we get
±A 84 106
I
22
/i=-—A, L = A /^ =—A
I 34 ^ 37 2 37 ^ 37
)

>
Current through branch CF, Example 143. Use Kirchhojfs rules to detemine the
Aa potential difference between the points A and D when no
34
current flows in the arm BE of the electric network shown in
Current through branch DE, Fig. 3.154(a). [CBSE OD 15]
10 30 E D
f + l2~ A. F| VW—f-
34
1 v-^
(1?) P.D. across the 5 Q resistance R I R

= (/, + /2)x5=—x5
34
V = 1.47V. in
.^3V
Example 141. Use Kirchhojfs rules to write the A I-
C
B
expressions for the currents /j, I2 and in the circuit 6V 4 V

diagram shown in Fig. 3.152. [CBSE OD 10]


Fig. 3.154 (a)
CURRENT ELECTRICITY 3.83

Solution. No current flows through the arm BE. Solution. Let and be the currents as shown

Let 1 be the current along the outer loop as shown in Fig. 3.156. Kirchhoff's second rule for the closed
in Fig. 3.154{&). loop ADCA gives
3Q E
F -VVN—f
I D
10-4(1,-/2)+2(/2+/3-Ji)-/,=0
/, or
7/, - 6 Ij - 2 /3 - 10 ...(1)
IV R1

JO
R For the closed loop ABCA, we get
2C1
4i.3V I
10-4/2-2(/2 + Z3)-/,=0
I * I C
or
+6/2+2/3 = 10 ...(2)
B
6V 4V For the closed loop BCDEB, we get
Fig. 3.154 (b) 5-2(/2+/3)-2{/2+/3-/i) = 0

w
Applying Kirchhoff' loop rule to the loop AFEBA, or
2/,-4/2-4/3 = -5 ...(3)

(2+3)/+K,xO = l+3 + 6 On solving equations (1), (2) and (3), we get

Flo
7 = 2A 7
I = 2.5 A, L=- A, L = 1- A
From A to D along AFD,
1
' 2 g ' 3

ee
yAD = 2x2-l + 3x2 =9 V. The currents in the various branches of the network
are :

Fr
Example 144. In the circuit Fig. 3.155, assuming point A
= Aa
to be at zero potential, use Kirchhojfs rules to determine the
potential at point B.
I
AB ~
I A; /c,=2U; / DEB
8

for
ur
4V = 0; I = 2-A.
lA D 3A CD BC
8 2
I I fB
Example 146. In the circuit shown in Fig. 3.157(a), E, F,
ks
R in G and H are cells of emfl V, 1 V, 3 V and 1 V, and their
Yo

internal resistances are 2 Q, 10, 3 Q and 1 £1, respectively.


oo

2V
3 A
4- i I Calculate (i) the potential difference between B and D and
eB

A C (ii) the potential difference across the terminals of each of the


cells G and H. [CBSE 18C]
Fig. 3.155
+
r

A B
Solution. From the loop BDCR.^B, we get
ou
ad

+
2x2+3R,=4 or R,=0
Y

^s = ^c=2 + V^ =2+0 = 2 V.
nd

D C
Example 145. Determine the current in each branch of the
Re

network shown in Fig. 3.156. [NCERTl


Fig. 3.157 (a)
Fi

Solution. In Fig. 3.157(b), the network has been


4£1
^3 redrawn showing the emfs and internal resistances of
-t. the cells explicitly.
in
2a I
la E B
I / A
A' —'»c 5V

^1-^2
k.10 V la
C A
0- H.,
IV

c
I
IV-^i-F hj
in la
4a + h

D
D *^\
h
Wv C
3V 3a

Fig. 3.156 Fig. 3.157 (b)


3.84 PHYSICS-XII

(i) Applying Kirchhoff s first law at junction D, Applying Kirchhoff's second law to loop BCDB,
we get we get
7= 7,+ 7, ●●●(0 +yx3-/^x4=0
Applying Kirchhoff s second law to loop ADBA, we 57
1
37-9/
8
=0 ...(2)
get Applying Kirchhoff's second law to loop ADCEA,
27+ 7 + 27j=2-l we get
or
37 + 27^ =1 ...(//) 2(7-7,) + 3(7-7j + 7p + 27=2
Applying Kirchhoff s second law to loop DCBD or
-57,+77 + 37^ = 2 ...(3)
372 + 72 -27j=3-1 Adding (2) and (3),
or
472 -27j =2 ...(Hi)
47-67 =2 ...(4)
On solving equations (/), (») and (Hi), we get 8

Multiplying (1) by 5 and (2) by 3 and subtracting,


7, 4 7, =— Aand 7=— A
^ 13 2 13 13 we get
-7 + 477 =0 7 = 477
P.D. between the points B and D, 8
or
8

V =7, x2=—V. From (4),


^ ’ 13 4x 477 -67 =2 or 182 7 =2
8 8 o

{«) P.D. between the terminals of G (giving current),


7 ^=^A.
K = S-7-r = 3- —x3 = 1.615 V 8 182 91
^ 2 13
Example 148. The four arms of a Whetstone bridge
P.D. between the terminals of H (taking current), (Fig. 3.159) have the following resistances :
K, = 6' + 7/ =1 + ~x 1 = 1.46 V. AB = 100a, BC=10Q, CD = 5Q and DA =60 0.
A galvanometer of 15 O resistance is connected across
Example 147. In a Wheatstone bridge, P = lf2, BD. Calculate the current through the galvanometer when a
Q=2 n, 7? =2 n, S=3Q and R^=4Q.. Find the current potential difference of 10 V is maintained across AC.
through the galvanometer in the unbalanced position of the [NCERT ; CBSE F 20)
bridge, when a battery of 2 V and internal resistance! Q. is
used.

Solution. The circuit for the given Wheatstone


bridge is shown in Fig. 3.158. Let 7, 7, and 7^ be the
currents as shown.

/ 1

Fig. 3.159

/ / Solution. Applying Kirchhoff's second law to loop


BADB, we get
1007, +157^-6072 =0
2n
or
207,+37^-12/2=0 ...(1)
Fig. 3.158
Considering the loop BCDB, we get
Applying Kirchhoff's second law to loop ABDA,
we get.
10(7,-7^)-157^-5(72+7p=0
7, xl+7^x4-(7-7,)x2=0 107,-307^-572 =0
3L1 -27 + 47 =0
or
8
...(1) 27,-67^-72=0 ...(2)
CURRENT ELECTRICITY

Considering the loop ADCEA, we get X r, + T, X + I2) R + X =€j


60/2 + 5(/2+ ^g) = 10 /j x2 + x2 + (/j + /2)x8 + x3=1.5
15/j+8/2=1.5 ●●●(0
65/2 + 57^ = 10
13/2 +7^=2 ...(3) Applying Kirchhoff's second law to the loop
Multiplying Eq. (2) by 10, we get CDEFC, we get
20/1
607^-10/2=0 ...(4) X + i2 ^ + ^2) ^ ^ ^2 ^ -^4 ^ ^2
From equations (1) and (4), we get /2XI + 7^x2 +{i^ + /2)x8+
8/1+14/2=2
637^-2/2=063 ...(»)
or 4/j +7/2=1
/2 = -/^,=31.5/^ On solving equations (i) and {ii), we get
Substituting the value of /2 in Eq. (3), we get 18

w
A
h- 146
and I2 = 146
13 (31.5/p+7^=2
or 410.5/ =2 Current through the 8 resistance wire is

Flo
or I A = 4.87 mA. /, + /, = — i^=i^A
S 410.5 146 146

ee
P.D. across the ends of 8D resistance wire
Example 149. Two cells of emfs 1.5 V and 2 V and
23
internal resistances 2 D and 1Q respectively have their

Fr
X 8 = 1.26 V.
negative terminals joined by a wire of 6 .Q and positive 146

terminals by a wire of 4 Q resistance. A third resistance wire ^ 50. aB, BC, CD and DA are resistors of I 1,2
0/8 a coHMecfs middle points of these wires. Draw the ^^^^iDrespectively connected in series. Between A and C is
for
ur
circuit diagram. Using Kirchhoff laws, find the potential ^ ^ of resistance 2 Q, A being positive. Between B
difference at the end of this third wire. [CBSE D 2000C] Disa2V cell ofl Q resistance, B being positive. Find
ks
Solution. As shown in Fig. 3.160, the positive current in each branch of the circuit.
Yo

terminals of cells and E2 ^^e connected to the wire Solution. The circuit arrangement and current
oo

AE of resistance 4 Q and negative terminals to the wire distribution is shown in Fig. 3.161.
BD of resistance 6D. The 8Q wire is connected
eB

B
between the middle points F and C of the wires AE and /,-/2
BD respectively. /i

R, = R,=|=2n
r

1 n in
2v-i-i n
ou
ad

D
O

R3 = R4=|=3n
<c
and
Y

h 2n /a-/, 2n
The distribution of current in various branches is 1 V o
^3-^2
nd
Re

shown in Fig. 3.160. F6- l- ■6e


2Q
r, = 20
Fi

/1
At V -AAV B Fig. 3.161
6, = 1.5 V Ra = 3n
R, = 2Q Applying Kirchhoff's second law to loops BADB,
7,+ 7, 7, + /2 ,1^1 BCDB and ADCEFA, we get
AW >■ C
R = 8Q 1.72+2./3+I.7^=2
72
R2 = 2Q f + I2+2 1^=2 or ...(1)
84*30
r2 = in or
1(/,-/2)-2(73-/,)+/i=2
I- AW D
or
47^-/2-273=2 ...(2)
6,»2V
and 2 73 + 2 (73 -/j) + 2 {73 - 72) = 1
Fig. 3.160 or
-2/1-272+6/3 = 1 ...(3)

Applying Kirchhoffs second law to the loop Solving equations (1), (2) and (3), we get
ABCFA, we get 7j=0.8A, 72 =0.2 A and 73 =0.5 A
PHYSICS-XII

Currents in different branches are Example 152. Two squares ABCD and BEFC have the
^AB = ^2 “ ' side BC in common. The sides are of conducting wires with
I resistances as follows : AB, BE, FC and CD each 2 Q; AD,
BC = - I2 = 0.6 A;
BC, EF each lO.. A cell ofemfl V and internal resistance
1.CD = - /3 = 0.3 A; 2 n is joined across AD. Find the currents in various
I
AD = 73 = 0.5 A; branches of the circuit.
^EF ~ h~ h~ Solution. The current distribution in various branches
of the circuit is shown in Fig. 3.163.
Example 151. Find the equivalent resistance between the
I A I1 B h
terminals A and B in the netivork shown in Fig. 3.162. I—►—9—►—^ vvv -E
Given each resistor R is of 10 El. 1-1 1
2f2 2n
h
I K I, L I,
Ap. AA-V—0 AAA—oM 2V-=t-2n in in in
R R
I-I 1 ' 7,-/2 h 2n I1 2n
h
R

1-1 1
0 R
0 R D
AAA < ^—AAA
C
Af

Fig. 3.163
p». » \A/v—o » AAA ■9B
R O R N

o Applying Kirchhoff's second law to the loop


containing the cell and AD, we get
2x 7 + lx(/-7j) = 2
or
37-7j = 2 ...(1)
Fig. 3.162
From the loop ABCDA, we get
Solution. Imagine a battery of emf S, having no 2 X 7, +lx(7j -/2) + 2 X 7^ -lx (7-7^)=0
internal resistance, connected between the points A
and B. The distribution of current through various
or
- 7 + 6 7^ - 72 = 0 ...(2)
branches is as shown in Fig. 3.162. Similarly, from the loop BEFCB, we get
Applying Kirchhoff's second law to loop KLOPK, 2 X 72 + 1x 72 + 2 X /2 -lx (7j -72)=0
we get or
-7j +6 72 =0 ...(3)
7i i? + (7, -72) K-2(7-7j) R = 0 Solving equations (1), (2) and (3), we get
or
47,-72=27 ...(1)
7 =
70 12 2
A, L=—A, L=—A
Similarly, from the loop LMNOL, we have 99 1 99 ' 2 99
Currents in different branches are
12 2
or
-7i+472=7 ...(2) ^AB=^-CD ^^ he = hr
From the loop AKPONBEA, we have
2(7-7,)R + {7-72)7^ = € ...(3)
7
AD = l-h=^A,
99
7
BC
= 7
1
2 99
Solving equations (1) and (2), we get 70
Current through the cell = 7 = — A
3/ and 99
^-5
Example 153. Two points A and B are maintained at a
Substituting these values in equation (3), we get
3 ^ f 2 ^ constant potential difference of 110 V. A third point is
2 7- - 7 R+ I-~ I R = E connected to A by two resistances of 100 and 200 Q in
5 5 J
parallel, and to B by a single resistance of300 .O. Find the
or -/R = E
5
...(4) current in each resistance and the potential difference
between A and C and between C and B.
If R' is the equivalent resistance between A and B,
then Solution, The circuit arrangement and the current
7K'=€ distribution is shown in Fig. 3.164.
...(5)
From (4) and (5), IR' = — IR
Applying Kirchhoff's second law to the loop
5 DEFGHID, we get
or
7
R' = - R = -xl0=14Q.
7,xl0O-(7-7,)x2OO = 0
5 5 or 300 I1 -200 7=0 ...(1)
CURRENT ELECTRICITY

C
F C I
O'

4^^ O
1

D, /
I
l~l i
1 110 V t— B
/ I

Fig. 3.164

Similarly, from loop ADIHGCBA, we get


(/-/^)2004-/x300=110
500 i-200 L1 =110 ...(2)

w
or

Solving equations (1) and (2), we get


1 =— A and The equivalent resistance of the network is

Flo
10
R' =
Total emf 6 SIR =—5 K^
.●. Current through 100 H resistance Total current 6 I 61 6

ee
But R=1Q

Fr
R' = -n
Current through 200 Q resistance 6
= 1-1
1 “
JLa Total current in the network is
10
for
ur
, - 6 10
Current through 300 Q resistance
6f = —
R' y=12A or /=2A
= / = ^A 6
ks
10
AA' D'C' - - fee- -2f-4 A
Yo

P.D. between A and C = P.D. across 100 D resistor


oo

I
BC
= I
DC ~ ^DD' ~ ~ ^AV ^A’B' = / = 2 A.
= L1 xlOO=-xlOO = 20V
eB

5
Example 155. Twelve wires each having a resistance ofrCl
P.D. between C and B = P.D. across 300 Cl resistor are connected to form a skeleton cube; find the resistance of
= 7x300 =~x300=90 V.
the cube between the two corners of the same edge.
r

10 Solution. Let a current x + 2y enter the junction A


ou
ad

Example 154. A battery o/lO V and negligible internal of the cube ABCDEFGH. From the symmetry of the
Y

resistance is connected across the diagonally opposite corners parallel paths, current distribution will be as shown in
of a cubical network consisting of 12 resistors each of resistance Fig. 3.166. H G
1D. Determine the equivalent resistance of the network and
nd
Re

2(y-z)
the current along each edge of the cidie. [NCERTl
Fi

y-2
Solution. Let 6/ be the current through the cell. E
F
Since the paths AA', AD and AB are symmetrically ’ y-2
placed, current through each of them is same, i.e., 21. At y
the junctions A', B and D the incoming current 27 splits ●/. Dy ►
z
c
equally into the two outgoing branches, the current y /
y
y
through each branch is 7, as shown in Fig 3.165. At the X

junctions J?, C and D', these currents reunite and the A B


x + 2y
x^ly
currents along B*C', D'C' and CC are 27 each. The total
current at junction C' is 67 again. Fig. 3.166
Applying Kirchhoff's second law to the loop Applying Kirchhoff's second law to the loop
ABCC' EA we get DHCCD, we get
-2 7R-77?-2 IR + t=0 g=5 IR
or
(y-2)r + 2(y-2)r+(y-2)r-zr=0
where R is the resistance of each edge and 6 is the emf or 4y r - 52 r = 0 or 5z = 4y or z=
4

of the battery. 5^
3.88 PHYSICS-XII

Applying Kirchhoff's second law to the loop ABCDA, or


-4AT + 5y=0
we get 4
or
xr-yr-zr-yr = 0 ^ 5
or
x-2y-z = 0 4 14r
lxr-¥- xr = 2x R or =2R

^-2y-|y=o
4 ^ 5 5
or

Hence R = lArCl.
14 5
or X =
y=7T^ Example 157. Tioelve wires each having a resistance of IQ
14
are connected to form a cube. Find the resistance of the cube
Let R be the resistance across AB. Then between two corners of a diagonal of one face of a cube.
P.D. across AB = xr Solution. Imagine a battery connected between
10 12 points A and C so that a current of 1 A enters junction
or + —X R = xr or — R =r
14 7 A. This current is divided equally along AB and AD.
The distribution of current in various branches is
7
Hence R = ~rQ.
12
shown in Fig. 3.168. These currents finally add so that a
current of 1 A flows out of junction C.
Example 156. Eleven equal wires each of resistance r form
£ i-2x-z H
the edges of an incomplete cube. Find the total resistance
from one end of the vacant edge of the cube to the other. /
l-2x
y

Solution. Let A and B be the vacant edges of the


cube. Let an emf € applied across AB send a current A X B
\-2x-z + y
in the circuit. Since the paths AD and AE are symme x~y
trical, the current 2x at A is divided into two equal X. F'.
C
parts X and x. At other points, the current is divided as y
shown in Fig. 3.167, so that again the currents combine x-y l-2x-2y
at B to give current 2x. Let R be the total resistance of D
the cube between A and B.
'A*.A
H 2(^-y) G
x-y
Fig. 3.168
y
£
F
l^-y x-y Applying Kirchhoff's second law to the loop
X
AEFDA, we get
X D>
C -{l-2x)-z + y + X-0 ...(1)
X ' y
X

2x 2x Similarly, from the loop BHGCB, we have


A B
6 -y - {l-2x ~z + y)-{l-2x + 2y) + {x ~y)=0
2x
V ...(2)
Again, from the loop FGCDF, we have
Fig. 3.167
-iy + z)-0-~^x + 2y) + (x-y)-y=0 ...(3)
Applying Kirchhoff's second law to the loop,
ABCDA, we get On solving equations (1), (2) and (3), we get
xr+ yr + xr=t
From Ohm's law.
X =
I A, y=0, z 1a
8

Now
E=2x. R
2xr+ yr = 2xR = lx? +1X 3- = i=iv
8 8 8 4
Applying Kirchhoff's second law to the loop
EFGHE, we get .●. Equivalent resistance between A and C,
yr-{x-y)r-2ix~y)r-{x-y)r=0 V
AC _ 3/4 3
R = =-n.
or
y-x + y-2x + 2y-X + y =0 I 1 4
CURRENT ELECTRICITY 3.89

Example 158. In the netzoorkas shoxun in Fig. 3.169, each Solution. Suppose a cell of emf 6 is connected
resistmice r is of 2 Q. Find the effective resistance bctioeen between A and B. Then the given circuit can be
points A and B. represented by an unbalanced Wheatstone bridge as
shown in Fig. 3.172. The distribution of current is also
shown.

h-h
r
o R
I1
h

G G
A B

Solution. The distribution of current is shown in /-/ 1


r

Fig. 3.170(fl). By symmetry, current in arm AE = current R


t-h + h "!
in arm EB. As the current in arm CE is equal to the

w
C h-h D

r
h
r
r
6

Flo
r r
h h Fig. 3.172
I1 r II r h^h

ee
O- -WV
A E B Applying Kirchhoff's second law to the loop 1, we

Fr
Fig. 3.170 (a) get
fr+ l^r-{I-l^)R=0
current in arm ED, so the resistance of the network will
ij( r + R)+ f2 / R =0 ...(1)
for
ur
not be affected if the wire CED is disconnected from or
the wire AEB at the point £, as shown in Fig. 3.170(1?). From the loop 2, we have
ks
if-I^)R-U-I^ + l2)r-l2r=0
Yo

f{R + r)-l2{R+2r)-Ir=0 ...(2)


oo

or

Solving equations (1) and (2), we get


eB

R + r
1 I
R+3r
Fig. 3.170 (b)
r

R-
^-I
ou

and h - ...(3)
ad

Resistance of wire ABD =r+r=2r


R+3r
2rx r 8r
Y

Resistance of wire ACDEB = r +


2r+ r
+ r = —
3
Similarly, from the loop 3, we have
{l-f)R + (I-I^+l2)r = ^
nd

As these two resistances are in parallel, so the


Re

equivalent resistance R between points A and B is or


(R+r)+ I^rA- /{R + r) = S
Fi

given by
Substituting the values of f and I2 from equation
i=JL A=Z_ or R =—
8r
(3), we get
R 2r 8r 8r 7

Given r = 2 Q, therefore, R =
8x2 16
=—a.
(R + r)^^ ^(R-r) Z+(R + r)/=6
R + 3r R+3r
7 7
3rR+r^
Example 159. Calculate the equivalent resistance between or
R+3r
the points A and B in the network shoiun in Fig. 3.171.
r R Equivalent resistance between A and B,
AAA/— —I
3rR + r^
o- r ■o I R+3r
A 6
r(3R + r)
Wv * Wv—' (R + 3r)
Fig. 3.171 R r
PHYSICS-XII

roblems For Practice 5. Fig. 3.177 shows n cells connected to form a series
circuit. Their internal resistances are related to their
1. Apply Kirchhoff's rules to the loops PRSP and emfs as = a , where a is a constant. Find (i) the
PRQPto write the expressions for the currents I2 current through the circuit and (ii) the potential
and in the circuit shown in Fig, 3.173. difference between the terminals of ;th battery.
[CBSE ODIO]
[Ans. (0 - (ii) 0]
Ans. -^A -^A il A
a

860 ' 215 ' 172 S,


1 ^2 €3 f3 t r,
200 Q 1-o-AAAr-^ l-o-VN/\ ho-VW-o-i
S R
I
5 V.

60 n
k Fig. 3.177

6. Two cells of emfs 3 V and 4 V and internal resis


Q
Fig. 3.173
4V
tances 1 n and 2 Q respectively are connected in
parallel so as to send current in the same direction
2. Use Kirchhoff's rules to determine the value of through an external resistance of 5Q.
the
current /j flowing in the circuit shown in Fig. 3.174. (/) Draw the circuit diagram.
30 Q
[CBSE D 13CJ {ii) Using Kirchhoff's laws, calculate {a) the current
/1
^ ● (Ans. /j = - 0.75 A) through each branch of the circuit. (6) p.d.
across the 5Q resistance. [CBSE OD 95, 96 C]

Ans. {a) A A, ^ A, ^ A {h) 2.35 V


20 a L 20 V
h
\ d
h c
h'
20 Q
7. In the electric network shown in Fig. 3.178, use
8, f Kirchhoff's rules to calculate the power consumed
80 V
h by the resistance R = 4Q. [CBSE D 14C]

6, = 12V r, =2fi (Ans. 9 W)


Fig. 3.174 B -Wr- C
80 V
20 n
3. Using Kirclilioff's A B
>VW K = 4Q
rules, calculate the A ■"D
current through the D
40 n
AAV
40 Q and 20 Q £, = 6V
resistors in the E
AVW h
adjacent circuit. +
ion ^
40 V
[CBSE D 19] Fig. 3.178
(Ans. 0 A, 4 A) Fig. 3.175
8. A network of resistors is connected to a battery of
4. The circuit diagram shown in Fig. 3.176 has two negligible internal resistance, as shown in Fig. 3.179.
cells Sj and €2 with emfs 4 V and 2 V respectively, Calculate the equivalent resistance between the
each one having an internal resistance of 2Q. The points A and D, and the value of the current
external resistance R is of 8Q. Find the magnitude (Ans. 1.25 n, 0.5 A)
and direction of currents flowing through the two 2Q 7,-/3
cells. [ISCE 98]
B Wr-*—OC
h
Ans. /,1 = -A,/2 = -1a 2Q
3 2 3
2Q
2D
I
I1 I
2j.
/, *I2 1“ D
AVv
6,1 T ^2 2Ci

3 3
R “I=2A

6
Fig. 3.176
Fig. 3.179
CURRENT ELECTRICITY

9. Using Kirchhoff's rules, determine the value of


unknown resistance R in the circuit shown in
Fig. 3.180 so that no current flows through 4Q
resistance. Also find the potential difference
between A and D. ICBSE D 121

(Ans. 3 V)
f in £ D
r—V/V-t

in 40 K
2V

..1 6V
Fig. 3.184

1
14. In the given electrical networks shown in
Figs. 3.185(n) and (Ij), identical cells each of emf S,

w
A B C
9V 3V
are giving same current /. Find the values of the
Fig. 3.180 resistors R, and in the network (l>).
11
Ans. 9.9 n, — n

Flo
10. Find the current flowing through each cell in the
circuit shown in Fig. 3.181. Also calculate the
potential difference across the terminals of each

ee
R1
/ //lO
cell. (Ans. 0, - 3 A, 3 A, 3 V) / r-*—WV

Fr
s1

10 V in
^6
I
no €
I
3 43 11 n

for
ur
<■
«
A I («) (b)
4V 20
Fig. 3.185
ks

15. What does the ammeter A read in the circuit


Yo

13 V in
oo

shown in Fig. 3.186 ? What if the positions of the


Fig. 3.181 cell and the ammeter are interchanged ?
eB

11. In the network shown in Fig. 3.182, (/) calculate the


Ans. Aa,1a'
11 11
current of the 6 V battery and (iT) determine the
r

potential difference between the points A and B. /


ou
ad

[Ans. (0 2 A (ii) 4 V] ’7 1
Y

C 1 D fA
1 2
14 V 4n \J
40 60
nd
Re

20
/1 /
B<H —Wv—►—< £ L-WV
Fi

60
10 V
40 h Fig. 3.186

AO- -wv 16. In the circuit shown in


20
Fig. 3.187, determine 5 0 50
the current in the
Fig. 3.182 Fig. 3.183 resistance CD and
equivalent resistance
12. In the network shown in Fig. 3.183, find (/) the
between the points A
currents and and {ii) the potential difference and B. The internal
between the points B and £. resistance of cell is
[Ans. (0 Ij = 2 A, fj = “ 3 A' ^3 1 ~2 negligible.
/
13. Calculate the potential difference between the (Ans. 70, 0.4 A) 14 V

junctions B and D in the Wheatstone's bridge shown


[Roorkee 89] (Ans. 0.2 V) Fig. 3.187
in Fig. 3.184.
3,92 PHYSICS-XII

17. A certain length of a uniform wire of resistance 12 Q 22. Twelve identical wires each of resistance 6Q are
is bent into a circle and two points, a quarter of arranged to form a skeleton cube. A current of 40 mA
circumference apart, are connected to a battery of is led into cube at one comer and out at the diago
emf 4 V and internal resistance 1 fl. Find the current nally opposite comer. Calculate the potential diffe
in the different parts of the circuit. rence developed across these comers and the effec
Ans. — A, — A tive resistance of the network. (Ans. 0.2 V, SH)
13 13 J 23. Twelve wires each having a resistance of 3Q are
18. In Fig. 3.188, ABCDA is a uniform circular wire of connected to form a cubical network. A battery of
resistance 2Q. AOC and BOD are two wires along 10 V and negligible internal resistance is connected
two perpendicular diameters B
across the diagonally opposite comers of this net
of the circle, each having work. Determine its equivalent resistance and the
same resistance IQ. A airrent along each edge of the cube. ICBSE OD 19J
battery of emf 6 and internal
resistance r is connected
A
O
^ 24. Find the currents /j, and 1^ through the three
resistors of the circuit shown in Fig. 3.191.
between the points A and D.
(Ans. Zero in each resistor)
Calculate the equivalent r
B D
resistance of the network. D
h I-
15
(Ans n) Fig. 3.188
3 V 1
3 V
/2 3V '73
■ 14

19. Acellofemf €, =2 V and internal resistance ^ = IQ 10 Q lOQ ion

is connected to another cell of emf 62 = 8 V and


internal resistance r2 = 2 Q through an external H
G F
£

resistance of 4 Q as shown in Fig. 3.189. Find the 3 V 3V 3 V

potential difference between point A and point C Fig. 3.191


[CBSE 20C] HINTS
24
A 1-AW
1 n
B
(Ans. y V) 1. By Kirchhoff's junction mle.
2 V
...(/)

4Q From loop PRSP, 2O/3 + 200/3 = ^ ...(//)

8 V
From loop PRQP, 20J3 + 60/^ = 4 ...(Hi)
2Q
D I--VW c On solving equations (i), (ii) apd (Hi), we get
h 11
L .2 — A, I = A.
860 215 ^ 172

Fig. 3.189 2. Applying Kirchhoff's junction rule at 'a', we get


20. Determine the current flowing through the h''' h
galvanometer G of the Wheatstone bridge shown in Applying Kirchhoff's loop mle to the loop ahdcba,
Fig. 3.190. (Ans. 0.0454 A) we get
B 30/j -f- 20/3 = 20
h h or
30/, +20(/j + /2>=20
I1 >0
or
50/j + 20/3 = 20
/=! A
C 1A 5/,2/3 = 2 -ii)
A\i-i i ©ion
Again, from the loop agfedcba, we get
A
■9
20 n
20/3 + 20/3 =80+20
D
or
20/3+ 20(/, + /3)= 100
or
20/3 +40/3 = 100
Fig. 3.190 or
/j + 2/3 = 5 ...(»)
21. The terminals of a battery of emf 3 V and internal Subtracting (//) from (i), we get
resistance 2.5 Q are joined to the diagonally opposite 4/3 =~3 or /j=-0.75A
comers of a cubical skeleton frame of 12 wires, each
The negative sign shows that the actual direction of
of resistance 3Q. Find the current in the battery.
airrent /, is opposite to that shown in the given
(Ans. 0.6 A) circuit diagram.
CURRENT ELECTRICITY 3.93 M
k f

3. In the loop ABCFA, resistance in AB. Their equivalent resistance


80 V = 2+4/3=10/30. This resistance is in parallel
20(/,+ l2) + 40/i =80 +
2on f = /i + h
i -VyAr->—|B with 2 O resistance in AD. The equivalent resistance
/''
or 607j + 20/2 = 80 I1 40 a /1
between A and D,
F 4 VA— 4 10
or
3/j + /2 = 4 o x2 5
R
/,
...(/) h
+
VvV-« D
3 ^ ^
ion
In the loop FCDEF, 40 V 6= /R = 2xl.25 = 2.5 V
-40/, + 10/2 = 40 Applying Kirchhoff's second law to the lower
Fig. 3.192
-4/, + /2 = 4 rectangular loop,
On subtracting (ii) from (i), we get 2/2 = £ = 2.5 V or l2==1.25A
71,1 = 0 /,1 =0 Now /, + /,= /

w
From (i), /2 = 4A i, = / - /2 = 2- 1.25 = 0.75 A
Current through 400 resistor = /, = 0 A From loop BCDB, we get

Flo
Current through 20 O resistor = /, + /2 = 4 A. 2(/, -/3)+2(/^-/3)-2/3 = 0
4. Applying Kirchhoff's second law to loop 1, we get
4/, -6/3 = 0

ee
or

/jr, ^2^2 ~ ^2

Fr
or /, = - i, = - X 0.75 = 0.50 A
or
2/, -2/2 = 4-2 ^ 6 ’ 6
or
/,-/2 = l ...(1) 9. Applying Kirchhoff's loop rule to the loop AFEBA,
for
ur
Similarly, from loop 2, we get (1+ l)/ + 4x0= -6+9
^2^2 ^2) ^2 /=1.5A
1Q p /
ks
or 2/2 + 8(/, + /2)= 2 Fr^WV—T D
Yo

or
4/3 + 5/3 = 1 ...(2)
oo

la 4n
R
On solving equations (1) and (2), we get
eB

/ ■:^6V
L1 =
- -3 A
’ A I-
B
(From -ve to +ve terminal inside 6,)
r

9V 3V
ou
ad

and -U Fig. 3.193


^ 3
Y

From the loop BEDCB, we get


(From +ve to -ve terminal inside 63) 1.5R+4xO= -3+6
5. Suppose a current / flows in the circuit in the indi
nd
Re

R=2Q
cated direction. Applying Kirchhoff's loop law, V AD = (l+l)xl.5 = 3 V.
Fi

6n
/r, + /rj + Ir^ + ... + /r„ = + 63 + S3 +... +

10. Applying Kirchhoff's first law at the junction B, we


or
£, + 62 62 + ... + 6 +
I! get
r, + T2 + rj + ... + r„ /, + /t + 13 — 0 ...(1)
Applying Kirchhoff's second law to the loop
_ 6, + 63 + 83 ... + € +
II
1
AE^ BEy A, we have
(6, +62 + 63 + - + ^ii)
a a

/, xl- /2 x2=(10-4)
(ii) P.D. between the terminals of ith battery 1,-2/2 = 6. ...(2)
1
= 6,-/r,=6,
I t I
. a 6, = o. I
Similarly, from the closed loop AE^BE^ A, we have
/2 x2-I3 xl= 4-13 or 272-/3= "9
a

8. Here R BCD= 2+2 = 4Q. It is in parallel with 2Q ...(3)


resistance in BD. Their equivalent resistance
4x2 4
Solving equations (1), (2) and (3), we get
- Q. This resistance is in series with 2 D
4+2 3
/.1 =0, i2 = - 3 A, [3 =3 A
PHYSICS-XIl

Thus, tine current in the 10 V cell is zero. The Similarly, from the loop ADCEA, we have
current given by the 13 V cell to the circuit is 3 A, I2 X 1.5 + I2 X 1 = 2
and the current taken by the 4 V cell from the circuit 2
is 3 A. h ~ 2.5
= 0.8 A

As there is no current in the 10 V cell, so the


Potential difference between the points A and B is
potential difference across its ends is equal to its
e.nn.f. i.e., 10 V. Since all the three cells are in
V^,-V^g = 1.0AxlQ-1.0V
parallel, the potential difference across the Potential difference between the points A and D is
terminals of each is 10 V. V^~Vd = 0.8 A X 1.5Q = 1.2 V
11. (/) The distribution of current in various branches of .-. Potential difference between the points B and D is
the circuit is shown in Fig. 3.194.
A B
h = 1.2-1.0= 0.2 V.

14. From the network of Fig. 3.195(a), E = 11 7

L3
R1

£i
I I /no
*r
2 V 4 V

\ Q-

h
O

M I
IIQ g-1-
I
3 4^ 11

6 V
(«) (&)
Fig. 3.194
Fig. 3.195
Applying Kirchhoff's second law to loop 1,
In the network Fig. 3.195(7?), the main current I
372 +(7i-/2) = 2
or
7, + 2 72 = 2 ...(0
passes through R^, a part -3 through the llQ
I 91
From loop (2), we get resistor and the remaining current, I =—
10 10
(7j -72) + 2(7^ + 73)=6
through the resistor
or
37j ~ 72 + 21^ = 6 ...(70
From loop (3), we get Applying Kirchhoffs law to the loop 1, we get
7 97
4/3+ 2(7^ -f 73) = 4 — xll-
10 10
X
7^2 = 0 or
or
27,+ 673 = 4
Similarly, from the loop 2, we get
or
7,+373=2 ...(777) 91
I R, -\ X R, = S or 9J 11
On solving equations (7), (77) and (777), we get ^ 10 ^
+
10
X
9
= 117

7,=2A R,=9.9n.
(”) =^2 + - =2 + 6-4 = 4 V. 15. From Kirchhoff's first law, 7 = 7j + 72
12. (0 Applying Kirchhoff's first law at the junction E, >■
7
^3 “ ^1 ^2 ...(7) i. I1
h
From the loop BCDEB, we get 1 2
A
5 V
- 67, + 472 = - 14 - 10 4Q
or
- 37, + 272 =- 12 ...(77) 2n
6H
7
From the loop ABEFA, we get
67, + 273 = 10
Fig. 3.196
or
37, + Ig = 5 ...(777)
On solving equations (7), (it) and (777), we get Applying Kirchhoff's second law for the loop 1 of
Fig. 3.196, we get
7, =2 A, 72=-3A, /3 = -1A 7, x4+7x2 = 5
(7/7) P.D. between points 6 and £ or
7, x4+ (7, + 72) x2= 5
= 10-67, =10-6x2 =~2 V. or
6 7, + 2 72 = 5 ...(1)
13. Applying Kirchhoff's second law to the loop ABCEA, Similarly, from the loop 2, we get
we get
7, X 1 + 7,1 X 1 = 2
73x6-7, x4=0
.. 7,1 =1.0 A
or
4 7, = 6 72- ...(2)
CURRENT ELECTRICITY 3.93 A

if R'is the equivalent resistance of the network, then |j*


Solving equations (1) and (2), ^2 “ ^ /(r+ R') = ^
This will be the reading of the ammeter.
On interchanging the cell and the ammeter, the 3
r + 5 /2 R - / (r + R') = -3 /2 (f + R')
circuit takes the form as shown in Fig. 3.197. Again,
we can show that or R' = 2^ = x0.5 = —a.
7 7 14

^2 = A A.
11
The currents in various branches are as shown in
Fig. 3.199 Applying Kirchhoff's second law to the
h I1 / loop ACDA, we get
r r 5V 4/,+ 2/=8 2V IQ
B

0
A
2 1 -l-AAA ^
4n r:>2/j + l=4 ...((■)

w
6Q From the loop /I
2n /,
I
ABCA, we get T 1

WV
{/-/|)xl'-f^x4=~2
rig. 3.19 7 D i—VA—

Flo
/
=> J-5/j=-2 8V
2il
16. Proceeding as in Example 151 on page 3.86, we
-O'O
obtain equivalent resistance between points A and Fig. 3.199

ee
B as Subtracting (ii) from (/), we get
7 7 ^A

Fr
R' = _R = 1x5 = 7Q (V R = .5!.>1 711 = 6 ^1 = 7
0 5

Main current. 6 , 24.,


= /,1 x4 = -x4 = —V.
AC
for 7 7
ur
6 14
Z = - — = 2A
R 7 211. From the loop ABDA, we get
[/ = 1A]
Current through 5Q resistance in arm CD 51, + 10/^-(l-/,)15=0
ks
5
- R =- /-= / = - / = -- x2=0.4 A.
1 1 or
2D/j + 10/^, = 15
Yo
=;
1
2 5 5 5 5 ...(0
4/, +2/^ = 3
oo

or

18. The current distribution is shown in Fig. 3.198. From the loop BCDB, we get
eB

= 0
Ii B
I3 10(/i-/p-20(l-/^ + /p-101
R
30T1 -40/^ =20
R = 0.5Q 1
3/.1 = 2 ...(ii)
r

or
ou

On solving equations (i) and (ii), we get


ad

]1 R
I-I 1 R h-h - A = 0.0454 A
'* = 22
Y

A' C
0

CjJ <rCi^
I >2 21, Proceeding as in Example 154 on page 3.87, we obtain
nd

tlie effective resistance.


Re

B i-l.
R R-,
R' = ^R
Fi

6
r
D h
5x3
But R = 3 n, therefore, R = = 2.5 Cl
6
Fig. 3.198
Total resistance of the circuit = 2.5 + 2.5 = 5.0 Cl
Applying Kirchhoff's law to different loops, we get
Current,
R(/-/,)+ R(/-J2) + (^ + '')^=^ ...(1)
emf
1 = — = 0.6 A.
Rfi + R(f,-/3)-R(;-/,) = 0 ...(2) 5.0
total resistance
...(3)
22. Proceeding as in Example 154 on page 3.87, we
R/2-R(/-/2)+ R(f2-f3)=0 ...(4)
obtain the effective resistance, R' = ^6 R
On simplifying and solving these equations, 5x6
But R = 6Q, therefore, R = —— = 5Q
/, = /2, h ~ 3'- / =
3 ^
6

P.D. developed = Resistance x Current


and -Lr+5L R = B
3 2 2 = 5x(40xl0”^) = 0.2 V.
3.96 PHYSICS-Xil

23. Proceed as in Example 154 on page 3.87. Principle. Vie basic principle of a potentiometer is that
1
AB - ^AA' ~ ^AD = A
D'C
= 1
B'C -A when a constant current flows through a Zaire of uniform
CC'"3 avss-sectional area and composition, the potential drop across
I BC BB' ~ ^A'B' ~ ^ AV - ^DC ~ ^ = ^A any length of the wire is directly proportional to that length.
dd'-3
In Fig. 3.200, if we connect a voltmeter between the
24. From the loop ABGHA, we get end A and the jockey /, it reads the potential difference
lOli =3-3 or 7.1 =0 P across the length I of the wire A]. By Ohm's law,
From tlie loop BCFGB, we get
V=IR=I.^ V R =
I 1
10/2 - 107, = 3 - 3 or 72 =0 A
^a\
From the loop CDEFC, we get For a wire of uniform cross-section and uniform
10/3 - 10/2 = 3 - 3 or 73=0. composition, resistivity p and area of cross-section A
are
constants. Therefore, when a steady current 1 flows
3.32 POTENTIOMETER through the wire.
57. What is a potentiometer ? Give its construction — = a constant, k
and principle. A

Potentiometer. An ideal voltmeter which does not Hence V = kl or Vail


change the original potential difference, needs to have
This is the principle of a potentiometer. A graph
infinite resistance. But a voltmeter cannot be designed drawn between V and / will be a straight line passing
to have an infinite resistance. Potentiometer is one
such device which does not draw any current from the through the origin O, as shown in Fig. 3.201.
circuit and still measures the potential difference. So it
acts as an ideal voltmeter.

A potentiometer is a device used to measure an unknown


emf or potential difference accurately.
Construction. As shown in Fig. 3.200, a potentio
meter consists of a long wire AB of uniform cross-
section, usually 4 to 10 m long, of material having high
resistivity and low temperature coefficient such as Fig. 3.201 Potential drop V cc length I
constantan or manganin. Usually, 1 m long separate
pieces of wire are fixed on a wooden board parallel to Potential gradient. The potential drop per unit length
each other. The wires are joined in series by thick of the potentiometer wire is known as potential gradient.Itis
copper strips. A metre scale is fixed parallel to the g'^^^n by
wires. The ends A and B are connected to a strong k=-
battery, a plug key K and a rheostat Rh. This circuit, l
called driving or auxiliary circuit, sends a constant SI unit of potential gradient = Vm
-]

current / through the wire AB. Thus, the potential -1

gradually falls from A to B. A jockey can slide along Practical unit of potential gradient = V cm

the length of the wire.


3.33 APPLICATIONS OF A POTENTIOMETER
58. With the help of a circuit diagram, explain how
A can a potentiometer be used to compare the emfs of two
0
f 100
primary cells.
/
Battery 1-1200
<- Comparison of emfs of two primary cells. Fig. 3.202
shows the circuit diagram for comparing the emfs of
300 two cells. A constant current is maintained in the
potentiometer wire AB by means of a battery of emf 6
Rh
400
through a key K and rheostat Rh. Let and ^2 be the
I ' ‘"I I III I " I ■! -I" 111111111J
emfs of the two primary cells which are to be
compared. The positive terminals of these cells are

connected to the end A of the potentiometer wire and


Fig. 3.200 Principle of a potentiometer.
their negative terminals are connected to a high
CURRENT ELECTRICITY

resistance box R.B., a galvanometer G and a jockey / r-mh. G


through a two way key. A high resistance K is inserted <z>
in the circuit from resistance box R.B. to prevent £
excessive currents flowing through the galvanometer.
A
Battery je.
+

L
7; Rh B
A h h
Battery _i_ g

Fig. 3.203 To determine the internal resistance

w
KQ of a cell by a potentiometer.

Rli
Close the key K,. A constant current flows through

Flo
the potentiometer wire. With key kept open, move
the jockey along AB till it balances the emf 6 of the cell.
Fig. 3.202 Comparirrg emfs of two cells by a potentiometer. Let /j be the balancing length of the wire. If k is the

ee
potential gradient, then emf of the cell will be

Fr
As the plug is inserted between a and c, the cell S 1 E = kl ]
gets introduced in the circuit. The jockey / is moved
along the wire AB till tlie galvanometer shows no With the help of resistance box R.B., introduce a
resistance R and close key K^. Find the balance point
for
ur
deflection. Let the position of the jockey be /j and
length of wire A]^ =/j. If k is the potential gradient for the terminal potential difference V of the cell. If I2 is
along the wdre AB, then at null point, the balancing length, then
ks
6,1 1
V = kl2
Yo

By inserting the plug between b and c, the null point


e.'i
oo

V
is again obtained for cell £2- het the balancing length be h
eB

A/2 = I2. Then Let rbe the internal resistance of the cell. If current I
£2 = cf/2 flows through cell when it is shunted with resistance
R, then from Ohm's law we get
r

Hence, -i £=/(R + r) and V= IR


ou
ad

I
1
£ R +r_
Y

If one of the two cells is a standard cell of known V R


emf, then emf of the other cell can be determined.
nd
Re

R
/1
Fi

In order to get the null point on the potentiometer or

wire, it is necessary that the emf, £ of the auxiliary R ^2


battery must be greater than both £^ and £2- h-h
Internal resistance. r=R
59. With the help of a circuit diagram, explain how ^2
can a potentiometer be used to measure the internal
resistance of a primary cell. 60. Why is a potentiometer preferred over a voltmeter
Internal resistance of a primary cell by a for measuring the emf of a cell ?
potentiometer. As shown in the Fig. 3.203, the -t-ve Superiority of a potentiometer to a voltmeter,
terminal of the cell of emf £ whose internal resistance r Potentiometer is a null method device. At null point, it
is to be measured is connected to the end A of the ^loes not draw any current from the cell and thus there
potentiometer wire and its negative terminal to a is no potential drop due to the internal resistance of the
galvanometer G and jockey /. A resistance box R.B. is cell. It measures the p.d. in an open circuit which is
connected across the cell through a key K2. equal to the actual emf of the cell.
3.98 PHYSICS-XII

On the other hand, a voltmeter draws a small Examples based on


current from the cell for its operation. So it measures (i) Comparison of EMFs of two Cells
tine terminal p.d. in a closed circuit whicln is less than tine
(jj) Measurement of Internal
emf of a cell. That is why a potentiometer is preferred
over a voltmeter for measuring the emf of a cell. Resistance of a Cell by a Potentiometer
Formulae Used
3.34 SENSITIVENESS OF A POTENTIOMETER € /
1. For comparing emfs of two cells,'I^ ^
61. What do you mean by the sensitivity of a 1 h
potentiometer ? Hoiu can we increase the sensitivity of a 2. For measuring internal resistance of a cell.
potentiometer ?
xR
Sensitivity of a potentiometer. A potentiometer is
sensitive if
3. Potential gradient of the potentiometer wire.
(0 it is capable of measuring very small potential k =
V
differences, and I
(ii) it shows a significant change in balancing length for 4. Unknown emf balanced against length l,i.=kl
a small change in the poteiitial difference being measured.
Units Used
The sensitivity of a potentiometer depends on the
potential gradient along its wire. Smaller the potential The emfs 6.j and 82 are in volt, lengths and I2 of
gradient, greater will be the sensitivity of the potentiometer. potentiometer wire in metre.
The sensitivity of a potentiometer can be increased
by reducing the potential gradient. This can be done in Example 160. A potentiometer wire is 10 m long and has
two ways ; a resistance of 18 Cl. It is connected to a battery of emf 5 V
(i) For a given potential difference, the sensitivity and internal resistance! Cl. Calculate the potential gradient
along the wire.
can be increased by increasing the length of the
potentiometer wire. Solution. Here / = 10 m, R = 18 O, 6 = 5 V, r =2 H
(ii) For a potentiometer wire of fixed length, the Current through the potentiometer wire,
potential gradient can be decreased by reducing 5
the current in tlie circuit witli the help of a rheostat. ^=1a
R + r 18+2 20 4
For Your Knowledge
.●. Potential gradient = = 0.45 Vm"^
> A potentiometer can be regarded as an ideal voltmeter I “4^10
with infinite resistance because it does not draw any
current from the source of emf at the null point.
Example 161. A potentiometer wire is supplied a constant
voltage of 3 V. A cell of emf 1.08 V is balanced by the voltage
> The principle of potentiometer requires that (/) the drop across 216 cm of the wire. Find the total length of the
potentiometer wire should be of uniform cross-
potentiometer wire.
section and (ii) the current through the wire should
remain constant. Solution. Here 8 =3 V, 8j = 1.08 V, =216 cm, / = ?
> The emf of the auxiliary battery must be greater than 8 / 8 3x216
As
the emf of the cell to be measured. 81 / ■■■ '=!->''1 = 1D8
= 600 cm.
*1

> The balance point cannot be obtained on the potentio


meter if the fall of potential along the potentiometer Example 162. Two cells of emfs 8^ and 82 (8j >82) are
wire due to the auxiliary battery is less than the emf of connected as shown in Fig. 3.204.
the cell to be measured.
A B C
^ The positive terminals of the auxiliary battery andthe 6, 8,
cell whose emf is to be determined must be connected
Fig. 3.204
to the zero end of the potentiometer.
> Other uses of a potentiometer. Any physical quantity When a potentiometer is connected between A and B, the
that can produce or control a potential difference can balancmg length of the potentiometer wire is 300 cm On
be measured using a potentiometer. Thus, a potentio connecting the same potentiometer between A and C, the
meter can be used to measure and control stress, balancing length is 100 an. Calculate the ratio of 8j and 82-
temperature, radiation, pH, frequency, etc. [CBSE F 201
CURRENT ELECTRICITY 3.99

Solution. As emf oc balancing length of the Example ]6A. In a potentiometer, a standard cell of etnf
potentiometer wire 5 V and of tiegJigible resistance maintains a steady current
When the potentiometer is connected between through the poterttiometer wire of length 10 m. Two primary
A and B, 6,1 oc300 cells of emfs Sj and ^2 joined in series with (i) same
polarity, and {ii) opposite polarity. The combination is
When potentiometer is connected between A and C, connected through a galvanometer and a jockey to the
Sj xiOO potentiometer. The balancing lengths in the two cases are
100 . ^2 100 found to be 700 cm and 100 cm respectively,
Hence or 1--=^
g1 300 €1 300 (i) Draw the necessary circuit diagram,
(ii) Find the value of the emfs of the two cells.
g
or
^2 1 2
or
1 _ - = 3: 2. [CBSE D 04C ; OD 20]
g1 3 3 g, 2 Solution, (j) The circuit diagram is shown in

w
Fig. 3.206.
Example 1 63. In Fig. 3.205, a long uniform potentiometer
g=5V
wire AB is having a constant potential gradient along its
length. The null points for the tivo primary cells of emfs gj

Flo
and ^2 connected in the manner shoion are obtained at a ''
,.

distance of 120 an and 300 cm from the end A. Find (i) gj / gj /


and (ii) position of null point for the cell g,.
.4

ee
How is the sensitivity of a potentiometer increased ?

Fr
l-h
[CBSE D 12]

Fig. 3.206
for
ur
5V 1 -1
(i7) Here k = = —Vm
300 cm
10m 2
ks
-120 cm-
When the cells are joined with same polarity,
Yo
oo

/I B

H ly g^1 +g,=fcL =-x 7.00 =3.5 V ...(/)


2 ^2
eB

When the cells are joined with opposite polarity,


^2
gT-g,=fcL=-x 1.00 =0.5 V ■■■{ii)
r

Fig. 3.205
ou
ad

On solving (j) and (j7), we get


Y

Solution. (0 Let k be the potential gradient in


volt/cm. Then
g^ = 2.0 V and g = 1.5 V.
Example 165. A 10 metre long wire of uniform
nd
Re

gj+g2 =300fc and gj-g2=120)t cross-section of 20 .0 resistance is used as a potentiometer


g,I =210fe and g^ = 90A- wire. This wire is connected in series with a battery of 5 V,
Fi

g along with an external resistance o/480n. If an unknown


Hence, Z emft is balanced at 600 an of this wire, calculate (i) the
g2 3 potential gradient of the potentiometer wire and (ii) the
value of the unknown emf^. [CBSE D 06]
(ii) As gj= 2101c 5V 480 a
Balancing length for cell gj is I I- AA/V
R
g
= 210 cm
/
600 cm ■H
●6b
The sensitivity of a potentiometer wire can be A'

increased by decreasing potential gradient either


through increasing length of the potentiometer wire or g
through increasing resistance put in series with the
main cell. Fig. 3.207
3.100 PHYSICS-XII

Solution. Current in the circuit or through the Example 167. The length of a potentiometer wire is 5 m It
potentiometer wire is is connected to a battery of constant emf Tor a given
1/ 5 V
1 = = 0.01 A Leclanche cell, the position of zero galvanometer deflection is
R
AB + R (20 +480) n obtained at 100 cm If the length of the potentiometer wire be
Resistance of potentiometer wire, made?) w instead of 5 m, calculate the length of wire for zero
deflection in the galvanometer for the same cell.
R^g=20n [CBSE F 97]
P.D. across the wire,
Solution. Here i = 5 m, /^ = 100 cm = 1 m, I' =8 m.
l/=7R^g=0.01x20=0.2V I' =?
1

Length of potentiometer wire, Let 8 be the emf of the Leclanche cell.


/ = 10 m =1^)00 cm IRl
In first case. 1
...(1)
.●. Potential gradient, /
^ 0-2 V -1 iRi:1
= 0.0002 V cm In second case, ...(2)
/ 1/100 cm r

Unknown emf balanced against 600 cm length is Comparing equations (1) and (2),
E = kl' =: 0.0002 X 600 = 0.12 V.
/' I
Example 166. In the circuit diagram given below, AB is a

uniform wire of resistance 15 ohm and length one metre. It is or 4- X I' = — X 8 = 1.6 m.
connected to a series arrangement of cell 8j of emf 2.0 V and l 5
negligible internal resistance and a resistor R. Terminal A is
also connected to an electrochemical cell 82 of emf 75 mV Example 168. A potentiometer wire of length 100 cm has
and a galvanometer G. In a resistance of 10 O. It is connected in series with a
this set-up, a balancing '1 R resistance and a battery of emf 2 V and of negligible internal
point is obtained at 30 cm
>VW resistance. A source of emf 10 mV is balanced against a

markfrom A. Caladate the } length of 4:0 cmofthe potentiometer wire. What is the value
A ■iB of the external resistance ?
resistance of R. If '^ore I [IITJ

to have an emf of300 mV, Solution. Fig. 3.209 shows a potentiometer wire of
where will you expect the length 100 cm connected in series to a cell of emf 2 V
balancing point to be ?
E, G and an unknown resistance R. The cell of emf 10 mV
[CBSE F 16] Fig. 3.208 balances length AJ = 40 cm of the wire.
Solution. Current through the potentiometer wire. R 2V
81 2 1
1 =
R+R R + 15 V/
AB 1..

Resistance of the 30 cm length of wire, which 100 cm


balances the emf 8.,,1' is <>B

15
'— 40 cm a:
R' = x30=4.5O
100
\
<£>
Now, 82 = Potential drop across R' 10 mV

.-. 75 xl0"3= — X 4.5


R + 15 Fig. 3.209
2x4.5 10
or R = -15=120-15 = 105 n. Resistance of wire Aj = x40=4D
75x10"^ 100

Current through wire A},


For &2 = 300 mV, the balancing length is given by
300 10 mV 10xl0“^V
X 30 =120 cm 1 = =2.5x10“^ A
1
75 4D 40.

As the length of the potentiometer wire is just Tlie same current flows through the potentio-
100 cm, so this balance point cannot be obtained on the meter wire and through the external resistance R.
wire.
Total resistance = (R + 10)Q
CURRENT ELECTRICITY 3.101
2 V
2.5xlO~^A = Example 170. A cell gives a balance with 85 an of a
(K + 10)Q potentiometer wire. When the terminals of the cell are
or R + 10 =
2
= 800
shorted through a resistance of7.5 H, the balance is obtained
2.5 X 10"^ at 75 cm Find the internal resistance of the cell. IISCE 951

R = 800-10 = 790 Q.
Solution. Here l-^ = 85 cm, l^ = 75 cm, R=7.5D.
Internal resistance.
Example 169. AB is 1 metre long uniform wire of 10 Q
resistance. Other data are as shown in Fig. 3.210. Calculate fS5-75\ = in.
r= R = 7.5
(i) potential-gradient along AB and (ii) length AO, when 75

galvanometer shows no deflection. [CBSE D 2000C, 16]


Example 171. A cell, shunted by a 8Q resistance, is
balanced across a potentiometer wire of length 3 m. The
balancing length is 2 m ivhen the cell is shunted by 4 Q

w
A'
resistance. Find the value of internal resistance of the cell.
IJEE Main June 22]
l-l
I
Solution. In first case, r= R^

Flo
h )
I
...(0

ee
1
R1
Fig. 3.210

Fr
Similarly, in second case,
Solution. (0 Total resistance of the primary circuit h
r— ...{ii)
= 15+ 10 = 25Q, emf = 2 V
for
ur
Current in the wire AB,
Subtracting (ii) from (0,
1 = — =0.QS A
25 I1 ^2
ks
P.D. across the wire AB
r
= l-f-1+1.2.
Yo

R, K,
oo

= Current x resistance of wire AB


r =
h h 2-3
= 8Q.
= 0.08x10=0.8 V / 3 2
eB

Potential gradient R1 ^2 8 4

P.D. 0.8
= 0.008 V cm
r

length 100 roblems For Practice


ou
ad

(ii) Resistance of secondary circuit 1. A potentiometer wire is 10 m long and a potential


Y

= 12 +0.3=1.50 difference of 6 V is maintained between its ends.


emf =1.5 V Find the emf of a cell which balances against a
nd

length of 180 cm of the potentiometer wire.


Re

Current in the secondary circuit = — = 1.0 A


^ 1.5 (Ans. 1.08 V)
Fi

The same is the current in 0.3 O resistor. 2. The resistance of a potentiometer wire of length
10 m is 20 n. A resistance box and a 2 volt
P.D. between points A and O
accumulator are connected in series with it. What
= P.D. across 0.3 O resistor in the
resistance should be introduced in the box to have a
zero-deflection condition
potential drop of one microvolt per millimetre of
= Current x resistance the potentiometer wire ? (Ans. 39800)
= li)x0.3=0.3 V 3. In a potentiometer arrangement, a cell of emf
1.20 volt gives a balance point at 30 cm length of the
Length AO wire. This cell is now replaced by another cell of
Potential difference
unknown emf. If the ratio of the emfs of the two
Potential gradient cells is 1.5, calculate the difference in the balancing
0.3 V length of the potentiometer wire in the two cases.
= 37.5 cm. ICBSE D 06C]
-1
0.008 V cm
(Ans. 10 cm)
3.102 PHYSICS-XII

4. Two cells of emfs and €2 are connected together 11. A standard cell of emf 1.08 V is balanced by the
in two ways shown in Fig. 3.211. The 'balance points' potential difference across 91 cm of a metre long
o- I o o- ■o wire supplied by a cell of emf 2 V through a series
resistor of resistance 2Q. The internal resistance of
Fig. 3.211 the cell is zero. Find the resistance per unit length of
in a given potentiometer experiment for these two the potentiometer wire. (Ans. 0.03 fi cm ')
combinations of cells are found to be at 351.0 cm 12. Potentiometer wire PQ of 1 m length is connected to
and 70.2 cm respectively. Calculate the ratio of the a standard cell By Another cell, €2/ of emf 1,02 V ISi:
emfs of the two cells. ICBSESP081 (Ans. 3: 2) connected as shown in the circuit diagram with a
5. A potentiometer has 400 cm long wire which is resistance Y and a switch, S. With switch Sopen,
connected to an auxiliary of steady voltage 4 V. A null position is obtained at a distance of 51 cm from
Leclanche cell gives null point at 140 cm and Daniel P. Calculate (i) potential gradient of the potentio
cell at 100 cm. (i) Compare emfs of the two cells. meter wire and (ii) emf of the cell €j. (Hi) When
(ti) If the length of wire is increased by 100 cm, find switch S is closed, will null point move towards P
the position of the null point with the first cell. or towards Q ? Give reason. ICBSE OD 041
[Ans. (/) 7 : 5, (ii) 175 cm] (Ans. 0.02 V cm"', 2 V, no effect)
6. With a certain cell, the balance point is obtained at
60 cm from the zero end of the potentiometer wire.
With another cell whose emf differs from that of the
first cell by 0.1 V, the balance point is obtained at
55 cm mark. Calculate the emf of the two cells.

(Ans. 1.2 V, 1.1 V) 1.02 V

G
7. A potentiometer wire has a potential gradient of
0.0025 volt/cm along its length. Calculate the length 6 s

of the wire at which null-point is obtained for a


1.025 volt standard cell. Also, find the emf of Fig. 3.213
another cell for which the null-point is obtained at
860 cm length. (Ans. 410 cm, 2.15 V) 13. A battery of 4 V and a variable resistance Rh are
8. AB is a potentiometer wire of length 100 cm. When connected in series with the wire AB of the
a cell ^2 is connected across AC, where AC = 75 cm, potentiometer. The length of the wire of the
no current flows from ^2- Find (i) the potential potentiometer is 1 metre. When a cell E, of emf
gradient along AB and (ii) emf of the cell 6^. The 1.5 volt is connected between points A and C, no
internal resistance of the cell is negligible. current flows through 82- Length of AC = 60cm.
[Ans. (/) 0.02 volt/cm (//) 1.5 V] (/) Find the potential difference between the ends
A and B of the potentiometer.
(ii) Would the method work, if the battery 8j is
replaced by a cell of emf of 1 V ? ICBSE D 03]
[Ans. (/) 2.5 V, (ii) No]

g1

4V
9. In a potentiometer arrangement for determining 100 cm
the emf of a cell, the balance point of the cell in open C
■6b
circuit is 350 cm. When a resistance of 9 Q is used in I.

60 cm
the external circuit of the cell, the balance point 62
shifts to 300 cm. Determine the internal resistance
of the cell.
h
<S>
ICBSE 18] (Ans. 1.5 Q)
10. A potentiometer wire of length 1 m has a resistance Fig. 3.214
of 10 D. It is connected to a 6 V battery in series with
a resistance of 5Q. Determine the emf of the 14. The potentiometer wire of length 200 cm has a
resistance of 20 D. It is connected in series with
primary cell which gives a balance point at 40 cm. a

[CBSE D 141 resistance 10 D and an accumulator of emf 6 V


(Ans. 1.6 V)
CURRENT ELECTRICITY 3A03

having negligible resistance. A source of 2.4 V is (ii) Let 6 be the emf of the auxiliary battery and 1 be
balanced against a length 'L' of the potentiometer the length of potentiometer wire. Then € = 4 V and
wire. Fmd the value of L [CBSE F 03] / = 400 cm.

(Ans. 120 cm) 6I _ Sj 140 _ 7


or

K
/ 4 “ 400 ” 20
R' = 10Q
-AVV-—11 (●)● gj=1.4V
6 V
When length is increased by 100 cm, new length,
/' = 400+ 100 = 500 cm
■6b
C
Now or 11 = J-
G. 4 500
1.4 X 500
2.4 V .'. New balancing length, /j = — =175 cm.

w
Fig. 3.215 6. Let the emf of the two cells be 6 and g - 0.1. Then
g 60
15. A potentiometer wire carries a steady current. The g=1.2V.
potential difference across 70 cm length of it balances g-0.1 55

Flo
the potential difference across a 2 Q coil supplied by emf of the other cell = 12 - 0.1 = 1.1 V.
a cell of emf 2.0 V and an unknown internal
1025
resistance r. When a IQ coil is placed in parallel = 410 cm.

ee
0.0025
with the 2Q coil, a length equal to 50 cm of the

Fr
potentiometer wire is required to balance the {,■,') £' = kr = 0.0025 X 860 = 2.15 V,
€1 2V
potential difference across the parallel combi 8. (I) k=
_
- = 0.02 Vem’L
nation. Find the value of r. (Ans. 0.5 Q) h 100 cm

for
ur
HINTS (u)g2=W2 = 0.02 x75= 1.5 V.
2. Resistance of the potentiometer w'ire, R = 20 Q h h 350-300^ 50
x9n=1.5Q.
9. r = R = x9Q =
ks
Length of the potentiometer wire = 10 m = lO"* mm 300 ) 300
Yo

-I
Required potential gradient, ^ = 1 pV mm V 6V
oo

10. / = = 0.4 A
Potential drop along the potentiometer wire, R
AB + R (10+5)n
eB

V = W = 10pV mnr' xl0''mm = lO^V = 10‘A V = IR


AB
= 0.4xl0 = 4.0V
Current through the potentiometer wire, 4.0 V 4.0 V -1
= 0.04 V cm
V _ 10^^
r

/ = = 5xl0-^A / 1 m 100 cm
R” 20
ou
ad

Unknown emf balanced against 40 cm of the wire,


If R' is the required resistance to be introduced in
Y

the resistance box, then


g = W' = 0.04 V cm X 40 cm = 1.6 V.

g -4 2 11. Let r ohm be the resistance per cm of the


nd

/ = or 5x10
Re

R+ R' 20+ R' potentiometer wire. Then


IR
AB _
gR AB 2xl00r -1
— V cm
Fi

or R'= 39800.
/ /(R+R^g) lOdcT+ToOr)
3. Here g, = 1.20 V, /, = 30 cm As the emf of 1.08 V balances against a length of
g k = 15
Also L = _L 91 cm, so
e ; 2xl00r 1.08
^2 '2 k = V cm
-I

I =-L- = ^ = 20 cm
91 T00~(2+'Ibdr) 91
^15 15
On solving, r = 0.029 - 0.03 cm
Difference in the balancing lengths, 102 V
= 30-20 = 10 cm. 12. (z)ic = -^- = 0.02 V cm "Z
^2 51cm
4. Proceed as in Example 162 on page 3.98. -1
{ii) g^ = kl PQ = 0.02 V cm X 100cm = 2 V.
5. (f) Here /, = 140 cm, /j = 100 cm
(Hi) With switch S closed, the null point is not
g
„_i
1
= Ii«=Z = 7:5.
_ affected because no current flows through the
^2 h 100 5 cell g, at the null point.
3.104 PHYSICS-XII

y /
13. (0 As
AB _ ‘1 3.35 WHEATSTONE BRIDGE
62 ^2
62. W]int is n Whentskme bridge ? When is the bridge
P.D. between A and B, said to be balanced ? Apply Kirchhoffs laws to derive the
I, 100 cm
V
AB X 1.5 = 2.5 V. balance condition of the Wheatstone bridge.
60 cm
Wheatstone bridge. It is an arrangement of four
(i7) No, this method would not work when 8^ = 1V, resistances used to determine one of these resistances quickly
because then 8, < 82 and null point cannot be
1 and accurately in terms of the remaining three resistances.
obtained through the potentiometer wire. This method was first suggested by a British physicist
6
14. / AB - = 0.2 A Sir Charles F. Wheatstone in 1843.
10 + 20 30
A Wheatstone bridge consists of four resistances P,
V AB = /
AB I?,,b-0.2x20 = 4V Q, R and S ; connected to form the arms of a
Potential gradient. quadrilateral ABCD. A battery of emf 8 is connected
y 4V between points A and C and a sensitive galvanometer
M = -1
k =
I 200 cm
= 0.02 V cm between Band D, as shown in Fig. 3.216.
Let S be the resistance to be measured. The
Potential difference
Balancing length, L = resistance R is so adjusted that there is no deflection in
Potential gradient the galvanometer. The bridge is said to balanced when
2.4 V
= 120 cm. the potential difference across the galvanometer is zero
-1
0.02 V cm so that there is no current through the galvanometer.
In the balanced condition of the bridge,
15. In first case. Current sent by the 2.0 V cell through
2 a coil. P R

8 2 Q s
'1
Total resistance 2 + r Q
Unknown resistance, S = — . R
P
Potential drop across 2 Q coil,
4 Knowing the ratio of resistances P and Q, and the
V^ = 2+ r 2+ r
resistance R, we can determine the unknown
resistance S. That is why the arms containing the
But
y^ cc 70 cm resistances P and Q are called ratio arms, the arm AD
4
●^^oc70 containing R standard arm and the arm CD containing S
...(/) the unknown arm.
2+ r

In second case. The combined resistance of the


parallel combination of 2Q and ID coil,
2x1

2TI 3
Current sent by the cell through the parallel
combination,
I I
8 2 6
7 = Total resistance (2/3)+r 2 + 3r

Potential drop across R^^,


6 4
y, = R„ K = - X - Fig. 3.216 Wheatstone bridge.
^ 3 2 + 3r 2 + 3r
But y, oc 50 cm Derivation of balance condition from Kirchhoff's
4 laws. In accordance with Kirchhoff's first law, the
oc50 ...(//)
2+3r currents through various branches are as shown in
Fig. 3.216.
Dividing (?) by (ii), we get
Applying Kirchhoffs second law to the loop ABDA
4 2 + 3r 70 we get
X or r = 0.5D.
2+ r 4 50
fP+I^G-I^R=0
CURRENT ELECTRICITY 3.105

where G is the resistance of the galvanometer. Again {Hi) The unknown resistance can be measured to a
applying Kirchhoff's second law to the loop BCDB, we very high degree of accuracy by increasing the
get ratio of the resistances in arms P and Q.

For Your Knowledge


In the balanced condition of the bridge, = 0. The
above equations become > When the Wheatstone bridge is balanced, the po
tential difference between the points B and D is zero.
I^P-I^R = 0 or
I^P=I^R ...(f)
> The Wheatstone bridge is most sensitive when the
and
Ifi-1^5 = 0 or resistances in the four arms are of the same order.

On dividing equation (f) by equation (if), we get > Wheatstone bridge method is not suitable for the
P R measurement of very low and very high resistances.
> In the balanced Wheatstone bridge, the resistance in
Q s

w
arm BD is ineffective. The equivalent resistance of the

This proves the condition for the balanced balanced Wheatstone bridge between the points A
Wheatstone bridge. and C will be
63. What do you mean by sensitivity of a Wheatstone (P+Q)(R+ S)

Flo
R
bridge ? On zvhat factors does it depend ? eq
P + Q+R+ s
Sensitivity of a Wheatstone bridge. A Wheatstone

ee
> If the bridge is balanced, then on interchanging the
bridge is said to be sensitive if it shows a large deflection in positions of the galvanometer and the battery there is

Fr
the galvanometer for a small change of resistance in the no effect on the balance of the bridge. That is why the
resistance arm.
arms BD and AC are called conjugate arms of the
The sensitivity of the Wheatstone bridge depends bridge.
for
ur
on two factors :
> The Wheatstone bridge is the simplest example of an
(f) Relative magnitudes of the resistances in the arrangement, the variants of which are used for a
four arms of the bridge. The bridge is most large number of electrical measurements. The
ks
sensitive when all the four resistances are of the
important applications of Wheatstone bridge are
Yo

same order.
metre bridge, Carey-Foster'S bridge and post office box.
oo

(ii) Relative positions of battery and galvanometer.


eB

According to Cfi//cfidtT for the greater sensitivity of 3.36 METRE BRIDGE OR SLIDE WIRE BRIDGE
the Wheatstone bridge, the battery should be so connec 65. What is a metre bridge ? With the help of a circuit
ted that the resistance in series with the resistance to be
r

diagram, explain how it can be used to find an unknown


ou

measured is greater than the resistance in parallel with it. resistance. Explain the principle of the experiment and
ad

According to Maxwell for the greater sensitivity of give the formula used.
Y

the Wheatstone bridge, out of the battery and the galvano Metre bridge or slide wire bridge. P is the simplest
meter, the one having the higher resistance should be practical application of the Wheatstone bridge that is used to
nd
Re

measure an unhwwn resistance.


connected between the junction of the tivo highest resistances
and the junction of the two loivest resistances. Principle. Its working is based on the principle of
Fi

Wheatstone bridge.
64. What are the advantages of measuring resistance
by Wheatstone bridge method over other methods ? When the bridge is balanced,
P R
Advantages of Wheatstone bridge method. The
bridge method has following advantages over other Q s
methods for measuring resistance : Construction. It consists of usually one metre long
(/) It is a null method. Hence the internal resistance magnarun wire of uniform cross-section, stretched
of the cell and the resistance of the galvano- along a metre scale fixed over a wooden board and
meter do not affect the null point. with its two ends soldered to two L-shaped thick
(ii) As the method does not involve any measure- copper strips A and C. Between these two copper
ment of current and potenHal difference, so the strips, another copper strip is fixed so as to provide
resistances of ammeters and voltmeters do not hvo gaps ab and a,(i,. A resistance box R.B. is connected
affect the measurements. in the gap ab and the unknown resistance S is
’¥.106 PHYSiCS-XII

connected in the gap a^by A source of emf B is Examples Based on


connected across AC. A movable jockey and a (I) Wheatstone Bridge
galvanometer are connected across BD, as shown in
(ii) Slide Wire Bridge
Fig. 3.217.
K Formulae Used
^1 (●> p R
B 1. For a balanced Wheatstone bridge, —
'Q s
R S If X is the unknown resistance
{M} >WSr
I D f P R
or X =
RQ

rps a
bi
Q X p

P B Q 2. In a slide wire bridge, if balance point is obtained


o at / cm from the zero end, then
I ■100-1
P R f (100 -1
or X = R
Q ” X " 100 - i I
u

Units Used
Fig. 3.217 Measurement of unknown resistance
by a metre bridge. All resistances are in ohm and distances in cm.

Working. After taking out a suitable resistance R Example 172. Find the value of resistance X in the circuit
from the resistance box, the jockey is moved along the shown in Fig. 3.218 so that the junctions Mand N are at the
wire AC till there is no deflection in the galvanometer, same potential. [ISCE 09)
This is the balanced condition of the Wheatstone h
bridge. If P and Q are the resistances of the parts AB 2V
and BC of the wire, then for the balanced condition of
2n 5n
the bridge, we have
_P _ R
Q“ S
Let total length of wire AC = 100 cm and AB = I cm,
then BC = (100 -1) cm. Since the bridge wire is of
uniform cross-section, therefore,
resistance of wire cc length of wire Solution. As no current flows through the middle
or
P resistance of AB arm MN, the circuit represents a balanced Wheat
Q resistance of BC stone bridge.
a I I — = ?. or X = -xl5 = 6Q.
15 5 5
o (100-0 100-/
Example 173. P, Q, R and S are four resistance wires of
where o is the resistance per unit length of the wire. resistances 2, 2, 2 and 3 ohms respectively. Find out the
Hence resistance with which S ynust he shunted in order that bridge
R _ I may be balanced.
S " 100-/ Solution. For a balanced Wheatstone bridge,
P R
or
R(100-/)
Q s
/

ButP=2f2, Q = 2n, R=2n


Knowing / and R, unknown resistance S can be 2 S
determined.
i.e., resistance S must have a total resistance of 2 O. In
Determination of resistivity. If ris the radius of the arm S, the resistance of 3 Q must be shunted with a
wire and /' its length, then resistivity of its material resistance r so that the combined resistance is oflQ..
will be
i.e., 1 1^1. or
1 1 1 1
SA S X Tcr^ r 2 3 6
p =
Y r
.'. Required shunt, r = 6 O.
CURRENT ELECTRICITY 3.107

Example 174. In a Wheatstone bridge arrangement, the In circuit 2, the interchange of the positions of the
ratio arms P and Q are nearly equal. The bridge is balanced batter}' and the galvanometer does not affect the
when R = 500 Q. On interchanging P and Q, the value of R balance condition of the Wheatstone bridge, so
for balancing is 505 Q. Find the value ofX and the ratio 6 _ ^2
PIQ. u~Y
Solution. For balanced Wheatstone bridge, 6x8
= 4Q
12
Q X
In the first case, R = 500 Q
^2 4 2
P ^ 500 ...(1) = 3:2.
Q X
Example 176. Calculate the current drawn from the

w
In the second case when P and Q are interchanged, network of resistors shown in Fig. 3.220.
R=505n [CBSE OD 09,15C]
Q 505 2Q

Flo
...(2) VvV
P X
A in 5C1 D
Multiplying equations (1) and (2), WA/—6—V\Ar-f—WV—*

ee
B C
500 X 505

Fr
1 = AA/V
X2 2Q

or
X = poo X 505
for
ur
4V
= 502.5 n
Fig. 3.220
Substituting the value of X in (1), we get
ks
P _ 500 Solution. The given network is equivalent to the
Yo

Q “ 502.5 circuit shown in Fig. 3.221(fl).


oo

1
= 1:1.005.
eB

1:005

Example 175. The galvanometer, in each of the two given


circuits, does not show any deflection. Find the ratio of the
r

resistors R.^ and R^, used in these two circuits.


ou

I I
ad

[CBSE OD 13)
Y

I
4V
nd
Re

Fig. 3.221 (a)


IQ _2Q _P _K
Fi

Now i.e.,
Q“ S
The given circuit is a balanced Wheatstone bridge.
The resistance of 5t2 in arm BD is ineffective. The
3.0 V
equivalent circuit reduces to the circuit shown in
I
Fig. 3.221(fc).
Circuit 1 Circuit 2
in B 2n
HWSr-o-A/W
Fig. 3.219 A C
2n D 4n
●—VvV-«-AAA»—'
Solution. In circuit 1, the Wheatstone bridge is in 1 .. '’I
the balanced condition, so 4V

4x9 I
= 60
R,~0 Fig. 3.221 (&)
3.108 PHYSICS-Xll

Resistances in AB and BC are in series, their Example 178. A potential difference of2V is applied
equivalent resistance =1 + 2=30. between the points A and B shown in network drawn in
Resistances in AD and DC are in series, their ^^8- -3-223. Calculate
equivalent resistance =2 + 4=60
The resistances of 3 O and 6 O are in parallel.
The equivalent resistance R between A and C is
3x6
R = =2 0
3 + 6

Current,
y
/ = - _4
- = 2 A.
R~2
Example 177. Each of the resistances in the network
shown in Fig. 3.222(a) equals R. Find the resistance between
two terminals A and C. IJEE Main 22) Fig. 3.223

(i) the equivalent resistance of the network between the


points A and B, and
(ii) the magnitudes of currents flowing in the arms
AFCEB and AFDEB. ICBSE OD 98)

Solution, (i) The equivalent network is shown in


Fig. 3.224(fl). It is a balanced Wheatstone bridge
because

Fig. 3.222 (a)

Solution. The network shown in Fig. 3.222(b) is the


equivalent network of the given network.
D
/K

Fig. 3.224 (a)

Hence the points C and D are at the same potential.


The resistance in arm CD is ineffective. The given
network reduces to the equivalent circuit sho\vn in
Fig. 3.222 (b) Fig. 3.224(&).
in in
It is a balanced Wheatstone bridge because c
WV"^WV-i
R^R
R R E in

D
in

Hence the points B and D must be at the same


potential. The resistance R in arm BD is ineffective. Fig. 3.224 (b)
Total resistance along ADC = R + R=2RQ
Total resistance along FCE =2+2=40,
Total resistance along ABC = R + R=2RCl
Total resistance along FDE =2+2=40
These two resistances form a parallel combination. These two resistances form a parallel combination.
.'. Effective resistance between A and C .’. Equivalent resistance between points A and B
2Rx2R 4x4 _ _
= RO. =2 0
2R+2R 4 + 4
CURRENT ELECTRICITY 3,109

V _2 V = 1A
R
(n) Total current in the circuit = —
R ~2Q -WV
R R
D
Current through arm AFCEB C AA/V -Wv^ £

= Current through arm AFDEB ■R R ■R

= i A = 0.5 A.
2

Example 179. Find the value of the unknown resistance X, Fig. 3.226 (a)
in the following circuit, if no current flows through the
section AO. Also calculate the current drawn by the circuit Solution. The equivalent circuits are shown below,
arm DE of tlie balanced Wheatstone
from the battery of emf 6V and negligible internal The resistance R in
resistance. [Fig. 3.225(a)] [CBSEOD02] bridge is ineffective.

w
R R
VSA>

Flo
R R
—WV WV
R

ee
I
-WV

Fr
-WV A

6V 2.4 a

Fig. 3.226 (&)


for
ur
ib)
Fig. 3.225 The equivalent resistance R' across AB is given by
Solution. The equivalent circuit for the given 1 _ 1 J_ 1 _ 4 _ 2
ks
network is shown in Fig. 3.225(b).
^~2R 2R R~2R R
Yo
oo

R' = R/2
As no current flows through the section AO, so the or
E E 2E
given circuit is a balanced Wheatstone bridge.
eB

Current through arm AB = —=


Hence

3x4
Current through arm DE = 0.
-=L
r

or X =
4 'X Example 181. Calculate the ratio of the heat produced in
ou

2
ad

the four arms of the Wheatstone bridge shown in Fig. 3.227.


The resistance of 10 Q in section AO is not effective. 40 n _ 60 O
Y

B
Solution. As

Total resistance along BAC =2 + 4 =6Q


nd

The bridge is balanced.


Re

Total resistance along BOC = 3 + 6 = 9 Q


P.D. across AB
Fi

These two resistances form a parallel combi “ P.D. across AD 60 ^


nation. The effective resistance between Band C is D
or 40 =60
6x9 18 L
R =
6+9
— =3.6n.
5 or il = “=1.5 E
/2 40
Total resistance in the circuit =3.6 + 2.4 =6 Q. Fig. 3.227
or
I, =1.5 1^
6 V
Current, / = = 1 A Heats produced in time t in different arms of
6D Wheatstone bridge are

Example 180. Six equal resistors, each of value R, are


H
AB = Rt =(1.5 1^)'^ X 40 X t =90 t
joined together as shown in Fig. 3.226(a). Calculate the H
BC = I^xl0xt= (1.5/2)^X10X t = 22.5 ll t
equivalent resistance across AB. If a supply of emf E is
connected across AB, compute the current through the arms
H
AD = Z^x60xt=60/2 t
DE and AB. ICBSE 5P 03] H DC = ^x 15x t=l5 ll t
. \
PHYSICS-XII

Hence the ratio of the heats produced in the four arms is Dividing (1) by (2),
^AB ● ^BC ■ ^AD '■ ^DC 22.5 I
X
120-1
10 100-/ /-20
= 90 ij t:22.5 l^t:60 l\ t
= 90:22.5:60:15 = 6:1.5:4:1. On solving, we get -120/ + 3600 = 0
/ = 60 cm
Example 182. In the following circuit, a metre bridge is
shown in its balanced state. The metre bridge wire has a From (1),
resistance of IQ.cm Calculate the value of the unknown X _ 60 or X=
60x10
= 15 a
resistance X and the current drawn from the battery of 10 100-60 40
negligible internal resistance.
Solution. In balanced condition, no current flows Example 184. The resistances in the two arms of the
metre bridge are R = 5Q and S respectively. When the
through the galvanometer.
resistance S is shunted with an equal resistance, the new
Here P = Resistance of wire A] = ^Q balance length is found to be 1.5 /j, where l.^ is the initial
Q = Resistance of wire BJ =60 balancing length. Calculate the value of S. [CBSE D 19]
R = X,S=60 Solution. For a R s

X 6a balanced metre rWSn


In the balanced con- R.B. D
I
dition. bridge,
G
R _ I
Q~ s
A’* *—40 cm-^1^4 60 cm —●

S~m-l A
P B Q
} c

X li cm (lOO-ii)an
or Iny?rs/ case.
60 “ 6 6 V I- < y
X = 4Q Fig. 3.228
5 h E
K
or
...(/)
Total resistance of wire /IB = 1000 S 100-/j Fig. 3.229

Total resistance of resistances X and 6 Q connec In second case.


5 _ 1.5/i
...{ii)
ted in series =4+6=100 S/2 100-1.5/ 1

This series combination is in parallel with wire AB. Dividing (ii) by (/), we get
Equivalent resistance =
10 X 100 _100 O 1.5f
10 +100 " 11 2 =
(100-1.5/j) /
1
emf of the battery =6 V 100
Current drawn from the battery, On solving, cm

emf 6
7 = = 0.66 A. 100
resistance 100/11 5
From (/),
3 100 ^ 1
^ 100-
100 200 ~2
Example 183. With a certain resistance in the left gap of a
3
slide wire metre bridge, the balance point is obtained when a
resistance of 10 Q is taken out from the resistance box. On S= 5x2n=10a

increasing the resistance from the resistance box by 12.5 Q, the


balance point shifts by 20 cm. Find the unknown resistance. Example 185. In a metre bridge, the null point is found at
a distance of 40 cm from A. If a resistance of 12 is
Solution. With unknown resistance X in the left
connected in parallel with S, the null point occurs at 50.0 cm
gap and known resistance of 10 Q in the right gap, jj-om a. Determine the values ofR and S. (Fig. 3.230)
suppose the balance point is obtained at / cm from the [CBSE D 10) 12 n
zero end. Then ^vw

X /
...(1) rVW, hWVi
10 100 - / R

When the resistance in the right gap is increased by


12.5 n, total resistance becomes 22.5 Q. The balance
point shifts towards zero end by 20 cm.
A c B

X f-20 1-20 1
...(2)
22.5 100-(/-20) 120-/ Fig. 3.230
CURRENT ELECTRICITY 3J11

Solution. point is found to shift by 10 cm towards the end A of the


wire. Find the position of null point if a resistance of 30Q
In first case, were connected in parallel with Y. (CBSE SP 08]
S " 60 " 3
X Y
R 50
In second case, rWVi rVAn
12 S 50
S,
] c
S + 12

or
12 _2 or S = 60
S S + 12 3 .4 ->C

and R=-xS=-x6=4Q i h
3 3

Example 186. A resistance R =2 Cl is connected to one of Fig. 3.232

w
the gaps in a metre bridge, which uses a wire of length 1 m. Solution. In first case,
An unknown resistance X >2 Cl is connected in the other gap X 60
X ^3
as shown in Fig. 3.231. The balance point is noticed at Tfrom
or
y 40 y 2
the positive end of the battery. On interchanging R and X, it

Flo
In second case.
is found that the balance point further shifts by 20 cm (away
from end A). Neglecting the end correction, calculate the
X
55=1

ee
y + 15 50
value of unknown resistance X used. [CBSE OD 08]

Fr
X
X y + 15 3
r©n — X

y X
= -xl
2
[

for
ur
or
G y 2

or y=3oa
A B
ks
{100 - /) cm
X = iy =ix30 = 45Q
Yo

2 2
oo

h
When a resistance of 30 Cl is connected in parallel
with Y, the resistance in the right gap becomes
eB

Fig. 3.231
30y 30 X 30
Solution. In first case, Y' = = 15Cl
30+ y "30+30
R _ I
r
ou

X ” 100-1 Suppose the null point occurs at I cm from end A


ad

Then
In second case.
Y

X I
or 45 _ /
X 1 + 20 1 + 20 15 100-/ 15 100-/
nd

R 100 -(/ + 20) 80-/


Re

or 300-3/ = /
On multiplying tlie two equations, 4/ = 300 1 = 75 cm.
Fi

or or

/ / + 20
1 = X Example 188. When two known resistances, R and S, are
100-/ 80-/
connected in the left and right gaps of a metre bridge, the
or 8000-180/ + /^ = /^+20/ balance point is found at a distance l.^ from the 'zero end' of
X
or 200/=8000 AW
or / = 40 cm
i-AWi hWvn
/ + 20 40+20 R
Now X = R = x2 = 3Q.
80-/ 80-40

Example 187. Figure 3.232 shows the experimental setup A B

of a metre bridge. The null point is found to be 60 cm away


from the end A with X and Y in position as shown. When a
resistance of 15 Cl is connected in series with Y, the null Fig. 3.233 i \
3.112 PHYSICS-Xli

the metre bridge wire. An unknown resistance X is now 4. Calculate the equivalent resistance between the
connected in parallel to the resistance Sand the balance point points A and B of the network shown in Fig. 3.237.
is now found at a distance l^from the zero end of the metre R
(Ans. R)
AAV
bridge wire. Obtain a formula for X in terms o//j, l^ and S. R R
R
[CBSE D 04C, IOC ; OD 09.17] AO- AW AAAr AAV OB
Solution. In first case, R
R I1 AAV
...(/)
S 100 - /1
Fig. 3.237
In second case, 5. Calculate the resistance between the points A and B
R
!i ●●●(n)
of the network shown in Fig. 3.238. (Ans. 80)
xs/(x + s) 100-/2
Dividing (//) by (/), we get
X + S_/2f 100-/ 1 or S
1 + — = /2fl00-/j
X ~T1 100-/,2 j X /j^lOO-/ ij
s
X = AAV
^2 100-/ 1
-1
/
Fig. 3.238
100 - ./
1 \ 2 ;
6. In the experimental set-up of metre bridge shown
roblems For Practice in Fig. 3.239, the null point is obtained at a distance
of 40 cm from A. If a 10 Q resistor is connected in

1. The Wheatstone's bridge series with R^, the null point shifts by 10 cm. Find
of Fig. 3.234 is showing the resistance R that should be connected in parallel
no deflection in the with (/^ + 10) O such that the null point shifts back
to its initial position. IJEE Main Jan 19]
galvanometer joined R1
R2 (Ans. 600)
between the points B
and D. Compute the pWVj AW
value of R.
I-
(Ans. 250) g:
Fig. 3.234
A J B
2. (/) Calculate the equivalent resistance of the given
electrical network between points A and B.
(ii) Also calculate the h ●(4
current through CD and
ACB, if a 10 V d.c. source is Fig. 3.239
connected between A and
7. The potentiometer wire AB shown in Fig. 3.240 is
B, and the value of /? is R
40 cm long. Where should the free end of the galvano
assumed as 20.
meter be connected on AB so that the galvanometer
[CBSE OD 08]
may show zero deflection ? (Ans. 16 cm from A)
[Ans. (/) R^g = RO AW-f—VW-,
(ii) I CD = 0, 8Q 12 0 6n R

/
ACB = 2.5 A]
AAA—f—WA-n
Fig. 3.235
3. Calculate the equivalent resistance between points ©
A and B of the network shown in Fig. 3.236.
11—o- C (►
A D B
20 [CBSE D 99] o
●vw A D B
(Ans. 20)
Fig. 3.240 Fig. 3.241
AO—♦—AA\—f—VSA—0—VW—f—o B
10 100 40
8. The potentiometer wire AB shown in Fig. 3.241 is
AW
20
50 cm long. When AD = 30 cm, no deflection occurs
in the galvanometer. Find R. (Ans. 40)
Fig. 3.236
CURRENT ELECTRICITY 3.113

9. Calculate the value of unknown resistance X and 15. In the metre bridge experimental set up, shown in
the current drawn by the circuit, assuming that no Fig. 3.246, the null point' U is obtained at a distance
current flows through the galvanometer. Assume of 40 cm from end A of the metre bridge wire. If a
resistance of lOQ is connected in series with X, null
the resistance per unit length of the wire AB to be
0.0ia/cm.(Fig. 3.242) ICBSE D 01] point is obtained at AD =60 cm. Calculate the
values of X and Y. ICBSE D 131
(Ans. 6Q, 5.5 A)
(Ans. 8 a, 12 Q)
16. In a metre-bridge experiment, two resistances P
A/vV and Q are connected in series in the left gap. When
X R(4 Q)
the resistance in the right gap is 50 £1, the balance
GO
point is at the centre of the slide wire. If P and Q are
60 cm connected in parallel in the left gap, the resistance
I
A
I in the right gap has to be changed to 12 Q so as to
obtain the balance point at the same position. Find

w
100 cm

P and Q. (Ans. P=30n,Q=20Q)


5V 2.0 V
17. In a metre bridge when the resistance in the left gap
is 2 n and an unknown resistance in the right gap,

Flo
Fig. 3.242 Fig. 3.243 the balance point is obtained at 40 cm from the zero
end. On shunting the unknown resistance with 2 O,
In Fig 3.243, P= 3Q, Q = 2Q, R= 6Q, S =4Q and

ee
10.
find the shift of the balance point on the bridge
X = 5 n. Calculate the current I. [CBSE D 921
(Ans. 22.5 cm)

Fr
wire.

(Ans. 0.6 A) 18. Figure 3.247 shows experimental set up of a metre


11. Each resistor r shown in Fig. 3.244 has a resistance bridge. When the two unknown resistances X and Y
inserted, the null point Dis obtained 40 cm from
of 10 Q and the battery has an emf of 6 V. Find the for
ur
are

current supplied by battery. (Ans. 0.6 A) the end A. When a resistance of 10 D is connected in
X
series with X, the null point shifts by 10 cm. Find
the position of the null point when the 10 Q
ks
r r

AMr ■WS* ► resistance is instead connected in series with


Yo

resistance 'Y'. Determine the values of the resis-


oo

r
tances X and Y. [CBSE D 09]
r r b(_AAA VW- A
eB

.►-►-VvV X Y (Ans. X = 20 Q,
6V rWVi Y = 30n,
B
1- /’ = 33.3 cm]
r
ou
ad

Fig. 3.244 Fig. 3.245 G


Y

12. Find the equivalent resistance between the points X


C
and Y of the network shown in Fig. 3.245. A D

(Ans. 10 Q)
nd
Re

< F
13. In a metre bridge, the length of the wire is 100 cm.
At what position will the balance point be obtained
Fi

Fig. 3.247
if the two resistances are in the ratio 2:3?
HINTS
(Ans. 40 cm)
100 200 100 R
14. In metre bridge, the null point is found at a distance 1.
100 R 40
or
100+ R
= 20

of 60.0 cm from A. If now a resistance of 5Q is


100+ R
connected in series with S, the null point occurs at
50 cm. Determine the values of R and S. (Fig. 3.246). R=25Q.
[CBSE DIO] rWVn
R s 2. Proceed as in Example 178 on page 3.108.
(Ans. 15Q,10fl) □ □
3.
A -1 = -2
As
2 4
A B
.-. The given circuit is a balanced Wheatstone bridge
as shown in Fig. 3.248. The resistance of 10 D is
Fig. 3.246 1 ●( I ineffective.
PHYSICS-XII

7. S _ AD _ I / -16 cm.
12 " DB " 40 - /
AD 30
8. R = 4Q.
AO -o B R DB 50 - 30

9. Resistance of wire y4/ = 60 x 0.01 = 0.60 D


Resistance of wire B/ = 40 x 0.01 = 0.40 H
When no current flows through the galvanometer,
Fig. 3.248 P R
or 0.60 ^ X
Q S 0.40 " 4
We have (1C2 + 2Q) and (2fl + 4fl) combinations
in parallel. X =
0.60 X 4
= 6n
3x6 0.40
R = = 2Q.
3+6 Total resistance of X and R in series = 6 + 4 = lOH
Total resistance of wire AB = 0.60 + 0.40 = l.Otl
4. The given circuit is equivalent to the circuit shown
in Fig. 3.249. The above two resistances are in parallel.
10x1 10
.'.Total resistance of the circuit = = —Q
10+1 11
£MF 5
Ao ■0 B Current, / = = 5.5 A.
Resistance 10/11

10. The circuit is a balanced Wheatstone bridge. Its


effective resistance R is given by
1 1 1 10
- or R = n.
R 3+2 6+4 10 3

Here R^R V
R~ R .'. Current, I = — = 0.6 A.
R 10/3
So it is a balanced Wheatstone bridge. We have T T
n. As - = -, so the given circuit is a balanced
resistances (R + R) and (R + R) in parallel. r r

2Rx2R V\Tieatstone bridge and the resistance r in the


.'. Equivalent resistance = = R.
vertical arm is ineffective. The circuit is then
2R+2R
equivalent to two resistances of 2r and 2r connected
■■ Resistance of 20n is ineffective. in parallel.
2rx2r
We have resistances of (lOQ + lOfl),(lOQ + lOQ) .'. Equivalent resistance, R = = r = 10Q
2r + 2r
and 40 fl in parallel.
-1 ^ ^ Current supplied by the battery of emf 6 V,
+ or R = 8Q.
R 20 20 40 40
/ = ®R A=o.6A.
10
6. In first case,
2. The equivalent circuit is shown in Fig. 3.250.
R2 ~ 60 ~ 3
In second case,
R, + 10 50 = 1
50

● ^ .2 R, =20n
R5 + IO 3 Fig. 3.250

In third case,
30xR The resistances in arm ABare ineffective.
= 20
30+ R 1 1 1 1
+ or R=10n.
30R= 600+20R R = 60Q R 10+10 10+10 10
CURRENT ELECTRICITY

X I ●●●(2)
P-Q= 10
13. For a balanced metre bridge, = ^qq j Solving (1) and (2), P=30£l and Q=20fJ.
17. If X is the unknown resistance, then
But
R " 3 2 40 2x60
or X = = 3Q
2 / X 100-40 40

3 100-/
When resistance X is shunted with 2 H resistor, the
200 effective resistance becomes
or 200-2/= 3/ or / = = 40 cm.
5 X x2 3x2
X' = = i.2n
X + 2”3+ 2
14, In_/irsf case, R^ M^3 ...(/)
S ~ 40 2
Now if the balance point is obtained at distance I'
In second case.
R _50 ...di/ from the left end, then
S+5”50

w
2 /' 2 V
or
5+5 3 X' 100-/' 1.2 100-/'
On dividing (i) by {ii), —^ = - /'= 62.5 cm

Flo
On solving, R = 15 O, S = 10 H Shift in the balance point

15. In/irsf case : X _ 40 _2 = /'-/ = 62.5 - 40 = 22.5 cm.

ee
y” 100-40 3
18. With the unknown resistances X and Y, the balance

Fr
In second case : X + 10 ^ 60 point is 40 cm from the end A .
Y 100-60
X 40 ^2 or Y=-X
X 10^3 Y~ 100-40 3 2
for
ur
y"^ Y 2
or

With 10 Q resistance in series with X, the balance

or 10^3_X ^3_2^5 point is at 40 + 10 = 50 cm from the end A.


ks
Y "2 Y ~2 3 6 X+10^ 50
Yo

10x6
Y "100-50
oo

.-. Y = = 12Q and X =-Y = - xl2 = 8Q.


3 3 or Y=X+10
5
eB

-X = X+10
16, When P and Q are connected in series in the left gap,
or
2

P+ Q^ 50 or X = 20O. and Y= 20+ 10 = 300.


50 ~ 100 - 50
r

When 10 O resistance is connected in series with Y,


ou
ad

P+Q=50n ...(1) let the balancing length be /'.


Y

When P and Q are connected in parallel in the left Then

gaP' X /'
nd

PQ Y+10 100-/'
Re

P+Q^50_^ 20 /'
Fi

or
12 50 30+10 100-/

PQ= 12(P+ Q) = 12x50 = 600 or /' = 33.3 cm.

(P - Q)2 =( p + Q)2 - 4 PQ = 50^ - 4 X 600 = 100

'I
Very Short Answer Conceptual Problems

Problem 1. Is electric current a scalar or a vector Problem 2. Is a wire carrying current charged ?
quantity ? Give reason. Solution. No. The current in a wire is due to flow of
Solutkm. Electric current is a scalar quantity. The electrons in a definite direction,
reason is that laws of ordinary algebra are used to add At any instant, the wire has as much negative charges
electric currents and laws of vector addition do not apply
as the positive charges.
to the addition of electric currents.
3.116 PHYSICS-XII

Problem 3. A steady current is flowing in a cylindrical Problem 11. A large number of free electrons are
conductor. Is there any electric field within the conductor ? present in metals. Why is there no current in the absence
Solution. Yes. It is under the influence of the electric of electric field across it ?

field only that the free electrons flow inside the conductor Solution. In the absence of an electric field, the motion
and constitute the electric current. of the electrons in a metal is random. There is no net flow
Problem 4. How is current kept continuous inside a charge across any section of the conductor. So no
conductor ? current flows in the metal.

Solution. By maintaining a potential difference Problem 12. Two wires of equal lengths, one of copper
between the two ends of the conductor. and the other of manganin have the same resistance.
Which wire will be thicker ? [CBSE OD 12]
Problem 5. For the flow of electricity through gases, /
they should be better exposed to some high energy Solution. As R = ■■■ A=!^
R
radiations like X-rays. Why ?
For both wires R and I are same and
Solution. High energy radiations cause ionisation of
gases ; the resulting positive ions and electrons increase Pcopper ^ Pmanganin
the conductivity of gases.
opper <An anganin
Problem 6. Does the emf represent a force or poten
tial energy or work done per unit charge or potential i.e., manganin wire is thicker than copper wire.
difference ? Does emf have electrostatic origin ? Problem 13. If a wire is stretched to double its original
Solution. The term emf is a misnomer. Literally, emf length without loss of mass, how will the resistivity of
the wire be influenced ?
means the force that pushes the electrons in a circuit. Since [CBSE D 99]
emf does not have simple electrostatic origin, so the Solution. The resistivity of a wire depends on the
concept of potential is not strictly applicable. It has the i^^ture of its material. The increase in length will not affect
nature of work done per unit charge and not that of force. resistivity.
Problem 7. What is the difference between resistance Problem 14. Two wires of equal cross-sectional area,
and resistor ? one of copper and other of manganin have the same
Solution. The property by virtue of which a material resistance. Which one will be longer ? [CBSE F 95]
I RA
opposes the flow of current through it, is called resistance Solution. As R = p — .-. 1 =
and any material that has some resistance is called a resistor. ^ A P

Problem 8. Differentiate between electrical For both wires R and A are same and
resistance and resistivity of a conductor. [CBSE OD 20]
Pcopper Pmar.ganin
Solution. Electrical resistance of a conductor is the
/ >l
ratio of the potential difference applied across its ends to copper manganin

the current flowing through it. Resistance of a conductor i-S-, copper wire is longer than manganin wire,
depends on its shape and size and also on the nature of its Problem 15. There are two conductors A and B of the
material. Resistivity of a material is the resistance offered same material, having lengths / and 21; and having radii
by a unit cube of that material. It depends on the nature of r and r/2 respectively. What is the ratio of their
the material and on the physical conditions like resistances ?
temperature and pressure.
Solution. A = i fll .!(’■/2
\2

Problem 9. What is the cause of resistance of a = -=1:8.


conductor ? ^ 21
^2 VI r 8

Solution. While drifting, the free electrons collide with Problem 16. Two wires A and B are of same metal,
the ions and atoms of the conductor, i.e., motion of the have the same area of cross-section and have their
electrons is opposed during the collisions. This is the basic lengths in the ratio 2 : 1. What will be the ratio of
cause of resistance in a conductor. currents flowing through them respectively when the
Problem 10. Resistivities of copper, constantan and silver same potential difference is applied across the length of
each of them ?
are 1.7 xl0“® Q m, 39.1 xlO"® Q m and 1.6 xlO-8 [CBSE D 98]
n m. V UA /
Solution. Current,
respectively. Which has the best conductivity ? R P^ r-K=p^]
A
[CBSE D 96C]
1 For the two wires p, A and V are same, therefore
Solution. Conductivity = 1
/oc-
Resistivity I
As silver has the lowest resistivity, so it has the best Hence -=1:2
conductivity. I 2
[V /^:/b=2:1]
B
CURRENT ELECTRICITY 3J17

Problem 17. Two wires A and B of the same metal Problem 23. When electrons drift in a metal from
and of the same length have their areas of cross-section lower to higher potential, does it mean that all the free
in the ratio of 2 :1. If the same potential difference is electrons of the metal are moving in the same direction ?
applied across each wire in turn, what will be the ratio (CBSED 12]
of the currents flowing in A and B ? (CBSE D 98] Solution. No, only the drift velocities of the electrons
V VA
Solution. Current, / = — = —
are
superposed over their random thermal velocities.
R p/ Problem 24. How does the drift velocity of electrons
For the two wires p, / and V are same, therefore in a metallic conductor vary with the increase in
temperature ? (CBSE D 02 ; OD 16]
I X A
Solution. The drift velocity decreases because of the
Hence increase in collision frequency of free electrons at higher
I.B A, 1 temperature.
Problem 18. Plot a graph showing the variation of Problem 25. If the temperature of a good conductor

w
resistance of a conducting wire as a function of its radius, increases, how does the relaxation time of electrons in
keeping the length of the wire and its temperature as the conductor change ? (CBSE OD 02]
constant. [CBSE F 13] Solution. With the increase in temperature, the

Flo
Solution. electrons collide more frequently with positive metal ions.
/ /
So their relaxation time decreases.
t
R = p-=p —

ee
A nr Problem 26. A potential difference V is applied
across a conductor of length 1. How is the drift velocity

Fr
1
i.e., R «-=■ affected when V is doubled and / is halved ?
r
[CBSE F 10]
Hence the graph of R Solution. Drift velocity,
for
ur
versus r is of the form shown
Fig. 3.251 eEz eV X elVx
in Fig. 3.251. V
d -
v", =
m ml m(//2)
Problem 19. The current flowing through a con
ks
ductor is 2 mA at 50 V and 3 mA at 60 V. Is it an ohmic or When V is doubled and I is halved, drift velocity
Yo

non-ohmic conductor ? (CBSE F 94] becomes 4 times original v^.


oo

Solution. The resistances in the two cases are Problem 27. Two conducting wires X and Y of same
diameter but different materials are joined in series across
eB

p-jj- 50 _ = 250000 a battery. If the number density of electrons in X is twice


^ I, 2x10-3 that in Y, find the ratio of drift velocity of electrons in the
two wires. (CBSE OD 10]
r

v; 60
2 _
ou

= 200000
^ /2 3x10-3
ad

Solution. In series.

- = 1:2.
en^Av^{X) = enyAv^j{Y) or
Y

or
As the resistance changes with current, so the given
conductor is non-ohmic.
Problem 28. When we switch on an electric bulb, it
nd

Problem 20. Does the value of resistance of a


Re

lights up almost instantaneously though drift velocity


conductor depend on the potential difference applied electrons in copper wires is very small. Give reason,
Fi

across it or the current passed through it ? Solution. When the electric circuit is closed, electric
Solution. The resistance of conductor is constant field is set up throughout the circuit almost instantly with
provided the potential difference applied or current the speed of light. Electrons in every part of the circuit
passed is not large. begin to drift in the opposite direction of electric field. A
Problem 21. Is the formula V=IR tree for non-homic ‘^“■'rent starts flowing in the circuit almost
instantaneously.
resistance also ?
Problem 29. Of metals and alloys, which have greater
Solution. Yes, the formula defines resistance and not
value of temperature coefficient of resistance.
Ohm's law. (CBSE D 95C]
Problem 22. Is V = TB Ohm's law ? Give correct form
Solution. Metals have greater value of temperature
of Ohm's law. coefficientof resistancethan alloys.
Solution. V = IRis not Ohm's law. It only defines and Problem 30. Why alloys like constantan and
gives value of resistance R. It is Ohm's relation. manganin are used for making standard resistors.
(CBSE D 16]
V' oc /, is Ohm's law.
3.118 PHYSICS-XII

3
Y
Solution. (0 The temperature coefficient of resistance Problem 37. Plot a graph I

for alloys is low. (ii) Alloys have has a high value of showing the variation of
resistivity. current ‘I' versus resistance 'R

Problem 31. Draw a graph showing variation of connected to a cell of emfg and
resistivity with temperature for nichrome. Which pro internal resistance Y.
perty of nichrome is used to make standard resistance [CBSE SP 18]
coils ? [CBSE OD 13C] e
Solution. 1 =
Solution. See Fig. 3.21(b) on page 3.24. Nichrome has a R+r Fig. 3.252
R

very small value of temperature coefficient of resistance


(a = 1.7 xlO"^°C“^). For this reason, it is used to Graph of / versus R is shown in Fig. 3.252.
make
standard resistance coils. Problem 38. You are given a primary and a secon
Problem 32. Light from a bathroom bulb gets dimmer dary cell of the same emf. From which cell will you be
able to draw larger current and why ? [CBSE OD 94]
for a moment when the geyser is switched on. Why ?
[Haryana 95] Solution. The internal resistance of a primary cell is
Solution. The current drawn by the high powered more than that of a secondary cell, so a secondary cell will
geyser from the supply initially lowers the voltage for a provide a larger current.
moment until the voltage gets stabilised by the Problem 39. A car battery has an emf of 12 V. Eight
transmission grid. ordinary cells connected in series can also supply 12 V.
Problem 33. When a motor car is started, the car But we do not use such an arrangement of cells in the
lights become slightly dim, why ? car. Why ?
Solution. When a motor car is started, its starter takes Solution. Eight cell arrangement cannot provide the
a high current from the battery, so a large potential drop high current needed to start the car engine because it has a
{V = Current X internal resistance) occurs at the terminals high internal resistance of about lOD while the internal
of the battery and the bulbs get dim. resistance of a car battery is just 0.1 Q.
Problem 34. Two students A and B were asked to Problem 40. Why does the conductivity of a semi
pick a resistor of 15 kD from a collection of carbon conductor increase with rise of temperature ?
resistors. A picked a resistor with bands of colours : Solution. As temperature increases, covalent bonds
brown, green, orange while B chose a resistor with begin to break in the semiconductor, setting free more and
bands of black, green, red. Who picked the correct more electrons. So the conductivity increases.
resistor ? [CBSE OD 13C] Problem 41. Two materials Si and Cu (or GaAs and
Solution. Ag) are cooled from 300 K to 60 K. What will be the effect
For student A: For student B : on their resistivity ? [CBSE F 13]
Brown Green Orange Black Green Red Solution. The resistivity of Si (a semiconductor)
i i i I i increases and that of Cu (a metallic conductor) decreases,
when cooled from 300 K to 60 K.
1 5 3 0 5 2
Problem 42. Why resistance becomes more in series
R^ = 15x10^0 = 15kn Rg =05x10^ = soon combination ?

Hence, the student A picked up the correct resistor of Solution. In series combination, the effective length of
15 kn. conductor increases. As Rk I, so resistance increases.

Problem 35. Is internal resistance a defect of a cell ? Problem 43. Why resistance becomes less in parallel
combination ?
Solution. Yes, because a part of the electrical energy
Solution. In parallel combination, the effective area of
obtained from the cell is used in sending current through
the interior of the cell itself due to its internal resistance. cross-section of the conductor increases. As R cc 1/ A, so
resistance decreases.
Problem 36. Why is the terminal voltage of a cell less
than its emf ? Problem 44. Why is a potentiometer named so ?
[CBSE D 13, OD 13C]
Solution. EMF = Terminal p.d. -f- P.D. across internal Solution. Because it is used to measure potential
difference.
resistance

E=V+Ir Problem 45. State the principle of working of a


potentiometer. [CBSE F 09 ; OD 14C ; D 16]
A real cell has always some internal resistance, a part
Solution. A potentiometer works on the principle that
of the emf is consumed in doing work against the internal
resistance in a closed circuit. when a steady current flows through a wire of uniform
cross-section and composition, the potential drop across any
So g> or V <E.
length of the wire is directly proportional to that length.
CURRENT ELECTRICITY 3.119

Problem 46. Why should the potentiometer wire be Solution. A potentiometer is said to be sensitive if (i) it
of uniform cross-section and composition ? (OD 14C) can measure very small potential differences, and {ii) for a
Solution. Only then it will have same resistance per small change in potential difference being measured, it
unit length throughout. Then potential difference will be shows a large change in balancing length.
proportional to length of the wire, as required by the Problem 56. How can the sensitivity of potentio
meter be increased ? ICBSE D IIC)
principle of potentiometer.
Problem 47. Why should the material of the Solution. The sensitivity can be increased by reducing
potentiometer wire be of high specific resistance ? the potential gradient. This can be done by (f) increasing
Solution. This makes the resistance of the entire length the length of the wire and (it) by reducing the current in
the main circuit.
of the wire sufficiently large and hence for a given
current, there is an appreciable potential drop. Problem 57. How can you make a potentiometer of
Problem 48. Why should the material of the potentio given wire length more sensitive using a resistance box ?
meter wire be of low temperature coefficient of resistance ? Solution. This can be done by introducing some

w
Solution. A material having low temperature resistance in the circuit through the resistance box. This
coefficient of resistance ensures that its resistance does decreases the current in the circuit. Consequently, the
not change appreciably due to heating. potential gradient decreases and hence sensitivity of the

Flo
potentiometer increases.
Problem 49. Of which material is a potentiometer
wire normally made and why ? [CBSE D nc] Problem 58. Why do we prefer a potentiometer with a

ee
longer bridge wire ? [CBSE OD 14C|
Solution. The potentiometer wire is usually made of
Solution. A potentiometer with a longer bridge wire

Fr
an alloy such as nichrome or manganin. Such an alloy has
high resistivity and low temperature coefficient of resistance. has a small potential gradient. Consequently, it is more
sensitive and hence preferred.
Problem 50. Can we use copper wire as a potentio
Problem 59. Why is a ten-wire potentiometer more
for
ur
meter wire ?
sensitive than a four-wire one ?
Solution. No. Resistivity of copper is small, so there
will not be an appreciable potential drop across the ends Solution. This is because potential gradient for ten-
wire potentiometer is smaller tlian that for a four-wire one.
ks
of potentiometer wire. Also temperature coefficient of
Yo

resistance of copper is large. Problem 60. Why is a Wheatstone bridge so called ?


oo

Problem 51. Why should the current be not passed Solution. It is so called because this method was first
through potentiometer wire for a long time ? suggested by a British physicist. Sir Charles F. Wheatstone
eB

Solution. This will heat up the potentiometer wire and in 1843. It is called a bridge because the galvanometer
will change its resistance. Potential drop per unit length of circuit forms a kind of bridge by cormecting two points
the wire will also change. having the same potential.
r
ou
ad

Problem 52. What type of cell should be used in the Problem 61. When is a Wheatstone bridge said to be
main circuit of the potentiometer and why ? balanced ?
Y

Solution. A Leclanche cell should be used in the main Solution. A Wheatstone bridge is said to be balanced if
circuit of the potentiometer. This is because of the fact that no current flows through its galvanometer arm. When the
nd
Re

Leclanche cell is useful, when the current is drawn for a Wheatstone bridge is balanced.
short time. Q S
Fi

Problem 53. The emf of the driving cell used in the Problem 62. What do you mean by sensitiveness of a
main circuit of the potentiometer should be more than Wheatstone bridge ?
the potential difference to be measured. Why ? Solution. A Wheatstone bridge is said to be sensitive if
[CBSE OD 14C1 it produces more deflection in the galvanometer for a
Solution. If it is not so, the balance point will not be small change of resistance in resistance arm.
obtained on the potentiometer wire. Problem 63. When is Wheatstone bridge most sensitive ?
Problem 54. Why should the jockey be not rubbed Solution. When all the four resistors P, Q R and S are

against the potentiometer wire ? nearly of the same magnitude.


Solution. Rubbing of jockey against the potentiometer Problem 64. Why is a slide wire bridge or metre
wire affects the uniformity of the cross-sectional area of bridge named so ?
the wire and hence changes the potential drop across the Solution. As in it a jockey is made to slide over the
wire.
bridge wire to get a null point, so it is called a slide-wire
Problem 55. What is meant by the sensitivity of a bridge. As it uses one metre long wire, so it is called a
potentiometer ? [CBSE D 03) metre bridge.
3.120 PHYSICS-XIl

Problem 65. Why should we get the null point in the Problem 72. What happens if the galvanometer and
middle of the metre bridge wire ? [CBSE D lie ; F 13] cell are interchanged at the balance point of the bridge ?
Solution. The metre bridge is most sensitive when the Would the galvanometer show any current ?
four resistances forming the Wheatstone bridge are equal. Solution. When galvanometer and cell are inter-
This is possible only if the balance point is somewhere changed, condition for balance of the bridge remains
near the middle of the wire. satisfied. So galvanometer will show no current.
Problem 66. What is end error in a metre bridge ? Problem 73. A household circuit has a fuse of 5 A
How is it overcome ? [CBSE D 19] rating. Calculate the maximum number of bulbs of
Solution. The end error in a metre bridge is an error rating 60 W - 220 V each which can be connected in this
which arises due to household circuit. [CBSE SP 031
(/) Ends of the wire not coinciding with the 0 cm/ Solution. Current drawn by one bulb
100 cm marks on the metre scale.
_ P _ 60 1a
(/[) Presence of contact resistance at the joints of the “ V “ 220 11
metre bridge wire with the metallic strips.
No. of bulbs that can be safely used with 5 A fuse
The end error can be avoided/overcome by finding 5 55
balancing lengths with two interchanged positions of R — = 18.33
3/11 3
and Sand taking the average value of S from two readings.
Problem 67. What are the advantages of a Wheat Hence 18 bulbs can be safely used with 5 A fuse.
stone bridge method of measuring resistance over other Problem 74. A constant voltage is applied between
methods ? the two ends of a uniform metallic wire. Heat Q is
Solution, (i) It is a null method, hence the result is free developed in it. If another wire, double the radius and
from the effect of extra resistances (cell resistances) of the twice the length is used, how much heat will be
circuit, (ii) Being null method, it is easier to detect a small developed in it ? (CBSE OD 96C]
change in deflection than to read a deflection directly. Solution. Q = t
2
. .t
Problem 68. Why are the connecting resistors in a R p/
metre bridge made of thick copper strips ? [CBSE F 13|
n(2rf
Solution. Thick copper strips offer minimum resistance Q'=v2. ● f=2Q.
p.2!
and hence avoid the error due to end resistance which
have not been taken into account in the bridge formula. Problem 75. Two wires A and B of the same material

Problem 69. Why is Wheatstone bridge (or metre and having same length, have their cross-sectional areas
in the ratio 1 : 4. What would be the ratio of heat
bridge) method considered unsuitable for the measure
ment of very low resistances ? produced in these wires when same voltage is applied
across each ? [CBSE SP 97]
Solution. For measuring low resistance, all other pi pi RA
resistances used should have low value to ensure the Solution. R^= — RB ~ =4
A ' 4A RB
sensitivity of the bridge. This requires a galvanometer of
very low resistance which itself would be very sensitive. Ratio of heats produced in the two wires will be
Also, the end-resistances and resistances of connecting
wires become comparable to the resistance being Ea_y111a ___
R 1
= 1:4.
B _

measured and may introduce error in the result. Pg K^/Rg R^ 4


Problem 70. Why is metre bridge method considered Problem 76. The element of the heater is very hot
unsuitable for the measurement of very high resistance ? while the wires carrying current are cold, why ?
Solution. For measuring high resistance, all other resis Solution. Heat produced, H = J^Rt .
tances forming tlie bridge should also be high so as to ensure Both heater element and conducting wires carry same
the sensitivity of the bridge. But this reduces the current current. But the heater element becomes very hot due to
through the galvanometer which becomes insensitive. its high resistance and the conducting wires remain cold
Problem 71. Why is Wheatstone bridge method due to their low resistance,
suitable for comparing the resistances of the same order Problem 77. Though same current flows through the
of magnitude ? [CBSE F 99] electric line wires and the bulb filament, yet only the
Solution. Metre bridge is based on the principle of filament glows. Why ?
Wheatstone bridge which is most sensitive when all the Solution. The bulb filament has a much higher
resistances are nearly of the same magnitude. So metre resistance than the electric line wires. Current passed
bridge is suitable for comparing the resistances of the through the high resistance filament produces a large
same order of magnitude. amount of heat {H = I^Rt) and makes it glow.
CURRENT ELECTRICITY 3.121

Problem 78. The temperature of the filament of an Solution.In parallel, same voltage V is applied to both
electric bulb is 2700°C when it glows. Why is it not the bulbs. But 500 W bulb has a smaller resistance
burnt up at such a high temperature ? {R = V^ ! P), so it will produce more heat per second
Solution. This is because the lamp's filament has high (P= / R) and will glow brighter than 100 W bulb.
melting point and is held in an atmosphere of inert gases Problem 85. Two heating coils, one of fine wire and
which prevent its oxidation. the other of thick wire, made of same material and of the
Problem 79. A toaster produces more heat than a light same length are connected in turn to a source of emf.
bulb when connected in parallel to the 220 V mains. Which of the coils will produce more heat ? [CBSE D 99C]
Which has a greater resistance ? [CBSE OD 2000C]
Solution. As P = , hence for same V, the thicker
Solution. The amount of heat produced by an electric R
current in time t is given by wire will produce more heat because of its smaller
resistance.
vh .
H=I^Rt =

w
i.e.. Hoc-
R R Problem 86. Why an electric bulb becomes dim when
an electric heater in parallel circuit is switched on ? Why
For the parallel combination, potential difference V is dimness decreases after some time ?
same. As toaster produces more heat, so light bulb should

Flo
have a greater resistance. Solution. Electric heater with more power has a
smaller element resistance. It draws a higher current.
Problem 80. Heat is generated continuously in an Some current from the bulb is diverted into heater. Bulb

ee
electric heater but its temperature becomes constant
becomes dim. After some time, heater element becomes
after some time. Why ?

Fr
hot. Its resistance increases. It takes less current. Current
Solution. When the temperature of the heater becomes through the bulb increases and its dimness decreases.
greater than the temperature of its surroundings, it begins
to lose heat to surroundings. A stage is reached when the Problem 87. Why does a bulb not become dim when
for
ur
rate of production of heat becomes equal to the rate of loss another bulb of same power in parallel circuit is
switched on ?
of heat and the temperature becomes constant.
Problem 81. A current is passed through a steel wire Solution. As the two bulbs have same power, their
ks
filament resistance is also same. When second bulb is
heating it to red hot. The half of the wire is immersed in
Yo

cold water. Which half of the wire will heat up more and switched on in parallel, total resistance decreases to one-
oo

why ? half and current becomes double. The double current divides
eB

Solution. The temperature of the wire outside the itself equally in the two bulbs. Current through each bulb
water is more than that inside the water. As resistance of is same as the original one. So there is no dimness.
the wire increases with temperature, the resistance of the Problem 88.Nichrome and copper wires, having same
r

part of the wire outside the water will also be higher. length and same area of cross-section, are connected
ou
ad

Since same current is flowing through the two parts, more across a battery in turn. In which case the rate of
heat will be produced in the part outside the water. production of heat will be higher ?
Y

Problem 82. The maximum power rating of a 20 Q Solution. As the resistivity of copper is less than that
resistor is 2 kilowatt. Can it be connected to a 220 V d.c. of nichrome, so resistance of copper wire is less than that
nd
Re

supply ? of nichrome wire of same length and diameter.


Solution. The maximum voltage that can be applied
Fi

across the resistor is


As rate of production of heat, P = R
1
V
max = = ^2000 X 20 = 200 V For given V, Pk—
R
So the given resistor cannot be connected to a 220 V
d.c. supply. Hence rate of production of heat in copper wire is
higher than that in nichrome wire.
Problem 83. Three bulbs 40 W, 60 W and 100 W are
Problem 89. Nichrome and copper wires of same
connected to 220 V mains. Which bulb will glow brightly,
if they are connected in series ? length and same radius are connected in series. Current
(Himachal 01]
I is passed through them. Which wire gets heated up
Solution. In the series circuit, the same current flows
more ? Justify your answer. (CBSE OD 17]
through each bulb. But the 40 W bulb has the highest
resistance {R=V^/P). The 40 W bulb produces Solution. In series, the current I is same through both
maximum heat per second (P= I^R), so it will glow more wires. Rate of production of heat, P = I^R Pec R
brightly than the other two bulbs. Nichrome wire is heated up more because resistivity
and hence resistance of nichrome wire is much more than
Problem 84. A 100 W and a 500 W bulb are joined in
parallel to the mains. Which bulb will glow brighter ? that of copper wire of same length and radius.
3A22 PHYSICS-XII

Problem 90. A heater joined in series with a 60 W bulb Problem 93. Equation P = 7^R seems to suggest that
is connected to the mains. If 60 W bulb is replaced by a the rate of joule heating in a resistor is reduced if the
100 W bulb, will the rate of heat produced by the heater resistance is made less, however, the equation P = /R
be more, less or remain the same ? [CBSEODOIC] seems to suggest the opposite. How do you reconcile
Solution. The resistance of the 100 W bulb is less than
● this apparent paradox ?
Solution. There is no paradox in the problem. When V
that of 60 W bulb. When 60 W bulb is replaced by 100 W
is constant and R is reduced, there is an increase in current
bulb, the resistance in the series circuit decreases and
and consequently an increase in joiale heating.
current increases. Hence the rate of heat produced by tire
heater will be more.
Problem 94. If three electric bulbs of 100 W, 50 W and
25 W connected in the electric fittings of a house are
Problem 91. A 60 W electric bulb connected in
lighted by the electric supply, what will be the total
parallel with a room heater is further connected across power consumed ?
the mains. If 60 W bulb is replaced by 100 W bulb, will Solution. Total power consumed
the heat produced by the heater be smaller, remain the = 100 + 50+ 25= 175 W
same or be larger and why ?
This is because the bulbs in house fitting are connected
Solution. In parallel connection, a part of current from in parallel and total power consumed is equal to the sum
the mains flows through the 60 W bulb and remaining powers of the individual bulbs,
current flows through the heater. When 60 W bulb is Problem 95. If a battery is short-circuitedby means of
replaced by 100 W bulb, the 100 W bulb now draws larger a heavy copper wire, its temperature irses. Why ?
current due to its smaller resistance than 60 W bulb, Tlie

current through the heater decreases and it produces Solution. Rate of production of heat, P = R
smaller heat.

Problem 92. Two 120 V light bulbs, one of 25 W and As the resistance of heavy copper wire is negligibly
other of 200 W were connected in series across a 240 V small, the rate of production of heat will be quite high, so
the temperature rises.
line. One bulb burnt out almost instantaneously. Which
Problem %. How does the use of a fuse wire protect
one was burnt and why ? ICBSE F 04]
the electrical installations ?

Solution. As R = — , so 25 watt bulb has more Solution. A fuse wire has high resistance and low
P
melting point. It is connected in series with the electric
resistance. In the series circuit, same current flows
installations. When the current exceeds the safety limit,
through botli the bulbs. The 25 W bulb develops more heat the fuse wire melts away. The circuit breaks and electrical
(H = I^Rt) and hence bums out almost instantaneously. installations are saved.

Short Answer Conceptual Problems

Problem 1. Clarify your elementary notions about electrons of the metal are moving in the same
current in a metallic conductor by answering the direction ? (CBSE D 12]
following queries : (e) Are the paths of electrons straight lines
(rt) The electron drift speed is estimated to be between successive collisions (with the
only a few mms~^ for currents in the range of positive ions of the metal) in the (0 absence
a few amperes ? How then is current established of electric field, (n) presence of electric field ?
almost the instant a circuit is closed ? [NCERT]
ib) The electron drift arises due to the force Solution, (fl) Electrons are present everywhere in an
experienced by electrons in the electric field electric circuit. As soon as the circuit is closed, electric
inside the conductor. But force should cause
field is set up throughout the circuit almost instantly
acceleration. Why then do the electrons (with the speed of light). Electrons in every part of the
acquire a steady average drift speed ? circuit begin to drift in the opposite direction of electric field.
(c) If the electron drift speed is so small, and the A current starts flowing in the circuit almost instantly.
electron's charge is small, how can we still
obtain large amounts of current in a con {h) Due to the force exerted by the external electric
ductor ?
field, each 'free' electron inside the conductor does accele

id) When electrons drift in a metal from lower to


rate and increase drift speed until it collides with a
positive ion of the metal. It loses its drift speed after collision
higher potential, does it mean that all 'free'
CURRENT ELECTRICITY 3,123

but starts to accelerate and increase its drift speed again {b) The internal resistance of a car battery decreases
only to suffer a collision again and so on. On the average, with increase in temperature. Therefore, on a warm day a
therefore, the electrons acquire only a drift speed, car battery gives large current which helps in starting the
(c) The current in a conductor is given by car engine,
/ = en Avj (c) It mainly differs in internal resistance but partly in
Although the electron's charge e and drift speed are its emf also. A nearly discharged lead acid secondary cell
very small quantities, yet we can obtain a large amount of possesses a very high internal resistance.
current in a conductor. This is because the free electron Problem 3. Choose the correct alternative :

density (number of electrons per unit volume) of a (fl) Doping a semiconductor (with small traces of
conductor is very large -10 m"'^. The drifts of a very impurity atoms) reduces/increases its resis
large number of free electrons add up to cause a large tivity.
current inside the conductor.
(&) The resistance of graphite and most non-

w
(d) No, only the drift velocities of the electrons are metals increases/decreases with increase in
superposed over their random (haphazard) thermal temperature,
velocities. In Fig. 3.253, the solid line shows the random (c) The resistivity of a semiconductor increases/
path followed by a free electron in the absence of an

Flo
decreases rapidly with increasing temperature.
external field ; the electron proceeds from A to B, making
six collisions on its path. The dotted curve shows how the Solution, (a) reduces (b) decreases (c) decreases.

ee
Problem 4. Write dimensional formula of electrical
random motion of the same electron gets modified when
an electric field is applied. Clearly, the electron drifts conductivity. [IIT 97]

Fr
slowly to the right, ending its motion at B rather than at B. Solution. Electrical conductivity
1 Length
Resistivity Resistance x Area
for
ur
L
.●. [Electrical conductivity] = ML^T-^A"^
ks
Yo

Problem 5. Two wires of the same material having


oo

lengths in the ratio 1: 2 and diameters in the ratio 2 ; 3


are connected in series with an accumulator. Compute
eB

the ratio of P.D. across the two wires.


Fig. 3.253 [CBSE SP 90]
r

Solution. Resistance,
(e) (/) In the absence of electric field, the paths are
ou
ad

straight lines, {ii) In the presence of electric field, the paths / / 4p/
R = P-=P
are curved. ( dY
Y

K
n

Problem 2. Answer the following questions : u


(a) It is easier to confine electric current to definite
nd

Ip'a
Re

paths (by the use of electric insulators) than


to direct heat flow along definite routes using
D
B
1 '3^9
2 I2J "8
Fi

= — X

heat insulators. Why ? R, ipl B /B

(b) It is easier to start a car engine on a warm day tiDY


B

than on a chilly day. Why ? P. D. across A


A
(c) In which respect, does a nearly discharged P.D, across B B
lead acid secondary cell differ mainly from a
freshly charged cell in its emf or in its
internal resistance ? RB 8

Solution, (r?) Typical electrical insulators {e.g., glass) [v For series resistances, = Ig]
differ in their resistivity from metals enormously, by a Problem 6. If the current supplied to a variable
factor of the order of 10^. The corresponding factor for resistor is constant, draw a graph between voltage and
thermal insulators versus thermal conductors is only 10^. resistance. [ISCE 96]
So it is easier to send electric currents to definite paths by
Solution. According to Ohm's law,
the use of insulators than to direct heat flow along definite
V^IR
routes using heat insulators.
3.124 PHYSICS-XII

V.
For constant 1, Hence graph 1 represents the series combination of
VxR other two resistors.

Hence the graph


between voltage V and
resistance R will be a

straight passing through R


the origin, as shown in
Fig. 3.254. Fig. 3.254

Problem 7. I-V graph for a metallic wire at two


different temperatures, and is as shown in
Fig. 3.255. Which of the two temperatures is lower and Fig. 3.256 (b)
why ? [CBSE OD 151 Problem 9. The I-V characteristics of two wires X and

I Y at constant temperatures are shown in Fig. 3.257. The


two wires have equal lengths and diameters. Which of
them is made up of material of larger resistivity ?
/I
7-2 [CBSE 20C1
I X

h
Y
V,0

Fig. 3.255
V
Solution. For the same potential VJ0 '

V, Fig. 3.257
Resistance at 7J, ^=-7^
1 Solution. Slope of I-V line for wire X > Slope of I-V
line for wire Y.
V0
Resistance at 7^, Rj = .’. Conductance of wire X > Conductance of wire Y
^2
or Resistance of wire X < Resistance of wire Y
R, <^2 / /
or
Since resistance of a metal increases with temperature, Px-^<Py^
/I 71
or
Px <Py
therefore, 7] < 7^- Thus wire Y has a larger resistivity.
Problem 8. The V-I graphs of two resistors, and their
Problem 10. The voltage-current graphs for two
series combination, are shown in Fig. 3.256(a). Which resistors of the same material and the same radii with
one of these graphs represents the series combination of
lengths Lj and Lj are shown in Fig. 3.258. If Lj > Lj, state
the other two ? Give reasons for your answer.
[CBSE SP 03) with reason, which of these graphs represents voltage-
!
current change for L^. (CBSE D 03]
3
2

Fig. 3.256 (a)


Fig. 3.258
Solution. Refer to Fig. 3.256(b), various resistances are
1/ y Solution. As Lj > so Rj > R2. Now the graph Bhas a
h ^"7'
‘2 ^"7
h
higher slope (V /1), so it corresponds to a greater
resistance. Clearly, graph B represents voltage-current
But
I^ < I2 < I^ ^ ^ ^ change for length Lj.
CURRENT ELECTRICITY 3.125'

Problem 11. The variation of the resistance of a Problem 13. Using the concept of drift velocity of
metallic conductor with temperature is shown in Fig. 3.259. charge carriers in a conductor, deduce the relationship
between current density and resistivity of the
A
(CBSE D 98C) conductor. (CBSE D 15C]

Solution. By the concept of drift velocity,


a eEx m
<u
u
} = enAvj, Vd -
in
/ P =
ne\
c
r: R
(/>
fA Current density,
cv
f
a
R0 7 etiAv^ = en.
eEx

i ^"a~' a m

O
Temperature (®C) 6 /
ne
2
T
or ; = E = -E .
Fig. 3.259 m P

w
(i) Calculate the temperature coefficient of resis Problem 14. A potential difference V is applied to a
tance from the graph. conductor of length /, diameter D. How are the electric
Hi) State why the resistance of the conductor field E, the drift velocity and the resistance R affected

Flo
increases with the rise in temperature. when (0 V is doubled in) I is doubled (in) D is doubled ?
Solution. (0 Let R be the resistance and 0 be the [CBSE OD 97]

ee
temperature of the conductor corresponding to some Solution. Electric field, E=
point A on the graph. Then temperature coefficient of the I

Fr
conductor will be R-R, eE eV
a =
Drift velocity. — X = —- X
RqxO m ml

for
ur
(jj) The resistance of a conductor increases with the ! 4p/
Resistance,
rise in temperature due to the increase in collision A~nD^
frequency of electrons with the positive metal ions.
ks
(i) When V is doubled, £ becomes double,
Problem 12. A wire of length Lq has a resistance Rq . It becomes double and R remains unchanged,
Yo
oo

is gradually stretched till its length becomes 1.5 Lq. (/■j) When / is doubled, E becomes half, becomes
half and R becomes double.
eB

(/) Plot the graph showing variation of its


resistance R with its length L during stretching, {Hi) When D is doubled, E remains unchanged,
(if) What will be its resistance when its length is also unchanged and R becomes one-fourth.
r

becomes 1.5 Lr.0 ? [CBSE OD 20] Problem 15. The current I flows through a wire of
ou
ad

radius r and the free electrons drift with velocity v^.


Solution, (i) When the wire is stretched to n times its
What is the drift velocity of electrons through a wire of
Y

original length, its new resistance becomes


same material but having double the radius, when a
f rf
R' = n^R= — R = ^.^ current 21 flows through it ?
nd
Re

I I
Solution. Initial drift velocity, = eiiA en x
Graph of resistance R' vs. length E is a parabola as
Fi

shown in Fig. 3.260 For a wire of same material (hence same n) and double
R'A the radius, the drift velocity with current 2 / will be
4R 27 1 I 1
V

3R
enxK (2r)^ 2 ‘ en x
2.25R
Problem 16. The following table gives the length of
2R
three copper wires, their diameters, and the applied
R potential difference across their ends. Arrange the wires
in increasing order according to the following :
I-o 1.5 L 0 2L 0 L (fl) The magnitude of the electric field within
them,
Fig. 3.260 (b) The drift speed of electrons through them,
and
(ii) When L= 1.5 Tq, R' = (1.5)^ R = 2.25R.
.126 PHYSIC5-XII

(c) The current density within them. [CBSE SP 18] Solution, (fl) In the absence of any electric field, the
Wire No. Length Diameter Potential difference
free electrons of a conductor move randomly in all
1 I 3d
directions due to thermal energy. At room temperature,
V
this random velocity is of the order of 10^ ms ’ and it does
2 21 d V not cause net of flow of charge in any direction.
3 31 2d 2V Tlie average velocity acquired by the free electrons of
Solution.
a conductor in the opposite direction of the externally
applied electric field is called I'rf
V 2V^
drift velocity. Drift velocity is
3/ very small, typically 1 mm/s. \
X

(b) Asv^ <x E dl (b) Current density,


ic)j = envj => jccv^ h<h<Jv j=env^

Problem 17. The thickness of a conductor conti Graph between j and ;


is a straight line.
nuously decreases from its one end (A) to another end Fig. 3.262
(B). It is connected across the terminals of a battery.
What will be the effect on the value of Problem 19. («) Write the relationship between
(a) electric field, mobility and drift velocity in a current-carrying conductor.
(b) current density, and
(c) mobility of the electron (b) Two aluminium wires have their lengths in the
ratio 2 : 3 and radii in the ratio 1: 3. These are connected
at a point on the conductor as one moves from end A to
end B ? in parallel across a battery of emf € and of negligible
[CBSE D 20C] internal resistance. Find the ratio of drift velocities of
Solution. A conductor of non-uniform cross-section is electrons in the two wires, [CBSE F 20]
a series combination of many conductors of non-uniform Drift velocity or u = — .
cross-section. So current I through them remains constant. Solution, (fl) Mobility =
Electric field E
B (b) Let the lengths be 21 and 3/ and area of
\!ijniJiD cross-sections A and 9 A.
L. For wires connected in parallel, V =
+ 21 3/ V
xp— ~enx9Av^2
- -

- = 3:2.
A 9A V 2

Fig. 3.261 Problem 20. Two metallic wires, Pj and P2 of the same
material and same length but different cross- sectional
(«)£ =
y _ jp _ / pi Ip =>
1
£cc — areas, A^ and A^ are joined together and connected to a
/ " / "7’A “7 A
source of emf. Find the ratio of the drift velocities of free
£ increases from end A to end B due to decrease electrons in the two wires when they are connected (i) in
in area of cross-section. series, and (it) in parallel. [CBSE F 17]
1
(b) j = — => i <x— Solution. (0 In series, the current remains the same.
^ A ’ A
I }
Current density j decreases from end A to end B.
«. «- -●
^1 ^2
e£x er = enA^v^2
(0p=-| niE
— = a constant
m or
V
rfl _ V

V
Mobility p does not change from end A to end B. d2
Fig. 3.263

I'robk-m 18, (.!) Differentiate between the random (ii) In parallel, the potential different is same but
velocity and the drift velocity of electrons in an the currents are different,
electrical conductor. Give their order of magnitudes, p/
(b) A conductor of uniform cross-sectional area is A
connected across a dc source of variable voltage.
Draw a graph showing variation of drift velocity of Also, V =/2R2 =CKpi>^2^
electrons )as a function of current density (y)in it. V
dl
Now, l^R^ =12^2 = 1.
[CBSE F 20] V
dl
CURRENT ELECTRICITY 3.127

Problem 21. The number densities of free electrons Problem 23. Two identical storage batteries, each
in three conductors X, V and Z are in the ratio 4:1:8, having emf 6 and internal resistance r, are connected, as
and their electrical conductivities are in the ratio 2:1:2. shown in Fig. 3.264. Determine the potentiai difference
For which conductor will the relaxation time of free set up between the points A and B.
e
electronsbe (n) maximum, and (h) minimum?
(CBSE F 20]
T —^—
Solution. Conductivity, a =
m A"
r
r
< 'B

-W^
o m
Relaxation time, t = —5-
ne
6
lam _ om Fig. 3.264
For material X, ^x = 2ne^
Solution. Clearly, the two batteries are connected in

w
2cjm _ a m senes.
For material Y, Ty =
2ne^ ne^ .-.Total emf the two batteries = 6 + 6 = 28
2am _ am Total resistance of the circuit = r + r - 2r
For material Z,

Flo
^z = 28 6
8ne^ Ane^ l = ~
Current in the circuit.
2r r
(a) Relaxation time is maximum for Y.

ee
(b) Relaxation time is minimum for Z. The potential difference beUveen the points A and B,
V^-Vg= emf of either cell - p.d. across its internal

Fr
Problem 22. A battery of emf € and internal resistance

resistance r sends currents Ij and Ij, when connected to = 8-Jr = 6 — xr = 0.


external resistances Rj and Rj respectively. Find the emf for
ur
r

and the internal resistance of the battery. Problem 24. A battery of emf 8 and internal
resistance r is connected to a variable external resistance
Solution. Current in first case,
ks
R. Find the value of R for which (i) current in the circuit
8
Yo

^1 = is maximum and (n) terminal p.d. across the battery is


oo

+ r
maximum. Also, find the maximum value of current in
case (j) and maximum terminal p.d. in case (ir).
eB

8= (R, +r) ...(0 8


Current in second case,
Solution, (i) Current in the circuit, I = R+ r

Current in the circuit will be maximum when R = 0.


r

8
^2 =
ou
ad

8 8
R2 + r I max
0+ r r
Y

8=l2(^^ + '*)
{ii) Terminal p.d. across tlie battery,
From equations (i) and (»), we have 8r
nd

P=8-/r = 8-
Re

l,{R^ + r)=l^iR^ + r) R + r
Fi

or
r(-f[ - ^2) " ^2^ For V to be maximum, 8r/(R+ r) should be mini
mum. This requires
8r
or
R = CO => .Vmax = 8- = 8.
h-h 00 + r

Putting the value of r in equation (i), we get Problem 25. Under what condition will the current in
a wire be the same when connected in series and in
g=J, R, parallel of n identical, cells each having internal
h ~h .
resistance r and external resistance R ? [CBSE OD 19)
Solution. Let internal resistance of each cell be r and its
= i
1
h emf be 8. When n cells are connected in series, the current
in the circuit is
f,f^(R^-R^) n 8
or 8 = /
1 “
R + nr
3.128 PHYSICS-XII

When the n cells are connected in parallel, the current (rf) Currents of the order of 0.1 A through the human
in the circuit is
g body are fatal. What causes the death : heating of
h ~ the body due to electric current or something else ?
R+ ' iiR + r
n
(e) A nerve fiber contains a membrane separating
But
two conducting 'fluids' maintained at a potential
difference. What is the order of this potential
difference ?
[NCERTJ
or R + nr = iiR + r or R =r.
R + nr nR + r Solution. («) About 10 kQ, it is mainly due to skin
through which current enters and leaves our body.
Problem 26. Why is the use of a potentiometer (l>) It is because our body is sensitive to minute
preferred over that of a voltmeter for measurement of currents even as low as a few mA.
emf of a cell or in comparing the emfs of two cell ?
(*■) "Pbis impression is misleading. There is no special
[CBSE OD 08,19 ; F 03 ; D 16C] attractive force that
keeps a person 'stuck' with a high
Soludon. Potentiometer is a null method device. No POwer line. Actually, current of the order of 0.05 A or even
current is drawn from the cell at null point, thus there is much less is enough to disorganise our nervous system,
no potential drop due to the internal resistance of
the cell. result is that the affected person may temporarily lose
So it measures the p.d. in an open circuit. But voltmeter ability to exercise his nervous control to get himself
requires a small current for its operation. So voltmeter 'free' from the high voltage point,
measures the p.d. in a closed circuit which is less than the
actual emf of the cell. (d) Tlie cause of death is not heating, though a person
may receive burns if the currents are too large. The cause
Problem 27. The variation of potential difference V of death is the interference caused by external currents in

with length / in case of two potentiometers A and B is as our highly sensitive nervous system which is basically
shown in Fig. 3.265. Which of these two will you prefer electrical in nature. External currents cause convulsive
for finding the emf of a cell or for comparing emfs of actions and especially interfere with the nerve processes
two primary cells ? Or, which of the two potentiometers, related to our heartbeating. Beyond a certain point, this
A and B, is more sensitive ? interference is fatal.
[CBSE OD 03C, 06 ; F 17]
(e) About 0.1 V.
A

S' Problem 29. Alloys of metals have greater resistivity


B
than that of their constituent metals. Why ?
Solution. In an alloy like nichrome, an electron sees a
p
Cl.
4,
disordered arrangement of Ni^-^ and Cr^^ ions and is
●O
scattered by them randomly and very frequently. This
decreases the relaxation time and increases the resistivity.
Length (I) Problem 30. A metal rod of square cross-sectional
area A having length / has current I flowing through it
Fig. 3.265 when a potential difference of V volt is applied across its
dV ends [Fig. 3.266(01. Now the rod is cut parallel to its
Solution. Potential gradient =
dl =slope of P-/graph. length into two identical pieces and joined as shown in
Fig. 3.266(i’0. What potential difference must be
To measure emf of a cell, or to compare the emfs of maintained across the length 21 so that the current in the
two primary cells, the potentiometer B is preferred over A rod is still / ?
(CBSE F 16)
because it has a smaller potential gradient and hence it is
more sensitive.
A I
Problem 28, Answer the following questions : I I I I

(a) What is the order of magnitude of the resis Fig. 3.266(0 Fig. 3.266(11)
tance of a (dry) human body ? I
Solution. In first case :
(b) If the resistance of our body is so large, why
does one experience a strong shock (some 21
times even fatal) when one accidentally In second case : R^^p A/2
touches the live wire of, say 240 V supply ?
(c) There is an impression among many people i = 2L-Y2 ^ il_A
that a person touching a high power line gets ^ ^ 4 R,
'stuck' with the line. Is that true ? Explain. Hence, Vj = 4 V.
CURRENT ELECTRICITY 3.129

Problem 31. 'n' identical cells each of emf 'S' and inter (Hi) P.D. across each ceil = S + /r
nal resistance 'r' are connected in series to resistor 'R'.
S'-,zS Sr-SV
(i) Deduce an expression for the internal resis R+ n R+n
tance Y of one cell in terms of the current 'I'
flowing through the circuit. P.D. across the series combination of cells.
Hi) How does the internal resistance of the cell Sr-SV^
vary with temperature ? [CBSE D 97C| V - ?j(S + Ir) = n
R+n
Solution,
Problem 33. A student connects a cell, of emf and
(i) Figure 3.267 shows H- H Cells
n identical cells con + internal resistance rj with a ceil of emf € j and internal
nected in series to a 1
resistance r 1' such that their combination has a net
resistor R.
h'
I
internal resistance less thanr.j. This combination is then
As the internal

w
R connected across a resistance R.
resistances of all the n
>VW
cells are connected in Draw a diagram of the 'set-up' and obtain an
series, so total internal expression for the current flowing through the
Fig. 3.267
resistance R. [CBSE OD 16C]

Flo
resistance = nr

Total resistance in the circuit = R + nr Solution. As the net internal resistance is less than rj

the two cells are connected in parallel, as shown in

ee
Total emf of the cells =
Fig. 3.269. '2
Current in the circuit = /

Fr
r
vw
eq
Using Ohm's law. 6 C
''i
Total emf
Total resistance =- g I VW
for
ur
eq
Current in the circuit I I
% 'i '2
R+ nr = — or nr= R
VW
i I re
ks
2 ¥2 R
eq
Yo
R h
or Fig. 3.269
oo

I n
€ '6 1
eq r,r2 1
(n) The internal resistance of a cell decreases with the 1 =
eB

R+ r
increase of temperature. eq ^1 + ^2 R + Vi
rj -t-r.
Problem 32. A number of identical cells, n, each of
1 Vi.
emf 6, internal resistance r connected in series are
r

r^J R(r^+r2)+r^r2
ou

charged by a d.c. source of emf using a resistor R.


ad

(i) Draw the circuit arrangement Problem 34. Three identical resistors, each of
Y

resistance R, when connected in series with a d.c. source,


Hi) Deduce the expressions for («) the charging
dissipate power X. If the resistors are connected in
current and (b) the potential difference across
parallel to the same d.c. source, how much power will be
nd
Re

the combination of the cells. [CBSE D 03C, 08]


dissipated ? [CBSE OD 98C ; D 99]
Solution. (/) Tlie circuit arrangement for charging n Solution. Let emf of the source = V volts
Fi

cells is shown below :


Resistance of each resistor = R ohms
M—n Cells—M
In series, total resistance = 3 R
I I- 1
V'2
Ir r .-. Power dissipated = 3R
= X or R =
/ / 3X
I

R
In parnllcl, total resistance R' is given by
1 AW
1_ 2 2 1-1 R
R’ = -
r^”r^r^r"r
or
3
Fig. 3.268
.'. Power dissipated =
V^_3V^_ 3V'^ = 9X.
(iT) Net emf - nE R' ~ ~ V^/3X
Total resistance = R + nr Problem 35. An accumulator of emf S and internal
f
Net emf resistance ris first connected to an external resistance Rj
-

Charging current, I =
Total resistance R + «r and then to an external resistance R^ for the same time.
3.130 PHYS!CS-X!I

For what value of r the heats dissipated in Rj and R2 Electron drift speed is estimated to be of the order of
will be same ? mm s Yet a large current of the order of few amperes
Solution. Heat dissipated in R^ in time t can be set-up in the wire. Explain briefly.
(CBSE OD 15]
= Heat dissipated in Rj in time f.
^2 2 Solution. As j = cE, so the graph of current density (;)
versus electric field (£) for any material is a straight line,
or
R,t = R^t
+ r
i?2 + r as shown in Fig. 3.270.

or

On simplification, we get: r =
Problem 36. Under what condition is the heat

produced in an electric circuit:


(/) directly proportional
(//) inversely proportional to the resistance of the
circuit ? [CBSE SP 04] Fig. 3.270
Solution. (0 Heat produced is directly proportional to
Slope of the graph = Conductivity (o)
the resistance if current in the circuit is kept constant
because H = I^Rt. The material Bwith less slope (smaller conductivity or
(ii) Heat produced is inversely proportional to the higher resistivity) is used for making standard resistances
resistance if voltage is kept constant because while the material A with greater slope (higher
conductivity) for making connecting wires.
H = t.
. R Electric current, I = enAv^
Problem 37. A student has two wires of iron and
Although both e and are very small, yet we can
copper of equal length and diameter. He first joins the obtain a large current. This is because free electron density
two wires in series and passes electric current through of a conductor is very large, u-10^m“^. The drifts of a
the combination which increases gradually. After that very large number of free electrons add up to cause a large
he joins the two wires in parallel and repeats the process current inside the conductor.
of passing current. Which wire will glow first in each
case and why ? Problem 39. Explain,, giving reasons, how the internal
Solution. (0 In series, same current 1 flows through resistance of a cell changes in the following cases :
iron and copper wires. (fl) When concentration of the electrolyte is
inaeased.
Here H=I^Rf =I^^t
A (&) When area of the anode is decreased,
i.e., H ocp (c) When temperature of the electrolyte is
increased. [CBSE F 08]
As p is more for iron than for copper, so iron wire will
start glowing first in series combination, Solution.

(n') In parallel, the voltage V is same for both the wires. (a) When concentration of the electrolyte is
increased, internal resistance of the cell
vh
Here H = = vhA i
i.e.,
1
Hoc- increases.
R p/ P
Rt’rtsyji. In more concentrated electrolyte, inter
As p is less for copper than for iron, so copper wire will ionic attractions increase and the movements
of the ions become difficult,
start glowing first in parallel combination.
(fc) When area of the anode is decreased, internal
Problem 38. Plot a graph showing the variation of resistance of the cell increases.
current density {j) versus the electric field (E) for two
conductors of different materials. What information Rcasoi}. Lesser area of the anode decreases its

from this plot regarding the properties of the tendency to attract oppositely charged ions,
conducting material, can be obtained which can be used (c) When temperature of the electrolyte is
to select suitable materials for use in making increased, internal resistance decreases.
({) standard resistance and (it) connecting wires in Reason. Both interionic attractions and viscous
electric circuits ?
forces decrease at higher temperature.
CURRENT ELECTRICITY 3.131

Problem 40. Answer the following : (b) In a potentiometer experiment, if the area of the
(fl) Why are the connections between the resistors cross-section of the wire increases uniformly from one
in a metre bridge made of thick copper strips ? end to other, draw a graph showing how potential
(fc) Why is it generally preferred to obtain the gf^dient would vary as the length of the wire increases
from one end. (CBSE OD 14C]
balance point in the middle of the metre
bridge wire ? Solution, (fl) (0 A potentiometer with a longer wire has
(c) Which material is used for the metre bridge ^ small potential gradient. Consequently, it is more
wire and why ? [CBSE OD 14] sensitive and hence preferred.
Solution, {a) Thick copper strips offer minimum (ii) Only then it will have same resistance per unit
resistance and hence avoid the error due to end resistance length throughout and V a: I, as required in the principle
which have not be taken into account in the bridge ^1^^ potentiometer,
formula. (Hi) If it is not so, then the balance point cannot be
(fe) The metre bridge is most sensitive when the four obtained on the potentiometer wire,

w
resistances forming the bridge are equal. For this the V _R1 _ pi I _ pi
(b) Potential gradient, k =
balance point must be somewhere near the middle of the I I A I A
wire.
The variation of cf us Z when A increases from one end

Flo
(c) Alloy such as manganin or nichrome which has to the other is as shown in Fig. 3.271.
low temperature coefficient of resistance and high

ee
resistivity.

Fr
Problem 41. (fl) In a potentiometer experiment, why t
is it necessary to
(i) use a long wire,
for
ur
(ii) have uniform area of cross-section of the wire
and

{Hi) use a driving cell whose emf is taken to be


ks
greater than the emfs of the primary cells ? Fig. 3.271
Yo
oo

HOTS
eB

Problems on Higher Order Thinking Skills


Problem 1. Do bends in a wire affect its resistance ? (b) Behoeen two successive collisions each free electro)i
r

eE
ou
ad

Solution. No, this is because the mean free path of acquires a velocih/ from 0 to v where v = X. What is the
free electrons in the wire is less than the radius of m
Y

curvature of the bend and so they can easily go round (average velocity of a free electron in the presence of an electric
the bends. The resistance is not affected as long as the ? Do all electrons have the same average velocity ?
nd

area of cross-section remains same at the bend.


(c) How does this average velocity of the free electrons, in
Re

Problem 2. A conductor of length I is connected to a d.c. the presence of an electric field vary luith temperature ?
Fi

ICBSE SP 2011]
source o/cm/'€'. If the length of the conductor is tripled by
stretching it, keeping '6' constant, explain how its drift Solution, (fl) (/) In the presence of electric field, the
velocity would be affected. [CBSE OD 15] paths are curved.
g eEx
(ii) In the absence of electric field, the paths are
Solution. E = 7/ and v^ = m straight lines between two successive collisions.
(];) Average velocity of a free electron in the
For given emf 6, the field £ becomes ^ rd when the
presence of an electric field = ●— .
length is tripled. m

.●. New drift velocity, v'^ = —


3
. No, there is a variation in the velocity for
individual electrons,

Problem 3. (a) Write the nature of path office electrons (c) As the temperature of a conductor increases,
in a conductor in the (i) presence of electric ifeld (ii) absence the thermal speed of free electrons increases and also
of electric ifeld. the amplitude of vibration of the metal ions increases.
3.132 PHYSICS-XII

The free electrons collide more frequently witli the Problem 7. In an electric kettle, water boils in
positive metal ions. The mean collision time t 20 minutes after the kettle is switched on. With the same
decreases. Hence average velocity of free electrons supply voltage if the wafer is to boil in 10 minutes, should
decreases. the length of the heating clement be decreased or increased ?
Problem 4. Is current density a vector or a scalar Solution. For a given supply voltage, the rate of
quantity 7 Deduce the relation between current density and production of heat is
potential difference across a current carrying conductor of v^A 1
length I, area of cross-section A. and number density of free p = Poc-
R I
electrons n. Hozv does the current density, in a conductor,
vary with Clearly, in order to double the rate of production of
(fl) increase in potential gradient ? heat, length of the wire should be decreased to half its
(b) increase in temperature ? original length.
(c) increase in length 7 Problem 8. E.xplain, with the help of a graph, the
variation of conductivity with temperature for a metallic
(d) increase in area of cross-section 7 conductor. [CBSE OD 041
(Assume that the other factors remain constant in each
case). ICBSE SP 2011] Solution. Conductivity of a metallic conductor is
given by
Solution. Current density is a vector quantity having 1
2
ne X
same direction as that of motion of the positive charge. a = —
m P
/ = enAv^
:. Current density. For metals, the number of free electrons is fixed. As
1 eE eV the temperature increases, the amplitude of vibration
) = = cnv^ =en—x = en of the atoms increases. The collisions of electrons with
A m ml
these atoms become more effective and frequent. The
or
ne\'\ V relaxation time x decreases. Hence the conductivity of a
} =
m I metallic conductor decreases with the increase of
temperature, as shown below graphically.
(a) j increases with increase in / /.
(b) j decreases with increase in temperature because T
T increases,
E
(c) j decreases with increase in length 1. 52,

(d) j remains same as it is independent of A >

Problem 5. A battery ofemf 2 V and internal resistance u

0.1 Q is being charged by a current of 5 A. What will be the


T3
C
o

direction of current inside the battery 7 What is the potetitial U

difference behveen the terminals of the battery 7 [IIT 80]


Temperature (BQ
Solution. During charging of a battery, the current
inside the battery flows from the positive to the Fig. 3.272
negative terminal of the battery, so the terminal p.d. is
greater than the emf. Problem 9. Figure 3.273 shozvs a piece of pure
Thus V=^+ Ir =2 +5x0.1 semiconductor, S in series with a variable resistor R, and a
= 2.5 V.
source of constant voltage V. Woidd you increase or decrease
the value of R to keep the reading of ammeter (A) constant.
Problem 6. Tzventy electric bulbs are connected in when semiconductor S is heated 7 Give reason.

series with the mains of a 220 V supply. After one bulb is ICBSE OD04,17;SP 23]
fused, the remaining 19 bulbs are again connected in series
across the same mahis. What luill be the effect on illumi S

nation 7

Solution. The combined resistance of 19 bulbs is


0
less than the combined resistance of 20 bulbs.
R
Therefore, the current will increase. As heat produced, V

H <x therefore, illumination will increase. Fig. 3.273


CURRENT ELECTRICITY 3.133

Solution. The value of R should be increased. As Problem 12. The current through n wire depends on
the temperature of semiconductor S increases, its time rts I = Iq + at, where /^ = 10 >1 and a = 4 /4s ^ Find
resistivity decreases. As a result, the circuit resistance the charge that flows across a sectmt of the wire in
decreases and current tends to increase. To keep the 10 seconds.
dq
reading of ammeter (/4) constant, the value of R has to Solution. As I = — = Iq + at
df
be increased.
at^
Problem 10. Explain hoxo electron mobility changes for q=\ ^o + <^t)^t = lQt + — + C

a good conductor lohett (i) the temperature of the conductor


is decreased at constant potential dijference and (ii) applied At f =0, =0, so constant of integration C =0.
potential difference is doubled at constant temperature.
[CBSE OD 06C; D 17C]

Solution. Electron mobility of a conductor,

w
4x(10f = 300 C.
ex Atf =10s. q =10 X 10 + 2
p =—
m
Problem 13. A unifor)n wire is cut into 10 segments
(/) When the temperature of the conductor decreases,

Flo
the relaxation time x of free electrons decreases,
i}icrcasing in lengtlt in equal steps, the resistance of the
shortest segment is R and the resistances of the other
so mobility p decreases.
segments increase in steps ofSQ. If the resistance of the

ee
(ii) Mobility p is independent of applied potential 2 R, ifnd the value of Rand hence ifnd the

Fr
difference. resistance of the original xoire.
Problem 11. Tu’O wires X, Y have the same resistivity, Solution. Resistance of first or shortest segment
but their cross-sectional areas are in the ratio 2 : 3 and = RQ
for
ur
lengths in the ratio 1 : 2 Thc,j are first connected in series of second segment
and then in parallel to a d.c. source. Find out the ratio of the
drift speeds of the electrons in the two wires for the two cases. =R+8 = R + 8xlQ
ks
[CBSE 08] Resistance of third segment
Yo
oo

= R + 16 = R + 8x2Q
Solution. Current, I = enAv^
Resistance of tenth segment
eB

(i) In series, current 1 is constant. So, A = R+ 8x9 = R+72Q

_ ■^Y _ 3.2 But resistance of longest segment = 2 R


r

v,(Y) 2
ou

2 R= R+72 R = 72Q.
ad

or

(if) In parallel, V =IR = constant Resistance of the original wire


Y

1
jee — = R + (R+8) + (R + 16) + ... + (R + 72)
R
= 10 R+8(1 + 2 + 3+ ... + 9)
nd
Re

I
But R X — = 10 R + 8x 45=10x72 +8x 45
A
Fi

= 10800.
A
loc — Problem 14. The resistivity of the material of a
I
conductor of uniform cross-section varies along its length ns
/
X _ P- =Po
■j d + OLV). Deduce the expression for the resistance of
or
I
Y Ay l^ 3 1 3 the conductor, fits length is L and area of cross-section is A.
Solution. The resistance of the small length dxof the
Also,
enA^v^jX) _2 v^jX) conductor at distance ;c from its one end will be
ly enAyV^iY) 3 ^^(y) dx dx
rfR = p —= Po(l + ca) —
2vA^)_4
Hence,
3v^{Y) 3 The resistance of the length L of the conductor will be

or R = dR = (1 + ca) dx
1 A
0
.134 PHYSICS-XII

2 ^
-Po L X Problem 17. Calculate the temperature at which the
A ^lo + a
2 resistance of a conductor becomes 20% more than that its
0
resistance at 27®C The value of the temperature coefficient
of resistance of the conductor is 2.0 x 10“^ / K
L+-aI^l .
2 ICBSE SP 08]
Solution. Here
Problem 15. A current of 5 A is passing through a

non-linear magnesium wire of cross-section 0.04 n3. At R2= Rj +20% of =1.2 Rj,
every point the direction of current densih/ is at an angle of
60“ with the unit vector of area of cross-section. Find the f = 27°C = 300 K a = 2.0 x 10~*
magnitude of electric ifeld at any point of the conductor. _ R2 - Rj _ 1.2 R,1 -R 1 = 1000
ti fj -
(Resistivity of magnesium p =44 x lO'^fim) R.a
1 R,1 X 2.0x10“^
[JEE Main July 21]
Solution. f = 1000 + f, = 1000 + 300
= 1300K or 1027° C.
I
Problem 18. Fiiv identical cells each of inteimal resistance
1Q and emf 5 V are connected in series and in parallel with an
external resistance ‘R\ For what mine of'R', current in series and
parallel combinations will remain the same ?
[JEE Main Aug 21]
I =] .A = }Acos60° Solution.
6, r S, r 6, r r r
2 / _ 2 X 5 _ 10^ Am ^
^ 0.04 " 4 -I 11 11 11 Ih
1 ■<'/ ^‘7 1
/=a£=-£
P

10^ R
£ = p/ = 44xl0 ®x Vm"^
56
4
^1 = R + 5r
= llxl0~^Vm^\

Problem 16. A carbon iflament has a resistance oflOO Q h~
R+ ^
at 0“C. What must he the resistance of a copper filnment 5
placed in series with carbon so that the combination has the
same resistance at all tempieratures ? Temperature coefficient
of resistance of carbon =-0.0007 and that of copper is
0.004“C"’.
Solution. As a =
Rq t
.-. Change in resistance = a Rq f h" I
6, ;● ' ‘'2

Let the resistance of copper placed in series with


carbon at 0“C be R, so that the combination has the
AAAr
same resistance at all temperatures. Then R

Increase in resistance of copper per °C h =h


= Decrease in resistance of carbon per °C 56 6
(a Rq 7) for copper =(a Rq t) for carbon R + 5r
R+ ^
0.004 X R x 7 = 0.0007 X 100 X t 5

0.0007 X 100 5R + r= R + 5r
R = = 17.5 n.
0.004 :. R = r= in.
CURRENT ELECTRICITY 3.135

?robleml9.Thelengthsondradiiofthreewiresofsame Problem 21. Calculate the steady-state current through


metal are in the ratios 2:3:4 and 3:4:5 respectively. They 2 Q resistor in the circuit shown in Fig. 3.276. The internal
are joined in parallel and included in a circuit having 5 A resistance of the battery is negligible and C = 2 f.iF
ICBSE F 101
current. Find current in each wire. IIITI

Solution. Let be the resistances of the 2n

wires. Then A B

„ L L L 2 3 4
AA/V
c 4fi
The ratio of the currents in the three wires must be
AAAr
inverse of the above ratio.
9 16 25
, I
6V 2.8 n
— = 54:64:75
● ● h-h-h 2■ 3 4 h-^-VW

w
As total current = 5 A, therefore
Fig. 3.276
5x 54
h = = 1.40 A,

Flo
193 Solution. In the steady state, capacitor offers
infinite resistance to d.c., so no current flows through
5 X 64
4 n resistor, which is thus ineffective.

ee
h = 1.66 A,
193
Effective resistance between points A and B is

Fr
5x75
= 1.94 A. 2x3
193 R' = -=1.2Q
2+3 5
for
ur
Problem 20. Five 4 Q resistances, a2V battery and an
Total resistance of the circuit.
ammeter are connected as shown in Fig. 3.274. Find the
ammeter reading. [BIT Ranchi 96] R = 1.2 +2.8 =4.0 n
ks
4£1
Yo

Current drawn from the battery.


A/W
oo

€ 6
-=1.5 A
4Q
R 4
eB

4Q
P.D. between points A and B,
AMr U=/R'=1.5xl.2=1.8V
r

2 V A
ou
ad

40 V 1.8
= 0.9 A.
—WV
Current through 2 Q resistor = y = —
Y

40
Problem 22. As shown in Fig. 3.277, a variable rheostat
of 2 kQ is used to control the potential difference across a
nd
Re

Fig. 3.274 500 Q load.

Solution. Tlie equivalent circuit is shown in


Fi

I
Fig. 3.275.

11
B
20 ,_50V
AAA,^—1 40
2V
h
20 1 1
h-0-1 C
500 0

AAAt

Fig. 3.275 Fig. 3.277


2x2
Equivalent resistance, R = 2 + 4=5Q (/) If the resistance ABis 500 D, what is the potential
+2
difference across the load ?
S 2 (ii) If the load is removed, ivhat should be the resistance
.-. Ammeter reading, 7 = y = — = 0.4 A. at BC to get 40 V between B and C ? [ISCE 96]
, 3.136 PHYSICS-XII

Solution. Here R AC
= 2kQ=2000a Kj_=500a When switch S is closed, the parallel combination
R..=500Q, R BC = R AC -K.n
AB =2000-500 =1500n
AB
of all the three resistors is in the circuit. Tine equivalent
Total resistance of the parallel combination of R BC resistance K' of the combination is given by
and R^ ,
1500 X 500 J_ = l +.113
- + - =
R' = -=3750 R! r r r r
1500 + 500
or R'=^
Total resistance of the circuit, 3
R=RAB + K’=500 + 375 = 875 0
Ammeter reading,
Current in the circuit. V 0.3 r
/' = — = 0.9 A.
50 V
^A R' r/3
R 875 O 35
Problem 24. If the galvanometer in the circuit
{/) The potential drop across Rj_ will be the same as (Fig. 3.279) rends zero, find the value of resistor R. Thel2 V
the potential drop across the parallel combination of negligible internal resistance. If cool air is blown
Rg^ and R^. across the wire zvoiind resistance, zuhnt effect will be noticed
ajid zvhy ?
.'. Potential drop across R^^ I
IISCE 97]
= V ~V
^AB
10 kn
2
= 50 - — X 500 = 50 - 28.57 = 21.43 V. wire
35 wound
+ 6
(//) If the load is removed, then the current will flow _o 12 V
through entire resistance RAC of the rheostat. R
2V
50 V
.'. Current, /' = -^A Cell

2000 n 40

To obtain a potential drop of 40 V between Band C, Fig. 3.279


the required resistance BC must be Solution. As the galvanometer shows no deflection,
R'
40 V
= 1600n.
so the p.d. across R is 2 V. If / is the current in the
BC “
1740 A circuit, then

Problem 23. In the circuit shozvn in Fig. 3.278, both the IR=2 volt or / =—
R
ammeter and the cell have negligible resistance. Three
12
external resistors are identical. When the szoitch S is opened, But 1 =
the ammeter readsO.6 A. What zoill the ammeter read zohen R + 10‘‘
the szoitch S is closed ? 2 12

C I R R + 10^

A/W On solving, R=2000Q=2kQ.


When cool air is blown across the wire wound
iWV resistor, its resistance decreases. The current in the
AMr
circuit increases. The voltage across R increases. The
galvanometer will show some deflection.
Fig. 3.278
Problem 25. A uniform wire of resistance R is shaped
Solution. Let r be the resistance of each resistor. into a regular n sided polygon, where n is even. Find the
When the switch S is opened, the parallel combination equivalent resistance between (i) opposite corners of the
of upper two resistors is in the circuit. Their equivalent polygon and (ii) adjacent corners of polygon.
resistance is R
Solution. Resistance of each side of polygon
R = rx r _ r n

r+r 2 (/) Equivalent resistance betzveen two opposite cortiers of


polygon. The resistances of the two parts of the polyg on

Potential drop across the cell, will be

V=/R=0.6x-=0.3r R = R, =- X — - —
2 1
^"2
CURRENT ELECTRICITY 3.137

These two parts form a parallel combination. So the Problem 27. Space between two concentric conducting
equivalent resistance between two opposite comers of spheres of radii a and b (b > a) is [died zvith a medium of
the polygon will be resistivity p. Prove that the resistance (i?) between the tzoo
-
R
X
R ...
spheres loiU be —
pfl_V
R' =
^1^2 - 2 2..R 4KVfl b [JEE Main April 19]
R R 4
K, + + Solution.
2 2

(/;■) Equivalent resistance between two adjacent corners


of the polygon. The resistances of the two parts will be
R,1 = Resistance of one side = —
n

Rj = Resistance of remaining (« -1) sides

w
connected in series

(tt-1) R
n

Flo
These two parts form a parallel combination. So the dR = p.
equivalent resistance of the polygon between two 4nx^
adjacent comers will be

ee
b

R (n-l)R rfR = -2- {x~-dx

Fr
— X
4ti
(n-l)R
R‘ - Jl
n a

R_^(«-1)R
2
n
r 1?
n n
for
ur
4ti 4n V t? h
Problem 26. Model of a torch battery of length I is to be
made up of a thin cylindrical bar of radius 'a' and a Problem 28. In a metre bridge, the wire of length 1 m
ks
concentric thin c\/lindrical shell of radius 'b' iflled in dR
has a non-uniform cross-section such that, the variation
Yo

betzeeen zuith an electrolyteof resistivity p [see Fig. 3.280(a)] dl


oo

dR
If the battery is connected to a resistance of imliie R, shozv of its resistance R zvith length I is dl ^—i= Tzuo equal
that the maximum joule heating in R xvill take place for 41
eB

R =
^4-1-
2k/ [JEE Main Sept. 20]
resistances are connected as shozvn in Fig. 3.282. The
galvanometer has zero deflection zvhen the jockey is at point
P. What is the length AP ? [JEE Main Jan 19]
r
ou
ad

.j
V
dx R' R'
Y

TAAArr—i-^\AAr-f
p X
nd
Re

K
Fi

a a
A B
b b
1-1
Fig. 3.280 (fl) ib)
Fig. 3.282
Solution. Refer to Fig. 3.280(/j). Maximum power is
Solution.
dissipated in the external resistance when R = r. dl~ 4l
f F 4 R i
R
\dR = c\r^''^dl
0 0
=> R=2C41
Resistance of the small element,
pdx For balanced Wlieatstone bridge,
dR = R' ic4l
2nxl 241 = 1
R' ~2C(1-V/)
u
R = -P -dx =
2nr X 2nl 2kI a / = -=0.25m.
a
4
&.138 PHYSICS-XII

Problem 29. Tvx> cells ofEMFs IV, 2 V and internal As R2 > Ry these resistances in increasing order are
resistance 2Q and lO respectively are connected in
(i) series, (ii) parallel. What should be the external resistance r, r + , r+ R V r+ R^, r+Kj + 1^
in the circuit so that the current through the resistance be the Kj +
same in the two cases ? In which case more heat is generated The currents in decreasing order are :
in the cells ? [CBSE SP 08)
4.2 A, 1.4 A, 1.05 A, 0.6 A, 0.42 A
Solution. Current in series circuit is given by 6
- = 4.2
1 + 2 3 r
...(1)
/ =
rj + + R 2+1 + 2 3+ R 6
= 1.05 A ...(2)
When the two cells are connected in parallel, r+ R
1

g _eq
+ ^25 _1x1+2x2 5 s
= 0.6 A ...(3)
rj+r2 2+1 3 r+ R2
6
1x2 2
r 5^2 , = 0.42 A ...(4)
+ r2 1+2 3 r + Rj + R2
6
= 1.4 A ...(5)
Current in the parallel circuit is given by R1R2
r +
5
g R^ + R2
eq 3 5
+ R 2 2 +3R
On dividing (1) by (2),
+ R
3 r+ R 4.2 R1
1 _
or 1 + =4
r 1.05 r
or
Rj =3r

3 5 On dividing (1) by (3),


3+R 2+3R r + R2 _ 4.2
r 0.6
or 6+9R = 15 + 5R

or R=9/4=2.25n
or 1 + i=7
r

More heat will be generated in series case due to or


R^=6r
larger resistance. From (1), g = 4.2 r
Problem 30. A cell of unknown emf 6 and internal Putting the above values in (4), we get
resistance r, two unknown resistances R, and R2 (R2 > Rj) 4.2 r
and a perfect ammeter are given. The current in the circuit is r + 3r+6r
= 0.42

measured in five difinent situations :


r = lQ
(i) Without any external resistance in the circuit, or

(ii) With resistance R^ only, Hence S = 4.2 V, r = 1Q, Rj = 3 O and R2 = 6 H.


(Hi) With resistance R2 only, Problem 31. A battery consists of 12 cells in series,
(lu) With both Rj and R^ used in series combhiation, and each having an emf € and internal resistor r. Some of the
(v) With R^ and R^ used in parallel combination. cells in the battery are connected with wrong polarity. This
battery is connected to another source ofemfl € and internal
The current obtained in the five cases are : resistance 2 r. An ammeter in the circuit reads 3 A when
0.42 A, 0.6 A, 1.05 A, 1.4 A, and 4.2 A battery and the source aid each other and 2 A in the same

but not necessarily in that order. Identify the currents in the when they oppose each other. Find how many cells
five cases listed above and calculated, r, Rj and R2. in the battery are connected with wrong polarity. [IIT 93]
[CBSE SP 08 ; D 121
Solution. Suppose n cells are connected wrongly in
Solution. Total resistances in the five cases are : the battery. Tlien (12 - n) cells give forward emf and n
(0 r. (ii) r+ R^, {Hi) r+ R^, cells give reverse emf.

R1R2 Effective emf of the battery


(iz;) r+ Rj + R^ (v)r +
Rj + R^ = (12-ii)g-«g={12-2«)g
CURRENT ELECTRICITY

Total resistance of the circuit with battery and Decrease in heat production,
source in both cases H-H' H
I \

xl00= 1- X 100
= 12r + 2r = 14r H H

Currents in the two cases must be proportional to 16


the emfs in the two cases 1- X 100 = 36%.
25
(12-2»)e + 2£_3 Problem 34.1” the circuit shown in Fig. 3284, the heat
(12-2n)E-2&~2 produced in 5 Cl resistor, due to the current flowing through
(14-2n)€ _ 3 it is 10 calorie per second. Find the heat produced in 4 ft
or
resistor.
(10*2n)€~2 HIT]
4Q
or
7-n_3 —Vs/V‘
5-«"2

w
1
A 5H B
or 14-2ji = 15-3h
■VW
n = l
Rg. 3.284

Flo
i.e., one cell has been connected with wrong polarity in
the battery. Solution.Heat produced per second in 5 ft resistor,

ee
-1
Problem 32. In the circuit shown in Fig. 3.283, each P = 10 cal s = 10x4.2
battery is o/ 5 V and has an internal resistance of 0.2 Cl.

Fr
What will be the reading of an ideal voltmeter connected As = P
R
across a battery ? HIT 97]

for
Potential difference between points A and B is
ur
i
V = VM=V10x42x5
ks
(V) Current through 4 ft resistor,
Yo

-Jiio
oo

i i i 7 = A
4 + 6 “ 10
eB

Fig. 3.283 Heat produced per second in 4ft resistor


^/m -1
Solution. Current in the circuit. = I^R = x4Js
r

10
ou
ad

8x5
= 25 A 210 4 -1 -1
nr 8x0.2 X — cal s = 2 cal s .
Y

100 4.2
Reading of the voltmeter, Problem 35. A heater is designed to operate with a
nd

power of 1000 W in a 100 V line. It is connected in


Re

V/ = g-7r = 5-25x0.2 =5-5 = 0.


combination with a resistance of lOCl and a resistance Rtoa
Problem 33. Determine the percentage by which the jqO V mains, as shown in Fig. 3.285. What should be the
Fi

illumination of a lamp mil decrease if the current drops by value of R so that the heater operates zoith a power of623 W?
20%. HIT 88]

Solution. If R is the resistance of the lamp and I is Heater


A B
the current flowing for time t, then heat produced is
R
H=I^Rt
When current drops by 20%, current in the circuit
100 V
—o o-
= 80%of/ = —/=-/
100 5
Rg. 3.285
Heat produced. Solution. Resistance of heater,
\2
H' = 1/ R' =
_100^ = 10 ft
^5 J 25 25
P ”1000
3.140 PHYSICS-XII

When the heater is connected as shown and the Problem 37. Determine the currents through the
power drops to 62.5 W, the p.d. across the heater resistors Ry and shaxm in Fig. 3.287.
would be
e,
y’=V^=V62.5xlO=25V [—VS/V— l-f-l)—VsAr-|
R 1
P.D. across 10 Q resistor
= 100 -25 = 75 V

Current in 10 H resistor. ♦

75
/ = — = 7.5 A Fig. 3.287
10
Solution. Suppose currents /j and I2 start from the
Current through heater, cells of emfs and S2 respectively, as shown in
V'
r=—
25
— = 2.5 A
Fig. 3.287(fl). Then currents through the three resistors
R' 10 R^, i?2 3rid R3 will be , I2 and + I2 respectively.
Current through resistance R i] h

= 1-I'=7.5-2.5 = 5A R1
tI
P.D. across R = P.D. across heater = V' = 25 V

F' 25
1
G G h

Resistance, R = — =50.
I-!' 5

Problem 36. Three equal resistors, each of R ohm, arc Fig. 3.287 (q)
connected as shown in Fig. 3.286. A battery of 2 volt and of Applying Kirchhoff's second law to the loops 1
infernal resistance 0.1 ohm is connected across the circuit. and 2, we get
Calculate the value ofR for which the heat generated in the
circuit is maximum. R, f^R,{f + l2) = ^ 1
or
(R^ + R^)f + R^l2=^, ...(1)

R R R and +
^2) “^2
—WV—^Wv—4—VW or
(R2 + R3) l2 + R3 f =^2 ...(2)

+
To eliminate I2, multiply (l)by (R2 + K3)and (2) by
f R3 and subtract. Then
(R, + R3KR2 + R3)- R^^ Jj =gj (Rj + R3) -EjR,
Fig. 3.286
or I
I “
Solution. As three resistors of R ohm each are (R, + R3)(R3 + R3)-R3^
connected in parallel, their equivalent resistance R' is
given by _ (R2 + ^3) ^2 ^
R^ R2 + R2 R3 + R3 Rj
J_ = l 1 1 =1
R' R R R R To eliminate f, multiply (2) by (R^ + R3) and (1)
R by R3 and subtract. Then
or R' = —
3 (R, + RjHRj + R3) - r2 (R, +
R3)-g,R3
Heat generated in the circuit is maximum when or
h~
External resistance = Internal resistance {R, + R^){R^+R^)-R^
i.e., R'=r £3(R, + R3)-6,R3
R R^ R2 + R2 R3 + R3 Rj
or
3
R2 + B2 ^
Ij +12-
or R=3r=3x0.1 = 0.3n. Rl Rj ●1^2 ^3 ^3 ^1
CURRENT ELECTRICITY 3.141:

Problem 38. 'n' cells, each of emf 'e' ami i}itermil


resistance Y are connected in a closed circuit so that the
positive teriniiial of a cell is joined to the negative ternihial of
the next, as shown in fig. 3.288(rt). Any tiuo points of the
circuit are connected by an extertial resistance R. Find the
current in R.

Fig. 3.289 (a)

Solution. The distribution of current for the given


i network is shown in Fig. 3.290(1?).
V, = 6V

w
R
■WV I1 Q-
.--'ion 0
Fig. 3.288 (a) I
1-1 I
P,= 10V ''

Flo
1
●Q
Solution. Suppose m cells are connected on one
side of R and remaining (» - m) cells on the other side 3

ee
of R. Then the equivalent circuit will be of the form V’3 = 5V
shown in Fig. 3.288(1?). The distribution of current is

Fr
shown. Fig. 3.289 (&)
H - m cells
Applying Kirchhoff's second law to the loop 1021,
for
ur
A —►—I 4 4 B
x +y x +y we get
P
I ’
V 101+20/^ =10-6
I I
ks
VAT or
101 + 20/^ =4
Yo

m cells
Similarly, from loop 0320, we get
oo

-Wv
D X
R 30(/-/i)-20/i=6-5
eB

Fig. 3.288 (b) or


301-50/^ =1
On solving equations (/) and (n), we get
Applying Kirchhoff's second law to the loop
r

/ = 0.2 A
ou

ABQCDPA, we get
ad

...(1)
Thus the current flowing through the resistance
{n - m) e=r{n- m) (x + y)+ xR
Y

is 0.2 A.
Similarly, from the loop QEFPDCQ, we get Problem 40. A part of the circuit in a steady state along
me = - mry - xR or mry = me + xR with the currents flowing in the branches, the values of
nd
Re

me + xR resistance, etc. is shown in Fig. 3.290. Calculate the energy


or y = HIT 86]
Fi

mr stored in the capacitor.


Substituting tliis value in (1), we get
me + xR Y
{n-m)e=r(n-m) x + + xR
mr

L WV
On simplifying, we get
x{mrn-rn? + iiR) = 0 C
2A
.V = 0 I ^ -W/ ^wv4-va
20 t 40
i.e.. Current through R is zero. 3 V
30

Problem 39. Find the current flowing through the 1 A


resistance R, of the circuit shozon in Fig. 3.289(r?). CiiK’n
R^ =10Q, R., =20n and R^ =3012 and the potentials of
pomts 1, 2 and 3 are -V^ =10 V, ¥2=6 V and = 5 V. Fig. 3.290
'.-142 PHYSICS-XII

Solution. In the steady state, when the capacitor is Problem 42. for the potentiometer circuit, shown i:: m
fully charged, there is no current in the arm AB.
Pig- 3-292, points x and y represent the two terminals of an
Applying Kirchhoff s first law at the junction A, the unknown em/6. A student observed that when the jockey is
current in arm AD ^
moved from the end A to the end B of the potentiometer wire,
= 1 + 2= 3A = current in arm DE the deflection in the galvanometer remahts in the same
Applying Kirchhoff's first law at the junction B, What are the two possible faults in the circuit that
current in arm EB could result in this observation ?
+
= 2-1 = 1 A 1

Now V'^^ = 5x3=15V,


=1x3=3 V, 4b
I
Vgg = 2 X 1=2 V y
P.D. across the ends A and B of the capacitor is
Fig. 3.292
=''/ID + +'"eb = 15 + 3 + 2 =20 V
If the galvanometer deflection at the end B is
Energy stored in the capacitor is
(i) more {ii) less
U = -CV^
2 ^x(4xl0“^)x(20)^ than that at the end A, which of the two faults, listed above,
would be there in the circuit ? Give reasons in support of your
answer in each case. -
= 8x 10"'^ J. [CBSE OD 07]

Solution. Two possible faults for one-sided


Problem 41. circuit shown in Fig. 3.291, find the deflection are as follows :
current through the 4 .Q resistor. HIT 981
(f2) The positive terminals of all the cells are not
3Q 2£2 2Q
r-VW
A
-Wv
B
-WWn
connected to the point A of the potentiometer.
I, /.
' '‘2 h (b) The emf of the driving cell is less than the emf of
the cell to be balanced.
9 V
8n 8Q 4Cl (/) In case (a) the deflection at the end Sis more than
that at end A.
L_wv -Wv -WVJ
2Q.
D
2Q C
2C1 Reason. The two emfs support each otlier and the
resultant emf becomes maximum at the end B.
Fig. 3.291 (») In case (b) the deflection at the end Bis less than
that at A
Solution. Resistance between B and C on the right
hand side of the circuit = ^ = 4Q Reason. The two emfs oppose each other and the
8+8
resultant deflection at the end B(where the main emf is
maximum) would be less than that at the end A
Resistance between A and D on the right hand side
8x8
of the circuit = = 4Q Problem 43. the potentiometer circuit shown in
8 +8 Fig. 3.293, the balance (null) point is at X. State with
Equivalent resistance of the circuit, reason, where the balance point will be shifted when
R
R=3+4+2=9n

Current drawn from the battery,


9 V X
1 = =1 A Ail
9Q
T7

At the junction A the current of 1 A is divided


equally between the 8 H resistance and the remaining Fig. 3.293
circuit of resistance 8 O.
(i) Resistance R is increased, keeping all parameters
At the junction B, the current of 0.5 A is divided unchanged,
equally between the 8Q resistor and the remaining (ii) Resistance S is increased, keeping R constant,
circuit of resistance 8 Q.
ini) Cell P is replaced by another cell whose emf is lower
Current through the 40 resistor = 0.25 A. than that of cell Q. rCBSE F 04 ; SP 20]
CURRENT ELECTRICITY 3.143

Solution, (i) When resistance R is increased, the Problem 45. The following circuit shows the use of
potential gradient k decreases. potentiometer to measure the internal resistance of a cell:
1 R
AB
6 r
k =
I VvV
AB

y
As
- AX }
B
AX
AX- ^ 6 I
Due to the decrease in k, length ^AX will increase i.e., I- 0
the balance point will shift towards the end R
R
{ii) No effect, because at null point no current is
drawn from the cell Q. Fig. 3.295
{in) Balance point is not found on wire AB because (0 When the key K is open, how does the balance point

w
in this situation the potential drop across the wire is change, if the current from the driver cell decreases ?
less than the emf of the cell Q.
(ii) When the key K is closed, how does the balance point
Problem 44. A potentiometer wire has a length Land a change ifR is increased, keeping the current from the

Flo
resistance Rq . It is connected to a battery and a resistance driver cell constant ? [CBSE OD 98C1
combination as shown. Obtain an expression for the poten
Solution. (0 When the current from the driver cell

ee
tial drop per unit length of this potentiometer wire.
decreases, the potential gradient across the potentio
meter wire decreases, and balance point shifts towards

Fr
R
I-WV-I S
+
the end B.
I-1-
s
L^WV (ii) When R is increased and K is kept closed,
for
ur
terminal p.d. of the cell increases and balance point
again shifts towards the end B.
■4
B Problem 46. A potentiometer wire of length 1 m is
ks
connected to a driver cell of emf 5 V as shown in Fig. 3.296.
Yo

Fig. 3.294 Wlten a cell of 1.5 V emf is


oo

I
used in the secondary
What is the maximum emf of a 'test cell' for which one
eB

circuit, the balance point is


can get a 'balance point' on this potentiometer wire ? What found to be 60 cm. On
^ 4B

1.5 V
precaution should one take, while connecting this 'test cell' replacing this cell and
in the circuit ? [CBSE OD 04C] I—0-(i>
r

using a cell of unknown


ou

G
ad

RS
emf the balance point shifts
Solution. Total resistance = Rq + R + S to 80 cm. Fig. 3.296
Y

B
(i) Calculate unknown emf of the cell,
Current in the circuit, 1 =
{ii) Explain with reason, whether the circuit works, if the
nd

RS
Re

Rq + R + S
driver cell is replaced with a cell of emf 1 V.
{Hi) Does the high resistance R, used in the secondary
Fi

Total P.D. across the ends of wire AB, circuit affect the balance point ? Justify your answer.
[CBSE D 08]

V = /Ro =
Ml
RS Solution. (0
Rq + R + S
I
1

80
V ^2 = 60
xl.5 = 2.0V.

P.D. per unit length, ^


RS
L Ro + R+S
{ii) The circuit will not work because with a driver
cell of 1 V, the balance point cannot be obtained for a
The emf of the test cell must be less than the total cell of emf 1.5 V on the wire AB.

P.D. at the ends of wire AB. The positive terminal of {Hi) No, the balance point is not affected by the high
the cell should be connected to the end A of the resistance R because no current flows through the cell
potentiometer wire. at the balance point.
3.144 PHYSICS-XII

Problem 47. Figure 5.297 shozvs the circuit diagram of and negligible internal resistance. The potential drop, across
a potentiometer for determining the emf't' of a cell of negli the 2 kCl resistor, is measured by (i) a 30 kQ voltmeter (ii) a
gible internal resistance. 1 A*n voltmeter and (in) both these voltmeters connected
IV R I across it, as shoiun in Fig. 3.298. If the voltmeter readings hi
the three cases are V^, V2 and respectively, arrange these
/ readings in descending order.
Hozv will the three readings compare with one another if
the potential drops were measured across the series combi
VA—(G nation of the 2 kCl and the 1 kCi resistor i.e., across the
points A and B ? [CBSE SP 04]
<f^O-
Key K Solution. (a) Lesser is the current drawn by a volt
meter, higher is the reading shown by it. As a voltmeter
Fig. 3.297
having higher resistance draws lesser current, so it
(/) What is the purpose of using high resistance ? shows higher potential difference.
(ii) Hozu does the position of balance point (/) change Now =30 kQ,
ivhen the resistance is increased ?
30 X1 30
(Hi) Why cannot the balance point be obtained, R3 = = —kQ i.e..
30+1 31
{a) when the emf^ is greater than 2 V, and
(b) when the key K is closed ? [CBSE F 09]

Solution. (0 High resistance protects the galvano- (b) When the potential drops are measured across
meter for positions far away from the balance point, by the points A and B, the same relation is obtained, i.e.,
decreasing current through it.
y;>y'>y'.
(ii) When the resistance is increased, the current'
in the driving circuit decreases. This decreases the Problem 49. For the circuit shown in Figure 3.299,
potential gradient across the wire AR The balance l^^l(^"cing length increase, decrease or remain .the
point shifts towards the end R same, if(i) Rj is decreased (ii) R., is increased
(Hi) (fl) When the emf Sis greater tlian 2 V, the balance change, (in each case) in the rest of the
point cannot be obtained on wire AB because the circuit ? Justify your anszver in each case.
[CBSE OD 191
potential drop across the wire AB will be less than 2 V. +
K1 R1
11- wv
(b) When the key K is closed, the cell of emf £ gets
short-circuited, so no balance point is obtained on ]
wire AB. A B

Problem 48. A series combination of a2 kQ resistor


and 1 kQ resistor, is connected across a battery of emf 6 V ^2 K2
30kn Ikn
L-VW—CP
© ©
A B A B
■VA—0 o-*—VA VA—0 Fig. 3.299
2kn 2kn
1 kn 1 kfi
Solution. (0 When R^ is decreased, the balancing
length decreases.
+

Reason. As Rj decreases, the current in the driving


6V
(0 30
6 V
circuit increases. This increases the potential gradient.
Hence a smaller length of the potentiometer wire is
© needed to balance the same emf.
© (//) When R2 is increased, the balancing length
A 1 kn B increases.
VA -VW 4
2kfl 1 kn
Reason. As R^ increases, the current I ~^/{r+ R^)
decreases. This increases the terminal p.d. (V=t-Ir)
across the unknown cell. Hence a larger length (with
6 V
(nO
the same potential gradient) is needed to balance the
terminal p.d.
Fig. 3.298
CURRENT ELECTRICITY 3.145

ProhlQm 50. A potentiometer circuit is set itp CIS shozL’ii of lamp Q increases because its resistance does not
in
Fig. 3300. The potential gradient across the potentio- change while the p.d. across it increases due to the
meter wire is 0.025 V / cm and the ammeter present in the decrease in p.d. across P.
circuit reads 0.1 A, when the tivo way key is completely Problem 52. /I resistance of R O draivs current from a
switched of.f The bakiice points, when the key between the j,„ Mtentiometer has n total |■esislancc of
terminals (i) 1 and 2 (ii) 1 and 3, is plugged in, are found to n (Fig. 3.302). A voltage is supplied to the poten
be at lengths 40 cm and WO cm respectively. Find the values tiometer. Derii>e an expression for the voltage across R when
of R and X. [CBSE SP 11]
the sliding contact is in the middle of the potentiometer.
[NCERT ; CBSE OD 14 ; D 17]
+
I h
■o o-

B
A R i)
A/W ■6c

w
B
R
VAr
R
£ X

Flo
AW A/W Fig. 3.302

0 Solution. When the slide is in the middle of the

ee
+
^2
potentiometer wire, only half the resistance of the

Fr
potentiometer wire (Rq / 2) is introduced between the
Fig. 3.300
points A and R Hence, the effective resistance between
the points A and B is
Rx0.l = kxl 1
for
ur
Solution.

or Rx0.1 = 0.025x40 5>-.r


R _ 2
or R = ioa AB ~
ks
/
+ R Rq+2R
R
Yo

_ ‘1 2
Also,
oo

R +X
Total resistance between A and C is
10 40
eB

R
0 _
R.R
0
R0
10 + X “lOO R
AC
= R
AB
2 R.+2R
0
2
or X = 15Q
r

Rq(Rq + 4R)
ou

Problem 51. The circuit shown in Fig. 3.301 contains a


ad

battery 'B, a rheostat 'R/i' and identical lamps P and 2(Ro + 2R)
Y

Q.What will happen to the brightness of the lamps, if the Current through the potentiometer wire will be
resistance through the rheostat is increased ? Give reasons.
V _2V-(Rq+2R)
[CBSE F 041 / =
nd
Re

R
AC R„(Rp + 4R)
B
Fi

+
M- Voltage across R will be
Q 2V(Rq + 2R) RqR
^AB ~ AB ~
R„(Ro + 4R)'(Ro+2R)
2VR
or VAB "
Rh
R„0 + 4R

Fig. 3.301 Problem 53. The length of a potentiometer wire is 600 cm


and it carries 40 niA current. For a cell of emf 2 volt and
Solution. The sum of the potential drops across the internal rcsistana* 10 ohm, the null-point is friund at 500 cm. If a
lamps P and Q is constant (equal to the battery wltmeter is connected across the cell, the balancing length of
voltage). As the resistance through the rheostat is the wire is decreased In/10 cm. Find (i) the resistance of the
increased, the current in the whole circuit decreases. So whole wire, (ii) reading of voltmeter, (Hi) resistance of
the brightness of lamp P decreases. But the brightness wltmeter. [CBSE OD 2000C1
3.146 PHYSICS-XII

Solution, (i) Let I be the current in the poten Reading of voltmeter,


tiometer wire AB and p the resistance per cm of AB. V’ = E-I'r
Then the potential gradient is
2 X 10
k = Jp or 1.96=2-
R + 10
Balancing length, AC = 500 cm
l-’.D. between A and C, On solving, we get R = 490 n.

V = kl = Ipl Problem 54. In the metre bridge experiment, a student


In no-deflection condition, the p.d. Vequals the emf ^^^erved a balanced point at the pomt /, where A} = 1. Draw
S of the cell. the equivalent Wheatstone bridge diagram for this set-up.
[ c (Fig. 3.305).
A9- ■OB R X
>WV
2 V
ion

I—aM—© 0
G

Fig. 3.303
A
J
€ = 7p /
g 2 V V
or

^ II (40x10-^ A)X 500 cm Fig. 3.305


-1
= 0.1Q cm
The values of R and X are both doubled and then
.. Total resistance of the wire AB interchanged. What zoould be the nezv position of the balance
point ? If, in this set-up, the galvanometer and battery are
= p X AB = 0.1x 600 =60n.
interchanged at the balance point position, how zvill the
(if) When voltmeter is connected across the cell, the balance point get affected ? [CBSE OD 11]
null-point is C, where AC =/' =490 cm. Solution. The equivalent Wheatstone bridge circuit
It-
I C
■OB
is shown in Fig. 3.306. After doubling R and X and
interchanging, let the new balance point be at length
C
2V
''ion _

l-VVW-(t) R X
G
A B

<Z> 0
Voltmeter
A.eft Length
Fig. 3.304 D
(100 - /)

P.D. benveen A and C', +

K
V'=kl' =IpV
Fig. 3.306
= 40 X 10"^ X 0.1 X 490 = 1.96 V.
We then have
As there is no current in the galvanometer G, the 2X
same (1.96 V) is the p.d. across the cell and this is the 2R (100-/')
reading of the voltmeter. R I
Also,
(Hi) The cell is sending current in the voltmeter. Let X (100-/)
R be the resistance of the voltmeter and r the internal
resistance of the cell. If follows that /' =(100-/)
The current supplied by the cell is No, there will be no change in the balance point
6 2
position, when the galvanometer and the battery are
1 =
R-i-r R + 10
A interchanged in the balanced point position in the
above set-up.
CURRENT ELECTRICITY 3.147

Problem 55. Write the condition of balance in Problem 56. A hatiery of iuternal resistance r (= 4 Q) ts
Wheatstone bridge. In the given Wheatstone bridge, the connected to the netioork of resistances, as shozvn in
R, if the Fig. 3.309. What must be the value of R, so that maximum
current in the resistor 3R, is zero. Find the value of
carbon resistor, connected in one arm of the bridge, has the power is delivered to the Jtetwork ? What is the maximum
[IIT 95]
colour sequence of red, red and orange [Fig. 3.307j. power ?
R R
A B
-VS/V -Vvv

●R 6R R
R 4R

r = 4Q -VW D
c

w
Fig. 3.309
1
V Solution. The equivalent circuit for the given
network is shown in Fig. 3.310.
Fig. 3.307

Flo
R 2R
The resistances, of BC and CD arms, are nozv inter B
VvV
changed and another carbon resistance X is connected in

ee
place of R so that the current through the arm BD is again 6R

Fr
4R
zero. Write the sequence of colour bands of this carbon 2R

r = 4fi AW c
resistor. Also find the value of current through it. D
[CBSE SPll]

for
ur
Solution. For the balanced Wheatstone bridge,
P R 2R R Fig. 3.310
or

Q s 2R S
ks
Clearly,
Yo

R = S = Resistance of carbon resistor 2R AR


oo

= 22xl0^a = 22kn
Thus the part of the circuit on the right side is a
eB

When the resistances in arms BC and CD are


balanced WTieatstone bridge. Hence resistance 6 R in
interchanged and another carbon resistor X is arm BD is ineffective. The equivalent circuit then
connected in place of R, the condition for balanced reduces to the circuit shown in Fig. 3.311.
r

bridge will be
ou
ad

R B 2R
2R X
A-wv-^-aw
Y

R 2R

or X = 4R = 88kn
2R 4R
nd

.●. The sequence of colours will be grey, grey and AVV---VvV c


Re

r = 4fi
D
orange.
Fi

Also equivalent
resistance, of the Fig. 3.311
balanced WTieatstone

bridge, is given by Effective resistance along ABC = R + 2R =3R


J__J_ Effective resistance along ADC = 2R + 4R=6R
R..cq ~ 3R^ 6R D
.●. Effective resistance between points A and C,
2+1 I- 3Rx6R
R’ = = 2R
6R
3R + 6R
.-. R. -2R Fig. 3.308
eq
Current drawn from the battery,
Current through the (new) carbon resistor £
-1 A-A / =
r + R' r+2R
~s''2R~ 6R'
3J48 PHYSICS-XII

Power delivered to the network. Solution. Refer to Fig. 3.312(1?).


^2
6
P^l^R' = x2R
r + 2R

According to maximum power theorem, power


delivered to the network will be maximum when

Resistance of the network = Internal resistance


of the battery
50 cm
or 2R=r=4Q I

or R = 2Q.
Fig. 3.312 {b)
^2
6
Also, Pmax x2x2= — Resistance of a frustum shaped conductor.
4 + 2x2 16
I
R =
Problem 57. Two resistances R 1
= X Q and R^ = ]Q ^ mb
arc connected to a wire AB of uniform resistiviti/, as shown r =
a +b 0.2+1
= 0.6 mm
in Fi^. 3.312(a). The radius of the wire varies lincarli/ along 2 2

Resistance of left half wire,


R1
pxO.50
R3 = 16^
Ttx 0.2 X 0.6 X10

A B
Resistance of right half wire,
p X 0.50
V
Ttx 0.6x lx 10
1

For the balanced Wheatstone bridge,


Fig. 3.312 (a) R
1 _

its axis from 0.2 mm at A to 1 mm at B. A galvanometer (G) K


4

connected to the centre of the wire, 50 cm from each end X 0.6x1


along its axis, shows zero deflection when A and B arc = 5 => x = 5n.
IQ 0.2 X 0.6
connected to a battery. Find the value of X. [JEE Adv 22]

Guidelines To NCERT Exercises

3.1. The storage battery of a car has an emfof 12 V. If the is the resistance of the resistor ? What is the terminal voltage of
internal resistance of the battery is 0.4 Q, ivhat is the maximum the battery when the circuit is closed ?
current that can be drawn from the battery ? Ans. As
Ans. Here £ = 12 V, r = 0.4 Q / =
e
R + r
The current drawn from the battery will be maximum
when the external resistance in the circuit is zero i.e.,

R = 0. or R+ r =
I

I
max
r ~ 0.4 € 10
R = --r = —-3=17Q
i 0.5
= 30 A.

3.2. A battery of emf 10 V and internal resistance 3Q is Terminal voltage.


connected to a resistor. If the current in the circuit is 0.5 A, what P = /R = 0.5xl7=8.5V.
CURRENT ELECTRICITY 3.149

3.3. 0) Three resistors of IQ, in and 3 Q are combined in 3.7. A silverwire has a resislauce of 2.1 Ci at 273° C a'
series. What is the total resistance of the combination ? (ii) If the resistaiice of 2.7 £i at 100°C Determine the temperature
combination is connected to a battery of emf 12 V and negligible coefficient of resistivity of silver,
internal resistance, obtain the potential drop across each Ans. Here R, = 2.1Q,t^ = 27.5°C, = 100°C
resistor.
Temperature coefficient of resistivity of silver,
Ans. (0 Kj = Rj + i?2 + ^ =
6 12 a =

(i7) Current in the circuit, I = — — = 2A R^it^-t,)


R 6
2.7-2.1 0.6
Potential drops across different resistors arc
2.1(100 - 27.5) 2.1x72.5
^ = /R,=2xl = 2V,
= 0.00394° C"\
P2=/R2 = 2x2 = 4V,
3.8. A heating element using nichrome connected to a 230 V
P3 = /J^ = 2x3 = 6V.
supply draws an initial current of 3.2 A which settles after a few

w
3.4. (i) Three resistors 2Q, 4Q and 5 Q are cmnbined in
seconds to a steady value of 2.8 A. What is the steady
parallel. Wlmt is the total resistance of the combination ? (ii) If temperature of the heating element if the room temperature is
the combination is connected to a battery of emf 20 V a/id
27° C ? Temperature coefficient of resistance of nichrome averaged

Flo
negligible internal resistance, determine the current through over the temperature range involved is 1.70 x 10 ^ °C .
each resistor, and the total current drawn from the battery.
Ans. Here P = 230 V, = 3,2 A,
(0^ = 4R + —^ + ~1 _1 1 51_19

ee
Ans. -1

1 R, R, 2 4 " 20 /2 = 2.8A, a = 1.70xl0'^°C

Fr
Resistance at room temperature,
20
R =—n. P 230
r 19 = 71.875 n
L1 3.2
(ii) Currents drawn through different resistors are for
ur
g _20 Resistance at steady temperature.
6 _20
— = 10 A (2= — — = 5A, 230
R,- 2 R2" 4
1
= 82.143 Q
^2 = 7-
ks
7, 2.8
Yo

r ^
7, = — = — = 4A
20 . .
oo

K3 5 Now a =

^ C^2 ~
eB

Total current drawn from the battery,


_ R2-R,
7= 7j + 72 + ig =10+5+ 4 = 19 A. ^2 “
R^ a
3.5. At room temperature (27° C), the resistance ofa heating 82.143-71.875
r

element is 100 Q. What is the temperature of the element if the


ou
ad

71.875x1.70x10
resistance is found to be 117 n, given that temperature
coefficient of the resistor material is 1.70 x 10^°C“^ 10.268x10“
Y

= 840.35°C
Ans. Here R^ =100 a R2=117n, f, =27°C, 71.875x1.7
a = 1.70x10“^ °C~
nd

.-. Steady temperature of element,


Re

R2-R,
As a = f2 = 840.35+ 27= 867.35°C.
Fi

Rj(f2-ti) 3.9. Determine the current in each branch of the network


117-100
= 1000
shown in Fig. 3.313.
^2 ^ R.1 a 100 x1.70x10"“

f2 = 1000 +fj =1000+ 27 = 1027°C.


3.6. A negligibly small current is passed through a loire of
length 15 m and uniform cross-section 6.0x10"^ m^ and its
resistance is measured to be 5.0 Q. What is the resistivity of the
material at the temperature of the experiment ?
Ans. Here / = 15 m, A = 6.0 x 10“^m^, R = 5.00
RA 5.0 X 6.0 X10" ^
Resistivity, p =
/ 15

= 2.0xl0"^Q m. Fig. 3.313


3A50 PHYSICS-Xil

Ans. Let I, /j, be the currents as shown in Fig. Ans. Here I = 35.9 cm, R = X = 7, S=Y = 12.5
3.314. We apply Kirchhoff's second rule to different loops. 100-/ 100 - 39.5
As S= xR 12.5 = xR
/ 39.5
12.5 X 39.5
or R = = 8.16n
60.5

Connections are made by thick copper strips to


minimise the resistances of connections which are not
accounted for in the above formula.
I
(ii) When X and V are interchanged.
R = y=12.5Q, S=X = 8.16n,/=?
L
GO 100-/
As xR .-. 8.16 = xl2.5
/ /
Fig. 3.314
or 8.16/= 1250-12.5/
For loop ABDA, 1250
or / = = 60.5 cm, from end A.
10/, + 5/3 -5/2 = 0 20.66
For loop BCDB,
(Hi) When the galvarfometer and cell are interchanged
5(/, -/3)-10(/2+ /3)-5/3 =0 at the balance point, the conditions of the balanced bridge
For loop ADCFGA, are still satisfied and so again the galvanometer will not

5/2 + 10(/2 +/3)+10(/j +/2) = 10 (V /, + /2 = /) show any current.


or
10/j - 5/2 + 5/3 = 0 ...(1) 3.11. A storage battery ofemf 8.0 V and internal resistance
5/j - 10/2 - 20/3 = 0 ...(2)
0.5 n is being charged by a 120 V dc supply using a senes
resistor of 15.5 Q. What is the terminal voltage of the battery
10/, + 25/3 + 10/3 = 10 —(3) during charging ? What is the purpose of having a series

Solving equations (1), (2) and (3), we get resistor in the charging circuit ? [CBSE SP 21]
/.1 = Aa Ans. When the storage battery of 8.0 volt is charged
17 ' 2 17 ' 3 17
with a dc supply of 120 V, the net emf in the circuit will be
Currents in different branches are
6'= 120-8.0= 112 V
/ AB ~ h ~—
17
A, fflc = “ -^3 = 17
A, f 120 V {●

^DC ~ iA
h~ 17 /''
I I
8V
I , _ 0.5Q I R = 15.5£l
/
AD = /2 = —
17
A, / BD = h3 A I—
1
vw-
i I

Total Current,
Fig. 3.316
/=/1 +/ 2 =^A.
Current in the circuit during charging
3.10. (i) In a metre bridge (Fig. 3.315), the balance point is I 112
found to be at 39.5 cm from the end A, when the resistor Y is of / = = 7A
R + r 15.5 + 0.5
12.5 Q . Determine the resistance of X. Why are the connections
between resistors in a Wheatstone or metre bridge made of thick The terminal voltage of the battery during charging,
copper strips ? (ii) Determine the balance point of the bridge above
V = S + /r = 8.0 + 7 X 0.5 = 11.5 V
if X and Y are interchanged. (Hi) What happens if the galvano
meter and cell are interchanged at the balance point of the bridge 7 The series resistor limits the current drawn from the
Would the galvanometer show any current 7 [CBSE D 05] external source. In its absence, the current will be dange
X y rously high.
3.11. In a potentiometer arrangement, a cell of emf 1.25 V
B
gives a balance point at 35.0 cm length of the wire. If the cell is
replaced by another cell and the balance point shifts to 63.0 cm.
what is the emf of the second cell 7
A Ans. Here 6, = 1.25 V, /, = 35.0cm, l.^. = 63.0cm
D
h 63x1.25
l^ ■o^ 62 = —/ >< 6. 1
=
35
= 2.25 V.
1

Fig. 3.315
CURRENT ELECTRiOTY 3.151

3.13. The number density of free electrons in a copper Terminal voltage.


conductor is 85 x 10^® m~^. How long does an electron take to V= 7R = 1.4 X 8.5 = 11.9 V.
drift from one end of a wire 3.0 m long to its other end ? The area (b)Here £ = 1.9V, r = 380n
of cross-section of the wire is 2.0 x 10“ ^ m^ and it is carrying a
current of 3.0 A. I S _ 1.9 A = 0.005 A
max
r ”380
Ans. Here « =8.5 x /=3m,
This secondary cell cannot drive the starting motor of
A =2.0 X 10“^ e = 1.6 x 10“^^ C, / =3.0 A
a car because that requires a large current of about 100 A
Drift speed. for a few seconds.

V
I
3.16. Two wires of equal length, one of aluminium and the
d ~
enA other of copper have the same resistance. Which of the tivo wires
3
ms
-1 is lighter ? Hence explain why aluminium wires are preferred
28
1.6x10
-19
X 8.5 X10 X 2 X10“^ for overhead power cables.

w
3 Given ; = 2.63 X 10"®Qm, = 1.72 x 10“® Clm,
ms ' = 1.1 X 10^ ms ^
16x85x2x10 relative density of Al = 2.7 and that of Cu = 8.9.
Ans. Mass = volume xdensity = Aid
Required time.

Flo
pd l^ /
I 3 = ^.ld = [vR = p--]
= 2.73 7.57 h. R R A
1.1x10““* ^

ee
As the two wires are of equal length and have the
3.14. The earth's surface has a negative surface charge same resistance, their mass ratio will be

Fr
density of 10~^ Cm~^. The potential difference of400 kV between _ Pcu ^u _ 1.72x10"'^ x8.9 = 2.1558-2.2
the top of the atmosphere and the surface results (due to the low m
Pai ^A1 2.63x10““ x2.7
Al
conductivity of the lower atmosphere) in a current of only
for
ur
1800 A over the entire globe. If there were no mechanisfn of i.e., copper wire is 2.2 times heavier than aluminium wire.
sustaining atmospheric electric field, how much tune (roughly) Since aluminium is ligliter, it is preferred for long sus
would be required to neutralise the earth's surface ? (Radius of pension of cables otherwise heavy cable may sag down
ks
the earth = 637 x 10^ m). due to its own weight.
Yo

Ans. Surface charge density, 3.17. What conclusion can you draw from the following
oo

a= 10"^Cm“2 observations on a resistor made of alloy manganin :


eB

Radius of the earth, Cunent Voltage Current Voltage


1(A) V m V
R = 6.37 X 10^ m
0.2 3.94 3.0 59.2
Current, 1 = 1800 A
r

0.4 7.87 4.0 78.8


ou

Total charge of the globe,


ad

0.6 11.8 5.0 98.6


q = surface area x o = 4n R^a 0.8 15.7 6.0 118.5
Y

= 4x3.14x(6.37xl0^)^xl0“^ 1.0 19.7 7.0 138.5


= 509.65 X 10^ C 2.0 39.4 8.0 158.0
nd
Re

Required time.
‘I 509.65 X 10^ Ans. We plot a graph between current I (along y-axis)
Fi

= 283.13 s-283 s. and voltage V (along x-axis) as shown in Fig. 3.317.


I 1800

3.15. (fl) Six lead-acid type of secondary cells each ofemfl.O V


and internal resistance 0.015 Q arc joined in series to provide a
supply to a resistance of 8.5 Ci. What are the current drawn from 7-

the supply and its terminal voltage ? 6-

(b) A secondary cell after long use has an emf of 1.9 V and a t5
large internal resistance ofSSOCi. What maximum current can 4-

be drawn from the cell ? Could the cell drive the starting motor 3-

of a car ? 2-

Ans. (fl) HereS=2V, r=0.015n, R = 8.5n, n = 6 1

When the cells are joined in series, the current is 0


nt 6x2 12
/ = A =1.4 A
R+ nr 8.5+6x0.015 8.59
Fig. 3.317 V-I graph for manganin.
V.
3.152 PHYSICS-XII

Since the V-/graph is almost a straight line, therefore, {b) Given the resistance of in,2Q,3Q, how will you
manganin resistor is an ohmic resistor for given ranges of combine them to get an equivalent resistance of:
votlage and current. As the current increases from 0 to 8 A, 11

the temperature increases but the resistance of manganin


Q
(//) ^5 n (Hi) 6 n {iv) —11 Q ?
does not change. This indicates that the temperature [CBSE F 15]
coefficient of resistivity of manganin alloy is negligibly (c) Determine the equivalent resistance of the following
small. netzvorks :

3.18. Aziszoer the fiillowing questions :


(a) A steady current flows in a metallic conductor of
non-unijbmi aoss-section. Say zvhich of these quantities
is constant along the conductor : current, current
de7isihj, electric field, drift speed ? [CBSE D 15C, I"]
{b) Is Ohm’s law universally applicable for all conducting
elements ? If not, give examples of elements which
do not obey 0/i;;i's lazo.
(c) A lozo voltage supply from zvhich one needs high
current must have very lozo internal resistance.
Why ?
(d) Why a high tension (H.T.) supply of say 6 kV must
have a very large mternal resistance ?
Ans. (a) Only current is constant because it is given to (b)
be steady. Other quantities : current density, electric field
and drift speed vary inversely with area of cross-section. Fig. 3.318
(&) No, Ohm's law is not universally applicable for all Ans. {a) For maximum effective resistance, all the n
conducting elements. Examples of non-ohmic elements resistors must be connected in series.
are vacuum diode, semiconductor diode, thyristor, gas .'. Maximum effective resistance,
discharge tube, electrolytic solution, etc.
= nR
(c) Tlie maximum current that can be drawn from a
voltage supply is given by For minimum effective resistance, all the n resistors
g must be connected in parallel. It is given by
I max
r 1 1 1 1 n
— — + —I 1- n terms =
R R R R
Clearly, will be large if r is small.
(d) If the internal resistance is not very large, then the Minimum effective resistance.
current will exceed the safety limits in case the circuit is R

short-circuited accidentally. n

3.19. Choose the correct alternative : Ratio of the maximum to minimum resistance is
{a) Alloys of metals usually have (greater/ lesser) R
s _
tiR
— =n^: 1.
resistivity than that of their constituent metals, R,, R/n 1
(b) Alloys usually have much (lozoer/higher) temperature
coejficients of resistatzce than pure metals. (l))Here R, = in, Rj = ^ = 3^^
(c) Tlze resistivity of the alloy manganin is nearly (/) When parallel combination of 1 n and 2 n resistors
independent of/increases rapidly with increase of connected in series with 3Q resistor [Fig. 3.319(a)], the
temperature. equivalent resistance is
(d) The resistivity of a typical insulator (e.g., amber) is Ml
R=R^ + R3 = R. + R2 «3
greater than that of a metal by a factor of the order of
aO^/10^). 1x2 2 11
+3=-+3= n.
Ans. (fl) greater (b) lower (c) is nearly independent of 1+2 3 3

(d) 10^.
(//) When parallel combination of 2 n and 3 H resistors
3.20. (a) Given n resistors each of resistance R, how will you is connected in series with 1 n resistor [Fig. 3.319(&)], the
combine them to get the (i) maximum, (ii) minimum effective equivalent resistance is
resistance ? Wlzat is the ratio of the maximum to minimum R = Ml + R,= 2x3 6 11
+1=-+ i=—n.
resistance ?
R, + R3 2+3 5 5
CURRENT ELECTRICITY 3.153 A

IQ 2Q 1 Q IQ IQ IQ la A
i—VsA/- 3Q IQ Vs/v—^-VA—®
2Q 3Q wv
IQ IQ IQ IQ IQ
L-WV
ia) ib)
5 IQ 5 IQ ^ IQ ■'i IQ ’'l IQ 8
— ●— ♦—Vv\— ♦-V'lA—®
1 Q

r-WV-i Fig. 3.321


IQ 2Q 3Q 2Q
VA-VvV-WV 4_VW-<^ Ans. Let the equivalent resistance of the infinite
3Q network be X. This network consists of infinite units of
three resistors of 1 n, 1 0,1 H. The addition of one more
(c) {d) such unit across AB will not affect the total resistance. The
network obtained by adding one more unit would appear
Fig. 3.319
as shown in Fig. 3.322.

w
A IQ
(in) When the three resistances are connected in series —®p
[Fig. 3.319(c)], the equivalent resistance is

Flo
X IQ
R = i\ + R^ + R^={i+2 + 3)n = ea
IQ
{iv) When all the resistances are connected in parallel i aAAt-oQ

ee
[Fig. 3.319(rf)], 8

1-J_+ J_+ ^ 1 1 1 11 Fig. 3.322

Fr
R R^ R^ 3 6
Resistance between A and B

Equivalent resistance, R = ^Q. = Resistance equivalent to parallel


for
ur
combination of X and 1H
(c) The network shown in Fig. 3.318(fl) is a series
combination of four identical units. One such unit is Xxl X

shown in Fig. 3.320(a) and it is equivalent to a parallel X+1 X+1


ks
combination of two resistances of 2 Q and 4 Q as shown in
Yo

Resistance between Pand Q


oo

Fig. 3.320(&). X X
= 1 + + 1=2 +
X + 1 X+ 1
eB

2Q
r-Wv This must be equal to the original resistance X.
X
X = 2 +
r

1+ X
ou
ad

4Q
or X^-2X-2=0
Y

(1») or X = 1±V3
Fig. 3.320 As the value of resistance cannot be negative, so
nd
Re

X = l+V3 = 2.732Q
Resistance R of one sucli unit is given by
g
Fi

emf 12
1-1 1 2+13 Current, / =
R"2^ 4“ 4 “4 Total resistance X + r 2.732 + 0.5

= 3.713 A
or R = in
3 3.22. Figure 3.323 shows a potentiometer with a cell of
.●. Resistance of the total network (4 such units) 2.0 V and internal resistance 0.40 .O maintaining a potential
drop across the resistor wire AB. A standard cell which
4_^ n.
= 4x
3”T maintains a constant emf of 1.02 V (for very moderate currents
upto a few A) gives a balance point at 67.3 cm length of the wire.
(lY) The network shown in Fig. 3.320(b) is a series To ensure very low currents drawn from the standard cell, a
combination of 5 resistors, each of resistance R. very high resistance of600 k n is put in series with it, which is
.-. Equivalent resistance = 5R. shorted close to the balance point. The standard cell is then
3.21.Determine the current drawn from a 12 V supply ivith replaced by a cell of unknown emf € and the balance pointfound
internal resistanceO.SFlby thefbllowing infinite network. Each similarly turns out to be at 82.3 cm length of the wire.
resistor has 1 n resistance. [NEET Sept 22] (a) WJiaf is the value o/£ ?
PHYSICS-XII

2 V, 0.4 Q
323. Figure 3.324 shows a potentiometer circuit for comparison
of two resistances. The balance point with a standard resistor
R - 10.0 n is found to be 583 on, while that with the unknown
A^■ ■●B
resistance X is 685 an Determine the value of X. What might
G
you do if you foiled tofind a balance pomt with the given ce//€ ?
— \—
600 ka
1.02 V
A B

(&) Wiat purpose does


Fig. 3.323

the high resistance of -vw


R
%
600 AQ have ? X
AAV-^
(c) Is the balance point affected by this high resistance ?
g
(d) Is the balance point affected by the internal
resistance of the driver cell ?
Fig. 3.324
(e) Would the method work in the above situation if the
driver cell of the potentiometer had an emfof 1.0 V Ans. Here R = 10.0n,/j = 583cm, X =?,/2 = 68.5cm
instead of 2.0 V ? Let Sj and ^2 be the potential drops across R and X
(f) Would the circuit work well for determining respectively and 1 be the current in potentiometer wire.
extremely small emf say of the order of a few mV Then
(such as the typical emf of a thermocouple) ? If not, £1 “ 7R “ R
how will you modify the circuit ? ^2 _ h
But
An=. =1.02V,/j = 67.3cm,62 =£=?^2 =®23cm g1 h R I,
Formula for the comparison of emfs by poten h 68.5
tiometer is or X = -^.R = X10 = 11.750
K 583
^2 _ h g 82.3
g1 1.02 “ 67.3 If there is no balance point, it means potential drops
across R or X are greater than the potential drop across the
or g = — X 1.02 = 1.25 V. potentiometer wire AB. We should reduce current in the
67.2 outside circuit (and hence potential drops across R and X)
(b) High resistance of 600 kO protects the suitably by putting a series resistor.
galvanometer for positions far away from the balance 3.24. Figure 3.325 shows a 2.0 V potentiometer used for the
point, by decreasing current through it. determination of internal resistance of a 1.5 V cell. The balance
point of the cell in open circuit is 76.3 cm. When a resistor of9.5CI
(c) No, balance point is not affected by high resistance
is used in the external circuit of the cell, the balance point shifts
because no current flows through the standard cell at the
to 64.8 cm length of the potentiometer wire. Determine the
balance point,
internal resistance of the cell.
(d) Yes, the balance point is affected by the internal 2.0 V
resistance of the driver cell. The internal resistance affects

the current through the potentiometer wire, so changes


the potential gradient and hence affects the balance point, A
T T B

(e) No, the arrangement will not work. If € is greater


1.5 V
than the emf of the driver cell of the potentiometer, there
will be no balance point on the wire AB. ©
if) The circuit as it is would be unsuitable, because the 9.5 n ;
balance point (for g of the order of a few mV) will be very L _
--V'Ar---'
close to the end A and the percentage error in measure
Fig. 3.325
ment will be very large. The circuit is modified by putting
a suitable resistor R in series with the wire AB so that Ans. Here /, = 76.3 cm, = 64.8 cm, R = 9.5 n
potential drop across AB is only slightly greater than the The formula for the internal resistance of a cell by
emf to be measured. Then the balance point wUl be at potentiometer method is
larger length of the wire and the percentage error will be r=R
h h = 9.5 76.3 - 64.8') 9.5 X 11.5 = 1.7n.
much smaller. 64.8 64.8
CURRENT ELECTRICIT 3J>5--

OTQs : Objective Type Questions


■fl’YPE A : Multiple Choice Questions (i mark each)

Based on Definitions of Electric Current and EMF 9. The colour sequence in a carbon resistor is red,
brown, orange and silver. The resistance of the resistor is
1. Which of the following is responsible for the flow
of current in a conductor ? (fl) 21x lO^n ±10% (6) 23x lO’n ±10%
(fl) protons (b) free electrons (c) 21x10^^0 ±5% (d) 12 X lO^n ± 5%
(c) positive ions (d) protons and holes 10. A carbon resistance is having a following
2. The current in a conductor varies with time t as
coding: green, orange, black, gold. The resistance of the

w
resistor is
7=2f+3f^, where I is in ampere and t in seconds.
Electric charge flowing through a section of the (fl) 53x10“ ±5% (b)53xl0’±5%
conductor during t =2 sec to f = 3 sec is (c) 53xl0“±10% (d) 53xl0±10%

Flo
(fl)lOC (&)24C (c)33C (d)44C Based on Drift Velocity and Mobility of Electrons
3. How much work is required to move a 6|iC

ee
11. If the potential difference V applied across a
charge from the negative to the positive terminal conductor is increased to 2 V with its temperature kept

Fr
battery of 9 V ? constant, the drift velocity of the free electrons in the
conductor will [CBSE SP 22]
(fl) 54xlO“^J (b) 54xlO“^J
(fl) remain the same
for
ur
(c) 54xlO“^J (d) 54x10“^2j
(&) become half of its previous value
Based on Ohm's Law, Resistance, Resistivity,
(c) be double of its irutial value
Conductance and Conductivity
ks
(d) become zero.
4. What is the unit for specific resistivity ?
Yo

12. A battery is connected to the conductor of


oo

(fl) ohm cm^


-1
(b) ohm cm non-uniform cross-sectional area. The quantities or
(c) ohm cm (d) ohm cm“^ quantity which remain constant is/are
eB

[CBSE SP 22]
5. A 1 m long wire is broken into two unequal parts (fl) electric field only
X and y. The X part of the wire is stretched into (b) drift speed and electric field
r

another wire W. Length of W is twice the length of X (c) electric field and current
ou
ad

and the resistance of W is twice that of Y. Find the ratio

of lengths of X and Y. (d) current only


Y

(fl) 1 : 4 (b) 1 : 2 13. Electric field (E) and current density (;) have
relation
(c)4:l (d) 2 : 1 [JEE Main July 22]
nd

(b) Eazj (c) £ X 4- (d) T)


Re

-1

6. What length of the wire (specific resistance (fl) £ X j


48x 10"®Qm) is needed to make a resistance of 4.2 Q ?
Fi

(Diameter = 0.4 mm) 14. The mobility of charge carriers increases with
(fl) 1.1 m (b) 3.1 m ( ; increase in the average collision time
(c) 2.1 m (d) 4.1 m (b) increase in the electric field
7. The ratio of masses of three wires is 1 : 2 : 3 and (c) increase in the mass of the charge carriers
that of their lengths is 3 : 2 :1. If the wires are made of (d) decrease in the charge of the mobile carriers
same material, the ratio of their resistances will be
15. Assume that each atom of copper contributes
(fl) 1 : 1 : 1 (b) 1 : 2 : 3 one electron. If the current flowing through a copper
(c) 9 : 4 : 1 (d) 27 : 6 : 1 wire of 1 mm diameter is 1.1 A, the drift velocity of
8. If a wire of resistivity p is stretched to double its electrons will be (density of Cu =9gcm“®, atomic wt.
length, then its new resistivity will of Cu = 63)
(fl) be half (b) be 4 times (fl) 0.3 mm/s (b) 0.5 mm/s
(c) be double (d) not change (c) 0.1 mm/s (d) 0.2 n\m/s
3.156 PHYSICS-XII

16. A current of 10 A exists in a wire of 21. Resistance of the wire is measured as 2 Q and
cross-sectional area of 5mm^ with a drift velocity of
2xl0”^ms“\ The number of free electrons in each 3Q at 10°C and 30°C respectively. Temperature
co-efficient of resistance of the material of the wire is
cubic metre of the wire is -1 -1
25 23 (fl) 0.033°C {b) - 0.033®C
(fl) 2 X10 (&)lxl0 -1 -1
25 (c) 0.011°C (d) - 0.055°C
(c) 625x10 (d) 2 X10^ [JEE Main March 21] [JEE Main June 22]
17. Drift velocity varies with the intensity of the 22. Atl000°C the resistance of a wire is found to be
electric field as per the relation four times its resistance atO°C. The temperature coefficient
1
of resistance of the material of the wire will be
(a) v^xE
(fl)0-004/®C (b)0-002/°C
(c) = constant (d) Vj cc £2 (c)0-003/“C (ri)0-001/°C
23. The temperature coefficient for a wire is
Based on Temperature Variation of 0.00125°C'"^. At 27°C, its resistance is ID. The
Resistivity and Superconductors temperature at which the resistance becomes 2Q is
(a) 1154 K (b) HOOK
18. By increasing the temperature, the specific
resistance of a conductor and a semiconductor (c) 1400 K id) 1127 K
[a) increases for both 24. The voltage V versus current I graphs for a
(&) decreases for both conductor at two different temperatures are shown in
(c) increases for a conductor and decreases for a the figure.
semiconductor

(d) decreases for a conductor and increases for a


semiconductor [CBSE SP 23]
19. The temperature (T) dependence of resistivity
of materials A and material Bis represented by Fig. (i)
and Fig. (ii), respectively. Identify material A and /
material B.
The relation between Tj and T2 is
P- P“
ia)T,>T2 (&)Tj<T2
(c)T,=T2 (d)Tj=2T2
25. We use alloys for making standard resistors
because they have
T T
(fl) low temperature coefficient of resistivity and
high specific resistance
Fig- (0 Fig- (")
(b) high temperature coefficient of resistivity and
low specific resistance
(a) Material A is copper and material B is (c) low temperature coefficient of resistivity and
germanium
low specific resistance
(b) Material A is germanium and material B is copper (d) high temperature coefficient of resistivity and
(c) Material A is nichrome and material B is high specific resistance
germanium Based on Series and
(d) Material A is copper and material B is nichrome Parallel Combinations of Resistances
[CBSE SP 23]
26. Two resistors of resistances and R2 (R2 < ^1)
20. The resistances of a wire at a temperature f°C
and 0®C are related by are cormected in parallel. For equivalent resistance R,
the correct statement is
(a) R, = Rq(1 + at) (b) R, = R^{l-at) (fl) K > Rj + J?2 ib) R2 < R < R^
(c) R, =R^(l + af) id) R^=R^il-at) (c) R2 < R < Rj + R2 (d) R < R2
3J57

27. An electric current is passed through a circuit (fl)o-oixio"^n (b) 4-44Q


containing two wires of same material, connected in (c)0-888n (d) lx 10"^ a
parallel. If the lengths and radii of the wires are in the
ratio of 3 ; 2 and 2 : 3, then the ratio of the currents 35. If you are provided three resistances 2 .0 ,3 Q
passing through the wires will be [CBSE SP 22] and 6 O, how will you connect them so as to obtain the
(fl) 2 : 3 (b)3:2 equivalent resistance of 4 H ?
(c) 8 : 27 (d) 27 : 8 ■VA—VvV
30 20
28. A wire has resistance of 120. It is bent in the
(«)
60
form of a circle. The effective resistance between the
two points on any diameter of the circle is
(fl) 12 O (b) 24 a
AM WV

w
(c)60 (d)3Q 30 60

29. A metallic wire of resistance 120 is bent to form (&)


20

a square. The resistance between the two diagonal MV

Flo
points would be
(fl)12 O (fa) 24 a AM

ee
30 20
(c)6 0 (d)3n (C) AM—o

Fr
o-

60
30. Three resistances of 20 each are connected in
M/V
a triangle. The resistance in ohms, between two
(d) none of these for
ur
vertices is

(fl)3 0 (fa) 4 a 36. Tl\e equivalent resistance between A and B is


ks
(c)60
Yo

31. By using only two resistance coils-singly, in


oo

series or in parallel one should be able to obtain


eB

resistance of 3, 4, 12 and 16 O.

The separate resistances of the coils are


(fl) 3 and 4 (b) 4 and 12
r

(fl) 3 ohms (fa) 5.5 ohms


ou

(c) 12 and 16 (d) 16 and 3


ad

(c) 7.5 ohms (d) 9.5 ohms [CBSE SP 22]


32. The current in a simple series circuit is 5.0 A.
Y

When an additional resistance of 2.0 O is inserted, the 37. Resistances of 1Q, 2 Q and 3 a are connected to
current drops to 4.0 A. The original resistance of the form a triangle. If a 1.5 V cell of negligible internal
nd
Re

circuit in ohms was resistance is connected across the 3 Q resistor, the


(a) 1.25 (fa) 8 current flowing through this resistor will be
Fi

(c) 10 (d) 20 (rt) 0.25 A (fa) 0.5 A


33. A parallel combination of two resistors, of la (c) 1.0 A (d) 1.5 A
each, is connected in series with a 1.5 a resistor. The
38. The magnitude of / in ampere is
total combination is connected across a 10 V battery.
60Q
The current flowing in the circuit is
I
(a) 5 A (fa) 20 A 15 5Q

(c) 0.2 A (d) 0.4 A o- > AA^V—WV >


1 A 1 A
34. Cu and Al wire each of length / = 20 cm and area lOfl

of cross section A = 50mm^. Their resistivity


Pcu =l'69xl0“*^am and =2-75xl0”®am. If they (a) 0.1 (fa) 0.3
are joined end to end, then total resistance of
combination is (c) 0.6 (d) none of these
3:158 PHYSICS-XIl

39. What is the value of R ?


43. The current I in the given circuit is
R / = 0.25A AW h
'Wv—► ion 5 V
..1
<> 2V 20 n
T 12 V ion i> 6on
AW

{a) 0.1 A (b) 0.3 A


(c) 0.2 A (d) 0.4 A
{a)42Q (&) 62 n
(c)84Q (d) none of these 44. In the given diagram, the reading of the
40. A current of 15 mA flows in the circuit as shown
ammeter (when the internal resistance of the battery
is zero) is
in the figure. The value of potential difference between
5
40
the points A and B will be («)^A A
29

10 kn 10 4n 10 V
wv (I^)yA 5n

< ●—WV— ►
5kn
(c)|a
10 kn id) 2 A

15 mA
45. All resistances in figure are in each. The value
of current '/'is [JEE Main June 22]
5kn A B

(a) 50 V (b) 75 V
rWW
(c) 150 V (d) 275 V [JEE Main July 22)
r-WV-
41. The equivalent resistance between points A and L-WV
B in the given network is
-WV
5n
Wv
rVW
-I-
^3V L_WV
l_\w

w|a
8
(^^)-A
(n) 65 Q (b) 20 n
8
(d)^A
8
(c)5Q (d)2Q
[JEE Main June 22] 46. In the circuit shown the value of I in ampere is
42, The current / in the given circuit is 4n

WV
AW 4n
2A
I
3n
I / rAWn 4n
o- ●o 4n AW
6n i^VWi
AW 4V i.6n
1 AW
(fl)l A (b)6A
(c)3A
(fl)l (b) 0.60
(d) 12 A
(c) 0.4 (d) 1.5
3Jy9

47. In the circuit shown in the figure, the potential 51. The reading of the ammeter in the circuit shown
below is
difference between X and Y will be
2a
40 a X y ■vw
●o o-
2a 2V

—11—®
in
120 V 20 a
I- ●WV 2a

(fl) zero {b) 20 V


(&)^A
1
(c) 60 V (d) 120 V («)^A
8
48. If the ammeter in the given circuit reads 2 A, the 1

w
(d)2A
resistance R is
^2^
30
52. The current drawn from the battery in the
f-A/vVi R
circuit shown below is

Flo
A/vV . 70
20
L^VVv--'
60 ■VAr t-ww

ee
15 V T

<S>

Fr
V 60 lO
0.5 0
6V

(6)2Q MV A/W
(a) in
for
ur
B
80 10 O
(c)3n (d)4a
{a)lA {b)2A
49. In the given circuit, the voltmeter records 5 V.
ks
(c) 1.5 A (d) 3 A
<v>
Yo

53. Find the equivalent resistance across AR


oo

AO
AAAv
eB

100 o 50 0
20
20
r

h <■> 20
ou
ad

10 V
20
The resistance of the voltmeter in ohms is
Y

20

(a) 200 (b) 100


nd

Bo
(d) 50
Re

(c) 10
50. The potential difference between the points A (fl)lQ {b)2n (c)3a (d)4n
Fi

and B of the circuit shown below is


54. The equivalent resistance between points A and
50 50 B of an infinite network of resistances, each of 10,
A B connected as shown, is
lO lO 1 O

50 AO—\AAr AAA/—I—VvV

lO 1 o lO
D c
50 50

Bo-
8

W|v (b)-V
9 (rt) infinite {b)2n

, ,1+V5 a
(c)|v (d)2V (d) zero
1.3.160 PHYSICS-XII

55. The equivalent resistance between the terminals 60. A cell of emf 1.5 V and internal resistance is
A and D in the following circuit is connected to two resistors of 5 and 8 n in series. The
5fi ion 5Q potential difference across the 5 Q resistor will be
AA/V AA/V—OB
(fl) 3.3 V {b)lV
ion ion (c) 0.5 V (d) 0.33 V

CO—VW—*—VW AAAr-oD 61. When a current of 0.2 A is drawn from a battery,


5n ion sn then potential difference between its terminals is 20 V
and when a current of 2 A is drawn then the potential
{a) 10 n (b) 20 Q difference drops to 16 V. The emf. mattery is
(c) 5n id) 30 Q (fl) 15.1 V (b) 20.4 \
56. The effective resistance between A and B in the (c) 18.9 V (d) 23.3 V
figure shown is
62. If 3 A of current is flowing between points P and
Q in the circuit, then the potential difference between
P and Q is
2V
3A
Bo- oQ
2n 4n

(fl)3n (b)4-50 (a) 30 V (b)22 V


(c) 4 a (d)60 (c) 20 V (d) 15 V
57. For the circuit shown in the figure, determine 63. For a cell, the terminal potential difference is
the charge of capacitor in steady state. 3.6 V, when the circuit is open. If the potential
3V 3V difference reduces to 3 V, when the cell is connected to
1 1 a resistance of 5Q, the internal resistance of the cell is
1 10 V (a) in (&)2 0
1 n
sn (c) 4Q {d)8 0
2n

l-AVv ■VvV- 64. A cell supplies a current of 0.9 A through a2Q
2Q
6V 1 mF
in
resistor and a current of 0.3 A through a 7Q resistor.
The internal resistance of the cell is

(«)4pC {b)eiiC (fl) 2.0 0 (b) 1.5 0


(c)lpC (d) Zero {c) 1.00 (d) 0.5 O
65. A straight line plot showing the terminal
Based on EMF, Infernal Resistance and potential difference (V") of a cell as a function of current
Terminal P.D. of Cells (I) drawn from it, is shown
58. A lead-acid battery of a car has an emf of 12 V. If in the figure. The internal
resistance of the cell would 5a
the internal resistance of tlie battery is 0.5 O, the
be then
maximum current that can be drawn from the battery o 4,2
vdll be
(fl) 2.8 O 2.8

{a) 30 A {b) 20 A (i>) 1.4 0 1.

(c)6A (d)24 A (c) 1.2 0


0.5 1.0 1.5 2.0
59. A battery of emf lOV and (d) zero [CBSE SP 22]
g I (ampere)
internal resistance 3O is

connected to a resistor as shown


Based on Grouping of Cells
in the figure. If the current in the R
circuit is 0-5A, what is the AA/y 66. Six identical cells, each of emf of 6 V, are

resistance of the resistor ? connected in parallel. The net emf across the battery is
(a) 130 (a) 6 V (b) 36 V
(b) 170
(c) 15 O (c) 0 V (d) between 6 V and 36 V
(d) 19 O
3.161

67. Two similar accumulators each of emf £ and 72. Two batteries of emf 4 V and 8 V with internal
internal resistance r are cormected as shown in the resistances 1 O and 2 respectively are connected to
adjacent figure. £
an external resistance R = 9 Q as shown in figure. The
Then, the potential I-
current in circuit and the potential difference between
difference between x P and Q respectively will be
and y is X‘ - ■■y 4V 8V
p in Q
(fl)2£ E i-AW Wv
{b)E h
''i

(c) zero
(d) none of these
68. To supply maximum current, cells should be R = 9n

arranged in 1 1

w
(b) _ A, 12 V
(a) series (1)) parallel (a) ^ A, 9 V 12
(c) mixed grouping
(d) depends on tlie internal and external resistance (c) 3i A, 3 V (d) - A, 4 V

Flo
6
69. A battery consists of a variable number (n) of
73. Two cells of emf 2€ and & with internal
identical cells, each having an internal resistance r

ee
connected in series. The terminals of the battery are resistance and Tj respectively are connected in series
short-circuited. A graph of current (1) in the circuit to an external resistor R (see figure). The value of R, at

Fr
versus the number of cells will be as shown in figure 2g e
I 1
(a) / (b)
for
ur
R
VA
ks
Yo

which the potential difference across the terminals of


oo

(c) id)
the first cell becomes zero is
/
I
eB

(a) (b) rj -r^

(c)y + ^2 h
^ [JEE Main March 21]
r
ou
ad

Based on Heating Effect of Electric Current,


70. Internal resistance is 0.5 each. If in C, the
Y

Electric Power and Electric Energy


negative is connected to B, the current will be
74. If the resistance of the wire is made four times
nd
Re

2V 2V 2V
h H
keeping potential difference across it constant, then the
number of times heat produced in it varies by
Fi

A B C

(fl)4 ml (c)2 id) I


4.5 a
75. A hot electric iron has a resistance of 80 and is
used on a 200 V source. The electrical energy spent, if it
(fl) 6/6 A (&) 6/5 A is used for 2 h, will be
(c) 2/5 A (d) 2/6 A (fl) 800 Wh (b) 2000 Wh
71. The combination of two identical cells, whether
(c) 1000 Wh (d) 8000 Wh
connected in series or parallel combination provides
the same current through an external resistance of 2 fl. 76. If percentage change in current through a resistor
The value of internal resistance of each cell is is 1%, then the change in power through it would be
(a) 2 a (b)4n ia) 1% (b) 2%
(c)6a (d)8a [JEE Main June 22] (c) 1.7% (d) 0.5%
3.162 PHYSICS-XII

77. An electric bulb of 500 watt at 100 volt is used in Time taken to boil the same amount of water when
a circuit having a 200 V supply. Calculate the both coils are connected in series is
resistance R to be connected in series with the bulb so
(fl) 5 min (b) 8.6 min
that the power delivered by the bulb is 500 W.
(c) 35 min (d) 30 min
(fl)5n (b) 20 a
85. An electric kettle has two heating elements. One
(c) 30 n id) 10 n [JEE Main Aug 21] brings it to boil in 10 min and the other in 15 min. If
78. Two wires A and B of same material and mass
two heating elements are connected in parallel, the
have their lengths in the ratio 1:2. On connecting them water in kettle will boil in
to the same source, the rate of heat dissipation in B is
(fl) 5 min (b) 6 min
found to be 5 W. The rate of heat dissipation in A, is
(c) 7 min id) 25 min
(fl) 10 W (b)5W
86. An electric kettle boils some water in 16 min.
(c) 20 W id) none of these
Due to some defect, it becomes necessary to remove
79. A constant voltage is applied between the two 10% turns of heating coil of the kettle. Now, how much
ends of a uniform metallic wire, heat' H' is developed time will it take to boil the same amount of water ?
in it. If another wire of the same material, double the
(fl) 19.6 min (b) 12.7 min
radius and twice the length as compared to original
(c) 14.4 min (d) 15;0 min
wire is used, then the heat developed in it will be
(fl)H/2 ib)H 87. Domestic electrical wiring has three wires
(c)2H (d) 4H (a) Positive, negative and neutral
(CBSE SP 22]
(b) Positive, negative and earth
80. Three resistors having values Pj, R^, and Rj are (c) Live, neutral and earth
connected in series to a battery. Suppose R 1 cames a
(rf) Positive, negative and live
current of 2.0 A, R2 has a resistance of 3.0 ohms, and R^
dissipates 6.0 watts of power. Then the voltage across Based on Kirchhoff's Laws
^ is 88. Kirchhoff's first law for the electric junction is
(fl)l V based on
(6)2V
(c)3V (d)4 V (CBSE SP 22] (fl) Law of conservation of charge
81. The power of a heater is 1000 W atl000°C What (b) Law of conservation of energy
will be its power at 400°C ? (c) Law of conservation of angular momentum
Given, temperature coefficient of resistance of (d) Law of conservation of mass
heater-wire is 1.4xl0“^®C”^
89. A current of 6 A enters
ia) 4.2xlO^W ib) 1.3xlO^W one comer P of an equilateral
(c)9.68xlO^W (d)1.08xl0^W triangle PQR having 3 wires of
82. An electric motor runs on DC source of emf resistance 2Q each and leaves

200 V and draws a current of 10 A. If the efficiency be by the comer R. The currents /j
40%, then the resistance of armature is in ampere is
[JEE Main Feb 21]
(fl)2n ib)8Q
ia)2A (&)3 A
(c) 12 n id) 160
83. A cell of constant emf is first connected to a (c)5 A (d)8A

resistance Rj and then connected to a resistance R^. If 90. In the given 2 V


power delivered in both cases is same, then the figure, the value of Vq
1 kn

internal resistance of the cell is will be J -VW


R]
(b) (fl)l V 4 V 1 kf2

ib)3V AAV

(0^ (d)
2
(c)4 V
(rf)8V
6V

I—Wy
ikn

84. A coil takes 15 min to boil a certain amount of (JEE Main July 22]
water. Another coil takes 20 min for the same process.
3.163

91. In the circuit diagram shown below, the electric 96. In a potentiometer arrangement, a cell of emf
current through branch BC is 1.5 V gives a balance point at 27 cm length of wire. If
B 2A 3A the cell is replaced by another cell and balance point
A ● WV ■Wv— ^ shifts to 54 cm, the emf of the second cell is
2A
(fl)3V {b) 1.5 V
(c) 0.75 V (d) 2.25 V
C A/VV D
97. A cell of emf 10 V and internal resistance30. is
connected to a uniform wire of length 500 cm and
"5A resistance 30. The potential gradient in the wire is
F 4 VW AMr G
(b) 10 mV cm'^
> -1
2A
(fl) 30 mV cm
(d) 4 mV cm"^
-1
(c) 20 mV cm
(fl)4A {&)2A
98. A 10 m long wire of uniform cross-section and

w
(c)5A (d) 10 A
20 O resistance is used in a potentiometer. The wire is
92. Electric current through resistance lOO in the connected in series with a battery of 5 V alongv^dth an
circuit shown below is 6 = 5 V 480 n

Flo
6V 2n
I ■vw
/1
●« -I VS^- 10 m

ee
6 m

ion Q

Fr
A/W

2n 6'
●<- I WV
for
external resistance of 480 Q. If an unknown emf S' is
ur
^2 6V
balanced at 6.0 m length of the wire, then the value of
(fl) 0 A (b) 0.5 A unknown emf is
ks
(c) 6/11 A {d)2A (fl) 1.2 V (b) 1.02 V
Yo

93. The value of is


oo

1 (c) 0.2 V (d) 0.12 V [CBSE SP 22]


99. Two cells when connected in series are balanced
eB

30 n
-VW on 8 m on a potentiometer wire. If the cells are con
I nected with polarities of one of the cells reversed, they
i .‘I 40 V
40 0 ^3 balance on 2 m. The ratio of the emfs of the two cells is
r

A/W I <■
ou

(fl)3:5 {b)5:3 (c) 3 : 4 (d) 4 : 3


ad

40 n 80 V 100. For a cell of emf 2 V, a balance is obtained for


Y

VW i
50 cm of the potentiometer wire. If the cell is shunted
by a 2 Q resistor and the balance is obtained across 40 cm
nd

(fl) 0.4 A {b) - 0.4 A of the wire, then the internal resistance of the cell is
Re

(c) 0.8 A (d) - 0.8 A (fl)in (&) 0.5 O


Fi

(c) 1.2 n (d) 2.5 O


Based on Potentiometer
101. In the given figure, there is a circuit of
94. The best instrument for accurate measurement
potentiometer of length AB = 10m. The resistance per
of emf of a cell
unit length is 0.10 per cm. Across AB, a battery of emf
(fl) potentiometer (b) metre bridge 6 and internal resistance
+
6 r

‘r' is connected. The maxi I -VW


(c) voltmeter
mum value of emf
(d) ammeter and voltmeter [CBSE SP 22]
measured by this I
95. Cell of emf 1 V is connected across a potentio potentiometer is A\^ -fP
meter, balancing length is 600 cm. What is balancing (fl) 5 V ib) 2.25 V 550 cm 450 cm
/
length for 2.5 V ?
{c) 6 V (d) 2.75 V +
(fl) 400 cm (b) 600 cm ●Wv—
K
[JEE Main July 21] 6V
20 n
(c) 1500 cm (d) 1200 cm
L 3,164 PHYSICS-XIl

102. Potentiometer wire of length 1 m is connected 11 n. Which of the following steps will not bring the
in series with 490 Q resistance and 2V battery. If bridge to balance again ?
0.2 mV / cm is the potential gradient, then resistance of p = ion Q = ioon
the potentiometer wire is AAAr
(fl) 4.9 n
(c) 5.9 n
(b) 7.9 a
(d) 6.9 Q
R = 20Q
1 -AAA^
S = 200 Q
103. In the potentiometer experiment there are 10
lines in the potentiometer. In first situation null point is h AW
at 7th line. If we want to obtain the null point in the 9th
line, then it can be done by («) increasing K by 2 O
(fl) increasing resistance R in series with the battery (b) increasing Q by 10
whose emf is to be measured, (c) increasing S by 20 Q
(i?) decreasing resistance R in series with the (d) making product RQ =2200 (Q)^
battery whose emf is to be measured, 108. In the circuit shown in figure, the value of
(c) increasing resistance R in main circuit, resistance x, when the potential difference between the
(d) decreasing resistance R in the main circuit. points B and D is zero, will be

Based on Wheatstone Bridge and Metre Bridge


104. In the balanced Wheat
stone's bridge circuit as shown in
the figure, when the key is pressed,
what will be the change in the
reading of the galvanometer ?
R
{a) remains same
(b) increased
(c) decreased
V
(d) none of these
B D
105. What is the current in BD ? O'
I-
(fl) zero
(ij) 0.033 A
A
(§) C (fl)9fl (b)8Q
{c)6n (d)4Q
(c) 0.066 A * O'
■Q 109. Seven resistances are connected as shown in
(d) none of these D the figure. The equivalent resistance between Aand Bis
106. The equivalent resistance I (fl)3n (b) 4n
{R^g) between the points A and B is 2V
(c) 4.5 n (d) 5n
c ion
AW
o 5? 3n
AO- ●9- wv wv OB
A 3Q B ion

5n 8n 6n 6n
o
.●V
D

110. In a metre bridge experiment, resistance box


(fl)6 0 (b) 7.50 (R = 20) is connected in the left gap and the unknown
resistance S in the right gap. If the balancing length be
(c) 4.50 (d)80 40 cm, the value of S is

107. The given figure shows a balanced Wheat («)2n (b)30


stone's network. Now, it is disturbed by changing P to (c) 40 (d) 2.5 O
111. Resistances in the two gaps of a metre bridge
are 10 Q and 30
112. Two resistances are connected in two gaps of a
respectively. If the resistances are metre bridge. The balance point is 20 cm from the zero
end. A resistance of 15 Q is connected in series with the
3.165
/
y

interchanged, the balance point shifts by


smaller of the two. The null point shifts to 40 cm. The
(fl) 33.3 cm (b) 66.67 cm value of the smaller resistance in ohms is
(c) 25 cm (rf) 50 cm («)3 {b)e (c)9 (d)12

Answers and Explanations T


,2 2^ ^
1. (b) In a conductor, current is carried by free electrons. R — —=9:2:-
. 1 'z ^ 1 2 3 3
2.(b) Given I=2t+3t^
dq = 27:6:1.
As dq=Idt={2t + 3t^)dt

w
dt
8. (rf) Resistivity depends on the nature of the
material and not on the dimensions of the conductor.
Charge passed from t =2s to t =3s is
3
9. (fl) Red Brown Orange Silver
q^{dq=\i2t + 3t^)dt +

Flo
4.
2
2 1 3 ±10%
= (9-4) + (27-8) = 24C.

ee
R= 21x10^0 ±10%.
3. (b) W = i/K = 6pCX 9 V = 54 )ij = 54 X10”^ J

Fr
10. (<?) Green Orange Black Gold
4. (c) The unit of resistivity is ohm cm. i
4^ 4^
5. (b) ±5%
5
for 3 0
ur
I 1 -/
R=53xl0®n±5%
a H D
X y
11- (c) v^=—T= — z
ks
m ml
Yo
21
D At constant temperature, relaxation time r remains
oo

a
w
constant. Also, m and e are constants.
eB

When wire X is stretched to twice its length, its


resistance increases 4 times.
When V becomes 2V, drift velocity becomes
Rw = 4Rj^ =2Ry double of its initial value.
r
ou

Jk-i 12. (d) A conductor of non-uniform cross-section is


ad

RY 2 a series combination of many conductors of different


Y

cross-sections. So I through them remains constant.


X- = 1:2.
nd

I Ry
Re

RA RxttD^ 4-2x22x(0-4xl0~^)^ uiuUjj


Fi

6. (a) 1 =
P 4p 7x4x48x10'' I'-

= l-lm.
+

7. (d) Mass = Volume x density = Aid V


m
or A = — / 1
V => z?. X —
Id d
enA A
I I l^d
I 1
A mild m E = £ cc —
aA A

i.e., Rx — I 1
m
;=-r ) X
A A
Given m^:m2'.m^ = l : 2 : 3
and V/2:/3=3:2:1 Thus , £ and j all change with A
3.166 PHYSICS-XII

V
13.(b)E ^p/. 24. (fl) 1? = —I = Slope of V-I graph
‘1'^ :
14. (fl) Mobility, )i =
m
i.e., {iccT
Slope of V-I graph at > Slope of V~I graph at T2
23 R^>
15. (c) No. of atoms in 63 g of copper = 6x 10
As the resistance of conductor increases with
Volume of 63 g of copper.
Mass
temperature, so Tj > T2.
63 g
= 7cm^=7xl0“^-"^
m

Density 9gcm"^ 25. (a) Alloys have low temperature coefficient of


23 resistivity and high specific resistance. With small a, R
6x10 6
n ~ = -xl0" m
29 _-3
will not change appreciably with temperature and
7x10“^^ 7 high specific resistance ensures that required length of
the resistor wire is small.
A = nr^ =7tx(0.5xl0“^)^m^
Drift velocity. 26. (d) Equivalent resistance of a parallel combi
nation is smaller than the smallest individual resistance.
I 1.1x7
V

enA 1.6xl0'^^x6xl0 29 X71x(0.5x10‘^)^ h


27. (c) R,=P~^ and f?2 = p —
= 0.1x10“^ ms -1 = 0.1 mms
nrf nr^
16. (c) I = enAv^ tlL=!lx^=~x
R
-3'\^ 27

n =
I R2 I2 2 [2 8

eAVd h_V/R, R2 8
10
-19
m
-3
I2 VIR2 R^ 27
x5xl0"^x2xl0"^
1.6x10

= 625xl0“m"^ 28. (rf) The two semicircular parts, each of 6 D, are in


eEx
parallel.
17. {<?) V d ~ ^ (9^ QC £ R =
6x6
= 3Q.
m
6 +6
18. (c) Temperature coefficient of resistivity (a) is 6x6
+ve for a conductor and -ve for a semiconductor. 29. (rf) R AB =3a
6+6
Specific resistance of a conductor increases and that of
a semiconductor decreases with the increase in B
3H
temperature.
19. (b) The resistivity of a semiconductor decreases
while that of a metal increases with the increase in 3Q 3fl

temperature. Material A is germanium and material B


is copper.
20. (fl) K, = R(j(l + at) /I

21. (a) R= Rfj(l + aAT) 30. (d)

2 = R(,(l + 10a) C

3= Kg(l + 30a)
2 _l + lQq 20 20
40.

3 ~l + 30a
a = — =0.033°Cr^. Ao wv oB /lo- ■OB
30 20 20
4x2
22. (b) a = = 0.003°C"^ R
AB ~
=-n
4+2 3
X t Rq X1000 1000
_R2~Ri_ 2-1 31. (b)
23. (b) t2-t,= °C=800°C ax

R^a ~ 1x0.00125
= =3 0
_

=800+ =800+27 =827°= 1100 K. R^ + R^


min
3.

or R,(16-Ri)_g 38. (fl) Resistance in upper arm, = 60 Q


16
Total resistance of lower two arms,
or
16Rj-R? =48 20x10 _20^
or
Rf-16Rj+48 = 0 20 + 10 3

On solving R^ = 4 Q, R2 = 12 Current through 60 H resistor,


32.{&)
; =
lAx R^_
In first case, y = /R = 5R
Ri + R2 60 + ^
In second case, V = /’R'=4(R+2) 20
— = 0.1A.
5R = 4(R+2) or R = SC1. 3 ^200
1x1
39. {a) Equivalent resistance R' of three resistances

w
33. (fl) R = + 1-5=20
1 +1 10 Cl, 20 Cl, 60 Cl connected in parallel is given by
1^ = 10 V
R' " 10 20 60 60 6

Flo
/ = ^=1^=5A.
R 2 R'=6Ci

ee
34. (fl)
^ = ^u + ^ai=[Pcu+Pai17 Now R and R' are connected in series

Fr
V 12
R+ R' = — = 48
(l-69 + 2-75)xlQ~^x20xl0~^ n I "0-25
-6
50x10
for
ur
R = 48-R'=48-6=42a.
= 1-776x10
-A
0-01xl0"^n
40. (d)
3x6 275
10x5
ks
35. (c) R = + 2 =2+2 =40 kO
+ 10 =
3 +6 «4B = 5 + 10
Yo
+ 5 15
oo

36. (c) The equivalent circuit is shown in the figure. 275


The effective resistance between points C and D = 15mAx kO=275 V.
eB

15
3x6
3+6
+ 8=100
41. (c) From the left of the network, a series combi
nation of 5 O and 5 O is in parallel with 10 O to give an
r

3Q equivalent resistance of 5 O. Tlie same pattern repeats


ou
ad

AAV four time.


D
C
Y

o
A 60 B
80
AAAr-1
42. (c) Current through 60 resistor
nd

30 0
Re

AAV
= ^X Current through 3 O resistor
Fi

Now the 100 and 300 resistances are in parallel.


The equivalent resistance between points A and B = ix2 =1 A
2
_ 10x30 = 7.5 O. /=2A + 1A = 3A
“10+30
43. (a) The two cells are connected oppositely.
37. (b) The series combination of 10 and 2 O is in
Total emf =5-2 =3 V
parallel with 3 O resistor. Total resistance =10+20 =30 0
R = — = -=-0 3V
3+3 6 2 Current = = 0-1 A
30 0

/ = V ^ 1-5 = 1.0 A
R ’3/2 44. (d) Reading of the ammeter
10 V
= 2 A
-x 1 = 0.5 A. = Current through 5 O resistor =
Current through the 3 O resistor = ^6 50
3:168 PHYSICS-XII

45.(b) Total resistance =2 + R

3R/2 V 6
/ =— or 2 =
R R 2 + R
nMAr- 3R/2
R I— 2+R = 3 or R=1Q
3R/2
49. (b) Voltage across parallel combination of
+
3R/2 voltmeter V and 100 O resistor
3V-:t R
L-WV
= Voltage across 50 Q resistor = 5 V.
I
Equivalent resistance of voltmeter and 100
resistor = 50 Q

7i?/4
or
lOOX Kyr = 50
r-VW-i
R 100 + Ry
-vw
UvW-<
or
2 Ry — 100 "f* Ry
+
7R/4
3 V-:t or
Ry = 100 n.
/ 50. (c) The given circuit can be redrawn as follows:
5f2 5Q B sn
= R + 7R _15R 15^ vv\—vw wv
eq
8 " 8 " 8 2/3 V 2/3 V 2/3 V
8
= -A 1
15/8 5
2V
46. (c) The equivalent circuits are as shown below:
A AW + A/W—\W—'c
5a D 5a sa
r 4a 2.4 a
AW
V=2V
I 6a
>—VW t ' 'I As the p.d. divides equally among the equal series
resistors, so

‘'^B = |x2V=iv
4v 1.6 a 4V 1.6 n
^^sA/V -H^W 3

4V 51. (b) The given circuit can be redrawn as follows:


Current, i= = 1 A
(2.4 +1.6) Q 2 V
2a

Now f X 4 =6X 7 or r=-/ AW y <A>


2 2a

But /=/+/' X' AW


2a
3 5
or 1=7+£/=£/ AW
2 2 2a
AW
or 7 = -A = 0.4 A
5
2
7? = -n
47. (d) As the circuit is open, no current flows xy-3
through it.
P.D. between X and Y =emf of the battery =120 V. R =2+-=-n
3 3
48. (fl) Equivalent resistance of 3H and 6f2 Current through the ammeter,
connected in parallel is 2V 3
3x6
R' = =2Q ?A 4
3+6 3
3.169

6x18
52. (fl) R AB =4.5n 56. The equivalent circuit is shown below :
~
6 + 18
2Q 2Q 2n
=2+4.5+8+0.5 = 15^ MV MV MV
eq
15V o- ●o
1 = = 1 A. A B
15Q MV MV MV
6Q 6Q 6Q
53. (fl)
2x6
AO AO R x3=4-5a
AB ~
2+6
20 20 lO
57. (b) In the steady state, the capacitor is fully
charged and no current flows through the branch
20 20 containing the capacitor

w
= -6 V
.-. 6+2x0+V^=0
20 lO
20
g = CV=lpFx(-6V) = ~6pC
Negative sign shows that the left plate of the

Flo
60 60

R
2x2
= 1Q
capacitor is negatively charged.
AB ~
2+2

ee
58. {d) 1 i=l?- = 24 A.
54. (c) Let X be the equivalent resistance between A max
r 0.5

Fr
and B. The network consists of infinite ur\its of two
6
resistors of 1Q, 1 fl. The addition of one more such unit 59. {b) J =
R +r
across AB will not affect total resistance. The network
for
ur
10
will then appear as shown below : or 0.5 =
R+3
P ^^ A
—V 1
ks
10
or K +3 = = 20
Yo

0.5
oo

in X
or K = 17fl.
eB

o-
& 1.5
60. (c) 1 = = 0.1 A.
Q 6
R +r (5 + 8) + 2
r

IxX P.D. across 5 Q resistor =0.1x 5 = 0.5 V.


ou
ad

Rn^=l + = K AB
PQ 1 +X
61. (f») As V = £-/r
Y

X 20= g-0.2r or 200=10£-2r


or 1 + = X In first case :
1 +X
In second case : 16 =S-2r
nd
Re

X^-X-1 = 0 S = 20.4 V.
or
On subtracting, 184 =9E or
Fi

l+Vl + 4 l + Vs
or X =
2 2
Q. 62. (c) Vp-Vq =e+ /(r+ K) =2 +3(2 + 4) = 20 V.
8-V {3.6-3
[●.■ X cannot be -ve] 63. (fl)r = R = x5=in.
V 3 J
55. (b)
5Q 64. id) In first case. 6=0.9 (2 + r)
AO—VW
In second case, 6 = 0.3 (7 + r)
On solving, r = 0.5 Cl.

Wv—OD
65. (fl)When 7=0,6 = V = 5.6 V
5fl
When 7 = 2 A, V=0
20x20
6-V 5.6-0
^ad = 5 + 20 + 20 + 5 = 20 n.
r = = 2.8Q.
7 2
^!k 3.170 PHYSICS-Xll

66. (f7) For parallel combination, 3g


73. (^/) 7 =
net emf = emf of a single cell = 6 V. K + Tj + r2
67. (c) The similar accumulators are connected in Terminal P.D. across first cell,
opposition to each other.
V=2l-lr^ =0
v,-v^=o.
68. (iJ) For maximum current, cells should be 26 = 7r^
connected in series. 36
26 = X K
6+6 6
69. (c) For a battery of two cells, I = R + r^ + V2 ^
r+r r
_ 3r.
Similarly, for a battery of n cells, ^ 'l ^2 ” “
2

7 = ^^6
nr r R = -i-
2
Clearly, the current in the circuit does not depend 1
on the number (n) of the cells. Hence the correct graph 74. (b) For constant V, H oc —
R
is (c).
Net emf 2+2-2 Vh 200x200x2x60x60
70. (rf) 7 = = ^A. 75.(c) H =
7? + 3r 4.5+3x0.5 6 R 80
71.(a) 200x200x2x60x60
Wh = 1000 Wh.
6 6 80x3600
r r

I—wv I—Wyr
76. ib) Power, P = I^R
1 AP M AR
xl00=2 —xl00 + xlOO
2C1 P I R
AAAr
R
=2x1%+0 = 2%.
26 77. (b)
i
P 500 500 W R Rs
2r + 2 = 5A
v~m
6 r 100 V 100 V
I—vw 100 V
' r/2 5A
S r
-1—WSr
= 20D. 200 V
I—vw
^2"
2n 2n
78. (c) R oc — =
/ /^p
AW AAAr
A Alp w

6 26 For same m, Rccl^


h~
-^2 r+4
Poclocl
2
R f
^1 = ^2
2r+2 = r+4
p.B 1
r=2D.

72. (c) Current in the circuit is P^ =4x5=20 W.


/ / 21 _R
7 =
Net emf 6,-6, 79. (c) R=p-V=p and 7^=p
Net resistance R + r, + r,
A Tir^ {2rf~2
8-4 vh V^t
~=-A. H = and H' = [V = constant]
9 +1 +2 12 3 R R'
1 H' _ R _ R =2 ^ H'=2H.
y, = P.D. across R=-x9 = 3V.
PQ 3 H R’ R/2
-'-7

3All\

80. (c) 85. (/j) From the above question, we have R-kt
I = 2A ^2 = 3 0 % For a parallel combination,

Pg = 6W 1 = J_ J- 1=1 1
'7 t h1

I tJ
t =
1-2 _ 10x15 _150 = 6 min.

P3 = V,I = I% fj +12 10 + 15 25

6=2^x R3 86. (c) H = cal


JR
= 1.5 a
^ 2
= constant
R
V3 = /R3=2x1.5=3V.

w
R = kt and R oc / /oc/

81. (rf)
R
400 _ Ro(l+4Q0a) ^l + 5.6xl0~^ _L056 90
~Ro(l + 1000a)"l + 1.4xlO
-1
R 1.14 X16 min = 14.4 min.
^2 “

Flo
1000
100
P. R 1.14
400 _ 1000 _
87. (c) Domestic electrical wiring has three wires -

ee
R R 1.056
1000 400 live (red), neutral (black) and earth (green).

Fr
1.14x1000 88. (fl) KirchhofPs first law is based on the law of
or
^400 “ = 1.08xl0^W.
1.056 conservation of charge.
89. (^7)
82. (c)Input power = W =200x 10 =2000W
for
ur
I’l +12 = 6 A
Output power = 40% of 2000 W = 800 W
For parallel currents along PQR and PR,
Power loss in heating the armature = 1200 W
ks
1200 '1^1 “
Yo

1^R = 1200 .-. R = = 12 a.


j'l X 4 = I2 2
oo

10x10

83. (a)Pj=P2 2'i ='2


eB

u1 +2L1 = 6
\2
& €
R = 2 A.
1
^R2 + rJ
r

Rj + r
ou

90. (c)
ad

^1
Y

1 kn
{R,+rf {R^ + rf ov 4
2V
0

2V
On solving, r = ^R^R^.
nd
Re

4V 1 kn
ov
84. (c) Heat produced in time / in a resistor R,
Fi

4V
vh 6V Ikn
H = cal ov
JR V.0
6V

or l=i£
R V
Using Kirchhoff s junction rule,
= constant for boiling a certain
2-V.0 4-y„0 6-V,0
amount of water = 0
1 1 1
or R=kt

For two resistance coils R^ and Rj connected in series 12-3Vo = 0


K0 = 4 V.
R = Rj + R2
kt = cf/j + kt2 91. (fl) Applying junction rule at B,
or
/ = /i + /2 = 15 + 20 = 35 min. I
BC
=2+2=4A
3.172 PHYSICS-XII

92. (c) For similar loops, = I2 100. (^) r = k-^2] K = r 50-40^ x2Q =0.5n
Applying second law to upper loop, . h , 1, 40

10{/^ + /2) + 2/i=6 101. (fi)


I
10x2/^+2/i=6 > B

^1 = 22

Current through lOQ resistance, -1 VA-


20 n
/j + /2=2x 22
A 11 R
6V

= 10x0.1xl00=100n
AB

93. (h) Applying Kirchhoff's loop rule to the upper 6 1


I = —A
mesh. AB ~
20 + 100 20
40 J3+30/1 =40 Maximum emf measured.
or
473+37^=4 ●●●(0
^AB ~ ^AB^AB ~ — Xl00=5 V.
From the lower mesh, 20

4073 + 4072 = 40+80 102. (rt) P.D. across potentiometer wire of length
100 cm.
or
73 + (73-7i) = 3
or
273-7^=3 -Hi) V^g = W =0.2 mVcm ^x 100cm =20 mV
V 20xl0'^V
On solving (0 and (21)/ 7^=-0.4A I
AB “
AB _
R R
AB AB
94.(fl) Potentiometer is best suited for the measure
ment of emf of a cell. € 2V
Also, 7
AB ~

?2 ^AB ^ + ^
95.(c)
61 I
1
2 20x10'^ 1

2.5 V 7?ab + 490 R AB 50 RAB


or
1V 600 cm
or I2 =2.5x600 = 1500 cm.
or

54 490
96.(22)
27
.-. €2=2^1 =2x1.5=3V. or R
AB ~
-4.9 a
99
10
97.(b) 7 = 103. (c) When the resistance R in the main circuit is
3 +3 3 increased, the current in the auxiliary circuit decreases.
-x3=5V This decreases the potential gradient. Hence the
ab"3 potential drop which earlier existed in 7th line can now
be obtained on 9th line.
5V 5xl0^mV
k = = 10 mV cm
500 cm 500 cm 104.(27) As the Wheatstone bridge is balanced, the
pressing of key K makes no effect. The reading of the
98.(27) Current through the potentiometer wire, galvanometer G remains the same.
6 5
7 = — = 0.01 A 30 Q _ 30 g
R + R' 480 + 20 500 105.(27) As
3^”3^
P.D. across this wire of length 10 m = IR'
= 0.01 X 20 = 0.2 V The Wheatstone bridge is balanced. No current
flows in arm BD.
V 0.2
= 0.02 Vm’^
Potential gradient, ^ = y 10 106. (c) As
2n
4Q
6g
12 0
= P.D. across wire of 6 m length
The Wheatstone bridge is balanced.
= kl = 0.02 X 6 = 0.12 V.
R =2+4=6
ACB
99.{b)
R = 6 + 12 =18
ADB
^1+^2 h _ l^+l2 _8+2 _5- = 5 : 3. 6x18
R = 4.5 0.
8,-62 gj l^-l2 8-2 3 AB ~
6 + 18
107. (c) When S is increased by 20 Q, we have The 8 Q resistance is ineffective.
11 20 8x8
R■AB ~ = 4n.
100 220 8+8

That is the bridge is not balanced. R


^ S=3Q.
no. (b) —= =>

108. (ft)For potential difference between Band Dto Q S 60 ~S


be zero.
15 +
6x6 111. id)
P_^R 6 + 15 6 +6 R _ / 10 /=25cm
In first case:
Q S 8;c
+3 4 +
4x4 S “l00-/~30
8 +x 4 +4
In second case : — r I' = 75 cm
21 18 8a:
=3 ^ +3 = 7 R 100-r 10
8x
+3 6 8 +x
Shift in the balance point = I' -1=75 -25 = 50 cm.

w
8 +x

8a: = 32 + 4a: => 4a: =32 a: = 8 0. 112. (c)


R_ / 20 1
109. (ft) The equivalent circuits are shown below : In ifrst case:

Flo
S 100-/ 100-20 4
A sri an B
o- ■o
Clearly, R<S

ee
R + 15 r 40
sn
In second case:
8Q an.
S ”l00-/' 100-40

Fr
or R 15^2
S "3
for
ur
or 15 _2_R _2_1.
S~3 S~3 4” 12
ks
or ^15x12 =36 0
Yo

5
oo

R = is=lx36 = 90.
eB

4 4
The circuit is a balanced Wheatstone bridge.
r
ou

■ Type B : Case Study Based Questions


ad
Y

PARAGRAPH 1 Electric Resistance


nd

Electrons move more easily through some conductors than others when a potential difference is applied.
Re

The opposition of a conductor to current is called its resistance. Collisions are the basic cause of collision.
Fi

When a p.d. is applied across the ends of a conductor, its free electrons get accelerated. On their way, they
frequently collide with the positive metal ions, i.e., their motion is opposed and this opposition to the flow
of electrons is called resistance. The number of collisions that the electrons make with atoms/ions depends
on the arrangement of atoms or ions in a conductor. So the resistance depends on the nature of the
material of the conductor. A long wire offers more resistance than short wire because there will be more
collisions in the long wire. A thick wire offers less resistance
than a thin wire because in a thick wire, more area of
cross-section is available for the flow of electrons. The resistance
of metals increases when their temperature increases. Certain
alloys such as constantan and manganin show very small
changes of resistance with temperature and are used to make
standard resistors. The resistance of semiconductors and
insulators decreases as their temperature increases.
.3:174 PHYSICS-XII

QUESTIONS (Answer any four of the following questions)


1. The resistance of a conductor is 4. A wire of resistance R is stretched to twice of its
{a) inversely proportional to the length original length. Its new resistance will be
(&) directly proportional to the square of the radius R
(a)4R
(c) inversely proportional to the square of the radius
(d) directly proportional to the square root of the (c)3R w-
R
length
2. The dimensions of a block are 1cm x 1cm x 100cm. If 5. From the graph between current I and voltage V,
the specific resistance of the material is 3 xlO'^Qm, then identify the portion corresponding to the negative
resistance
the resistance between two opposite rectangular bases is
1“
{a)3xlQ-^n {b)3xl0"^n (a) AB
(c)3xl0~^n (rf)3xlO“^Q (b) BC

3. Two wires of the same material have lengths / and 21 (c)CD


and areas of cross-section 4 A and A respectively. The (d) DE
ratio of their specific resistances would be
(a) 1:2 (b) 8 : 1
(c) 1 : 8 (rf) 1 : 1

PARAGRAPH 2 Internal Resistance of a Cell

Any source of emf has some internal resistance itself, called internal or source resistance. When we connect
the terminals of a cell, a current flows in the wire from positive terminal of the cell towards the negative
terminal. But inside the electrolyte of the cell, the positive ions flow from the lower to the higher potential
(or negative ions from the higher to the lower potential) against the background of other ions and neutral
atoms of the electrolyte. So the electrolyte offers some resistance to the flow of current inside the cell.
The resistance offered by the electrolyte to the flow of current between its electrodes is called the internal
resistance of the cell. It depends on nature and concentration of electrolyte, separation and common area
of the electrodes dipped in the electrolyte, and temperature of the electrolyte. Internal resistance causes
energy loss which occurs inside a battery when a current is driven round an external circuit. The greater the
cunent, the greater the energy loss and the small is the terminal p.d. of the battery. The internal resistance
of a battery increases with age and so reduces the current it can drive. ©
The knowledge of internal resistance becomes important when we e
consider how a source of emf can deliver maximum power to an r

appliance connected to it. According to maximum power theorem this


occurs when the internal resistance of the source equals the resistance
of the appliance. R

s
QUESTIONS (Answer any four of the following questions)
6. The internal resistance of a cell
7. A battery of emf 10 V and internal resistance 3f2 is
(a) always acts in the cell in open circuit connected to a resistor. The current in the circuit is 0.5 A.
(b) acts only in dosed circuit and it reduces the EMF The terminal voltage of the battery when the drcuit is
(c) acts ordy in closed circuit and it reduces the closed is
current
(a) 10 V (fc) zero
(d) none of these. (c) 1.5 V id) 8.5 V
3.175 ...

8. A cell of emf 6 and internal resistance r is connected 9. The maximum power drawn out of the cell from a
across an external resistance il The graph showing the source is given by
variation of P.D. across R versus R is

(«) {b) {a) — ib)-r


V . 2r 4r

6 e
e
(c) id) —
r 3r

where r is internal resistance of the cell.


O R

ic) (^0 10. A battery of 16 V and internal resistance ^ is


connected to an external resistance R. Find the value of
V.
e current so that power in circuit is maximum.
(a) 8 A {h)2A

w
(c) 16 A (d)4 A

0 R

PARAGRAPH 3 Heating Effect of Current

Flo
ee
Consider a resistor connected to a source of emf. The energy of the source gets dissipated entirely in the

Fr
form of heat. This phenomenon of the production of heat in a resistor by the flow of current through it is
called heating effect of current or Joule heating. When a potential difference is applied across the ends of a
conductor, its free electrons get accelerated in the opposite direction of the applied field. for
ur
The speed of the electrons does not increase beyond a constant drift speed. This
is because during the course of their motion electrons collide more frequently
ks

with the positive metal ions. The kinetic energy gained by the free electrons
Yo
oo

during the interval of free acceleration is transferred to the metal ions at the
time of collision. The ions vibrate more vigourly i.e., the conductor gets heated.
eB

\
The energy supplied by the source of emf appears as heat. According to Joule's A

1
law of heating, the amount heat produced is ff =I^Rt joule. Electric heater,
r

electric iron, electric bulb, electric stove, etc.; are the devices which work on
ou
ad

this law and convert electric energy into heat energy.


Y

QUESTIONS (Answer any four of the following questions)


nd
Re

11. If two identical heaters each rated as (1000 W, 220 (a) 40 W (b)fw
V) are connected in parallel to 220 V, then the total power
Fi

(c) 90W (d) lOW


consumed is
14. 3 identical bulbs are connected in series and these
(fl)200W (b)2500W (c) 250 W (d) 2000 W
together dissipate a power P. If now the bulbs are
P
12. Two wires having / corvnected in parallel, then the power dissipated will be
resistances R and 2R are (a)- (b) 3P (c)9P (d)-
3 ^ ' 9
connected in parallel, then ratio ^ 2R R
of heat generated in Rand2Ris 15.Bulb B^{100W-250V)andbuIb ^{100W-200V)are
(a) 3: 2 (fc) 2 : 1 connected across 250 V. What is potential drop across ?
(c) 1 : 4 (d)4:l (a) 200 V

13. When 4 equal resistors are connected in series with (b) 250 V B, B2
a battery, they dissipate a power of 10 W. What will be the (c) 98 V
power dissipated through any of them if it is individually (d) 48 V 1

connected across the same battery ? 250V


3.176 PHYSICS-XII

Answers
i I 1
l.(c) ^=P-7=P^
A %r^
=> Roc — 12. (d) For wires connected in parallel, potential
difference V is constant.
2. (p) The resistance between opposite rectangular H =
vh
faces is i.e., Hcc-
R R
h 3.7xl0"^xl0'^
R^
^Ixh o. =3.7xl0“^n. ■ _{2Rf 4
1x10"^
H2 1 4.1
3. (d) The specific resistance does not depend on I
13. (a) For equal resistors, = I’
and A, it depends on nature of the material only.
. , R' I' A 21 A
For a series combination, —= —+ —4- —+ -L
R I A' I A/2 F P2 P3 P,
5. (c) In the portion CD, the current I decreases with 1 = J- J_ J_ J_-_-
or or P = 40 W.
the increase in voltage V. This portion corresponds to 10 P
1
n 1

negative resistance.
14. (c) When the bulbs are connected in series,
6. (c) Internal resistance of a cell acts only in a P-Yl-Yl
closed circuit and it reduces the current.
Rs
7.(rf) V=E-Ir =10-0.5x3 =10-1.5 =8.5 V- When the bulbs are connected in parallel,
€ ^ g
F =
8. (fl) V = 7R = R+r
R =
R/3 ~ R
=3x3P=9P.

As R increases, V also increases.


15. (c) Resistance of first bulb,
When R ^0, V =0 R _ 250x250 =625 0
’ P1 100
When R X, V = g
Resistance of second bulb.
9. (p) Maximum power is drawn when r = R
g g g _ 200x200
/ =
r+R r+r 2r
^ ^2P 100
= 400 0

\2
P_... = fR= —
In a series circuit, potential divides in proportion to
resistance. Potential drop across bulb is
xr =
max
2r 4r

P' _= ^ xV =
400
x250V
10* (d) For maximum power, r = R R,+ R^ 625 + 400
1 = g _ g 16 = 4 A. =97-56V*98 V.
R +r 2r ~2x2
11. (d) For a parallel combination,
P = Pj + P^ = 1000 +1000 = 2000 W-

■^YPE C : Assertions and Reasons


DIRECTIONS

In the following questions (1-11), a statement of Assertion (A) is followed by a statement of Reason [R].
Mark the correct choice as :

(a) If both assertion and reason are true and reason is the correct explanation of the assertion.
(6) If both assertion and reason are true but reason is not the correct explanation of the assertion,
(c) If assertion is true but reason is false.
(d) If both assertion and reason are false/ass.ertion is false and reason is true.
3,177

1. Assertion. The dimensional formula for product of maintained at a higher temperature. The null point can be
resistance and conductance is same as for dielectric constant. obtained at the same point as before by decreasing the
value of standard resistance.
Reason. Both have dimensions of time constant.
[AIIMS 2009] Reason. Resistance of metal increases with increase in
temperature. [AIIMS 15)
2. Assertion. Electrons move from a region of higher
potential to a region of lower potential. 8. Assertion. The effective
Reason. An electron has less potential energy at a resistance of network between

point where potential is higher and vice-versa. (AIIMS 13) Rand Qis |r.
o p -wv Wv-
3. Assertion. A wire carrying an electric current has no
electric field around it. Reason. Symmetry can be
Reason. Rate of flow of electrons in one direction is
applied to the network with
respect to centre. [AIIMS 15)
equal to the rate of flow of protons in opposite direction.

w
4. Assertion. Though large number of free electrons 9. Assertion. The conductivity of an electrolyte is very
are present in the metal. Yet there is no current in the low as compared to a metal at room temperature.
absence of electric field.
Reason. The number density of free ions in electrolyte

Flo
Reason. In the absence of electric field electrons move is much smaller as compared to number density of free
randomly in all directions. [AIIMS 11] electrons in metals. Further, ions drift much more slowly,

ee
5. Assertion. The value of temperature coefficient of being heavier. [AIIMS 15]
resistance is positive for metals. 10. Assertion. An electrical bulb starts glowing

Fr
Reason. The temperature coefficient of resistance for instantly as it is switched on.
insulator is also positive. [AIIMS 11]
Reason. Drift speed of electrons in a metallic wire is
6. Assertion. A thermoelectric refrigerator is based for
ur
on very large. [AIIMS 17]
the Peltier effect.
11. Assertion. A current flows in a conductor only
Reason. A thermocouple may be used as a radiation when there is an electric field within the conductor.
ks
detector. [AIIMS 12]
Reason. The drift velocity of electrons in presence of
Yo

7. Assertion. In a metre bridge experiment, null point


oo

electric field decreases. [AIIMS 18]


for an unknown resistance is put inside an enclosure
eB

Answers ▼

1. (c) Resistance x conductance = R x -i = [M^’l'^T®] 8. (d) Considering the symmetry about the centre, the
r

circuit can be redrawn as below :


ou

Dielectric constant k is also dimensionless.


ad

But time constant has the dimension of time. 8r/3


Y

2. (fl) Both assertion and reason are true and reason is A/W
2r
the correct explanation of the assertion.
nd

oQ
Re

3. (c) As many electrons enter one end of the wire as


they go out from the other end. There is no net charge on WV
Fi

the wire and so there is no electric field around it.


Assertion is true but the reason is not true.
8
R
4. (a) Both assertion and reason are true and the reason r 8r pQ"15'-
is the correct explanation of the assertion.
5. (c) Assertion is true but reason is false, a is +ve for 9. (fl) Both assertion and reason are true and reason is
metals but -ve for insulators. the correct explanation of the assertion.
6. ((;) Both assertion and reason are true but the reason 10. (c) Assertion is true but the reason is false because
is not a correct explanation of the assertion. the drift speed of electrons is very small.
7. (d) The unknown resistance (X) increases with the 11. (c) As soon as the electric field is set up within the
R I conductor, electrons start drifting in its opposite direction
mcrease in temperature. — =
^ X 100-/ but the electrons begin to gain drift velocity only in the
The same null point can be obtained by increasing the presence of external electric field. So the assertion is true
and the reason is false.
standard resistance (R).
.178 PHYSICS-XII

Text Based Exercises


■^YPE A : Very Short Answer Questions (i mark each)
I Define electric current. What is the SI unit of electric 26. Write the expression for the drift velocity of charge
current ?
carriers in a conductor of length across which a
2. Write the relation between a coulomb and an potential difference ‘V is applied. [CBSE OD 14C[
ampere. [ISCE 96]
27. How does one explain increase in resistivity of a
3. What does the direction of electric current signify in metal with increase of temperature ?
an electric circuit ?
[CBSE OD 14C]
4. What is electromotive force ? State its SI Unit. 28. Define the term mobility of charge carriers in a
[Punjab 2000] conductor. Write its SI unit. [CBSE D 14 ; OD 15]
5. State the condition in which terminal voltage across 29. Plot a graph showing variation of resistivity of a
a secondary cell is equal to its emf ? [CBSE D 2000] conductor (copper) with temperature.
6. Define an emf of one volt. [CBSE D 14 ; F 15)
7. State Ohm's law. [ISCE 95] 30. Plot a graph showing variation of current versus
8. Name the colours corresponding to the digits 4 and voltage for the material GaAs. [CBSE D 14]
7 in the colour code scheme for carbon resistors. 31.
Sketch a graph showing variation of resistivity of
[CBSE SP 15] carbon with temperature. (CBSE D 06[
9. Define resistance and state its SI unit. [CBSE D 92C] Or
10. Define Ohm.
Show on a graph, the variation of resistivity with
11 Define conductance of a material. Give its SI unit. temperature for a typical semiconductor Si.
(CBSE D 02] [CBSE D 12, 14 ; OD 19]
12. Define electrical conductivity of a material. Give its 32.
Name two materials whose resistivity decreases
SI unit. [CBSE D 03, 14 ; OD 17C] with the rise of temperature.
13. How much is the resistance of an air-gap ? 33,
How does the conductance of a semi-conducting
14. How much is the resistance of a closed plug-key ? material change with irse in temperature ?
15, Which metal has the lowest resistivity ? 34.
Of copper and nichrome, which one has possibly
16. Define resistivity of a material. State its SI unit. larger value of temperature coefficient of
resistance ? [CBSE D 95C]
[ISCE 93]
35.
17. What is the order of resistivity of an insulator ? Write two properties of manganin which make it a
suitable material for making standard resistors.
18. What is the ratio of the resistivity of a typical
insulator to that of a metal ? [CBSE 20C]
36.
19. What is the average velocity of free electrons in a How does the resistance of an insulator change
with temperature ?
metal at room temperature ?
37. Name two parameters which determine the
20. Give the order of magnitude of the number density resistivity of a material.
of free electrons in a metal.
38.
How is the conductivity of an electrolyte affec
21. Give the order of magnitude of thermal velocity ted by the increase of temperature ? [CBSE D 95]
and drift velocity of free electrons in a conductor
39.
carrying current at room temperature. If potential difference V applied across a conductor
22. What is the order of resistivity of conductor ? is increased to 2 V, how will the drift velocity of the
electrons change ? [CBSE OD 2000C]
23. Define temperature coefficient of resistivity. 40.
24. How does the random motion of free electrons in a
What is a non-ohmic device ? State one example.
[Punjab 02]
conductor get affected when a potential difference 41.
is applied across its ends ? [CBSE D 14C]
Give an example of non-ohmic device which shows
up negative resistance.
25. Define the term 'drift velocity' of charge carriers in 42. A cell of emf and internal resistance W draws a
a conductor and write its relationship with the current Write the relation between terminal
current flowing through it. [CBSE D 14] voltage 'V' in terms of S, I and r. [CBSE OD 13]
3.179.

43. Two identical cells, each of emf 6, having negligible 63. The applied p.d. across a given resistance is altered
internal resistance r, are connected in parallel with so that heat produced per second increases by a
each other aaoss an external resistance R. What is the factor of 9. By what factor does the applied p.d.
(CBSE OD 13] change ? [CBSE OD 17]
current through this resistance ?
44. A cell of emf '£' and internal resistance 'r' is 64. Two electric bulbs are rated at 220 V - 100 W and
connected across a variable external resistor 'R'. 220V-60W. Which one of these has greater
Plot the variation of voltage drop 'V' across the resistance and why ? (CBSE SP 03]
resistor as a function of 'R'. [CBSE 20C] 65. The maximum power dissipated in a 10,000 Q
45. A 4fl non-insulated resistance wire is bent 180"^ in resistor is 1W. What is the maximum current ?
the middle and the two halves are twisted together. IISCE 93]
What will be its new resistance ? [CBSE D IOC]
66. What is the safest voltage you can safely put across
46. Can Kirchhoff's laws be applied to both d.c. and a.c. a 98 n, 0.5 W resistor ? [ISCE 97]
circuits ?
67. How much charge flows through a 250 V, 1,000 W

w
47. On what conservation principle is the Kirchhoff's heater in one minute ? [ISCE 96]
first law based ?
68. A heating element is marked 210 V, 630 W. What is
48. On what conservation principle is the Kirchhoff's the value of the current drawn by the element when

Flo
second law based ?
connected to a 210 V dc source ? [CBSE D 13]
49. Name the device used for measuring the emf of a 69. A heating element is marked 210 V, 630 W. Find the

ee
cell. [CBSE D 96]
resistance of the element when connected to a 210 V
Name the device used for measuring the internal

Fr
50.
dc source. [CBSE D 13]
resistance of a secondary cell. [CBSE D 96]
70. Two resistors of 2Q and 40 are connected in
51. Define potential gradient. Give its SI unit. parallel to a constant d.c. voltage. In which case
for
ur
52. What is a Wheatstone bridge ? [CBSE D 03] more heat is produced ? [CBSE D 98C ]
53. Write the principle of working of a metre bridge. 71. Two bulbs whose resistances are in the ratio 1:2 are
[CBSE OD 17] connected in parallel to a source of constant
ks
54. Give the sequence of colour bands found on a 47 kD voltage. What will be the ratio of power dissipation
Yo

+10% carbon resistor. [CBSE 20C] in these bulbs ? [CBSE D 2000C]


oo

55. A carbon resistor is shown in Fig. 3.326 Using 72. Distinguish between kilowatt and kilowatt hour.
eB

colour code, write the value of the resistance. 73. Which has a greater resistance-kW electric heater
Violet Red [CBSE OD 19] or a 100 W filament bulb both marked for 220 V ?

M
Green [CBSE D OIC]
r

A 74. The coil of a heater is cut into two equal halves and
ou
ad

only one of them is used into heater. What is the


ratio of the heat produced by this half coil to that by
Y

Fig. 3.326
original coil ?
56. If the resistances in the three successive arms of a
75. What do you mean by the maximum power rating
nd
Re

balanced Wheatstone bridge are 1, 2 and 36 D of a resistor ?


respectively, what will be the resistance of the
Fi

galvanometer placed in the fourth arm ?


76. Express power transferred per unit volume into
57. Current I flows through a potential drop V across a joule heat in a resistor in terms of current density.
conductor. What is the rate of production of heat ? 77. Write two special characteristics of the wire of an
[CBSE D 93C] electric heater. [CBSE D 94]
78. What are the characteristics of a fuse wire ?
58. The rate of production of heat is given by P = VI.
Is this relation valid for a non-ohmic conductor ? 79. What is the difference between a heater wire and a
fuse wire ?
59. Of which physical quantity is the unit kilowatt
hour ? 80. Two identical heaters rated 220 V, 1000 W are
60. What do you mean by 1 unit of electric energy in placed in series with each other across a 220 V line.
domestic use ? What is their combined power ?
61. How many joules of energy are equivalent to 1 kWh ? 81. Write an expression for the resistivity of a metallic
62. How man)' kilowatt hours (kWh) are there in one conductor showing its variation over a limited
joule ? [CBSE OD 99C] range of temperature. [CBSE D 08C]
3.180 PHYSICS-Xll

82. The plot of the variation 87. A parallel combination of two cells of emf's and
of potential difference ^2/ and internal resistances, ^ and r2, is used to
across a combination of supply current to a load of resistance K Write the
three identic?! cells in expression for the current through the load in
series, versus current is terms of €,,82, and r2-
shown in Fig. 3.327. I
ICBSE SPll)
What is the emf of each 88. Under what condition can we draw maximum
cell? current from a secondary cell ? ICBSE F 10]
(CBSE D 08 ; OD 16] 89. Write any two factors on which the internal
83. The/-V characteristics of resistance of a cell depends. ICBSE OD 10]
a resistor are observed t
90. Write two factors on which the sensitivity of a
to deviate from a
potentiometer depends. ICBSE D 13C]
straight line for higher 91. Graph showing the variation of current versus
values of current as
voltage for a material GaAs is shown in Fig, 3.330.
shown in Fig. 3.328. Identify the region of (j) negative resistance
Why? ICBSE SP 08] Fig. 3.328 (n) where Ohm's law is obeyed. ICBSE D 15]
84. A (O series (u) parallel combination of two given
resistors is connected one-by-one, across a cell. In t
which case will the terminal potential difference,
across the cell, have a higher value ? ICBSE OD 08C]
c
01
b>

85. Two identical slabs of given metal are joined together, 3


u

in two different ways, as shown in Figs. 3.329(i) A

Voltage V —»

Fig. 3.330

92. Two electric bulbs P and Q have their resistances


(//)
in the ratio of 1 : 2. They are connected in series
.329
across a battery. Find the ratio of the power
and (ii). What is the ratio of the resistances of these dissipation in these bulbs. (CBSE 18]
two combinations ? (CBSE D IOC] 93. Define mobility of a charge carrier. What is its
relation with relaxation time ?
86. A resistance R is connected across a cell, of emf S
and internal resistance n A potentiometer now 94. How does the mobility of electrons in a conductor
measures the p.d., between the terminals of the change, if the potential difference applied across the
cell, as V. State the expression for V in terms of 6, V conductor is doubled, keeping the length and tempe
and R. ICBSE D 11] rature of the conductor constant ? ICBSE D 19]

Answers

1. The electric current is defined as the rate of flow called electromotive force or emf of the source. SI
of electric charge through any section of a unit of emf is volt.
conductor.
5. When no current is drawn from the cell, its terminal
Electric current = Total charge flowing or /=5 voltage is equal to its emf.
Time taken f
6. If an electric cell supplies an energy of 1 joule for the
The SI unit of electric current is ampere (A). flow of 1 coulomb of charge through the whole circuit
1 coulomb (including the cell), then its emf is said to be 1 volt.
2. 1 ampere =
1 second 7. Ohm's law states that the electric current I passing
3. The direction of conventional current in an electric through a conductor is proportional to the potential
difference V applied across its ends, provided the
circuit tells the direction of flow of positive charges
in that circuit.
temperature aiid other physical conditions remain
unchanged, i.e., V oz I ot V = Rl, where R is called
4. The work done per unit charge by a source in resistance of the conductor.
taking the charge once round the complete circuit is
8. Yellow and violet respectively.
3,181

9. Resistance of a conductor is the property by virtue 27. With the increase in temperature, the relaxation time
of which it opposes the flow of current through it. It ... I m ].
T decreases and hence resistivity p - —5- mcreases.
is equal to the ratio of the potential difference ne X

applied across the conductor to the current flowing


through it. SI unit of resistance is ohm{Q).
28. Refer to point 21 of Glimpses.
V 29. See Fig. 3.21(fl).
R = —
I
30. See Fig. 3.27(b).
10. The resistance of a conductor is said to be 1 ohm if
31. See Fig. 3.22 on page 3.25.
1 ampere of current flows through it on applying a
32. Germanium and silicon.
potential difference of 1 volt across its ends
1 ohm =
1 volt
or ia =
IV 33. With the rise in temperature, the conductance of a
1 ampere lA semi-conducting material increases exponentially.
11. The ease with which a conductor allows a current to 34. Copper,

w
flow through it is called its conductance. It is equal 35. (i) Low temperature coefficient of resistivity.
to the reciprocal of resistance. (ii) High resistivity.
1
Conductance (G) = 36. The resistance of an insulator decreases with the

Flo
Resistance (R) increase of temperature.
-I
SI unit of conductance is ohm or mho. 37. The resistivity of a material depends on (0 its number

ee
12. The conductivity of a material is equal to the density of free electrons, {ii) the relaxation time.

Fr
reciprocal of its resistivity. 38. The conductivity of an electrolyte increases with
1
the increase in its temperature.
Conductivity (a) =
Resistivity (p) eEx eVx
39. Drift velocity,
for =
ur
SI unit of conductivity is ohm"’m
-1
or mho m ^ m ml

13. Infinity. Clearly, when V is increased to 2V, drift velocity


ks
14. Negligibly small. also gets doubled.
Yo

15. Silver. 40. A device which does not obey Ohm's law is called a
oo

non-ohmic device. Semiconductor diodes, ther


16. The resistivity of a material is the resistance offered mistors, etc. are non-ohmic devices.
eB

by a unit cube of that material. Its SI unit is Dm.


41. Thyristor.
17. The resistivity of an insulator like glass or rubber is 42. V=6-/r
of the order of 10® - Qm.
r

18. The ratio of 10^. 43. Effective emf of the parallel combination
ou
ad

19. Zero.
= emf of any one cell = S
Total emf €
Y

20. Number density of free electrons in a metal 1 =


Total resistance R
= 10'^m'3.
nd

44. Sec Fig. 3.91(b) on page 3.46.


Re

21. Thermal velocity of free electrons = lO^ms


Drift velocity of free electrons = 1 mm s“^ 45. The length of the wire becomes half of the original
Fi

22. Conductors have resistivities less than 10“^Qm. length while area of cross-section is doubled.

23. The temperature coefficient of resistivity is defined


R' ^V! A' _V _A^IJ2.
R~ U A l' A' I ' 2A 4
as the change in resistivity per unit resistivity per
degree rise in temperature. Mathematically, or R' = - R = -x4 = lQ.
4 4
P-Po _ 1 frp
a - 46. Yes, Kirchhoff's laws can be applied to both d.c.
and a.c. circuits.
The unit of a is C°"^ or K"’. 47. Kirchhoff's first law is based on the law of conser

24. Random motion gets partially directed towards the vation of charge.
48. Kirchhoff's second law is based on the law of
higher potential side.
conservation of energy.
25. Refer to point 17 of Glimpses. 49. Potentiometer.
eVx
26. v^ = ml
50. Potentiometer.
3.182 PHYSICS-XII

' 51. The potential drop per unit length of the poten¬ 68. / =— P ^ 630 = 3 A.
tiometer wire is known as its potential gradient. Its V 210
SI unit is volt per metre (Vm"^). 210x210
69. R= = 700.
52. A Wheatstone bridge is an arrangement of four P 630
resistances used to determine quickly and accu- V^t
rately one of these resistances in terms of other 70. Heat produced, H = i.e., Hoci
R R
three resistances.
Thus heat produced in 2 O resistor is more than that
53. The working of a metre bridge is based on the in 4Q resistor.
principle of Wheatstone bridge. When the bridge is
balanced, — -3.
Q S P2 V^IR^ ^ ^
54. Yellow, violet, orange and silver. 72. Kilowatt is the unit of electric power while kilowatt
hour is the unit of electric energy.
55. R = 57 X 10^0 ± 20%. See Fig. 3.15 on page 3.9. -1
1 kilowatt = 1000 W = 1000 Js
56. HereP=in,Q=2aR = 36Q,S = ?
For a balanced Wheatstone bridge, 1 kilowatt hour = 3.6 x lO'’ J.
220x220
P^R RxQ^36x2 = 72Q.
73. Resistance of heater = = 48.4n
Q~ S P 1
,P1 1000

220x220
57. Rate of production of heat is P=V! Resistance of bulb = 484n
58. Yes, it is valid. Pi 100

59. Kilowatt hour is the unit of electrical energy. Thus the 100 W bulb has a greater resistance.
74. Let original heat produced.
60. 1 unit of electric energy = 1 kWh. This means that
vh
when an appliance of power 1000 watt is operated H,1 =
on mains for 1 hour, it consumes 1 unit of electric R

energy. Resistance of half coil = R / 2


61. 1 kWh = 1 killowatt xl hour Heat produced in half coil,
= 1000 watt X 3600 s = 3.6 x 10^ J. V^t ^2V^t
Hi = = 2:1
1 R/2~ R H1
62. 1J =
3.6 X 10
j kWh = 2.778x10-’' kWh.
75. The maximum power rating of a resistor is the
maximum power that it can dissipate in the form of
63. Heat produced per second, P =
R heat without undergoing melting.
V = yfM 76. P=3r=3.^: Volume, V = Al
A '
As the heat produced per second increases 9 times,
so V ^3sPxR=ZV .●. Power transferred per unit volume

Thus, the applied p.d. must increase 3 times the _ P_/V/A p=


V Al A
original p.d.
64. R =
2 where J is the current density.
—. For a given voltage, R —.
77. A heater wire should have (i) high melting point,
So 60 W bulb has greater resistance than 100 W and (i7) high resistivity.
bulb.
78. A fuse wire must have high resistivity and low
fp. 1 melting point.
65. I max max
— = 0.01 A.
V R M 10,000 100 79. The melting point of a heater wire is very high
while that of a fuse wire is very low.
66. Vmax
80. Total power P dissipated by the series combination
P 1000 w . . is given by
67. Current, I = — = 4 A
V 250 V
1 =1 -L=_A_ 1 1

Charge that flows in 1 minute, P iP 1000 1000 500


t/ = R = 4 X 60 = 240 C. or P=500 W.
3.183

81. The resistivity p at any temperature T is given by 88. When the external resistance in the circuit is zero,

P =Po [1+ o.(T-Tq)] the current drawn from the secondary cell is max.

where pQ is the resistivity at a lower reference I
max

temperature 7^ and a is the temperature coefficient


r

of resistivity. 89. The internal resistance of a cell depends on (i) the


82. Total emf of three cells in series nature of the electrolyte and (ii) concentration of the
= P.D. corresponding to zero current = 6 V electrolyte.
90. The sensitivity of a potentiometer depends on the
The emf of each cell = ^ = 2 V. potential gradient along its wires. This, in turn,
83. At higher values of current, the resistor gets heated depends on (i) length of the potentiometer wire and
up and its resistance increases. The resistor (ii) the value of resistance put in series with the
driver cell.
becomes non-ohmic and hence I-V graph deviates
from the straight line. 91. (0 In the region DE, I decreases with increasing V.

w
84. In the case of series combination of the two given + veAV
= -ve R.
resistors, the the terminal p.d. will have a higher value. -veA/
I
85. For each slab, R=p

Flo
A {0} AB/ BCis the region where Ohm's law is obeyed.
R
21
_/___R 92. Power dissipation, P= 7^R
= p—=2R ; R2=p 2A " 2

ee
1
A As the bulbs are connected in series, so current I is
same in the two bulbs.

Fr
R 2R
1 _
= 4:1.
^ R/2
E-v P, 1% 2 "
for
ur
86. Internal resistance, r= R.
^ ) 93. Refer to point 21 of Glimpses.
6 94. Mobility of an electron,
ks
eq
87. 1 =
Yo

R+r Vz_ R(rj+r2)+ijr2


eq R+
When V is doubled, does not change.
oo

h + h
eB

■ Type B : Short Answer Questions (2 marks each)


1. Distinguish between electromotive force and '■■etween the current density and the drift velocity
r

terminal potential difference of a cell. What are their


ou

)f electrons. [CBSE OD 20]


ad

units ? [CBSE OD HC! 7. Define relaxation time of electrons in a conductor.


Y

2. Explain how the average velocity of free electron Explain how it varies with increase in temperature
in a metal at constant temperature, in an elecrri of a conductor. State the relation between resistivity
field, remains constant even though the electrons
nd

and relaxation time.


Re

[CBSE D 2000]
are being constantly accelerated by this electric field ?
8. A conductor of length is connected to a d.c.
3. Define the electrical resistivity of a material. How
Fi

source of potential 'V". If the length of the


it is related to the electrical conductivity ? Of
conductor is tripled, by stretching it, keeping ‘V
the factors, length, area of cross-section, nature of
constant, explain how do the following factors
material and temperature - which ones control the
vary in the conductor :
resistivity value of conductor ? [CBSE F 98]
(i) Drift speed of electrons, (//) Resistance and
4. Derive the expression for the drift velocity of free
electrons in terms of relaxation time and electric (Hi) Resistivity. [CBSE D 2000]

field applied across a conductor. [CBSE SP 20] 9. Define the term 'mobility' of charge carriers in a

5. Explain the term 'drift velocity' of electrons in a current-carrying conductor. Obtain the relation for
conductor. Hence obtain the expression for the mobility in terms of relaxation time. [CBSE OD 20]
current through a conductor in terms of 'drift 10. Define ionic mobility. Write its relationship with
velocity'. [CBSE OD 13, 13C, 15C ; SP 23) relaxation time. Give its SI unit. How does one
6. Define the term 'drift velocity' of electrons in a understand the temperature dependence of resis
current-carrying conductor. Obtain the relationship tivity of a semiconductor. (CBSE F 10 ; OD 13C]
3.184 PHYSICS-Xll

11. Define the term resistance. Give physical expla (i) In which bar is drift speed of free electrons
nation of the opposition offered by a conductor to greater ?
the flow of current through it. (ii) If the same constant current continues to flow
12. Three resistances R^, and are connected in for a long time, how will the voltage drop
series. Find their equivalent resistance. across A and B be affected ?

13. Three resistances R^, Rj and R^ are connected in Justify each answer. [CBSE SP 18]
parallel. Find the equivalent resistance of the 29. State the two Kirchhoff's rules used in electric
parallel combination. [Punjab 03]
networks. How are these rules justified ?
14. What are superconductors ? Give two applications of [CBSE D 14,17 ; OD 15]
the phenomenon of superconductivity. [CBSE F 03)
30. The diagram below shows a potentiometer set up.
15. What is Meissner effect ? What does it indicate On touching the jockey near to the end X of the
about the magnetic nature of superconductors ? potentiometer wire, the galvanometer pointer
16. What are ohmic and non-ohmic resistors ? Give one deflects to left. On touching the jockey near to end Y
example of each. [Haryana 02) E
17. State the conditions under which Ohm's law is not I— f- I
R
obeyed in a conductor. [CBSE D 92] I }
X‘- -6Y
18. What is internal resistance of a cell ? On what
factors does it depend ?
19. When a battery of emf € and internal resistance r is
Fig. 3.332 pL-O
tr7> 7
connected to a resistance R, a current I flows through of the potentiometer, the galvanometer pointer
it. Derive the relation between E, /, r and K again deflects to left but now by a larger amount.
20. Figure 3.331 shows a cell of emf 6 and internal resis Identify the fault in the circuit and explain, using
tance r, connected to appropriate equations or otherwise, how it leads to
a voltmeter V and a such a one-sided deflection. [CBSE SP 18]
variable resistance R
ET 31. State, with the help of a suitable diagram, the
Deduce the relation 'll' ® r principle on which the working of a metre bridge is
ship among V, E, R based. Under what condition is the error in
and r. How will V
determining the unknown resistance minimized ?
vary when R is (CBSE F 10,13, OD 15C)
reduced. [ISCE 98) Fig. 3.331
32. For the circuit diagram of
21. A cell of emf 'E' and internal resistance V is
a Wheatstone bridge
connected across a variable load resistor R Draw
shown in Fig. 3.333, use
the plots of the terminal voltage V versus (i) R and Kirchhoff's laws to
(ii) the current I. [CBSE D 15] obtain its balance
22. Two identical cells, each of emf E and internal condition. [CBSE D 09C]
resistance r are connected in parallel to an external 33. Following circuit was set
resistance R Find the expression for the total up in a metre bridge 6
current flowing in the circuit. [CBSE F 96]
experiment to determine Fig. 3.333
23. Name any one material having a small value of the value of X of an
temperaturecoefficient of resistance. Write one use unknown resistance.
of this material.
Resistance box X
24. Define the terms electric energy and electric power. D
—Wv
R
Give their units.
] [
25. Derive an expression for the heat produced in a Si
resistor R when voltage drop across it is V. A B

26. Give four reasons why nichrome element is


commonly used in household heating appliances. Fig. 3.334
■o^
27. What is a safety fuse ? Explain its function.
28. Two material bars A and B of equal area of (fl) Write the formula to be used for finding X
cross-section, are connected in series to a DC from the observations.

supply. A is made of usual resistance wire and B of (b) If the resistance R is increased, what will
an }i-type semiconductor. happen to balancing length ? [CBSE SP 18]
3.

Answers
20, Refer answer to Q. 35 on page 3.46.
1. EMF Terminal Voltage
Sr S
(i) It is the potential dif It is the potential dif We get, V =
ference between two ference between two R-i- r l + (r/R)
terminals of the cells terminals when a
when no current is current passes through Clearly, V decreases when R is reduced.
drawn from it. it.
It is an effect.
21. (i) See Fig. 3.91(&) (i7) See Fig. 3.91(c) on page 3.46.
(ii) It is a cause,
6 2€
(ill) The SI unit is volt. The SI unit is volt. 22. / =
rxr 2R+r
R + R+
2. Refer to the solution of Problem 1(b) on page 3.122. r-i- r 2

3. Resistivity of a material is the resistance of a 23- Alloy like manganin has a small value of tempe
conductor of that material having unit length and rature coefficient of resistivity. It is used for making
unit area of cross-section. standard resistances.

w
1
Conductivity = 24. Refer answer to Q.44 and Q. 45. on pages 3.60 & 3.61.
Resistivity 25. Refer answer to Q. 43 on page 3.60.
Resistivity of conductor depends on the nature of 26. Refer answer to Q. 54 (Application 1) on page 3.64.

Flo
its material and its temperature. 27. Refer answer to Q. 54 (Application 1) on page 3.64.
4. Refer answer to Q. 19 on page 3.16 28. (i) For both bars A and B, I = enAv^ is same. But n

ee
5. Refer answer to Q. 19 on page 3.16. is much smaller for B than that for A. So drift

Fr
6. Refer answer to Q. 19 on page 3.16. speed is greater for B («-type semiconductor)
than for A.
7. The average time that elapses between two successive
collisions of an electron in a conductor is called {it) Voltage drop across A will increase because the
for
ur
relaxation time (t). resistance of A increases with the increase in tem
m
perature. Voltage drop across B will decreases
It is related to resistivity p as p = —5—
ne^ T because resistance of B (semiconductor)
ks
decreases with the increase in temperature.
With the increase in temperature, the electrons
Yo

29. Refer to point 49 of Glimpses.


oo

collide more frequently with positive metal ions. So


their relaxation time decreases. 30. The positive terminal of cell 6^ is not connected to
eB

eV / terminal X.
8. Drift speed, v^= — t; Resistance, R=p-
ml Applying Kirchhoff's second law to loop PCJX,
When I is tripled (i) drift-speed becomes 1/3 times
r

the original (ii) resistance becomes 3 times the Clearly, (or deflection) will be minimum when
ou
ad

original resistance (in) resistivity is not affected. / is minimum i.e., when jockey is touched near
9. Refer answer to Q. 24 on page 3.22.
Y

end X. Also, Vg(or deflection) will be maximum


10. Refer to point 21 of Glimpses. Resistivity, p =—^ ● ne T
when I is maximum i.e., when jockey is touched
near end Y.
nd
Re

As temperature increases, average speed of electrons 31. See Fig. 3.216. The working of a metre bridge is
increases. This increases collision frequency and based on the principle of Wheatstone bridge.
Fi

decreases relaxation time t. But n increases more When the bridge is balanced i.e., no current flows
rapidly with temperature. The increase in n more through the galvanometer arm,
than compensates the decrease in r. So p of P R
semiconductors decreases with temperature. Q S
11. Refer answer to Q. 21 on page 3.17. Error in determination of resistance can be
12. Refer answer to Q. 32 on page 3.30. minimised by adjusting the balance point near the
13. Refer answer to Q. 33 on page 3.31. middle of the metre bridge wire.
14. Refer answer to Q. 30 on page 3.30. 32. Refer answer to Q. 62 on page 3.104.
15. Refer answer to Q. 29 on page 3.29. 3"^. {a) For the balanced metre bridge,
16. Refer answer to Q. 27 on page 3.28. ^ = 3.AB _ I
.-. X =
(1Q0-/)R
17. Refer to points 23 and 24 of Glimpses. X R
BC (100-/) /

18. Refer to point 24 of Glimpses. (b) Balancing length will increase on increasing
19. Refer answer to Q. 35 on page 3.46. resistance K
PHYSICS-Xll

YPE C :: Long Answer Questions I (3 marks each)

1. (a) Define the terms drift velocity and relaxation 9. Define conductivity of a conductor and state its SI
time, giving their physical significance. unit. State and explain the variation of conductivity of
{b) A conductor of length L is connected to a dc (a) good conductor (b) ionic conductor with
source of emf 6. If this conductor is replaced temperature. (CBSE D 01, 08]
by another conductor of same material and
10. (fl) Define internal resistance of a cell,
same area of cross-section but of length 5L,
how will the drift velocity change ? (fc) A cell of emf 6 and internal resistance r is
connected across a variable resistor R. Plot the
[CBSE D 11,19C]
2. What is meant by drift velocity of free electrons ?
shape of graphs showing variation of terminal
voltage V with (i) Rand {it) circuit current I.
Derive Ohm's law on the basis of the theory of
electron drift. [CBSE D 20]
[CBSE D 03]
11. A variable resistor R is connected across a cell of
3. Are the paths of electrons straight lines between emf 6 and internal resistance r.
successive collisions (with positive ions of the
metal) in the (/) absence of electric field (ii) presence (fl) Draw the circuit diagram,
of electric field ? Establish a relation between (b) Plot the graph showing variation of potential
drift velocity 'v^' of an electron in a conductor drop across R as function of R
of cross-section 'A', carrying current' /' and concen (c) At what value of R current in circuit will be
tration 'n' of free electrons per unit volume of maximum ? [CBSE SP 21]
conductor. Hence obtain the relation between
current density and drift velocity. [CBSE OD 03]
12. Distinguish between emf (6) and terminal voltage
(V) of a cell having internal resistance 'r'. Draw a
4. {a) Define the term 'conductivity' of a metallic plot showing the variation of terminal voltage (V) vs.
wire. Write its SI unit.
the current (/) drawn from the cell. Using this plot,
(b) Using the concept of free electrons in a how does one determine the emf and the internal
conductor, derive the expression for the resistance of the cell ? [CBSE OD 14,14C ; F 16]
conductivity of a wire in terms of number 13. A cell of emf € and internal resistance r is connected
density and relaxation time. Hence obtain the
across a variable resistance R Plot graphs showing
relation between current density and the the variation of (i) E and R, (ii) terminal p.d. V with
applied electric field E. [CBSE 18] R Predict from the second graph under which V
5. Define relaxation time of the free electrons drifting becomes equal to E. [CBSE D 09]
in a conductor. How is it related to the drift velocity 14. Two cells of emfs 6j and£2/andintemalresistances
of free electrons ? Use this relation to deduce the
and r2 respectively are connected in series. Obtain
expression for the electrical resistivity of the
material. expressions for the equivalent (i) emf and
[CBSE OD 12]
(ii) resistance of the combination. [CBSE 18C]
6. Define the term resistivity of a conductor. Give its
SI uiut. Show that the resistance of a conductor is 15. Two cells of emfs Ej and E2 and internal resistances
given by [CBSE OD 02]
Tj and T2 respectively are connected in parallel as
ml shown in the figure.
R =
ne^ xA ' '■1

where the symbols have their usual meanings.


7. Define resistivity of a conductor. Plot a graph
showing the variation of resistivity with /
4
/
4
temperature for a metallic conductor. How does A B
one explain such a behaviour, using the
mathematical expression of the resistivity of a
material.
[CBSE D 01, 08]
8. Draw a plot showing the variation of resistivity of a
(0 conductor and (if) semiconductor, with the Deduce the expression for the
increase in temperature. (f) equivalent emf of the combination
How does one explain this behaviour in terms of (if) equivalent internal resistance of the combination
number density of charge carriers and the (iff) potential difference between the points A and B.
relaxation time ?
[CBSE D 14C] [CBSE 19C; SP 23]
3,187

16. (a) Draw a graph showing the variation of current I


+
versus voltage in an electrolyte when an 1-^ G
external resistance is also connected.
±
(b) (i) The graph between resistance (R) and
temperature (T) for Hg is shown in Fig. 3.335(a).
A h l2
Explain the behaviour of Hg near 4 K. Battery _i. g
(if) In which region of the graph shown in
Fig. 3.335(b) is the resistance negative and
why ? [CBSE OD 19J
I—®‘
Rh
Jb
^ 0.16
Rg. 3336

w
a
O'

0.08 (a) Give reason for the observation and suggest


how this difficulty can be resolved (b) What is the
t/^ ia)
function of resistance R used in the circuit ?

Flo
0 2 4 6 8
How will the change in its value affect the null
T(K)
point, (c) How can the sensitivity of the

ee
potentiometer be increased ? [CBSE OD 19}

Fr
22. (a) State the two Kirchhoff's rules used in the
<
analysis of electric circuits and explain them,
(b) Derive the equation of the balanced state in a
for
ur
Wheatstone bridge using Kirchhoff s laws.
[CBSESP23J
U
(b) 23. (a) Derive the condition of balance for Wheatstone
ks
bridge,
Yo

(b) Draw the circuit diagram of a metre bridge to


oo

0
Voltage (V)
explain how it is based on Wheatstone bridge.
eB

[CBSE OD 20}
Fig. 3.335
24. Draw a circuit diagram which can be used to deter
17. Two bulbs are rated (ij/V) and (i^/V^)- If they are mine the resistance of a given wire. Explain the
cormected (i) in series and (ii) in parallel across a
r

principle of the experiment and give the formula


ou

supply V, find the power dissipated in the two


ad

used. Why cannot such an arrangement be used for


combinations in terms of and [CBSE D 06]
measuring very low resistances ?
Y

18. Draw the circuit diagram and describe the use of a 25. State the principle of working of Wheatstone
potentiometer for comparing the emfs of the two bridge. With the help of a circuit diagram, explain
nd

primary cells experimentally. Why is potentiometer


Re

how it is used in the determination of the specific


preferred over a voltmeter for measuring emf of a resistance of the material of a wire using metre
Fi

cell ? [CBSE20q [CBSE20C]


bridge.
19. State the principleof a potentiometer.With the help of 26. (a) You are required to select a carbon resistor of
a circuit diagram, describe a method to find the inter resistance 56 kQ + 10% from a shopkeeper.
nal resistance of a primary cell. What would be the sequence of colour bands
[CBSE D 03; OD 13]
required to code the desired resistor,
20. You are required to find the internal resistance of a (b) Write two characteristic properties of the
primary cell in die laboratory. Draw a circuit diagram material of a metre bridge wire,
of the apparatus you will use to determine it. Explain (c) What precautions do you take to minimize the
the principle of the experiment. Give the formula error in finding the unknown resistance of a
used. [CBSE D 08C, 16, 20C]
wire by using metre bridge ? [CBSE 19Q
21. A student uses the circuit diagram of a
potentiometer as shown in Fig. 3.336. For a steady
current / passing through the potentiometer wire
he gets a null point for the cell and not for 62-
Ir. 3.188 PHYSICS-XII

Answers

1 ● (fl) Drift velocity. The average velocity acquired by 10. (a) Refer answer to Q. 34 on page 3.45.
the free electrons of a conductor in the opposite (b) (0 See Fig. 3.91(1)) on page 3.46.
direction of the externally applied electric field is
called drift velocity. (/i) See Fig. 3.91(c) on page 3.46.
11. (fl) The given circuit
Significance. The drift velocity determines the net R

current flowing across any cross-section of the diagram shows a V/v


conductor. variable resistance R _
connected across a
Relaxation time. It is the average time the elapses cell of emf £ and _
between two successive collisions of an electron rhAVv
internal resistance r.
drifting in a conductor.
(b) See Fig. 3.91(b) on page 3.46. Cell
Significance. It is an important factor which (c) Current in the circuit will be
determines the drift velocity acquired by free maximum when R = 0. Fig. 3.337
electrons under a given external electric field. It
also determines the electrical conductivity of a 12. Refer answer to Q. 35 on page 3.46.
conductor at different temperatures. 13. Refer answer to Q. 35 on page 3.46.
...

14. Refer answer to Q. 37 on page 3.51.


ml
15. Refer answer to Q. 38 on page 3.52.
When length is increased to 5Z., drift velocity 16. (a)
becomes 1/5 times the original v^.
2. Refer answer to Q. 19 on page 3.16. +7
CUSO4 solution
with Cu electrodes
3. Refer answer to Problem 1(e) on page 3.123 and
Q. 19 on page 3.16.
4. (a) The conductivity of a conductor is the
-V
reciprocal of its resistivity. Its SI unit isD"^ m .
-1

(&) Refer answer to Q. 19 on page 3.16.


5. Refer answer to Q. 1 on page 3.190. -I
6. Refer answer to Q. 1 on page 3.190. or

7. Resistivity of a material is the resistance of a 7

conductor of that material having unit length and Water voltameter


unit area of cross-section. The SI unit of resistivity is (water acidulated
ohm metre (Dm). with dil, H2SO4)
m
Resistivity of a conductor =
ne\
With the increase in temperature, the amplitude of
vibration of positive ions increases. The electrons V,0
suffer collisions more frequently. The relaxation
time T decreases. Hence the resistivity of a conductor Fig. 3.338
increases with the increase in temperature.
8. Refer answer to Q. 25 on page 3.24. (b) At the temperature of 4IC the resistance of Hg
9. The conductivity of a conductor is the reciprocal of becomes zero i.e., Hg becomes a superconductor
at4K.
its resistivity. Its SI unit isQ m

(fl) The conductivity of a good conductor (c) The resistance is -ve in the region BC
because here current decreases with the increase in
decreases with the increase in temperature. For
explanation, refer answer to Q. 7 above, voltage.
17. The resistances of the two bulbs will be
(b) The conductivity of an ionic conductor
increases with the increase of temperature. As R. = —
the temperature increases, the electrostatic
attraction between cations and anions

decreases, the ions are more free to move and and


^2 =
so the conductivity increases. P2
3.189

(0 When the bulbs are connected in series : (c) Sensitivity can be increased by reducing the
potential gradient by (i) increasing the length
= Rj + Rj of potentiometer wire, (ii) reducing the value
of 6, (/») increasing resistance of the rheostat.
or 22. (fl) Refer to point 47 of Glimpses,
Pi (b) Refer answer to Q. 62 on page 3.104.
or J_=J_ A 23. (fl) Refer answer to Q. 62 on page 3.104.
(b) See Fig. 3.217 on page 3.106.
PiPl 24. Refer answer to Q. 65 on page 3.105.
Ps = Metre bridge becomes insensitive for very low
resistance. Moreover, the end resistances become
{ii) When the bulbs are connected in parallel: comparable to the unknown low resistance and

w
1 1 1 cannot be neglected.
or
R
1 Pi 25. A Wheatstone bridge is an arrangement of four
resistances P, Qj R and S joined to form a

Flo
or
quadrilateral ABCD with a battery between A and C
18. Refer answers to Q. 58 and Q. 59 on pages 3.96 & 3.97. and a sensitive galvanometer between B and D.

ee
When no current flows through the galvanometer
19. Refer answer to Q. 59 on page 3.97. P R

Fr
20. Refer answer to Q. 59 on page 3.97. Q S
21. (a) Reason. The positive terminals of both cells Refer answer to Q. 65 on page 3.105.
for
ur
and 6 are connected to the point A while 26. {a) Green, blue, orange, silver,
negative terminal of 62 is connected to A. (b) (0 High resistivity.
The difficulty can be resolved by interchanging (ii) Low temperature coefficient of resistivity,
ks
the terminals of Sj.
Yo

(c) (0 Uniformity of wire.


(b) Resistance R protects the galvanometer by
oo

(ii) Balance point near the midpoint of the wire.


reducing correct flowing through it.
eB

■ Type D : Long Answer Questions II (5 marks each)


r
ou
ad

1. (n) Why do the 'free electrons', in a metal wire, 3. (i) Derive an expression for drift velocity of elec¬
'flowing by themselves', not cause any current trons in a conductor. Hence deduce Ohm's law.
Y

flow in the wire ? {ii) A wire whose cross-sectional area is increasing


(b) Define 'drift velocity' and obtain an expression linearly from its one end to the other, is con
nd
Re

for the current flowing in a wire, in terms of the nected across a battery of V volts. Which of the
'drift velocity' of the free electrons, following quantities remains constant in the
Fi

wire ?
(c) Use the above expression to show that the
'resistivity', of the material of a wire, is inversely (fl) drift speed (b) current density
proportional to the 'relaxation time' for the (c) electric current (rf) electric field
'free electrons' in the metal. Justify your answer. [CBSE D 171
[CBSE OD 16C] 4. (fi) Derive a relation between the internal
2. {a) Define the term drift velocity, resistance, emf and terminal potential
difference of a cell from which current I is
(b) On the basis of electron drift, derive an
expression for resistivity of a conductor in drawn. Draw V vs. I graph for a cell and
terms of number density of free electrons and explain its significance.
relaxation time. On what factors does (b) Two cells of different emfs and internal
resistivity of a conductor depend ? resistances are connected in parallel with one
(c) Why alloys like constantan and manganin are another. Derive the expression for the
used for making standard resistors ? equivalent emf and equivalent internal
[CBSE D 05, 16] resistance of the combination. [CBSE OD 19]
3.190 PHYSICS-XII

5. Define the term potential gradient. Using this 9. (a) Draw a circuit diagram of a metre bridge used
concept, explain the method for comparison of to determine the unknown resistance R of a
emfs of two primary cells using a potentiometer. given wire. Hence derive the expression for R
Establish the relation used. Write two possible in terms of the known resistance S
causes of potentiometer giving only one-sided (b) What does the term 'end error' in a metre
deflection.
[CBSE D13I
bridge circuit mean and how is it corrected ?
6- (fl) State the working principle of a potentiometer. How will be balancing point be affected, if the
Draw a circuit diagram to compare the emfs of
two primary cells. Derive the formula used.
positions of the battery and galvanometer are
interchanged in a metre bridge experiment ?
(&) Which material is used for potentiometer wire Give reason for your answer.
and why ?
[CBSE D lie, 17C]
(c) How can the sensitivity of a potentiometer be 10. (fl) Obtain the condition under which the current
increased ?
[CBSE D 11 C; F 17]
flowing, in the 'current detecting device', used
7. (fl) State the working principle of a potentiometer. in the circuit shown in Fig. 3.339, becomes zero.
With help of the circuit diagram, explain
how the internal resistance of a cell is p Q
determined.
W'Af—f—VW
Current
(Resistance
’ =Ro) '■
!
detecting i>-
(&) How will the balancing length be affected in device

the potentiometer circuit when (j) the >vw


internal resistance of the driver cell increases and X R

(ii) the series resistor connected to the driver


cell is reduced ? Justify your answer.
[CBSE D 17CJ
8. Deduce the condition for balance in a Wheatstone Fig. 3.339
bridge. Using the principle of Wheatstone bridge,
describe the method to determine the specific (b) Describe briefly the device, based on the above
resistance of a wire in the laboratory. Draw the condition. Draw a circuit diagram for this device
circuit diagram and write the formula used. Why and discuss, in brief, how it is used for finding
an unknown resistance.
it is considered important to obtain the balance [CBSE OD 16C]
point near the midpoint of the wire ? [CBSE D 04]

Answers

1. («) While flowing themselves, the free electrons eE


have random motion in a metal. There in no = u + a T = 0-

net flow of charge across any section of the


VI

wire. So no current flows in the wire. -> eE


or V X
(b) Drift velocity. Force exerted on a free electron d -
m
—>

by the external field E is Current. If nis the number of free electrons per
F =-eE or via =-e£
unit volume, then the amount of charge
crossing area A in time At will be
eE' Acf = enAv^At
a
m

I = — = enAv,
Average thermal velocity of free electrons in a
conductor is zero, zT = 0. eEx eVx
(c) Ohm's law. =
m ml
The average time elapsed between two
successive collisions of a drifting electron is eVx
J=enAv^=evA.
relaxation time(r). The average velocity gained ml

by a free electron during this time is the drift ml


or

velocity given by / ne\A


At a fixed temperature, tlie quantities m, I, n, e, r 7. (fl) Refer answer to Q. 59 on page 3.97.
and A are all constant for a given conductor. (b) (/) When the internal resistance of the driver
cell increases, the current in the driving circuit
— = a constant, R decreases. This decreases the potential gradient.
I
Hence a larger length of the potentiometer
This proves Ohm's law. wire is needed to balance the unknown emf.
ml
Resistivity. Resistance, R = ne\A {ii) When the series resistor connected to the
driver cell is reduced, the current in the driving
} circuit increases. This increases the potential
Also, R = p—
A gradient. Hence a smaller length of the
m potentiometer wire is needed to balance the
Resistivity, p = —^ unknown emf.
T

8. Refer answers to Q. 62 on page 3.104 and Q. 65 on


2. (fl) Refer to point 17 of Glimpses,

w
page 3.105 and solution of Problem 65 on page 3.120.
(b) Refer answer to Q. 1 above,
9. (fl) Refer answer to Q. 65 on page 3.105.
(c) Alloys like constantan and manganin have very
ib) Tlie error which arises due to resistance of
weak dependence of resistivity on temperature.

Flo
3. (/) Refer answer to Q. 1 above.
copper strips and the connecting wires at both
ends of the metre bridge is called end error. By
(ii) Refer answer to Exercise 3.18(a) on page 3,152. using thick copper strips, we can minimise the

ee
4. (a) Refer answer to Q. 35 on page 3.46. error due to end resistance.

Fr
(b) Refer answer to Q. 38 on page 3.52. If the positions of the battery and galvano
5. Refer answer to Q. 58 on page 3.96 and Problem 42 meter are interchanged, the condition for
on page 3.142. balance of the bridge remains the same. So the
for
ur
6. (fl) Refer answer to Q. 57 and Q. 58 on page 3.96. position of balancing point is not affected.
(b) Refer to the solution of Problem 49 on page 3.119. 10. (fl) For the derivation of condition for balanced
(c) It can be made more sensitive by decreasing Wheatstone bridge, refer answer to Q. 62 on
ks
current in the main circuit decreasing potential page 3.104.
Yo

(b) Refer answer to Q. 65 on page 3.105.


oo

gradient/increasing resistance put in series


with the potentiometer wire.
r eB
ou
ad
Y
nd
Re
Fi
COMPETITION SECTION

Currenf Eleclridfy
GLIMPSES

1- Current electricity. The study of electric charges round the complete circuit. It is equal to the
in motion is called current electricity. terminal p.d. measured in open circuit.
2. Electric current. The flow of electric charges EMF =
Work done W
through a conductor constitutes electric current. or

Charge
Quantitatively, electric current across an area
held perpendicular to the direction of flow of 6* SI unit of emf is volt. If an electrochemical cell
charge is defined as the amount of charge supplies energy of 1 joule for tlie flow of 1 coulomb
flowing across that area per unit time. of charge through the whole circuit (including
the cell), then its emf is said to be one volt.
For a steady flow of charge, I=~
t
7. Ohm's law. The current flowing through a con-
If the rate of flow of charge varies with time, ductor is directly proportional to the potential
then
difference across its ends, provided the tem
I = lim perature and other physical conditions remain
Af-»0 At dt unchanged.
SI unit of current is ampere (A).
VocI or V^RI or - = R
1 coulomb I
1 ampere = or 1A = ICs'L
1 second
Here R is called the resistance of the conductor.
Conventional and electronic currents. The 8. Resistance. It is the property by virtue of which
direction of motion of positive charges is taken a conductor opposes the flow of charges
as the direction of conventional current. Electrons through it. It is equal to the ratio of the potential
being negatively charged, so the direction of difference applied across the conductor to the
electronic current is opposite to that of the current flowing through it. It depends on the
conventional current.
length / and area of cross-section A of the
4. Electric current is a scalar quantity. Although conductor through the relation :
we represent current with an arrow, yet it is a /
scalar quantity. Electric currents do not obey the R =
p —, p =resistivity of the material.
laws of vector addition.

5- Electromotive force (emf). The emf of a source 9. SI unit of resistance is ohm (Q). The resistance of
a conductor is 1 ohm if a current of 1 ampere
may be defined as the work done by the source flows through it on applying a potential
in taking a unit positive charge from its lower difference of 1 volt across its ends.
potential terminal to the higher potential
1 volt
terminal. Or, it is the energy supplied by the 1 ohm = or in=lVA
-I

source in taking a unit positive charge once 1 ampere

(3.192)
CURRENT ELECTRICITY {Competition Section) 3.193

15. Colour code for carbon resistors.


10. Resistivity or specific resistance, It is the
resistance offered by a unit cube of the material Colour Number Multiplier
of a conductor. ,0
Black 0 10
RA
P =
Brown 1 10^
I
Red 1 1(3
SI unit of p = Q m 3 10^
Orange
It depends on the nature of the material of the Yellow 4

conductor and the physical conditions like Green 5 10^


temperature, pressure, etc. Blue 6 10^
Violet 7 10^
11. Current density. It is the amount of charge
Grey 8 10'

w
flowing per second per unit area normal to the
White 9 10^
flow of charge. It is a vector quantity having the
same direction as that of the motion of the
How to remember colour code :

Flo
positive charge. B B R O Y of Great Britain had Very Good Wife
For normal flow of charge, i i i

ee
0 1 2 3 4 5 6 7 8 9
. <]lt I

Fr
Tolerence :

Gold Silver No colour


In general, I = jA cos Q = ] . A ± 5% ± 10% ± 20%
for
ur
SI unit of current density = Am”^. A set of coloured co-axial bands is printed on
the resistor which reveals the following facts :
12. Conductance. It is the reciprocal of resistance.
ks
(1) The first band indicates the^Irsf significant
Yo

1
Conductance = figure.
oo

Resistance
(2) The second band indicates the second
eB

1 significant figure.
or G =-
R
(3) The third band indicates the power of ten
with which the above two significant figures
r

SI unit of conductance = ohm ^ = mho


ou

must be multiplied to get the resistance


ad

= siemen (S). value in ohms.


Y

13. Conductivity or specific conductance. It is the (4) The last band indicates the tolerence in per
reciprocal of resistivity. cent of the indicated value.
nd
Re

1 16. Carriers of current. In metals, free electrons are


Conductivity =
the charge carriers. In ionised gases, electrons
Fi

Resistivity
and positively charged ions are the charge
or a = —
1 carriers. In an electrolyte, both positive and
P negative ions are the charge carriers. In
semiconductors like Ge and Si, conduction is
SI unit of conductivity due to electrons and holes. A hole is a vacant
state from which an electron has been removed
-1
- ohm * m ^ = mho m ^ = S m ^
and acts as a positive charge carrier.
14. ■i.stivities of different substances. Metals have
17. Drift velocity and relaxation time. The average
low resistivities in the range of 10"® D m to velocity acquired by the free electrons of a con
10"'’ Cl m. Insulators have resistivities more than
ductor in the opposite direction of the externally
10“* Cl m. Semiconductors have intermediate
applied electric field is called drift velocity (v^).
resistivities lying between 10”^ D m to lO'^ Q m. The average time that elapses between the two
.194 PHYSICS-XII

successive collisions of an electron is called


For an electron.
relaxation time (t).
e£ t ml m
V
d ~ ; R = ; P= — For a hole.
m ne^ T A ne T

SI unit of mobility = m^ V“^ s


-1
I = en Avj ; j =env^
Here n =no. of free electrons per unit volume or Practical unit of mobility = cm^ s'\
free electron density and m = mass of an electron. 22. Relation between electric current and mobility.
18. Other forms of Ohm's law. In terms of vector For a conductor,

quantities like current density / and electric I =en A\i^ E


For a semiconductor.
field E, Ohm's law may be expressed as
-*
l = e AE{n\x^^ p\i^)
and
j =a E or
E =pj
where n and p are the electron and hole
The equation E =p j leads to another state densities.

ment of Ohm's law i.e., a conducting material 23. Ohmic conductors. The conductors which obey
obeys Ohm's law when the resistivity of the Ohm’s law are called Ohmic conductors. For
material does not depend on the magnitude and these conductors, V-I graph is a straight line
direction of the applied electric field.
passing through the origin. For example, a
19. Temperature coefficient of resistance (a). It is metallic conductor for small currents is an
defined as the change in resistance per unit Ohmic conductor.

original resistance per degree rise in 24. Non-ohmic conductors. The conductors which
temperature. It is given by do not obey Ohm's law are called non-ohmic
a =
R,-R, conductors. The non-ohmic situations may be of
Rj (f2 t-j^) the following types :
If =0°C and
t2 = t°C, then (/) The straight line V-I graph does not pass
through the origin.
a = or
Rj = Rq (1 + a t) (ii) V-I relationship is non-linear.
R^xt
(Hi) V-I relationship depends on the sign of V
The unit of a is or K“‘. for the same absolute value of V.

20. Effect of temperature on resistance. For metals a (iv) V-I relationship is non-unique.
is positive i.e., resistance of metals increases Examples of non-ohmic conductors are water
with the increase in temperature. voltameter, thyristor, a p-n junction, etc.
For semiconductors and msulators, a is negative 25. Superconductivity. The phenomenon of
i.e., their resistance decreases with the increase complete loss of resistivity by certain metals
in temperature. and alloys when they are cooled below a certain
For alloys like constantan and manganin, the temperature is called superconductivity. The
temperature coefficient of resistance a is very temperature at which a substance undergoes a
small. So they are used for making standard transition from normal conductor to super
resistors. conductor in a zero magnetic field is called
transition or critical temperature (T^).
21. Mobility of a charge carrier. The mobility of a
charge carrier is the drift velocity acquired by it 26. Meissner effect. Tlie expulsion of the magnetic
flux from a superconducting material when it is
in a unit electric field. It is given by
cooled to a temperature below the critical
E m temperature in a magnetic field is called
Meissner effect.
CURRENT ELECTRICITY {Competition Section) 3.195

27. Resistances in series. When a number of 32. Cells in series. If n cells of emf E and internal
resistances are connected in series, their resistance r each are connected in series, then
equivalent resistance (Rj) is equal to the sum of current drawn through external resistance R is
the individual resistances. nt
1 =
Rj = R| + R2 Rg + -. R + nr

28. Resistances in parallel. When a number of 33. Cells in parallel. If m cells are connected in
resistances are connected in parallel, the parallel, then current drawn through external
reciprocal of their equivalent resistance (Rp) is resistance R is
equal to the sum of the reciprocals of the m€
individual resistances. / =
1 1 1 1 mR 4- r
+...
R1 ^ % 34. Cells in mixed grouping. If n cells are connected

w
in series in each row and m such rows are
For two resistances in parallel,
connected in parallel, then current drawn

Flo
through an external resistance R is
P R, + R2
mnt
/ =
29. Division of current in resistors joined in

ee
mR + nr
parallel. The current is divided in resistors,

Fr
connected in parallel, in the inverse ratio of For maximum current, the external resistance
their resistances. must be equal to the total internal resistance, i.e.,
h nr

for R= —
ur
. I
^1 =
R^ + R2 m

R or mR = nr.
1
. 7
ks
R\ + R2 35. Heating effect of current. The phenomenon of
Yo

the production of heat in a resistor by the flow


oo

30. Internal resistance (r). The resistance offered by


of an electric current through it is called heating
the electrolyte of a cell to the flow of current
eB

effect of current or Joule heating. It is an


between its electrodes is called internal
irreversible process.
resistance of the cell. It depends on (/) nature of
the electrolyte, («/) concentration of the 36. Joule's law of heating. It states that the amount
r

of heat H produced in a resistor is


ou

electrolyte, {Hi) distance between the electrodes,


ad

{iv) common area of the electrodes dipped in the (i) directly proportional to the square of
Y

electrolyte and (u) temperature of the current for a given R,


electrolyte. (ii) directly proportional to the resistance R for
nd
Re

31 Relations between emf (E), terminal potential a given /,


difference (V) and internal resistance (r). The (Hi) inversely proportional to the resistance R
Fi

potential drop across the terminals of a cell for a given V, and


when a current is being drawn from it is called (iv) directly proportional to the time t for which
its terminal potential difference. It is less than the current flows through the resistor.
the emf of the cell in a closed circuit.

ER Mathematically, this law can be expressed as


E = V + 7r; V"=E-7r; V = IR =
H = Vlt joule
R + r
V^t
E-y E-y E E = I^Rt joule = R
joule
r = = R ; 1 = : I max
I V R +r ' r
Vlt
or H = cal
4.18
Terminal p.d. of a cell when it is being charged
is I'^Rt vh
cal = cal.
V=E+7r, y>E. 4.18 4.18 R
3.196 PHYSICS-Xll

37. Electric power. It is the rate at which an electric power is equal to the sum of the reciprocals of
appliance converts electric energy into other the individual powers of the appliances which
forms of energy. Or, it is the rate at which work is have been manufactured for working on the
done by a source of emf in maintaining an same voltage.
electric current through a circuit. 1 1 1 1
Electric power, - = — + — + — +
P R1
Pi Pi
W 0
p=—= y/ = /2R = 43. Power consumed by a parallel combination of
t R
appliances. The effective power is equal to the
38. SI unit of power is watt. The power of an sum of the powers of the individual appliances.
appliance is one watt if one ampere of current
flows through it on applying a potential P = Pj + + Pg 4-
difference of 1 volt across it.
44. Efficiency of a source of emf. It is the ratio of the
1 watt =
1 joule output power to the input power. If a source of
1 second emf € and internal resistance r is connected to an
external resistance, then its efficiency will be
= 1 voltx 1 ampere
Output power
or 1 W = 1 Js"^ = 1 VA Ti =
Input power
1 kilowatt (kW)=1000W.
VI _V _ R
39. Electric energy. It is the total work done in Bl B R + r
maintaining an electric current in an electric
45. Maximum Power Theorem. It states that the
circuit for a given time.
output power of a source of emf is maximum
Electric energy = Electric power x time when the external resistance in the circuit is
W =Pf equal to internal resistance of the circuit i.e.,
when R = r.
= VI t joule = I^Rt joule
Pmax
40. Units of electric energy. The commercial unit of 4r
electric energy is kilowatt-hour (kWh) or Board
The efficiency of a source of emf is 50% when it
of Trade (B.O.T.) unit. It is the electric energy
delivers maximum power.
consumed by an appliance of power 1000 watt
in one hour. 46. Efficiency of an electric device. It is the ratio of
the output power to the input power.
1 kWh =1000 Wh
Output power
= 1000 Wx 3600 s =3.6x10^1 11 =
Input power
Another unit of electric energy is watt hour.
(f) For an electric motor.
1 watt hour = 1 watt x 1 hour
Output mechanical power
= 3.6x10^ J. 11 =
Input electric power
41. Power rating. The power rating of an electrical
(n) Input electric power
appliance is the electrical energy consumed per
second by the appliance when connected across = Output mechanical power
+ Power lost as heat
the marked voltage of the mains.
V'^ 1
p = ~ = rR = vi.
The power output of an electric motor is
R maximum when its back emf is one-half the

42. source emf, provided the resistance of the


Power consumed by a series combination of
appliances. The reciprocal of the effective windings of the motor is negligible.
CURRENT ELECTRICITY (Competition Section) 3A97

47. Kirchhoff's laws. These laws enable us to (j) To compare the emfs of two cells. If and I2
determine the currents and voltages in different are the balancing lengths of the potentiometer
parts of the electrical circuits. wire for the cells of emfs 6^ and €2 respectively,
then
First law or junction rule. In an electric circuit,
the algebraic sum of currents at any junction is
zero. Or, the sum of currents entering a junction /
1
is equal to the sum of the currents leaving that
junction. (if) To find the internal resistance r of a cell. If
Mathematically, is the balancing length of the potentiometer
2 /=0 wire without shunt and I2 the balancing length
with shunt R across the cell, then internal

w
Justification. This law is based on the law of resistance of the cell will be
conservation of charge. When the currents in a ^-V L-I
1
r = X R = 2-x R
circuit are steady, charges cannot accumulate or V h

Flo
originate at any point of the circuit.
50. Wheatstone bridge. It is an arrangement of four
Second law or loop rule. Around any loop of a

ee
resistances P, Q, R and S joined to form a
network, the sum of changes in potential must
quadrilateral ABCD with a battery between A

Fr
be zero. Or, the algebraic of the emfs in any loop
and C and a sensitive galvanometer between B
of a circuit is equal to the sum of the products of
and D. The resistances are so adjusted that no
currents and resistances in it.
for
ur
current flows through the galvanometer. The
2 AV=0 bridge is then said to be balanced. In the
balanced condition.
ks
or 2 € = 2 JR
Yo
oo

Justification. This law is based on the law of Q S


conservation of energy. As the electrostatic
eB

force is a conservative force, the total work done Knowing any three resistances, the fourth
by it along any closed path must be zero. resistance can be computed. A Wheatstone
bridge is most sensitive when the resistances in
r

48. Galvanometer. It is a sensitive device to detect


ou

its four arms are of the same order.


ad

current in a circuit. It produces a deflection


Y

proportional to the electric current flowing 51. Slide wire bridge or metre bridge. It is an
through it. application of wheatstone bridge in which R is
nd

fixed and a balance point is obtained by varying


Re

49. Potentiometer. It is a device used to compare P and Q i.e., by adjusting the position of a jockey
Fi

emfs of two sources. Its working is based on on a 100 cm long resistance wire stretched
the principle that when a constant current between two terminals. If the balance point is
flows through a wire of uniform cross- obtained at length I, then
sectional area and composition, the p.d. across
P_R /
any length of the wire is directly proportional
to that length. Q~ S 100-/
Vocl
or r 100- / ^ R
/
or V = kl
SA
where k is the potential drop per unit length Resistivity, p = /
which is called potential gradient. Poten Sx KT^
tiometer has two main uses.
I
PHYSICS-XII

JEE Advance

Multiple Choice Questions with 4. The current I and


one correct answer
voltage V curves for a given
metallic wire at two different
1. The potential difference applied to an X-ray tube
temperatures and Tj ^^e
is 5 kV and the current through it is 3.2 mA. Then the
shown in the figure. Then,
number of electrons striking the target per second is
16
(<i) 2 X10 (1j)5x10^ (a)T^>T^ (b)r,<T2
17 15
(c)lxlO (rf)4xl0 [IIT 2002] ic)T,=T^ (d) 7^=212 I1IM989; VMMC14I

2. Express which of the following set up can be 5. Six identical resistors are connected as shown in
used to verify Ohm's law ? the figure.
(«) (b) R

I rA/^{V Wv Q

(V) 0 (V)
■R‘
R‘
R R

'R

(C) id)
R

<A> O
The equivalent resistance will be
(a) Maximum between P and R
(b) Maximum between Q and R
<v> O (c) Maximum between P and Q
(d) all are equal.
3. The three resistances of equal value are arranged
in the different combinations shown below. Arrange 6. Find out the value of current through 2Q resis
them in increasing order of power dissipation. tance for the given circuit.

W\ VvV-
(0 10 V 5n lofi 20 V

Vv\— 2H
AA/V
>
(a) zero {b)2A
AA/V (c)5A (rf)4 A lUT 05; AJEEE 03]
(/'■)
7. A constant voltage is applied between the two
AA/V ends of a uniform metallic wire. Some heat is
AVv
developed in it. The heat developed is doubled, if
AAV AW (fl) both the length and radius of wire are halved.
AA/v (b) both length and radius of wire are doubled,
AAV (c) the radius of wire is doubled.
m {iv)
(d) the length of the wire is doubled. (IIT 1980]
(fl) 111 < 11 < IV < I (b) II < III < IV < I 8. A 100 W bulb Bj, and two 60 W bulbs and
(c) 1 < IV < in < II (d) I <III<II<IV [IIT 2003]
are connected to a 250 V source as shown in the figure.
CURRENT ELECTRICITY (Competition Section) 3.199

B.
13. In the circuit shown in the figure, P * R. The
^ ^ reading of the galvanometer is same with switch S
B3
open or closed. Then,

250 V

Now W^, and W3 are the output powers of the


bulbs Bj, % respectively. Then
(fl) Wj >W2=W3 (b)lVj>W2>W3 I

(c)W^ <W2=W3 (rf)lV^<W2<W3 [IIT 2002]

w
9. In the circuit as shown in the figure, the heat
produced in the 5 H resistor due to the current flowing (c) 1q = Jq (HT 1999]

Flo
in it is 10 cal s"’. 14. In the given circuit, it is observed that the
4C1 6fl
current / is independent of the value of the resistance
AA/V VvV

ee
^6-

Fr
AO- ●OB AA/V
Bs
5a R. R3
AA/V
for
ur
The heat generated in 4 Q is
-1 -1
(fl) 1 cals {b) 2 cal s R,
ks
-1 -1
(c) 3 cal s (rf) 4 cal s [[IT 1985]
Yo
oo

10. A wire of length L and 3 identical cells of


Then the resistance values must satisfy
negligible internal resistances are connected in series.
eB

Due to the current, the temperature of the wire is (n) R^R2R^ —


raised by AT in time t. A number N of similar cells is 1 1
(M _L + _ =
1
now connected in series with a wire of the same
r
ou

material and cross-section but of length 2L. The tem


ad

perature of the wire is raised by the same amount AT in (c) R^R_^ = R2R^
Y

the same time. The value of N is


(d) RjRg = = RgRg (IlT 20011
(a) 4 (1;) 6 (c) 8 (d) 9 [IIT 21X11]
15. The effective resistance between points P and Q
nd
Re

11. An ideal gas is filled in a closed rigid and


of the electrical circuit shown in the figure is
thermally insulated container. A coil of 100 D resistor
Fi

2R A 2R
carrying current 1A for 5 minutes supplies heat to the ^WV iL^NAA^—I
gas. The change in internal energy of the gas is
2R
(a) 10 kJ (b) 30 kj
r
(c) 20 kJ id) 0 kJ [IIT 2005] po- aaAt
B
AAV oQ
12. In the given circuit, the potential drop across the
2R
capacitor must be
V 2R 2R
R
{a)V i AW AW
^-AAAr-
(b) VI2
V c 2Rr 8R(R + r)
(c) V/3 («) (^)
R + r 3R + r
(d) 2V/3 5R
2V
(IIT 2001] 2R (c)2r+4R + 2r
1 AW [IIT20Q2;IPUEE11]
3.200 PHYSICS-XII

16. In the shown arrangement of the experiment of dissipated by the configuration Kl, K1 and R3 is PI, P2
the metre bridge if AC corresponding to null deflection and P3, respectively, then
I y (fl) PI > P2 > P3 (b) PI > P3 > P2

R1
(c) P2 > PI > P3 (d) P3 > P2 > PI
A/W Wv To verify Ohm's law, a student is provided with
G. a test resistor Rj., a high resistance Ry a small
B
resistance R2, two identical galvanometers Gj and C2
A‘
and a variable voltage source V. The correct circuit to
X

of galvanometer is x, what would be its value if the carry out the experiment is
radius of the wire AB is doubled ? Gi
(«)
{a) X ^2

(c) 4x Rt G2 R1

17. Consider a AAA-


thin square sheet of
side Land thickness t,
made of a material of
/

resistivity p. The resis


tance between two Gi
_L (b)
opposite faces, shown t
Rl
by the shaded areas T L^VW-'
in the figure is
jR’j- Gz ^2
(fl) directly proportional to L MV 3MW-1
(b) directly proportional to t
(c) independent of L V

(d) independent of f /

Figure shows three resistor configurations Rl, (C) Gj R1


R2 and R3 connected to 3 V battery. If the power ZIHWv^
G2
MV
in Rt
R2
1 V

in

in id) G, ^2
in h-MAn
G2
VW
Rj R1

7^ III!

in. Incandescent bulbs are designed by keeping in


mind that the resistance of their filament increases
with the increase in temperature. If at room
temperature, 100 W, 60 W and 40 W bulbs have
filament resistance R 100' RgQ and R^q, respectively, the
relation between these resistances is
CURRENT ELECTRICITY (Competition Section) 3.201

1 I-
- (f’)K 100 - ^40 + ^60
R R R
100 40 60
R 90 Q
1 1 VAr AAV
{c)R 100 >R..>R
60 40
>
R R R
100 60 ●40

|HT 2010]

21. A metre bridge is set-up as shown, to determine 40.0 cm


an unknown resistance X using standard 10 ohm resistor.
The galvanometer shows null point when taping key Tine unknown resistance is
is at 52 cm mark. The ^
X ion (fl) 60 + 0.15 n (l7) 135 ±0.56 0
end-corrections are
rAVVi rVsAn I[IT Adv. 14)
(c) 60 ±0.25 0 (d) 135 ±0.23 0
1 cm and 2 cm respec
[
tively for the ends A Multiple Choice Questions with one or more

w
and B. The determined than one correct answers
value of 'X' is A B
24. Read the following statements carefully :
(fl) 10.2 ohm

Flo
y The resistivity of a semiconductor decreases
(&) 10.6 ohm
with increase of temperature.

ee
(c) 10.8 ohm (d) 11.1 ohm [IIT 20111
Z In a conducting solid, the rate of collisions
22. An infinite line charge of uniform electric

Fr
between free electrons and ions increases witii
charge density X lies along the axis of an electrically increase of temperature.
conducting infinite cylindrical shell of radius R. At
Select the correct statement(s) from the following:
for
ur
time f = 0, the space inside the cylinder is filled with a
(a) Y is true but Z is false (6) Y is false but Z is true
material of permittivity £ and electrical conductivity a.
The electrical conduction in the material follows (c) Both y and Z are tnie
ks
Ohm's law. Which one of the following graphs best (d) y is true and Z is the correct reason for Y
Yo

[HI
describes the subsequent variation of the magnitude of
oo

current density /(f) at any point in the material ? 25. When a potential difference is applied across,
eB

(fl) (&) the current passing through


Ho¬
(a) an insulator at 0 K is zero
(b) a semiconductorat 0 K is zero
r
ou

(c) a metal at 0 K is finite


ad

(d) a p-n diode at 300 K is finite, if it is reverse


Y

biased. lin' 19‘^'|


t
(0,0) (0,0) 26.Jn an aluminum (Al) bar of square cross-section,
nd
Re

a square hole is drilled and is filled with iron (Fe) as


(c) (d) shown in the figure. The electrical resistivities of Al
Fi

no no
and Fe are2.7x 10“®nm and 1.0 x 10“^n m, respectively.
The electrical resistance
I
between the two faces P and Q
I
of the composite bar is >
I
“ T

2475
(«)
64
1875
(0,0) (b)
LJEE Adv, lo| 64
1875
2.5.
During an experiment with a metre bridge, the
/
(0 pn z.
49
galvanometer shows a null point when the jockey is 2475
pressed at 40.0 cm using a standard resistance of 90 O, as (b) pD Fe ^
132
shown in the figure. The least count of the scale used in 2 mm P
IJEE Adv. 15]
the metre bridge is 1 mm. 7 mm
3.202 PHYSICS-XII

27. For the circuit shown in the figure The current in resistance would be zero if

(a) and R^=R^ = R^


(b) = ^2 and R^ =2R^ = Rj
2kfi R.
(c) V^=2V2and2Kj=2R2 = R3
24 V-ir (rf) 2 -Vj =1^2 snd 2 = 1?2 = ^3
6kQ R, R, 1.5 kn
’ ], An incandescent bulb has a thin filament of
tungsten that is heated to high temperature by passing
(a) the current 1 tluough the battery is 7.5 mA an electric current. The hot filament emits black-body
(b) the potential difference across is 18 V. radiation. The filament is observed to break up at
random locations after a sufficiently long time of
(c) ratio of powers dissipated in R^ and R^ is 3. operation due to non-uniform evaporation of tungsten
(d) if Rj and R2 are interchanged, magnitude of the from the filament. If the bulb is powered at constant
power dissipated in R^, will decrease by a factor voltage, which of the following statement(s) is(are) true ?
of 9.
(fl) The temperature distribution over the filament
is uniform
28, For the resistance network shown in the figure,
choose the correct option(s). (&) The resistance over small sections of the
I- 2Q filament decreases with time

2f2 (c) The filament emits more light at higher band of


frequencies before it breaks up
1 Cl in
{d) The filament consumes less electrical power
O' towards the end of the life of the bulb
MV i'j
7
4f2
52. In the circuit shown, initially there is no charge
12 V
on capacitors and keys S, and $2 are open. The values
of the capacitors are Cj = 10 pF, C2 = 30 pF and
(rt) The current through RQ is zero. C3 = Q = 80 pF.
C4
{b) =3A.
Jj. r-i p 30 n
I MV
(c) The potential at S is less than that at Q. 5 V
9^S
(d) 72=2 A c,

Heater of an electric kettle is made of a wire of


C3
length Land diameter d. It takes 4 minutes to raise the 70 Q
30 fi

temperature of 0.5 kg water by 40 K. This heater is 10 V


replaced by a new heater having two wires of the same I AA^ 1
material, each of length Land diameter2d. The way these Q 100 n
wires are connected is given in the options. How much
Which of the statement(s) is/are correct ?
time in minutes will it take to raise the temperature of
the same amount of water by 40K ? (n) The key Sj is kept closed for long time such that
(ii) 4 if wires are in parallel
capacitors are fully charged. Now key $2 is
closed at this time, the instantaneous current
(b) 2 if wires are in series across 30 H resistor (between points P and Q)
(c) 1 if wires are in series will be 0.2 A (round off to 1st decimal place),
(d) 0.5 if wires are in parallel. (b) If key Sj is kept closed for long time such that
V R capacitors are fully charged, the voltage
R, difference between points Pand Q will be 10 V.
Two ideal batteries of emf
MV (c) At time ■ = 0, tlie key S, is closed, the instantaneous
Vj and V, and three resistances current -n the dosed circuit will be 25 mA.
R^,R2 and R^ are connected as
shown in the figure.
V,
(d) If key Sj is kept closed for long time such that
R-. capacitors are fully charged, the voltage across
MV
the capacitor Cj will be 4 V. III I A5'. Ah'-'
CURRENT ELECTRICITY (Competition Section) ^■203 J
33. Shown in the figure is a semicircular metallic AM»—VW wv
R R,
strip that has thickness t and resistivity p. Its inner
radius is and outer radius is R2. If a voltage V'q is
■9^ R2
applied between its two ends, a current I flows in it. AVv
Si Rj/2 AW
^3
A B -a^o AW-
Circuit-1 S2 2R3

/I B
Circuit-2
In addition, it is observed that a transverse voltage AV
develops between its inner and outer surfaces due to Qj and Q2 are the power dissipations in Circuit-1

w
purely kinetic effects of moving electrons (ignore any and Circuit-2 when the switches and $2 are in closed
role of the magnetic field due to the current). Then conditions, respectively.
(figure is schematic and not drawn to scale) Which of the following statement(s) is(are)

Flo
VJ
0 correct ?
(«) 1 = In
R
np K *'1 (a) When a voltage source of 6 V is connected

ee
(fc) The outer surface is at a higher voltage than the across A and B in both circuits, Pj < P2-

Fr
inner surface (b) When a constant current source of 2 A is
(c) The outer surface is at a lower voltage than the connected across A and B in both circuits, Pj >
inner surface (c) When a voltage source of 6 V is connected
for
ur
(rf) AF oc f IJliE Adv. 2020] across A and Bin Circuit-1, Qj > P^.
(d) When a constant airrent source of 2 A is con
34. The figure shows a circuit having eight
nected across A and B in both circuits, Q2 < Qj
ks
resistances of 1 each, labelled Kj to Rg, and two ideal
Yo

batteries with voltages =12 V and £2 =6 V. [JEI- Adv 2:.|


oo

Reasoning Type
eB

Instructions. Each question contains statement - 1


(assertion) and statement - 2 (reason). Of these
r

statements mark correct choice if


ou
ad

(fl) statements - 1 and 2 are true and statement - 2


Y

is a correct explanation for statement - 1.


(b) statements - 1 and 2 are true and statement - 2
nd

is not a correct explanation for statement - 1


Re

Which of the following statement(s) is(are) correct ?


(c) statement - 1 is true, statement - 2 is false
Fi

(fl) The magnitude of current flowing through R■^ is


7.2 A. (d) statement - 1 is false, statement - 2 is true.
(1?) The magnitude of current flowing through Rj is
1.2 A.
36.Statement 1. In a Metre Bridge experiment, null
(c) The magnitude of current flowing through R^ is point for an unknown resistance is measured. Now,
4.8 A. the unknown resistance is put inside an enclosure
(d) The magnitude of current flowing through R^ is maintained at a higher temperature. The null point can
2.4 A. iJEE Adv 22] be obtained at the same point as before by decreasing
the value of the standard resistance.
35.In Circuit-1 and Circuit-2 shown in the figures,
Rj = 1 n, R2 = 2 n and R^ = 3 Q. Statement 2. Resistance of a metal increases with

Pj and P2 are the power dissipations in Circuit-1 increase in temperature. Ill i


and Circuit-2 when the switches and ^ are in open
conditions, respectively.
3.204 PHYSICS-XII

Linked Comprc.iension V 's


Paragraph for Q. 37 to 38 ' When two identical batteries of internal
Electrical resistance of certain materials, known as resistance 1 Q each are connected in series across a

superconductors, changes abruptly from a nonzero resistor R, the rate of heat produced in R is Jy When
value to zero as their temperature is lowered below the same batteries are connected in parallel across R.
a critical temperature 7^(0). An interesting the rate is f2- If =2.25 J^, then what is the value of R
property of superconductors is that their critical in n ?

temperature becomes smaller than T^(0) if they are Two batteries of different emfs and different
placed in a magnetic field, i.e., the critical internal resistances are connected as shown. What is
temperature 7^(6) is a function of the magnetic the voltage across AB in volts ? ' .
field strength B. The dependence of 7^ (B) on B is
shown in the figure. -I—vw
6V IQ
rc(B). Ao- ■OB

Tc(0) -I—VAr
3V 2Q

In the following circuit, the current through the


O B
resistor R(=2Q)is 1 amperes.
The value of / is —
In the graphs below, the resistance K of a
superconductor is shown as a function of its
R(= 2 Q) IQ
temperature T for two different magnetic fields Bj -Wv-
(solid line) and (dashed line). If is larger than B,, 2Q.(>
%
which of the following graphs shows the correct 6Q 2Q
variation of R and T in these fields ?

(a) (b) 6.5 V lOQ

R
AAV
R

12 n 4Q

Bj, Si
O o
T T ■ In order to measure the internal resistance of a
(C) (rf) cell of emf S, a metre bridge of wire resistance
R
Rq = 50 Q, a resistance Rq /2, another cell of emf S /2
(internal resistance r) and a galvanometer G are used in
a circuit, as shown in the figure. If the null point is
found at / =72 cm, then the value of = n.

O T O T
RJ2
l-AW

A superconductor has 7^(0) = 100K When a


magnetic field of 7.5 tesla is applied, its 7^ decreases to
75 K. For this material one can definitely say that when I

(fl)B = 5tesla, 7^(B) = 80K


(6) B = 5 tesla, 75 K <r^(B) <100 K
(c) B = 10 tesla, 75 K < 7^ (B) < 100 K
(d) B = 10 tesla, r^(B) =70 K
/ A
CURRENT ELECTRICITY Seaion) 3.205

^UJmcTtcc.' ' :My;! i'i vj Question Stem for Question Nos. 44 and 45
In the balanced condition, the values of the In the circuit shown below, the switch S is
resistances of the four arms of a Wheatstone bridge are connected to position P for a long time so that the
shown in the figure below. charge on the capacitor becomes tjjjiC. Then S is
switched to position Q. After a long time, the charge on
O
the capacitor is <jo|iC.
R, = 60 n p 1^2 “ ^
t 5 in 2n
oN>— ■Wv—1
50 V-i P9Q
S T

^ 1 V T 1 nF T 2V
AAV
R3=30on Q R4 = 50on

w
The resistance has temperature coefficient
0.0004 °C“^ If the temperature of is increased by ! 1, The magnitude of is
100°C, the voltage developed between S and T will be

Flo
volt. := ,The magnitude of q^ is

ee
Answei'.s- aiu! !Y\'pl:-^ii:Ui{)n>'

Fr
q _ ne
1. (.;) As 1 = -!- For the same potential Vq ,
t ~ t
v:
It 3-2xl0-^xl At temperature ,T1" R _ '0
= 2x10^^ for 1 "
ur
n =
-19
I1
e 1-6x10
K0
2. (n) In Ohm’s law, we verify for a resistor that At temperature T^, I^ =
ks
V = IR.
Yo
oo

As I2 <
On O
Since resistance of a metal increases with tempe
eB

O O rature, so % > Tj or Tj < Tj .


3
5. :● R
pQ-f/; R PR = —R
r

~
11
ou
ad

R
Only option («) fits this formula. PQ
IS maximum.
Y

6. ,.The current through20 resistancf will be zero


3-(.) R,„
eq
=R+R+R=3R, P, = l\3R)
because it is not a part of any closed loop.
nd
Re

2Rx R _2R 2R' V- \/2


R P. =
eq 2R+R~ 3 ' II
3 ) 7. ,● R-
R / /
Fi

1 R % P
nr
Ml R
eq 1
+ 1+1 3 ' j
.2
R R R or I.C., Px —
/
Rx R
i\ R + R =
R+R 2 When both I and r are doubled, P gets doubled.

Clearly, P„, < Pjj < P^ < ,P.


8. ( , 1
Voltage across B, is maximum, so glows
4. with maximum brightness.
In the series combination of Bj and £^, the bulb of
lesser wattage glows brighter than B, i.c., W, > Wp
Hence < W^ < W^.
9. I Refer to the solution of Problem 34 on
page 3.139.
.206 PHYSICS-XII

10.(/)) Let R and m be the resistance and mass of the 15. (if) If a potential difference is applied between P
first wire. Then the resistance of second wire would be and Q, no current will flow in arms AB and BC. The
2R and mass 2m.
resistances in arms AB and BC are ineffective. The
Let 6 = emf of each cell equivalent circuit then takes the form as shown below.
and
c= specific heat capacity of the material of AAV
the wire. AAV
2R 2R
3E
For the first wire. current /j = r r
R Po AAV AW oQ
and heat produced. l}Rt
I
= mcAT
NE 2R 2R
For the second wire, current =
2R AAV AW

and heat produced, ll{2 R)t =2mcAT 1 1


1 1 _ R+r
Clearly, = 1^. R
PQ
4R 4R 2r 2R 2r 2Rr

Hence N = 6. 2Rr
or R
PQ- R + r
11. (h) Change in internal energy of the gas R
AC _ ^
= Heat produced in the coil 16. (it) At the null point
100-.T
= /^Rf=(l)^j<100x5x60J
= 30000] =30 kj. When the radius of the wire AB is doubled, both
R
and
AC decrease. But the ratio still remains equal
12. (C-)
to R, / R2. Hence the balancing length AC = x remains
V'
R unaffected.
I
1 AW L _p
17. (c) R =p
d-—= p —
V' C A ^tL t

B
A .’. Resistance is independent of L
2V
2R 18. (c) Equivalent resistances of the three configu
rations are
1 AW
}
Ri = 1 n. R2 =1/2H, R3=2n
Traversing the loop anticlockwise from A,
IR + V-2V+2IR=:0
Power, p = —
R
y
or 3/R = y or 1 =
3R
For same V, Pocl
R

Potential drop across C, As R2 < Rl < R3 P2 > PI > P3.

3 19. (c) Gj behaves as voltmeter by connecting it in


13. {(}) Whether the switch S is open or closed, the series with high resistance Rj and applying it across Rj.
reading of the galvanometer Gis not affected. This indi in parallel. G2 behaves as ammeter by shunting it with
cates that no current flows through switch S. Therefore, small resistance R^ and connecting in series with Rj.
Vg = Vq and the Wheatstone bridge is balanced. y2
20. Of) p = ~.
R '
Hence Ip - Iq and Ip = Iq- Only option («) is correct.
14. ( l ) As I is independent of R^, no current flows where R is the resistance at working temperature
1 1
through Rg. Thus the junction of R, and is at the - X P >
1 1
>
R
same potential as the junction of R3 and R^. Hence for a R
100
R
60
R
40

balanced Wheatstone bridge. 21. (b) Here,


R
1 _
or
R,R, = R,Ry /, = 52 + end correction = 52 +1 = 53 cm
R. R4 /2 = 48 + end correction = 48 + 2 = 50 cm
CURRENT ELECTRICITY (Competition Section) 3.207

X _10 1X10“'X 50x10"^


Now or R
Fe
= -x10“-'‘Q
h h 53 50 4x10"^ 4

53 53
X = xl0=—= 10.6n. 5000 „ ^
= —^1^^ =1250 fiQ
or
50 5

22. . ) Let X be the linear charge density at any Both the bars are in parallel.
instant t and Xn0 at time f =0. Then,
30x1250 1875
X R pa.
£ = R..xR.
A1 Fe
”30 + 1250 ' 64
2ner

Using differential form of Ohm's law 27. ,1 i.'i In option (a),


X
j = aE = <j 6x1,5 _16 kQ
2 Tier 6 + 1.5 ” 5
But 1 =jx A

w
24V
dq aX / =
16
= 7.5 mA => Option (r?) is correct.
X 2 nrl ka
df 2TtKr 5

Flo
Negative sign shows that charge flows radially ; /
outwards through the cylinder.

ee
d gXI 2kClJ>R I 6kn, R.
(U) = - R,i. = 1.5 ka Ri_ = 1.5 kn
dt

Fr
E
24 V 24 V

dX
=^-2df 6kQ R, 2kQ R 1

X E

for
ur
0
~(o/e}t =7.5mAx2kn=15V
As J oc X, so j = JqC R
1
ks
Thus j{t) decays exponentially with time t. Hence R
=24-15=9 V => Option (Ij) is wrong.
Yo

option (/7) is correct.


oo

23. i( 'For the balanced wheatstone bridge, Pr 1 _ 15-/2 ^25 ^ Option (c) is wrong.
eB

R 40
=> R =
40
x90 =60 0 P.
R-, 9-/6 ~ 3
90 100-40 60
9'
/ Pr I
r

As R = x90 1.5
100-/
ou
ad

When R^ and R-, are interchanged,


Y

R " / '*'100-/ R 6T 2 X L5 48 ,kO


=—
‘‘●J 2^1.5 7
5J. OJ
nd
Re

AR = X 60 = 0.25 n
40 60 24 V
/ = - = '-A
Fi

48/7kO 2
Hence, R = (60 ± 0.25)0.
^7

24. Both the statements Y and Z are correct.


V;L =24--x6=3
^ V
2
25. {(/), (/j) and (,/) A semiconductor is a perfect
insulator at 0 K. No current flows through an insulator 3-
or a semiconductor at 0 K. A metal becomes a super
conductor at 0 K, current flowing through it at 0 K will
1.5
V 9^ ”9
be infinite. In reverse biasing at 300 K, a small finite ^ Option (d) is correct.
current flows through a p-n diode due to minority 28. I.-! 1.-1, (. I. (.'1
charge carriers.
p/ _ 2.7 X10“^ X 50x10“^ Due to input and output symmetry’, =Vq and
26. (b) Rai=^ Vg = Vj.. The resistances of 10 in PQ and 10 in ST are
A (7^-2^)xl0“^
ineffective. The current through PQ is zero. The
= 3xl0"'’0=30p0 equivalent circuits are as shown on next page.
3.208 PHYSICS-XII

^2 2 Cl p 2 Cl 5 2Cl B C
»—^ —●■ AW Using KVL in ABCDA,
R1
0+V^-iRj =0 V

4n Q 4n r 4q V.
1
I1 ■Wv—►—Wv—●—VW i = A' AW <D
R
1

1 _ ^2
^3
R1
12 V R1 + R3
2A 6n
*—AW yjK] + VjRg - V|Rj + y2^-i
Yl-Yl
R
1 A 12 n 1

3A AW
Options (a), (b) and (rf) satisfy the above condition.
31. (c). id)

12 V
Since evaporation is given to be non-uniform,
temperature distribution must be non-uniform.
6x12
R Q=4Q => Option (rt) is incorrect.
eq
6 + 12
Due to evaporation, cross-sectional area of wire
12 decreases. Hence resistance of such a section
A =3 A
increases.

12 ^ Option (b) is incorrect.


h “ 6 + 12 x3A = 2 A
When filament breaks up, the temperature of the
Vp-V5=2x2=4V filament will be higher. According to Wien's law (?. oc III
1

T
or cc T), the filament emits more light at higher
band of frequencies.
29.(/>). (</)
H _ HR => Option (c) is correct.
t « R
P As voltage is constant, consumed electrical power.
For original wire.
R

As resistance R increases with temperature, the


For each new wire, R'=p—— = —
{2df 4 filament consumes less electrical power towards tlie
end of the life of the bulb.
R
If wires are in series, R, = R' + R' = 2 R' =
s 2 => Option (d) is correct.
t 4 32. (c),(d)
^ ^ ^ Option (b) is correct. 80 mP son
R' R 4V P4V
AW
If wires are in parallel, R^ = — 8
q 5 V
t 4
-=—=0.5 min. => Option (d) is correct.
Cj * 10 nF 1
8 8 Cj'SO |iF
R'
C3 = 80 {iF
30. (n), (b). (d) 70 n son

10 V
Using KVL in ABCDEFA, we get
IQV j AW
—/R| + ^2 — /R^ + V| = 0 0 Qo 100 n
0

V + v
At t =0, when switch Sj is closed, the capacitors are
Ri + R2 short-circuited. The equivalent circuit is as shown.
CURRENT ELECTRICITY (Cumti-.'iitiuii Section)
R
I—VW ^2
Si P 5V 30 n 1= dl =
P7i a: pTC R
v.'M>
70 0

100 o
Q Option (fl) is correct.
AMr
As the electric field £ provides inward centripetal
R . =200n force to the electron, it should act radially outward.
eq

1 = —A=25mA
200

Option (c) is correct.


At steady state, with Sj closed, the equivalent
circuit is as shown.

w
80 mF 5V
I-
P
C4 V
‘^oufer
<V^inner => Option (c) is correct.

Flo
Cl lOjiF C3 80
mv
2

Also, ^ = eE
Q

ee
r

2
/
J_-i_ J_ J_=2 E = !!!2d.=21

Fr
[}=enAVj]
C cq " 10 ^ 80 80 ” 8 er cr V t-’»tA J
AV = f Erfr => AV oc
C,eq =8pF, Q = 8pFx5V = 40pC
for
ur
40 pC ^ Option (d) is correct.
= 4V,
''c,= 10 pF 34. !.;i, (. ), 11 )
ks
40 pC = 0.5 V
Tlie points A and B are at the same potential. So
Yo

=^C = they can be merged as shown in the equivalent circuit


80 pF
oo

diagram.
P.D. betvc'een P and Q = =4 V
eB

^ Option (if) is correct and option (1?) is incorrect.


As the switch $2 is closed, the equivalent circuit is
r

as shown.
ou
ad

P cfHl—WV
Y

4 V 4V 10 Vt 4V 10 V
nd
Re

91
70 130 n 30 n —Q 30 Q
Fi

2 Using KVL along loop 1,


1 1
lx.r + .t-t- {X
2 2
Q Q

6 4A: + y = 12
1 = 0.079 A - 0.08 A
PQ ” 91 Along loop 2,
30 +
2 12

Option (fi) is incorrect.


-.v-4y = 24
33.(.r),(c-),(<0 ~Ax-\6y = 96 ...{it)
dR = pi _pX7VC
^
A tdx
Adding (/) and (n), we get
-15y =108
= Yq,-Yq^
108
dJ 7.2 A
dR y = -
picc 15
>".210 PHYSiCS-XII

From (f). 1
For same voltage, P oc —
=> Qi > Fj
R
4.r = 12-y = 12+7.2=19.2
A- = 4.8 A Option (c) is correct.
Current through Rj = y = 7.2 A For same current, P oc R and R^ <
4.8-7.2
Current through = 1.2 A Q] < Qz Option (d) is incorrect.
2
R(100-0
Current through R3 = a = 4.8 A 36. (</) R unknown
I
1
Current through R5 = ~ A = 2.4 A As the temperature increases, the value of
unknown resistance increases. To get the same
35. u/i, (,'■), ii I
balancing length /, the value of R should increase.
Ifi 2Q 3n 1 Q Hence statement - 1 is false and statement - 2 is true.
-Wv —vw AW
2n
37. (,' ) As ^ > Bj; critical temperature becomes less.
AW So option (a) is correct.
■0^0- w\
S, 1 3 0 38.
(1)} As critical temperature decreases
2
AW continuously, with increase in magnetic field.
60 For B = 5T ; >75 K but <100 K
A B
-ofo
>2
\W- .●. Option (b) is correct.
Circuit-1

39.
A B
Circuit-2 0 0 0 4

When switches Sj and $2 are open. Equivalent n2


resistances are 28
In case 1, /i = R

5x1 l,R+2r y
^ 2 =1 + ^_16 s2
ll’ll 28
^^2 R

1 1 1 1 11
R+2J lr = in]
— — + — + — 25
Rc
^2 =-
11
Q
\2
^2 1 2 3 6
8 28
\2
In case 2, h ~ R = R
For constant voltage, P = Pocl R-k- UR-HlJ
R R 2;

R^_ > Rc^ ^ Pj < Pj ^ Option (rt) is correct.


J] _ r2R+i'i^ 9
= 2.25=-
For constant current, P = I^R ^ P <x R h R+2 4

^ ^2 ^ Pj > P2 => Option (b) is correct.


or
2R + 1 3
R+2 2
When switches Sj and Sj are closed. Equivalent
resistances are
R = 4n.
5X.1
An 40.
11
^-2 0 0 0 5
1
= 1 + 1 1 1=11=1
^2 1 2 3'*^6" 6 ”1 1 =
6-3
= 1A
3
=e> Ri =- in
^2 2 V.A -6 + lxl-K„=0
B
CURRENT ELECTRICITY (Competition Section) 3.211

41.

0 0 0 1

Using the concept of balanced wheatstone bridge, the resistors of 8 Q and 10 n can be removed. We get the
successive equivalent circuits as follows :
2H
i—VW
2a 2a
2a 1

6a 2a

10 a sO

C> -ijr- 6.5 V 6.5 V


G
c:C> T 6-5 V If)

CM

12 a 4a
4a

w
6.5 V
/ = = 1 A.

Flo
6.5 V

42.

ee
50 50
0 0 0 3 A
^1 = 372 A, /,=

Fr
600
●●i
^ I—VvA AAAr-^
50 50
for
ur
-60 X
H G = 10022-60/1 =100x 600 372
e/2 ' 50 500 (62 -60 100
e/2
B
= 50 = 0.2688 V - 0.27 V.
A' >
ks
P 6 62 6x62 372
Yo
72 cm
28 cm 44. :.;v>
oo

in B 20 /,
With switch S in v/v AW—4
g
eB

Current in primary circuit, / = R position P. After 1 V T


+ n
1 long time, we can 1 pF

replace the capa-


r

P.D. between the points across which secondary citor by open circuit, A
ou
ad

cell is connected.
,=^
3 = 1a
Y

1
^ 3
2 [2 100
nd

g 6
Re

2 (3R 0
— + r
100 ^.B = ^B-^^=2-2xi=|y
Fi

1
L 2

+ —
32^ _ 1562?o C}^ = V^C = ^V^1^F = 1.33^C.
^ 2 100 45. ii.i'r
156x50 3x50
^1 = = 78-75=3Q. With switch S in position Q. After long time, the
100 2
current in the circuit is in B 2n '2
43. 0.27 100 a V/v f -^W—*
60 a /, p ^2
●wv X r ■VAr '22 = -^
3
-^2V
S< 50 V 'T
l^^+2-2x| = ,V6
X
A
I -VW
Q 500 a
2?3 = 312a

2^3 = R3(1 + oAT) = 300(1 + 0.0004 x 100) H = 312 Q ^2 = ^abC = |vx1hF = 0.67^C.
,212 PHYSICS-XII

JEE Main

Based on Ohm's Law, Resistance, Resistiviry, 7. The colour coding on a carbon resistor is shown
Conductance and Conductivity in the given figure. The resistance value of the given
resistor is
1. A current through a wire depends on time as
I = a^t + where = 20 A / s and p = 8 As“^. Find the o- ●o
charge crossed through a section of the wire in 15 s.

{a) 2100 C {h) 11250 C Gold

(c) 2250 C (d) 260 C i! F'-T 2i Red


Green
2. The length of a given cylindrical wire is increased Violet
to double of its original length. The percentage
increase in the resistance of the wire will be (a) (7500 ±750) a (b) (5700 ±375) a

(fl) 200% (b) 100% (c) (5700 ± 285) n {d) (7500 ±375)0
IJFH M.Tin Awj. 2] \
(c) 50% id) 300% ilEE Main 221
8. A 200 Q resistor has a certain colour code. If one
3. An aluminium wire is stretched to make its
length, 0.4% larger. The percentage change in replaces the red colour by green in the code, the new
resistance is resistance will be

(a) 0.4% ib) 0.2% («) 400 O (b) 500 n


(c) 0.8% id) 0.6% .M.iin kiiii’ 22] (c) 300 O (d) icon MuFi April I'-'i

4. Consider four conducting materials copper,


Based on Drift Velocity and
tungsten, mercury and aluminium with resistivity p^,
Mobility of Electrons
Pr'PM respectively.
Then 9. Suppose the drift velocity in a material varied
(^)Pm>Pa >Pc (b) Pc >P^ >Pt- with the applied electric field £ as oc «/£. Then V-I
graph for a wire made of such a material is best given
WP/1 >Pm>Pc i<^) Pa>Pt>Pc by
|!th .Main 2r! («) (b)
V
5, In the following T' refers to current and other
symbols have their usual meaning choose the option
that corresponds to the dimensions of electrical
conductivity :
1 I

(a) (b) IvT^L^T^I


(c) id) jlEE Muiu i6| (c) V id)

6. A conducting wire of length '1area of cross-


section A and electric resistivity p is connected between
the terminals of a battery. A potential difference V is
developed between its ends, causing an electric I

current. lEE jM :i;i l?j

If the length of the wire of the same material is K).

doubled and the area of cross-section is halved, the (A) The drift velocity of electrons decreases with
resultant current would be
the increase in the temperature of conductor.
V^A 3 VA
(n)4 (b)T (B) The drift velocity is inversely proportional to
pi 4 pi the area of cross-section of given conductor.
1 pi 1 W1
(C) The drift velocity does not depend on the
(c)- id)
4 VA 4 pi applied potential difference to the conductor.
[lEE jVI.Mn M.iu-h 211
CURRENT ELECTRICITY (Compoiiii...n h,

(D) The drift velocity of electron is inversely In the light of the above statements, choose the
proportional to the length of the conductor. correct answer from the options given below :
(£) The drift velocity increases with the increase in (fl) Both A and R are true and R is the correct
the temperature of conductor. explanation of A.
Choose the most appropriate answer from the {b) Both A and R are true and R is not the correct
options given below : explanation of A.
(fl) (A) and (6) only (b) (A) and (D) only (c) A is true but R is false,
(c) (B) and (£) only (d) (B) and (C) only (rf) A is false but R is true. ! Ai.ii:

■ The difference in the variation of resistance with


! ! - In a conductor, if the number of conduction temperature in a metal and a semiconductor arises
electrons per unit volume is 8.5xl0^®m”^ and mean essentially due to the difference in the
free time is 25 fs (femto second), its approximate

w
(fl) crystal structure
resistivity is {m^ = 9.1 x 10"^^kg)
(&) type of bonding
(fl) 10"^ Qm (b) lO^Qm
(c) variation of scattering mechanism with temp.
(c) 10'^ Qm (d) 10-®nm

Flo
(d) variation of the number of charge carriers with
temperature. : i! ●

ee
I:. When 5 V potential difference is applied across
'T. The thermistors are usually made of
a wire of length 0.1 m, the drift speed of electrons is

Fr
2.5xl0’^ms"^. If the electron density in the wire is (fl) metals with low temp, coefficient of resistivity.
8x10 m“ , the resistivity of the material is close to (b) metals with high temp, coefficient of resistivity,
(fl) 1.6xl0"®nm {b)1.6xlO~^Om for
(c) metals oxides with high temperature coefficient
ur
(c)1.6xl0^nm (d) 1.6xlO~^Qm n. of resistivity,
'■ '● A current of 5 A passes through a copper (d) semiconducting materials having low
ks
conductor (resistivity = 1.7 x 10"®nm) of radius of temperature coefficient of resistivity.
Yo

cross-section 5 mm. Find the mobility of the charges if


oo

their drift velocity is 1.1 x 10"^ m/s. i'-- The resistance of bulb filament is 100 O at a
eB

(a) 1.3 m^/Vs (b) 1.5 m^/Vs temperature of 100°C. If its temperature coefficient of
(c) 1.8 m^/Vs (d)l.O m^/Vs resistance be 0X)05°C"’, its resistance will become
l i ■ ' *n \ i', 11
200 Q at a temperature of
r
ou

Mobility of electrons in a semiconductor is


ad

(a) 500°C (b) 200° C


defined as the ratio of their drift velocity to the applied (c) 300 °C (d) 400°C
Y

electric field. If, for an «-type semiconductor, the


density of electrons is lO^^m”^ and their mobility is The resistance of a wire is 5 Q at 50°Cand6Q at
1.6 m^ /(V.s), then the resistivity of the semiconductor
nd
Re

(since it is an n-type semiconductor contribution of 100°C. The resistance of the wire at0°C will be
Fi

holes is ignored) is close to (fl)3Q (b)2n


(a) 2 Qm (b) 0.2 Qm (c)lQ (d)4Q ‘ :
(c) 0.4 Qm (d) 4 Qm |i
-H- The temperature dependence of resistances of
Bc;soJ :>ii Temperature- Vana.lion c-’ ilesislivity Cu and undoped Si in the temperature range 300 - 400
^ Given below are two statements: one is labelled K, is best described by
as Assertion A and the other is labelled as Reason R. (a) Linear increase for Cu, exponential increase
for Si.
Assertion A. Alloys such as constantan and
manganin are used in making standard (b) Linear increase for Cu, exponential decrease
resistance coils. for Si.

Reason R. Constantan and manganin have (c) Linear decrease for Cu, linear decrease for Si.
very small value of temperature coefficient of (d) Linear increase for Cu, linear increase for Si.
resistance. 1,1) ' lin ■ 'I
3.21-I PHYSICS-XIl

:i. Statement 1. The temperature dependence of 1=>. The resistance of the series combination of two
resistance is usually given as R = R^il + oAt). The resistances is S. When they are joined in parallel, the
resistance of a wire changes from 100 H to 150 Q when total resistance is P. If S = nP, then the minimum
its temperature is increased from 27° C to 227° C This possible value of n is
implies that a =2.5xl0”^/°C (fl)4 (1j)3 (c)8/9 {d)2 |.\ii i!,.iuj
Statement 2. R = Kq(1 + oAT) is valid only when the 2(». A Copper (Cu) rod of length 25 cm and
change in the temperature AT is small and cross-sectional area 3mm^ is joined with a similar
AR=(R-Rq)« Rq. Aluminium (Al) rod as shown in figure. Find the resis
(a) Statement 1 is true. Statement 2 is false. tance of the combination between the ends A and B.

{b) Statement 1 is true. Statement 2 is true. Statement 2 (Take Resistivity of Copper = 1.7 x lO'^Qm
is the correct explanation of Statement 1. Resistivity of Aluminium = 2.6 x 10“^Om)
(c) Statement1 is true. Statement2 is true. Statement2 (a) 2.170 mO
is not the correct explanation of Statement 1.
(&) 1.420 mO. a Cu
>
(rf) Statement 1 is false. Statement 2 is true. Ao- ■oB
(c) 0.0858 mO
22. Two conductors have the same resistance at 0°C
but their temperature coefficients of resistance are
(d) 0.858 mQ d Al
>
|li l. V|.,[n lnlv 21
and a 2- The respective temperature coefficients of their
series and parallel combinations are nearly 27, In the circuit, the galvanometer G shows zero
+ tt2 Oj +' '^7
a ,,, a, +a. deflection. If the batteries A and B have negligible
(^) internal resistance, the value of the resistor R will be
2

a, -i-a, soon
(c)a, + a2, (rf) ttj + a 2' I—vw <2>
tti + tt2
[.Ml II 111: il'( I I I 1|

23. In an experiment, the resistance of a material is 12V T B R A T 2V

plotted as a function of temperature (in some range).


As shown in the figure, it is a straight line.

In R(T) (<7) 200 Q (b) 100


(c) 500 a {d) 1000 n
\ll l-.l 2iinr'; COSE F 13]
2s. The total current

1/r supplied to the circuit as


shown in figure by the 5 V
One may conclude that
battery is
{a) R(T) = ^0 (b) R{T)=R^e
-rdTg2
C«)l A {b)2A
j2
(c)4A (rf)6A
(c) RiT) = R^e
-T^/T^
id) R{T) = R,e^'^^o {III \I<iin JiiiH' .I2[
111 I I '

2^1.The current /j(in A) flowing through 1Q resistor


Based on Series and Parallel Combinations in the following circuit is IQ
h
of Resistors (a) 0.2 MV in

21. An electric current is passed through a circuit (b) 0.4 “ in '-MVn


containing two wires of the same material, connected (c) 0.25
MV
in parallel. If the lengths and radii of the wires are in
{d)0.5 MV
the ratio of 4/3 and 2/3, tlien the ratio of the currents 2n
passing through the wire will be
1
{a) 3 w- (c)8/9 (d)2 IV
1 Alt l-.l
CURRENT ELECTRICITY (Competition Section) 3.215

30. A cell Sj of emf 6 V and internal resistance 2 O is Based on Internal Resistance,


connected with another cell of 4 V and internal EMF and Terminal P.D. of Cells
resistance 8Q (as shown in the figure). The potential 35. An energy source will supply a constant current
difference across points X and Y is into the load, if its internal resistance is
(a) 3.6 V p X
^2 (fl) zero
(fo) 10.0 V (&) non-zero but less than the resistance of the load
6V,2a 4V,Sn
(c) 5.6 V (c) equal to the resistance of the load
(rf) 2.0 V |[IT M.iiii Fob 2i| (d) very large as compared to the load resistance,
lAlCEl-
31. In the given circuit diagram when the current
3b. In the given circuit, an 15n
reaches steady state in the circuit, the charge on the ideal voltmeter connected rAAArn in
capacitor of capacitance C will be across the 10 n resistance

w
ion
(«) ce 6 reads 2 V. The internal
resistance r, of each cell is
(i,) ce—5— VvV
(rj + r) (n)0.5O {b)1.5n 11-

Flo
(c) on (d)in 1.5 V, 1.5 V
AAV
(c) ce—^ 111 1. Mom AiMil ]Mj
rn,rn

ee
C
(r+r2) ’’2
AAV 37, To verify Ohm's law, a student connects the

Fr
id) voltmeter across the batter}' as, shown in the figure.
(rj+r) The measured voltage is plotted as a function of the
32. The supply voltage to a room is 120 V. The current, and the following graph is obtained :
for
ur
resistance of the lead wires is 6 Q. A 60 W bulb is
already switched on. What is the decrease of voltage
ks
across the bulb, when a 240 W heater is switched on in MVW
Internal
Yo

parallel to the bulb ? resistance

0 Ammeter
oo

(fl) zero volt (b) 2.9 volt


eB

(c) 13.3 volt (d) 10.04 volt llliL M.iin l.'l VA


33. What will be the most suitable combination of
\
three resistors A = 2Q, B = 4n,C = 6n,so that — n
r

1.
I 3 y
ou
ad

is the equivalent resistance of combination ?


Y

(fl) Parallel combination of A and C connected in


series with B. 0
1000 mA 7
nd
Re

(fe) Parallel combination of A and B connected in


If Vq is almost zero, identify the correct statement:
series with C.
(/?) The emf of the battery is 1.5 V and its internal
Fi

(c) Series combination of A and C connected in resistance is 1.5 H


parallel with R (b) Tlie emf of the batterv is 1.5 V and the value of R
(d) Series combination of B and C connected in is 1.5 Q

parallel with A [ll'l- Main Iniu' 22| (c) The value of the resistance K is 1.5
34. A metal wire of resistance 3 Q is elongated to (rf) The potential difference across the battery is
1.5 V when it sends a current of 1000 mA
make a uniform wire of double its previous length.
This new wire is now bent and the ends joined to make jI[ I M.im April 1''|

a circle. If two points on this circle make an angle 60° at 38. Two cells of same emf but different internal
the centre, the equivalent resistance between these two resistances q and are connected in series with a
points will be resistance R. The value of resistance R, for which the
lliT. Main Ai>ril !^)I
potential difference across second cell is zero, is
3 (b)^n 2 (c)ln 2 {d)^n o (fl) rj -rj (&) fj {C)r^ {d) [ll i Mam ..2I
^.2/6 PHYSICS-XIl

'''●A 10 V battery with internal resistance 1Q and a -i5. Two equal resistances when connected in series
15 V battery with internal resistance 0.6 Q are to a battery, consume electric power of 60 W. If these
connected in parallel to a resistances are now connected in parallel combination
10 V
voltmeter as shown in the 1 fl
to the same battery, the electric power consumed
I—vw will be
figure. The reading in the
voltmeter will be close to 15 V (fl) 60 W (b) 30 W
0.6 n
(fl) 11.9 V I Wr (c) 120 W (d) 240 W
(b) 13.1 V ! A heater coil is cut into two equal parts and only
(c) 12.5 V one part is now used in the heater. The heat generated
<Y> will now be
(d) 24.5 V ’ i; ■

(fl) doubled (6) four times


Based on Heating tzffed of Flectr;-: C i:i re 111
(c) one fourth (d) halved
P!
iric ''0"'er 'jnc' Flociric iTioru'-
A wire, when connected to 220 V mains supply,
* ■ The resistance of hot tungsten filament is about has power dissipation Py Now the wire is cut into two
10 times the cold resistance. What will be the resistance equal pieces which are connected in parallel to the
of 100 W - 200 V lamp, when not in use ? same supply. Power dissipation in this case is P2- Then
(fl)40n {b)20Q P^-.P^is
(c)400n (d)2n (fl)l (&)4 (c)2 (d)3
i I A resistor develops 500 J of thermal energy in
* Two electric bulbs, rated at (25 W, 220 V) and
20 s when a current of 1.5 A is passed through it. If the (100 W, 220 V), are connected in series across a 220
currentis increased from 1.5 A to 3 A, what will be the
voltage source. If the 25 W and 100 W bulbs draw
energy developed in 20 s ?
powers P^ and P^, then
(fl) 2000 J (b) 1000 J
(c) 1500 J id) 500 J |l \ K :U I (fl) Pj =9 W, ?2 = 16W (b) Pj =4 W, ?2=16 W
d. A current of 2 mA was passed through an (c) P, = 16 W, P2 = 4 W (d) Pj = 16 W, P2 = 9 W
unknown resistor which dissipated a power of 4.4 W. [li .1 M.iin Jan i"|
Dissipated power when an ideal power supply of 11 V
is connected across it is An electric power line, having a total resistance
of 2 n, delivers IkW at 220 V. The efficiency of the
(fl)llxl0“'‘’w (i?)llxl0“'^W transmission line is approximately
(c)llxlO^W (rf)llxl0“^W (fl) 85% (b) 96%
● ’ A 2 W carbon resistor is colour coded with green, (c) 72% id) 91% [II'.I: Main j'l.
black, red and brown respectively. The maximum
current which can be passed through this resistor is In a large building, there are 15 bulbs of 40 W,
5 bulbs of 100 W, 5 fans of 80 W and 1 heater of 1 kW.
(fl) 20 mA ib) 0.4 mA
The voltage of the electric mains is 220 V. The minimum
(c) 100 mA id) 63 mA
capacity of the main fuse of the building will be
'1- The resistive network shown below is connected
(fl) 8 A ib) 10 A
to a D.C. source of 16 V. The power consumed by the
network is 4 watt. The value of R is (c) 12 A id) 14 A
!: :

4R 6R ' ● In a building there are 15 bulbs of 45 W, 15 bulbs


r-AM^ R r^/W-i R of 100 W, 15 small fans of 10 W and 2 heaters of 1 kW.
4R AW 12R ■wv The voltage of electric mains is 220 V. The minimum
WV—* L^WV-' fuse capacity (rated value) of the building will be
(fl) 15 A ib) 10 A
1
(c) 25 A id) 20 A
e = i6v
'- In the given figure, the emf of the cell is 2.2 V
(fl)8f2 ib)in and its internal resistance is 0.6 Q. Calculate the power
(c)16n (d)6Q i' !■ dissipated in the whole circuit.
CURRENT ELECTRICITY (Competition Section) 3,217

4Q
In the above circuit the current in each resistance is ●'

A («)1 A {b) 0.25 A


(c) 0.5 A (rf)OA [JEE Main 17]

57. In the given circuit the cells have zero internal


8n
resistance. The currents (in amperes) passing through
resistances and respectively, are
2.2Y r=0.6n

(a) 1.32 W (b) 0.65 W


R\ 20 n Rj 20 Cl
(c) 2.2 W (d) 4.4 W [JFE-Main Aug 21]
+ +
53. One kg of water, at 20°C, is heated in an electric

w
lOV 10 V
kettle whose heating element has a mean (temperature
averaged) resistance of 20 Q. The rms voltage in the (a) 1, 2 (b) 0,1
mains is 200 V. Ignoring heat loss from the kettle, time

Flo
(c) 0.5, 0 id) 2, 2 [JEE Main 19]
taken for water to evaporate fully, is close to
58. In the given circuit diagram, a wire is joining
[Specific heat of water =4200 J/(kg“C),

ee
points Band D. The current in this wire is
Latent heat of water =2260 kJA:g] in
B
2C1

Fr
r*^AW-t-VvV
(a) 16 minutes (b) 3 minutes A C
(c) 22 minutes (d) 10 minutes 4Q 3C1
Wv—'
for
ur
fJEE Main April 19] D

54. A battery of 3.0 V is connected to a resistor


dissipating 0.5 W of power. If the terminal voltage of
ks
the battery is 2.5 V, the power dissipated within the 20 V
Yo

internal resistance is (fl)4 A (b) 0.4 A


oo

(a) 0.10 W (b) 0.072 W (c)2A (rf) zero [JEE Main Jan 20]
eB

(c) 0.50 W (d) 0.125 W 59. In the circuit showm, the potential difference
[JEE Main Sept. 20] between A and 6 is
IQ 1 V
Based on Grouping of Cells and
r

M AMr
ou
ad

Point Potential Method


5Q IQ 2V lOQ
Y

55. In the electric network shown, when no current AO—Wv AAA, L


D C
flows through the 4fl resistor in the arm EB the IQ 3V

potential difference between the points A and D will


nd

N AAA
Re

be
F 2Q £ D (/7) 6 V (1;)3V
Fi

I—
(fl)6V 2V
(c)2V (d) 1 V [JEE Main Jan 19]
(&)3V 4Q
60.For the circuit shown, with Rj =1.0 Q, R2 = 2.0 Cl,
2Q R
(c)5V S, =2V and 62 =€3 the potential difference
4V
(d)4 V between the points 'a' and 'b' is approximately (in V)
[JEl Main 15) 1 R1 R1
A B C
9V 3V VA AW
56. '3

2V 2V 2V 6 I
R1
1 h 1
AW
IQ IQ 1 Q R, b

1 1 (a) 2.7 ib) 3.7


2V 2V 2V (c) 2.3 (rf) 3.3 (JEE Main Jan 19]
3.218 PHYSICS-XII

61. A cell of internal resistance r drives current 66.

through an external resistance i^.The power delivered 20 n 5Q

by the cell to the external resistance will be maximum VA»—t—


when
140V_=. 6Q 90 V

{a) R = 1000r (b) R=r


(c) R=2r (d) R = 0.001r ii;i; \]m<-

62. Determine the charge on the capacitor in the The value of current in the 6 Q resistance is

following circuit: (fl)8A (b) 10 A


60 20
(c)6A {d)4 A
■WSr
67. bi the given circuit diagram, the currents,
T 72V 40 10 O 10 nF /j =0.3 A,/^ =0.8Aand Ig =0.4 A, are flowing as shown.
Pi—►—Wv—^
^3
(a) 200 pC (b) 60 pC
(c) 10 pC (rf)2pC IJEEM.iin April I'';

63. In the figure shown, the current in the 10 V -I 1

battery is close to ^ A WV- R


50
The currents I2, and Ig, respectively are
100 ■z^lOV
(fl) 1.1 A, 0.4 A, 0.4 A
(l?)-0.4 A, 0.4 A, 1.1 A
L-jl—VA—*—Wv
20 40
(c) 1.1 A,-0.4 A, 0.4 A
20 V
(d)l.l A,-0.4 A, 0.4 A II'! \l.iit, l,ti. >'

(rt) 0.36 A from negative to positive terminal


68. In the circuit, given in the figure, currents in
(b) 0.71 A from positive to negative terminal different branches and value of one resistor are shown.
(c) 0.21 A from positive to negative terminal 2V
D
(d) 0.42 A from positive to negative terminal lA"
r VA I h B

llli- M.iin :i']


20

Based on Kirchhoff's Laws


64. The Kirchhoff's first law (Z/ =0) and second A C 2A
IV

law (ZfR = Z&), are respectively based on


Then potential at point Bwith respect to the point
(fl) conservation of momentum, conservation of charge A is

(fc) conservation of charge, conservation of energy (fl)+1V (&) -2V


(c) conservation of charge, conservation of momentum (c) +2 V (d) -IV
(d) conservation of energy, conservation of charge 69. The value
l.AllUA.-
of (Hjrrent /1
65. In the circuit shown, the current in the Ifl flowing from A
resistor is to C in the circuit i

(fl) 1.3 A, from P to Q 6 V


p 2n diagram is
{b) 0 A
I r-VvVn (fl)l A
(c) 0.13 A, from Q to P (6) 4 A
in 9V
(d) 0.13 A, from P to Q (c)5A
3n 3Q
[JEr M.iin 13) UWSr AAV-i (d)2A
Q Ml I M.iin
CURRENT ELECTRICITY (Competition Section) 3.219

70. When the switch S, in the circuit shown, is 74. A potentiometer wire AB having length L and
closed then the value of current i will be resistance 12 r is joined to a cell D of emf € and internal
g
resistance r. A cell C having emf — and internal
20 V 1 h 10 V
-o
A 2C1 ” 4£i B resistance 3r is connected. The length A] at which the
2n
galvanometer as shown in the figure shows no
deflection is
s

1 V=0 +

D
J
(a) 3 A (b)5A A B

C
(c)4A id) 2 A ||!:H Main J.in n'l

w
<G)J
+
h
Based on Potentiometer i,3r
2
71. In a potentiometer experiment, the balancing

Flo
11
with a cell is at length 240 cm. On shunting the cell («) ii’)
12 24
with a resistance of 2 Q, the balancing length becomes

ee
120 cm. The internal resistance of the cell is
(c)^L {d)^l

Fr
12 24 IIEE M.iin (an 1-'|
ia)in (b)0-5Q
(c) 4 0 {d)2Q iAlEKh 2(m!5| 75. An ideal battery of 4 V and resistance R are
72. In the given potentiometer circuit arrangement, connected in series in the primary circuit of a
for
ur
the balancing length AC is measured to be 250 cm. potentiometer of length 1 m and resistance 5 O. Tlie
When the galvanometer connection is shifted from value of R, to give a potential difference of 5 mV across
point (1) to point (2) in the given diagram, the 10 cm of potentiometer wire, is
ks
balancing length becomes 400 cm. The ratio of the emf (a) 490 n (&) 480 O
Yo
oo


of two cells, is (c) 395 (rf) 495 O [JEEMainJnn19]
^2
eB

7(S. In the circuit shown, a four-wire potentiometer


<●} I- is made of a 400 cm long wire, which extends between
K ..

V
A and B. The resistance per unit length for the potentio
r

C
A' B meter wire is r= 0.01 O/cm. If an ideal voltmeter is
ou
ad

1.5 V, 1.5 ,V
Y

0.5 a 0.5 Q
(D 0) \ I
Ar^'
0
I
50 cm
nd

\
Re

1 Q
Fi

8 {}
B' 1(X) cm

connected as shown with jockey / at 50 cm from end A,


ill ! Main liil;.- 21!
the expected reading of the voltmeter will be
73. The given potentiometer has its wire of (fl) 0.20 V (b) 0.25 V
resistance 10 Q. When the sliding contact is in the
(c) 0.75 V (rf) 0.50 V [ICE Main April 191
middle of the potentiometer wire, the potential drop
across 2 resistor is 20 V
77. The length of a potentiometer wire is 1200 cm
and it carries a current of 60 mA. For a cell of emf 5 V
(fl) 10 V (b) 5 V and internal resistance of 20D, the null point on it is
40 40 found to be at 1000 cm. Tlie resistance of whole wire is
(c) — V (d)—V
A ●WAV B
4 k

9 11
2Q (fi) 60 n (b) 120 a
I.. \.i ain Juh' VAr (c) 80 a (d) 100 Q IJEE Main Jan 20)
3.220 PHYSICS-XII

78. A potentiometer wire PQ of Im length is 81. The current i drawn from the 5 V source will be

connected to a standard cell Another cell E2 of emf ion

1.02 V is connected with a resistance 'r' and switch

S (as shown in the figure). With switch S open, the null /t


sn ion ^ 20 n D
^-vw
B
AAAr AA/V

ion
1-.
AVv
} I-
Q
I 5V
r

e, rAVSn Og (rt) 0.67 A (b) 0.17 A


1
(c) 0.33 A (d) 0.5 A (AIEEE 20l)f.l
S 82. In a Wheatstone bridge, three resistances P, Q
and R are connected in the three arms and the fourth arm
position is obtained at a distance of 49 cm from Q. The
potential gradient in the potentiometer wire is is formed by two resistances and $2 cormected in
parallel.
(fl) 0.04 V/cm ib) 0.01 V/cm
The condition for the bridge to be balanced will be
(c) 0.02 V/cm (d) 0.03 V/cm R
IfEE Main Sept. 20] {b)- =
Q 2S,S2 Q S, + S2
Based on Wheatstone Bridge 2R P_R(S, + S2)
and Metre Bridge (c)- = (rf)
Q Sj + S2 Q s,s.
79. Which of the following statements is false ? (AIHEE 2006]
(a) Wheatstone bridge is most sensitive when all 83. In a metre bridge experiment, null point is
the four resistances are of the same order of
obtained at 20 cm from one end of the wire when
magnitude resistance X is balanced against another resistance Y. If
(b) In a balanced wheatstone bridge if the cell and X < y, then where will be the new position of the null
the galvanometer are exchanged, the null point point from the same end, if one decides to balance a
is disturbed resistance of 4X against Y ?
(c) A rheostat can be used as a potential divider («) 50 cm (b) 80 cm
(rf) Kirchhoffs second law represents energy (c) 40 cm (d) 70 cm (AIEEE 2004]
conservation [JEEMnin 17] 84. On interchanging the resistances, the balance
80. The Wheatstone point of a metre bridge shifts to the left by 10 cm. The
resistance of their series combination is IkO. How
bridge shown in the
much was the resistance on the left slot before
figure here, gets
balanced when the interchanging the resistance ?
carbon resistor used as (fl) 990 O (b) 5050
R-^ has the colour code (c) 550 O (d) 910 0 [JEE Main
(Orange, Red, Brown). 85. In a Wheatstone bridge (see figure), resistances
+
The resistors R2 and R^ P and Q are approximately equal. When R - 400 O, the
are 800 and 400, bridge is balanced. On interchanging P and Q, the
respectively. value of R, for balance is 4050.
Assuming the colour code for the carbon resistors The value of Q is close to
gives their accurate values, the colour code for the (a) 403.5 O
carbon resistor used as R^ would be ((;) 404.5 O
(fl) Brown, Blue, Black (()) Red, Green, Brown
(c) 401.5 0
(c) Grey, Black, Brown (d) 402.5 O
(rf) Brown, Blue, Brown IJEE Main Jan 19] IJEE Main Jan 19]
CURRENT ELECTRICITY (Competition Section) 3.221

86. The resistance of the metre bridge AB in the 90. In the given circuit ‘a' is an arbitrary constant,
given figure is 4Q. With a cell of emf £=0.5 V and The value of m for which the equivalent circuit
rheostat resistance R.,« =2 Q, the null ^point is obtained . ●
resistance IS minimum, will be
, ,:ii P
V2
6
h The value of .y is [JEE Main June 221
t
ma

-wv a/m
!
Ilia
WV
—wv a/m
Wv
Rh AAAr
6V ma

A/W
at some point J. When the cell is replaced by another

w
one of emf £ = 62 th® same null point / is found for h
=6f^. The emf £j is
(fl) 0.6 V (&) 0.5 V 91. In a metre bridge experiment S is a standard

Flo
(c) 0.3 V (rf) 0.4 V (JEE Main Jan 19| resistance. R is a resistance wire. It is found that
87. The four arms of a Wheatstone bridge have balancing length is / =25 cm. If R is replaced by a wire

ee
resistances as shown in the figure. A galvanometer of of half length and half diameter that of R of same

Fr
material, then the balancing distance /' (in cm) will now
be

for
ur
R
S

] [
ks
Yo
oo

I I
eB

10 V
I-
15 Q resistance is connected across BD. Calculate tlie
r

current through the galvanometer when a potential [JEK M.iin Jan 201
ou
ad

difference of lOV is maintained across AC.


92. A metre bridge setup is shown in the figure. It is
(a) 2.44 pA {b) 4.87 pA
Y

used to determine an unknown resistance R using a


(c) 2.44 mA {d) 4.87 mA given resistor of 15 H. The galvanometer (G) shows null
[JEE Main Mnnli 211 deflection when tapping key is at 43 cm mark from end
nd
Re

A. If the end correction for end A is 2 cm, then the


Numerical Value Type Questions
Fi

determined .value of R will be n.


88. The series combination of two batteries, both of
the same emf 10 V, but different internal resistance of
5V
20 n and 5 Cl, is connected to the parallel combination !●
of two resistors 30 Q and R n. The voltage difference K

across the battery of internal resistance 20 H is zero, the } B


A
value of R (in Q) is

5
IJEE Main Jan 20]

89. The balancing length for a cell is 560 cm in a


potentiometer experiment. When an external
resistance of lOO is connected in parallel to the cell, the AMr Wv
balancing length changes by 60 cm. If the internal 15 n R

N
resistance of the cell is —H, where N is an integer, then [JEE Main June 221
10
value of N is IJEE Main Jan 20|
3.222 PHYSICS-XII

Answers and Explanations


1. {h) i = + =20t + 8t^ 9. (/I) VjXyfE
15
I - enAv^ Iccv^ I QC VV
7 = dq^\idt= \{20t+8t^)dt fccV
0
15
Hence, V-I graph is a parabola symmetric about
V-axis.
I 3 Jo
10. (h)
= 10(15^-0^) + -(15^-0^)
3
eEi eVj
V
d ~
m ml
= 2250 + 9000=11250 C.
As temperature increases, relaxation time t decreases
2. (i/i When the length is increased to 2/, its area of
and so Vj decreases. So statement A is correct.
cross-section decreases io A12.
As Uj cc -, statement D is correct.
New resistance. d I'
/' 21 I
K' =p— =p—^ =4p — = 4R 11. [,/)
^ A' ^ AI2 ^A m

Change in resistance, ’’ nA
R’-R 9.1x10
-31
xl00 = X100 = 300%. Qm
R R -15
8.5 X 10“ X (1.6 X lO"'^)^ X 25 X10
3. (c) V = IA = constant => A x- 9.1
I X10"^= 1.67 X 10'®Qm.
8.5x2.56x25
I
K =p => Rx/2
A
12. f.'i V = IR = enAv^ x t-
A
AR M
X100 = 2 —X100 = 2 X 0.4% = 0.8%. 5
R I P = -19
envj 1.6x10 x8xl0^®x2.4xl0^x0.1
4. I,!) Pm>Pt>Pa>Pc- = 1.6xl0"^nm.
5. in) j = cE 13. (,;i
l^L-' V
[CJ] = L-
d _
=> [a] = Mobility, p =
E M^L’T'^ E pj pi
liji
l.lxl0~^xTtx(5xl0~^)^ m^/Vs
6. i./i New resistance. R' =
p(2/)_4pl 1.7xl0”°x5
8

A/2 A
11x3.14x5
y 1VA xlO'V^/Vs
Resultant current, /= — 17
R' 4 pi
= 10.15 X 10"^ -1.0 m^/Vs.
7. I,;)

Violet Green Red Gold 14. iO I = gE = nev^


i i 4-
^d
7 5 2 ±5% a = ne— = neu.„
E

1 _ 1 _ 1
R=75xlO^Q±5% P =
= (7500 ± 375) n. o ~ nep ^ ~ 10 X 1.6 X10"^^ x 1.6
1 = 0.4 Qm.
8. (/-) 200 0=20x10 ^ Red, Black, Brown
15. t..') As the alloys have very small value of
Green, Black, Brown
1
temperature coefficient of resistance, their resistance
50xl0‘ =500 0.
remains constant over wide ranges of temperature.
CURRENT ELECTRiOTY (Competition Section)
In R{7) ‘ ‘
16. .In metals, the number of density of charge 23. (|i) Using intercept
form of a line, in R(To) .
carriers (tree electrons) is independent of temperature
while it increases exponentially with temperature in ^ +^
- y = 11
case of semiconductors. a b

17. . ;A thermistor is made of metal oxides with


= 1 h 1/T^
high temperature coefficient of resistivity. 0

18. 1.’
_ ^2“^! 200-100 2'l
Z0
^2 R,1 a 100x0 005 In R(T) = In R(To) 1- T

= 200 + fj = 200 +100 = 300° C.


19. Rf - Rq = a Ryf => In R(T)-In R(To) =-In R(To);^
T,?0
R(T)

w
and R
100 -Ro = aRQxlOO = R(7o)f
^(^o)
On dividing, we get
0

Flo
^ ^ _ 50 KCr) = [R(To)feT2
●^100 ^ j-2

ee
2

or ^ ^=- or 10-2R^|=6-Ro R= R„e


0
.

Fr
6-Rq 2 n2

or
R, = 4Q. 24. i;>)
R^ I2 \ ^2 h J\J
for
ur
20. 1' .The resistance of conductor (Cu) increases
\2
linearly and that of semiconductor (Si) decreases = —x
4 (3 =3
exponentially with the increase in the temperature 3 I 2)
ks
from 300 K to 400 K. For wires connected in parallel,
Yo

or ^R-\~^2^2
oo

21. iAs relation R = Ry(l+oAT) is valid only ^1=^2


when AR « Rq, so statement -1 is false and statement I
R2 _ 1
eB

1 _
or
2 is true.
^2 R
1
3

22..: R^ = R^ + R2 25. (,;| In series. R^ + R2 = S


r

_^ = P
ou

dR^ _ dRi dR^


ad

In parallel ' +
dT dT
Y

Ra^ = Rjtt^ + R^ttj But S = nP

Ri=R2 R,R2
nd

As
Re

Ri + R2 = n
Rj + R2
R — R| + R2 — 2 Rj
Fi

For n to be minimum, R^ = R^ = R (say)


*^2 RR
a
^ “ 2 .-. R+R = n or 4R^ = nR^ or n=4.
R+R
J_ = J_ J_ 26. (,-'i
Rp “ Rl R2 _ -^U ^A1 - I PCuPaI
R
1 dRp 1 dR^ 1 dR^
eq
+ '^‘PCu+PaI
dT -16
_25xl0~^ 1.7x2.6x10 Q X

” 3x10-^
-8
4.3x10
a
JL - ^1 . ^2
25x1.7x2.6
Rp R 1 xl0“^Q
3x4.3
a
ttj + tt2
= 8.57 xl0"‘‘n = 0.857 mH.
3.224 PHYSICS-XII

27. (i?) As no current flows through G, the potential 31. (c) In the steady state, no current will flow
difference across R = 2 V. Hencs current through R will be through the capacitor.
2 t
/ = — i ~
g r
R
r+^
12 I
Also, 1 = P.D. across C
500+ R
= P.D. across AB
2 12 c
Q . gr^ ''a
K 500+R -
c
= zr2 = wv—● s
r+ ^
or 1000 + 2R = 12K or R=100Q.
Q = cg-1-
28. (1;) The equivalent circuits are shown below. r+ r2
120x120
32. (d) Resistance of bulb = 240 0
60

5n 5fi 120x120
Resistance of heater = = 60 0
5 240
5Q = T 5V 2.5 a

240x60
2.5 n Resistance of heater and bulb = = 480
240 + 60

J =
5 V
= 2 A.
1
2.5 0 240 n 6n

Case 1
29. in)
120 V
0.5 a 2a 2.5 X 2 10
aAA VW 4.5
240 a

Wv "I
VW—1
2a
6a 4sa 6a

h WV
Case 2
60 a
IV

y 1x9
/ = = 0.9 A 120 V 120 V
R 10
i-q

I 0.9 In case 1, voltage across bulb is


h“ 4.5
x2 =
4.5
x2 =
5 240
^1 = 246 xl20=117.073V
Current through each 10 resistance,
In case 2, voltage across bulb is
i=lA = 0.2 A.
^^2 5 48
1^2 = —X120
54
= 106.66 V
30. (c)
^1-^2 _6-4 Decrease of voltage across the bulb,
1 = = 0.2 A
r^ + r2 2+8 - ^2 = 117.073 - 106.66 ^ 10.04 V.
33. (^) Parallel combination of A and B should be
As the current inside
e1 ^2 connected in series with C.
the cell €2 flows from +ve P X

to -ve terminal, so R
6 V2a 4V, 8 a
eq + R^
/' ■ ^A^^B
2x4 4 22
= 4 + 0.2x 8=5.6 V. + 6=-+6= —a
2 +4 3 3
CURRENT ELECTRICITY (Competition Section) 3.225

34. ia) e 26
I
K =p = 3n R+r^ + ty
A

21
=>
jR + Tj + 7^ = 2 r2
R'=p = 4R=12Q.
AI2
60 n
39. (r) The two cells are connected in opposition, so
^1 = 360 xl2 =2Q
the net current in the circuit is

R^=U -2=10 0 1 = iLiia. = -A - A


2x10 5 r^+T2 0.6 + 1 1.6 8
R =-o.
eq
2+10 3
Reading of the voltmeter
35. (il) When r » R, current = Terminal p.d. across either cell

w
25
8 8
/ = — = constant = 8j-/rj =15-—X
8
0.6
r+ R r
= 15-1.875=13.125 V.

Flo
36. (a)
15x10 200x200
= 6Q 2Q 40. (c) R = = 400 O.

ee
25
-wv VvV P 100

Fr
41. (<0 H = I^Rt
r r For same R and t, H <x

for
ur
1.5 V 1.5 V

2V
H
1
/?1
1 = 1a
60 3 3^
ks
^2 = 2 x 500J = 2000 J.
(1.5)
Yo

2x1.5
R.. =8 + 2r = =9 r = 0.5O.
oo

1/3 P 4.4
42. (^0 R = ^
eB

37. (/7) When J=0, g=V = 1.5V 2xl0“^x2xl0


When voltmeter reading is zero, = 11x10^0
Ko=g-7r = 0 When connected across 11 V supply,
r
ou
ad

8_ 1.5 V _1.5V F = _ rixii = llxl0"^W.


r = = 1.5 O.
I 1000 mA lA R 11x10^
Y

43. i.n
38.{a)
Green Black Red Brown
nd

6 r, 8
Re

I—Wy I—WSr i i i i
Fi

5 0 2 ±1%
/' ■

R = 50 X 10^0 ±1%
R P 2 1
7 = A = 20 mA.
Current in the circuit is R 50x10^ 50
28 4Rx4R 6Rxl2R
7 = 44. {,!) R - ●+ R 4- + R
^9
R + + r2 4R + 4R 18R

=2R+R+4R+R=8R
P.D. across the second cell,

K=6-7r2=0 P =
16x16
=4 W
8 R 8R
7 = e<7

^2
R = 80.
3.226 PHYSICS-Xll

1/2 51. .. Total power consumption


45. . : 1 In series combination, = =60W
2R = 15x45 + 15x100 + 15x10 + 2x10^ =W
In parallel combination. 1 =
4325 _ 4325 = 19.66 A
1/2 V ~ 220
R/2
= 4Pg =4x60 =240 W. Required fuse capacity =● 20 A.
y2 y2 52. The equivalent circuit is
46. (fOHj = R
f, ^2 = R/2
t
4Q

H, vw
H1
=2 or
H,=2H,. nn
VW B
A
6Q
y2 wv
47. (/j) Pj = R 8Q ' ■!
I-.
wv
When the wire is cut into two equal pieces, the resis
tance of each part becomes R/2. Effective resistance
when the two parts are connected in parallel will be 2.2 V, r = 0.6 n

2 2 _ ^ 1 = —+ 1 1 1^1^15
K + -S 4 4 12 6 8 24
2 2
24
y2
Pz = R
= 4P1 or A = 4. R
^
= —= i.6n
15
4
Pi
R total = 1.6 + 0.6=2.2 0

48. (c) ● R, = —
y2 _ (220)2 = 1936 0 P =
y2 2.2 X 2.2
W = 2.2 W.
’ P 25 R 2.2
total

_ (220)2 = 484 0 y.2..j


^2 = 100 53. I. Q=Pxt^
rms

/ = y2 ^ 220 _ 1 ^ Q = mcAT + ml
Rj + R2 2420 11
/
1
x2 = lx 4200 X 80 + lx 2260 x 10^
20
P^ = fR^=—
11;
xl936=16W
2596x10^x20
t = s = 1298 s c:^22 min.
( 1 f x484=4 4x10^
P,= f2i^ = 11
W.

54. ,, P„R = /2r = o.5W


R
P IQOOW _50 -V0r
49. (10 y„R = Ri =2.5 V
y “ 200V “ 11
0.5 W
T 5012 5000 / = = 0.2 A
!—WV-*
Power loss = 'fR = — x2W= W 2.5 V
11 121 r
e
1000
VnR =t-Ir
Efficiency = X100%
5000 2.5 =3-0.2 r
1000 +
121
r = 2.5a
1 121
5
X 100% =
126
X100%-96%.
P^ = fr = (0.2 )2 X 2.5 W = 0.10 W.
1 +
121 55. Let Vq = 0. Then y£=0
50. u ) Total power,
As no current flows through the 4fl resistor.
P = 15x40 +5x100 +5x80 + 1x1000 = 2500 W
y, = -4V
^ P ^ 2500 = 11.36 A
" y “ 220 V^=9-4 = 5V
Hence, the minimum capacity of the main fuse
should be 12 A.
Hence, y^-yp=5V.
CURRENT ELECTRICITY (Competition Section) 3.227

56. (d) If we take potential at point A equal to 0 volt, 60. lil) Three cells are connected in parallel between
then the potentials at other points will be as shown in points a and b.
the figure.
1
6V 4V 2V ov
y =
2 Rj R^ 2 R^
1 1 1
— + +
2R
1 R2 2R2
2 4 4
I- K
_ 2 2 2 -5 10
6V 4V 2V OV — = 3.3 V.

P.D. across each resistance = 0 V l+l + .l 3/2 3


2 2 2
Current through each resistance = 0 A. 61. (,' iMaximum power is delivered by the cell to
57. (.-) the external resistance when

w
ov OV ov external resistance R = internal resistance r.
4x12
62. !wi R^=6 + =90.
20 n ^2<2on

Flo
R1 4 + 12

I 72 V
h - .'2 / = = 8 A

ee
h 90
ov 10 V ov
10 V lOV

Fr
We use point potential method.
10
/j = 20
—A = 0.5 A 72V
8A
wv
24V 2A 2Q 20 V

for
ur
6a ” 6 A
7, = 0.
72 V T loa 10 ^iF
4a
58.
ks
41 31
Yo
0 0
j la B 5 2a 0 0
oo

A -1 C £/ = CV = 10 pFx 20 V = 200 pC
eB

40 5 30

1..
»~wv D2I
^ ».wv-* 63. (c)
I
I
5 5 »
1
r

20 V 10 V g 10 V
ou
ad

20 V 10a
70 40 R 40
Y

1x4 2x3
R = 2D
5 5
e 20x10 _ 200
nd

20 V
Re

/ = _=10A cq 17 ~ 17
2
Fi

I
I
-=2 A. R 7x10^70^
BD-g cq 17 ~17
200
59. (,n Potential difference across AB will be equal -10
to battery equivalent across CD. 1 = IZ
70
4+'--
17
- - ^2 >3 30
^.4B = v:CD 1 1 1
= 0.21 A.
— + — + 138
"1 ^2 ^3 Current flows from positive to negative terminal of
1 2 3 10 V battery.
_ 1 1 1 _ -=2
6 V. 64. i; ! Kirchhoff's first law is based on the
-1,1,1-3 conservation of charge and second law is based on the
1 1 1
conservation of energy.
■ V

. 3.228 PHYSICS-XII

65. (c) Let X be the p.d. across 1 resistor from Q to 69. (fl) =
P. Then, =4 + 4=8fi

x-6 a:-0 :c + 9 8
= 0 z
,● _ ’AC _ -=1 A.
1 “
3 1 5 R 8
AC

or X
rS + 15 + 3^ = 2--
9 70. ib)
15 5 5 20 V 'i V 10 V
o-

2fi 4n
or = i 11=1.
^ 5 ^ 23 23 2fi

/ = x-0 ^ 3 = 0.13 A, from Q to P.


I

1 "23
’#●0 V
66. (b)
140 V 20 n V 5Q
90 V
By KCL, i^ + i^- I
W/ p 20-V 10-V V^-0
6H 2 4 2

V=10V
0 0 0 . 10
t = = 5 A.
2
Sum of currents away from junction P is zero.
V-140 V-90 V-0 71. (d) Internal resistance of the cell is
= 0
20 5 6
r= R
1,-1, =2
240-120 ^ = 20.
31^-420 + 12^-1080 + 10^ = 0
h 120

2Sy= 1500
72. (/z) g, X 250
V^ = 60 V
6^+63 ==^400
60-0
Current in 6 O resistance =
6
= 10 A. ^1+^2 _ 400 _8
g1 250 "5
67. ((Z) We use Kirchhoff's current law.
3 g i_5
At junction S, 1+TT^=1 + -
^1 5
g2 3
^4 “ ^3 ^5
73. (C-)
7g = J^-/^=0.8-0.4 = 0.4 A 20 V 20 V

At junction R, I-

h
5n 5Cl
l2 = J^-/i=0.8-(-0.3) = l.lA Vs/V ■Wv—VW
5C1
At junction Q, 2Q
7

10
0.4+/g = 1.1-0.3 y. Z—x20V = 10x20 ^40 V.
20 10 45 " 9
J, = 0.4 A. + 5
7
68. (fz) 2V
£ D 74. (</) y^, = IR /ij
Using KVL along -VW I- B

ACDB, 20 6 g
x-xl2r
2 12r+r L
y^ +1+1X2-2 = yg 1 A
lA
13L
yB
*■ AM—'F
y^ =3-2 =1V.
X =
A C 2A
1 V
24
CURRENT ELECTRICITY (Competition Section) 3229 /

75.(Cl 79. (/’) In a balanced Wheatstone bridge, the null


4 V
R
point remains unchanged even if cell and galvano
I—Wy meter are interchanged.
V ’ .J 80. (i/) Rj (Orange, Red, Brown) =32x10^ =320n
10 mV
Ao
10cm
■6 B For balanced bridge,
/
R
1 m 1 _ ^3
V., =5mV=5xlO‘^V R, R,
Al
320x40
R — xl0=0.5Q = 160 0
AJ - 100 80

V 5x10"^ Colour code for R^ = Brown, Blue, Brown.

w
1 = Al _ = 10‘^A
R
A}
0.5 81. (f/) The equivalent circuit is shown in the figure.
4 4
Also, 1 =

Flo
R+R R+5
AB

— = 10“^

ee
R +5

Fr
R + 5 = 400

R = 395 0.
I

for
ur
76. (b) R^g =400x0.01 = 40
3V
/ = = 0.5 A 5V
(l + l + 4)0
ks
5 0 _10O
Yo

As
Voltmeter reading, lOO "2^
oo

V., = /R„ = 0.5 X 50 X 0.01 V= 0.25 V.


A/ AJ
The Wheatstone bridge is balanced. The 10 O
eB

resistance in arm BC is ineffective.


77. (<f) Potential gradient
5V Resistance along ABD = 5 +10 = 15 O
r

1000 cm 1200 cm
ou

Resistance along ACD = 10 + 20 = 30 O


ad

Sp=6V The effective resistance is


Y

6V 15x30
R = 1000 R = = 100
AB ~
nd

I 60xl0'^A 15 + 30
Re

5V
Fi

Current, 1 = = 0.5 A.
1 100

1000 cm
82. ((/) Equivalent resistance of the parallel
A'
60 mA
B combination of and $2 is
1200 cm

+
I \AAr Si + %
5V 20

For the balanced Wheatstone bridge,


78. li) Balancing length is measured from P.
_P_R_ R(Sj + S2)
/ = 100-49 = 51cm
Q"s
Potential gradient.
83. (.M In a balanced metre bridge.
fc = €2 _1-02V
R I
= 0.02 V/cm.
} 51 cm S 100-1
3.230 PHYSICS-XII

In first caseR = X, S = Y, /=20cm 87. I,/)

X _ 20 _ X
y"l00-20 ~4
In second case : R = 4X, S = Y, /' = ?
10 V ov

4x _ r or 4x 1 _ r
y " 100-/' 4 100-/'

● or /'=100-/' or /’ = 50 cm.

1
R /
1 _
84. i. ' In first case, 10 V
^2 100-/
Sum of currents away from junction B is zero.
On interchanging resistances,
x-10 x-y 3r-0 = 0
R^ _ /-lO 10 0 10
R,1 110-/
53x-2y = 30 -(0
/ /-lO
= 1 Sum of currents away from junction D is zero.
lOO-Z^'llO-/
/2-10/ = 11000-110/-100/ + /^
y-10 ^ y-x ^ y-0 =0
60 15 5
200/= 11000
17y-4;c = 10 ...(»)
/ = 55 cm
On solving (i) and (n)/ we get
R, + R2 = loooa X = 0.865 A and y = 0.792 A
R2 = (1000-Ri)a
x-y _ 0.073^
1000-Rj^_45 15 ~ 15
55
= 4.87 mA.

1000 ^45 , ^ ^ 100 88.


R 55 55
1
I 20 Q sa

R^ =550n. ^ h-Wv
10 V 10 V
85. ! .' For balanced bridge
30 0
P _ R ^ 400 A/W
Q~ X~ X
R
When P and Q are interchanged, A/vV
Q _ R _ 405
P “X “ X Terminal voltages across the two batteries are
P
— X — =
Q 400 X
405 Vj =6j -Ir^ =0
10-1x20=0
Q P X X
1 = 0.5
X = V400x405 = 402.5 H.

86. (c)
0.5 V 6 V ^2 = ^2 - X 5 = 7.5 V
R (2 + 4)0
AJ
0.5 = 7.5 , 7_^
30 R
6 V
Also, 7.5
R
AI (6 + 4)0 => = 0.25
R
^2 6 =s>
3
E-=-x0.5=0.3V. R=30O.
05 ”10 2 5
CURRENT ELECTRICITY (Competition Section) 3.231

89. 12.0(' 91. 40.00

So R0 '0 R0 ^ ^ pL ^ 4pL
I I Wv ^ Tof-

560 cm 560 cm

R' =
<1 = 2R

€ g CG 7t
rrff
2)
r
N
AM- R
10
10 a In first case, S ~ 75 ~ 3
Let X be the potential gradient. Then, 6 = 560x 2R /
In second case, —

w
After connecting the 10 H resistance in parallel, S 100-/
SxlO
= 500x 1 I
10+r 2x- =
3 100-/

Flo
560x10
= 500
I - 40 cm.
10+r

ee
56 = 50+ 5r 92. '

Fr
6
JV ^12 Adding the end correction for end A, effective
r = — => N = 12.
5 10 10 lengths become
A/= 43 + 2 =45 cm
90. 03.00
for
ur
1 a
R = -?na + 6/ = 102 -45 = 57cm
3 2 m
Using condition of balanced Wheatstone bridge,
ks
For minimum value of R,
Yo

dR _ a a
oo

=0 R~ 57
d7u 3 2
eB

a a 57
=> R= —xl5
3 2n? 2 45

= 19 a.
[3
r

a:
w = x = 3.
ou

U ^2
ad
Y
nd
Re

Based on Ohm's Law, Resistance, Resistivity, (d) the resistance and the specific resistance will
both remain unchanged. [CBSE 20041
Conductance and Conductivity
Fi

1. The reciprocal of resistance is 3. Two solid conductors are made up of same material,
(/j) mobility
have same length and same resistance. One of them has
(fl) reactance
a circular cross-section of area Aj and the other one
(c) conductivity {d) conductance
●\EI-.r ; ●● :2! has a square cross-section of area A^.The ratio Aj / A2 is
{a) 2 (b) 1.5 (c) 1 id) 0.8 (NEET 20)
2. The electric resistance of a certain wire of iron is
R. If its length and radius are both doubled, then 1. Three copper wires have lengths and cross-
sectional areas as (/, A) ; (21,AI2) and {112,2 A).
(a) the resistance will be doubled and the specific
Resistance is minimum in
resistance will be halved.
(fl) wire of cross-sectional area A / 2
(b) the resistance will be halved and the specific
resistance will remain unchanged, {b) wire of cross-sectional area A
(c) the resistance will be halved and the specific (c) wire of cross-sectional area 2 A
resistance will be doubled. (d) same in all the three cases [CBSE 1997)
. 3.232 PHYSICS-XII

5. Tlie resistance of a wire is' R' ohm. If it is melted Match Column I and Column II with appropriate
and stretched to 'tf times its original length, its new relations.
resistance will be Column 1 Column 11
R m
(fl) uR (b) (p) Drift velocity (0
R
ic) n^R {d)~ (q) Electrical resistivity
n |\i 1 r i7| eE
(r) Relaxation period m T
6. A wire of resistance 10 is elongated by 10%. m

The resistance of the elongated wire is (s) Current density (iv) i


J
(a) 10.1 a (b) 11.1 n
(c) 12.1 n (d) 13.1 Q 'i-'i (rt) (p) - (27/), (q) - (iv), (r) - (i), (s) - (ii)
7. From the graph between current (/) and voltage (b) (p) - (in), (q) - (iv), (r) - (ii), (s) - (i)
(y) as shown in the figure, identify the portion (c) (p) - (in), (q) - {/), (r) - (iv), (s) - (//)
corresponding to negative resistance. (d) (p)-(///), (f?) - (//), (r) - (iv), (s) - (i) i\il.l2l|
11. A charged particle having drift velocity of
7.5 X lO’^ms"^ in an electric field of3 x 10“^'^Vm“\ has a
mobility in m^V~*s~^ of
(fl) 2.5x10^ (b) 2.5 X 10^
-15 15
(c) 2.25x10 (d) 2.25x10 [NEHT 20|

Based on Temperature Variation of Resistivity


(a) AB (b) BC and Superconductors
(c) CD (d) DE iCBSI: 19971
12. As the temperature increases, the electrical
8. The colour code of a resistance is given below : resistance

(«) increases for both conductors and semiconductors


o-
(b) decreases for both conductors and semiconductors
(c) increases for conductors but decreases for
Yellow Violet Brown Gold
semiconductors

The values of resistance and tolerance, respectively. (d) decreases for conductors but increases for
are semiconductors [NEET july 22]
(27) 47 Idl, 10% (Zj) 4.7 Vn. 5% 1.1. The specific resistance of a conductor increases wifii
(c) 470 n, 5% (d) 470 kn, 5% [K1 t ;r 20) (a) increase in temperature
(Z?) increase in cross-sectional area
Based on Drift Velocity and
(c) decrease in length
Mobility of Electrons
(d) decrease in cross-sectional area [CBSh 2002]
9. A copper wire of length 10 m and radius
14. Si and Cu are cooled from 300 K to a
10"^
m has electric resistance of 10 Q. The current
temperature of 60 K. Then resistivity
(a) for Si increases and for Cu decreases
density in the wire for an electric field strength of
10 Vm
-1 :
is
(b) for Cu increases and for Si decreases
(c) decreases for both Si and Cu
(/7)10^Am"^ (b) lO^Am"^
(d) increases for both Si and Cu [CBSE 2001]
(c)]0‘^Am~2 (d) lO^Am'^ INEETJuly 22|
15. Which one of the following bonds produces a
10. Column I gives certain physical terms asso solid that reflects light in the visible region and whose
ciated with flow of current through a metallic conductor. electrical conductivity decreases with temperature and
Column II gives some mathematical relations has high rnelting point ?
involving electrical quantities. (fl) metallic bonding
CURRENT ELECTRICITY (Competition Section) 3.233

(b) van der WaaTs bonding 21. Two wires of the same metal have same length,
(c) ionic bonding but their cross-sections are in the ratio 3 : 1. They are
joined in series. The resistance of thicker wire is 10
(d) covalent bonding [CBSE Pre 2010]
The total resistance of the combination will be
16. Tlie solids which have the negative temperature
coefficient of resistance are
<n
(fl) insulators only (D) semiconductors only (c) 40 n (d)ioon [CBSE 19951
(c) insulators and semiconductors
22. The current in the given circuit is
(d) metals [\1-KT 20]
i A
17. Which of the following graph represents the >-

variation of resistivity (p) with temperature (7) for


copper ? 6Q

w
30
4.8 V X
(«) P-

wv
c B

Flo
60

(a) 8.31 A (/;) 6.28 A

ee
T T
(c) 4.92 A [CBSE 19991
(c) (d)

Fr
P“
23. Three resistors having resistances r,, ^ and are
connected as shown in the given circuit.

for ^2
ur
>2

* o—► o
T T A I1 '3 B
ks
[NEET20] l
Yo

The ratio -2- of currents in terms of resistances used


oo

Based on Series and Parallel I


1
Combination of Resistances in the circuit is
eB

18. The effective resistance of a parallel connection {b)-3L.


that consists of four wires of equal length, equal area of "2 + ^3 r^ + r^
r

cross-section and same material is 0.25 Q. What will be


(d)-^
ou
ad

tlie effective resistance if they are connected in series ?


"i + 's [.\EET21]
(«) 0.25 a (/;) 0.5 Cl
Y

(c)in
24. The equivalent resistance of the infinite network
{d)4Cl [NEET 211
given below is
nd

19. Resistances n, each of rohm, when connected in


Re

in in in
parallel give an equivalent resistance of R ohm. If these
A^—VW—●—VA—I—VA—t—V/v—
Fi

resistances were connected in series, the combination


would have a resistance (in ohms) equal to in 1 n in
R
(a) n^R (^)-5
in in in in
n B*—VA—*—^AA^—i—VA
R
{c)~ (rf) tiR {a) 2 Cl {h) (l + ^/2)^
n
[CBSE 2004]
(c) (l4-^)a id) (1 + V5)Q
20. A wire of resistance 12 ohms per metre is bent to
[NEET Sept. 22]
form a complete circle of radius 10 cm. The resistance
25. For the circuit shown in 2V 4V
between its two diametrically opposite points, A and B 1 n in

as shown in the figure is the figure, the current I will be I—VA— 1—VA—I
(fl)3Q (b) 6'kCI
(rt) 0.5 A (b) 0.75 A
A| ■>B
(c)l A (d) 1.5 A
(c)6n (d) 0.6 nCl > AAV
(NEET 20] 1
[CBSE PMT09] 4n
3.234 PHYSICS-XII

26. The equivalent resistance between A and B for 31. In the circuit shown, the cells A and B have
the mesh shown in the figure is negligible resistances. For =12V, =500 0 and
R = lOOn, the galvanometer (G) shows no deflection.
R1
VA

R VB
T

{a) 4.8 Q (b) 7.2 O


(c) 16 0 (d) 30 O |\l I I 2('i
The value of Vg is
27. A wire of resistance R is bent to form a square ((J)4V, (f’)2V
ABCD as shown in the figure. The effective resistance
(c) 12 V (d) 6 V
between E and C is (£ is midpoint of ^ B
arm CD) IJEEM.iin April I')| [ 32. When the key K is pressed at time f = 0, then
{a)~R (F)R which of the following statements about the current /
4
in the resistor ABof the given circuit is true ?
1 D
E
ic) R A 1000n B
16 64

28. You are given several identical resistors each of


resistance R=10O and each capable of carrying a IpF 1000 n
maximum current lA. It is required to make suitable
combination of these resistors to produce a resistance 2V

of 5 .O, which can cany a current of 4 A. The minimum A


K
number of such resistors required is
(«)4 {b) 10 {a) 7=1 mA at all f (I?) 7=2 mA at all f
(c)8 (d) 20 (c) 7 oscillates between 1 mA and 2 mA
29. Two metal wires of identical dimensions are (d) at f = 0,7 = 2 mA and with time, it goes to 1 mA
connected in series. If and are the conductivities |AI \K; 1''^'!: : t !^^i

of the metal wires respectively, the effective conduc 33. A, B and C are voltmeters of resistances R, 1.5R
tivity of the combination is lAli’M i 13 ; )Li. Main 22) and 3R respectively as shown in the figure. When some
20i02
in) (b)
Oi + 02
o

^1+^2 X
(c) (b)
2(7^02 CT1O2 <c>
30. A ring is made of a wire having a resistance potential difference is applied between X and Y, the
=120. Find the points A and B as shown in the voltmeter readings are V^, Vg and respectively.
figure, at which a current carrying conductor should Tlien
be connected so that the resistance R of the sub-circuit
g
(«)V^^Vg = V^ ib)V^=V,^V^
between these points is equal to -O.
[AlI'Ml Pi-L' 1:1 (d)V^=Vg=V^ I Ml’M' r-i

Based on Internal Resistance,


h 8
EMF and Terminal P.D. of Cells
I 1
{b) J- = 34. A cell having an emf 6 and internal resistance r
^2 3 is connected across a variable external resistance R. As
the resistance R is increased, the plot of potential
difference V across R is given by
'2
CURRENT ELECTRICITY (Competition Section) 3.235

40. A set of 'n' equal resistors, of value 'R' each, are ■


connected in series to a battery of emf 'S' and internal
resistance 'R'. The current drawn is I. Now, the 'u'
resistors are connected in parallel to the same battery.
Then the current drawn from battery becomes 10 /. The
value of'«' is

R (a) 10 (b)9
(c) 20 (d) 11 [MEET 18)

Based on Grouping of Cells


41. Two batteries, one of emf 18 V and internal
resistance 2Q. and the other of emf 12 V and internal

w
o R o R O

Flo
18 V
[AIPMT12; VMMC 14] h

35. A student measures the terminal potential diffe ^Wv-

ee
rence (V) of a cell (of emf 6 and internal resistance r) as 2Q

12 V
a function of the current (/) flowing through it. The

Fr
slope, and intercept, of the graph between V and 1,
IQ
then, respectively, equal
for
ur
(fl) -r and 6 {b) r and - 6 resistance 1Q, are connected as shown. The voltmeter V
(c) -€ and r (c) 6 and -r [CBSE PMT 09] will record a reading of
ks
36. The internal resistance of a 2.1 V cell which («) 30 V (b) 18 V
Yo

gives a current of 0.2 A through a resistance of 10 Q is (c) 15 V (d) 14 V [CBSE 2005]


oo

(fl) 0.2 n {b)0.5Q 42. A battery consists of a variable number 'n' of


eB

(c) 0.8 n (rf)i.on (NEET 131 identical cells (having internal resistance 'r' each)
which are connected in series. The terminals of the
37. A cell of emf 4 V and internal resistance 0.5 .Q is
connected to a 7.5 .Q external resistance. The terminal battery are short-circuited and the current I is
r

measured. Which of the graphs shows the correct


ou

potential difference of the cell is


ad

relationship between / and n ?


(a) 3.75 V (b) 4.25 V
Y

(«) (b)
(c)4V id) 0.375 V [iVEET Sept. 22j
/ ]
38. For a cell, the terminal difference is 2 -2 V, when
nd
Re

circuit is open and reduces to 1-8V, when cell is


Fi

connected to a resistance R = 5 fl. The internal


resistance of cell r is

10
(«)-n o It

10
ic) id)
(c) —n id) - a / I
9 9 [CBSE 2002]

39. A current of 2 A flows through a 2 O resistor


when connected across a battery. Tlie same battery
supplies a current of 0.5 A when connected across a 9 Q
resistor. The internal resistance of the battery is
O It
(fl) 0.5 n ib) 1/3 n
ic) 1/4 n (rf)lQ [CBSE 2011) [\'EET 18)
3.236 PHYSICS-XII

43. Two cells, having the same emf are connected in 50. Power dissipated across the 8 .Q resistor in the
series through an external resistance R. Cells have circuit shown here is2 W. Tlie power dissipated in the
internal resistances and r2(r^ > respectively. When 3 n resistor is
the circuit is closed, the potential difference across the
first cell is zero. The value of R is AAA VW
la 3Q
(a) (b) r, -r^ o

8Q

^ ^ 2 iCBsr. AAV

Based on Heating Effect of Electric Current, (rt)0-5 W (b) 1 W


Electric Power and Electric Energy (c) 2W {d)3V/ H iNi j'lih]

44. Which of the following acts as a ciraiit 51. In India, electricity is supplied for domestic use
protection device ? at 220 V. It is supplied at 110 V in U.S.A. If the
(/z) Inductor (&) Switch resistance of a 60 W bulb for use in India is R, the
resistance of a 60 W bulb for use in U.S.A. will be
(c) Fuse (rf) Conductor |\; 1 I b-|

45. A fuse wire is a wire of


{a)R {b) 2R
R R
(a) low resistance and low melting point,
j( !;s! j0041
(b) high resistance and low melting point,
52. The total power dissipated in watts in the circuit
(c) low resistance and high melting point. shown here is
(d) high resistance and high melting point. 6Q
[C15S1- AAV
46. Two resistors of resistances, 100 and 200 H are
3a
connected in parallel in an electrical circuit. The ratio of AAV
the thermal energy developed in 100 Cl to that in 200
4D 18 V
in a given time is
AAV It
(fl) 1 : 2 (b)2:l
(c) 1 : 4 (rf)4:l [NEEl Julv 22| («) 16 {b) 40
47. A battery is charged at a potential of 15 V for (c)54 (d)4 K l-M 2007]

8 hours, when the current flowing is 10 A. The battery 53. Two cities are 150 km apart. Electric power is
on discharge supplies a current of 5 A for 15 hours. The sent from one city to another city through copper wires.
mean terminal voltage during discharge is 14V. The The fall of potential per km is 8 volt and the average
watt-hour efficiency of the battery is resistance per km is O.SQ.The power loss in the wires is
(a) 82.5% (b) 80% (a) 19.2 kW (&) 19.2 J
(c) 90% (rf) 87.5% [CBSF Katjj (c) 12.2 kW {d) 19.2 W lAIi’MT l-ij

48. The power of an electric bulb marked as 40 W 54. The power dissipated in the circuit shown in the
and 200 V used in a circuit of supply voltage 100 V figure is 30 watts. The value of R is
R
will be
AAV
(ff)100 w (h) 40 W
(c) 20 W’ id) 10 W [Oist 5Q
VW
49. When three identical bulbs of 60 watt-200 volt

rating are connected in series to a 200 volt supply, the 100 V

power drawn by them will be h

(fl) 60 watt (&) 180 watt (fl) 20 Cl (b) 15 Cl


(c) 10 watt (d) 20 watt ICBSE 21 ill 11 (c)ion (d) 30 a [All’MI' 12]
CURRENT ELECTRICITY (Competition Section) 3.237

55. If power dissipated in the 9 H resistor in the Based on Kirchhoff's Laws


circuit shown is 36 W, the potential difference across
the 2 Q resistor is
59. Consider the following two statements :

90 (A) Kirchhoff's junction law follows from the


rWn conservation of charge.
60
l_v^
(B) Kirchhoff's loop law follows from the
conservation of energy.

I—vw Which of the following is correct ?


V 20
(fl) Both(y4) and (B) are wrong
(fl)4V (ft)8V (ft) (A) is correct and (B) is wrong
(d)2V (c) (v4) is wrong and (B) is correct

w
(c) 10 V
INF 2i'l 1 : \'MM( I.":! {d) Both (A) and (B) are correct (CHSE2010]
56. A filament bulb (500 W, 100 V) is to be used in a (9). in the circuit shown in the figure, if the potential
230 V main supply. When a resistance R is connected

Flo
at point A is taken to be zero, the potential at point Bis
in series, it works perfectly and the bulb consumes R 2V
500 W.
1 D

ee
B
I h
The value of R is 1 A'

Fr
(fl) 260 (ft) 13 O 20 2A

(c) 2300 (d) 46 O 1A 2A


H I
A
for
ur
57. Six similar bulbs are connected as shown in the C
^3
I V
figure with a DC source of emf S and zero internal
resistance. (a)-2V (ft)+1V
ks
(c)-lV (d) +2 V ]U^^F 2011]
Yo

A B
oo

61. See the electric circuit shown in this figure.


Which of the following equations is a correct equation
i
eB

1 for it ?

1 1 R
r

t
ou
ad

i
e 1
-♦ I wv
Y

The ratio of power consumption by the bulbs ^2


when (i) all are glowing and (i7) in the situation when 1
nd
Re

two from section A and one from section 6 are glowing,


will be (rt) =0
Fi

(fl) 9 : 4 (ft) 1 : 2 (ft)


(c)2:l (d)4:9 i\i;ll'9
(c)E^-0\ + i2)R+ijrj=0
58. The charge flowing through a resistance R
varies with time t as Q = at~bl^, where a and ft are (d) -0\+i^)R-i\r^=0 ICBSETMT 09]

positive constants. 62. For the circuit given below, the Kirchhoff's loop
The total heat produced in R is rule for the loop BCD£B is given by the equation
a^R a^R I
1 B '2
(«) (b) A,—► VW-^c
6ft 3ft
R!

a^R a^R T T ^2
(c) (^)- "h
2ft
D
|\FI i I'- ij
^3
.238 PHYSICS-XII

(fl) ~l2R^ + ^2 ^3 “0 67. In a potentiometer circuit, a cell of EMF 1.5 V


(b) + gives balance point at 36 cm length of wire. If another
cell of EMF 2.5 V replaces the first cell, then at what
(c) i2^ + ^2~^3~h^l =0 length of the wire, the balance point occurs ?
(fl) 60 cm (&) 21.6 cm
(d) + + [NEET 20|
(c) 64 cm (d) 62 cm [NEET21I
63. The potential difference between the
68. A potentiometer circuit is set up as shown. Tlie
points A and B in the given figure is potential gradient, across the potentiometer wire, is
k volt/cm and the ammeter, present in the circuit, reads
3 V
2Q
+
IQ H 1.0 A when tw'o way key is switched off. The balance
O-
A / = 2A
VvV^I—Vv\—o points, when the key between the terminals (i) 1 and 2
B

(n) 1 and 3, is plugged in, are found to be at lengths


(fl) +6V (I;) +9V /j cm and cm respectively. Tlie magnitudes of the
(c) -3V (d) +3V
potential drops across the resistors R and X are then
[MEET Ifi IJl
respectively equal to
64.
In the network shown in the figure, each
resistance is in. The effective resistance between the
i h
points A and Bis
1 Q
vw
A
T B

IQ IQ 1 Q 2
IQ 3

IQ 1 Q
Y/s,—i—
R X
AO- AAAr wv ■OB

(»)ln {b)^n (a) k{l., -/j) and {b)kl^ andlc(/2-/,)


(c)7Q {d)^-n (c) cf{/2-lj)and kl^ (rf) kl^ and kl2
ICBSE 1990)
[CBSE 10]

Based on Potentiometer 69. A potentiometer wire has length 4 m and


resistance 8 n. Tlie resistance that must be connected in
65. A potentiometer is an accurate and versatile series with the wire and an acaimulator of emf 2 V, so
device to make electrical measurements of EMF, as to get a potential gradient 1 mV per cm on the wire
because the method involves IS

(a) cells (a) 400 (b) 44 Q


(b) potential gradients (b) 48 Q (rf)32 O (A1PMT15]
(c) a condition of no current flow through the 70. A potentiometer circuit has been set up for
galvanometer finding the internal resistance of a given cell. The main
(d) a combination of cells, galvanometer and battery, used across the potentiometer wire, has an emf
resistances (NEET 17] 2.0 V and a negligible internal resistance. The potentio¬
meter wire itself is 10 m long. When the resistance R,
66. The resistivity of potentiometer wire is 10~^fi m.
cormected across the given cell, has values of
Its area of cross-section is lO'^m". When a current
(i) infinity (ii) 9.5 a
f = 01A flows through tlie wire, its potential gradient
is the 'balancing lengths', on the potentiometer wire are
found to be 3 and 2.85 m, respectively. The value of
(rt)10"^Vm
-1
{b) 10~*Vm
-1
internal resistance of the cell is
-1 -I
(c)O-lVm (rf) 10 Vm (a) 0.95 n (b) 0.5 a
(CBSE 20011 (c) 0.75 Q (d) 0.25 Q [AIPMT 14]
3.239
CURRENT ELECTRICITY (Competition Section)

71. A potentiometer wire is 100 cm long and a variable resistance Y as shown in the figure. For the
constant potential difference is maintained across it. most precise measurement of X, the resistances P and Q
Two cells are cormected in series first to support one (rt) should be very large and unequal
another and then in opposite directions. The balance (1)) do not play any significant role
points are obtained at 50 cm and 10 cm from the positive (c) should be approximately equal to 2X
end of the wire in the two cases. The ratio of emfs is
{b)5:A
(d) should be approximately equal and are small
{«) 5 : 1 [NEET July 22]
(c) 3 : 4 (d) 3 : 2
73. In a Wheastone's bridge, all the four arms have
"2. A potentiometer wire of length L and a e
qual resistance R. If the resistance of the galvanometer
resistance r are connected in series with a battery of arm is also R, the equivalent resistance of the
emf and a resistance r^. An unknown emf 6 is combination as seen by the battery is
balanced at a length I of the potentiometer wire. The {a) R (b) 2R

w
emf 6 will be given by : R
R
L6.r
0 0 [CBSE 20031
(«) (i»)
{r+r^)I

Flo
7h. A bridge circuit is shown in the figure. The
Br.r
0 I U equivalent resistance between points A and B is
{C)

ee
(r+r^) L \ir\ii I'n IV

Fr
73. The sliding contact C is at one fourth of the
length of the potentiometer wire (AB) from A as shown
V,0
for
ur
ks
Yo

Ra
oo

A B
eB

■wv [CBSE 2000)


R
77. The resistances of the four arms P, Q, R and S in
in the circuit diagram. If the resistance of the wire AB is
r

Wheatstone's bridge are 10 ohm, 30 ohm, 30 ohm


ou

Rq, then the potential drop (V) across the resistor R is


ad

4WR 2WR and 90 ohm, respectively. The emf and internal


Y

(«)
0
(b)
0
resistance of the cell are 7 volt and 5 ohm respectively.
4R.+ R
3Ro + 16R 0
If the galvanometer resistance is 50 ohm, the current
nd

2V.R drawn from the cell will be


Re

4K.R
0 0
(c) (d)
3Rq + R 2R.+3R
0 |\ri:i Si'i'i- (fl) 1.0 A {b) 0.2 A
Fi

(c) 0.1 A (d) 2.0 A jNTEET 13]


Based on Wheatstone Bridge and
78. For the network shovvm in the figure, the value
Metre Bridge
of the current / is
74. A Wheatstone bridge is used to determine the 9V
value of unknown resistance X by adjusting the 2Q
35

18V

sa ' ’
5V
(c) —
9
5V
id)
18 '( HM 2Un3]
3.240
PHYSICS-Xli

’ 79. Three resistances P, Q, R each of 2 n and an C

unknown resistance S form the four arms of a Wheat


stone bridge circuit. When a resistance of 60 ir is R
R
connected in parallel to S, the bridge gets balanced. R

What is the value of S ? F


A
(a) 2 0 (b)3Q R B
(c)60 (d) lO fCBSE 2007]
D
●vw £
R
80. The resistance of each arm of a Wheatstone
bridge is 10 O. A resistance of 10 O is connected in (a)3VIR (b)V/R
series with the galvanometer. Then, the equivalent (c) V/2R {d)2VIR [CBSE 2004)
resistance of the bridge across the battery will be 84. In a metre bridge, the balancing length from the
(a) 10 O (b) 15 O left end (standard resistance of 10 is in the right gap) is
(c) 20 O (d) 400 [CBSE 2001] found to be 20 cm. The value of the unknown
resistance is
81. In the circuit shown, if a conducting wire is
connected between the points B and D, the current in (rt)0-8O {b)0-5.Q
this wire will (c)l-40 (d) 0-25 0 [CBSE 1999]
85. An unknown resistance R^ is connected in series
with resistance of 10 ohm. This combination is con
nected to one gap of a metre bridge, while other gap is
connected to another resistance ^2- Th® balance point
is at 50 cm. Now when the 10 ohm resistance is
removed, the balance point shifts to 40 cm. Then, the
value of Rj (in ohm) is
(a) 60 (b) 40
(c) 20 (d) 10 [CBSE 2002]
(a) flow from B to D 86. The resistances in the two arms of the metre
(b) flow from D to B bridge are 5
and R Q, respectively. When the
resistance R is shunted with an equal resistance, the
(c) flow in the direction which will be decided by new balance point is at 1.6/
the value of V. r

(d) be zero. [CBSE 2006] < >

82. In tire network shown in figure, each resistance sn RQ

is equal to 2 Q. The resistance between the points A r'WVi rA'Wi


and Bis

c
G

NJ' A B
●O' h 100-/ 1
O'.
*0
F The resistance 'R', is
A
B
(a) 15 Q (b) 20 a
D E (c) 25 n (d) 10 Q [AlPMT 14]
2n

(a) IQ
87. A resistance wire connected in the left gap of a

(b)2Q.
metre bridge balances a 10 D resistance in the right gap
(c)3Q (d)4Q [CBSE 1995) Poirif which divides the bridge wire in the ratio
83. Five equal resistances each of resistance R are 3 : 2. If the length of the resistance wire is 1.5 m, then
connected as shown in the figure. A battery of V volt i IS
the length of 1Q of the resistance wire is
-1
connected between A and R The current flowing i in (a) 1.0x10 m
(&)1.5xl0~^ m
AFCEB will be
(c) 1.5xl0"^m (d)l.OxlO'^ m ]NEET 20]
CURRENT ELECTRICITY (Competition Section) 3.241

Answers and Explanations


1
1. (rf) Conductance = 8. (r)
Resistance
Yollow Violet Brown Gold
2. {/?) Original resistance. i i
/ / 4 7 1 ±5%
K=p_=p
A nr R = 47 X10 a ± 5% = 470 Q ± 5%.

When both length and radius are doubled.


j = csE- 1e=-Le
IE
9. un
21
R' = P 7 p RA Rxnr^
n{2rf 10x10
1 /

w
-2 \2
2‘^rtr^ ~2 lOx jc
10

■Jn
I pi
3. (c) R =
= 10®Am'l

Flo
R
10. i.n
For both conductors p, I and R are same.

ee
, , eE
Aj - ^2 ip) v^=~x ^ {in)

Fr
m

4. {c) The resistances of the three wires are


W/ = a£ = -
P
=> p = ^/ ^ {iv)
/ 21 4/
for
ur
R
.p-; R2=P
1
A/2 ^ a' (r) T =
m
^ (0
ne^p
- 42 _ 1
ks
I
{s)l=enAv^ => )=— = env^ => (/i)
Yo

A
oo

Clearly, the resistance of third wire of cross-


sectional area 2A is minimum. V 7.5xl0~*ms“^
eB

d _
11. (<fl -10 -1
E 3x10 Vm
5. (c) R=p 1= i!
A ^ Al = 2.5x lO^m^wV^
r
ou

m
Ro:f
ad

12. k' l p = —
ne~x
Y

R' fl (nlf = n
2 For conductors, x decreases with temperature and
R I I so resistance increases.
nd
Re

R' = n^R. For semiconductors, n increases rapidly with


temperature and so resistance decreases.
Fi

6. (c) New length, V = 1 +10% of / = 1 ● 1/ 13. The specific resistance of a conductor


As V = Al = AT increases with the increase in temperature.
AJ'-=M 14. Un Si is a semiconductor, its resistivity
A' I ~ increases with the decrease in temperature. Cu is a
conductor, its resistivity decreases with the decrease
K=L^A = 1-1x1.1=1-21 in temperature.
R A A'
15. (-n The conductivity of metals decreases with
R' = l-21R=l-21xlO increase in temperature.
= 12in. 16. [i) Both insulators and semiconductors have
7. (f) In the portion CD, the current I decreases with riega^ive temperature coefficient of resistance,
the increase in voltage V. So this portion corresponds 17. yi') At low temperatures, the resistivity of
to negative resistance. copper increases as a higher power of temperature.
<,●
3.242 PHYSICS-XII

R
18. (</) R = 0.25 0 24. (i ) Refer to the answer of Exercise 3.21 on
p 4
page 3.153.
R = 10
2+4
25. (c) 7 = - = 1 A.
Rj = 4R=40. R 1 +1 +4

19. (ii) When n resistances are connected in parallel, 12x6


26.(0 + + 8=4 + 4 + 8=160.
1 1 1 1 1 n
12+6
— = - + - + - + -+.../i terms =
R r r r r r
11. Ul)
or r = tiR 7R
R EAC
When n resistances are connected in series, 8

R' = r+r+}■+...n terms = nr R


R
EC ~
8
or R'=h(«R) = «^R.
J_-A
20. (i!) Length of wire ~2nr=2K'y. 0.10 m R.eq " 7R R ” 7R
Resistance of whole wire = 0.2 7c x 12 = 2.4 k O
7R
Resistance of each half part = 1.2 nO R
«? 64

1.2 Ttn
28. (c)To carry a current of 4 A, we need four paths,
A/W
y40- -OB
each carrying 1 A current, let the resistance of each
path be r. As four such resistances are connected in
AAAr
1.2 n a parallel, r/4 = 5 O or r = 20 O. So we should connect the
10 O resistances in each path.
1.2n
R = 0.67tO. .-. Total number of resistances =4x2=8.
AB ~ 2
29. l/'i
1
21. (c) As R oc / /
A
n
a
1 °2

rC
or
R^ =3R^ =3x10=300 2/ /
1
/
1_=J_+_L
R^=Ri + R2=10+30 = 400. a
eq O2A ^2

22. (il) The two resistances of 6 O each are in parallel 20,02


o
with the 3 O resistance. «/
01+O2

R 12x3 _36 _12 ^ 30. (d) Let resistance of length /, of wire =xQ.
eq 12+3" 15" 5
Then the resistance of length /j of wire ={12 - .r)0
S 4-8
7 = - = 2 0 A. These two resistances are in parallel
R 12/5
e<]
xjl2~x) _8 12x-a:^
or =8
23. (/;) h~h'^ '3
;c + (12-^:)~3 4

. 1 or -12^ + 32 =0 :r=40 or 80
I oc -
r
For.r = 4 0,
X 4 _1
=>.
I2 n-x 12-4”2
h h '3 ’2 /
1
8
For X =8 O, =2

=>
I
3 _ h h 12-8

h "2 + ^3 Hence, the only correct option is (d).


CURRENT ELECTRICITY (Competition Section) 3.243

31. (h) As no current flows through the galvano- 37. (<?)


meter.
7.5 Q
Vr,B = P.D. across R = RI WV
R
12 1
But / = = —A
R,I + R 500 + 100 50 e r

.-. ,ys = R/=100x-l = 2 V. 4V 0.5 n


50
e 4
32. (:/) When the key is pressed at time f=0, the / = = 0.5 A
R+r 73 + 0.5
current starts growing in the circuit. The 1 pF capacitor
bypasses the 100 Q resistance connected in parallel Terminal voltage.
with it. Only resistance AB is effective. Hence initial V = RI = 73 X 0.5 = 3.75 V.

w
current in the circuit is
38. Internal resistance of the cell is
g 2V
1 = =2xl0“^A = 2mA.
R 1000 n

Flo
AB r = R
y
When the capacitor gets fully charged, no current

ee
flows through it. r2-2-l-8'l 10
5D = n.

Fr
1-8 9
Then R„ =1000 + 1000 =2000
eff
2V
39. (/') Let rO be the internal resistance of the
/ = = 10"^A = lmA
battery
2000 n
for
ur
g
33. ((/) In first case: = 2A
r+ 2
ks
1.5 R
g
VAr In second case:
Yo
= 0.5 A
r+9
oo

R
o—vw -o
r+9
X y
1q.
eB

= 4 or r =
AW r +2 3
3R
g
40. (iJ) / = ...(/)
r

R„ = R uR+ R
ou

R
ad

^9

AW-o fiE
X
10/ = ...(»)
Y

R R + nR
+ R
B and C are in parallel, Vg = V^ n
nd
Re

In a series circuit, V cc R From (t) and (/i), we get


y. =ycombination
. Hg g
Fi

A
= 10
R + nR nR+ R
Hence, V^=Vg = V^.
n = 10.

34. {b} Refer answer to Q. 35 on page 3.45. 41. hi) The two cells are connected in opposition, so
35. {a) y = g - /r the net current in the circuit is

or y = -r/+g ^]-^2 _1B-12


1 = = 2 A
On comparing with y = nix + c, we get n + ^2 2+1

slope =~r and intercept =g Reading of the voltmeter,


g 2.1 = Terminal p.d. across either cell
36. (b) r = --R = -10
I 0.2 = gi-/r^ =18-2x2
= 10.5-10 = 0.5 0. = 14 V.
3.244 PHYSICS-XII

42. {-;) 48. (0 P = i.c., P<xV^


nr r R

.●. I is independent of n and it is constant. ?l^YL


/ P, Vl
/o| or

^2 (100)- ^
49. ((f) For the three bulbs connected in series
R = Pj + ^2 "t
O
1/2 y2 y2 y3
or
43. (/’) Given

=g-/r^ =0 1-J_ _L J_=_L


P~60 60^60 "20
or
6
or i =
or P = 20 W.

50. ((f)
R
>yVV /, in 3n
WV
u. I
r, 6,r. o—► >—o

AAA^
8Q
2g
Also, / =
P.D. across (1D + 3 D) resistors
R + ty+.r,
=: P.D. across 8 O resistor
g 26
or Ij X 4 = /2 X 8
h R + Ty+r2 or
/i=2/2
or
R+ + ^2 = 2 or R = - r2- Power dissipated across 8Q resistor
44. (c) When excess current flows, fuse wire melts /^x8=2W or
f^=0-25
and breaks the circuit.
Power dissipated across 3D resistor
45. (t) A fuse wire is a wire of high resistance and = ljx3 =(2/2)^ x3 = 4x0-25x3 =3W.
low melting point.
51. (c) As the power rating of the bulb is same in
1
46. ih) P = => Pec — [Pissame] both cases.
R R vr
1 _
p2
'^2
Pi _^_200 = 2:1.
R
] R2
P2 R 1 100 or
220x220 110x110
R
47. ((f) Input energy when the battery is charged 1
1 1
= V/t = 15 VxlO Ax8h=1200 Wh or Ky= -R, =-R.
^2 4 1 4 [● ● -Rj = R]
Energy released when the battery is discharged
52. ((1 The parallel combination of 6 O and 3 Cl
= 14 Vx5 Axl5h=1050Wh
resistances is in series with the 4D resistance. The

Watt hour efficiency of the battery equivalent resistance is


6x3
_ Energy output _ 1050 R =
6 +3
+ 4=6D

Energy input 1200


18x18
P = = 54 W.
=0-875 = 87-5%. R 6
CURRENT ELECTRICITY (Competition Section)

R R 2R
53. {.n Total resistance of the wires * 150 x 0.5 = 75 Q R 1 —
eif 3 3 3
Current through the wires,
36^
1=^ 8
= 16A R1 =
R 2R
AR 0.5

p= I~R={16fx75 When two bulbs of section A and one bulb of

= 19200 W = 19.2 kW. section B glow :


1/2 R
54. ic) P =
rWY-i R
R
eq R

100
30 =
5R

w
5+ R I-
e
150 R = 500 +100 R
R 3R

Flo
R=ioa R + R =
cq 2 2
55. ) Current through 9 H resistor

ee
e 2^
p 36 Pi =

Fr
— =2A R 3R
Mr 9 eq

Current through 6 Q resistor Pi _ 3^2 3R = 9:4.


P2 2R 262for
ur
_ V 9x2 =3A
~R~ 6
58. (.0
Total current = 5 A
ks
Yo

Potential difference across 2 Q resistor / = = a-2bt


oo

dt
=2x5 = 10 V.
(7
eB

When f=—. Z=0


56. c.-i 2 b'
Bulb a
That is current will exist till f = —.
r

i
R
21;
ou

A/VV
ad

100 V 130 V
500 W Total heat produced in resistance R,
Y

2b

H=\l^Rdt= Ua~2btfRdt
nd
Re

230V
0 0
Fi

P 500 W a

1 = = 5A 2b
V lOOV
= (fl2 + 4b^t^-4abt)Rdt
5R = 130V 0
17

R = 26Q
=J>
t^llh
= <72^ + 4f,2 X R
3 2
57. (n ) When all the bulbs glow : Jo
R R
AAV AW 2/ . \3 \2
2 4fc a r
R R
a —I — -lab — R
WV AAV lb 3 \2b lb
R R
VA VA
a^R
R =
2b'^6i> 26 66
e
3.246 PHYSICS-XII

59. (. ) KirchhofPs junction law follows from the 64. (i/) Refer to the solution of Example 158 on

conservation of charge and Kirchhoffs loop law page 3.89.


follows from the conservation of energy. Putting r = lQ, we get
60. (('I
2 =1x2+2R3 f?3=0 R 8r_8xl 8
= -n.
AB
7
R1 2V
D
rVW 11
B
65. (c) Reading of potentiometer is accurate because
1 A
2n
it does not draw any current from the battery at the
R^: balance point.
1 A
1 A 2A 66. (iO Resistance of 1 m long potentiometer wire is
aM I ■MMt
l
IV ^ ^3 R=p^ =io-7x-L=oiq
-6
A 10

P.D. across 1 m length,


or
0 + l = P3
P = 7R=0-lx0-l=10'^V
KB = + lV.
Potential gradient.
61. (./)
==10"^ Vm"\
J, + l2 R I Im
D ■Wv C

67. {a)
B1
^-!z
I
1
A I vw B
B1 I
1
1.5 "36
ri
£ M 36x25
h ~ 15
= 60 cm.

Applying KVL to the loop ABCD, we get 68. (/'j


In case (i), R 1

In case (ii), ^ ^2
62. (c) Applying KVL along loop BCDEB,

-^7^-62 +^3 ■^'3^ = 0 69. (if) P.D. across the potentiometer wire,
V = kl =lmVcm“^ x 400cm
=> = 400 mV = 0.4 V

63. (1^1 Current through the potentiometer wire,

3V 1 = 2.=^ 1a
2n 10
~ R~ 8 20
o-
I—ya»—®
A 2 A B If R' is the required series resistance, then
B
Vg = V,-2x2-3-2xl = V^-9 / = or 1 ^ 2
R+R' 20 ’ 8 + R'
R' =320.

Alternative
70. (/>)
V 2n
3V in VB I 3 ^
2A + r = -1--1 R = ~-l x9.5
5*V I'
o-
9V h 2.85

0.15
X 9.5 = 0.5 0.
^A-^B=9-0=9V. 2.85
CURRENT ELECTRICITY (Competition Section) 3247

71. i.<'i Let k be the potential gradient in V/ cm. 75. (i7) In the balanced condition, the resistance R of
Then the galvanometer is ineffective. We now have {R+ R)
and {R+ R) resistances in parallel.
£,+62 = 50^ 2Rx2R
R = R.
2R+2R

gj=30fcand £2=20^ 76. ((/) The Wheatstone bridge is balanced, because


3Q _6Q
= - = 3 : 2. in
£2 2
The 7 n resistance is ineffective, we now have
72. (i ) Current through the potentiometer wire. (3 Q + 4 Cl) and (6 O + 8 £2) resistances in parallel.

1 = 0 7x14 14
R

w
Q.
r+ n AB ~
1 7 + 14 3

Potential gradient. 77.(h)


£.r
;c=^ = 0

Flo
L {r+r^)L

ee
0
I
£ = W =
(r+fj) L

Fr
73. i.?.i

V,0
for
ur
h
■vw
7V 5Q

_ 40x120
ks
R = 30 C2
~ 120 + 40
Yo

VA-VVW 7V
oo

A B
1 = = 0.2 A.
3Ro/4 (30 + 5)n
eB

AVW 78. ((/) Tine circuit is a balanced Wheatstone bridge,


R because
4n _6n
r

Rx 2D ~3Q
ou
ad

R 4 _
AC
K + ^ 4R+R 0 The 4 £2 resistance in arm BD is ineffective. We
Y

4 now have (4£2+2£2) and (6£2+3£2) resistances in


RR parallel.
3R^ _ R^{16R + 3R^)
nd

0 6x9 18
R
Re

“"4(1^+4R)
AB ~ R Q
4R+ R 4
0 eq 6+9 “ 5
Fi

In a series combination, potential divides in direct Current, J = V _ V _5V


ratio of resistance. R..eq ~18/5 " 18
R
AC 79. [h] The fourth arm has resistances S and 6 £2 in
X V
AC ~ 0
R 6S
AB
parallel with equivalent resistance = a
RR, 6+S

4R+ Rq xK„0 =
4RV,0
For the balanced Wheatstone bridge,
Ro(16R +3Rq) 16R + 3R 0 P R

4(R, + 4R) Q 6S
6+S
74. (rj ) Measurement of resistance is most precise
2 _2(6+5)
when Wheatstone bridge is most .sensitive. This is done or
2 " 6S
by making resistances in the ratio arms equal. So
option (d) is correct. or 3S=6+S or S = 3£2.
3.248 PHYSICS-XII

80. fiO Connecting a resistance in series with the 85. (r) In first case,
galvanometer does not effect the balanced condition of
R^+10^ 50
the bridge. We just have (10 Q +10 Q) and (10 Q +10 H) or
R| +10 — R2
resistances in parallel.
i?2 "100-50
20x20 In second case 40 ^2
R = 10 a ' 100 -60 3
aj
20+20

81. (fj) Current through arm ABQ On solving, Rj = 20 Q.


V 86. (.;)
/ =
4+4 ~8 5 I1
In fhst case. ●●●(0
P.D. between A and B, R 100-/j
V
x4= — In second case 5 ^ i.ei^ -in)
8 2 ' R/2 100-1.6/j
Similarly, Xl= — On dividing (ii) by {i], we get
1 +3 4
1.6(100-1,)
V _V_V 100-1.6/
1
4 ” 4
or 200-3.2/,1 =160-1.6/ 1
or
D
® 4 or 1.6/,1 = 40 =>
/, =25

From (i), ^R
25
It follows that Vp > Vg. Hence the current will flow =;> R=15Q.
from O to Bin arm BD. 75

82. (/j) Refer to the solution of Example 177 on 87. 1,7)


page 3.108. R 10 a
AW

83. (c) As in problem 82, R AB = R

V ^2
Total current in the main circuit = —
R e /
This current divides equally into two parts along 1 (●)

paths FCE and FDE of equal resistances.


R
V
Current flowing along AFC£B =
ZR
10 "/2‘2
R I R= 15H
84. id) - =
100-/ Rx/

R 20 cm 1x1.5
or
in 100 cm -20 cm k~ ■h- 15
= 0.1 m.

20
or R=—xin=o-25 n.
80
C H A PT E R

●t
Magnefic Effed

w
of Currenf

Flo
ee
Fr
for
ur
ks
4.1 CONCEPT OF yvVAGNETIC FIELD Magnetic effect of current : Historical note. The
Yo

relation between electricity and magnetism was first


oo

1. Briefly explain the concept of magnetic field. noticed by an Italian Jurist. Gian Demenico Romagnosi in
Concept of magnetic field. A magnet attracts small
eB

1802. He found that an electric current flowing in a


pieces of iron, cobalt, nickel etc. The space around a wire affects a magnetic needle, and published his
magnet within which its influence can be experienced is observations in a local newspaper, Gazetta di Trentino.
r

called its magnetic field. However, it is now known that However, his observations were overlooked. The fact
ou
ad

all magnetic phenomena result from forces between that a magnetic field is associated with an electric
electric charges in motion. current was rediscovered in 1820 by a Danish Physicist,
Y

Hans Christian Oersted. His observations are explained


In order to explain the interaction between two charges below.
in motion, it is useful to introduce the concept of magnetic
nd
Re

field, and to describe the interaction in two stages : Oersted's experimentConsider a magnetic needle
SN pivoted over a stand. Hold a wire AB parallel to
Fi

1. A moving charge or a current sets up or creates a


magnetic field in the space surrounding it. the needle SN and connect it to a cell and a plug-key,
as shown in Fig. 4.1.
2. The magnetic field exerts a force on a moving
charge or a current in the field. It is observed that :

Like electric field, magnetic field is a vector field, 1. When the wire is held above the needle and the
that is, a vector associated with each point in space. We current flows from the south to the north, the
use the symbol B for a magnetic field. north pole of the magnetic needle gets deflected
towards the west, as shown in Fig. 4.1(fl).
4.2 OERSTED'S EXPERIMENT 2. When the direction of the current is reversed, so
that it flows from the north to the south, the
2. Describe Oersted's experiment leading to the dis
covery of magnetic effect of cunent. State Ampere's north pole of the magnetic needle gets deflected
swimming rule. towards the'east, as shown in Fig. 4.1(b).

(4.1)
PHYSICS-XIl

<4

N'

S'

o
O w
Fig. 4.2 Ampere's swimming rule.
(«) S ■►N
+
<4 E 4.3 BIOT-SAVART LAW
3. State and explain Biot-Savart law for the magnetic
S'
field produced by a current element. Define the SI unit of
B
A
magnetic field from this law.
S N
Biot-Savart law. Oersted experiment showed that a
N' current carrying conductor produces a magnetic field
around it. It is convenient to assume that this field is
made of contributions from different segments of the
conductor, called current elements. A current element is
denoted by ^, which has the same direction as that of
ib)
current I. From a series of experiments on current
Hg. 4.1 Deflection of a magnetic needle under carrying conductors of simple shapes, two French
the influence of electric current. physicists Jean-Baptiste Biot and Felix Savart, in 1820,
deduced an expression for the magnetic field of a
3. When the wire is placed below the needle, the current element which is known as Biot-Savart law.
direction of deflection of the needle is again
reversed. Statement. As shown in Fig. 4.3, consider a current
4. When the current in the wire is stopped element rff of a conductor XV carrying current 1. Let P
flowing, the magnetic needle comes back into
be the point where the magnetic field dB due to the
its initial position,
Since a magnetic needle can be deflected by a current element is to be calculated. Let the position
magnetic field only, it follows from the above
experiment that a current carrying conductor produces a
vector of point P relative to element dl be r. Let 0 be
magnetic field around it. the angle between df and r .
Ampere's swimming rule. This rule predicts the
direction of deflection of the magnetic needle in the
Oersted's experiment, it can be stated as follows :
Imagine a man swimming along the wire in the
0
direction of the flow of the current with his face always
T
turned towards the magnetic needle, then the north
dl
pole of the needle will get deflected towards his left i
hand, as shown in Rg. 4.2.
The direction can also be remembered with the
help of the word SNOW. It indicates that if the X
current flows from South to North and the wire is
held Over the needle, the north pole is deflected
towards the West. Hg. 4.3 Biot-Savart law.
MAGNETIC EFFECT OF CURRENT 43

According to Biot-Savart law. the magnitude of advances gives the direction d. Thus the direction of
the field dB is
dB is perpendicular to and into the plane of paper, as has
1. directly proportional to the current I through the been shown by encircled cross ® at point P in Fig. 4.3.
conductor,
dBccI Special Cases
2. directly proportional to the length dl of the current 1. If 0 = 0°, sin 0 = 0, so that dB = 0
element, i.e., the magnetic field is zero at points on the
dBccdl axis of the current element.

3. directly proportional to sin 0, 2. If 0 = 90°, sin 0 = 1, so that dB is maximum i.e.,


dB cc sin 0 the magnetic field due to a current element is
maximum in a plane passing through the element
4. inversely proportional to the square of the distance r
and perpendicular to its axis.

w
of the point P from the current element.

dBoc\ r
SI unit of magnetic field from Biot-Savart law. The
SI unit of magnetic field is tesla, named after the great

Flo
Combining all these four factors, we get Yugoslav inventor and scientist Nikola Tesla. According
I dl sin Q
to Biot-Savart law.

ee
dBcc
r^ I dl sin Q
dB =

Fr
I dl sin 0 4 71
or dB=K.
If I=lA,dl =lm,r=l m and 0 =90° so that sin 0=1,
then
for
ur
The proportionality constant K depends on the
ho
medium between the observation point P and the dB =
4 TC
current element and the system of units chosen. For
ks
free space and in SI units. 471x10“^
= 10 ^ tesla
Yo
oo

ho 4 7t
K =
Ak
= 10 ^ T mA ^ (orWbm ^A ’)
Thus one tesla is 10^ times the magnetic field pro
eB

Here is a constant called permeability of free duced by a conducting wire of length one metre and carrying
space. So the Biot-Savart law in SI units may be current of one ampere at a distance of one metre from it and
expressed as
r

perpendicular to it.
ou

= ho ^ sin 0
ad

4tc 4.4 BIOT-SAVART LAW VS. COULOMB'S LAW


Y

We can write the above equation as 4. Give some points of similarities and differences
go ^ dir sin Q betiveen Biot-Savart law for the magnetic field and
nd
Re

dB =
4k r^ Coulomb's law for the electrostatic field.
Comparison of Biot-Savart law with Coulomb's
Fi

As the direction of dh is perpendicular to the plane law. According to Coulomb's law, the electric field
of dt and r^. so from the above equation, we get the produced by a charged element at a distance r is given
by
vector form of the Biot-Savart law as 1 dq
rf£ =

rfg _hpf X ^
4716,,0

47C According to Biot-Savart law, the magnetic field


produced by a current element I dl at a distance r is given
Direction of d^.The direction of dB is the direction by
of the vector d? x r . It is given by right hand screw rule. dB = gQ 1 dl sin &
Ak
If we place a right handed screw at point P per
pendicular to the plane of paper and turn its handle On comparing the above two equations, we note
the following points of similarities and differences
from dt to r, then the direction in which the screw between the two laws.
PHY5ICS-XII

Points of similarity : Examples Based on


1. Both fields depend inversely on the square of Biot-Savart Law
the distance from the source to the point of
observation. Formula Used
Pq Idl sin 0
2. Both are long range fields. Biot-Savart law, dB =
4n
3. The principle of superposition is applicable to
both fields. This is because the magnetic field is Units Used
linearly related to its source, namely, the current
Magnetic field B is in tesla, current / in ampere
element 1 ^ and the electrostatic field is related and distance r in metre.

linearly to its source, namely, the electric charge. Constant Used


Points of difference : —7 —1

Permeability constant, p g = 4;c x 10 Tm A


1. The magnetic field is produced by a vector
source ; the current element J df. The electro
Example 1. A wire placed along the north-south direction
static field is produced by a scalar source : the carries a current of 8 A from south to north. Find the
electric charge dq. magnetic field due to a 1 cm piece of wire at a point 200 cm
2. The direction of the electrostatic field is along north-east from the piece.
the displacement vector joining the source and Solution. Tine problem is illustrated in Fig. 4.4.
the field point. The direction of the magnetic
field is perpendicular to the plane containing /TP
the displacement vector ^ and the current r.' N
45° '
element IM. dl

W E
3. In Bio-Savart law, the magnitude of the O

magnetic field is proportional to the sine of the


an
gle between the current element / d/ and
displacement vector r while there is no such Fig. 4.4

angle dependence in the Coulomb's law for the As the distance OP is much larger than the length
electrostatic field. Along the axial line of the of the wire, we can treat the wire as a small current
current element 0=0°, sin 0=0 and hence element.
dB = 0.
Here /=8A, fr/=1 cm =1 x 10“^ m,
5. Write a relation between pg, Bq and c r=200cm=2m, 0=45°

Relation between Pg, £g and c. We know that I dl sin 0

= 9 X 10^ Nm^ Cr^ 4n


47te 0
4tu X 10 ^ 8 X 1X 10 ^ X sin 45°
and ^ = 10"^ Tm A"^ 4n 2^
4n

f4ne 0 = 1.4 X 10"^ T.


Po^o =
(Po
4k 1
The direction of the magnetic field at point P is
1 1 normally into the plane of paper.
= 10‘^x
9x10^ (3x10®)^
—^ A

Example 2. An element Al =Axi is placed at the origin


But 3 X 10® ms ^ = speed of light in vacuum (c) and carries a large current 1 = 10 A What is the magnetic
1
field on the y-axis at a distance of 0.5 m. Ax = lcm
INCERT; OD 19]

1 Solution. Here frl = Aj: =1 cm =10“^m, 1=10 A,


or

-JPo ^0 r = y=0.5m, 0=90°, pg/4;: = 10"^ Tm A‘^


MAGNETIC EFFECT OF CURRENT

-y

p
p@

0.5 m
o
x'-< X

/Ai
♦●X

■w Ax H-
z

Fig. 4.5 Fig. 4.6

According to Biot-Savart law, HINTS

w
Pq Idl sin 0 1. Proceed as in Example 1.
2. Proceed as in Example 2.
3. Proceed as in Example 2.

Flo
10”^xl0xl0“^xsin 90°
(0.5)2
We shall now apply Biot-Savart law to calculate the

ee
= 4x 10"® T magnetic field due to {/) a straight current carrying

Fr
The direction of the field dh will be the direction of conductor and (n) a circular current loop.
vector ^ X r . But 4.5 MAGNETIC FIELD DUE TO A LONG
for
ur
A A A A A
STRAIGHT CURRENT-CARRYING
X r = Aa: 2 X y / = Ax y (2 X y ) ^ Ax y k
CONDUCTOR
Hence field dB is in the + 2-direction.
6. Apply Biot-Savart law to derive an expression for
ks

the magnetic field produced at a point due to the current


Yo
r
oo

roblems For Practice flowing through a straight wire of infinite length. Also
draw the sketch of the magnetic field. State the rules used
eB

1. A wire placed along east-west direction carries a


current of 10 A from west to east direction. for finding the direction of this magnetic field.
Determine the magnetic field due to a 1.8 cm piece Magnetic field due to a long straight current-
carrying conductor. As shown in Fig. 4.7, consider a
r

of wire at a point 300 cm north-east from the piece.


ou

(Ans. 1.4 X10"^ T, normally out straight conductor XY carrying current I. We wish to
ad

of the plane of paper) find its magnetic field at the point P whose
Y

2. A small current element I df, with ^ = 2k mm perpendicular distance from the wire is a i.e., PQ = a.
Y
nd

and / = 2 A is centred at the origin. Find magnetic


Re

field dB at the following points :


Fi

(i) On the x-axis at x = 3 m.


-n j
(Ans. 4.44 x 10 ;T) Q *JP
(ii) On the x-axis at x = - 6 m.
-11 : I
(Ans.-1.11x10 ;T)
0
(Hi) On the z-axis at 2 = 3 m. (Ans. 0)
dl O

3. An element a1 =Axi is placed at the origin (as shown


in Fig. 4.6) and carries a current / = 2 A. Find out the U. /
magnetic field at a point Pon the i/-axis at a distance X
of 1.0 m due to the element Ax = 1cm. Give also the
direction of the field produced. [CBSE D 09C]
Fig. 4.7 Magnetic field due to a straight current-
(Ans. 2xl0"'^T, in -i- z-direction) carrying conductor.
4^6 PHYSICS-XII

Consider a small current element ^ of the Special Cases


1. If the conductor XY is infimtely long and the point P
conductor at O. Its distance from Q is / i.e., OQ = 1. Let r
lies near the middle of the conductor, then = ij^ = ti/2.
be the position vector of point P relative to the current B= [sin90° + sin90°]
element and 0 be the angle between d! and 4tm
I
According to Biot-Savart law, the magnitude of the or B = 1^0
2na
field dh due to the current element ^ will be
2. If the conductor XY is infinitely long but the point P
dB = Pq i d/ sin 0 lies near the end Y (or X), then (|)^ =90® and (j^ =0®.
4 Ti I
B = Its. [sin 90° + sin0°] =
4jifl 4na
From right AOQP,
0 + (l) =90° Clearly, the magnetic field due to an infinitely long
or 0 = 90° -
straight current carrying conductor at its one end is
just half of that at any point near its middle, provided
sin 0 = sin (90° - <|)) = cos the two points are at the same perpendicular distance
a from the conductor.
Also cos (fi = -
r
3. If the conductor is of finite length L and the point
or r =
a
= a sec (f)
P lies on its perpendicular bisector, then 4^ = (^ = (j) and
cos 4» L/2 L
sin 41 =
As tan (ji = -
/
a
yja^ + (L/2f -/WT?
Pq I
l = a tan <f>
B= [sin (j) + sin
4tm
On differentiating, we get 2L

dl = a sec ([) d^ 47tfl


■^4fl^ + I?
Hence dB =
fig /(fl sec^ 4» d(|)) COS ({)
●y n or B =
Po^^
4 71 a sec 4)
2na^j^a^ + 1?
^0^
or dB = cos 4> d^
Ana Direction of magnetic field. For an infinitely long
conductor.
According to right hand rule, the direction of the
magnetic field at the P due to all such current elements B =
2na
will be in the same direction, namely, normally into the
-» 1
60c-
plane of paper. Hence the total field B at the point P i.e.,
a

due to the entire conductor is obtained by integrating


Clearly, the magnitude of the magnetic field will be
the above equation within the limits - 4>| and 4^. same at all points located at the same distance from the
^2 ‘{'2
Po^ conductor. Hence the magnetic lines of force of a straight
B= dB = cos 4> d^
Ana
I I

= ^[sin4)]?;2
Arm
.0:
Ana
[sin4^-sin(-4)j)]

■'! 1
or B =
4tU2
[ sin 4\ + sin <^ ]
(«) (i’)
This equation gives magnetic field due to a finite
wire in terms of the angles subtended at the Fig. 4.8 Magnetic lines of foce of a straight
observation point by the ends of the wire. current-carrying conductor.
MAGNETIC EFFECT OF CURRENT

current-carrying conductor are concentric circles with the e

wire at the centre and in a plane perpendicular to the wire.


[A line of force is a curve, the tangent to which at any
point gives the direction of magnetic field at that
point]. If the current flows upwards, the lines of force
have anticlockwise sense [Fig. 4.8(fl)j and if the current
flows downwards, then the lines of force have
clockwise sense [Fig. 4.8(F)].
o
Distance -*
Rules for finding the direction of magnetic field
due to straight current carrying conductor. Either of pig. 4.11 Variation of B with distance from a straight conductor,
the following two rules can be used for this purpose :
1. Right hand thumb rule. If we hold the straight Examples based on
conductor in the grip of our right hand in such a luay that

w
the extended thumb points in the direction of current, then Magnetic Field due to Straight
the direction of the curl of the fijigers will give the direction Current'Carrying Conductor
of the magnetic field (Fig. 4.9). Formulae Used

Flo
1. Magnetic field due to a straight conductor of finite
length.

ee
Fr
B= (sin (|j + sin 4^)
4tm

2. Magnetic field due to an infinitely long straight


conductor.
for
ur
2raj
ks
S Units Used
Yo

N S Magnetic field B is in tesla, current I in ampere


oo

and distance a in metre.


eB

Example 3. A current of 10 A is floiuing east to west in a


Fig. 4.9 Right hand rule for field due to a straight conductor.
long wire kept horizontally in the east-west direction. Find
magnetic field in a horizontal plane at a distance of
r

2. Maxwell's cork screw


ou
ad

(i) 10 cm north
rule. If a right handed screw be
(ii) 20 cm south from the wire;
Y

rotated along the wire so that it


advances in the direction of and in the vertical plane at a distance of
current, then the direction in (Hi) 40 cm downward and
nd
Re

ivhich the thumb rotates gives the (iv) 50 cm upward.


direction of the magnetic field
Fi

Solution, (i) Magnetic field in a horizontal plane at


(Fig. 4.10). 10 cm north of the wire is
Variation of magnetic field 4nxl0"^xl0
= 2 X 10"^ T
with distance from straight 27cr 2tcx0.10
current carrying conductor.
For a straight current-carrying According to right hand thumb rule, the direction
conductor. of the magnetic field will be downward in the vertical
Fig. 4.10 Cork screw rule
for field due to a
plane.
Boc- straight conductor. (ii) Magnetic field at 20 cm south of the wire is
a
4tix10‘^x10
Ss = = 1 X 10“® T
27TX0.20
Thus the graph plotted between the magnetic field
B and the distance a from the straight conductor is a The magnetic field will point upward in the vertical
hyperbola, as shown in Fig. 4.11. plane.
PHYSICS-XII

{Hi) Magnetic field 40 cm just down the wire is


4tix10“^x10 2 Tin
1 2Tcr2
B,D “
= 5 X 10'® T
271x0.40
4tix 10 ^ r 20 30
2ti 0.10 0.30
The magnetic field will point south in a horizontal
plane. = 2 X 10 ^ T, pointing normally outward.
(I'y) Magnetic field 50 cm just above the wire is {ii) At point Q, both 6j and will point normally
4jix lO'^x 10 inward.
= 4x 10"^ T
2tcx 0.50 47Txl0~^r 20 30
+
2ti 0.10 O.IOJ
The magnetic field will point norlh in a horizontal
plane. = 10"*^ T, pointing normally inward.
Example 4. A long wire with a small current element {Hi) At point R, Bj points normally inward and ^
of length 1 cm is placed at the origin and carries a current of points normally outward.
10 A along the X-axis. Find out the magnitude and direction 4tix10-^ r 30 20
of the magnetic field due to the element on the Y-axis at a 2ti 0.10 0.30 .
distance 0.5 mfrom it. [CBSE OD 19]
Solution. Y = 4.5 X 10"^ T, pointing normally outward.
Here / = 10 A, Example 6. Two parallel wires P and Q placed at a sepa
P
dl = l cm = 0.01 m ration of r = 6an carry electric currents f=5A and
OP =0.5 m, 0.5 m I2 =2 A in opposite directions as shown in Fig. 4.13. Find
0=90®.
the point on the line PQ where the resultant magnetic field is
zero.
O 0
I1 h
Magnetic field 10 A
●►X

at point P,
dl 0 0
P Q R
. -^t^^sine r - X

4ti r^
10”^xl0x0.01xsin90® Fig. 4.13
8
T = 4xlO"°T.
(0.5f Solution. At the required point, the resultant
magnetic field will be zero when the fields due to the
According to right hand thumb rule, the direction two wires have equal magnitude and opposite direc
of the magnetic field at the point P points normally tions. Such point should lie either to the left of P or to
outwards.
the right of Q. But the wire Q has a smaller current, the
Example 5. Figure 4.12 shows two current-carrying wires point should lie closer to and to the right of Q. Let this
1 and 2. Find the magnitudes and directions of the magnetic point be R at distance x from Q, as shown in Fig. 4.13.
field at points P, Q and R. Field due to current f at point R,
20 cm
Po^i
Bi =
2K{r+x)'
20 A 30A
normally into the plane of paper.
Field due to current at point R,
●*— 10 cm —●
P
10 cm
Q
10 cm —«*— 10 cm
R
Pq h
2kx '

normally out of the plane of plane


But Bi = ^
Fig. 4.12
-^1 _ h
Solution, (i) According to right hand grip rule, the r + x X

field Bj of wire 1 at point P will point normally outward ^2 ^ _ 2 A X 6 cm = 4 cm.


while the field ^ of wire 2 will point normally inward, or X =

hence h-h 5A-2A


MAGNETIC EFFECT OF CURRENT

Example 7. Use Biot-Savart law to obtain an expression I 4.5 X 10'^


a = m

for the magnetic field at the centre of a coil bent in the form of 2 tan 60® 2j3
a square of side 2a carr\/ing current 1. 3x471x10 ^xl.0x2-\/3
Solution. Refer to Fig. 4.14. Magnetic field at O due B = [sin 60®+ sin 60°]
4ti X 4.5 X 10'^
to finite length of wire AB is
6^^x 10"^[V3 ,Js' = 4 X 10'® T,
B,1 = (sin a + sin P) 4.5 2 2
47tfl

directed normally outwards.


(sin 45® + sin 45°) =
4na 47Cfl
Example 9. Figure 4.16 shows a right-angled isosceles
A
I
D
APQR having its base equal to a. A current of I ampere is
passing downwards along a thin straight wire cutting the

w
''
plane of paper normally as shown at Q. Likewise a similar
I I zvire carries an equal current passing normally upwards at
45°>'
R. Find the magnitude and direction of the magnetic
induction B at P. Assume the wires to be infinitely long.

Flo
I
B C [ISCE 97]
2a

ee
Fig. 4.14

Fr
The magnetic field at O due to conductors BC, CD
and DA will also be of same magnitude and direction.
for
ur
Therefore, resultant field at O is

B = 4 Bj =
ks
4TCfl na
Yo

Solution. Let PQ = QR - r. In right APQR,


directed normally outwards.
oo

Example 8. A current of 1.0 A is flowing in the sides of an or


"~>I2
eB

equilateral triangle of side 4.5x10"^ m. Find the magnetic


field at the centroid of the triangle. [Roorkee 91] Magnetic induction at point P due to the conductor
Solution. The situarion is shown in Fig. 4.15. The passing through Q,
r
ou

magnetic field at the centre O due to the current


ad

through side PQ is given by , acting along PR


2nr 2ita -J2 TUI
Y

4na
[sin 0j + sin 82] Magnetic induction at point Pdue to the conductor
nd

passing through R,
Re

where a is the distance of PQ from O and 0j, 02 are the


angles as shown. The magnetic field due to each of the , acting along PQ
Fi

three sides is the same in magnitude and direction,


therefore, total magnetic field at O is As the two fields at point P are acting along
perpendicular directions, the resultant magnetic
B = 3 Bj = [sin 0j + sin ©2] induction at point P is
47Cfl

Here / =1.0 A, 0^ = 02 =60°


|o.Q =47txlO”^Tm A'^ Fo^ +
Fo^ = ^/2.
PS na yj2 na ■J2 na
OS
- tan 0j P Q

or 6=
in na
or = tan 60°
a
Fig. 4.15 This field acts towards the midpoint of QR.
PHYSiCS-XII

roblems For Practice 9. A long straight wire carrying a current of 200 A,


runs through a cubical box, entering and leaving
1. A straight wire carries a current of 3 A. Calculate through holes in the centres of opposite faces, as
the magnitude of the magnetic field at a point 10 cm shown in Fig. 4.19. Each side of the box is of 20 cm.
away from the wire. [CBSE D 96]

(Ans. 6 X 10"^ T) *A
D

2. At what distance from a long straight wire carrying


a current of 12 A will the magnetic field be equal to
F Q C
3 X 10"^ Wb m"^. (Ans. 8 x 10"^ m)
3. A long straight wire carrying a current of 30 A is B*

placed -Ain an external uniform magnetic field of


4.0x10 T parallel to the current. Find the
Fig. 4.19
magnitude of the resultant magnetic field at a point
2.0 cm away from the wire. (Ans. 5x10'^ T) Consider an element PQ of the wire 1 cm long at the
4. What current must flow in an infinitely long centre of the box. Calculate the magnetic field
straight wire to give a flux density of 3x10“^ T at produced by this element at the points A, B, C and
6 cm from the wire ? (Ans. 9 A) D. The points A, B and C are the centres of the faces
5. A vertical wire in which a current is flowing of the cube and D is the midpoint of one edge.
produces a neutral point with the earth's magnetic (Ans. 20 X10"^ T, 20 x 10"^ T, 0, 7.07 x lO”^ T)
field at a distance of 10 cm from the wire. What is
10. A long straight telephone cable contains six wires,
the current if B^H = 1.8 x 10"^ 1 ' (Ans. 90 A)
each carrying a current of 0.5 A. The distance
6. Figure 4.17 shows two between the wires is negligible. What is the
h 5.0 cm-*
long, straight wires magnitude of magnetic field at a distance of 10 cm
carrying electric currents from the cable (i) if the currents in all the six wires
of 10 A each in opposite 10 A '' lOA are in same direction (ii) if four wires carry current
directions. The separa in one direction and the other two in opposite
tion between the wires is P
direction. [Ans. (0 6.0xl0"^T, (») 2.0x10'^ T]
5.0 cm. Find the magnetic
field at a point P midway 11. Calculate the magnetic induction at the centre of a
between the wires.
Fig. 4.17 coil bent in the form of a square of side 10 cm
(Ans. 1.6x10“^ T) carrying a current of 10 A. [Punjab 01]

7. Two long parallel wires are placed at a distance of (Ans. 1.13 X 10"^ T)
16 cm from each other in air. Each wire has a 12. A closed circuit is in the form of a regular hexagon
current of 4 A. Calculate the magnetic field at of side ih If the circuit carries current I, what is
midpoint between them when the currents in them magnetic induction at the centre of the hexagon ?
are (i) in the same direction and {ii} in opposite [IPUEE 13]
directions. [Ans. (0 Zero (/0 2xl0"^ T]
Ans. B =
8. Two infinitely long insulated wires are kept per Tt a

pendicular to each other. They carry currents /j = 2 A


13. Two straight long conductors AOB and COD are
and I2 = 1.5 A. (/) Find the magnitude and direction
perpendicular to each other and carry currents
of the magnetic field at P. {ii) If the direction of
current is reversed in one of the wires, what would
and I2 respectively. Find the magnitude of the mag
netic field at a point Pat a distance a from the point
be the magnitude of the field B ?
Oin a direction perpendicular to the plane ABCD.
[Ans. (/) 2x10 ® T, normally into Po
Ans.
the plane of paper (ii) zero] 2TTrt

id,
h
14. Two insulating infinitely long conductors carrying
currents and I2 lie mutually perpendicular to
3 cm each other in the same plane, as shown in Fig. 4.20.
*P Find the magnetic field at the point P{a, b).
4 cm
j \
Ans.^ h — , directed inward
Fig. 4.18 2k {b a
MAGNETIC EFFECT OF CURRENT

CO

8. (/)
Tj 4n X 10“^ X 2 = 10"^ T,
a P{a, b) 2rrrj 2nx4xl0”^
I1
b normally into the plane of paper.
CO- - - - CO
M-O ^2 = 10'^ T,
0
h ^2 =
271^2 2n X 3 X 10”^
normally into the plane of paper
B= =2x10“^ T,
CO

normally into the plane of paper.


Fig. 4.20
(lY) When current in any one wire is reversed, the
two fields will be in opposite directions, so that
HINTS B = zero.

Hq/ _ 4n X10 ^ x3

w
1. B = = 6.0x10'^ T. 9. Here 7 = 200 A, PQ = d/= 1cm = 0.01 m
2nr 2iixO”lO
For point A or B, r=10cm=0.1m, 0 = 90°,
2. r =
\1qI 4ti X X 12 = 8 xl0"^m. therefore

Flo
2^ 27tx3xl0"^ 7 dl sin 0

3. Field due to straight current carrying wire is 47T

ee
10"^ x200x0.01xsin 90°
\1q I 4n xlO”^ x30 = 3.0x10""^ T = 20x10"® T
(0.1)2

Fr
2nr 2n X 2.0 X 10~^

This field will act perpendicular to the external For point C, 0 = 0°, therefore
field B2 = 4.0x10“^T.
for 7 dl sin 0°
ur
= 0.
^ 4n .2
B= b2 =.J(3x10"'‘)2 + (4.0x10“^)2 1

For point D,
= 5x10'^ T.
ks

r = t/i?7T? = ICK/2 = 0.1 V2m, 0 = 45°


Yo
cm
27crB 2ji x6 X10 ^ x3xl0 ^
oo

4. 7 = = 9 A.
^0 4rcxl0"^ 7 dl sin 45° 10~^ x 200x0,01x1
B -liio
eB

D
4n xV2
5. If neutral point is obtained at distance r from the
wire, then = 7.07x10"® T.
lifli 10, (i) Net current, 7 =0.5 x 6 = 3.0 A,r = 10 cm = 0.1 m
r

= B
H
ou

2nr
4n X 10"^ X 3.0
ad

-4 B= = 6.0 xlO"® T.
27ir B 2jt X 0.10 X 1.8 X10 2nr 2k X 0.1
Y

H _ = 90 A.
or 7 =
4n X10"^
(ii) Net current, 7 = 0.5 x 4 - 0.5 x 2 = 1.0 A
6. According to right hand thumb rule, the direction |i,Q 7 4n X 10"^ X 1.0 = 2.0x10"®
nd
Re

B = T.
of magnetic field due to current in each wire is ^
2nr 2k X 01
perpendicular to and pointing into the plane of
Fi

paper. Hence total field at point P is 11. Refer to Fig. 4.21. Magnetic field at 0 due to finite
wire AB,
B = 2x iV.
271 r
D 10 cm C

2 X 4ti X10"^ X10


= 1.6 xlO"* T.
2k X 2.5 X 10"^
5 10 cm ^ ,10cm
r = ~ cm = 2.5x10"^ m .Aj A
2
,'45^5°'.
7. (i) When the currents are in same direction. '5 cm

B=B,-B, A 10 cm B

(li) When the currents are in opposite directions.


B= B, + Bj. Fig. 4.21
PHYSICS-XII

(sin a + sin p) As < fl, so S, > B|


4tw
Hence the net magnetic field at the point P,
4tt X 10"^ X 10 1
(sin 45° + sin 45°) B=K~R
4k X 0.05 ^ ^ 2n[b a

= 2.83xlO“^T
directed normally inward.
Total magnetic induction at O,
B = 46, = 4 X 2.83 x 10“^ = 1.13 x 10“'' T, 4.6 MAGNETIC FIELD AT THE CENTRE OF
directed normally outward. CIRCULAR CURRENT LOOP

12. ON = fl' = a
2 0^ V3(7
7. Apply Biot-Savart law to derive an expression for
4 2
E D
the magnetic field at the centre of a current-carrying
circular loop.
Magnetic field at the centre of a circular current
loop. As shown in Fig. 4.23, consider a circular loop of
'0/ wire of radius rcarrying current I. We wish to calculate
F C
its magnetic field at the
centre O. Tlie entire loop
|30'’N, [ can be divided into a
large number of small
V'^g/2 ^!: a/2 '
current elements.
A N B I

Consider a current
Fig. 4.22
element dt of the loop.
Magnetic field at O due to current in AB is
According to Biot-Savart
[sin a + sin p] law, the magnetic field at
4k a' the centre O due to this
Po ‘ element is
Fig. 4.23 Magnetic field at the
[sin 30° + sin 30°] =
4kV3a/2
dB = ^ df_ X r centre of a circular current loop.

Total field at 0 = 6 = 4k
Kfl
The field at point O points normally into the plane
13. Magnetic field at P due to current in wire AOB, of paper, as shown by encircled cross 0. The direction
1^0 A of df is along the tangent, sodt 1 r . Consequently, the
2kk
magnetic field at the centre 0 due to this current
Magnetic field at Pdue to current in wire COD, element is
d/sin90° dl
dB =
B> = 4k 4k
2m

As the two conductors are perpendicular to each The magnetic field due to all such current elements
other, so 6^ and ^ will also be perpendicular to each will point into the plane of paper at centre O. Hence
other. Hence the resultant magnetic field at Pis the total magnetic field at the centre O is
\2 ll/2 PqI dl _ Ppi
2m
+
PQ^2 6= J dB= J 4k 4Kr^'
dl
2m ,
./ = .2Kr
_ jio 4Kr^ 47ir^
2k«
aj +12
14. Magnetic field at point Pdue to current /V or 6=
2r
directed normally inward
2ka If instead of a single loop, there is a coil of N turns,
Magnetic field at point Pdue to current J2' all wound over one another, then
Po^2 B =
2k/j
directed normally outward 2r
MAGNETIC EFFECT OF CURRENT 4.13

4.7 MAGNETIC FIELD ON THE AXIS OF Total magnetic field at the point F in the direction
CP is
A CIRCULAR CURRENT LOOP
B = dB sin ^
8. Apply Biot-Savart law to find the magnetic field Idl
dB - iio
a
due to a circular current-carrying loop at a point on the But sin 4>= - and
4 Tt
axis of the loop. State the rules used to find the direction
s

of this magnetic field. B=


Fo _Idf a
4 JT s
Magnetic field along the axis of a circular current
loop. Consider a circular loop of wire of radius a and Since pg and I are constant, and s and a are same for
carrying current I, as shown in Fig. 4.24. Let the plane all points on the circular loop, we have
of the loop be perpendicular to the plane of paper. We
B= dl =
Fg/fl .2tw =
Fg/fl^
wish to find field B at an axial point F at a distance r 4tcs^ 4ro^ 2s^
from the centre C.

w
[●.' til = circumference =2ti fl]
dB
dB, or 6 =
di
2{r^ + a^f^

Flo
As the direction of the field is along +ve
X-direction, so we can write

ee
>X
P
dB:,
B =

Fr
z I
2(r^ + a^f^
If the coil consists of N turns, then

for
ur
B=
2{r^ + a^f^
Fig. 4.24 Magnetic field on the axis of a
circular current loop. Special Cases
ks

1. At the centre of the current loop, r-0, therefore


Yo

Consider a current element fr/ at the top of the loop.


oo

It has an outward coming current. B=


eB

2a^ 2a
If s be the position vector of point F relative to the
N lA
ho
element dt, then from Biot-Savart law, the field at or B=
2TCfl^
r

point F due to the current element is


ou
ad

Fo I dl sin 0 where A = = area of the circular current loop. The


dB =
field is directed perpendicular to the plane of the
Y

4 Tt
current loop.
Since df 1 i.e., 0 =90°, therefore 2. At the axial points lying far away from the coil,
nd
Re

Idl r » a, so that
dB =
Fo
Fi

4 7C B =
2r3 2nf^

The field dB lies in the plane of paper and is This field is directed along the axis of the loop and
falls off as the cube of the distance from the current loop.
perpendicular to s, as shown by FQ . Let 4>be the angle
between OP and CP. Then dB can be resolved into two 3. At an axial point at a distance equal to the
rectangular components. radius of the coil i.e., r = a, we have

1. dB sin (j) along the axis, Fq _Fo NI


B=
2. dB cos (j) perpendicular to the axis. 2{a^ + a^f^ 2^^^a‘
For any two diametrically opposite elements of the
loop, the components perpendicular to the axis of the Direction of the magnetic field. Fig. 4.25 shows the
loop will be equal and opposite and will cancel out. magnetic lines of force of a circular wire carrying
Their axial components will be in the same direction, current. The lines of force near the wire are almost
i.e., along CP and get added up. concentric circles. As we move radially towards the
PHYSICS-XII

centre of the loop, the concentric circles become larger the magnetic field along the axis of a circular loop with
and larger i.e., the lines of force become less and less distance from its centre. The value of Bis maximumat
curved. If the plane of the circular loop is held the centre, and it decreases as we go away from the
perpendicular to the magnetic meridian, the lines at centre, on either side of the loop,
the centre are almost straight, parallel and perpen
dicular to the plane of the loop. Thus the magnetic field Examples based on
is uniform at the centre of the loop. Magnetic Field due to a Circular Coil
N
Formulae Used

1. Magnetic field at the centre of a circular loop,


B =
Ir

n 2. Magnetic field at an axial point of a circular loop,


I B =

Units Used
S
Magnetic field B is in tesla, current in ampere,
Fig. 4.25 Magnetic lines of force of a circular current loop. distances r and a in metre.

Constant Used
Rules for finding the direction of a magnetic field
due to a circular current loop. Either of the following Pq = 4tcx 10 ^ Tm A \
two rules can be used for finding the direction of
Example 10. The plane of a circular coil is horizontal. It
1. Right hand thumb rule. Ifzve curl the palm of our has 10 turns each of radius 8 cm. A current of 2 A flows
right hand around the circular wire with the fingers through it. The current appears to flow clockwise from a
pointing in the direction of the current, then the extended point above the coil. Find the magnitude and direction of the
thumb gives the direction of the magnetic field.
magnetic field at the centre of the coil due to the current.
2. Clock rule. This rule gives the polarity of any face of Solution. Here N =10, r = 8 cm = 0.08 m, / = 2 A
the circular current loop. If the current round any face of the
coil is in anticlockwise direction, it behaves like a north pole. B= Pq NJ _ 47tx 10"^ X 10x2 = 1.57 X 10"^ T
If the current flows in the clockwise direction, it behaves like 2r 2 X 0.08

a south pole (Fig. 4.26). As the current flows clockwise when seen from
above the coil, the magnetic field at the centre of the
coil points vertically downwards.
Example 11. the Bohr model of hydrogen atom, an
electron revolves around the nucleus in a circular orbit of
-11
radius 5.11 x 10 m at a frequency of 6.8 x 10^^ Hz. What
Fig. 4,26 Clock rule.
is the magnetic field set up at the centre of the orbit ?
Variation of the magnetic field along the axis of a Solution. If n is the frequency of revolution of the
circular current loop. Fig. 4.27 shows the variation of electron, then
15 -19
I = ne=6.8 x 10 X 1.6x10 =6.8xl.6xl0“^A

B= Po / _ 4n X 10"^ X 6.8 X 1.6 X 10"^ = 13.4 T.


-11
2r 2 X 5.11 X 10

Example 12. TTie radius of the first orbit of hydrogen atom


is 0.5 A. The electron moves in an orbit with a uniform
speed of 2.2 x 10^ Calculate (i) the equivalent current
(ii) the equivalent dipole moment and (Hi) the magnetic field
O Distance -> produced at the centre of the nucleus due to the motion of
Fig, 4.27 Variation of B along the axis of a
thiselectron ? Use Pq / 4n = 10~^ NA~^ atid electronic charge
= 1.6x10"^^ C. PSCE 98]
circular current loop.
MAGNETIC EFFECT OF CURRENT 4.15

Solution, (i) Here r = 0.5 A =0.5 x 10


-10
m, Here a =12 cm =12 x 10 ^m, r = 5 cm = 5 x 10 ^ m,
V =2.2 X 10^ ms"^ B
centre
=0.50x 10“^T

Period of revolution of electron, (12 x


-10 Raxial X 0.50 X 10“^ T
T =
2;cr 2x22x0.5x10
= Axl0'*5g [144xl0"^+25xl0^f/^
7x2.2x10^ 7
(12)^x0.50x10
V -4

= 3.9 X 10”® T.
Equivalent current. 169x13
-19

1 = Charge _ e 1.6x10 x7
-15
= 1.12x 10"^ A. Example 15. Tzuo identical circular coils of radius 0.1 m,
Time T 10
each having 20 turns are mounted co-axially 0.1 m apart. A
{ii) Equivalent dipole moment, current of 0.5 A is passed through both of them (i) in the
m= M = J X nr^ same direction, (ii) in the opposite directions. Find the
magjiefic field at the centre of each coil.

w
= 1.12 X10^^ X 3.14 X (0.5 X10"^*^)^ Am^
Solution. Here a =0.1 m, N = 20, r= 0.1 m, / = 0.5 A
= 8.8x10"^'* Am ^
Magnetic field at the centre of each coil due to its
(Hi) Magnetic field produced at the centre of the

Flo
own current is
nucleus.
Pol _4;rx 10"^ X 1.12x10”^ B,1 = _ Pq M _ 4jt X 10 ^ X 20 X 0.5 = 6.28 X 10"^ T
B = = 14.07 T.

ee
●10 2fi 2x0.1
2r 2x0.5x10

Magnetic field at the centre of one coil due to the

Fr
Example 13. A circular coil, having 100 turns of wire, of current in the other coil is
radius (nearly) 20 cm each, lies in the XY plane with its
centre at the origin of co-ordinates. Find the magnetic field, I^qN/a^
for
ur
at the point {0,0,20f3 an), when this coil carries a current of 2(A2 + r^2
-1a 4ti:x10”^x20x0.5x(0.1)^ 0.628 X 10"^
\n) (CBSE D 16C)
ks
Solution. Here N = lOO, a = 20 cm = 0.2 m
2[(0.i)^+(0.1)^f^^ [2 X (O.l)^]^^^
Yo

0.628 X 10“^
oo

z=20V3 cm =0.2^3 m, 1=-A =2.22 X 10'® T.


7t 2^/2 X 10"®
eB

The coil lies in XV-plane and the field point (i) When the currents are in the same direction, the
(0,0,20V3)lies on 2-axis. resultant field at the centre of each coil is
r

Magnetic field at the axial field point, B = 6, + ^ = 6.28 X 10"® + 2.22 x 10"®
ou
ad

^2^
47txl0"^xl00x X (0.2)2 = 8.50 X 10"® T.
\TtJ
Y

B= T
{ii) When the currents are in opposite directions,
2(a2 + z2)3/2 2[(0.2)^ + (0.2^^)^f/^ the resultant field is
4 X10"® X 0,04 0.16x10"®
nd

B = Bj - ^ = 6.28 X 10"® - 2.22 x 10"®


Re

T = T
~2(0.2)®(l + 3)®''^ 2 X 0.008x8
= 4.06 X 10"® T.
Fi

= -Jx
8
10"‘*T =25x 10"^T = 25 pT. Example 16. Two coaxial circular loops L, and of radii
3 cm and 4 cm are placed as shown. What should be the magni
Example 14. The magnetic field due to a current-carrying
tude and direction of the current in the loop so that the net
circular loop of radius 12 cm at its centre is 0.50 x 10“^ T.
magnetic field at the pomt O be zero ? (CBSE SP 081
Find the magnetic field due to this loop at a point on the axis
at a distance of 5.0 cm from the centre.
Solution. B and Baxial
2{a^^r^f^
centre
2a

3
Baxial _ a

B
centre (A^ +
3
A
or B X B
axial centre
6 PHYSICS-XII

Solution. For the net magnetic field at the point O centre of the arc ? Would your answer change if the wire were
to be zero, the direction of current in loop should be bent into a semicircular arc of the same radius but in the
opposite to that in loop L,. opposite way as shown in Fig. 4.29(b) ? [NCERTJ
Magnitude of magnetic = Magnitude of magnetic Solution, (i) Magnetic field at the centre of the arc is
field due to current field due to current
B=
IjinL, 4r

or
HoJl(0-03f n 0/2(0-04)' Here 7 = 12 A, r =2.0 cm =0.02 m,
Pq =4tcx 10”^ TmA
3/2 “ -1
3/2
2 [(0.03)^ + (0.04)^* 2 [(0.04)2 ^ (0.03)21
47ux10”^x12
_(0.03)^ B = = 1.9 X 10“^ T
or — X 1 A = 0.56 A. 4 X 0.02
"“(0.04)2 16
According to irght hand rule, the direction of the
Example 17. A long wire having a semi-circular loop of
field is normally into the plane of paper,
radius r carries a current I, as shown in Fig. 4.28. Find the
magnetic field due to entire wire at the point O. [ii] The magnetic field will be of same magnitude,
R B=1.9x 10'“^ T
, L

The direction of the field is normally out of the plane


1 I
of paper.
P Q 0 s T
Example 19. A long wire
Fig. 4.28 is bent as shmon in Fig. 4.30. , I

What will be the magnitude


Solution. Magnetic field due to linear portion. Any
and direction of the field at I I
element ^ of linear portions like PQ or ST will make the centre O of the circular
O

angles 0 or tc with the position vector r . Therefore, portion, if a current I is >


P
>
f● 1 j j .. 1● ● passed through the wire ? I I
field at u due to linear portion is . , , .
Assume that the various

1^0 sin 9 portions of the wire do not Fig. 4.30


B=
4n -p touch at point P.
Magnetic field due to semi-circular portion. Any element Solution. The system consists of a straight
i on this porHonwiU be perpendicular to the posiHon conductor and a circular loop. Field due to straight
conductor at point O is
vector r, therefore, field due to one such element at
point will be e,= 27tr ' up the plane of paper
Idl sinn/l _ Pp Idl
471 r2 4 71 r2 Field due to circular loop at point O is

Magnetic field due to the entire circular portion is 2r'


up the plane of paper
given by
/ .●. Total field at O is
B= dB = dl =
Po ^ .Kr = ^*0 ^
n
4 7cr2 47ir2 4r
B=B, + ^ = 1 + — , up the plane of paper.
2r nj
PqI
.'. Total magnetic field at point O = 4r Example 20. Figure 4.31 shows a current loop having two
circular segments and joined by two radial lines. Find the
Example 18. A straight wire carrying a current of 12 A is
magnetic field at the centre O.
bent into a semicircular arc of radius 2.0 cm as shown in I

Fig. 4.29(a). What is the direction and magnitude of B at the

(fc)

Fig. 4.29 Fig. 4.31


MAGNETIC EFFECT OF CURRENT

Solution. Since the point O lies on lines SP and QR, 5A

so the magnetic field at O due to these straight portions


is zero.
b d
The magnetic field at O due to the circular segment O
PQ is
5A
>
^1 =
Fig. 4.33
Here, / = length of arc PQ = a a
Solution. Here I = 7 =2.5 A,
Po fa abc adc
B, =^ , directed normally upward
^ 4n a r = Oa = Ob = Oc = Od = 5 cm = 5 x 10“ ^ m.
Similarly, the magnetic field at O due to the circular The magnetic induction at O due to the current in
segment SR is part abc of the coil is equal and opposite to the

w
^
_^ i_E: directed normally^ downward.
4rt b
magnetic induction due to the current in part adc. So
magnetic induction at O due to the coil is zero.
The resultant field at O is Magnetic induction at O due to the straight conduc

Flo
Pofafl 1' tor pa (a half infinite segment) is
B=B,-B,=
4n b_ _ 1 Upf _47txlQ-^x5 = 10“^T,

ee
PqI (xih-a) ^" 2 27ir” 471x5x10“^

Fr
or B=
4Kab
normally out of the plane of paper.
Example 21. The wire shown in Fig. 4.32 carries a current Similarly, magnetic induction at O due to straight
for
ur
of 10 A. Determine the magnitude of the magneticfield at the conductor qc is
^2 = ^=10-^T,
centre O. Given radius of the bent coil is 3 cm.
[Punjab 01; AIIMS 13] 4Kr
ks

normally out of the plane of paper.


Yo
oo

Total magnetic induction at O is


B= B^ + £^=10“^ +10“^ = 2x 10“^ T,
eB

normally out of the plane of paper.


Example 23. The current-loop PQRSTP formed by two
r

circular segments of radii and carries a current of I


ou
ad

Fig. 4.32 ampere. Find the magnetic field at the common centre O.
Arc
Y

Solution. As 0 (rad) = What zvill be the ifeld if angle a =90° ?


Radius
Solution. Tlie magnetic field at O due to each of the
3k I Snr
straight parts PQ and RS is zero because 0 = 0°, for each
nd

or / =
Re

2 2 of them.

According to Biot-Savart law, magnetic field at the


Fi

centre O is

0 I 3nr_Po 3 nl
4tc 4tu 2 4tc 2 r
I

_ 47tx 10"^ 3 ^ 10
4k 2*7 3x10'^
= 1.57xlO“^T.
Exomple 22. In Fig. 4.33, abed is a circular coil of
non-insiilated thin uniform conductor. Conductors pa and
Magnetic field at tlie centre O due to circular segment
qc are very long straight parallel conductors tangential to
the coil at the points a and c. If a current of 5 A enters the coil QR of radius R2 is
I
from p to a, find the magnetic induction at O, the centre of a=^. 2 I'2
the coil. The diameter of the coil is 10 cm. ^ 4ti R^
4.18 PHYSICS-Xll

Here, Solution. Given / = 10A,


I2 = length of circular segment QR-a R2 Length of each wire = 44 cm = 4L (say)
1 directed normally downward (fl) Suppose the wire is bent into a circle of radius K.
4tc R2 Then its perimeter 2 tc R = 4 L
Similarly, the magnetic field at O due to the circular Magnetic field at the centre of the circular wire is
segment STP is
B= pQ /7i_Pq Ik
-(1)
Pq /(2jr-g) 2R 2kR 4L
47T
, directed normally downward
Now suppose the wire B is bent into a square of
Hence the resultant field at O is
side L. We know that the magnetic field due to a wire
a 27i-a of finite length whose ends make angles a and p with
B= B, + 62 = +
the perpendicular dropped on wire from the given
471 [R2 R
1 J
point at distance r from it is given by
directed normally downward
dB = (sin a + sin P)
If a=90° = rt/2,then 4rtr

B=
K
+
37t i_pqj r 1 3
T
I
D
[2 R^ 2R 1 8
/

Example 24. A current


I -5.0 Aflows along a thin wire i
}
shaped as shown in Fig. 4.35. f O I
The radius of the curved part of
A

s R
the wire is equal to R= 120 mrn,
J
B C
i
the angle 2 4> = 90°. Find the
magnetic induction of the field at A B
Fig. 4.36
the point O. Fig. 4.35
.'. Magnetic field at O due to conductor AB is
Solution. Magnetic induction at O due to the line
segment AB is dB =
I
(sin 45° + sin 45°) =
4%. L/2 4nL
[sin (|)+ sin ([)]
4tc R cos
[V a=p = 45°,r=L/2]
= to 11 tan 4>/ acting normally downwards By symmetry, magnetic field at O due to all the
471 ■ R
four sides of the square will be in the same direction.
Magnetic field at O due to the current through arc Hence total field at O due to the current-carrying
segment is square is
I

^ 4ti R {2n-2^), acting normally downwards B=4x


47tL 4kL
...(2)
Total magnetic induction at O,
/ Comparing equations (1) and (2), we find that the
S= + ^=^.-^[7r-(t)+ tan(|)] square wire produces a greater field at its centre.
4ti X 10“^ X 5 7U 7T (&) Magnetic field at the centre of the circular wire is
71— + tan
27tx0.120 . 4 4
B= P,)/7T _ 4tCX10“^x10x 71 ^
2 xlO'^x 5x3.356 4L 44x10“^
= 2.8 X 10'^ T.
0.120 = 0.9 X 10"^ T [●.● 4 L= 44 cm]
Example 25. Two wires A and B have the same length Magnetic field at the centre of the square wire is
equal to 44 cm and carry a current of 10 A each. Wire A is 8l2 xp^I _8x 1.414X 47cX 10 X 10
B =
bent into a circle and zuire B mto a square, (a) Which zvire 4tiL 7CX 44x 10 ^
produces a greater magnetic field at the centre ? (b) Obtain
the tnagnitudes of the fields at the centres of the tzoo wires. ^ 1.0 X 10"^ T.
MAGNETIC EFFECT OF CURRENT 4.19

Example 26. Two identical coils Pand Q each of radius Rare 5, Two identical circular wires P and Q each of radius
lying in perpendicular planes such that they have a common R and carrying current T are kept in perpen
centre. Find the magnitude and direction of the magnetic field dicular planes such that they have a common centre
at the common centre of the two coils, if they carry currents as shown in Fig. 4.39. Find
Q
equal to I and 431 respectively. [CBSE F 16 ; OD 17] the magnitude and
direction of the net
p magnetic field at the
S2 common centre of the two
coils. ICBSE D 12] I

(Ans , at an angle of
■ flR
45° with either of the

two fields) Fig. 4.39

w
Fig. 4.37 (a)
6. Two identical loops Pand Qeach of radius 5 cm are
ib)
lying in perpendicular planes such that they have a
Solution. Bp -]V vertically upwards. common centre as shown in Fig. 4.40. Find the

Flo
2K' magnitude and direction of
_PqV3J the net magnetic field at the
% along horizontal

ee
2R ' common centre of the two
coils, if the carry currents

Fr
Resultant field at the centre is
-!l/2
equal to 3 A and 4 A P
respectively. ICBSE OD 17]
8 = 7^7^ -I-

for
(Ans. 20ti|.iT, 0 = tan
-1
ur
2R 2R

1/2 _( ^ Fig. 4.40


(1 + 3) 7, A straight wire, of length — metre, is bent into a
ks
2R R
circular shape. If the wire were to carry a current of
Yo

tan 9 = BP ^ 1
oo

=> 0=30° 5 A, calculate the magnetic field, due to it, before


BQ~ 43 bending, at a point distant 0.01 times the radius of
eB

the circle formed from it. Also calculate the magnetic


■' Problems For Practice
field, at the centre of the circular loop formed, for
the same value of current. ICBSE OD 04C]
1. Consider a tightly wound 100 turn coil of radius 10 cm,
r

carrying a current of 1 A. What is the magnitude of (Ans. 1.256 X10“^ T)


ou
ad

the magnetic field at the centre of the coil ? 8. A circular coil has 35 turns and a mean radius of
4.0 cm. It carries a current of 1.2 A. Find the
Y

INCERT] (Ans. 6.28 x 10“^ T)


magnetic field (i) at a point on tlie axis of the coil at a
2. A circular loop of one turn carries a current of 5.0 A. distance of 40 cm from its centre and (//) at the centre
nd

If the magnetic field at the centre is 0.20 mT, find I Ans. (/) 6.5 X 10'^ T (ii) 6.6 x 10^ T]
Re

of tlie coil.
the radius of the loop. (Ans. 1.57 cm)
9. A thick straight copper wire, carrying a current of
Fi

3. What current has to be maintained in a circular coil 10 A is bent into a semicircular arc of radius 7.0 cm

of wire of 50 turns and 2.54 cm in radius in order to as shown in Fig. 4.41(rt). (;) State the direction and
just cancel the effect of earth's magnetic field at a calculate the magnitude of magnetic field at tlie
place where the horizontal component of earth's centre of arc. (;7) How would your answer change if
field is 1.86 x 10"^ T ? (Ans. 0.015 A) the same wire were bent into a semicircular arc of the
4. A straight wire carrying a current of 5 A is bent into same radius but in opposite way as shown in
a semicircular arc of radius 2 cm as shown in Fig. 4.41(1)) ?
Fig. 4.38. Find the magnitude and direction of the [Ans. (0 4.5 X 10"^T, outside the plane of paper,
magnetic field at the centre of the arc. ^-►-.^ (») 4.5xlO"^T, into the plane of paper]

Fig. 4.38 (CBSE OD 19]


(n)
(Ans. 7.85 x 10”^T, normally mwards)
Fig. 4.41
●o.
4.20
.

PHYSICS-XII

w
' 10. A long wire is bent as 15. A metallic wire is
shown in Fig. 4.42. Find the bent into the shape
magnitude and direction of ^ I shown in Fig. 4.47
the magnetic field at the and carries a current I.
centre O of the circular I If O is the common

part, if a current I is passed centre of all the three

through the wire. circular arcs of radii r,


n 2r and 3r, find the
lAns. 1 — normally into I
2R[ 7T magnetic field at the V r

the plane of paper]


point O.
I
''A
4
II. Figure 4.43 shows two Ans. e.
/
semicircular loops of 24nr
Fig. 4.47
radii R, and R2 carrying normally inward
current /. Find the magni
tude and direction of the HINTS

magnetic field at the 1, As the coil is tightly wound, so radius of each turn,
common centre O. r = 10cm = 0.1 m

[Ans.
Mo / 1 1
HgM 4;t X 10'^ X 100x1
4 [R,'"r^)'
B =
2r ” 2x^
normally downward] = 2tcx10^ = 6.28x10“^T.

12. A circular segment of radius 10 cm subtends an


2. Radius, r =
PoWf _ 471x10“^ xlx5.0
angle of 60° at its centre. A current of 9 A is flowing 2B 2x0.20x10^3
through it. Find the magnitude and direction of the = 1.57xl0-2m =1.57 cm.
magnetic field produced at the centre (Fig. 4.44).
(Ans. 9.42x10"^ T) 3, B= B H or
2r
= BH

47rxl0"^x50x/
or = 1.86x10"3
2x2.54x10'^
1.86x2x2.54
/ = = 0.015 A.
47t X 50

10"^ x5
4. B= = 7.85 xlO"^ T.
4r 4x0.02

6. Magnetic field at the centre of a circular coil.


B =
2R

Fig. 4.44 Fig. 4.45 Mqx3


Bp = 2x5x10”^ T = 12n X10 ^T, along vertical
13. A current of I ampere is flowing through the bent
Mo x4
wire shown in Fig. 4.45. Find the magnitude and B. T =16nxlO“*T,along horizontal
^ 2x5x10"^
direction of the magnetic field at point O.
la Resultant field,
Ans. B = ^ directed normally downward
4k r
12^ + 1? X Tu X10“^ T=207rjiT.
14. In Fig. 4.46, the curved portion is a semi-circle and
the straight wires are long. Find the magnetic field Let the resultant field make angle 0 with the
vertical. Then,
at the point O.
I
Ans.
Mo ^
2d { K
1.^11 12kx10~^ _ 3
tan0 =
16rcxl0"^ ~ 4
_i 4
●O
0 = tan ' - .
3
rt 1
1 7. Here 2nr = — metre r-—~ 0.25 m
2 4

Fig. 4.46
MAGNETIC EFFECT OF CURRENT 421

Magnetic field due to straight wire, According to Biot-Savart law, magnetic field at the
47T X 10 ^ x5 centre O is
B = Mpf _ Hpf
ll
2nr' 2n X 0.01 r 27t x 0.01 x 0.25
R _ 1^0
:„u = lio. I Jir _ ^ 0 ^ ^

= 4x10“^ T 4k ' in' 3 4n 3 r

Magnetic field at the centre of the circular loop, _ 10"^ X 3.14x9 = 9.42xl0"* T.
B =
Pfl/ 4tcx10”^x5 = 1.256x10'^ T. 3x0.10
2r 2x0.25

S, (i)N:=35, / = 1.2A, = 4.0 cm = 0.04 m,


13. Any element if/ on the arc will be perpendicular to
r = 40 cm = 0.40 m the position vector r^, so the field due to one such
element at the centre Owill be
B
4k xl0‘^ x35 xl.2x(0.04)^
2[(0.04)^ + (0.40)^f^^
axial
2(n^ + r2p dB =
W/ sin K / 2 _ |i Q hil

w
4k in '
4 X 3.14 X 10’^ X 35 X 1.2 x 0.0016
2x0.1616x0.402 Magnetic field due to the entire arc at the centre O,
= 6.5xlO“^T.

Flo
B = dB = if/ = ./
inr^ inr^
(n) .Bcentre
_^N/ = 6.6x10"'* T.
But / = length of arc = ar

ee
2l7

9. (i) Magnetic field at the centre of the arc is B =

Fr
. ar = —
4k 4k r
B =
1^0 ^
4r
1-1- Magnetic field at point O due to any airrent element
is perpendicular to and points out of the plane of
for
ur
Here/ = 10 A, r = 7cm = 0.07 m,
paper.
^^0 =
= 4k X 10'^ TmA"*
_ 4kx10“^x10 Magnetic field at O due to the upper straight wire is
ks
B = = 4.5x10"® T 1 1
4x0.07 X ---‘1
Yo

I'^lnOni) 2nd
oo

The direction of the field is normally outside the


plane of paper, Similarly, field at O due to lower straight wire is
eB

(n) B = 4.5 xlO"^ T. The field B will point normally E. =


2nd
into the plane of paper,
in. Magnitude of the magnetic field at O due to the Field at O due to the semicircle of radius if / 2 is
r
ou

1 I
straight part of the wire is
u
ad

●“0

I 2 2 (if/2) 2d
B| = ^, normally out of the plane of paper
Y

2k R Resultant field at O,

Magnetic field at the centre O due to the current 6=^-t-^+^ = ■U^ .


nd
Re

2d n
loop of radius R is
15. Magnetic field at Odue to the straight parts of the
Fi

Mo ^
^2 = 2R
, normally into the plane of paper wire will be zero. Magnetic fields at O due to the
three circular arcs of radii r, 2r and 3r are
Resultant field at 0 is
acting normally inward
Mo 4- 1 4k r
B=B,-^ = 2R .n)
B,= Mo^ _e acting normally outward
normally into the plane of paper. 4k 2r'

n. B= ^ + B2 = Mpf ^ Mpf ^ MqJ T ^ 1 4k 3r


. —-, acting normally inward
^
4R,1 4R2 4 R 2y
Thus the total magnetic field at the centre Ois
K
12. Here 0 = 60° = rad
3 B= ^ - Bj + B,= 4k I r 2r 3r.
/ /
As 0(rad) =
K
/ =
K r
5MqT
3
or
3
0, acting normally inward.
r r 24Kr
4.22 PHYSICS-XII

4.8 AMPERE'S CIRCUITAL LAW AND ITS


as centre. The magnitude of the field B is same for all
APPLICATION TO INFINITELY LONG
points on the circle. To evaluate the line integral of the
STRAIGHT WIRE
magnetic field 6 along the circle, we consider a small
9. (a) State Ampere's circuital lazv and prove it for the
magnetic ifeld produced by a straight current-carrying current element fr? along the circle. At every point on

conductor.
the circle, both B and ^ are tangential to the circle so
Ampere's circuital law. Just as Gauss's law is an
that the angle between them is zero.
alternative form of Coulomb's law in electrostatics,
similarly we have Ampere's circuital law as an B .dl = Bdl cos 0°= Bdl
alternative form of Biot-Savart law in magnetostatics.
Ampere's circuital law gives a relationship between Hence the line integral of the magnetic field along
the circular path is
the line integral of a magnetic field B and the total —> /
current / which produces this field. B.dl = n Bdl=Bn dl = ^0 A
2nr
Ampere's circuital law states that the line integral of the
—>
_fV .2Kr
magnetic ifeld B around any closed path is equal to ji 0 2nr
(permeability constant) times the total current I threading or
passing through this closed path. Mathematically, n B.dl

n bJi=Pq 1 This proves Ampere's law. This law is valid for any
assembly of current and for any arbitrary closed loop.
In a simplified form, Ampere's circuital law states that if 9. (b) Calculate, using Ampere's circuital theorem,
field B is directed along the tangent to every point on the magnetic field due to an infinitely long wire carrying
a current I.
perimeter L of a closed curve and its magnitude is constant
along the curve, then Application of Ampere's law to a straight
conductor. Fig. 4.49 shows a circular loop of radius r
BL=^i^ I
around an infinitely long straight wire carrying current
where I is the net current enclosed by the closed curve. —►

/. As the field lines are circular, the field B at any point


The closed curve is called Amperean loop which is a
geometrical entity and not a real wire loop. of the circular loop is directed along the tangent to the
Proof for a straight current-carrying conductor.
Consider an infinitely long straight conductor carrying
a current /. From Biot-Savart law, the magnitude of the
magnetic field B due to the current-carrying conductor
at a point, distant r from it is given by
B=
2Tzr
I
1

Fig. 4.49

B
circle at that point. By symmetry, the magnitude of
Line of field B is same at every point of the circular loop.
force d7 Therefore,

1
I B.dt Bdlcos0°= Bn dl= B.2nr

From Ampere's circuital law,


Fig. 4.48 Ampere's circuital law.
B.2nr =

As shown in Fig. 4.48, the field B is directed along B=


the circumference of the circle of radius rwith the wire 2;cr
MAGNETIC EFFECT OF CURRENT 4.23

For Your Knowledge the solenoid, current enters the plane of paper at points
marked 0 and leaves the plane of paper at points
Ampere's circuital law is not independent of the Biot- marked ©. The magnetic field at points close to a single
Savart law. It can be derived from the Biot-Savart law. turn of the solenoid is in the form of concentric circles
Its relationship to tine Biot-Savart law is similar to the like that of a straight current carrying wire. The
relationship between Gauss's law and Coulomb's law. resultant field of the solenoid is the vector sum of the
> Both Ampere's circuital law and Biot-Savart law fields due to all the turns of the solenoid. Obviously
relate magnetic field to the electric current. the fields due to the neighbouring turns add up along
> Ampere's and Gauss's laws relate one physical the axis of the solenoid but they cancel out in the
quantity (magnetic or electric quantity) on the perpendicular direction. At outside points such as Q,
boundary or periphery to another physical quantity the fields of the points marked 0 tend to cancel out the
(current or charge), called source, in the interior. fields of the points marked ©. Tlius the field at interior
midpoint P is uniform and strong. The field at the

w
> Ampere's circuital law holds for steady currents
which do not change with time. exterior midpoint Q is weak and is along the axis of the
solenoid with no perpendicular component. Fig. 4.51
> Although both Ampere's law and Biot-Savart law are
shows the field pattern of a solenoid of finite length.
equivalent in physical content, yet the Ampere's law

Flo
is more useful under certain symmetrical situations.
The mathematics of finding the magnetic field of a

ee
solenoid and toroid becomes much simpler if we
apply Ampere's law.

Fr
4.9 MAGNETIC FIELD INSIDE A
S
for N
ur
STRAIGHT SOLENOID

10. Give a qualitative discussion of the magnetic ifeld


produced by a straight solenoid. Apply Ampere's
ks
circuital lazv to calculate magnetic field inside a straight
Yo

solenoid.
oo

Magnetic field of a straight solenoid : A quali


eB

tative discussion. A solenoid means an insulated copper Fig 4.51 Magnetic field of a finite solenoid.
wire wound closely in the form of a helix. The word solenoid
comes from a Greek word meaning channel and was The polarity of any end of the solenoid can be deter
r

first used by Ampere. By a long solenoid, we mean that mined by using clock rule or Ampere's right hand rule.
ou
ad

the length of the solenoid is very large as compared to Ampere's right hand rule. Grasp the solenoid with the
Y

its diameter.
right hand so that the fingers point along the direction of the
current the extended thumb will then indicate the face of the
nd

solenoid that has north polarity (Fig. 4.52).


Re

●Q
Fi

Direction of
\ current

P'
N-pole
m )
I

Right hand

Fig. 4.52 Ampere's rule for polarity of a solenoid.


Fig 4.50 Magnetic field due to a section of a long solenoid.
Calculation of magnetic field inside a long straight
Figure 4.50 shows an enlarged view of the magnetic solenoid. The magnetic field inside a closely wound
field due to a section of a solenoid. At various turns of long solenoid is uniform everywhere and zero outside
PHYSICS-Xll

.Q
It can be easily shown that the magnetic field at the
dr —
end of the solenoid is just one half of that at its middle.
A Thus
CH H
■> B 1 r
I end
a'— —'b

B
Figure 4.54 shows the variation of magnetic field on
the axis of a long straight solenoid with distance x from
*■
its centre.
V-l,Nx|»|«|K|x|x|xl;jxMx|.|x|x|x|xM.Uh[xMx|>i

Fig. 4.53 The magnetic field of a very long solenoid, B

it. Fig. 4.53 shows the sectional view of a long solenoid.


At various turns of the solenoid, current comes out of B
the plane of paper at points marked O and enters the 2

plane of paper at points marked ®. To determine the


magnetic field B at any inside point, consider a End of O End of
rectangular closed path abed as the Amperean loop. solenoid <- Distance ->■ solenoid
According to Ampere's circuital law,
Fig. 4.54 Variation of magnetic field along
I B.rft the axis of solenoid.

= p Q X Total current through the loop abed


4.10 MAGNETIC FIELD DUE TO A
Now =f B.^ TOROIDAL SOLENOID
c a

b.M
a
+ +
B.^ + j B.d! 11. Apply Ampere's circuital laiv to find the magnetic
b c d
field both inside and outside of a toroidal solenoid.
But
f B. d! = f‘" Bdl cos 90° =0 Magnetic field due to a toroidal solenoid. A solenoid
bent into the form of a closed ring is called a toroidal
It a solenoid. Alternatively, it is an anchor ring (torous)
B.d! = Bdl cos 90° =0 around which a large number of turns of a metallic wire
are wound, as shown in Fig. 4.55. We shall see that the
magnetic field B has a constant magnitude everywhere
B.d! =0
inside the toroid while it is zero in the open space
interior (point F) and exterior (point Q) to the toroid.
as B = 0 for points outside the solenoid.
B.S =f
a

B dl cos 0° = B dl = Bi
a a

where,
I = length of the side ab of the rectangular loop abed.
Let number of turns per unit length of the
solenoid = n

Then number of turns in length / of the solenoid


Fig. 4.55 A toroidal solenoid.
= nl

Thus the current / of the solenoid threads the loop Figure 4.56 shows a sectional view of the toroidal
abed, nl times.
solenoid. The direction of the magnetic field inside is
.●. Total current threading the loop abed = nil clockwise as per the right-hand thumb rule for circular
Hence
B/ = Po»// or
B = Pq7iI loops. Tliree circular Amperean loops are shown by
MAGNETIC EFFECT OF CURRENT

For Your Knowledge >


The magnetic field inside a toroidal solenoid is
4.25

v
/
independent of its radius and depends only on the
current and the number of turns per unit length. The
field inside the toroid has constant magnitude and
tangential direction at every point.
In ideal toroid, the coils are circular imd magnetic
field is zero external to the toroid. In a real toroid, the
turns form a helix and there is a small magnetic field
external to the toroid.

Toroids are expected to play a key role in the Tokamak


Fig. 4.56 A sectional view of the toroidal solenoid. which acts as a magnetic container for the fusion of

w
plasma in fusion (thermonuclear)power reactors.
dashed lines. By symmetry, the magnetic field should
be tangential to them and constant in magnitude for Examples based on
each of the loops.

Flo
Ampere's Circuital Law and Magnetic Field due to
1. For points in the open space interior to the (/) Straight Solenoid (//) Toroidal Solenoid
toroid. Let 6j be the magnitude of the magnetic field

ee
Formulae Used
along the Amperean loop 1 of radius rj.

Fr
Length of the loop 1, L, = 2 tc 1. Ampere's circuital law, h B dl =
As the loop encloses no current, so / = 0 When B is directed along tangent to every point
Applying Ampere's circuital law,
for
on closed curve L,
ur
^ ~ 0 ^ 2. Magnetic field due to straight solenoid,
or
Bj x2Tcrj =}igxO (i) At a point well inside the solenoid, B = p g nl
ks
or
B,=0 (ii) At either end of the solenoid, ^ I'd
Yo
oo

Thus the magnetic field at any point P in the open space


interior to the toroid is zero. Here n is the number of turns per unit length.
eB

2. For points inside the toroid. Let B be the 3. Magnetic field inside a toroidal solenoid, B = p g n/
magnitude of the magnetic field along the Amperean Magnetic field is zero outside the toroid.
loop 2 of radius r. Units Used
r
ou

Length of loop 2, 1^=2nr B is in tesla, current / in ampere and n in m .


ad

If N is the total number of turns in the toroid and /


Y

the current in the toroid, then total current enclosed by Example 27. A solenoid coil of 300 turnshn is carrying a
the loop 2 = Nl current of 5 A. The length of the solenoid is 0.5 m and has a
radius of 1 cm. Find the magnitude of the magnetic field
nd
Re

Applying Ampere's circuital law, inside the solenoid. (CBSE F 04]


Bx 2jrr= pg X Nl
Fi

Solution. Here n =300 tums/m, I -5 A


or B =
27tr B = pgHf = 4k X 10“^ X 300 X 5 = 1.9 x 10“^ T.
If r be the average radius of the toroid and n the Example 28. A solenoid of length 0.5 m has a radius ofl cm
and is made up of500 turns. It carries a current of 5 A. What
number of turns per unit length, then
is the magnitude of the magnetic field inside the solenoid ?
N -2nrn
[NCERT]

B^PgUf Solution. Number of turns per unit length,


3. For points in the open space exterior to the _ N _ 500 = 1000 turns / m
toroid. Each turn of the toroid passes twice through the / ”0.5m
area enclosed by the Amperean loop 3. But for each
Here / =0.5 m and r =0.01 m i.e., I » a. So we can
turn, the current coming out of the plane of paper is
cancelled by the current going into the plane of paper. use formula for magnetic field inside a long solenoid.
Thus, / = 0 and hence = 0. B = pgH/ = 4k X 10“^ X1000 X 5 = 6.28 x 10'^ T.
4.26 PHYSICS-XII

Example 29. A 0.5 m long solenoid has 500 turns and has Now / =6.0A, =4 Tix 10 ^ TmA n =2800 m -1
a flux density of 2.52 x 10" ^ T at its centre. Find the current
B = 4 7t X 10"^ X 2800 X 6 T = 2.1 X 10"^ T
in the solenoid. Given
^^0 = 4jtxl0"^ [ISCE 951

Solution. Number of turns per unit length, This value of B is for both on and off the axis, since
n = —
N 500
= 1000 turns/m
for an infinitely long solenoid, the internal field near
/ 0.5 m the centre is uniform over the entire cross-section,

As (fc) Magnetic field at the ends of the solenoid is


B = Po«f
B 2.52 X IQ-3 B = 1.05 X 10"^T.
1 = = 2.0 A 6riQ 2
4tcx 10“^ X 1000
(c) The outside field near the centre of a long
Example 30. A copper wire having a resistarice of 0.01 Q solenoid is negligible compared to the internal field.
per metre is used to wind a 400 turn solenoid of radius Example 32. A coil wrapped around a toroid has inner
1.0 cm and length 20 cm. Find the emf of a battery which radius of 20.0 cm and an outer radius of 25.0 cm. If the wire
when connected across the solenoid ivould produce a wrapping makes 800 turns and carries a current of 12.0 A,
magnetic field of 10~^ T near the centre of the solenoid. what are the maximum and minimum values of the magnetic
Solution. Length of wire used field within the toroid ?
= 2 7T r X No. of turns Solution. Let a and b denote the inner and outer
= 27t X 1.0 X 10“^ X 400 ni radii of the toroid. Then
-1 N 47t X 10"^ X 800x12.0
Resistance per unit length =0.0in m Bmax = ^on/ = p 0 / =
.'. Total resistance of wire,
2im 2 7CX 20.0x10"^
R = 2n:xl.0x 10“^ x 400 x 0.01 = 9.6xlO"^T = 9.6 mT.
= 8tcx lO'^n N 4tix 10"^ X 800x 12.0
^min =l^0«^=^^0 2 Kb
/ =
2;ix25.0x 10“^
No. of turns per unit length.
400
n = =2000 m"^ = 7.68 X 10"^T = 7.68 mT.
20x10"^
Example 33. (i) A straight thick long wire of uniform cross-
As
B = Po«/ = )io« — section of radius 'a' is carrying a steady current I Use
R
Ampere’s circuital law to obtain a relation showing the
BR 10“^ X 871x10"^ variation of the magnetic field (B^) inside and outside the
e= = 1 V.
4tix10“^x2000 wire with distance r, {r <a)and(r> a)of the field point from
the centre of its cross-section. Plot a graph showing the
Example 31. A solenoid 50 cm long has 4 layers of variation of field B with distance r.
windings of350 turtjs each. The radius of the lowest layer is
(n) Calculate the ratio of magnetic field at a point o/2
1.4 cm. If the current carried is 6.0 A, estimate the
above the surface of the wire to that at a point a / 2 below its
magnitude of B (a) near the centre of the solenoid on its axis surface. What is the maximum value of the field of this wire ?
and off its axis, (b) near its ends on its axis, (c) outside the [NCERT ; CBSE D 10 ; OD I6C]
solenoid near its centre.

Solution, (fl) The magnitude of the magnetic field


at or near the centre of the solenoid is given by
B=pp n I
where n is the number of turns per unit length. This
expression for B can also be used if the solenoid has
more tlian one layer of windings because the radius of
the wire does not enter this equation. Therefore,

n =
No. of turns per layer x No. of layers Fig. 4.57 A steady current I distributed uniformly
Length of the solenoid across a wire of radius a.

350x4
= 2800 m"^ Solution. (0 Application of Ampere's law to a
0.50 long straight cylindrical wire. By symmetry, the
MAGNETIC EFFECT OF CURRENT 4.27

magnetic lines of force will be circles, with their centres {ii) Suppose the point P lies at distance a 12 above
on the axis of the cylinder and in planes perpendicular the surface of the wire and point Q lies at distance a 12
to the axis of the cylinder. So we consider Amperean below the surface. [Fig. 4.58(b)]
loop as a circle of radius r. Magnetic field at point Pat distance r=3al2 from
Field at outside points. The Amperean loop is a the axis of the wire is

circle labelled 2 having radius r>a.


Length of the loop, L-2kt 2rcr 27^3/2) a 3tui
Net current enclosed by the loop = I Magnetic field at point Q at distance r = al2 from
the axis of the wire is
By Ampere's circuital law,
_Pofr_ PqJ rg
BL=^o/ 2m^ 27mH2 4tw
or 8x 2;ir=
Bp 4tm = 4:3.

w
or B=
2 nr
[For r>a] Bq 3m \IqI
Clearly, B is maximum on the surface of the wire
1
B«- i.e., at r-a.
[For outside points]

Flo
i.e..
r
Hence, B.max
Field at inside points. Tlie Amperean loop is a circle 2m

ee
labelled 1 with r<a.
Example 34. A wire of radius 0.5 an carries a current of

Fr
Length of the loop, L = 2nr 100 A, which is uniformly distributed over its cross-section.
Clearly, the current enclosed by loop 1 is less than I. Find the magnetic field (i) at 0.1 cm from the axis of the wire,
As the current distribution is uniform, the fraction of / (ii) at the surface of the wire and (Hi) at a point outside the
for
ur
enclosed is wire 0.2 cm from the surface of the zvire.
I Solution. Here R =0.5 cm =0.5 x 10““ m, / =100 A
r =
ks
m
We use the results of the above example.
Yo

Applying Ampere's law,


(0
oo

BL=Po /'
47tx10“^x 100x0.1x10'^
eB

Ir^
or
Bx2nr=[iQ — 2;cx (0.5x10'^)^
= 8.0x10'^ T.
r

B= [For r < fl]


ou

or
— ^ r
4jix10'^x100
ad

2m7 J
(“) ®surface = 27tR
_

271x0.5x10'^
Y

i.e., B<x r [For inside points]


= 4.0x 10'^ T.
Thus the field B is proportional to r as we move
nd

from the axis of the cylinder towards its surface and (Hi) Here r =0.5 + 0.2 =0.7 cm =0.7 x 10'^
Re

then it decreases as -. The variation of Bwith distance r


B _ 1^0^ _ 47t X 10'^ X 100
Fi

r outside
2Tcr~27rx 0.7x10'^
from the centre of the wire is shown in Fig. 4.58(a).
= 2.86x lO'^T.
B
roblems For Practice
p*
a/2 1. A long solenoid consists of 20 turns per cm. What
a/2 current is necessary to produce a magnetic field of
20 mT inside the solenoid ? (Ans. 8.0 A)
2. A long solenoid is made by closely winding a wire
of radius 0.5 mm over a cylindrical non-magnetic
(«) (b) frame so that successive turns nearly touch each
other. What will be the magnetic field at the centre
Fig. 4.58 (a) Sketch of the magnitude of the magnetic of the solenoid if a currentofS A flows through it?
field for the long conductor of radius a. (Ans. 2ti X 10"^ T)
(.1 4.28 PHYSICS-XII
V

3. Tlie magnetic field at the centre of a 50 cm long 5. Number of turns per \init length,
solenoid is 4.0 x 10"^ T when a current of 8.0 A 5 X 1000 -I
= 2500 m
flows through it. What is the number of turns in the II -
2.0
solenoid ? Take n =3.14. (Ans. 1990)
4. A solenoid is 1.0 m long and 3.0 cm in diameter. It
B = gpji/ = 471 X 10“^ x2500x5.0 = 1.57 xl0"^T.
6. Mean radius of toroid,
has five layers of windings of 850 turns each and 20+ 22
carries a current of 5.0 A. (f) What is B at its centre ? r- = 21cm = 0.21 m
2
(/'/) What is the magnetic flux for a cross-section
of the solenoid at the centre ? Number of turns per unit length
_ 4200 _ 4200 1000
[Ans. (/) 2.67 x 10“^ T, (z7) 1.9 x 10~^ Wb]
-1
m
2ti r 2ti X 0.21 71
5. A solenoid is 2.0 m long and 3.0 cm in diameter. It has
5 layers of winding of 1000 turns each and carries a (i) Field inside the core of the toroid,
1000
current of 5.0 A. What is the magnetic field at the = 4jr xlO"^ X X10= 0.04 T.
centre ? Use the standard value of g 0- 71

(Ans. 1.57xlO"^T) (ii) Magnetic field outside the toroid is zero.


6. A toroid has a core of inner radius 20 cm and outer (Hi) Magnetic field in the empty space surrounded
radius 22 cm around which 4200 turns of a wire are by toroid is zero.
wound. If the airrent in the wire is 10 A, what is the 7. Current enclosed by the loop of radius r.
magnetic field (i) inside the core of toroid (ii) outside /' =
i
X Ttr
the toroid and (Hi) in the empty space surrounded tzR^
by the toroid. [Ans. (i) 0.04 f (H) Zero (Hi) Zero] Using Ampere's circuital law,
7. A long straight solid conductor of radius 4 cm
carries a current of 2 A, which is uniformly distri Ir^
B =
buted over its circular cross-section. Find the B x27ir= Pq or
2kR^
magnetic field at a distance of 3 cm from the axis of 4tix10'^x2x3x10'-
the conductor. (Ans. 7.5 X 10" ^ T) = 7.5xlO"^T.
27ix(4x10“^)^
HINTS

1. Here ;i = 20cm“^ =20x10^ m'\ 4.11 FORCE ON A MOVING CHARGE


B = 20 mT = 20 X10" ^ T IN A MAGNETIC FIELD

B 20 X lO"^ 12. State the factors on which the force acting on a


Current, I = = 8.0 A.
g qIi 4ti X 10 ^ X 20 X 10^ charge moving in a magnetic field depends. Write the
expression for this force. When is this force minimum and
2. Diameter of the wire = 2 x 0.5 = 1.0 mm - 10“^ m
maximum ? Define magnetic field. Also define the SI unit
.●. Number of turns per unit length, of magnetic field.
1— = ioV -1 Magnetic force on a moving charge. The electric
10"^ m charges moving in a magnetic field experience a force, while
-1 there is no such force on static charges. This fact was
Also, /=5A,gQ=47ixl0 ' Tm A first recognized by Hendrik Antoon Lorentz, a great
B = gQ 7zl = 4tc X 10~^ xlO^ x5 = 27u xlO"^ T. Dutch physicist, nearly a century ago.

3. B =
Suppose a positive charge q moves with velocity v
/
in a magnetic field B and v makes an angle 0 with B,
Bl 4.0 X 10~- X 0.50
N = = 1990. as shown in Fig. 4.59. It is found from experiments that
g^J‘4x3.14x10“^ x8 the charge q moving in the magnetic field B expe
4. Number of turns per unit length, riences a force ? such that
5 x850 -1
= 4250 m
n =
1.0
1. the force is proportional to the magnitude of the
magnetic field, i.e., F cc B
(j) B = go J7/ = 47txl0'^ x4250x5.0 = 2.67xlO'^T. 2. the force is proportional to the charge q, i.e., F <x:q
(ii) (j)g = BA = B X nr^ 3. the force is proportional to the component of the velocity
= 2.67 X 10'^ X 3.14 X (1.5 X10"^)^ V in the perpendicular direction of the field B, i.e.,
= 1.9x10"=’ Wb. F oc u sin 0
MAGNETIC EFFECT OF CURRENT 4.29

1. Fleming's left hand rule. Stretch the thtmib and


the first hvo fingers of the left hand nnitmUi/ per
pendicular to each other. If the forefinger points in
the direction of the magnetic field, central finger in
the direction of current, then the thumb gives the
direction of the force on the charged particle.
{Fig. 4.60)

Fig. 4.59 Magnetic Lorentz force.


Combining the above factors, we get

w
F oc Bqv sin 0
or
F = kqvB sin 0 Current

The unit of magnetic field is so defined that the

Flo
proportionality constant k becomes unity in the above
equation. Thus
Fig. 4.60 Fleming's left hand rule.

ee
F = qvB sin 9
2. Right hand (palm) rule. Open the right hand and

Fr
This force deflects the charged particle sideways
and is called magnetic Lorentz force. As the direction place it so that tips of the fingers point in the
of F is perpendicular to both v and B, so we can direction of the field B and thumb in the direction of
for
ur
express F in terms of the vector product of v and B as
velocity v of the positive charge, then the palm faces
towards the force F, as shown in Fig. 4.61.
ks
F = q{v X B )
Yo

B
Figure 4.59 shows the relationship among the direc-
oo

tions of vectors F , v and B. Vectors v and 6 lie in the


eB

XY-plane. Tlie direction of F is perpendicular to this


plane and points along + Z-axis i.e., F acts in the
r
ou
ad

direction of x B.
Y

Special Cases
Case 1. ltv=0, then F=0
nd
Re

Thus a stationary charged particle does not experience


Fi

any force in a magnetic field.


Fig. 4.61 Right hand palm rule.
Case 2. If 0 =0° or 180°, then F =0

Thus a charged particle moving parallel or antiparallel to Definition of magnetic field. We know that
a magnetic ifeld does not experience any force in the B =
F
magnetic field. q V sin 0
Case 3. If 9 =90°, then F = q vB sm90° - qvB
Uq = l, v=l, 0=90°, sin 90° = 1, then B=F
Thus a charged particle experiences the maximum force Thus the magnetic field at a point may be defined as the force
when it moves perpendicular to the magnetic field. acting on a unit charge moving with a unit velocity at right
angles to the direction of the field.
Rules for finding the direction of force on a charged
particle moving perpendicular to a magnetic field. The SI unit of magnetic field. Again, we use
F
direction of magnetic Lorentz force F can be B=
determined by using either of the following two rules: qv sin
430 PHYSICS-XII

-1
Iff=lN, q=lC, i? = lms , 0=90° then
This force acts perpendicular to the plane of v and
IN —^
SI unit of B =
1 C.lms ^.sin90° B and depends on the velocity v of the charge.
IN The total force, or the Lorentz force, experienced by
1 A.l m Cs = A] the charge q due to both electric and magnetic field is
-1 -1
given by
= 1NA m -1 tesla.
e t
m

Thus tlie SI unit of magnetic field is tesla (T). ->

or
F =q(E +v B )
One tesla is that magnetic field in which a charge of
1 C moving with a velocity of 1 ms at right angles to
the field experiences a force of one newton. . For Your Knowledge
A static charge is a source of electric field only while a
A field of one tesla is a very strong magnetic field. moving charge is source of both electric and magnetic
Very often the magnetic fields are expressed in terms fields.
of a smaller unit, called the gauss (G).
A moving charge produces a magnetic field which, in
1 gauss =10 tesla turn, exerts a force on another moving charge.
Table 4.1 Some Typical Magnetic Fields A stationary source does not produce any magnetic
field to interact with an external magnetic field. Hence
8
Surface of a neutron star 10° T no force is exerted on stationary charge in a magnetic
field.
Large field in the laboratory IT
An electric charge always experiences a force in an electric
Field near a bar magnet 10“^ T
field, whether the charge is staHonaiy' or in motion.
Field on the earth's surface lO"** T A charge moving parallel or antiparallel to the direction
Field in interstellar space 10 -12 Y of the magnetic field does not experience any magnetic
Lorentz force.

If in a field, the force experienced by a moving charge


Dimensions of magnetic field. Clearly,
depends on the strength of the field and not on the
m MLT^^ velocity of the charge, then the field must be an electric
[B] = field.
[<7][i’][sin 0] AT.LT“^1
If in a field, the force experienced by a moving charge
depends not only on the strengthof the field but also on
Here A represents current. the velocity of the charge, then the field must be a
magnetic field.
4.12 LORENTZ FORCE
13. What is Lorentz force ? Write an expression for it. Examples Based on
Lorentz force. The total force experienced by a charged Force on Moving Charges in a
particle moving in a region xvhere both electric and magnetic Magnetic Field
fields are present, is called Lorentz force. Formulae Used

A charge in an electric field £ experiences the Force on a charge q moving with velocity in a
electric force. magnetic field at an angle 9 with it is
F = qvB sin 9
The direction of the force is given by Fleming's
left hand rule.
This force acts in the direction of field £ and is
Units Used
independent of the velocity of the charge.
The magnetic force experienced by the charge q Force F is in newton, charge q in coulomb, velocity
V in ms"^ and B in tesla.
moving with velocity v in the magnetic field B is
given by Example 35. A proton enters a magnetic field of flux
density 2.5 T with a velocity of 1.5 x lO^s”^ at an angle of
=q{v X B) 30° ivith the field. Find the force on the proton.
MAGNETIC EFFECT OF CURRENT 431

Solution. Here ^ = e = 1.6 x 10 As the electron moves northwards, direction of

i?=1.5xl0^ms“\ 6 = 2.5 T, 0=30° current is eastwards. According to Fleming's left hand


rule, the magnetic force on the electron acts vertically
Force, F = c\vB sin 0 upwards.
= 1.6 X 10“^^ X 1.5 X 10^ X 2.5 X sin 30° Example 38. A positive charge of 1.5 pC is moving with a
= 3x 10"’^N. speed o/ 2 X 10^ along the positive X-axis. A magnetic
—^ A A

Example 36. An alpha particle -1 is projected vei ■tically B={0.2j + 0.4 k) tesla acts in space. Find the
upward with a speed of 3 x 10^ kms in a region where a magnetic force acting on the charge.
magnetic field of magnitude 1.0 T exists in the direction Solution. Here^/=1.5pC = 1.5x 10"^C,
south to north. Find that magnetic force that acts on the
particle. V =2 .y 10^ i ms \ B=(0.2;' +0.4;c)T
Solution. Charge on a-particle, Magnetic force on the positive charge is

w
-19
= + 2e =2 X 1.6X 10 c
F =q{v X 6)
Here z; =3x lO'^ km s"^ =3x 10^ ms"\ S = 1.0T,
0 = 90°.
= 1.5xl0“‘'’[2xl0S' x(0.2/ +0.4Jt)]

Flo
= 3.0 [0.2 f x/ +0.4f xk 1

ee
B N A A A A A A A

90 = {0.6k -1.2/ )N. [v ixj =k,i xk =-/]

Fr
W E Example 39. A 5.0 MeV proton is falling vertically down
ward through a region of magnetic field 1.5 T acting horizon
for
tally frorn south to north. Find the magnitude and the direc
ur
S

tion of the magneticforce exerted on the proton. Take mass of


Fig. 4.62 the proton as 1.6 x 10 kg.
ks
Magnetic force on the a-particle is Solution. Kinetic energy of the proton is
Yo

1
^ = 5.0 MeV = 5 X 1.6 X 10'^^ J
oo

— mv
F = qvB sin 0 2
-19
= 2 X 1.6 X 10 x3xl0^xl.0xsin 90° -13
eB

-12 or V
2 2 X 5 X 1.6 X 10
= 9.6 X 10 N
m

According to Fleming's left hand rule, the magnetic 10x1.6x10


-13
14
r

= 10x10
force on the a-particle acts towards west. -27
ou

1.6x10
ad

Example 37. An electron is moving northzvards with a


V = 3.16x 10^ m
velocity ofS.O x lO^ms"^ in a uniform magnetic field of 10 T
Y

directed eastwards. Find the magnitude and the direction of Force on the proton is
the force on the electron. F = qv Bsin90°
nd
Re

-19
Solution. q= e = 1.6x10 C, v =3.0 X 10^ ms \ = 1.6x 10
-19
X 3.16x10^x1.5x1
Fi

B = 10 T,0=9O°. = 7.58 X 10“^^ N


According to Fleming's left hand rule, the magnetic
N
force on the proton acts eastwards.

/ W E
Example 40. A long straight
wire AB carries a current of 4 A.
B

/ proton P
A travels at
4 X 10^ m/s, parallel to the wire,
4 A

I S
0.2 m from it and in a direction 0.2 m
fP
opposite to the current as shown
Fig. 4.63 in Fig. 4.64. Calculate the force
which the magnetic ifeld of
The magnitude of magnetic force on the electron is 6
current exerts on the proton. Also A
4 X 10 m/s
F = qvB sin 0 = 1.6 x 10“^^ x 3 x 10^ x 10 x sin 90° specify the direction of the force.
= 4.8x 10"^^ N [CBSE OD 02,19] Fig. 4.64
432 PHYSICS-XII
"ii:

Solution. Magnetic field at point P due to the current of 3 A. Calculate tlie magnitude of the axial
current in wire AB, magnetic field inside the solenoid. If an electron
_ 47TX 10 ^ X 4 = 4xlO"^T were to move with a speed of 2 x 10^ ms“' along
B =
2m 271x0.2 the axis of this current carrying solenoid, what
would be the force experienced by this electron ?
This field acts on the proton normally into the plane [CBSED08CI (Ans.0.38T, 0)
of paper. According to Fleming's left hand rule, a
5. An electron is moving at 10^ ms"' in a direction
magnetic force acts on the proton towards right in the
parallel to a current of 5 A, flowing through an
plane of paper. The magnitude of this force is
infinitely long straight wire, separated by a perpen
f = qvB sin 90° dicular distance of 10 cm in air. Calculate the
-19
= 1.6x10 X 4 X 10^ X 4 X 10"^ X 1 magnitude of the force experienced by the electron.
= 2.56 X 10
-18
N. [CBSED99] (Ans.l.6xl0"'® N)
Example 41. Copper has 8.0x10^^ electrons per cubic 6. A proton of energy 3.4 MeV moves vertically down
metre. A copper zoire of length 1 m and cross-sectional area
wards tlirough a horizontal magnetic field of 3 T which
acts from south to north. What is the force on the
8.0 X 10"^ carrying a current and lying at right angle to
proton ? Mass of proton is 1.7 x 10 kg; chargee on
a magnetic field of strength 5 x 10“^ T experiences a force of proton is 1.6 x 10"'^ C. (Ans. 12.15 x 10" ^ N)
8.0 X 10"^ N. Calculate the drift velocity of free elcctrozis in
the zoire. HINTS

1. = c = 1.6 X10“'^ C, y = 5.0x10^ ms


-1
Solution, n =8 x lO^^m"^, / =1 m
B = 1.0 Wbm“^,e = 30°
A = 8xl0‘^m^, e = 1.6 X 10 -19 c
Force, F = qvB sin 0
Total charge contained in the wire, = 1.6 X 10"'^ X 5.0 X 10^ X 1.0 X sin 30°
q = Volume of wire x ne = Alne = 4.0x10"'^ N.
= 8 X lO"*^ X 1 X 8 X 10^® X 1.6 X 10"'^ C
2. (i) Here m = 6.65x10^^ kg,
= 102.4 X 10"'' C
(? = + 2e = 2 X 1.6 X 10“'^C B = 0.2 T,
If is the drift speed of electrons, then r? = 6 X 10^ ms"', 0 = 90°
F - qv^ 6 sin 90° = qv^ B F = qvB sin 90°
F 8.0 X 10"^ -1 = 2 X 1.6 X 10"’^ X 6 X 10^ X 0.2 X 1N
.'. V ms

~ gS"l02.4xl0^x 5x10"^ = 3.84x10"''* N


-14
= 1.56xl0"‘‘ms"'. F 3.84 X 10
= 5.77x10'^ ms"
a = —
-27
m 6.65 X 10
^ Problems For Practice -19
3. F = (?yBsin90° = 1.6xl0 xl0^x3xl
1. An electron moving with a velocity of 5.0 x lO^ms
-1 -12
= 4.8x10 N
enters a magnetic field of l.'O Wb m"^ at an angle of According to Fleming's left hand rule, the force acts
30°. Calculate the force on the electron.
vertically upwards.
(Ans. 4.0xl0"'^N)
[4 qNI _ 4;r X10“^ x 1500 x 3
2. An a-particle of mass 6.65x10"^'^ kg and charge 4. B =
I 1.5
T = 0.038 T

twice that of an electron but of positive sign travels


Force, F = ez>B sin 0° = 0.
at right angles to a magnetic field with a speed of
6 x lO^ms"'. The strength of the magnetic field is 5. Magnetic field of the straight wire carrying a current
0.2 T. (i) Calculate the force on the a-particle. of 2 A, at a distance of 10 cm or 0.1 m from it is
(ii) Also calculate its acceleration. Fq I 4jix10"^x_5^^0-5t
[Ans. (0 3.84 x lO"'’* N (/i) 5.77 x 10'“ ms"^] 27rr 2;i X O.l

3. An electron is moving northwards with a velocity This field acts perpendicular to the direction of the
of 10^ ms"' in a magnetic field of 3 T, directed electron. So magnetic force on the electron is
eastwards. Calculate the instantaneous force on the
F = qv Bsin 90°
electron. (Ans. 4.8 x 10"'^ N, vertically upwards) = 1.6x10
-19
xlO^ xlO"^ X 1=1.6x10"'® N.
4. A solenoid, of length 1.5 m, has a radius of 1.5 cm
and has a total of 1500 turns wound on it. It carries a 6. Proceed as in Example 39 on page 4.31.
MAGNETIC EFFECT OF CURRENT 433

4.13 WORK DONE BY A AAAGNETIC FORCE Following three cases are possible :
ON A CHARGED PARTICLE IS ZERO 1. When the initial velocity is parallel to the
14. Show that the work done by a magnetic ifeld on a magnetic field. Here 0 =0“ so F = i;yBsin0°=0.
moving charged particle is always zero. Thus the parallel magnetic field does not exert any
Work done by a magnetic force on a charged force on the moving charged particle. The charged
particle. Tlie magnetic force F =q{v x B) always acts particle will continue to move along the line of force.
2. When the initial velocity is perpendicular to the
perpendicular to the velocity v or the direction of
magnetic field. Here 0 = 90°, so F = qvB sin 90° = qvB =
motion of charge q. Therefore,
a maximum force. As the magnetic force acts on a
F .V = q{v X B). V =0 particle perpendicular to its velocity, it does not do any
work on the particle. It does not change the kinetic
According to Newton's second law.
energy or speed of the particle.

w
d V
F =ma = m Figure 4.65 shows a magnetic field B directed
dt
—»
normally into the plane of paper, as shown by small
d V crosses. A charge + q is projected with a speed v in the

Flo
m . V =0
dt plane of the paper. The velocity is perpendicular to the
X

ee
-*

m d V -* d V
or . V + V . = 0 X

Fr
2 dt dt X

or
z {v . z; ) = 0 X
2 dt
for
ur
XB
d fl
or — mv = 0 X
dt\.2
[v .V = v^] X
ks
dK
Yo
X
or = 0
oo

dt X

or K = constant
eB

Fig. 4.65 A positively charged particle moving in a


Thus a magnetic force does not change the kinetic energy magnetic field directed into the plane of paper.
of the charged particle. This indicates that the speed of the
particle does not change. According to the work-energy
r

magnetic field. A force F = qvB acts on the particle


theorem, the change in kinetic energy is equal to the work
ou
ad

done on the particle by the net force. Hence the work perpendicular to both v and 6. This force conti
Y

done on the charged particle by the magnetic force is zero. nuously deflects the particle sideways without changing
its speed and the particle will move along a circle
4.14 MOTION OF A CHARGED PARTICLE perpendicular to the field. Thus the magnetic force
nd
Re

IN A UNIFORM MAGNETIC FIELD provides the centripetal force. Let r be the radius of the
circular path. Now
Fi

15. Discuss the motion of a charged particle in a 2


uniform magnetic field with initial velocity (i) parallel to Centripetal force.
mv
= Magnetic force, qvB
the field, (ii) perpendicular to the magnetic field and (Hi) r

at an arbitrary angle with the field direction. or r =


mv

Motion of a charged particle in a uniform magnetic qB


field. When a charged particle having charge q and
Tlius the radius of the circular orbit is inversely
velocity v enters a magnetic field B, it experiences a proportional to the specific charge (charge to mass
force ratio q! m) and to the ma^etic field.
f = q(v X B) Period of revolution =
Circumference

The direction of this force is perpendicular to both Speed


V and B. The magnitude of this force is or
_2nr _2k mv 27tm

V V qB qB
F = qv 6 sin 0
434 PHYSICS-XII

Clearly, the time period is independent of v and r. If The period of revolution is


the particle moves faster, the radius is larger, it has to 2nr 2n rfwsind_2nm
T =
move along a larger circle so that the time taken is the
same.
V
1 y sin 0 qB qB

Tlie frequency of revolution is Thus a charged particle moving in a uniform


magnetic field has two concurrent motions : a linear
f =
T 2-nm motion in the direction of B (along X-axis) and a

This frequency is called cyclotron frequency. circular motion in a plane perpendicular to B (in
YZ-plane). Hence the resultant path of the charged
3. When the initial velocity makes an arbitrary
particle will be a helix, with its axis along the direction
angle with the field direction. A uniform magnetic
of B.
field B is set up along +ve X-axis. A particle of charge q
The linear distance travelled by the charged particle
and mass enters the field 6 with velocity v inclined in the direction of the magnetic field during its period
->

at angle 0 with the direction of the field B, as shown in of revolution is called pitch of the helical path.
2 nm 2 nmv cos 0
Fig. 4.66.
pitch = U|| X T = i? cos 0 X qB qB
Y

4.15 MOTION OF A CHARGE IN


V PERPENDICULAR MAGNETIC AND
1, V
s h h h h ELECTRIC FIELDS

<7©
B
16. Electric and magnetic fields are applied mutually
T
O
V, X perpendicular to each other. Show that a charged particle
will follow a straight line path perpendicular to both of
these ifelds, if its velocity is £/ Bin magnitude.
z
Velocity selector. Suppose a beam of charged
Pitch
particles, say electrons, possessing a range of speeds
Fig. 4.66 Helical motion of charged particle in a magnetic field. passes through a slit and then enters a region in
which crossed (perpendicular) electric and magnetic
The velocity v can be resolved into two rectangular fields exist. As shown in Fig. 4.67, the electric field ^
acts in the downward direction and deflects the
components :
electrons in the upward direction. The magnetic field
1. The component ZJy along the direction of the field
B acts normally into the plane of paper and deflects
i.e., along X-axis. Clearly
the electrons in the downward direction.
t?ll = i? cos 0 s, S,
The parallel component remains unaffected by the +
Deflection
magnetic field and so the charged particle continues to B
V' X X
move along the field with a speed of v cos 0.
2. The component perpendicular to the direction
/^X |x X lx x\ ,
Electron X X[X|x|^X,X'r Undeflected electron
of the field i.e., in the YZ-plane. Clearly ‘ X X |X |X I X'X_'
' —^ I I I ^ -
= u sin 0 s EyXyX^X y X
Due to this component of velocity, the charged
particle experiences a force f = qVj_ B which acts " \ by S

Region of
perpendicular to both and B. This force makes the crossed fields

particle move along a circular path in the YZ-plane. Fig. 4.67 Motion of an electron in a region of
The radius of the circular path is ^ crossed magnetic and electric fields.
mv mv sin 0
r = i_ _ Only those electrons will pass undeflected through
qB qB the slit $2 on which the electric and magnetic forces are
MAGNETIC EFFECT OF CURRENT 4.35

equal and opposite. The velocity v of the undeflected Solution. The electron moves along semicircular
electrons is given by trajectory inside the magnetic field and comes out, as
eE = evB or v = —
£ shovvTi in Fig. 4.68(F). Radius r of the path is given by
B 2
mv

Such an arrangement can be used to select charged = qvB


r
particles of a particular velocity out of a beam in which
9.1x10 x4xl0'*
-31
the particles are moving with different speeds. This
mv
or m

arrangement is called velocity selector or velocity qB 1.6 X10“^^ X10"^


filter. This method was used by /./. Thomson to
9.1x4
determine the charge to mass ratio (c/ »j)of an electron. xlO'^m =22.75 xl0"^m
1.6
Examples based on
Motion of Charges in Electric X X X

and Magnetic Fields

w
e X Xb X
Formulae Used

1. Electric force on a charge, = qE


e ^ X X

X X X

Flo
2. Magnetic force on a charge, F^j=qv Bsin 0
3. In a perpendicular magnetic field, the charge Fig. 4.68(b)

ee
follows a circular path.
mz^ Time taken to come out of the region of magnetic

Fr
mv
qv B = or r =
r qB field.
2rt m qB nr 22x22.75x10"^
T = and / =
for
ur
qB 2nm V 7x4x10^
4. When v makes angle ©with B, the charge follows = 17.875 X 10"^s -1.8 X lO'^s.
ks
helical path.
Example 43. An electron travels in a circular path of
Yo

mv mv sin 6 2nm
radius 20 cm in a magnetic ifeld 2 x 10"^ T. (i) Calculate
± -
oo

r = ● T= —
qB qB V qB
1
the speed of the electron, (ii) What is the potential difference
eB

Pitch of helix, h = v,T = v cos 0. T =


2nmv cos 0
through zvhich the electron inust be accelerated to acquire
qB this speed ?
5. K.E. gained by an electron when accelerated Solution. Here r = 20 cm =20 x 10"^m,
r

through a potential difference V,


ou

C m = 9.1x 10"^^ kg
-19
B = 2xlO'^T, e = 1.6xl0
ad

2eV
- mv^ = cP (/) Magnetic force on the electron
Y

V -
2 m
= Centripetal force on electron
Units Used 0
nd

mv~
Re

evB =
EisinVm'^ orNC“^Bin tesla, u in ms'^,r in metre. r

eBr
Fi

.-. Speed, V =
Example 42. An electron moving horizontally zuith a m

velocity o/4x 10^??j/s enters a region of uniform magnetic 1.6xl0"^‘^x2xl0"^x20xl0"^


field of 10"^ T acting vertically dowmvard as shozvn in 9.1x10
-31

Fig. 4.68(a). Draiv its trajectory and find out the time it takes
to come out of the region of magnetic field. [CBSE F 15] = 7.0 X 10^ ms"^
(ii) If V is the p.d. required to give speed v to the
X X X
electron, then
X Xb X cV = -i mv^
- e
X X X
x(7DxlO^)^
-31
mii^ 9.1 X 10
or V^ =
X X X - 19
2e 2x1.6x10

Fig. 4.68 (a) = 13.9 x 10^ V -14 kV.


PHYSICS-XII

Example 44. An electron after being accelerated through a (ii) Current carried by proton beam,
potential difference of 10^ V enters n uniform magnetic field / = 0.8mA=8xl0“‘* A
of 0.04 T perpendicular to its direction of motion. Calculate
Number of protons striking the screen per second,
the radius of curvature of its trajectory. (CBSE F 1?]
-4
Solution. Here V =10'* V, B = 0.04 T, _ / _ 8x 10 = 5x 10*^s'*
-19 -31 ~e“l.6xl0 -19
c = 1.6xl0 C m = 9.1 X 10 kg
An electron accelerated through a p.d. V acquires a
Wp= 1.675x10'^^ kg
Force with which a proton beam strikes a target on
velocity v given by
the screen.
1 '2eV dp
- mv 2 ~ ,,
eV or v = F = — = nv
2 m dt P

As the electron describes a circular path of radius of = 1.675 X 10“^^ X 5 X 10*^ X 2 X 10^ N
r in the perpendicular magnetic field B, therefore. = 1.675 X 10"® N.
2
mv
= ev B Example 46. An electron beam passes through a mag
r
netic field of 2 X 10"^ Wb nf^ and an electric field of
or r -
mv m
^2eV _ f2meV 3.4 X 10 ^Vnf^, both acting simultaneously. If the path of
eB eB m eB the electron remains undeviated, calculate the speed of
the electrons. If the electric field is removed, what will be
^2 X 9.1 X 10" X 1.6 X 10" X 10^ the radius of the circular path ? Mass of an electron
1.6x10
-19
x0.04
=9.1 X 10"®* kg.
Solution. Here 6=2x10"® Wbm"^
-23
5.4 X 10 ●1
£ = 3.4x10^ Vm
-19
1.6x10 x0.04
Magnetic force on the electron
= 8.43 X 10 ®m = 8.43 mm. = Electric force on the electron

evB= eE
Example 45. If a particle of charge q is moving with
or

velocity v along the z-axis and the magnetic field Bis acting .'. Velocity of electrons,
along the y-axis, use the expression F =q{v x B) to find £ 3.4x10^ -1
V =— =■ ^ ms = 1.7x10^ ms"*
the direction of the force F acting on it. 6 2x10"®
A beam of proton passes undeflected with a horizontal When electric field has been removed.
velocity v, through a region of electric and magnetic fields, Force exerted by the magnetic field on an electron
mutually pcrpendicidar to each other and normal to the
= Centripetal force on an electron
direction of the beam. If the magnitudes of the electric and 2
magnetic fields are 100 kV/m and 50 mT respectively, i.e., evB =
mv

calculate : (/) velocity v of the beam, (ii) force with which it R

strikes a target on a screen, if the proton beam current is mv 9.1 X 10"®* X 1.7x10^
R =
equal to 0.80 mA. {CBSE OD 08]
or
eB 1.6xl0"*^x2xl0"®
Solution. P =q{vx B) = q{vj x Bk ) = 4.8 X 10"^ m = 4.8 cm.
A A A

= qvBj xk =qvBi Example 47. In a chamber a unifon7i magnetic field of 8.0 G


Thus the force F acts on the charge q along the +ve (1 0=10""* T) is maintained. An electron with a speed of
4.0 X 10 ^ ms ”* enters the chamber in a direction normal to
AT-direction.
the ifeld.
(?) For undeflected proton beam,
(/) Describe the path of the electron,
qvB - qE (ii) What is the frequency of revolution of the electron ?
-1
£ 100 kVm 100x10® Vm"* (Hi) What happens to the path of the electron if it
v = —
6 50 mT 50 X 10'®T progressively loses its energy due to collisions with
the atoms or molecules of the environment ?
= 2x10® ms"*. [NCERT]
MAGNETIC EFFECT OF CURRENT 437

Solution. (/) The path of the electron is a circle of Thus the electron moves in a circle of radius
radius r given by 11.32 m, as shown in Fig. 4.69. As it covers a distance
r =
mv
PQ= 30 cm, it goes down through a vertical distance
eB

Here B = 8.0 G =8.0 x 10”^T, i; = 4.0 x 10^ ms“\ pi 30


-19 -31
e = 1.6xl0 C, m = 9.1xl0 kg
-31
9.1x10 X 4.0x10^
r =
-19 -4
1.6x10 X 8.0 X 10

= 2.8x10'^ m=2.8 cm.


The sense of rotation of the electron in its orbit can

be ascertained from the direction of the centripetal

w
Fig. 4.69
force f =-e{v x^). Thus if we look along the
direction of B, the electron revolves clockwise. equal to PA. If 0 is the angle subtended by arc PQ at the
centre O, then

Flo
(b) The frequency of revolution of the electron in its PA = OP - OA = r - r cos 0 = r (1 - cos 0)
circular orbit is

ee
eB 1.6x10
-19
X 8.0 X 10
-4 Arc 30x 10”^
Now 0 = = 0.02650 rad
/=

Fr
27rm 27tx9.1x 10
-31 Radius 11.32

8 0.02650 X 180
= 0.22x10° Hz = 22 MHz. = 1.52°
7t

for
ur
(c) In successive collisions, electron loses its speed cos 0 = cos 1.52 ° = 0.99965
progressively. If after collision its velocity vector
Hence PA = 11.32 (1 - 0.99965)
remains in the same plane of the initial circular orbit,
ks
the radius of the circular orbit will decrease in = 3.9744x 10“^ m=^4 mm.
Yo

proportion to the decreasing speed. Otherwise, the patli Example 49. A beam of protons enters a uniform magnetic
oo

of the electron will be helical between two collisions.


field of 0.3 T with a velocity of 4 x 10^ ms~^ at an angle of
eB

Example 48. A monoenergefic electron beam of initial 60° to the field. Find the radius of the helical path taken
energy 18 keV moving horizontally is subjected to a by the beam. Also find the pitch of the helix (distance
horizontal magnetic field of 0.4 G normal to its initial travelled by a proton parallel to the magnetic field during
one period of rotation). Mass of proton is 1.67 x 10"^^ kg.
r

direction. Calculate the vertical deflection of the beam over a


ou
ad

distance of 30 cm. [CBSE 5P 981 [JEE Main Sept 20]


Solution. Under the action of the magnetic field, Solution. The components of the proton's velocity
Y

the electrons will move along a circular path. parallel and perpendicular to the magnetic field are
Centripetal force on an electron
nd

= V cos60° = 4x 10^ X - = 2 X 10^ ms“^


Re

2
= Magnetic force on an electron
Fi

2 Jo
mv
= evB V.1 = u sin 60° = 4 X 10^ X = 3.464x10^ ms"^
r
2

or
mv
■^2?».l/2 The component U|| makes the electron move along
the field B while v^ makes the proton move along a
r =
eB eB
circular path. Hence the path of the proton is a helix.
Here m = 9.11 x 10“^^ kg, e = 1.6x 10'^^ C, The radius r of the helix is given by
B=0.40G=0.40xl0“^T
_ mtC
1

K.E. = - mv^ = 18 keV = 18 x 1.6 x 10"’^ J qv^B=^ r


2
mv
1 _
1.67 X 10"^^ X 3.464 X 10^
-31 -16 or r =
x9.11xl0 X 18 X 1.6 X 10 -19
r =
-19 -4
qB 1.6 X 10 xO.3
1.6x10 X 0.4 X 10
= 12xl0“^m = 1.2 cm.
= 11.32 m
s 4.38

Period of revolution of the electron is

T =
27tr
V
1
2 X 3.14 X 12x10"^
3.464x10^
PHYSICS-Xil

5. An electron beam passes through a magnetic field


of 2xlO"^Wbm“^ and an electric field of
1.0 X 10^ V m"^ both acting simultaneously. If the
path of the electrons remains undeviated, calculate
= 21.75 X 10'° s
8
the speed of the electrons. If the electric field is
removed, what will be the radius of the circular
Pitch of the helix is
path ? (Ans. 5 x 10^ ms"^ 1.43 cm)
p=U|| X 1=2x10^x21.75x10"® 6. An electron moving perpendicular to a uniform
= 43.5 X 10~®m = 4.35 cm. magnetic field completes a circular orbit in 10~^ s.
Calculate the value of the magnetic field. Mass of
Example 50. A proton projected in a magnetic ifeld of
electron = 9 x 10"^^ kg. (Ans. 3.5 X10"^ T)
0.02 T travels along a helical path of radius 5.0 cm and pitch
20 cm. Find the components of the velocity of the proton 7. Find the flux density of the magnetic field to cause
along and perpendicular to the magnetic field. Take the mass 62.5 eV electron to move in a circular path of radius
of the proton = 1.6 x 10"^^ kg. 5 cm. Given = 9.1 x 10“^^ kg and e = 1.6 x 10“^^ C.
mv
± (Ans. 5.335x10“^ T)
Solution. Radius of helical path, r =
qB 8. An electron of energy 2000 eV describes a circular
path in a magnetic field of 0.2 T. What is the radius
rqB_5xlO”^x 1.6x10
-19
x0.02
of path ? Take =9x10"^' kg, e = 1.6x10
-19
.●. V c.
A. -27
m 1.6 X 10
(Ans. 0.75 mm)
= 1.0 X 10® ms"’ 9. What should be the minimum magnitude and
Period of revolution.
direction of the magnetic field that must be produced
at tire equator of earth so that a proton may go
27ir 27TX 5 X 10 ^ round the earth with a speed of l.OxlO^ms -1 ,
T = = nx 10 ® S
V 1.0 X 10® Earth's radius is 6.4 x 10^ m.
1

(Ans. 1.63 X 10~® T, perpendicular to the


V,
Pitch _20x 10"^ = 6.37 X 10'* ms"’. equator in a horizontal direction)
T 71X 10~® 10. A stream of charged particles possessing a range of
speeds enters region I after passing through a slit S[
roblems For Practice (Fig. 4.70). In region I there exist crossed (perpen-

1. An electron entering a magnetic field of 10”^ T with


a velocity of 10^ ms”’ describes a circle of radius +

6 X10” ® m. Calculate e/m of the electron.


(Ans. 1.67x10” Ckg-’) S, S2
2. An electron after being accelerated through a poten *■ -1

tial difference of 100 V enters a uniform magnetic E


B
1000 ms

field of 0.004 T perpendicular to its direction of T T


motion. Calculate the radius of the path described
I II
by the electron. [CBSE OD 02]

(Ans. 8.4 mm)


3. A particle having a charge of 100 gC and a mass of Fig. 4.70
10 mg is projected in a uniform magnetic field of
25 mT with a speed of 10 ms”’ in a direction perpen dicular) electric and magnetic fields. The electric
dicular to the field. What will be the period of revolu field has magnitude 100Vm”’. We want the
tion of the particle in the magnetic field ? particles emerging from slit ^ into region II to have
(Ans. 25 s) a fixed velocity of 1000 ms”’. What should be the
4. An electron having a kinetic energy of 100 eV value of the uniform magnetic field in region I ?
circulates in a path of radius 10 cm in a magnetic (Ans. 0.1 T)
field. Find the magnetic field and the number of 11. A proton, a deutron and an alpha particle, after
revolutions made by the electron per second. being accelerated through the same potential
(Ans. 3.4 X 10~* T, 9.4 x 10^ rps) difference, enter a region of uniform magnetic
MAGNETIC EFFECT OF CURRENT

-J2 X 9 X lO'^
-19
X 1.6x10 X2000
field B, in a direction perpendicular to B . Compare -19
1.6x10 xO.2
their kinetic energies. If the radius of proton's
circular path is 5 cm, what will be the radii of the = 7.5 X lO”"* m - 0.75 mm.
paths of deutron and alpha particle ? inv 1.67x10
-27
xlO'
(Ans. 1 : 1 ; 2, = 7.07cm, = 10 cm) 9. B = = 1.63x10”“ T.
qr 1.6x10
-19
X 6.4x10^
12. A particle having a charge of 5.0 pC and a mass of
5.0x10”^^ kg is projected with a velocity of 10. For the particles to go undeflected,
1.0 km s"’ in a magnetic field of magnitude 5.0 mT. Force due to electric field = Force due to magnetic
The angle between the magnetic field and the field

velocity is sin”’ (0.90). Show that the path of the qE = qvB


particle will be a helix. Find the diameter of the £ 100 Vm
-1

helix and its pitch. (Ans. 36 cm, 55 cm) or B=- = 0.1 T.


V 1000 ms“’
HINTS

w
1. Use- =
V
11. For a given p.d., the kinetic energy of a charged
m rB particle is proportional to its charge.
2. Proceed as in Example 45 on page 4.36.

Flo
e:e:2e = l:l\2.
-6
2n tti 2ti X 10 X 10
?. T = = 25 s. Radius of the circular path of any particle of kinetic
100x10“^ X 25x10”^

ee
qB energy K
^2 meV 2K 4^

Fr
2Tzm mv m
4. Use B = and T =
er eB qB qB ] m qB
As eE = evB
for
ur
= 5 cm
.●. For proton, =
V =
E _ 1.0 X 10^ = 5x10^ ms'’ eB
B ** 2 X 10”^
^2x2»l/x'^
ks
When electric field is removed, electrons follow For deutron, =
eB eB
Yo

circular path.
oo

mv
2
- evB
= 42r^ - 1.414 X 5 cm = 7.07 cm.
^ ^2x4x„,^x2K,,
eB

r
-31
mv 9.1x10 xSxlU*’ For a-particle. a
or r = 2cB 2eB
eB 1.6x10'’^ x2xl0”^
= 10 cm
r

= 1.43xl0”^m =1.43 cm.


ou

12. Here (? = 5.0 pC = 5 X 10"^ C, m = 5xlQ kg,


ad

2ntn 2nm
6. AsT = B = V - 1.0 km s = 10'^ ms”’, B = 5.0mT = 5xl0”^T
-1
Y

eB eT
As 0 = sin”’(O.9O), so sin 0 = 0.90
7. Here — mv^ = 62.5eV = 62.5 x 1.6 x 10”’^ J
2
G = ^1-sin^ 0 = 71 - 0.M = ^ 0.436
nd

cos
Re

= 10”’^ J V = n sin 0= 10^ X 0.90= 0.9 X10^ ms“‘


1
Fi

-17
= i; cos 0 = 10"'^ X 0.436 = 4.36 x 10^ ms
-1
2x10
V-
m
Velocity component yy moves the electron along the
2x10
-17
field and along circular path. Hence the motion
-31
= 4.69 X10^ ms’’ is helical.
\ 9.1x10 2m V 2x5x10
-12
X 0.9x10^
1
mv 9.1x10' X4.69X10*’
●31 Diameter = 2r =
5 X 10'^ X 5 X 10”^
B =
-19
qB
er 1.6x10 X 5 X10’^
= 0.36 m = 36 cm
= 5.335 xlO”'’ T.
27tr 3.14x0.36
2eV T = = 1.25xl0”^s
S. 1 = eV or v = V
1
0.9x10^
2 m

Pitch = V, xT = 4.36xlO^ X 1.25x10’^


r =
nw m _ ^2meV
eB eB V m eB = 0.55 m = 55 cm.
4A0 PHYSICS-XII

4.16 CYCLOTRON Electromagnet


17. Wltat is a cyclotron ? Discuss the principle, con N

struction, theory and working of a cyclotron. What is the HF Oscillator.


maximum kinetic energy acquired by the accelerated Spiral path
charged particles ? Give the limitations and uses of a of proton
cyclotron.
Cyclotron. It is a device used to accelerate charged D1 ^2
particles like protons, deutrons, a-particles, etc., to very high
-Accelerated
energies. It was invented by E.O. Lawrence and M.S.
proton beam
Livingston in 1934 at Berkeley, California University.
Target
Principle. A charged particle can be accelerated to very "B

high energies by making it pass through a moderate electric S

field a number of times. This can be done with the help of a


perpendicular magnetic field which throws the charged particle
into a circular motion, the frequency of which does not depend
(«)
on the speed of the particle and the radius of the circular orbit.
Construction. As shown in Fig. 4.71, a cyclotron
consists of the following main parts : Magnetic field into
1. It consists of two small, hollow, metallic I HF Oscillator the plane of paper

half-cylinders and D2, called dees as they are X XX X


in the shape of D. X X > X\X X
2. They are mounted inside a vacuum chamber X X X

between the poles of a powerful electromagnet.


D * X X\X D2
X X X
3. The dees are connected to the source of high X 'X
Deflection
frequency alternating voltage of few hundred
kilovolts. plate
X
X XXX X Proton
4. The beam of charged particles to be accelerated X X X
source

is injected into the dees near their centre, in a X X


plane perpendicular to the magnetic field. X X X
«—Vacuum
5. The charged particles are pulled out of the dees ^ a X a
Accelerated
^ ^
chamber
by a deflecting plate (which is negatively charged) proton beam
through a window W. Target

6. The whole device is in high vacuum (pressure


~ 10"^ mm of Hg) so that the air molecules may (b)

not collide with the charged particles.


Fig. 4.71 Cyclotron (a) Front view {b) Section diagram.
Theory, bet a particle of charge q and mass m enter
a region of magnetic field B with a velocity v, normal Hence frequency of revolution of the particle will be
to the field B. The particle follows a circular path, the 1 _
fc = T 2 nm
necessary centripetal force being provided by the
magnetic field. Tlierefore,
Clearly, this frequency is independent of both the
Magnetic force on charge q velocity of the particle and the radius of the orbit and is
= Centripetal force on charge q called cyclotron frequency or magnetic resonance
mv
2
mv frequency. This is the key fact which is made use of in
or
qv B sin 90°= or r =
the operation of a cyclotron.
r
qB
Working. Suppose a positive ion, say a proton,
Period of revolution of the charged particle is
given by enters the gap between the two dees and finds dee Dj
to be negative. It gets accelerated towards dee Dj. As it
P_27ir_2n mv 2k m
enters the dee D,, it does not experience any electric
v V
qB qB field due to shielding effect of the metallic dee. The
MAGNETIC EFFECT OF CURRENT 4Al

perpendicular magnetic field throws it into a circular Uses of cyclotron :


path. At the instant the proton comes out of dee Dy it 1. The high energy particles produced in a cyclotron
finds dee positive and dee D2 negative. It now gets are used to bombard nuclei and study the resulting
accelerated towards dee D2. It moves faster through D2 nuclear reactions and hence investigate nuclear
describing a larger semicircle than before. Thus if the structure.
frequency of the applied voltage is kept exactly the
2. The high energy particles are used to produce
same as the frequency of revolution of the proton, then
other high energy particles, such as neutrons, by
every time the proton reaches the gap between the two collisions. These fast neutrons are used in atomic
dees, the electric field is reversed and proton receives a reactors.
push and finally it acquires very high energy. This is
called the cyclotron's resonance condition. The proton 3. It is used to implant ions into solids and modify
follows a spiral path. The accelerated proton is ejected their properties or even synthesise new materials.
through a window by a deflecting voltage and hits the 4. It is used to produce radioactive isotopes which

w
target. are used in hospitals for diagnosis and treatment.
Maximum K.E. of the accelerated ions. The ions
will attain maximum velocity near the periphery of the For Your Knowledge

Flo
dees. If Vq is the maximum velocity acquired by the
ions and is the radius of the dees, then > As the magnetic force on a charged particle acts
perpendicular to the velocity, it does not do any work

ee
2
mv
0 on the particle. As a result, the kinetic energy or the
= qVQB or V

Fr
0 ”
'b m speed of the particle does not change due to the
magnetic force.
The maximum kinetic energy of the ions will be
> When a charged particle is projected into a uniform
for
ur
fC, = - mUn = — m -—- magnetic field with its initial velocity perpendicular
^2 ° 2 t m to the field, the magnetic force acts on the charged
particle perpendicular to both the magnetic field and
ks
or its direction of motion. This force produces
Yo

2m centripetal force to make the particle move in a circle


oo

in a plane perpendicular to the magnetic field.


Limitations of cyclotron :
eB

> When a charged particle moves perpendicular to a


1. According to the Einstein's special theory of relativity, uniform magnetic field : (i) its path is circular in a
the mass of a particle increases with the increase in its plane perpendicular to the magnetic field and its
velocity as
r

direction of motion, {it) the radius of the circular path


ou
ad

m =
is proportional to its momentum, {Hi) the kinetic
energy and speed of the particle do not change,
Y

(iu) the force acting on the particle is independent of


where is the rest mass of the particle. At high the radius of the circular orbit but is proportional to
velocities, the cyclotron frequency (/^ =qBI2nm) will
nd

its speed i.e., F xr° and F xv and (u) the period of


Re

decrease due to increase in mass. This will throw the revolution of the charged particle is independent of
particles out of resonance with the oscillating field.
Fi

its speed and the radius of its circular orbit.


That is, as the ions reach the gap between the dees, the > When a charged particle is projected into a uniform
polarity of the dees is not reversed at that instant. magnetic field at an arbitrary angle with the field, the
Consequently the ions are not accelerated further. component of the initial velocity parallel to the magnetic
The above drawback is overcome either by increasing field will make the particle move along the direction
magnetic field as in a synchrotron or by decreasing the of the field while the perpendicular component will
frequency of the alternating electric field as in a compel it to follow a circular path. As a result, the
particle will follow a helical path with its axis parallel
synchro-cyclotron. to the field.
2. Electrons cannot be accelerated in a cyclotron. A
> In a cyclotron, it is the electric field which accelerates
large increase in their energy increases their velocity to the charged particles. The magnetic field does not
a very large extent. This throws the electrons out of change the speed, it only makes the charged particle
step with the oscillating field. to cross the same electric field again and again by
3. Neutrons, being electrically neutral, cannot be making it move along a circular path.
accelerated in a cyclotron.
PHYSICS-XII

Examples based on The operating magnetic field for accelerating


Cyclotron protons is
-27
Formulae Used
2 x3.14xl.67xl0 X 10^
B =
-19
e 1.6x10
For the accelerated charged particle,
qBr = 0.66 T.
1. Velocity, v =
m
Kinetic energy of the emerging beam will be
2mu
2. Period of revolution, T =
qB Kmax ^ _ (1.6 X 10“^^)^ X (0.66)^ x (0.6)^
-27
2-, 2 X 1.67 X 10
3. Cyclotron frequency, = £l -12
2nm 1.2x10

_^w = 1.2 X 10"*^ J = -13


MeV
4. Maximum kinetic energy, 1.602x10
ax
2 m
= 7.4 MeV.
where R is the radius of the dees.
Example 53. In a cyclotron, a magnetic induction of 1.4 T
Units Used
is used to accelerate protons. Hozo rapidly should the electric
Bis in tesla, vin ms“^ r in metre. Tin second and field between the dees be reversed ? The mass and charge of
proton are 1.67 x kg and 1.6 x 10“ C respectively.
fi in Hz.
Solution. Here B = 1.4T, m = 1.67xl0“^^ kg,
Example 51. Deutrojis are accelerated in a cyclotron that c = 1.6xl0 -19 c
has an oscillatoryfrequency ofltf Hz and a dee radius of 50 cm.
The time required by a charged particle to complete
(i) Wliat is the strength of the magnetic field needed to acce a semicircle in a dee is
lerate the deutrons ? (ii) What is the energy ofdeutrons emerging -27
from the cyclotron. Mass of a deutron=3.34 x 10 kg and t =
Jtm _ 3.14x1.67x10 = 2.34xl0“®s
charge of a deutron =1.6 x 10“^^ C eB~ 1.6 X10“^^ X 1.4
Solution. V =10^ Hz, R = 50 cm =0.50 m, Thus the direction of electric field should reverse

m = 3.34xl0“^^kg, <7 = 1.6x10“^^ C after every 2.34 x 10“ ® s.


qB The frequency of the applied electric field should be
(/) Cyclotron frequency, = 1 1
2k m
/c = 2f = 2.14 X 10^ Hz.
2x2.34x10"®
B =
iKmfi _2x3.14x3.34xl0~^^xl0^
-19
'I 1.6x10 Example 54. If the maximum value of accelerating potential
= 1.3 T. provided by a radio frequency oscillator be 20 kV, find the
number of revolutiojis made by a proton in a cyclotron to
(") ^ ax
2 m
achieve one fifth of the speed of light. Mass of a proton
(1.6 X 10“^^)^ X (1.3)^ X (0.50)^ = 1.67xW~^‘^ kg.
2x3.34x10
-27 Solution. In a cyclotron, a proton gains energy eV,
-12
when it crosses a region of potential difference V. In
1.62 X 10
= 1.62x10
-12
J = MeV one revolution, the particle crosses the gap twice. So
-13
1.6x10 the energy gained in each revolution =2 eV.
= 10.125 MeV. Suppose the particle makes n revolutions before
emerging from the dees. The gain in its kinetic energy
Example 52. A cyclotron's oscillator frequency is 10 MHz. will be
Wliat should be the operating magnetic field for accelerating 2
1A mv
protons ? If the radius of the 'dees' is 60 cm, what is the kinetic
9
— mv =2 neV or n =
2 4eV
energy of the proton beam produced by the accelerator ?
(e = 1.60x 10"^ C nip =1.67 x 10“^^ kg). Express your -13 Given V = —
c 3x10
8

= 0.6x10® ms"\
answer in units of MeV (1 MeV = 1.602 x 10 /)● 5 5
[CBSE OD 05 ; NCERT]

Solution. Here fi =10 MHz =10^ Hz, m = 1.67xl0“^^ kg


x(0.6x 10®)^
-27
-19 1.67x10
e = 1.6xl0 c. R =60 cm =0.6 m. n = = 470 revolutions.
4 X 1.6 x 10“^^ X 20x10®
m
^=1.67x10"^^ kg
MAGNETIC EFFECT OF CURRENT

-27
■Km 3.14x6.68x10 8
roblems For Practice 4. t = = 5.25 X 10'°s
-19
qB 3.2 X 10 xl.25

1. An electron of energy 10,000 eV describes a circular Direction of electric field should be reversed after
path in a plane at right angles to a uniform every 5.25 x 10~*s.
magnetic field of 0.01 T. (a) What is the radius of the
Applied frequency.
circular orbit ? (t) What is the cyclotron frequency ? 1 1
(c) What is the period of its revolution ? (rf) What is -8
= 9.5x10^ Hz
2t 2x5.25x10
the direction of revolution as viewed by an
observer looking in the direction of the field ? As r =
mv

(Ans. 3.4 X10"^ m, 2.8 x 10®s"^, qB


3.6 X10“^s, clockwise sense). -19
rqB 0.25 x 3.2 x 10 X 1.25
2. The protons are accelerated by a cyclotron, when a V =
m 6.68x10
-27

magnetic field of 2.0 T is applied perpendicular to

w
the plane of the dees. Calculate the energy of the = 1.5 X 10^ ms"‘‘
proton in MeV, if the circular path of the protons
K = mv
has a radius of 40 cm before the protons leave the 2
cyclotron. Given mass of a proton = 1.67 x 10“^^kg.

Flo
..

(Ans. 30.6 MeV) ~ 2^ ^ ^ ^ ^

ee
3. A cyclotron has an oscillatory frequency of 12 MHz _ y 5 jq
-13
J
and a dee radius of 50 cm. Calculate the magnetic “

Fr
field required to accelerate deutrons of mass
3.3x10""^^ kg and charge 1.6 X10"^^ C. What is the 4.17 FORCE ON A CURRENT-CARRYING
energy of the deutrons emerging from the cyclotron ? CONDUCTOR IN A MAGNETIC FIELD
for
ur
(Ans. 1.56 T, 14.7 MeV) 18. Describe an experiment to illustrate that a current-
-27
4. Alpha particles of mass 6.68 x 10 kg and charge carrying conductor experiences a force in a magnetic
-19
3.2x10
C are accelerated in a cyclotron in which fi^ld. What is the cause of this force ?
ks
a magnetic field of 1.25 T is applied perpendicular
Yo

Force on a current-carrying conductor in a magnetic


to the dees. How rapidly should the electric field
oo

between the dees be reversed ? What are the


field. When a conductor carrying a current is placed in
an external magnetic field, it experiences a mechanical
eB

velocity and kinetic energy of an alpha particle


when it moves in a circular orbit of radius 25 cm ?
force. To demonstrate this force, take a small
aluminium rod AB. Suspend it horizontally by means
(Ans. 9.5 X 10^ Hz, 1.5 x 10^ ms ,7.5x10"’^ J)
-1

of connecting wires from a stand, as shown in Fig. 4.72.


r
ou

HINTS
ad

/-V

q^^R^ (1.6xlQ-^^)^ x(2.0)-x(0.40)^


Y

2. Kn ax
2 m 2x1.67x10
-27

(1.6 X 10'^^)^ X (2.0)^ X (0.40)^


nd
Re

MeV
-27 -13
2x1.67x10 X 1.6x10
Fi

= 30.6 MeV.

qB
3. As /, = 27rm

.-. B=
2 Tim/. _ 2 X 3.142 x 3.3 x 10"^^ x 12 x 10*^
-19
‘1 1.6x10

= 1.56 T.

Fig. 4.72 Force on a current in a magnetic field.


_ q^^R} _ (1.6 X lQ-^^)^x(1.56)^ x (0.50)^
ax -17
2 m 2x3.3x10
Place a strong horse-shoe magnet in such a way that
(1.6 X 10"*^)^ X (1.56)^ X (0.50)^ the rod is between the two poles with the field directed
MeV
2x3.3x10
-27
X 1.6x10
-13 upwards. Now, if a current is passed through the rod
from A to R the rod gets deflected to the right. If we
= 14.7 MeV. reverse the direction of current or interchange the
s 4.44

poles of the magnet, the deflection of the rod is also


reversed. The direction of force is perpendicular to
both the current and the magnetic field and is given by
PHYSICS-XII

If 11 represents a current element vector in the


direction of current, then vectors / and will have
Fleming's left hand rule. opposite directions and we can take
Cause of the force on a current-carrying conductor -lv^=vj
in a magnetic field. A current is an assembly of moving
charges and a magnetic field exerts a force on a moving ? = enA -yB)
charge. That is why a current-carrying conductor when
But enAv^ = current, /
placed in a magnetic field experiences a sideways force
as the force experienced by the moving charges (free Hence ? = I{T x B)
electrons) is transmitted to the conductor as a whole.
Magnitude of force. The magnitude of the force on
19. Derive an expression for the force experienced by a the current-carrying conductor is given by
current carrying straight conductor placed in a magnetic F=llBsmQ
field. Under what conditions, is this force
(i) zero and
where 0 is the angle between the direction of the
(ii) maximum ?
magnetic field and the direction of flow of current.
State the rule used to determine the direction of this
force. Special Cases
Expression for the force on a current carrying (0 If 0=0° or 180°, then
conductor in a magnetic field. As shown in Fig. 4.73, f = I/B{0)=0
consider a conductor RS of length I, area of cross-
section A, carrying current I along direction R-^ Sat Thus a current-carrying conductor placed parallel to the
direction of the magneticfield does not experience anyforce,
an angle 0 with Y-axis. The field B acts along +ve
{ii) If 0 =90°, then
Y-direction. The electrons drift towards end R with
f =1/Bsin90° = //B
velocity v^. Each electron experiences a magnetic or F.max = BB
Lorentz force along +i?eZ-axis, which is given by
Thus a current-carrying conductor placed perpendicular to
the direction of a magnetic field experiences a maximum force.

Direction of force. The direction of force on a


current-carrying conductor placed in a perpendicular
magnetic field is given by Fleming's left hand rule.
Stretch the thumb and the first two fingers of the left hand in
mutually perpendicular directions. If the forefinger points in
the direction of the magnetic field, central finger in the
direction of current, then the thumb gives the direction of
force on the conductor. If the field B is along
+ Y-direction, the current I along -i- X-direction, then
the force acts along + Z-direction.

Fig. 4.73 Force on a current in a magnetic field. Examples based on


Force on a Current-Carrying
If n is the number of free electrons per unit Conductor in a Magnetic Field
volume, then total number of electrons in the con
ductor is Formulae Used

N = nx volume = nAl
1. F =I{TxB) 2. F=//Bsin0
Total force on the conductor is
3- =

? = N / = nAl [“ e (X B )] Units Used

= enA [-/X B ] Force F is in newton, current I in ampere, length /


in metre and field B in tesla.
MAGNETIC EFFECT OF CURRENT 4.45

Example 55. A wire of length I carries a current I along the cular to the plane of paper, as shown in Fig. 4.74. Find the
—* A A A
magnitude and direction of the magnetic forces on the four
X-axis. A magnetic field B = B^ii + j + k ) tesla exists in
sides of the frame.
space. Find the inagnitude of the magnetic force on the wire.
I
Solution. As the wire carries current / along the d
A

X-axis, so / =l i

Also, B = ^^(i + j + k ) tesla


Magnetic force on the wire is b I

F =1{1 B)=I[lhB^{i+j-^h]
Fig. 4.74
= x(f + / + cf )]

w
A A A A A A Solution. By symmetry, the current through each
= Bq I l[i X / + / X j + i X ] of the four sides will be 1 A. Also,
/ = 20 cm =0.20 m, B=0.25T
= ^//(0^ + ^' -j ) = {k -j )B^Il

Flo
Magnitude of force on each side is
Magnitude of the magnetic force is f = 7/Bsin90°

ee
F = ^l^ + {-lf £Jj//=V2 B^ll newton. = lx0.20x0.25x 1=0.05 N

Fr
Example 56. The horizontal component of the earth's By Fleming's left hand rule, forces on ab and dc will
be towards left and on ad and be downward.
magnetic field at a certain place is 3.0xl0"^r and the
direction of the field is from the geographic south to the Example 58. A magnetic field of 1.0 T is produced by an
for
ur
geographic north. A very long straight conductor is carrying electromagnet in a cylindrical region of radius 4.0 cm, as
a steady current of 1 A. What is the force per unit length on shown in Fig. 4.75. A zuire, carrying
it when it is placed on a horizontal table and the direction of current of 2.0 A, is placed perpen
ks
N
the current is (a) east to west, (b) south to north ? dicular to and intersecting the axis
Yo

[NCERT] of the cylindrical region. Find the


oo

Solution. The force on a conductor of length / magnetic force acting on the wire.
eB

placed in a magnetic field 6, and carrying current /, is Solution. Clearly, the mag
S
F = IIB sin e netic field acts vertically down
wards while the current flows
The force per unit length will be
r

horizontally, so
ou

Fig. 4.75
ad

0=90°.
/ = — = /B sin 0
I
Y

Length of the wire in the cylindrical region


where 0 is the angle that the conductor makes with the = 2r=2 X 4.0 cm =0.08 m
direction of 6 .
nd
Re

f = //Bsin 90° =2.0 x 0.08 x 1.0 x 1 = 0.16 N

(fl) When the current flows east to west, 0 =90°. This force acts on the wire normally into the plane
Fi

/=/Bsin90° = 1x3.0 X 10“^ xl of paper.


= 3.0xlO-^Nm"^ Example 59. A straight wire of mass 200 g and length
1.5 m carries a current of 2 A. It is suspended in mid-air by a
According to Fleming's left hand rule, this force
acts vertically downwards, uniform horizontal magnetic field B. What is the magnitude
(b) When the current flows from south to north. of the magnetic field ? [NCERT ; CBSE F 15]
0=0° Solution. Suppose that a wire AB carries a current
/=//sin0° = 0 of 2 A in the direction as shown in Fig. 4.76. The weight
mg of the wire acts vertically downwards. Therefore,
Thus the force per unit length of the conductor is
zero.
according to Fleming's left hand rule, the magnetic field
B must act perpendicularly into the plane of paper so
Example 57. A current of 2 A enters at the corner 'a' of a
squareframe of side 20 cm and leaves at opposite corner 'c'. A that the magnetic force F on the wire acts vertically
magnetic field of 6 = 0.25 7 acts in a direction perpendi- upwards.
PHYSICS-XII

X X X F X X X

X X X X X X

X / X X I X X
A B
X X X X X X

X X X X X X

X X X mg X X X
Fig. 4.77
Fig. 4.76
Mgsin30°=//Bcos30°
For mid-air suspension.
' MA gtan30°
Magnetic force on the wire = Weight of the wire 7 =
/ B
IIB sin 90° = mg
mg
0.30 X 9.8 x(lV3) A
or B= 0.15
II
2x9.8x1.732
But m = 200 g=0.2 kg, g=9.8ms^ 7=1.5m, A
7=2 A 3
0.2 X 9.8
B=- = 0.65 T. = 11.32 A.
1.5x2
Example 62. A short conductor of length 5.0 cm is placed
Example 60. What is the force on a zuire of length 4.0 cm parallel to a long conductor of length 1.5 m near its centre,
placed inside a solenoid near its centre, making an angle of The conductors carry currents 4.0 A and 3.0 A respectively
60° zvith its axis ? The zvire carries a current of 12 A and the in the same direction. What is the total force experienced
magnetic field due to the solenoid has a magnitude of 0.25 T. by the long conductor zvhen they are 3.0 cm apart ?
[NCERT]
Solution. The force on a conductor of length /
placed irr a magnetic field B, and carrying current 7, is Solution. As the two conductors have different
F = llB sin 0 lengths, the longer conductor may be considered to be
where 0 is the angle that the conductor makes with the of infinite length. Therefore, magnetic field produced
direction of B.
by it at a distance of 3 cm (0.03 m) is given by

Since the field due to a solenoid near its centre is B = M-o I2 _ 4ti:x10"^x3 T=2xl0“^T
2it r 2k.x 0.03
along its axis, so 0 =60°.
Also7=12 A, 7=4.0cm=0.04m, B=0.25T Force on the short conductor due to this magnetic
field will be
F = 12 X 0.04 X 0.25 sin 60° = 0.10 N.
F=T/,B
ri
= 4x5x 10"^ x2 X10“^ N
Example 61. On a smooth plane inclined at 30° zvith the
horizontal, a thin current-carrying metallic rod is placed = 4x10'^ N
parallel to the horizontal ground. The plane is located in a According to Newton's third law, the longer
uniform magneticfield of 0.15 T in the vertical direction. For conductor will also experience a force of reaction equal
zohat value of current can the rod remain stationary ? The to
-6
4.0 X 10 N. As the currents are in the same
mass per unit length of the rod is 0.30 kg m~^. (NCERT) direction, the force is attractive.
Solution. As shown in Fig. 4.77, various forces Example 63. A solenoid 60 cm long and of radius 4.0 cm
acting on the current-carrying horizontal rod are has 3 layers ofzvindings of 300 turns each. A 2.0 cm long
(i) its weight Mg acting vertically downwards. zvire of mass 2.5 g lies inside the solenoid near its centre
(ii) horizontal force F^^^ = IIB due to the vertical normal to its axis; both the zvire and the axis of the solenoid
magnetic field B. are in the horizontal plane. The zvire is connected through
tzvo leads parallel to the axis of the solejioid to an external
In order that the rod remains stationary, the
battery zohich supplies a current of 6.0 A in the zvire. What
component of the weight of the rod along the inclined
value of current in the zvindings of the solenoid can support
plane must balance the component of force F^^^ = IIB
along the inclined plane.
the zveight of the zvire ? g =9.8 ms~^. [NCERT]
MAGNETIC EFFECT OF CURRENT

Solution. Let I be the current in the windings of the roblems For Practice
solenoid which can support the weight of the wire. Tlie
magnetic field inside the solenoid along its axis will be 1. A current of 1 A flows in a wire of length 0.1 m in a

B=p.QnI magnetic field of 0.5 T. Calculate the force acting on


Total number of turns the wire when the wire makes an angle of (a) 90° (b)
Now 7J =
0°, with respect to the magnetic field.
Length of the solenoid
(Ans. 0.05 N, 0)
300x3
= 1500 turns m ' 2. A current of 5.0 A is flowing upward in a long
~ 60 X10~^
vertical wire placed in a uniform horizontal
6= 47tx 10~^x 1500 X I =6nx 10^/ tesla northward magnetic field of 0.02 T. How much
force and in what direction will the field exert on
This field acts perpendicular to the current carrying
wire, therefore, the magnetic force on the wire will be 0.06 m length of the wire ?

w
F=l'lB= 6x(2x10“^)x6tcx10‘^ / newton (Ans. 6 X10 ^ N, towards west)
3. What is the magnitude of force on a wire of length
The current I would support the wire if the above
0.04 m placed inside a solenoid near its centre,
force equals the weight of the wire,

Flo
making an angle of 30° with its axis ? The wire
i.e., 6 X 2 X 10“^ X 6tix 10“^ / = 2.5X10“^ X 9.8
carries a current of 12 A and the magnetic field due
2.5xl0~^ X 9.8 . . to the solenoid is of magnitude 0.25 T.

ee
or 1 = ^ A = 108.3 A.
-6 [CBSE OD 10 Cl
72x3.14x10

Fr
(Ans. 0.06 N)
Example 64. Figure 4.78 shows a triangular loop PQR 4. A long straight conductor P carrying a current of
carrying current I. The triangle is equilateral with side equal 2 A is placed parallel to a short conductor Q of
for
ur
to 1. If a uniform magnetic field B exists parallel to PQ, then length 0.05 m carrying a current of 3 A. The two
find the forces acting on the three wires separately.
R
conductors are 0.10 m apart.
ks
Calculate
Yo

(fl) the magnetic field due to P at Q


oo

I B
!
(b) the approximate force on Q.
eB

(Ans.4xl0~^T,6xlO“^ N)
I
5. A straight wire 1 m long carries a current of 10 A at
Q
right angles to a uniform magnetic field of 1 Wb m"^.
r

Fig. 4.78 Find the mechanical force on the wire and the
ou
ad

power required to move it at 15 ms"^ in a plane at


Solution. As B || PQ, so force on wire PQ is
Y

right angles to the field. (Ans. 10 N, 150 W)


6. A wire AB making an angle of 30° with a horizontal
P, = / PQx B
nd

is supported by a magnetic field of 0.65 T, directed


Re

or
fj = 7x PQx Bx sin0°=0 normally into the plane of paper. If the wire carries
Fi

Force on wire QR, a current of 5 A, determine its mass per unit length.
(Ans. 0.2872 kg m"’)
?2=/QK xB
or P, = IIB sin 120
O.V3 BB
^ 2
By right hand rule, this force acts normally into the
plane of paper.
Force on wire RP,

^ = / RP xB
/3
or P, = //Bsinl20° = — IIB
^ 2
This force acts normally out of the plane of paper. Fig. 4.79
s 4.48

7. A horizontal wire 0.1 m long carries a current of


5 A. Find the magnitude and direction of the
magnetic field which can support the weight of
PHYSICS-XII

(b) Force on Q. F = IIB sin 0


= 3x0.05x4x10"^ xsin 90°
= 6.0xl0"^T.
wire assuming that its mass is kg m""*. 5. F= I/Bsin90° = 10xlxlxl = 10N
(Ans. 5.88 x 10“^ T) P=f:;=10xl5 = 150 W.
8. A conductor of length 10 cm is placed per 6. Force on wire AB, F = IIB sin 90° = IIB
pendicular to a uniform magnetic field of strength
100 oersted. If a charge of 5 C passes through it in Component of the force in the vertically upward
/o
5 s, find the force experienced by the conductor. direction = Fcos 30° = IIB. —
2
(Ans. 10“^ N)
If m is the mass per unit length of wire, then its
9. A conductor of length 20 cm is placed (f) parallel
weight = mig
(ii) perpendicular (Hi) inclined at an angle 30° to a
uniform magnetic field of 2 T. If a charge of 10 C
passes through it in 5 s, find the force experienced by
mig=IlB.^
the conductor. [Ans. (/) zero (») 0.8 N (m) 0.4 N] IB.43 5x0.65xV3
or m =
10. A current of 5.0 A exists in the loop shown in 2g 2x9.8
Fig. 4.80. The wire AB has a length of 50 cm and lies
= 0.2872 kg m"^.
in a magnetic field of 0.20 T. What is the magnetic
force acting on the wire ? 7. In equilibrium.
(Ans. 0.50 N, towards the inside of the loop) Magnetic force on wire = Weight of wire
or IIB sin 90° = mg
3x10"^ x9.8
or B = ^.^ = = 5.88x10"^ T.
X X X X X I I 5

X X X X X 8. F=//Ssin0 = ^./Bsin9O°
A B t
X X X^ X X
5x0.10xl00xl0"‘^xl
= 10"^ N.
Fig. 4.80 5

9. Proceed as in Problem 8 above.


11. A horizontal wire 0.1 m long having mass 3 g
carries a current of 5 A. Find the magnitude of the 10. F = IIB sin 0 = 5.0 X 0.50 x 0.20 x sin 90° = 0.50 N.
magnetic field which must act at 30° to the length of 11. F = IIBsin Q = mg
the wire in order to support its weight ? mg 3x10"^ x9.8
.●. B = = 0.1176 T.
(Ans. 0.1176 T) 11 sin 0 0.1 X 5 xsin 30°
12. Find the magnitude of the magnetic force on the
12. F = BB sin 0 = 5 X 0.30 x 0.25 sin (180° - 120°)
segment PQ placed in a magnetic field of 0.25 T, if a
= 5 X 0.30 X 0.25 X sin 60° = 0.32 N.
current of 5 A flows through it, as shown in
Fig. 4.81. (Ans. 0.32 N)
P
4.18 FORCES BETWEEN TWO PARALLEL
CURRENT-CARRYING CONDUCTORS
30 cm

20. Hozv will you show experimentally the existence


120°
of (i) attractive forces between parallel currents and
(ii) repulsive forces between anti-parallel currents ?
B
I Q I Forces between two parallel current-carrying
conductors. It was first observed by Ampere in 1820
Fig. 4.81
that two parallel straight conductors carrying currents in
HINTS
the same direction attract each other and those carrying
currents in the opposite directions repel each other.
3. F = IIB sin 0 = 12 X 0.04 x 0.25 sin 30° = 0.06 N,
Experiment 1. As shown in Fig. 4.82, the upper
4. {a) Magnetic field due to P at Q is ends of two wires are connected to the -ve terminal of
I 4jrxl0"^x2 a battery and their lower ends are connected to the +ve
B = = 4x10"^ T.
2k a 271x0.10 terminal of the battery through a mercury bath. When
MAGNETIC EFFECT OF CURRENT 4.49

the circuit is completed, the current flows in the two B D B D


wires in the same direction. The two wires are found to

be closer to each other, indicating a force of attraction


between them. B1 B1
/.1 - ^2- ^2

f2

/
+
F F

●>

}I /1 h

Hg /

w
T, P,

/
B.

Flo
Fig. 4.82 Attractive force between parallel currents.

A C A C
Experiment 2. As shown in Fig. 4.83, two wires are

ee
connected to a battery through a mercury bath in such

Fr
Fig. 4.84 (a) Parallel currents attract,
a way that current flows in them in succession. When
(b) Antiparallel currents repel.
the circuit is closed, the currents in the two wires flow
in opposite directions. The two wires move away from
for
This field acts perpendicular to the wire CD and
ur
each other, indicating a force of repulsion between
them. points into the plane of paper. It exerts a force on
current carrying wire CD. The force acting on length /
ks
of the wire CD will be
Yo

hp ^1 _ M-0 ^2
oo

./
= I2 IBj sin 90°= 1^1 ● 2kv 2k r
eB

Force per unit length.


r

/ 27rr
ou
ad

According to Fleming's left hand rule, this force


Y

acts at right angles to CD, towards AB in the plane of


the paper. Similarly, an equal force is exerted on the
nd
Re

wire AB by the field of wire CD. Thus when the currents


Fig. 4.83 Repulsive force between antiparallel currents. in the two wires are in the same direction, the forces
Fi

between them are attractive. It can be easily seen that


21. Derive an expression for the force per unit length —>

between two mifnitely long straight parallel current F,=-F,


carrying wires. Hence define one ampere. Also define
coulomb in terms of ampere. As shown in Fig. 4.84(1?), when the currents in the
Expression for the force between two parallel two parallel wires flow in opposite directions (anti
current-carrying wires. As shown in Fig. 4.84(fl), con parallel), the forces between the two wires are repulsive.
Thus,
sider two long parallel wires AB and CD carrying
currents f and l2- Let r be the separation between Parallel ciirrertts attract and antiparallel currents repel.
them.
Definition of ampere.
The magnetic field produced by current 7| at any
point on wire CD is When f = /^ = 1 A and r = 1 m, we get
Fq
f=^=2 X 10“^ Nm
-1
B,1 =
2k r 2k
s 4.50

One amycrc is that value ofsteadif current, xvhich


flowing in each of the two parallel infinitely long
PHYSICS-Xll

on

conductors of negligible cross-section placed in vacuum at


Solution. The magnetic force per unit length on the
wire Q due to the current in wire P is
h h
a distance ofl m from each other, piroduces between them a 2k' r
force ofl X 10”^ newton per metre of their length.
Definition of coulomb in terms of ampere. If a 25 A
steady current of 1 ampere is set up in a conductor, Q
then the quantity of charge that flows through its
cross-section in 1 second is called one coulomb. W
50 A
1C=1 As

Examples based on Fig. 4.85


Forces between Parallel
The currents in P and Q must have opposite
Current-Carrying Wires directions, only then Q will experience a repulsive
Formulae Used force which would balance the weight of Q.
1. Force per unit length, / =
ho 1] T p ^ 1^(1 ^1 h = W
2n r 2k r

2. Force on length I of one of the wires,


or r= —.
ho _2x 10"^ X 50x25
_ h 0 ^1 h ^ ● 2k W 0.075
2k r
= 3.33 X 10"^ m = 3.33 mm.
Units Used
Example 67. A current balance (or ampere balance) is a
Force is in newton, currents in ampere, distance r device for measuring currents. The current to be measured is
in metre and field B in tesla. arranged to go through two long parallel wires of equal
Constant Used length in opposite directions one of which is linked to the
pivot of the balance. The resulting repulsive force on the wire
Pq = 4n X 10" ^ TmA"'. is balanced by putting a suitable mass in the scale pan
hanging on the other side of the pivot. In one measurement,
Example 65. A current of 5.0 A flows through each oftxvo the mass in the scale pmn is 30.0 g, the length of the wires is
parallel long wires. The wires are 2.5 cm apmrt. Calculate the 50.0 cm each, and the separation between them is 10.0 mm.
force acting per unit length of each wire. Use the standard What is the value of the current being measured ? Take
value of constant required. What u'ill be the nature of the g =9.80 ms “ ^ and assinne that the arms of the balance are
force, if both currents flozo in the same direction ? equal. [NCERT]

Solution. Here /j = ^2 = 5 A, Solution, m =30.0 g =0.03 kg, / = 50 cm =0.50 m,


r = 2.5 cm =2.5x 10"^m, =4tix 10"^ TmA"^ r = 10.0 mm =0.01 m,;^> =9.8 ms“^
Force per unit length between two parallel conductors.
Force acting per unit length of each wire, LI
1'2

hp^l^2 _ 47tx 10”^x 5x5 2k r


/=
Inr 2;tx2.5xl0“^ .'. Force on a conductor of length /,
= 2x 10"^ Nm"^ F = ^.
2k r
As the currents in both the wires flow in the same
When the pan is balanced.
direction, the force will be attractive.
Weight in scale pan = Balancing force
Example 66. A long horizontal wire P c<7rr/t’s a current of I.C., mg =
iio Iffl .1
50 A. It is rigidly ifxed. Another ifne loire Q is pdaced 2k r

directly above and pmallel to P. The zveight of the wire Q is /2 = 2 K mgr _ 2 7t X 0.03 x 9.8 x 0.01
0.075 Nm~^ and it carries a current of 25 A. Find the
or

hn^ 4rtxl0"^x0.50
position of the wire Q from the wire P so that Q remains
= 29400
suspended due to the magnetic repulsion. Also indicate the
direction of current in Q with rcspiecf to P. / = n^9400 = 171.46 A.
MAGNETIC EFFECT OF CURRENT

B
Example 68. A rectangular loop of sides 25 cw and W cm A *■

carrying a current of 15 A is placed zuith its longer side


parallel to a long straight conductor 2.0 cm apart carrying a D

current of 25 A. What is the net force on the loop ? E F


(CBSE OD 05)
Solution. Consider the rectangular loop ABCD Fig. 4.87
placed near a long straight conductor XY, as shown in Force per unit length of AB due to current in CD is
Fig. 4.86. The arm AB will get attracted, while CD will
get repelled. Forces on arms BC and AD, being equal, 471X 10"^ X 5 X 5
/,= 271x1.0x10”^
opposite and collinear, will cancel each other. 2 717'
D C-r
= 5.0 X 10”^ Nm \ directed downward
Force per unit length of AB due to current in EF is

w
rj= 12 cm
4ti X 10”^ X 5 X 5
T
A
6
/z = 271x2.0x10”-
= 2 cm /, = 15A

Flo
X
/, = 25A
= 2.5 X 10”^ Nm \ directed downward
Total force per unit length of AB is

ee
Fig. 4.86
/=/l + /2

Fr
Current through the rectangular loop, /j =15 A -4
Nm directed downward
= 7.5 X 10
Current through the long wire XY, =25 A

Force on AB,
Force per unit length of CD due to current in AB is
for
ur
ILL 4tt X 10”^ X 5 X 5
F1 =^. ~ X length of conductor AB /i = 271x1.0x10”^
47t
'i
ks
10”^x2xl5x25 = 5.0 X 10”^ Nm ^ directed upward
X 25x10”^
Yo

2.0 X 10”^ Force per unit length of CD due to current in EF is


oo

= 9.375 X 10”^N (Attractive) 4tix 10”^ X 5 X 5


eB

fi ~ 271x1.0x10”^
Force on CD,
2 LI. = 5.0 X 10”^ Nm \ directed downward
— X length of conductor CD
r

^ 471 .-. Total force per unit length of CD =zero.


ou

»2
ad

10”^x2xl5x25
X 25x10"^ roblems For Practice
Y

12.0 X 10”^
-A 1. A long horizontal rigidly supported wire carries a
nd

= 1.5625x10 (Rqmlsive) airrent of 100 A. Directly above it and parallel to it


Re

.-. Net force on the loop, IS a fine wire that carries a current of 200 A and
Fi

F= Fj -F2 =9.375x10 -1.5625x10


-4 -4
weighs 0.05 Nm”^ How far above the wire should
= 7.8125x10”^ N-7.8X 10”^ N (Attractive) the second wire be kept to support it by magnetic
repulsion ? (Ans. 8 cm)
Thus the force on the loop will act towards the long 2. A wire AB is carrying a steady current of 12 A and
conductor (attractive) if the current in its closer side is is lying on the table. Another wire CD carrying 5 A
in the same direction as the current in the long is held directly above AB at a height of 1 mm. Find
conductor, otherwise it will be repulsive. the mass per unit length of the wire CD so that it
Example 69. In Fig. 4.87, the wires AB, CD and EF are remains suspended at its position when left free.
long and have identical resistances. The separation between Give the direction of the current flowing in CD with
the neighbouring wires is 1.0 cm. The wires AE and BF have respect to that in AB.
negligible resistance and the ammeter reads 15 A. Calculate [Take the value of g = 10 ms”^]. [CBSE OD 131

the magnetic force per unit length of AB and CD. (Ans. 1.2 X 10”^ kg m”\ in the opposite direction)
Solution. By symmetry, current through each of 3. Two very long, straight, parallel wires A and B
the wires AB, CD and EF is 5 A. carry currents of 10 A and 20 A respectively, and
s 4.52

are at a distance 20 cm apart, as shown in Fig. 4.88.


If a third wire C (length 15 cm) having a current of
10 A is placed between them, how much force will
PHYSICS-XII

HINTS

1. Force of repulsion per unit length,


471 X 10"^ X 100x200 -1
act on C ? The direction of current in all the three /= = 0.05 Nm
2ti X r
wires is same. (Ans. 3.0 X 10“^ N, towards B) -3
A C B 4x10
r = m = 8 cm.
0.05
A B
2. Weight per unit length of upper wire
= Magnetic force per unit length
0 *■
w
I ^ mg ^MqV2
/ / I 2nr
D C

20 cm -»
Mass per unit length
_ m _ Pq/j 12 _ 47T xlO"^ xl2x5
Fig. 4.88 Fig. 4.89
I iKrg 27ix10~^x10
4. In Fig. 4.89, ABCD is a rectangular loop made of = 1.2x10'^ Icgm"’
uniform wire. The length AD=BC= 1cm. Tlie The direction of current in CD must be opposite to
sides AB and DC are much longer than AD or BC. that of current in AB so that the force between the
Find the magnetic force per unit length acting on two wires is repulsive.
the wire DC due to the wire AB if the ammeter
3, Force on C due to A,
reads 10 A. (Ans. 5 X 10"'* Nm"’, attractive) 4ti xlO"^ xlOxlO x0.15
K1 =
5. A rectangular loop of wire of size 2cm x 5 cm carries 2ti xO.lO
a steady current of 1 A. A straight long wire = 3.0 X 10"^ N, towards A
carrying 4 A current is
Force on C due to B,
kept near the loop as 2 cm

shown in Fig. 4.90. If the / = 4 A


4nxl0“^x20xl0x0.15
P2 =
loop and the wire are 271x0.10
1 A
coplanar, find (/) the E = 6.0 X 10“^ N, towards B
torque acting on the loop in Net force on C
and (ii) the magnitude = B, - = 3.0 X10^ N, towards B.
and direction of the force

on the loop due to the 4n X 10”^ X 5 X 5


1 cm 4. / =
current-carrying wire. 27tr 27t X 0.01
ICBSE D 121
Fig. 4.90 = 5 xlO Nm \ attractive
[Ans. (/) T = 0 (//) f = 2.67 pN, towards 5. (r) As direction of the magnetic field due to the
the straight wire] straight conductor is parallel to the area vector
6. A square loop of side 20 cm carrying current of 1 A (both normal to the plane of the loop), so
is kept near an infinite long straight wire carrying a torque r = 0.
current of 2 A in the same plane as shown in (//) Proceed as in Example 68 on page 4.51
Fig. 4.91. Calculate the magnitudeand direction of
6. F =
the net force exerted on the loop due to the current- 27tr
carrying conductor. ICBSE OD 15C]
.’. Net force on sides ab and cd
(Ans. 5.33 xlO'^N)
2 A _ I '
2ti:
T 'i ^2.
10 cm
I A
47ixl0"^x2xlx20xl0"^r 1 1
b -r N
3(1 cm
2ti 10x10"^ 30x10"^
20 cm 20
= 4xl0'^x20 N = 5.33xlO’^N
10x30
f
20 cm
This force is directed towards the infinitely long
Fig. 4.91 straight wire.
MAGNETIC EFFECT OF CURRENT 4.53

4.19 TORQUE EXPERIENCED BY A CURRENT opposite and coUinear (along the axis of the loop), so
LOOP IN A UNIFORM MAGNETIC FIELD their resultant is zero.

22. Derive an expression for the torque acting on a The side PQ experiences a normal inward force
current carrying loop suspended in a uniform magnetic equal to WB while the side RS experiences an equal
jjgld normal outward force. Tliese two forces form a couple
which exerts a torque given by
Torque on a current loop in a uniform magnetic
field. As shown in Fig. 4.92(a), consider a rectangular T = Force X perpendicular distance
coil PQRS suspended in a uniform magnetic field B, = IbBx a sin 0 = IB A sin 0
with its axis perpendicular to the field.
If the rectangular loop has N turns, the torque
increases N times i.e.,

w
T = NIBA sin 0

But NIA = m, the magnetic moment of the loop, so

Flo
T = mB sin 0

In vector notation, the torque x is given by

ee
N S

Fr
X = m X B

The direction of the torque x is such that it rotates


for
ur
the loop anticlockwise about the axis of suspension.

Special Cases
ks
(?) When 0 =0°, X =0,/.c., the torque is minimum
Yo

when the plane of the loop is perpendicular to


oo

the magnetic field.


eB

(//) When B =90°, x = NIBA, i.e., the torque is


maximum when the plane of the loop is parallel
to the magnetic field. Thus
r

= N1BA
ou

X
ad

max

nnwn
Y

For Your Knowledge


nd
Re

> The expression for torque (x = NIBA sin 0) holds for a


Fi

Fig. 4.92 (a) A rectangular loop PQRS in a uniform magnetic


planar loop of any shape. Tlius the torque on a planar
current loop depends on current, strength of magnetic
field B . (b) Top view of the loop, magnetic dipole field and area of the loop. It is independent of the shape
moment m is shown. of the loop.
> For a planar current loop of a given perimeter sus
Let / = current flowing through the coil PQRS pended in a magnetic field, the torque is maximum
a,b = sides of the coil PQRS when the loop is circular in shape. Tliis is because for
a given perimeter, a circle has maximum area.
A = ab = area of the coil
> The expression x* = th x ^ for the torque on a airrent
0 = angle between the direction of B and
loop in a magnetic field is analogous to the expression
normal to the plane of the coil.
X* = ^ X ^ for the torque on an electric dipole in an
According to Fleming's left hand rule, the electric field. This supports the fact that a current loop
magnetic forces on sides PS and QR are equal. is a magnetic dipole.
4.54 PHYSICS-XII

> The torque on a current loop in a magnetic field is the Solution. Here/= 10 A, N= 100, B = 3T,
operating principle of the electric motor and most
A = 40 cm X 20 cm = 800 cm^ = 8 x 10“^
electric meters used for measuring currents and
voltages, called galvanometers. 0=90^-60° =30°

> If the direction of the magnetic field makes an angle a = Angle between B and the normal to
with the plane of the current loop, then
the plane of the coil
e+ a = 9O°or0 = 9O°-a
Torque,
X = N/BA sin (90° - a) =/VfB/l cos a.
t = NIBA sin 0
> In a uniform magnetic field, the net magnetic force on
a current loop is zero but torque acting on it may be = 100 X 10 X 3 X 8 X 10“ ^ X sin 30°
zero or non-zero. = 120 Nm.

> In a non-uniform magnetic field, the net magnetic Example 72. Given a uniform magnetic field of 100 G in
force on a current loop is non-zero but torque acting an east to zvest direction and a 44 cm long zoire zoith a
on it may be zero or non-zero. current carrying capacity of almost 10 A. What is the shape
and orientation of the loop made of this zoire zvhich yields
maximum turning effect on the loop ? What is the magni
Examples based on tude of the maximum torque ?
Torques on Current Loops Solution. B = 100 G = 100 x 10“'^ T, / = 10 A
Formulae Used The torque on the planar loop will be maximum if
its area is maximum. Since for a given perimeter, a
Torque on a current loop in a magnetic field,
circle encloses maximum area, therefore, the wire should
T = MBA sin 9 = mB sin 0
be bent into a circular loop of radius r given by
where m = N/A = magnetic dipole moment of the 27tr= 44
current loop.
M_44x7 = 7 cm =0.07 m
In vector form, x = m x B . r-
2:i“2x22
Units Used
.'. Area of the circular loop,
Current / is in ampere, area A in m field B in tesla, . ■7

torque x in Nm and magnetic moment m in Am^. A = Tir =


yX (0.07)^ =0.0154
Again, for maximum torque, the loop must be
Example 70. The maximum torque acting on a coil of oriented with its plane in N-S direction.
effective area 0.04 n? is 4 x 10“® Nm zvhen the current in it
Then
is 100 pA Find the magnetic induction in zvhich it is kept.
= /BA = 10 x 100 X 10“^ X 0.0154 Nm
Solution. A =0.04 m^, x max
8
= 4x10“° Nm, max

= 1.54xlO"^Nm.
/ = 100pA=10^ A, N=1
As X = NIBA Example 73. A circular coil of 25 turns and radius 6.0 cm,
carrying a current of 10 A, is suspended vertically in a
max

Magnetic induction. uniform magnetic field of magnitude 1.2 T. The field lines
run horizontally in the plane of the coil. Calculate the force
max

NIA
and the torque on the coil due to the magnetic field. In zvhich
direction should a balancmg torque be applied to prevent the
-8
4x10 coil from turning ?
lx 10^x0.04
Solution. Consider any element ^ of the wire.
= 10“^ Wbm“^ Force on this element is / f^x For each element d!,
Example 71. Calculate the torque on a 100 turn there is another element -df on the current loop.
rectangular coil of length 40 cm and breadth 20 cm, carrying Forces on each pair of such elements cancel out.
a current of 20 A, zvhen placed making an angle of 60° ivith a Hence net force on the coil in a uniform magnetic field
magnetic ifeld of 3 T. is zero.
magnetic effect of current 4.55

The torque on the coil is zero when m is parallel or

I
antiparallel to 6, i.e., when it lies in the VZ-plane. The
-► coil will be in stable equilibrium when m is parallel to
B
B and in unstable equilibrium when iii is antiparallel
H
to B.

Example 75. A 100-hmis coil kept in n magnetic field


Fig. 4.93 B =0.05 Wbnf-, carrie^i a current of 1 A, as shozvn in
Fig. 4.94. Find the torque acting on the coil.
In Fig. 4.93, n is a unit vector normal to the plane of
P B
the loop, directed outward. The angle between n and

w
B is 90°. The magnitude of the torque acting on the N 15 cm S

loop is

Flo
T = MBA sin 0

= 25 X 10 X 1.2 X K {O.Oef X sin 90° 15 cm —H

ee
= 3.4 Nm

Fr
Fig. 4.94
This torque acts in the vertically upward direction
producing turning effect in the direction of curved Solution. Here the angle between the axis of
arrow. To prevent the coil from turning, a balancing
for
ur
torque t' = x must be applied. rotation of the coil and the magnetic field B is 90°.
Example 74. A rectangular coil of sides 8 cm and 6 cm N=100, /=1A,
ks
having 2000 turns and carrying a current of 200 mA is A = 15 cm X 15 cm =225 x 10 "^m^,
Yo

placed in a uniform magnetic field of 0.2 T directed along the B = 0.5Wbm'^, 9=90°
oo

+ ve X-axis.

(?) What is the maximum torque the coil can experience ? Torque,
eB

In which orientation does it experience the maximum T = NIBA sin 0


torque ? = 100 X 1X 0.5 X 225 X lO'"* x sin 90°
r

(ii) For which orientations of the coil is the torque zero ? = 1.125 Nm
ou

When is this equilibrium stable and when is it


ad

unstable ? INCERT] As the force on the arm PQ acts upwards and that
Y

Solution. I =8 cm =0.08 m, b =6 cm =0.06 m on SR downwards, so the torque acts anticlockwise.


N=2000, ;=200mA=0.2 A, B=0.2T Example 76. A parallelogram-shaped coil PQRS ofsides 0.7 m
nd
Re

The magnitude of the magnetic dipole moment is and 0.5 m carries a current of'1.5 A, as shown in Fig. 4.95. It is
Fi

given by placed in a magnetic field B = 40 T parallel to PS. Find (i)


m = NM=2000x0.2 X (0.08 x 0.06) = 1.92 Am' forces on the sides of the coil and (ii) torque on the coil.
The direction of m is normal to area A of the coil from ■p 1.5 A Q
S-pole to N-pole. Magnitude of torque on the coil is
T = mB sin 0
<0 B
<a-
For maximum torque, m must be perpendicular to B.
Therefore, .60°
S
t
max
= mB = 1.92 X 0.2 = 0.384 Nm 0.7 m

Thus the torque on the coil is maximum whenever


Fig. 4.95
the X-axis lies in the plane of the coil.
4.56 PHYSICS-XII

Solution. As the magnetic field B acts parallel to Initial torque.


sides PS and QR, no forces act on these sides. x= mBsin0° = 0

Force on side PQ is Final torque.


P = //Bsin 0 =1.5 X 0.7 x 40 x sin 60° T = mBsin90° = 10 x 2 x 1 = 20 Nm.

= 1.5 X 0.7 X 40 X 0.866 = 36.37 N (iv) By Newton's second law,


According to Fleming's left hand rule, the force F , , d(£t , do) dQ d(a
x= la = I — = I — = I. —. Q)
will act normally upward. dt dO df dB

Similarly, force on side SR will also be 36.37 N, but But T = niB sin 0

it will be directed normally inward. doi


. (0= mBsin 0 or /o)d{o = ?hB sin 0 dB
dB
(ii) As the forces on the sides PQ and SR are equal,
opposite and parallel, they form a couple which exerts When 0 changes from 0 to ti / 2, suppose the angular
a torque. speed changes from 0 to co. Integrating above equation
X = Force x ± distance between the two forces within these limits of 0 and co, we get
= Fx FS sin 60° =36.37x0.5x0.866
(0 ti/2
I cotico= niB sin 0 do
= 15.75 Nm.
0 0

Example 77. A 100 turn closely woiuid circular coil of 2

radius 10 cm carries a current of 3.2 A. (i) What is the field


^ =mB[-cos0]
1 n/2
0
at the centre of the coil ? (ii) What is the magnetic moment of JO

this arrangement ? 1 2 ^
/co = - mB cos — cos 0 = inB
The coil is placed in a vertical plane and is free
to rotate 2 2
about a horizontal axis which coincides with its diameter. A
uniform magnetic field of2T in the horizontal direction or 12 mB 12x10x2
co = = 20 rads"^
exists such that initially the axis of the coil is in the direction I 0.1

of the ifeld. The coil rotates through an angle of 90° under


Example 78. A solenoid of length 0.4 m and having
the influence of the magnetic field. (Hi) What are the 500 turns of wire carries a current o/3 A. A thin coil having
magnitudes of the torques on the coil in the initial and final 10 turns of wire and of radius 0.01 m carries a current of 0.4 A.
position ? (iv) What is the angular speed acquired by the coil Calculate the torque required to hold the coil in the middle of
ivhen it has rotated by 90° ? TheM.l. of the coil isO.l kg n?. the solenoid with its axis perpendicular to the axis of the
[NCERTJ solenoid (pg =4tcx 10'^ V-s) A-m).
Solution. (0 Here N =100, / =3.2 A,
Solution. For solenoid, / =0.4 m, = 500, f=3 A
r=10 cm =0.1 m
For coil, N2= 10, r= 0.01m, /2=0.4A
Magnetic field at the centre of the coil,
Field inside the solenoid.
47tx lO'^x 100x3.2
B= [v 71x3.2-10] B=
2r 2x0.1 I
-, along the axis of solenoid.
4xl0"^xl0 Magnetic moment of coil,
= 2 X 10“^ T
2x0.1
m = ^2 /2 A = 1^2 /2 along the axis of coil.
The direction of the field is given by right hand As tlxe axis of the coil is perpendicular to the axis of
thumb rule.
solenoid, m and B will be perpendicular to each other.
(ii) Magnetic moment associated with the coil,
Required torque,
m = NIA = N/ X Tcr^ = 100 x 3.2 x 3.14 x (O.lf z = m B sin 0
= 10 Am^
. sin 90°
/
The direction of m is given by right hand rule. 47cx10 ^x 500x3
= 10x0.4x3;tx(0.01)^ xl
(Hi) Torque, x = mB sin 0 0.4

Initially, 0 = 0° = 67C^ X 10"^ =6x9.87x10"^ =5.92 X 10“* Nm.


MAGNETIC EFFECT OF CURRENT 4.51

Assume the area of cross-section of the wire to be


roblems For Practice
10” ^ and the free electron density is 10^^/m^.
ICBSE OD 08!
1. What is the maximum torque on a rectangular coil
-24
of area 5 cm x 12 cm of 600 turns, when carrying a [Ans. {(i) zero {b) zero (c) 1.5x10 N]
current of 10” ^ A in a magnetic field of 0.10 T ? HINTS
(Ans. 3.6x10“^ Nm) 1. A = 5cm X 12cm = 60x10
-4„2
m
2. What torque acts on a 40 turn coil of 100 cm^ area T
max
= NIBA
carrying a current of 10 A held with its axis at right = 600 x 10“^ X 0.10x60x10
angles to a uniform magnetic of 0.2 T ?
= 3.6xlO”*Nm-
(Ans. 0.8 Nm)
2. T = NIBA sin G = 40 X 10 X 0.2 x 100 x 10”^ sin 90°
3. A square shaped plane coil of area 100 cm^ of
= 0.8 Nm.
200 turns carries a steady current of 5 A. It is placed
in a uniform magnetic field of 0.2 T acting 3. T = NIBAsin 0 = 200 x5 xO.2 x 100 x 10“'^sin{90°-60“)

w
perpendicular to the plane of the coil. Calculate the = 1 Nm.
torque on the coil when its plane makes an angle of 4. r = IBA sin 0 = 2.0 x 2.0 x 10”^ x 0.40 x 0.25 x sin 90°
60° with the direction of the field. In which

Flo
= 4.0x10”^ Nm.
orientation will the coil be in stable equilibrium ? T 0.2
[CBSE OD 15C1 5. B =
NIA sin 0 100x2x0.05x0.04 xsin 90°

ee
(Ans. 1 Nm)
= 0.5 T.

Fr
4. A rectangular coil PQRS is placed in a uniform
6. X = N/B(7ir^) sin 0
magnetic field B, as shown in Fig. 4.96. Find the
= 500xl.0x0.40x3.14x(0.02)^ xsin 30°
torque on the coil when it carries a current of 2.0 A.
for
ur
= 0.1256 = 0.13 Nm.
Tlie magnitude of the field B is 2.0 x 10”^ T. 7. Proceed as in Exercise 4.25 on page 4.102.
(Ans. 4.0xl0”^Nm)
ks
> 4.20 MOVING COIL GALVANOMETER
Yo

40 cm
oo

23. Describe the principle, construction and zvorkitig


-♦

■►B
of a pivofed-t\/pe moving coil galvanometer. Define its
eB

in
, I
figure of merit.
Moving coil galvanometer. A galvanometer is a device
<N

to detect current in a circuit. The commonly used moving


r

coil galvanometer is named so because it uses a


ou
ad

current-carrying coil that rotates (or moves) in a


Y

Fig. 4.96 magnetic field due to the torque acting on it.


5. A rectangular coil of 100 turns has length 5 cm and In a D'Arsonval galvanometer, the coil is suspended
nd

on a phosphor-bronze wire. It is highly sensitive and


Re

width 4 cm. It is placed with its plane parallel to a


uniform magnetic field and a current of 2 A is requires careful handling. In Weston galvanometer, the
Fi

passed through the coil. If the torque acting on the coil is pivoted between two jewellel bearings. It is rugged
coil is 0.2 Nm, find the magnitude of the magnetic and portable though less sensitive, and is generally
field. (Ans. 0.5 T) used in laboratories. The basic principle of both types
6. A circular coil of radius 2.0 cm has 500 turns and
of galvanometers is same.
carries a current of 1.0 A. Its axis makes an angle of Principle. A current carrying coil placed in a magnetic
30° with the uniform magnetic field of 0.40 T that experiences a current dependent torque, which tends to
exists in the space. Find the torque acting on the rotate the coil and produces angular deflection.
coil. (Ans. 0.13 Nm) Construction. As shown in Fig. 4.97, a Weston
7. A circular coil of 200 turns and radius 10 cm is (pivoted-type) galvanometer consists of a rectangular
placed in a uniform magnetic field of 0.5 T, normal coil of fine insulated copper wire wound on a light
to the plane of the coil. If the current in the coil is non-magnetic metallic (aluminium) frame. The two
3.0 A, calculate the (/j) total torque on the coil, ends of the axle of this frame are pivoted between two
(b) total force on the coil, (c) average force on each jewelled bearings. The motion of the coil is controlled
electron in the coil, due to the magnetic field. by a pair of hair springs of phosphor-bronze. The inner
4.58 PHYSICS-Xll

ends of the springs are soldered to the two ends of the Tlieory and working. In Fig. 4.98(fi), we have
coil and the outer ends are connected to the binding / = current flowing through the coil PQRS
screws. The springs provide the restoring torque and a,b= sides of the rectangular coil PQRS
serve as current leads. A light aluminium pointer
A= nb- area of the coil
attached to the coil measures its deflection on a
suitable scale. N = number of turns in the coil.

Scale
s
I

Pointer— Permanent
T P
magnet

\
I
►B

> J
►B
N down
R
S

Pivot
a F
I
t-iron'

(n) (b)
Uniform radial
Fig. 4.98 (a) Rectangular loop PQRS in a uniform
magnetic field
magnetic field. (&) Top view of the loop.
(«)

Since the field is radial, the plane of the coil always


r
=1 remains parallel to the field B. The magnetic forces on
Pointer
sides PQ and SR are equal, opposite and collinear, so
their resultant is zero. According to Fleming's left rule,
the side PS experiences a normal inward force equal to
.4^
MIbB while the side QR experiences an equal normal
Concave Terminals
outward force. The two forces on sides PS and QR are
pole
Soft-iron equal and opposite. They form a couple and exert a
(.ylinder >● Jewelled torque given by
Hair bearing T = Force x Perpendicular distance
spring
= NlbB X a sin 90° = NIB (nb) = NIBA
Here 0 =90°, because the normal to the plane of coil
Fig. 4.97 (a) Top view (t) Front view of a
pivoted-type galvanometer. remains perpendicular to the field B in all positions.
The torque x deflects the coil through an angle a. A
The coil is symmetrically placed between the restoring torque is set up in the coil due to the elasticity
cylindrical pole pieces of a strong permanent horse- of the springs such that
shoe magnet. oc a or - fca
^restoring ^restoring
A cylindrical soft iron core is mounted symme where k is the torsion constant of the springs i.e.,
trically between the concave poles of the horse-shoe torque required to produce unit angular twist. In
magnet. This makes the lines of force pointing along equilibrium position,
the radii of a circle. Such a field is called a radial field. Restoring torque = Deflecting torque
Tine plane of a coil rotating in such a field remains ka = NIBA
parallel to the field in all positions, as shown in
NBA
Fig. A.97(a). Also, the soft iron cylinder, due to its high or a = . I
k
permeability, intensifies the magnetic field and hence
increases the sensitivity of the galvanometer. or a oc /
Thus the deflection produced in the galvanometer
coil is proportional to the current flowing through it.
Consequently, the instrument can be provided with a scale
MAGNETIC EFFECT OF CURRENT

2. By increasing the magnetic field B. This can be


done by using a strong horse-shoe magnet and
placing a soft iron core within the coil.
4.59

/
with equal divisions along a circular scale to indicate 3. By increasing the area A of the coil. However,
equal steps in current. Such a scale is called linear scale. increasing A beyond a certain limit will make
k
Also, 1 = . a = Ga the galvanometer bulky and unmanageable.
NBA
4. By decreasing the value of torsion constant k. The
The factor G = k I NBA is constant for a galvano torsion constant k is made small by using sus
meter and is called galvanometer consta7it or current pension wire and springs of phosphor bronze.
reduction factor of the galvanometer. 25. Give the advantages and disadvajitages of using a
Figure of merit of a galvanometer. It is defined as the moving coil galvanometer,
current which produces a deflection of one scale division in Advantages of a moving coil galvanometer :
the galvanometer and is given by

w
1. As the deflection of the coil is proportional to
the current passed through it, so a linear scale
a NBA can be used to measure the deflection.

Flo
2. A moving coil galvanometer can be made highly
4.21 SENSITIVITY OF A GALVANOMETER
sensitive by increasing N, B, A and decreasing k.
24. When is a galvanometer said to be sensitive ? 3. As the coil is placed in a strong magnetic field of

ee
Define current sensitivity and voltage sensitivity of a a powerful magnet, its deflection is not affected

Fr
galvanometer. State the factors on zvhich the sensitivity of by external magnetic fields. This enables us to
a moving coil galvanometer depends. How can zve use the galvanometer in any position.
increase the sensitivity of a galvanometer ? 4. As the coil is wound over a metallic frame, the
for
ur
Sensitivity of a galvanometer. A galvanometer is said eddy currents produced in the frame bring the
to be sensitive if it shows large scale deflection even zvhen a coil to rest quickly.
small current is passed through it or a small voltage is Disadvantages of a moving coil galvanometer ;
ks
applied across it. 1. The main disadvantage is that its sensitiveness
Yo
oo

Current sensitivity. If is d^ned as the d^ection produced cannot be changed at will.


in the galvanometer when a unit current flozvs through it. 2. All types of moving coil galvanometers are
eB

NBA
Current sensitivity, =
a
easily damaged by overloading. A current greater
I k than that which the instrument is intended to

Voltage sensitivity. It is dotted as the d^ection produced measure will burn out its hair-springs or
r

in the galvanometer zvhen a unit potential difference is suspension.


ou
ad

applied across its ends.


For Your Knowledge
Y

a a NBA
Voltage sensitivity, =
V IR kR
> If the radial field were not present in a moving coil
nd

Current sensitivity galvanometer, for example, if the soft iron cylinder


Re

Clearly, voltage sensitivity =


R were removed, then the torque would be NBA! sin 6
Fi

and I would be proportional a / sin 0. The scale would


Factors on which the sensitivity of a moving coil then be iion-liuear and difficult to calibrate or to read
galvanometer depends :
accurately.
1. Number of turns N in its coil.
^ Phosphor-bronze is used for suspension or hair
2. Magnetic field B. springs because of several reasons :
3. Area A of the coil.
1. It is a good conductor of electricity.
4. Torsion constant k of the spring and suspension 2. It does not oxidise.
wire.
3. It is perfectly elastic.
Factors by which the sensitivity of a moving coil 4. It has very little elastic after effect.
galvanometer can be increased :
5. It is non-magnetic.
1. By increasing the number of turns N of the coil. But 6. Of all materials, it has the minimum value for
the value of N cannot be increased beyond a
certain limit because that will make the restoring torque per unit twist i.e., smallest
torsion constant k.
galvanometer bulky and increase its resistance R.
PHYSICS-Xll

You might also like